0% found this document useful (0 votes)
374 views848 pages

‎⁨ملف كامل⁩

The document contains a series of medical exam questions and answers related to various surgical and medical topics. It covers contraindications for surgeries, clinical manifestations of conditions, fluid therapy, wound healing, and complications in surgical procedures. Each question is followed by the correct answer, providing a comprehensive review for medical students or professionals preparing for examinations.

Uploaded by

55j97sxvwh
Copyright
© © All Rights Reserved
We take content rights seriously. If you suspect this is your content, claim it here.
Available Formats
Download as PDF, TXT or read online on Scribd
0% found this document useful (0 votes)
374 views848 pages

‎⁨ملف كامل⁩

The document contains a series of medical exam questions and answers related to various surgical and medical topics. It covers contraindications for surgeries, clinical manifestations of conditions, fluid therapy, wound healing, and complications in surgical procedures. Each question is followed by the correct answer, providing a comprehensive review for medical students or professionals preparing for examinations.

Uploaded by

55j97sxvwh
Copyright
© © All Rights Reserved
We take content rights seriously. If you suspect this is your content, claim it here.
Available Formats
Download as PDF, TXT or read online on Scribd
You are on page 1/ 848

1st year exam 2020 MOH

Which of the following is contraindication for day-case inguinal


hernia surgery?
A. Age of 70 years old.
B. Well controlled epilepsy .
C. Well control Diabetes type 1.
D. American Society Anesthesiologist (ASA) score 3.
E. Body mass index of 35kg/m2.

ANSWER D

One of the following statements regarding cellulitis is not true?


A. This is non-suppurative invasive infection of tissues.
B. It is poorly localized.
C. Could be caused by Clostridium perfringens.
D. Caused by beta hemolytic streptococcus.
E. Systemic signs are not present.

ANSWER E

In response to reversible hypovolemic shock, all of the following


indices decrease except
A. Cardiac output.
B. Systemic vascular resistance.
C. Blood volume in venous capacitance.
D. Central venous pressure.
E. Mixed venous oxygen saturation.
ANSWER: B

In an average adult, all the following are compatible with class


II hemorrhage except
A. Blood loss of 1000cc.
B. Heart rate 130 per minute.
C. Postural hypotension.
D. Anxiety.
E. Decreased central venous pressure.
ANSWER: B

All the following are clinical manifestations of hyperkalemia


except
A. Abdominal pain.
B. Paralysis.
C. Weakness.
D. Constipation.
E. Arrythmia.
ANSWER: D

A 20 years old male patient, presented complaining of abdominal


pain and vomiting for 24 hours due to intestinal obstruction, he
is thirsty with dry mouth, mucous membranes and axillae, he has
mild tachycardia but normal supine blood pressure, his weight is
80kg. Which of the following parenteral fluid therapy formulas may
be more suitable for him in the first 24 hours?
Dr. wessam alzaidat - General surgery
A. 8 L glucose saline 0.45% .
B. 1 L normal saline and 2 L glucose water 5%.
C. 2 L normal saline and 1 L glucose water 5%.
D. 4 L glucose saline 0.18%.
E. 4 L normal saline.
ANSWER: A

All the following are true about liver anatomy except


A. Cantlie’s line extends roughly between the gallbladder fossa
and the inferior vena cava.
B. The right lobe accounts around two thirds of the liver size.
C. The Caudate lobe lies posterior to the level of the porta
hepatis.
D. The anterior branch of the right portal vein usually supplies
liver segments V and VI.
E. The right hepatic vein drains venous tributaries from segments
V, VI, VII and VIII.
ANSWER: D

All the following are true regarding the pancreatic ducts except
A. The duct of Santorini embryologically comes from the dorsal
pancreatic bud.
B. The ventral bud forms most of the pancreas.
C. In most people most of the pancreas is drained via the major
ampulla.
D. In most people the two pancreatic ducts are communicated.
E. The ampulla of Vator located between the proximal two thirds
and the distal one third of the second part of duodenum.
Answer: B

Which of the following does not pass through the diaphragmatic


openings?
A. Aorta
B. Inferior Vena Cava
C. Esophagus
D. Left gastric artery
E. Left phrenic nerve
ANSWER: D

.01In a total thyroidectomy for advanced papillary cancer, you


notice the tumor on the right is adherent to a nerve in the
tracheo-esophageal groove. This nerve injury or resection will
result in :
A. Loss of high-pitched voice
B. Stridor with need for tracheostomy
C. Unilateral right vocal cord paralysis
D. Loss of sensation above the vocal cords.
E. Loss of sensation in the skin of the right muscular triangle
ANSWER: C

A patient asks you when he can allow water to touch his closed
surgical wound. Which of the following is true pertaining to wound
healing& external environment ?
Dr. wessam alzaidat - General surgery
A. One week, as epithelialization and granulation tissue formation
need to occur to protect the wound further, even with meticulous
closure
B. 24 hours, as meticulous closure with epithelialization seals
the wound from external environment, in the absence of excessive
tension on wound
C. Two weeks, as acellular collagen starts forming and maturation
phase occurs
D. Five days, as epithelialization requires this long, and
collagen formation begins then
E. Three weeks to allow collagen type III, which is present in
normal skin, to fully form
ANSWER: B

In maturation phase of wound healing, one of the following is


true:
A. Fibroblasts continue to produce collagen and interstitial
matrix
B. Angiogenesis continues to provide nutrients to the healing
wound
C. Skin loses its elasticity as Elastin never forms
D. Cells gradually undergo apoptosis, and a cellular collagen
continues to remodel
E. Typically starts at 6 weeks after the injury
ANSWER: D

Which of the sutures listed below would be most suitable for


suturing the femoral artery following an embolectomy?
A. 5/0 polyglactin
B. 5/0 polypropylene
C. 5/0 silk
D. 5/0 polyglycolic acid
E. 5/0 polyester
ANSWER: B

A24-year-old woman is scheduled for an elective cholecystectomy.


The best method of identifying a potential bleeding is which of
the following ?
A. Platelet count
B. A complete history and physical examination
C. Bleeding time
D. Lee-White clotting time
E. Prothrombin time (PT)
ANSWER: B

A 60-year-old woman who underwent a mastectomy for breast cancer 2


years earlier presents to the emergency department with headache,
backache, and frequent vomiting. She is extremely thirsty and
stuporous. Which test is most likely to identify the cause ?

A. CT scan of the head


B. X-ray of spine
C. Serum sodium determination
Dr. wessam alzaidat - General surgery
D. Serum calcium determination
E. Serum glucose determination
ANSWER: D

A 43-year-old woman with von Willebrand’s disease is scheduled for


cholecystectomy. It can be stated that preoperative evaluation
will reveal which of the following?
A. Normal bleeding time, PT, and PTT
B. Platelet aggregate with restocetin
C. Increased bleeding time and PTT, and normal PT
D. Increased bleeding time and PT, and normal PTT
E. Increased bleeding time, and normal PT and PTT
ANSWER: C

A 64-year-old woman undergoing radical gastrectomy under general


anesthesia is transfused with 2 U of packed RBCs. A hemolytic
transfusion reaction during anesthesia will be characterized by
which of the following?
A. Shaking chills and muscle spasms
B. Fever and oliguria
C. Hyperpyrexia and hypotension
D. Tachycardia and cyanosis
E. Bleeding and hypotension
ANSWER: E

After undergoing subtotal gastrectomy for carcinoma of the


stomach, a 64-year-old woman is receiving peripheral parenteral
nutrition. To increase calories by the peripheral route, what
should be prescribed ?
A. D5W in normal saline
B. Multivitamin infusion
C. D25W (25% dextrose in water )
D. Soybean oil (intra lipid)
E. Lactulose
ANSWER: D

What is the definition of overweight?


A. BMI > 25 kg/m2
B. BMI = 25 kg/m2
C. BMI 25-29.9 kg/m2
D. BMI 25-30 kg/m2
E. BMI > 30 kg/m2
ANSWER: C

In nutritional therapy, the commonest cause of complications is


A. Overfeeding.
B. Protein insufficiency.
C. Dehydration.
D. Hyponatremia.
E. Vitamins deficiency.
ANSWER: A

All of the following are true regarding human body fluids except
Dr. wessam alzaidat - General surgery
A. Water constitutes about 60% of the weight of an average adult.
B. Most of body water presents intracellular.
C. An average adult, about 75 kg weight, needs about 1400cc of
water daily.
D. Urine constitutes the major source of water loss.
E. Interstitial water is much more than intravascular water.
ANSWER: C

In response to trauma, the serum levels of all of the following


hormones increase except
A. T3.
B. Adrenalin.
C. Insulin .
D. Growth hormone.
E. Cortisol.
ANSWER: C

All of the following increase the amount of insensible water loss


except.
A. Dry weather.
B. Fever.
C. Diarrhea .
D. Tracheotomy.
E. Hot weather.
ANSWER: C

Which of the following operations need a prophylactic antibiotic?


A. Herniotomy.
B. Appendicectomy for a suppurative appendix .
C. Drainage for a perianal abscess.
D. Surgery for a perforated peptic ulcer.
E. Thyroid surgery.
ANSWER: B

The commonest causative organisms for cellulitis are


A. Streptococci .
B. Staphylococci.
C. E- coli.
D. Bacteroides.
E. Klebsiella species.
ANSWER: A

All the following are true regarding respiratory alkalosis except


A. Caused by increase respiratory CO2 washing.
B. Kidneys compensate rapidly for the condition .
C. Compensation never overcorrect the pH.
D. Rebreathing mask can be used in treatment.
E. May be associated by carpopedal spasm.
ANSWER: B

All of the following are causes of metabolic acidosis except


A. Pancreatic fistula.
B. Gastric outlet obstruction .
Dr. wessam alzaidat - General surgery
C. Salicylic acid overdose.
D. Methanol toxicity.
E. Sepsis.
ANSWER: B

Donated blood is screened for all of the following except


A. Hepatitis B virus.
B. Hepatitis C virus.
C. HIV1.
D. Syphilis.
E. Cytomegalo virus.
ANSWER: E

Which of the following blood products carries the highest risk of


transmission of bacterial infection?
A. Packed RBCs.
B. Fresh frozen plasma.
C. Platelets concentrates .
D. Cryoprecipitate.
E. Whole blood.
ANSWER: C

Which of the following is an advantage of transjugular central


venous line over a central venous line inserted through a
peripheral vein?
A. We can give higher concentration of glucose solutions.
B. We can keep it for a longer period .
C. Less risk of pneumothorax.
D. Less risk of arrythmia.
E. Less risk of air embolism .
ANSWER: B

In the management of a patient who has massive trauma and


hypotension, all of the following should be done except
A. Stop hemorrhage.
B. Correct coagulopathy.
C. Elevate systolic pressure above 110mmHg .
D. Control sepsis or major contamination.
E. Give blood.
ANSWER: C

In comparison between healing by primary and secondary intentions


for a wound involving skin and subcutaneous fat, healing by
primary intension has
A. More inflammatory response.
B. Less tensile strength.
C. Less chance to develop abscess.
D. Less scar .
E. More granulation tissue formation.
ANSWER: D

All of the following are true about polypropylene sutures except


A. It is monofilament.
Dr. wessam alzaidat - General surgery
B. It preserves its tensile strength for many years.
C. It is non absorbable.
D. It develops high tissue reaction .
E. It has high memory .
ANSWER: D

All of the following increase the risk of wound infection except


A. Obesity.
B. Jaundice.
C. Uremia.
D. Chemotherapy.
E. Pathing at the first postoperative day .
ANSWER: E

Which of the following sutures carries a higher risk of wound


infection?
A. Silk .
B. Polypropylene.
C. Nylon.
D. Polyglactin.
E. Polydioxanone.
ANSWER: A

Cancer cells have all of the following features except


A. Resist signals that inhibit growth.
B. Develop early apoptosis .
C. Acquire angiogenic competence.
D. Has genomic instability.
E. Acquire the ability to invade.
ANSWER: B

All of the following are steps done in processing a specimen for


histological examination except
A. Fixation in formalin for one day.
B. Make representative tissue blocks.
C. Place blocks in paraffin wax.
D. Make thin slices on glass slides.
E. Stain slides with Indian ink.
ANSWER: E

All of the following are of the first step of pain stepladder


booklet advised by WHO to control chronic pain caused by
malignancy, except
A. Tramadol.
B. Aspirin.
C. Paracetamol.
D. Nonsteroidal anti-inflammatory drugs.
E. Tricyclic antidepressants.
ANSWER: A

Measures to reduce the risk of venous thromboembolism include all


of the following except
A. Prevent preoperative dehydration.
Dr. wessam alzaidat - General surgery
B. Start to apply graduated compressing lower limbs stockings
immediately after surgery.
C. Apply intraoperative foot impulse device.
D. Use intraoperative pneumatic compressing device.
E. Start prophylactic low molecular weight heparin preoperatively.
ANSWER: B

All of the following combinations between an ulcer’s edge and


pathology are true except
A. Sloping edge: healing ulcer .
B. Undermined edge: bed sore.
C. Rolled edge: basal cell carcinoma.
D. Everted edge: tuberculosis.
E. Punched-out edge: syphilis.
ANSWER: D

All of the following are proven examples of congenital sinuses


except
A. Preauricular.
B. Umbilical.
C. Pilonidal.
D. Urachus.
E. Coccygeal.
ANSWER: C

Which of the following incidents is considered an adverse event?


A. Superficial thrombophlebitis in a peripheral vein caused by a
cannula.
B. A needle found near a patient’s bed.
C. Sciatic nerve injury after an intramuscular injection.
D. Intraoperative bleeding of 100cc from the right colic artery
during right hemicolectomy which was stopped at the same
operation.
E. Laparoscopic cholecystectomy converted to open due to sever
inflammation.
ANSWER: C

All of the following should be checked before induction of


anesthesia in a patient planed for paraumbilical hernia repair
except
A. Confirm the patient’s name.
B. Confirm the diagnosis.
C. Check the consent.
D. Mark the site of the procedure.
E. Ask for known allergies.
ANSWER: D

Which of the following forceps is unsuitable to grasp the bowel?


A. Nontoothed.
B. Allis.
C. Babcock.
D. Kocher.
E. Cecal forceps.
Dr. wessam alzaidat - General surgery
ANSWER: D

All the following are causes of clubbing except


A. Lung cancer.
B. Lung abscess.
C. Empyema.
D. Gastric cancer.
E. Cirrhosis.
ANSWER: D

All the following are true about venous ulcers except


A. Associated with varicose veins.
B. Usually on the lateral aspect of the leg.
C. Usually shallow.
D. The affected limb is usually warm.
E. More common in women.
ANSWER: B

Which of the following organs creates the least immunological


response in deceased donor organ transplantation?
A. Liver.
B. Kidney.
C. Cornea.
D. Pancreas.
E. Small bowel.
ANSWER: C

All the following are true about trichilemmal cyst except


A. More common in the scalp.
B. Has a distinct odor if ruptured.
C. Has a thin wall.
D. The wall is a complete mature epidermis with granular layer.
E. If causes symptoms, the entire cyst is removed.
ANSWER: D

Amnesia is a main action of all the following drugs except


A. Midazolam.
B. Barbiturates.
C. Etomidate.
D. Ketamine.
E. Fentanyl.
ANSWER: E

All the following are true about laryngeal mask airway except
A. It sits in the oropharynx.
B. It is passed to its site under vision.
C. It has a cuff which is inflated to push the soft tissue away
from the laryngeal inlet.
D. It does not pass through the vocal cords.
E. It does not fully protect against aspiration.
ANSWER: B

Dr. wessam alzaidat - General surgery


2nd year exam 2020 MOH
Low pitch sound after thyroidectomy is usually attributed to
malfunction of:
A. Omohyoid muscle.
B. Sternohyoid muscle.
C. Cricothyroid muscle .
D. Thyrohyoid muscle.
E. Sternothyroid muscle.
ANSWER: C

Serum calcium level is usually elevated in all of the followings


except:
A. Primary hyperparathyroidism .
B. Secondary hyperparathyroidism .
C. Tertiary hyperparathyroidism .
D. Hyperparathyroidism due to ectopic adenoma .
E. Vitamin D intoxication.
ANSWER: B

Unilateral bloody nipple discharge from one duct orifice is most


commonly caused by:

A. Paget's disease of the nipple.


B. Carcinoma in situ.
C. Inflammatory carcinoma.
D. Subareolar mastitis.
E. Intraductal papilloma.
ANSWER: E

Which of the following has the highest risk for Colo-rectal


carcinoma?
A. Ulcerative colitis.
B. Crohn’s disease.
C. Familial polyposis coli.
D. Diverticulosis.
E. Juvenile polyps.
ANSWER: C

A 35 years old male patient developed large amount of fresh


bleeding per rectum 10 days post hemorrhoidectomy. The most
probable cause for his bleeding is
A. Sloughing of a vessel wall.
B. Uncontrolled bleeding missed at the time of surgery.
C. Clot dislodgment.
D. Injury by defecation.
E. Vasodilation.
ANSWER: A

Neoplastic cells have the ability to spread. One of the following


is true:
A. Lymph node metastases permeate the sinusoids of the node and
later spread throughout the subcapsular space
B. Carcinoma in situ has no detectable invasion beyond the
Dr. wessam alzaidat - General surgery
basement membrane
C. Lymphatic involvement is common with sarcomas
D. The metastatic process of malignancy is highly efficient, as
once they shed into the bloodstream, they produce metastasis
almost always
E. Epithelial neoplasm spread only through hematogenous route
ANSWER: B

A 47-year-old woman presents to the emergency room after a motor


vehicle accident, mildly lethargic and confused, with normal
airway and vitals. She complains of a headache, and has a frontal
scalp hematoma. After stabilizing her cervical spine and doing
primary survey, she becomes more lethargic, moving only to pain.
Which of the following is the most likely to be present?
A. Subdural hematoma
B. Extradural hematoma
C. Cerebral contusion
D. Intracerebral hematoma
E. Diffuse axonal injury
ANSWER: B

If you want to review a reference paper addressing the best method


of management yielding the best outcome, which of the following
has the best level of evidence?
A. A well-powered double-blinded randomized controlled trial
B. A case-series
C. A retrospective series of more than 1000 patients
D. Meta-analysis of retrospective series
E. A case-report
ANSWER: A

All of the following are gases used to provide pneumoperitoneum in


laparoscopic surgery except?
A. Methane
B. Carbon dioxide
C. Helium
D. Nitrous oxide
E. Argon.
ANSWER: A

In which region of the GI tract is the longitudinal muscle of the


muscularis arranged into distinct longitudinal bundles?
A. Duodenum
B. Jejunum
C. Ileum
D. Colon
E. Stomach
ANSWER: D

All of the following conditions are associated with low cardiac


output except
A. Anaphylactic shock.
B. Cardiac tamponade.
Dr. wessam alzaidat - General surgery
C. Tension pneumothorax.
D. Massive pulmonary embolism.
E. Air embolism.
ANSWER: A

All the following statements are true regarding allergic blood


transfusion reactions except
A. They are caused by donor antigens or antibodies.
B. They are more commonly occur with transfusion of packed red
blood cells.
C. They are usually mild.
D. Usually treated by antihistamines.
E. Epinephrine is occasionally needed in the management of the
reaction.
ANSWER: B

All the following are true about refeeding syndrome except


A. It occurs more with patients on parenteral nutrition than
patients on enteral formulas.
B. It results mainly from decreased levels of some electrolytes .
C. It results in multiorgan failure.
D. It is prevented by increasing calorie intake slowly and regular
administration of minerals and vitamins.
E. It is treated by stopping proteins in the given formula.
ANSWER: E

In considering a donor for living related liver transplantation,


one of the following is a contra indication from the donor stand
point .
A. Replaced or accessory right or left hepatic artery
B. Active Non-Alcoholic Steatohepatitis (NASH) with fibrosis
C. Atherosclerosis of the aorta
D. Two right bile ducts
E. Dual right hepatic vein
ANSWER: B

A 43-year-old man was transported to the emergency department


after a high voltage electrical injury at work. After intubation,
resuscitation and exposure, you notice a significant burn in his
right upper extremity and his feet. In regards to his electrical
burn, all of the following are true except:
A. Admission under cardiac monitoring due to high risk for
arrhythmias
B. Electrical burn necrosis may extend beyond the burn entry site
in his extremity
C. Repeated debridement and fasciotomy may be necessary
D. Allowing necrotic tissue to spontaneously separate on its own
prior to debridement is preferable, as the extent of necrosis may
be extensive
E. Main vessel thrombosis and potential need for amputation is a
risk in extremity electrical burns
ANSWER: D

Dr. wessam alzaidat - General surgery


While performing a laparoscopic appendectomy in a thin female
patient, the consultant surgeon notices a prominent peristalsing
structure crossing the right external iliac artery, traversing to
the medial part of the pelvis. This tube-like structure
represents :
A. Right round ligament
B. Right ureter
C. Right broad ligament
D. Right gonadal vessel
E. Hypogastric plexus
ANSWER: B

A patient with an open wound inquires about the best option to


provide the best aesthetic (cosmetic) result. Which of the
following gives the best cosmetic result?
A. Nothing can be done
B. Applying a vacuum dressing will almost result in a scar-less
wound
C. Frequent dressings
D. Placement of silver dressing
E. Delayed primary closure
ANSWER: E

What is the most likely initial symptom to occur as a result of


local anesthetic toxicity?
A. Seizures
B. Restlessness
C. Slurred speech
D. Coma
E. Muscle weakness
ANSWER: B

A 55-year-old man with Crohn’s disease had undergone resection of


small bowel and anastomosis. Ten days later, he is found to have
bilious drainage of 1 L/d from the drains. He is started on total
parenteral nutrition (TPN). Four days later, his arterial blood
gases (ABGs) are pH, 7.25; PO2, 98 mm Hg; and PCO2, 40 mm Hg. His
anion gap is 10. The most likely cause is which of the following ?
A. Diabetic ketoacidosis
B. Renal failure
C. Hypovolemic shock
D. Small-bowel fistula
E. Uncompensated metabolic alkalosis
ANSWER: D

After undergoing a transurethral resection of the prostate, a 65-


year-old man experiences excessive bleeding attributed to
fibrinolysis. It is appropriate to administer which of the
following ?
A. Heparin
B. Warfarin (Coumadin )
C. Volume expanders and cryoprecipitate
D. Aminocaproic acid (Amicar )
Dr. wessam alzaidat - General surgery
E. Fresh-frozen plasma and vitamin K
ANSWER: D

A 70-year-old woman has low cardiac output with increased


pulmonary capillary wedge pressure (PCWP) and increased systemic
vascular resistance. What should be the drug of choice ?
A. Dopamine
B. Norepinephrine
C. Dobutamine
D. Epinephrine
E. Phenylephrine
ANSWER: C

Which of the following is not a limitation of minimal access


surgery?
A. Technically more demanding
B. Loss of tactile feedback
C. Extraction of large specimens
D. Poor vision
E. Difficulty with hemostasis.
ANSER: D

Which of the following is not a complication associated with


pneumoperitoneum?
A. Hyperthermia
B. Acidosis
C. Cardiac arrhythmias
D. Gas embolism
E. Reduced cardiac return
ANSWER: A

All the following statements about the anatomy of the esophagus


are true except
A. The narrowest point of esophagus is at the level of
cricopharyngeal muscle
B. The Meissner plexus is located in the submucosa
C. The Auerbach plexus is located between the muscle layers of the
esophagus
D. The esophagus lacks the serosal layer
E. The outer longitudinal muscle layer is an extension of the
crichopharyngeus muscle
ANSWER: E

All the following statements regarding the blood supply of the


stomach are true except
A. The left gastroepiploic artery commonly arises from splenic
artery
B. Ligation of the left gastric artery can result in acute left
sided hepatic ischemia
C. The stomach is well protected from ischemia because of rich
collateral circulation
D. The short gastric arteries provide significant blood supply to
the body of the stomach
Dr. wessam alzaidat - General surgery
E. The gastroepiploic artery usually originates from the
gastroduodenal artery
ANSWER: D

Which of the following sutures is natural?


A. Silk.
B. Polyglycolic.
C. Polydioxanone.
D. Polyproline.
E. Polyglactin.
ANSWER: A

Braided sutures
A. Are non-absorbable.
B. Are natural.
C. Have better knotting.
D. Have stronger memory.
E. Carries less risk of infection
ANSWER: C

Which of the following is true about hemophilia A?


A. Is factor VIII deficiency.
B. Inherited as autosomal dominant.
C. Males and females are equally affected.
D. Fresh frozen plasma is the treatment of choice.
E. The platelet function is impaired in patients with hemophilia.
ANSWER: A

All the following are risk factors for surgical site infection
except
A. Obesity.
B. Female gender.
C. Uremia.
D. Obesity.
E. Jaundice.
Answer: B

All the following are true about cellulitis except


A. It is a non-suppurative infection.
B. It is well localized.
C. Commonly caused by beta hemolytic streptococci.
D. Fever is common.
E. Blood cultures are usually negative.
ANSWER: B

Which of the following antibiotics is most effective against


Clostridia difficile?
A. Imipenem.
B. Metronidazole.
C. Vancomycin.
D. Ciprofloxacin.
E. Ceftazidime.
ANSWER: C
Dr. wessam alzaidat - General surgery
With regard to bipolar and monopolar devices, which of the
following is true?
A. Both use direct current (DC.)
B. Both uses alternating current (AC.)
C. Bipolar uses AC, while monopolar uses DC.
D. Bipolar uses DC, while monopolar uses AC.
E. Neither requires an electrosurgical unit.
ANSWER: B

Which of the following sites for central venous catheter placement


is associated with the lowest rate of catheter-associated blood
stream infections?
A. Internal jugular vein
B. Subclavian vein
C. Femoral vein
D. Both A and B are equivalent
E. All sites are equivalent if sterile technique is maintained
ANSWER: B

All the following are causes of wide anion gap metabolic acidosis
except
A. Lactic acidosis.
B. Ketoacidosis.
C. Pancreatic fistula.
D. Alcohol toxicity.
E. Salicylic acid overdose.
ANSWER: C

Autologous intraoperative blood donation can be an option in which


of the following conditions?
A. Aortocoronary bypass.
B. Mastectomy and breast reconstruction for breast cancer.
C. Sigmoid resection for volvulus.
D. Surgery for abdominal abscess.
E. Liver transplant for cirrhosis and small hepatocellular
carcinoma.
ANSWER: A

Cryoprecipitate is stored at
A. 2 to 6 degrees C.
B. -2 to -6 degrees C.
C. Room temperature.
D. -30 degrees C.
E. -70 degrees C.
ANSWER: D

All commonly develop as consequences of reperfusion syndrome


except
A. Hyperkalemia.
B. Myocardial suppression.
C. Vasoconstriction.
D. Acute lung injury.
Dr. wessam alzaidat - General surgery
E. Acute renal injury.
ANSWER: C

Early after a skin cut wound, all the following occur at the site
of injury except
A. Vasodilation.
B. Platelets aggregation.
C. Infiltration by macrophages and polymorphonuclear cells.
D. Release of serotonin.
E. Swelling of the injury site.
ANSWER: A

All the following are true regarding necrotizing soft tissue


infections except
A. They are usually polymicrobial.
B. Beta hemolytic streptococci are usually part of the causative
microbes.
C. Clostridia perfrengens causes gas gangrene.
D. Pain is usually out-of-proportion to physical signs.
E. Treatment is high dose of a wide spectrum antibiotic changed
according to culture result when ready.
ANSWER: E

A 40-year-old lady presented with 2cm left breast mass in the


retro-areolar region proved by a true-cut biopsy to be invasive
ductal carcinoma. Metastatic work up was free. You discussed the
condition with her and offered mastectomy with sentinel axillary
LN sampling. She refused to do any breast surgery. What should you
do?
A. Insist on your plan.
B. Discuss the matter with her family.
C. Refer her to another surgeon.
D. Discuss with her other treatment options although not as
effective as surgery.
E. Inform the police that she refuses treatment.
ANSWER: D

Complications of trans-arterial angiography include all the


following except
A. Renal impairment.
B. Deep vein thrombosis.
C. Arterial dissection.
D. Hematoma .
E. Distal embolism.
ANSWER: B

All the following is true about the abdominal wall anatomy except
A. The rectus abdominus muscles lie between the anterior and
posterior rectus sheets all the way from xiphoid to symphysis
pubis.
B. The internal oblique muscle lies between the external oblique
and the transversus abdominis.
C. The transversalis fascia lies under the transversus abdominis
Dr. wessam alzaidat - General surgery
muscle .
D. The latissimus dorsi lies posterior to the external oblique
muscle.
E. The linea semilunaris lies at the lateral border of the rectus
abdominis.
ANSWER: A

Richter’s hernia defined as


A. An inguinal hernia that contains ovary.
B. An inguinal hernia that contains appendix.
C. An irreducible hernia.
D. A hernia that involve only part of the bowel wall.
E. A posterior lumber hernia.
ANSWER: D

All the following about the inguinal anatomy are true except
A. The deep inguinal ring is a defect in the transversalis fascia.
B. The deep inguinal ring lies midway between the symphysis pubis
and the anterior superior iliac spine.
C. The inferior epigastric vessels lie just medial to the deep
inguinal ring.
D. The external oblique aponeurosis forms the anterior wall of the
inguinal canal.
E. The inguinal ligament constitutes the floor of the inguinal
canal.
ANSWER: B

Which of the following is a contraindication for a day-case


surgery?
A. A patient over the age of 60 years.
B. A surgery that should be done under general anesthesia.
C. A patient on warfarin.
D. A hypertensive patient.
E. A laparoscopic surgery.
ANSWER: C

A 20-year-old male underwent surgery for hemorrhoids, which kind


of postoperative analgesia you prefer to give him in the first 3
days?
A. Paracetamol.
B. Nonsteroidal anti-inflammatory.
C. Local lidocaine gel.
D. Carbamazepine.
E. Diazepam.
ANSWER: B

All the following are true about thyroid neoplasms except


A. Follicular neoplasms are unifocal.
B. Papillary cancers carries an excellent prognosis.
C. Papillary cancer metastasize commonly to cervical lymph nodes.
D. Lymph node metastasis in papillary thyroid cancer is the main
determinant of prognosis.
E. Medullary cancer commonly metastasizes to lymph nodes.
Dr. wessam alzaidat - General surgery
ANSWER: D

Which of the following is a proven advantage of gastro-intestinal


staplers?
A. Less infectious complications.
B. Less bleeding.
C. Shortens surgery time.
D. Less anastomotic leak.
E. Less cost.
ANSWER: C

All the followings contribute to gastroesophageal reflux except


A. Presence of Hiatal hernia
B. Cigarette smoking
C. High protein diet
D. Obesity
E. Abnormal esophageal motility
ANSWER: C

All the following statements regarding gastrointestinal tract


diverticular disease are true except
A. Most colonic diverticula are false diverticula
B. Meckel's diverticulum is a pulsion type diverticulum
C. The most common way of presentation in children with
symptomatic Meckel's diverticulum is Gastrointestinal bleeding
D. False diverticula are usually acquired
E. Sigmoid colon is the most affected site
ANSWER: B

Dr. wessam alzaidat - General surgery


3rd year exam 2020 MOH
In primary hyperparathyroidism which of the following tests is
most important in anticipating post-surgery bone hunger syndrome:
A. Alkaline phosphatase .
B. Erythrocyte sedimentation rate (ESR.)
C. C reactive protein.
D. Preoperative blood calcium level.
E. Preoperative blood phosphate level.
ANSWER: A

Most common site of breast metastasis is:


A. Lung.
B. Brain.
C. Bone.
D. Adrenal.
E. Liver.
ANSWER: C

Which of the following is more common to cause massive colonic


bleeding?
A. Cancer.
B. Ulcerative colitis.
C. Angiodysplasia.
D. Diverticulitis.
E. Granulomatous colitis.
ANSWER: C

Which pelvic floor muscle provides an important function in


maintaining the rectal angle?
A. Pubococcygeus.
B. Iliococcygeus.
C. Puboiliacus.
D. Puborectalis.
E. External and internal sphincter.
ANSWER: D

Which bacterial genus produces the most potent exotoxins?


A. Staphylococcus.
B. Pseudomonas.
C. Klebsiella.
D. Streptococcus.
E. Serratia.
ANSWER: D

The treatment of choice for type III choledochal cyst


(choledochocele) is
A. Excision.
B. Sphincterotomy.
C. Whipple operation.
D. Observation.
E. Biliary bypass.
ANSWER: B

Dr. wessam alzaidat - General surgery


About nutrition in patients with acute pancreatitis, one is true
A. The patient should avoid enteral nutrition for the first 36
hours to rest the pancreas.
B. Parenteral nutrition should be started early.
C. Enteral nutrition is indicated to start within the first 24
hours after admission in most cases.
D. Elemental formulas are superior to the standard polymeric
formulas.
E. The patient diet should be advanced by the treating physician
according to the physical findings and not left for the patient
himself control.
ANSWER: C

A 24-year-old man was involved in a motor vehicle accident. His


car caught fire with evidence of carbonaceous sputum, edematous
oropharynx, tachypnea, and stridor. The patient is hemodynamically
stable with oxygen saturation of 97%. Before attempting
intubation, which of the following maneuvers is necessary?
A. Insert an endotracheal tube for his inhalational injury
B. Axial stabilization of his cervical spine
C. Administer paralyzing agents
D. Administer morphine for sedation
E. Immediate needle cricothyroidotomy
ANSWER: B

During cholecystectomy in a 67-year-old woman, there is severe


bleeding from accidental injury to the hepatic artery. The patient
requires transfusion of 2000 ML of blood. After the operation, 24-
hour urine output varies between 1250 and 2700 mL/d. She was
adequately hydrated, but BUN levels continue to rise 10–12 mg
daily over a 5-day period. What is the most p0ssible explanation ?
A. Progressive bleeding
B. High-output renal failure
C. Post cholecystectomy syndrome
D. Glomerulonephritis
E. Obstructive jaundice
ANSWER: B

Which of the following is not a limitation of minimal access


surgery?
A. Technically more demanding
B. Loss of tactile feedback
C. Extraction of large specimens
D. Poor vision
E. Difficulty with hemostasis.
ANSWER: D

Axillary lymph node dissection is not an option in treating one of


the following conditions:
A. 2cm pure non comedo type intraductal carcinoma.
B. 1cm infiltrating lobular carcinoma.
C. 8mm infiltrating ductal carcinoma.
D. A pure medullary cancer in the upper inner quadrant.
Dr. wessam alzaidat - General surgery
E. 3cm non otherwise specified invasive ductal carcinoma.
ANSWER: A

In below knee amputation for a diabetic foot; Tibia should be


transected:
A. 4-8 cm below tibial plate.
B. 8- 12 cm below tibial plate.
C. 12-16 cm below tibial plate.
D. 16- 20 cm below tibial plate.
E. 20- 24 cm below tibial plate.
ANSWER: B

Indications for operation in a patient with primary


hyperparathyroidism include all of the followings except:
A. Age older than 50.
B. Nephrolithiasis.
C. A substantial decline in renal function.
D. A substantial decline in bone mass.
E. Depression and fatigue.
ANSWER: A

The most important and proved benefit of external beam radiation


after wide excision of an invasive breast cancer is the ability to
improve one of the following outcomes:
A. Recurrence of cancer in the ipsilateral breast.
B. Patient survival time.
C. Regional nodal recurrence.
D. Patient cure rate.
E. Distant metastasis upon follow up.
ANSWER: A

Which of the following is true regarding the management of short


bowel syndrome?
A. Glutamine should be avoided.
B. Octreotide is the cornerstone of management.
C. Proton pump inhibitor is contraindicated.
D. Early enteral feeding is indicated.
E. Patients who still require TPN after 3 months will require
permanent TPN.
ANSWER D

Which of the following pancreatic tumors has the lowest risk of


malignancy?
A. Mucinous cystic neoplasm.
B. Serous cystic neoplasm.
C. Intraductal papillary mucinous neoplasm.
D. Solid pseudopapillary tumor.
E. Acinar cell tumor.
ANSWER: B

With regards to genetic susceptibility to cancer, one of the


following is true:
A. All women with BRCA 1 mutation develop breast cancer
Dr. wessam alzaidat - General surgery
B. RET-protoncogene is uniquely associated with colon cancer
C. P53 is rarely found in breast cancer
D. BRCA2 may be associated with ovarian cancer
E. MSH2 & MLH2 are present in familial adenomatous polyposis coli
ANSWER: D

In regard to the following physical findings, one of the following


is true:
A. Virchow’s node is an early sign of abdominal malignancy
B. Sister Mary Joseph’s sign is a periumbilical tumor deposits
from intra-abdominal malignancy
C. Blumer’s shelf is due to a direct extension of a rectal tumor
D. Horner’s syndrome is due to invasion of the thoracic
sympathetic chain
E. Krukenberg’s tumor is a primary ovarian malignancy
ANSWER: B

A 40- year-old 80 -Kg man sustained a 40% burn of the total


surface area, with a significant inhalational injury. After
intubation and initial resuscitation and admission, the parkland
formula was calculated and started lactated ringer’s solution at
800cc/hr. Six hours later, the patient became oliguric. What is
the best management to follow?
A. Increase the volume of lactated ringer’s solution infused
B. Bolus of Hetastarch solution
C. Trial of small dose of furosemide
D. Low-dose dopamine (vasopressor) infusion
E. Insert Swanz-Ganz catheter and measure pulmonary wedge pressure
ANSWER: A

A male infant, born at term, appears well following delivery. Six


hours later, he is noted to have bilious vomiting by the
pediatricians. On examination, he seems well and his abdomen is
soft and non-tender. What is the best course of action?
A. Arrange an abdominal x-ray
B. Undertake a test feed
C. Perform serial abdominal examinations
D. Arrange an upper GI contrast study
E. Arrange a laparotomy
ANSWER: D

Regarding undescended testes, one is true:


A. No decent takes place after birth
B. Orchidopexy reduce the risk of malignancy
C. Laparoscopy is the diagnostic investigation of choice in
impalpable testes
D. Fertility is not affected in unilateral cases
E. Orchidopexy should be performed at the age of 6 years
ANSWER: C

You have just completed a laparotomy for peritonitis due to a


perforated peptic ulcer. What is the best surgical strategy for
avoidance of a complete abdominal wound dehisence?
Dr. wessam alzaidat - General surgery
A. Use of skin clips to close the skin rather than sub cuticular
sutures
B. Careful approximation of the peritonum with non absorbable
sutures
C. Mass closure of the midline wound using a 1 polydiaxone suture
D. Direct apposition of the rectus muscle rather than linea alba
aponeurosis
E. Mass closure of the midline wound using a 3/0 polypropylene
suture
ANSWER: C

A 24-year-old man who is admitted to the intensive care unit (ICU)


following severe head injury develops seizures on the fourth day
of hospitalization. His urine output is 500 mL over 24 hours,
sodium is 115 mEq/L, and serum and urine osmolality are 250 and
800 mOsm, respectively. The metabolic abnormality is due to which
of the following ?
A. Administration of D5W (5% dextrose in water) and 0.33 normal
saline
B. Syndrome of inappropriate secretion of antidiuretic hormone
(SIADH )
C. Decreased antidiuretic hormone (ADH) secretion
D. Nasogastric suction
E. Renal insufficiency
ANSWER: B

A 22 years old male patient, presented with sever dyspnea and


chest discomfort for 30 minutes, with distended neck veins and
hypotension, which of the following should be done first?
A. Insertion of a large bore needle in the chest.
B. Chest x-ray.
C. ECG .
D. Insert a chest tube.
E. Endotracheal intubation.
ANSWER: A

All of the following are true regarding fresh frozen plasma (FFP)
except
A. Stored at -40 degrease C.
B. Shelf life is 6 months .
C. Rh D positive FFP can be given to Rh D negative patients .
D. It is a source for vitamin dependent clotting factors.
E. It is a source of factor V.
ANSWER: B

In response to trauma, all the following happen except


A. Decrease in basal metabolic rate .
B. Increase nitrogen requirements.
C. Hyperglycemia.
D. Loss of ketoadaptation.
E. Increase serum growth hormone level.
ANSWER: A

Dr. wessam alzaidat - General surgery


Necrotizing soft tissue infections are associated with all of the
following except
A. Pain.
B. Skin blistering.
C. Skin involved more than the subcutaneous tissue.
D. Greyish discharge.
E. Focal skin gangrene in some cases.
ANSWER: C

All of the following about nylon sutures are true except


A. It is a poly amide polymer.
B. It loses 15% to 20% of its tensile strength per year.
C. It never degrades.
D. Has low tissue reaction.
E. Can be used for skin closure.
ANSWER: C

All of the following are true regarding surgical audit except


A. It aims to improve patients’ care.
B. It involves comparing aspects of care against defined
standards.
C. It can be an essential component for revalidation of the
surgeon.
D. If the results are far from standards the surgeon should stop
working.
E. An audit should answer predetermined questions.
ANSWER: D

Regarding rapid sequence induction of anesthesia, all of the


following are true except
A. Used to allow the airway to be rapidly secured.
B. Used for patients with high risk of regurgitation.
C. Intravenous anesthetic agents can be used.
D. Slow acting muscle relaxant is used.
E. Commonly needed in emergency surgeries.
ANSWER: D

All of the following have less favorable prognosis in skin


squamous cell carcinoma except
A. Depth more than 6mm.
B. High histological grade.
C. Surface size more than 2 cm.
D. Tumors on the trunk.
E. Tumors that arise in burn scars.
ANSWER: D

All the following statements concerning caustic esophageal injury


with an alkaline substance (lye) are true except
A. Injury to esophagus is the result of liquefactive necrosis
B. Endoscopy should not be performed within the first 72 hours
because of the risk of perforation
C. Lye can be neutralized with half strength vinegar if the
patient is seen in the first hour of ingestion
Dr. wessam alzaidat - General surgery
D. To decrease long term stricture rate, endoscopic dilatation
should be performed only after re-epithelialization
E. As compared to injury caused by of acidic liquid, alkaline
substances cause more deep tissue penetration
ANSWER: B

When comparing the intestinal subtype to the diffuse subtype of


gastric adenocarcinoma, which of the followings is true
A. Intestinal subtype is more common in younger patients.
B. intestinal subtype carries worse prognosis.
C. intestinal subtype usually has no identifiable precursor.
D. intestinal subtype usually arises in the cardia.
E. intestinal subtype usually forms discrete ulcer or mass.
ANSWER: E

All the following statements regarding Gastrointestinal Stromal


tumors (GISTs)are true except
A. The diagnosis of GIST is based on the presence of more than 10
mitotic figures on high-power field microscopy .
B. GISTs usually don't metastasize to lymph nodes.
C. GISTs are usually resistant to conventional chemotherapy and
radiotherapy.
D. Extra gastric GISTs carry worse prognosis .
E. Complete surgical resection is the standard of treatment .
ANSWER: A

In laparoscopic adrenal surgery for pheochromocytoma; hypotension


is anticipated upon:
A. Induction of anesthesia.
B. Insufflation of gas.
C. Ligation of adrenal arteries.
D. Ligation of adrenal vein .
E. Manipulation of the mass.
ANSWER: D

All the following are true about coronary arteries (CA) anatomy
except
A. They are branches of the ascending aorta .
B. They arise from ostia in the aortic sinuses.
C. Three coronary arteries arise from the aorta.
D. The worst CA stenosis prognosis is that located at the left
main CA.
E. The right CA passes anteriorly between the right atrial
appendage and the pulmonary trunk.
ANSWER: C

All the following are true about the venous ulcer except:
A. Usually has a sloping edge.
B. The floor contains granulation tissue covered by slough and
exudate.
C. An elevation in the edge may indicate malignant change.
D. Characteristically develop at the gaiter’s area.
E. Most of them develop at the lateral aspect of the calf.
Dr. wessam alzaidat - General surgery
ANSWER: E

All the following are true about anal tumors except:


A. The commonest serotype of human papilloma virus that causes
anal squamous cell carcinoma (SCC) is type 6 serotype.
B. Bleeding is the commonest presentation for SCC.
C. The first line treatment for SCC is chemoradiation .
D. Anal adenocarcinoma is more aggressive than SCC.
E. Symptomatic anal GIST carries worse prognosis.
ANSWER: A

Regarding Warthin tumor of the salivary glands all are true except
A. There is no risk of malignancy.
B. Can be treated by enucleation.
C. It may be cystic.
D. Occurs in male in 90% 0f cases.
E. Occurs equally in parotid and submandibular glands.
ANSWER: E

The first branch to the colon from the superior mesenteric artery
is
A. Ileocolic.
B. Right colic.
C. Middle colic.
D. Left colic.
E. Superior rectal.
ANSWER: C

A 6-year-old male presented to the emergency room with his mother,


his mother said that he felt from the second floor. He was
conscious, talking and walking, his pulse was 150 per minute, his
pressure 90 over 50 mmHg. He has bruises over the scalp and chest.
Within the last 6 months he was presented to the emergency twice
for falling down, one of them he had leg fracture. All the
following are suitable actions to do with this child except
A. Admit him to the ICU.
B. Take blood sample for CBC.
C. Do FAST ultrasound.
D. Do chest X-ray.
E. Inform child protection authorities about the case.
ANSWER: A

A 35-year-old female who is known to have insulin dependent


diabetes, operated for perforated duodenal ulcer. 6 hours after
surgery you were called to see her, she was stuporous, has
tachycardia, normal pulse with urine output 1200cc over 6 hours.
You asked for lab tests and revealed Hb 14.5, WBC 15000, blood
sugar 550 mg per dL. She most likely has
A. Hypernatremia.
B. Post-operative bleeding.
C. Wide anion gap metabolic acidosis.
D. High output renal failure.
E. Overhydration.
Dr. wessam alzaidat - General surgery
ANSWER: C

A 68-year-old smoker male presented with 6 months history of pain


during walking in his right thigh and calf. You suspect that he
has a significant disease at which of the following arteries?
A. Aorta.
B. Common iliac.
C. Femoral bifurcation.
D. Superficial femoral.
E. Deep femoral.
ANSWER: B

Mesh plug is more suitable to repair


A. Direct inguinal hernia.
B. Indirect inguinal hernia.
C. Paraumbilical hernia.
D. Femoral hernia.
E. Incisional hernia.
ANSWER: D

In split thickness skin graft we harvest


A. Part of the thickness of the epidermis.
B. The whole thickness of the epidermis leaving the whole dermis.
C. The whole thickness of the epidermis with part of the dermis.
D. The whole thickness of the epidermis and dermis.
E. The whole thickness of the skin with the superficial part of
the subcutaneous tissue.
ANSWER: C

All the following are true about liver hydatid cysts except
A. Echinococcus granulosus is the main causative organism.
B. The adult worm present in the dog intestine.
C. The causative organism is a tapeworm.
D. In human, the ova pass to the liver through intestinal
lymphatics.
E. Ultrasound is a good screening test for the condition.
ANSWER: D

After cholecystectomy you opened the gall bladder and found -in
addition to stones- pink to red mucosa with yellow elevations
looking like strawberry. This appearance in called
A. Cholesterosis.
B. Cholesterol polyposis.
C. Cholecystitis glandularis proliferans.
D. Intramuscular stones.
E. Typhoid cholecystitis.
ANSWER: A

A barium swallow is good in identifying all the following


pathologies except
A. Large hiatal hernia.
B. Gastroesophageal reflux.
C. Esophageal motility disorders.
Dr. wessam alzaidat - General surgery
D. Esophageal strictures.
E. Barrett’s mucosa
ANSWER: E

All the following are alarming symptoms that indicate the need for
upper GI endoscopy in a 40-year-old female except
A. Recurrent epigastric pain.
B. Weight loss.
C. Dysphagia.
D. Frequent recurrent vomiting.
E. Anemia.
ANSWER: A

All the following about adenomatosis polyposis coli APC gene are
true except
A. It is a tumor suppressor gene.
B. APC defects are present in patients with familial adenomatosis
polyposis FAP.
C. APC defects are rarely present in sporadic cases of colorectal
cancer.
D. Mutations of both alleles of APC gene is necessary to initiate
polyp formation.
E. In FAP the site of mutation in the APC gene correlates with the
disease severity.
ANSWER: C

Dr. wessam alzaidat - General surgery


4th year exam 2020 MOH

The most sensitive test for pheochromocytoma is:


A. Plasma free metanephrines .
B. 24 h urinary catecholamines.
C. 24 h urinary Vanillylmandelic acid (VMA.)
D. Clonidine suppression test.
E. Plasma catecholamines.
ANSWER: A

One of the following statements is true concerning mammography:


A. Up to 80% of cancers detected mammographically are not
palpable.
B. One third of palpable breast cancers are not detected by
mammography.
C. The sensitivity of mammography increases with age.
D. The American Cancer Society currently recommends routine
screening mammography beginning at age 50.
E. Only about 10% of nonpalpable lesions detection
mammographically are found to be malignant at biopsy.
ANSWER: C

Which of the following arteries is most likely to be involved with


serious atherosclerosis?
A. The right coronary artery.
B. The left coronary artery.
C. The left anterior descending coronary artery.
D. The circumflex coronary artery.
E. The obtuse marginal artery.
ANSWER: C

The most common cause of rectovaginal fistula is:


A. Colon carcinoma.
B. Diverticulitis.
C. Crohns disease.
D. Obstetric injury.
E. Bladder carcinoma.
ANSWER: D

In nutritional therapy, the commonest cause of complications is


A. Overfeeding.
B. Protein insufficiency.
C. Dehydration.
D. Hyponatremia.
E. Vitamins deficiency.
ANSWER: A

Biliary radionuclide scanning as HIDA scan can be valuable in the


diagnosis of the following conditions except
A. Hepatocellular dysfunction.
B. Biliary atresia.
C. Common bile duct injury.
D. Acute cholecystitis.
Dr. wessam alzaidat - General surgery
E. Bile leak.
ANSWER: A

Ghrelin
A. Decreases insulin release.
B. Decreases glucagon release.
C. Inhibits GI secretion.
D. Inhibits pancreatic exocrine secretion.
E. Is released from pancreatic beta cells.
ANSWER: A

After significant blunt trauma, a patient arrives to the emergency


room with Glasgow Coma Scale (GCS) of 4/15. His oxygen saturation
is poor at 89% after applying a non-rebreathing mask. His eyes are
bruised and blood-stained fluid is coming out of his ears. The
method of airway management that is best avoided :
A. Oropharyngeal intubation
B. Laryngeal mask airway
C. Cricothyroidotomy is oropharyngeal is unsuccessful
D. Nasopharyngeal intubation
E. Oropharyngeal intubation with bronchoscopy guidance
ANSWER: D

A 48-year-old man with circumferential burn in his right arm,


presented to the emergency room. After initial assessment and
resuscitation, he started complaining of increasing pain,
paresthesia and weakening peripheral pulses. Which is the best
next action?
A. Angiography
B. Escharotomy
C. Electrolyte assay and replacement
D. Fluid resuscitation
E. Fasciotomy
ANSWER: B

If a patient undergoing thoracotomy receives intercostal blocks


with bupivacaine, his or her postoperative period will
A. Be little different from controls
B. Show marked improvement in respiratory function over controls
C. Show little difference in vital capacity but marked pain relief
D. Be marked by hyperventilation
E. Be marked by increased incidence of atelectasis
ANSWER: B

All of the following may be acceptable operative approaches to


management of the thoracic outlet syndrome except:
A. Scalenectomy.
B. Excision of a cervical rib.
C. Thoracoplasty.
D. First rib resection.
E. Division of anomalous fibromuscular bands.
ANSWER: C

Dr. wessam alzaidat - General surgery


Lateral neck dissection for papillary thyroid tumor includes lymph
nodes in :
A. Levels 1, 2 and 3.
B. Levels 2, 3 and 4.
C. Levels 3, 4 and 5.
D. Levels 4, 5 and 6.
E. All levels from 1 to 6.
ANSWER: B

In parotid surgery, the most frequent injury involves:


A. Lesser occipital nerve.
B. Greater auricular nerve.
C. Facial nerve.
D. Transverse cervical nerve .
E. Hypoglossal nerve.
ANSWER: B

One of the following is a relative indication for mitral valve


replacement, as opposed to mitral valve repair:
A. Extensive leaflet calcification.
B. Mitral regurgitation.
C. Chordal rupture of the anterior mitral leaflet.
D. Significant annular dilatation.
E. Association with mild tricuspid regurge.
ANSWER: A

One of the following is true regarding short bowel syndrome in


adults?
A. Competent ileocecal valve is associated with decreased
malabsorption.
B. It is defined as less than 300 cm of residual small bowel.
C. Resection of the ileum is better tolerated than resection of
the jejunum.
D. The presence of an intact colon does not alter the severity.
E. It is most commonly caused by multiple operations requiring
small bowel resection.
ANSWER: A

A 34 years old a symptomatic female has an incidental finding of


right liver lobe mass in ultrasound. CT scan showed a well
circumscribed 5 by 5 cm mass with homogenous enhancement in the
arterial phase with central scar and isodense in venous phase. The
most probable diagnosis is
A. Hepatic adenoma.
B. Hemangioma.
C. Focal nodular hyperplasia.
D. Hepatocellular carcinoma.
E. Cholangiocarcinoma.
Answer: C

During a gastrectomy for cancer, you attending surgeon asks you


what lymph nodes are not involved D2 lymphadenectomy:
A. Hepatic artery lymph nodes
Dr. wessam alzaidat - General surgery
B. Splenic artery lymph nodes
C. Celiac trunk lymph nodes
D. Left portal vein lymph nodes
E. Para-aortic lymph nodes
ANSWER: E

Which of the following would be requires a biopsy confirming the


presence of cancer first prior to definitive management ?
A. An ‘apple core’ lesion in the ascending colon
B. A pancreatic head mass with painless jaundice
C. A palpable breast mass with skin puckering
D. Highly vascular painful mass in the thigh, abutting but not
invading the neurovascular bundle
E. A 2 cm parotid mass with an intact facial nerve
ANSWER: C

On exploration of a 43-year-old male patient who sustained blunt


trauma with pelvic fracture and free abdominal fluid on FAST
ultrasonography, you find a central (Zone I) expanding hematoma
and a contained pelvic (Zone III) hematoma. Which of the following
is the best method of management ?
A. Observation of both hematomas
B. Explore both hematomas
C. Observe the central hematoma, but explore the pelvic hematoma
D. Just apply an external pelvic fixator and close the abdomen
since no free blood is present
E. Explore the central hematoma after obtaining proximal and
distal control, and observe the pelvic hematoma
ANSWER: E

A 14 month old child is admitted with colicky abdominal pain and


on investigation is found to have an ileo-ileal intussusception.
What is the best course of action?
A. Attempt hydrostatic reduction with barium enema
B. Attempt pneumatic reduction with air insufflation
C. Undertake a laparotomy
D. Undertake a colonoscopy
E. Undertake a flexible sigmoidoscopy
ANSWER: C

A patient is being weaned from mechanical ventilation. Weaning


parameters are obtained prior to deciding on extubation.
Successful weaning from a ventilator is suggested by the presence
of which of the following ?
A. An alveolar arterial gradient of more than 350 mm Hg
B. APaO2/FiO2 ratio of <200
C. APaCO2 over 55 mm Hg
D. A tidal volume of over 5 mL/kg
E. A minute ventilation of 12 L/min
ANSWER: D

Giant gastric ulcers (5 cm or longer), one of the following is


true.
Dr. wessam alzaidat - General surgery
A. Commonly present with free perforation
B. Are unlikely to have foci of microscopic malignancy
C. Are prone to severe hemorrhage
D. Are associated with mechanical gastric outlet obstruction
E. Do not differ from ordinary gastric ulcers
ANSWER: C

A 25 years old female who underwent sleeve gastrectomy for obesity


6 months ago, presented with 3 months history of vomiting, then
she started visual disturbance and inability to stand, then
rapidly developed deterioration in the level of consciousness.
Mostly she has
A. Sever protein malnutrition .
B. Hypoglycemia.
C. Dumbing syndrome.
D. Thiamin deficiency.
E. Steatohepatitis.
ANSWER: D

All of the following conditions are associated with low cardiac


output except
A. Early septic shock.
B. Cardiac tamponade.
C. Tension pneumothorax.
D. Massive pulmonary embolism.
E. Air embolism.
ANSWER: A

Capacitance coupling may injure organs in laparoscopic surgery.


What is the best way to eliminate this risk?
A. Decrease the coagulation power.
B. Use entirely plastic ports.
C. Put pads around hollow organs.
D. Immerse the tissues in normal saline.
E. Use 30 degrees lens.
ANSWER: B

Regarding tetanus, all of the following are true except


A. A short period of prodromal symptoms carries a better
prognosis.
B. The causative organism is gram positive.
C. The causative organism is rod in shape.
D. Wound debridement may be a part of treatment.
E. Benzyl penicillin is the antibiotic of choice.
ANSWER: A

All of the following are true regarding amebic infection except


A. The vast majority of carriers are asymptomatic.
B. Amebic liver abscess needs drainage usually.
C. The parasite reaches the liver via the portal circulation.
D. Most of amebic liver abscesses located in the right liver lobe.
E. Ameboma presents as a colonic mass.
ANSWER: B
Dr. wessam alzaidat - General surgery
All of the following carry a higher risk of post ERCP pancreatitis
except
A. Male gender.
B. Young age.
C. Normal serum bilirubin.
D. Balloon dilatation of the papillary sphincter.
E. Previous ERCP-related pancreatitis.
ANSWER: A

All the following statements about the esophageal sphincters are


true except
A. The upper and lower esophageal sphincters are high pressure
zones rather than actual anatomic landmarks.
B. On swallowing, the upper esophageal sphincter pressure reaches
up to 90 mmHg.
C. The inferior constrictor muscle is the main contributor to the
upper esophageal sphincter high pressure zone.
D. Vagal mediated relaxation of the lower esophageal sphincter
occurs during food transit.
E. Gastrin and motilin increase lower esophageal pressure.
ANSWER: C

When comparing the intestinal subtype to the diffuse subtype of


gastric adenocarcinoma, which of the followings is true
A. intestinal subtype is more common in younger patients.
B. intestinal subtype carries worse prognosis.
C. intestinal subtype usually has no identifiable precursor.
D. intestinal subtype usually arises in the cardia.
E. intestinal subtype usually forms discrete ulcer or mass.
ANSWER: E

A 50-year-old postmenopausal woman with no family history of


breast cancer underwent bilateral breast reduction surgery and was
found to have lobular carcinoma in situ in the reduction specimen
from the left breast. Her preoperative mammogram was normal, and
she has no history of prior breast abnormalities. The patient
should be advised that lobular carcinoma in situ
A. Should be treated like ductal carcinoma in situ.
B. Is an incidental finding in the breast tissue that does not
increase the risk of breast cancer.
C. Is a risk factor for the future development of breast cancer
only in the breast in which it was found.
D. Is a risk factor for the future development of breast cancer in
both breasts.
E. Is a precursor to invasive lobular breast cancer and should be
treated by mastectomy.
ANSWER: D

All the following are true about testicular torsion except


A. It occurs most often in adolescents.
B. It occurs usually outside the tunica vaginalis.
C. The pain is not always scrotal.
Dr. wessam alzaidat - General surgery
D. The torsion must be relieved within 6 hours from the onset of
pain.
E. After detorsion, bilateral orchidopexy is indicated.
ANSWER: B

All the following are true regarding infantile hypertrophic


pyloric stenosis are true except
A. Associated with bilious vomiting.
B. Occur mostly between 2 and 8 weeks of age.
C. More in males.
D. After a feed, a visible gastric peristalsis from left to right
can be seen.
E. The diseased infant develops hypochloremic hypokalemic
alkalosis .
ANSWER: A

All of the followings are true except:


A. Grafts depend on recipient site for nutrition while flaps have
their own blood supply.
B. Imbibition occurs in the first stage after grafting (0-3 days)
in which the graft receives nutrients from plasma through direct
contact.
C. Full thickness skin graft develops more secondary contracture
than split thickness skin graft (STSG .)
D. STSG has high a higher chance of graft survival .
E. Scalp is a possible donor site for facial skin loss.
ANSWER: C

All the following are compatible with Milan criteria for the
indications of liver transplantation for patients with
hepatocellular carcinoma except
A. Single tumor up to 5cm in longest dimension.
B. Up to 3 tumors the largest is up to 3cm in longest dimension.
C. Absence of gross vascular invasion.
D. Absence of extrahepatic spread.
E. Absence of cirrhosis.
ANSWER: E

All the following are true about gallbladder hydrops except


A. It is caused by an impacted stone in the cystic duct.
B. The gallbladder is distended and may be palpable.
C. The gallbladder is full with mucous without bile.
D. The patient usually has right upper abdominal tenderness.
E. Cholecystectomy is indicated.
ANSWER: D

The use of noradrenaline gives the best response in shock due to


A. Sever pancreatitis.
B. Anaphylaxis.
C. Sever GI bleeding.
D. Cardiac tamponade.
E. Acute heart failure.
ANSWER: B
Dr. wessam alzaidat - General surgery
Which of the following is not part from the intrinsic-common
pathway of coagulation
A. Factor II.
B. Factor V.
C. Factor VII.
D. Factor VIII.
E. Factor XI.
ANSWER: C

In the genetic model for colorectal tumorigenesis, which of the


following genetic hits occur latest?
A. APC mutation.
B. DNA methylation.
C. Loss of p53.
D. K-ras mutation.
E. Loss of DCC.
ANSWER: C

All the following are true about Cyclosporine except


A. It is a calcineurin inhibitor.
B. Suppresses the activation of B-lymphocytes.
C. It is metabolized by the cytochrome P450 system.
D. It is nephrotoxic.
E. It is neurotoxic.
ANSWER: B

All the following are true regarding pressure ulcers except


A. Ischemia results if the pressure on the tissue exceeds the
pressure of the microcirculation (30 mmHg.)
B. Ischial tuberosities are the commonest areas for pressure
ulcers.
C. Skin is more susceptible than muscle to pressure related
ischemic injury.
D. Relief of pressure is a corner-stone in management.
E. Antibiotic treatment is not indicated in many cases.
ANSWER: C

The breast receives blood supply from all the following arteries
except
A. Perforating branches from the internal mammary artery.
B. Branches from the anterior intercostal arteries.
C. The highest thoracic artery.
D. Lateral thoracic artery.
E. The brachial artery.
ANSWER: E

All the following are true about invasive lobular carcinoma of the
breast except
A. It is less common than invasive ductal carcinoma.
B. It is commonly bilateral.
C. It is commonly multifocal.
D. About 30% express estrogen receptors.
Dr. wessam alzaidat - General surgery
E. It has insidious growth pattern.
ANSWER: D

Right ankle-brachial pressure index is measured as


A. The mean of right posterior tibial PT and dorsalis pedis DP
systolic pressure divided by the right arm systolic pressure.
B. The mean of the right PT and DP systolic pressures divided by
the mean of both arms’ systolic pressures.
C. The highest mean arterial pressure of right PT and DP divided
by the mean right arm pressure.
D. The highest systolic pressure of the right PT and DP divided by
the right arm systolic pressure.
E. The highest systolic pressure of the right PT and DP divided by
the highest of both arms systolic pressure.
ANSWER: E

All the following increase the risk for thrombo-embolic phenomena


except
A. Factor V Leiden.
B. Antithrombin deficiency.
C. Protein C deficiency.
D. Protein S deficiency.
E. Factor XI elevation.
ANSWER: E

All the following are true about desmoid tumors of the abdominal
wall except
A. They originate from the Musculo-aponeurotic structures of the
anterior abdominal wall.
B. They infiltrate locally their surroundings.
C. They rarely metastasize distally.
D. They may associate familial adenomatosis polyposis FAP.
E. Surgical excision with safety margin is indicated if feasible.
ANSWER: C

In Lichtenstein repair of inguinal hernia, which of the following


techniques is indicated to decrease medial recurrence of the
hernia?
A. The lateral border of the mesh is split above and below the
spermatic cord.
B. The medial aspect of the mesh is fixed to the pubic tubercle.
C. The mesh is fixed to the shelving edge of the inguinal
ligament.
D. The medial edge of the mesh is sutured to the anterior rectus
sheath.
E. The mesh is tailored to fit well around the spermatic cord but
not too tight.
ANSWER: D

All the following are true about the spleen except


A. Most accessory spleens locate in the splenic hilum and vascular
pedicle.
B. Most splenic ligaments are avascular.
Dr. wessam alzaidat - General surgery
C. In most cases the tail of pancreas locates either abutting the
spleen or within 1 cm of the hilum.
D. In about 70% of people the splenic artery divides near its
hilum into short terminal branches.
E. It receives part of its blood supply from the short gastric
vessels.
ANSWER: D

All the following are true about acute cholecystitis except


A. The vast majority is caused by gallstones.
B. The initiating event is typically a stone of the cystic duct.
C. Initially acute cholecystitis is an inflammation.
D. Increase prostaglandin synthesis amplifies the inflammatory
process .
E. Secondary bacterial infection happens in 75% of cases.
ANSWER: E

All the following are known presentations of somatostatinomas


except
A. Hypoglycemia.
B. Gallstones.
C. Steatorrhea.
D. Jaundice.
E. Abdominal pain.
ANSWER: A

Dr. wessam alzaidat - General surgery


5th year exam 2020 MOH

One of the followings is used in the treatment of Conn`s syndrome


due to adrenal hyperplasia:
A. Mitotane .
B. Spironolactone.
C. Metyrapone.
D. Ketoconazole.
E. Prednisolone.
ANSWER: B

Surgery for thyroglossal cyst usually includes excision of:


A. Part of Cricoid cartilage.
B. Part of Hyoid bone .
C. Part of Thyroid cartilage.
D. Part of overlying skin.
E. Part of Tongue muscles.
ANSWER: B

The most appropriate management of diverticulitis with an


associated 3-cm pelvic abscess in stable patient is:
A. Hartmann procedure.
B. Sigmoid resection with primary anastomoses.
C. Parenteral antibiotics.
D. Percutaneous drainage.
E. Exploratory laparotomy.
ANSWER: C

Healthy 22-year-old pregnant female who develops Clostridium


difficile colitis after antibiotic treatment should receive which
of the following regimens?
A. Per oral metronidazole.
B. Per oral vancomycin.
C. IV metronidazole.
D. IV vancomycin.
E. No treatment in pregnant patients.
ANSWER: B

All the following are true about bilirubin except


A. It is the breakdown product of heme.
B. The unconjugated bilirubin is bound to albumin in the
circulation.
C. In the liver it is conjugated to glucuronic acid.
D. The majority of conjugated bilirubin is excreted normally in
urine.
E. Urobilinogen is formed by the action of enteric bacteria on
bilirubin.
ANSWER: D

All the following are true about liver hydatid cyst except
A. It is more common in the right liver lobe.
B. The sensitivity of ELISA is about 85%.
C. When possible, complete surgical excision of the cyst without
Dr. wessam alzaidat - General surgery
rupture is preferred.
D. Small cysts can be managed by albendazole and observation.
E. Water Lilly appearance indicate high activity of the cyst.
ANSWER: E

To prevent hyperacute rejection in solid organ transplant, one of


the following is true:
A. Ensure HLA matching is present
B. Give FK 506 (Prograf)
C. Ensure ABO blood group compatibility
D. Give Prednisone
E. Give Azathioprine
ANSWER: C

Parkland formula is used to calculate fluid requirements of burn


patients. For a 70-kg adult with a 30% second degree burn, the
total fluid requirements in the first eight hours is:
A. 1.4 liters
B. 2.8 liters
C. 4.2 liters
D. 7 liters
E. 8.4 liters
ANSWER: C

In a trial comparing One Anastomosis Gastric Bypass (OAGB) with


Roux-en-Y Gastric Bypass (RYGB), patients with type II diabetes
achieved complete remission in 38% in RYGB VS 60% of in OAGB, with
P value of 0.28. With respect to the P value, one of the following
is true:
A. OAGB resulted in a statistically significant higher remission
rate
B. OAGB resulted in higher remission rate but did not reach
statistical significance
C. OAGB resulted in a higher remission rate with less than 5%
probability to be due to chance
D. P value in this scenario indicates the results are well-powered
E. P value actually indicates results of RYGB are better
ANSWER: B

Frozen section has the least value in differentiating:


A. Malignant from benign thyroid nodules.
B. Parathyroid adenoma from hyperplasia .
C. Papillary from Follicular thyroid neoplasm.
D. Reactive from Malignant lymph node.
E. Parathyroid gland from goitrous thyroid nodule.
ANSWER: B

In thyroid surgery; routine central neck dissection is an


essential part in the treatment of :
A. Papillary thyroid carcinoma .
B. Follicular thyroid carcinoma.
C. Medullary thyroid carcinoma .
D. Anaplastic carcinoma.
Dr. wessam alzaidat - General surgery
E. Thyroid Lymphoma.
ANSWER: C

All are true regarding parotid pleomorphic adenoma except:


A. Presents usually as a slowly growing painless lump.
B. Can present with facial nerve weakness.
C. Treatment does not include radiotherapy when totally excised.
D. Treatment does not include neck dissection.
E. The minimum accepted operation is a superficial parotidectomy.
ANSWER: E

In a patient with a zone 3 penetrating neck injury presented to


the emergency room with neck hematoma and external bleeding
through the wound controlled by pressure, he is fully conscious
and has stable vital signs. The next step is:
A. Admission to theatre and neck exploration under general
anesthesia.
B. Wound exploration in emergency room under local anesthesia.
C. Angiogram .
D. Bronchoscopy and Esophagoscopy.
E. Admission to an intensive care unit, maintain semi setting
position and keep the pressure dressing for 24 hours.
ANSWER: C

The most common cause for recurrent diverticulitis after sigmoid


resection is:
A. Pan colonic diverticular disease.
B. Inadequate distal resection margin.
C. Use of a side-to-end stapled technique.
D. Dietary indiscretion .
E. Early enteral feeding.
ANSWER: B

All the following are true regarding alcoholic chronic


pancreatitis except
A. It doesn’t occur in patients with low daily consumption.
B. The risk of the disease is high among persons who consume 150
cc or more of alcohol daily.
C. The onset of symptoms typically starts after more than 10 years
of alcohol abuse.
D. The disease appears in about 10% of alcohol abusers.
E. Genetic factors may play a role in the development of the
disease.
ANSWER: A

All the following are true about insulinomas except


A. About 90% are benign.
B. About 90% are solitary.
C. About 90% are sporadic.
D. About 10% are associated with multiple endocrine neoplasia type
2.
E. Larger tumors are more likely to be malignant.
ANSWER: D
Dr. wessam alzaidat - General surgery
All of the following carry a higher risk of post ERCP pancreatitis
except
A. Female gender.
B. Young age.
C. Very high serum bilirubin.
D. Balloon dilatation of the papillary sphincter.
E. Previous ERCP-related pancreatitis.
ANSWER: C

All the following are true about ultrasound guided percutaneous


drainage of the gall bladder with a pig-tail catheter except
A. It is advised to avoid passing the catheter through the liver.
B. It is indicated to decompress a purulent gall bladder in a
patient not fit for surgery.
C. When the gall bladder inflammation improves, the catheter is
removed.
D. If the general condition of the patient improved, the gall
bladder can be removed later.
E. The catheter can be used later for dissolution therapy if
indicated.
ANSWER: A

In a laparoscopic inguinal hernia repair through a trans-abdominal


approach in a male patient, the consultant surgeon discusses with
you what forms the triangle of doom. The medial border of the
triangle represents :
A. Cooper’s ligament
B. Vas deferens
C. Gonadal vessels
D. Iliac vessels
E. Iliopubic tract
ANSWER: B

With regards to metastatic disease, one of the following is true:


A. Axillary lymph node dissection is essential for staging of
sarcoma of the breast
B. Primary brain tumors have a predilection to metastasize to the
lungs
C. Bone is an infrequent site for metastasis for cancer of the
breast and prostate
D. Melanoma tends to metastasize to lungs, brain, and the
gastrointestinal tract
E. Tumor thrombus of renal cell tumor invades Aorta more
frequently than Inferior Vena Cava
ANSWER: D

A 30-year-old man, suffers a stab wound medial to the left nipple.


He had a weak pulse in the ambulance 10 minutes ago, but has no
palpable pulse or recordable blood pressure on your examination,
while his pupils are still reactive. The initial surgical approach
is:
A. Laparotomy
Dr. wessam alzaidat - General surgery
B. Left postero-lateral thoracotomy
C. Left clavicular incision
D. Left antero-lateral thoracotomy
E. Right antero-lateral thoracotomy
ANSWER: D

A 14-year-old boy is diagnosed with a duodenal hematoma after a


blunt trauma. He has no other injuries or findings. Please choose
the true answer pertaining to his diagnosis:
A. Patients present commonly one week after injury
B. Patients generally present with peritonitis
C. Patients almost always need exploration of the hematoma since
it is in zone I
D. Patients usually present with obstruction symptoms and need
total parenteral nutrition till hematoma resolves
E. Patients generally eat well and can be managed well at home
ANSWER: D

A 73-year-old man undergoes a laparotomy for mesenteric


infarction. An extensive small bowel resection is undertaken. A
planned re-look laparotomy is scheduled to occur in 24 hours'
time. What is the most appropriate closure strategy in this
situation?
A. Mass closure of the abdomen obeying Jenkins rule using 2 PDS
B. Application of a Bogota bag
C. Mass closure of the abdomen obeying Jenkins rule using 2/0 PDS
D. Mass closure of the abdomen obeying Jenkins rule using 2 nylon
E. Mass closure of the abdomen obeying Jenkins rule using 2/0
nylon
ANSWER: B

A group of surgeons wish to determine whether patients are


receiving adequate deep vein thrombosis prophylaxis following
surgery. What is the most appropriate process to determine this?
A. Departmental review
B. Peer review
C. Financial audit
D. Systems based audit
E. Standards based audit
ANSWER: E

Absolute contraindications for bariatric surgery include:


A. Cardiomyopathy
B. Pickwickian syndrome
C. Type 1 diabetes mellitus
D. Nonalcoholic steatotic hepatitis
E. Recent coronary arteries stents
ANSWER: E

Which of the following statements regarding management of


intussusception in adults is true?
A. Barium enema with pneumatic decompression
B. Exploratory laparotomy, manual reduction and resection of the
Dr. wessam alzaidat - General surgery
involved segment
C. Exploratory laparotomy and manual reduction
D. Exploratory laparotomy and intestinal bypass
E. Nasogastric tube placement, intravenous fluid, and trial of
nonoperative management
ANSWER: B

All are true about Colonic Diverticulosis except:


A. 10-20% of all colonic diverticulosis develops symptoms. Rest
remains asymptomatic.
B. Low fiber diet is implicated in etiology .
C. Sigmoidoscopy is a useful tool for evaluation of acute
diverticulitis
D. Elective colectomy in uncomplicated diverticulitis is rare and
forms only 1% of cases with diverticulosis.
E. It may present with abdominal pain, constipation, diarrhea or
passage of blood or mucus from the rectum.
ANSWER: C

67years old female with 12 months history of dysphagia is


admitted to the hospital for pneumonia. Gastrographin swallow
shows a tracheoesopageal fistula. Upper endoscopy biopsies showed
squamous cell carcinoma. What is the preferred treatment?
A. Esophagectomy
B. Cervical esophagostomy and percutaneous endoscopic gastrostomy
(PEG) tube placement.
C. Esophageal stent placement
D. Radiotherapy
E. Chemoradiotherapy followed by esophagectomy
ANSWER: C

Cytokines involved in the initial proinflammatory response include


all of the following except:
A. Interleukin-6
B. Interleukin-10
C. Tumor necrosis factor-α
D. Interleukin-1
E. Interleukin-8
ANSWER: B

Which of the following is the best parameter for monitoring septic


shock?
A. Central venous pressure (CVP)
B. Vasopressor requirement
C. Urine output
D. Serum lactate
E. Mental status changes
ANSWER: D

Which of the following clinical conditions is identified by the


presence of antibodies in the serum against HBsAg (anti- HBs) in
the absence of hepatitis B core antigen (anti-HBc) and HBsAg? The
patient
Dr. wessam alzaidat - General surgery
A. Is susceptible to HBV infection.
B. Is immune because of HBV vaccination .
C. Has an active acute infection with HBV.
D. Has chronic active hepatitis with HBV.
E. Has recovered from an HBV infection with subsequent natural
immunity.
ANSWER: B

Which of the following is the primary fuel source in prolonged


simple starvation?
A. Liver glycogen
B. Muscle protein
C. Muscle glycogen
D. Ketone bodies
E. Dextrose
ANSWER: D

All of the following are true regarding head injury EXCEPT :


A. Epidural hematomas are associated with a lucid interval .
B. Subdural hematomas are due to lacerations of bridging veins .
C. Epidural hematomas are usually associated with a skull
fracture .
D. Some small subdural hematomas can be managed nonoperatively .
E. Epidural hematomas generally have a worse prognosis than
subdural hematomas.
ANSWER: E

One month after an antrectomy with Billroth II reconstruction, a


patient presents with colicky abdominal pain, distention, bilious
emesis, and failure to pass gas. This most likely represents:
A. Blind loop syndrome
B. Afferent loop syndrome
C. Reflux gastritis
D. Efferent loop syndrome
E. Vitamin deficiency
ANSWER: D

A 49-year-old woman is diagnosed with Paget’s disease of the


nipple. What is the most appropriate treatment?
A. Excision of nipple-areola complex
B. Nipple-areola conserving recentralization surgery
C. Radiotherapy alone
D. Six-monthly mammography follow-up
E. Total duct excision
ANSWER: A

Which breast’s pathology, presents with dark, multifocal, painless


skin nodules and is most commonly associated with radiation and
long-standing lymphoedema ?
A. Primary angiosarcoma of the breast
B. Lymphoma of the breast
C. Melanoma of the breast
D. Secondary angiosarcoma of the breast
Dr. wessam alzaidat - General surgery
E. Pseudoangiomatous stromal hyperplasia
ANSWER: D

A 48-year-old woman had established renal failure secondary to


diabetic nephropathy, for 10 years, until 1 year ago when she
underwent a successful renal transplant. She now presents to acute
medical services with painful white lesions on her shins and
forearms, and is found to have a serum calcium level of 3.1
mmol/L. the single most likely diagnosis is?
A. MEN-2B syndrome
B. Pleomorphic adenoma
C. Primary hyperparathyroidism
D. Secondary hyperparathyroidism
E. Tertiary hyperparathyroidism
ANSWER: E

When assessing a patient in clinic with a suspected melanoma,


which one of the following would be most in keeping with
development of a nodular subtype?
A. Development within a long-standing mole
B. Development de novo
C. Development from a Hutchinson freckle
D. Development within a sun-protected site
E. Development from a Spitz nevus
ANSWER: B

All the following are true regarding inguinal hernia and hydrocele
in children except
A. Inguinal hernias are more common on the right side.
B. Inguinal hernias are almost always indirect.
C. The majority of hydroceles resolve spontaneously.
D. In infants, scrotal transillumination differentiates well
between the two conditions.
E. In many cases the diagnosis of indirect hernia in infants
depends upon the history.
ANSWER: D

All the following are true regarding the treatment of lower limbs
arterial stenosis except:
A. A femoral artery puncture is used to access the vessels.
B. Balloon dilatation is commonly used for short segment stenosis.
C. The results are better when treating vessels below the knee.
D. If the dilated segment fails to keep dilated after the balloon
in removed, a metallic stent is applied.
E. Surgical operations are reserved for patients with severe
symptoms when angioplasty has failed or is not possible.
ANSWER: C

All are true about colonic volvulus except:


A. The commonest site of colonic volvulus is the sigmoid.
B. Cecal volvulus commonly presents in young females.
C. Volvulus can present in the splenic flexure.
D. Pregnant women are less prone for volvulus.
Dr. wessam alzaidat - General surgery
E. Sigmoid volvulus in pregnancy is classically treated by
Hartmann’s procedure.
ANSWER: D

All the following increase the risk of breast cancer except


A. Intraductal papilloma.
B. Florid hyperplasia.
C. A typical lobular hyperplasia.
D. Lobular carcinoma insitu.
E. Ductal carcinoma insitu.
ANSWER: A

All the following are true about BRCA1 mutation except


A. BRCA1 is a tumor suppressor gene.
B. It represents at least 80% of hereditary ovarian cancers.
C. Female mutation carriers have higher lifetime risk to develop
breast cancer than ovarian cancer.
D. Most of BRCA1 related breast cancers have positive hormone
receptors.
E. It is the commonest known cause of hereditary breast cancer.
ANSWER: D

A 44-year-old female has left breast microcalcifications of 6cm in


maximum dimension. True-cut biopsy showed ductal carcinoma insitu.
Which of the following represent the best therapeutic option?
A. Wide local excision.
B. Unilateral mastectomy.
C. Bilateral mastectomy.
D. Prophylactic hormonal therapy.
E. Observation.
ANSWER: B

All the following are true about abdominal aortic aneurysms AAA
except
A. Most are infrarenal.
B. Most are fusiform in shape.
C. Most are due to atherosclerosis.
D. Associated peripheral arterial occlusive disease presents in
most cases.
E. CT scan is required for definitive evaluation of AAA.
ANSWER: D

Stab avulsion in treatment of varicose veins is best done for


A. Small branch varicosity.
B. Large branch varicosity.
C. Long saphenous vein.
D. Short saphenous vein.
E. Sapheno-popliteal incompetence .
ANSWER: B

A 30-year-old female with idiopathic thrombocytopenic purpura


scheduled for splenectomy, her platelets count is 15000 per cubic
mm. what is best recommended to prepare her for splenectomy?
Dr. wessam alzaidat - General surgery
A. High dose pulse steroid therapy.
B. IV immunoglobulin.
C. Rituximab.
D. Thrombopoietin.
E. Thrombopoietin receptor agonist.
ANSWER: B

Z-plasty is a technique used best to


A. Excise a chest wall nevus.
B. Repair a cleft lip.
C. Lengthen a hand scar contracture.
D. Do mastectomy for a centrally located tumor.
E. Suture a transverse cut wound in the scalp.
ANSWER: C

One of the following is a well-known disadvantage of nitrous oxide


A. Expansion of closed air spaces.
B. Hepatic toxicity.
C. Slow elimination.
D. High cost.
E. Arrythmia.
ANSWER: A

In the surgical therapy of chronic anal fissure, we


A. Divide about 50% of the external anal sphincter.
B. Divide the whole internal anal sphincter.
C. Divide about 30% of internal sphincter.
D. Excise the anal fissure.
E. Excise part of the internal sphincter
ANSWER: C

Dr. wessam alzaidat - General surgery


1st year exam 6/2021 MOH
1- Which of the following does not pass through the inguinal canal?
A. Round ligament
B. Vas deferens
C. Genital branch of genitofemoral nerve
D. Broad ligament
E. Testicular artery
ANSWER: D

2- Four days after an Ivor Lewis esophagectomy, you start enteral feeding through the
jejunostomy feeding tube. The long chain fatty acids contained in the feeds:
A. Are not found in chylomicrons
B. Are only synthesized in the body
C. Enter the circulation via lymphatics
D. Enter the circulation via the portal system
E. Are not absorbed at all
ANSWER: C

3- A 65-year-old woman, presents with a tender mass below and medial to the pubic tubercle.
One Of the following is not a boundary to this mass:
A. Femoral artery
B. Femoral vein
C. Inguinal ligament
D. Pectineal ligament
E. Lacunar ligament
ANSWER: A

4- Within 24 hours of a surgical wound, the predominant cell type is:


A. Polymorphonuclear cells
B. Macrophages
C. Lymphocytes
D. Fibroblasts
E. The wound is acellular
ANSWER: A

5- A patient who previously sustained a large open gluteal laceration, comes to your clinic with a
smaller wound to your surprise. This is explained by one of the following:
A. Lymphocytes
B. Macrophages
C. Fibroblasts
D. Myofibroblasts
E. PMNs
ANSWER: D

6- With regard to thyroid anatomy, One following statements is incorrect:


A. The inferior thyroid artery arises directly from the external carotid artery.
B. The thyroid ima artery arises directly from the aorta in 1% to 4% of patients.
C. The ligament of Berry is located near the entry point of the RLN.
D. Venous drainage of the thyroid gland is via the superior, middle, and inferior branches.
E. The superior and middle thyroid veins drain into the jugular vein.
ANSWER: A

7- All the following are true about the liver except


Dr. wessam alzaidat - General surgery
A. It’s weight is about 2% of the body weight.
B. It is surrounded by a fibrous sheath called Glisson’s capsule.
C. The round ligament is the remnant of the obliterated umbilical artery.
D. The falciform ligament anchors the liver to the anterior abdominal wall.
E. The ligamentum venosum is the obliterated ductus venosus.
ANSWER: C

8- The gallbladder wall contains all the following layers except


A. Mucosa.
B. Submucosa.
C. Muscularis.
D. Subserosa.
E. Serosa.
ANSWER: B

9- All the following are true about the spleen except


A. It is mesodermal in origin.
B. Most accessory spleens present in the hilum of the spleen.
C. The gastrosplenic ligament contains the short gastric vessels.
D. In most people the splenic artery divides far from the spleen and many long vessels enter the
spleen over three fourth of its medial surface.
E. The white pulp constitutes about 75% of the spleen.
ANSWER: E

10- Which of the following body fluids contains the highest concentration of bicarbonate?
A. Stomach.
B. Small intestine.
C. Colon.
D. Pancreas.
E. Bile.
ANSWER: D

11- All the following are manifestations of hypercalcemia except


A. Abdominal pain.
B. Hypertension.
C. Polyurea.
D. Hyperreflexia.
E. Arrhythmia.
ANSWER: D

12- Which of the following has the highest effect in increasing nitrogen wasting?
A. Major burn.
B. Skeletal trauma.
C. Severe sepsis.
D. Elective surgery.
E. Starvation.
ANSWER: A

13- All the following are true about platelets except


A. They don’t have nucleus.
B. Up to 30% may be sequestered in the spleen.
C. They have an average life span of 3 weeks.
D. They play an integral role in the formation of the hemostatic plug.
E. Heparin does not interfere with platelets aggregation.
Dr. wessam alzaidat - General surgery
ANSWER: C

14- Febrile nonhemolytic transfusion reaction is caused by


A. Preformed cytokines or host antibodies to donor lymphocytes.
B. Infusion of contaminated blood.
C. Transfusion of large volume of blood.
D. Anti-HLA antibodies in the transfused blood.
E. Transfusion of ABO incompatible blood.
ANSWER: A

15- In which region of the GI tract is the longitudinal muscle of the muscularis arranged into
distinct longitudinal bundles?

A. Duodenum
B. Jejunum
C. Ileum
D. Colon
E. stomach

ANSWER: D

16- A 30-kg child has an estimated hourly fluid requirement of:

A. 40 ml/hour
B. 50 ml/hour
C. 60 ml/hour
D. 70 ml/hour
E. 80 ml/hour

ANSWER: D

17- Which of the following is contraindication for day-case inguinal hernia


surgery?

A. Age of 70 years old.

B. Well controlled epilepsy.

C. Well control Diabetes type 1.

D. American Society Anesthesiologist (ASA) score 3.

E. Body mass index of 35kg/m2.

ANSWER D

18- One of the following statements regarding cellulitis is not true?

A. This is non-suppurative invasive infection of tissues.

B. It is poorly localized.

C. Could be caused by Clostridium perfringens.

Dr. wessam alzaidat - General surgery


D. Caused by beta hemolytic streptococcus.

E. Systemic signs are not present.

ANSWER E

19- In response to reversible hypovolemic shock, all of the following indices decrease except

A. Cardiac output.

B. Systemic vascular resistance.

C. Blood volume in venous capacitance.

D. Central venous pressure.

E. Mixed venous oxygen saturation.

ANSWER: B

20- In an average adult, all the following are compatible with class II hemorrhage except

A. Blood loss of 1000cc.

B. Heart rate 130 per minute.

C. Postural hypotension.

D. Anxiety.

E. Decreased central venous pressure.

ANSWER: B

21- A24-year-old woman is scheduled for an elective cholecystectomy. The best method of identifying a
potential bleeding is which of the following?
A. Platelet count
B. A complete history and physical examination
C. Bleeding time
D. Lee-White clotting time
E. Prothrombin time (PT)
ANSWER: B

22- After undergoing subtotal gastrectomy for carcinoma of the stomach, a 64-year-old woman is receiving
peripheral parenteral nutrition. To increase calories by the peripheral route, what should be prescribed?
A. D5W in normal saline
B. Multivitamin infusion
C. D25W (25% dextrose in water)
D. Soybean oil (intra lipid)
E. Lactulose
ANSWER: D

23- What is the definition of overweight?


A. BMI > 25 kg/m2
B. BMI = 25 kg/m2
Dr. wessam alzaidat - General surgery
C. BMI 25-29.9 kg/m2
D. BMI 25-28 kg/m2
E. BMI > 30 kg/m2
ANSWER: C

24- In response to trauma, the serum levels of all of the following hormones increase except
A. T3.
B. Adrenalin.
C. Insulin.
D. Growth hormone.
E. Cortisol.
ANSWER: C

25- All of the following increase the amount of insensible water loss except.
A. Dry weather.
B. Fever.
C. Diarrhea.
D. Tracheotomy.
E. Hot weather.
ANSWER: C

26- Which of the following operations needs antibiotics as prophylaxis?


A. Herniotomy.
B. Appendicectomy for a suppurative appendix.
C. Drainage for a perianal abscess.
D. Surgery for a perforated peptic ulcer.
E. Thyroid surgery.
ANSWER: B

27- All of the following are causes of metabolic acidosis except


A. Pancreatic fistula.
B. Gastric outlet obstruction.
C. Salicylic acid overdose.
D. Methanol toxicity.
E. Sepsis.
ANSWER: B

28- Which of the following is an advantage of transjugular central venous line over a central venous line
inserted through a peripheral vein?
A. We can give higher concentration of glucose solutions.
B. We can keep it for a longer period.
C. Less risk of pneumothorax.
D. Less risk of arrythmia.
E. Less risk of air embolism.
ANSWER: B

29- In the management of a patient who has massive trauma and hypotension, all of the following should
be done except
A. Stop hemorrhage.
B. Correct coagulopathy.
C. Elevate systolic pressure above 110mmHg.
D. Control sepsis or major contamination.
E. Give blood.
ANSWER: C
Dr. wessam alzaidat - General surgery
30- Cancer cells have all of the following features except
A. Resist signals that inhibit growth.
B. Develop early apoptosis.
C. Acquire angiogenic competence.
D. Has genomic instability.
E. Acquire the ability to invade.
ANSWER: B

31- All of the following are steps done in processing a specimen for histological examination except
A. Fixation in formalin for one day.
B. Make representative tissue blocks.
C. Place blocks in paraffin wax.
D. Make thin slices on glass slides.
E. Stain slides with Indian ink.
ANSWER: E

32- All of the following are of the first step of pain stepladder booklet advised by WHO to control chronic
pain caused by malignancy, except
A. Tramadol.
B. Aspirin.
C. Paracetamol.
D. Nonsteroidal anti-inflammatory drugs.
E. Tricyclic antidepressants.
ANSWER: A

33- Measures to reduce the risk of venous thromboembolism include all of the following except
A. Prevent preoperative dehydration.
B. Start to apply graduated compressing lower limbs stockings immediately after surgery.
C. Apply intraoperative foot impulse device.
D. Use intraoperative pneumatic compressing device.
E. Start prophylactic low molecular weight heparin preoperatively.
ANSWER: B

34- All of the following combinations between an ulcer’s edge and pathology are true except
A. Sloping edge: healing ulcer.
B. Undermined edge: bed sore.
C. Rolled edge: basal cell carcinoma.
D. Everted edge: tuberculosis.
E. Punched-out edge: syphilis.
ANSWER: D

35- All of the following are proven examples of congenital sinuses except
A. Preauricular.
B. Umbilical.
C. Pilonidal.
D. Urachus.
E. Coccygeal.
ANSWER: C

36- Which of the following incidents is considered an adverse event?


A. Superficial thrombophlebitis in a peripheral vein caused by a cannula.
B. A needle found near a patient’s bed.
C. Sciatic nerve injury after an intramuscular injection.
Dr. wessam alzaidat - General surgery
D. Intraoperative bleeding of 100cc from the right colic artery during right hemicolectomy which was
stopped at the same operation.
E. Laparoscopic cholecystectomy converted to open due to sever inflammation.
ANSWER: C

37- All of the following should be checked before induction of anesthesia in a patient planed for
paraumbilical hernia repair except
A. Confirm the patient’s name.
B. Confirm the diagnosis.
C. Check the consent.
D. Mark the site of the procedure.
E. Ask for known allergies.
ANSWER: D

38- Which of the following forceps is unsuitable to grasp the bowel?


A. Nontoothed.
B. Allis.
C. Babcock.
D. Kocher.
E. Cecal forceps.
ANSWER: D

39- Which of the following organs creates the least immunological response in deceased donor organ
transplantation?
A. Liver.
B. Kidney.
C. Cornea.
D. Pancreas.
E. Small bowel.
ANSWER: C

40- All the following are true about trichilemmal cyst except
A. More common in the scalp.
B. Has a distinct odor if ruptured.
C. Has a thin wall.
D. The wall is a complete mature epidermis with granular layer.
E. If causes symptoms, the entire cyst is removed.
ANSWER: D

41- Amnesia is a main action of all the following drugs except


A. Midazolam.
B. Barbiturates.
C. Etomidate.
D. Ketamine.
E. Fentanyl.
ANSWER: E

42- A healing ulcer has all the following features except


A. Sloping edge.
B. Healthy granulation tissue filling the floor.
C. A lot of discharge.
D. Absence of dead tissue.
E. Good tissue vascularity.
ANSWER: C
Dr. wessam alzaidat - General surgery
43- All the following are indications for insertion of a Folly’s catheter except
A. A patient with sever pancreatitis.
B. A female trauma patient who has femur fracture.
C. A male trauma patient who has blunt liver injury.
D. A child admitted for acute appendicitis.
E. A male presented to the emergency room with urine retention.
ANSWER: D

44- A 34-year-old female has left thyroid nodule about 2 by 3 cm. Which is the best pre-operative
diagnostic modality?
A. Fine needle aspiration cytology.
B. True-cut biopsy.
C. Incisional biopsy.
D. TSH serum level.
E. Thyroid isotope scan.
ANSWER: A

45- A 25-year-old female underwent total thyroidectomy for Graves disease, 8 hours after surgery she
informed the nurses that she has numbness in her face and fingers. This complaint is most likely caused
by
A. Hypothyroidism.
B. Hypocalcemia.
C. Recurrent laryngeal nerve injury.
D. Neck hematoma.
E. Anesthesia induced neuropathy.
ANSWER: B

46- Which of the following is a cause of wide anion gape metabolic acidosis?
A. Nasogastric tube.
B. Diabetic ketoacidosis.
C. Pancreatic fistula.
D. Diarrhea.
E. Chronic renal failure.
ANSWER: B

47- Which of the following surgeries carries the highest risk for deep vein thrombosis?
A. Surgery for subdural hemorrhage.
B. Liver resection for tumor.
C. Hysterectomy.
D. Total hip replacement.
E. Nephrectomy.
ANSWER: D

48- A 20-year-old male presented with an asymptomatic, 4 by 4 cm soft mass in his abdominal wall, not
attached to skin. The most probable diagnosis is
A. Lipoma.
B. Sebaceous cyst.
C. Soft tissue sarcoma.
D. Accessory breast.
E. Intradermal nevus.
ANSWER: A

49- The surgeon did total gastrectomy for gastric cancer, before closing the abdomen he noticed
Dr. wessam alzaidat - General surgery
significant change in the color of the left lateral segment of the liver. This mostly indicates the presence
of
A. Replaced right hepatic artery.
B. Accessory right hepatic artery.
C. Replaced left hepatic artery.
D. Accessory left hepatic artery.
E. Replaced common hepatic artery.
ANSWER: C

50- All the following features are characteristic of dysplastic nevus except
A. Variegated pigmentation.
B. Ill-defined borders.
C. Irregular surface.
D. Size more than 5 mm in diameter.
E. Blue color.
ANSWER: E

Dr. wessam alzaidat - General surgery


2nd year exam 6/2021 MOH
1- Which of the following statements is true about the liver enzymes ALT and AST?
A. ALT is more specific for liver disease than AST.
B. Both participate in glycolysis.
C. They are usually severely elevated in cases of liver cirrhosis.
D. In alcoholic liver disease, an ALT:AST ratio more than 2 is common.
E. In fulminant hepatitis, ALT and AST levels are usually moderately elevated.
ANSWER: A

2- All the following are true about Gilbert’s syndrome except


A. It is characterized by decrease activity of glucuronyltransferase.
B. It causes increase in conjugated bilirubin level.
C. It is a benign condition.
D. Presents with jaundice during stress conditions or fasting.
E. No treatment is required for this syndrome.
ANSWER: B

3- All the following about the vascular system of the gallbladder are true except
A. The cystic artery is usually a branch of the right hepatic artery.
B. When the cystic artery reaches the neck of the gallbladder it divides into anterior and
posterior branches.
C. Usually, a single cystic vein drains the gallbladder to the portal vein.
D. Frequently, a visible lymph node overlies the cystic artery near its end.
E. The nerves of the gallbladder come through the celiac plexus.
ANSWER: C

4- Which of the following is true regarding gallstones?


A. Most persons with gallstones have symptoms related to them.
B. Most gallstones are pure cholesterol stones.
C. Pure cholesterol stones are usually large and single.
D. Most gallbladder stones are radiopaque.
E. Black stones are usually formed secondary to bacterial infection caused by bile stasis.
ANSWER: C

5- Which of the following statements is true?


A. The inferior mesenteric vein often joins the splenic vein near its junction with the superior
mesenteric-portal vein.
B. The superior mesenteric artery runs to the right and parallel to the superior mesenteric
vein.
C. The pancreas is wrapped by peritoneum.
D. All the arteries that supply the pancreas are branches of the celiac trunk.
E. The body of the pancreas constitutes about 65% of the size of the pancreas.
ANSWER: A

6- Which of the following statements is true?


A. 1 milliequivalent calcium ion equals 2 millimoles.
B. Normal serum osmolality is between 250 and 270 milliosmoles.
C. The intracellular fluid ionic concentration is more than that in the interstitial fluid.
D. Sodium is the main intracellular cation.
E. Proteins in the plasma is more than proteins intracellular.
ANSWER: C

Dr. wessam alzaidat - General surgery


7- The most important factor in the formation of uric acid calculi is:
A. Concentration of uric acid in the urine
B. Volume of urine
C. Limited solubility of uric acid in acidic solutions
D. Excessive dietary intake of purines
E. Presence of symptomatic gout
Answer: C

8- The action of parathyroid hormone (PTH) on the kidney IS to:


A. Increase calcium and phosphorus absorption
B. Decrease calcium and phosphorus absorption
C. Increase calcium and decrease phosphorus absorption
D. Increase phosphorus and decrease calcium absorption
E. Promote production of active Vitamin D
Answer: E

9- The evaluation of a comatose patient with a head injury begins with:


A. The cardiovascular system.
B. Pupillary reflexes.
C. Establishment of an airway.
D. Computed tomography (CT) of the brain.
E. History.
Answer: C

10- Which of the following cells produces HCL?


A. Beta cells
B. Oxyntic cells
C. Chief cells
D. Alpha cells
E. Surface mucus cells
Answer: B

11- Secretion of Gastric juice is stimulated by


A. Gastrin
B. Cholecystokinin
C. Enterogastrin
D. Pepsin
E. lipase
Answer: A

12- Posterior superior iliac spine lies at:

A. L5 level
B. S1 level
C. S2 level
D. S3 level
E. L4 level
Answer: C

13- Which is the most common cause of peptic ulcer disease (PUD) of the following?

Dr. wessam alzaidat - General surgery


A. Smoking
B. Non steroidal anti inflammatory drugs
C. Zollinger-Ellison syndrome
D. Ethanol excess
E. Family history of PUD
Answer: B

14- With regard to bipolar and monopolar devices -


A. Both use direct current (DC).
B. Both use alternating current (AC).
C. Bipolar uses AC, while monopolar uses DC.
D. Bipolar uses DC, while monopolar uses AC.
E. Neither requires an electrosurgical unit (ESU).
Coagulation

ANSWER: B

15- Which of the following has been shown to reduce


the incidence of acquired central venous line sepsis?

A. Administration of intravenous (IV) vancomycin prophylaxis at the time of line insertion

B. Utilizing the femoral vein site of insertion

C. Using a multiple lumen catheter

D. Daily review and removal of the catheter at the earliest opportunity

E. Changing the lines every seven days


ANSWER: D

16- All the following are clinical manifestations of hyperkalemia except


A. Abdominal pain.
B. Paralysis.
C. Weakness.
D. Constipation.
E. Arrythmia.
ANSWER: D

17- A 20 years old male patient, presented complaining of abdominal pain and vomiting for 24
hours due to intestinal obstruction, he is thirsty with dry mouth, mucous membranes and axillae, he
has mild tachycardia but normal supine blood pressure, his weight is 80kg. Which of the following
parenteral fluid therapy formulas may be more suitable for him in the first 24 hours?
A. 8 L glucose saline 0.45%.
B. 1 L normal saline and 2 L glucose water 5%.
C. 2 L normal saline and 1 L glucose water 5%.
D. 4 L glucose saline 0.18%.
E. 4 L normal saline.
ANSWER: A

18- All the following are true about liver anatomy except

Dr. wessam alzaidat - General surgery


A. Cantlie’s line extends roughly between the gallbladder fossa and the inferior vena cava.
B. The right lobe accounts around two thirds of the liver size.
C. The Caudate lobe lies posterior to the level of the porta hepatis.
D. The anterior branch of the right portal vein usually supplies liver segments V and VI.
E. The right hepatic vein drains venous tributaries from segments V, VI, VII and VIII.
ANSWER: D

19- All the following are true regarding the pancreatic ducts except
A. The duct of Santorini embryologically comes from the dorsal pancreatic bud.
B. The ventral bud forms most of the pancreas.
C. In most people most of the pancreas is drained via the major ampulla.
D. In most people the two pancreatic ducts are communicated.
E. The ampulla of Vator located between the proximal two thirds and the distal one third of the
second part of duodenum.
Answer: B

20- Which of the following does not pass through the diaphragmatic openings?
A. Aorta
B. Inferior Vena Cava
C. Esophagus
D. Left gastric artery
E. Left phrenic nerve
ANSWER: D

21- In a total thyroidectomy for advanced papillary cancer, you notice the tumor on the right is
adherent to a nerve in the tracheo-esophageal groove. This nerve injury or resection will result in:
A. Loss of high-pitched voice
B. Stridor with need for tracheostomy
C. Unilateral right vocal cord paralysis
D. Loss of sensation above the vocal cords.
E. Loss of sensation in the skin of the right muscular triangle
ANSWER: C

22- In maturation phase of wound healing, one of the following is true:


A. Fibroblasts continue to produce collagen and interstitial matrix
B. Angiogenesis continues to provide nutrients to the healing wound
C. Skin loses its elasticity as Elastin never forms
D. Cells gradually undergo apoptosis, and a cellular collagen continues to remodel
E. Typically starts at 6 weeks after the injury
ANSWER: D

23- Which of the sutures listed below would be most suitable for suturing the femoral artery
following an embolectomy?
A. 5/0 polyglactin
B. 5/0 polypropylene
C. 5/0 silk
D. 5/0 polyglycolic acid
E. 5/0 polyester
ANSWER: B

24- A 60-year-old woman who underwent a mastectomy for breast cancer 2 years earlier

Dr. wessam alzaidat - General surgery


presents to the emergency department with headache, backache, and frequent vomiting. She is
extremely thirsty and stuporous. Which test is most likely to identify the cause?
A. CT scan of the head
B. X-ray of spine
C. Serum sodium determination
D. Serum calcium determination
E. Serum glucose determination
ANSWER: D

25- A 43-year-old woman with von Willebrand’s disease is scheduled for cholecystectomy. It
can be stated that preoperative evaluation will reveal which of the following?
A. Normal bleeding time, PT, and PTT
B. Platelet aggregate with restocetin
C. Increased bleeding time and PTT, and normal PT
D. Increased bleeding time and PT, and normal PTT
E. Increased bleeding time, and normal PT and PTT
ANSWER: C

26- A 64-year-old woman undergoing radical gastrectomy under general anesthesia is


transfused with 2 U of packed RBCs. A hemolytic transfusion reaction during anesthesia will be
characterized by which of the following?
A. Shaking chills and muscle spasms
B. Fever and oliguria
C. Hyperpyrexia and hypotension
D. Tachycardia and cyanosis
E. Bleeding and hypotension
ANSWER: E

27- In nutritional therapy, the commonest cause of complications is


A. Overfeeding.
B. Protein insufficiency.
C. Dehydration.
D. Hyponatremia.
E. Vitamins deficiency.
ANSWER: A

28- All of the following are true regarding human body fluids except
A. Water constitutes about 60% of the weight of an average adult.
B. Most of body water presents intracellular.
C. An average adult, about 75 kg weight, needs about 1400cc of water daily.
D. Urine constitutes the major source of water loss.
E. Interstitial water is much more than intravascular water.
ANSWER: C

29- In response to trauma, the serum levels of all of the following hormones increase except
A. T3.
B. Adrenalin.
C. Insulin.
D. Growth hormone.
E. Cortisol.
ANSWER: C

Dr. wessam alzaidat - General surgery


30- All the following are true regarding respiratory alkalosis except
A. Caused by increase respiratory CO2 washing.
B. Kidneys compensate rapidly for the condition.
C. Compensation never overcorrect the pH.
D. Rebreathing mask can be used in treatment.
E. May be associated by carpopedal spasm.
ANSWER: B

31- In comparison between healing by primary and secondary intentions for a wound involving
skin and subcutaneous fat, healing by primary intension has
A. More inflammatory response.
B. Less tensile strength.
C. Less chance to develop abscess.
D. Less scar.
E. More granulation tissue formation.
ANSWER: D

32- All of the following are true about polypropylene sutures except
A. It is monofilament.
B. It preserves its tensile strength for many years.
C. It is non absorbable.
D. It develops high tissue reaction.
E. It has high memory.
ANSWER: D

33- All of the following increase the risk of wound infection except
A. Obesity.
B. Jaundice.
C. Uremia.
D. Chemotherapy.
E. Pathing at the first postoperative day.
ANSWER: E

34- All the following are causes of clubbing except


A. Lung cancer.
B. Lung abscess.
C. Empyema.
D. Gastric cancer.
E. Cirrhosis.
ANSWER: D

35- All the following are true about venous ulcers except
A. Associated with varicose veins.
B. Usually on the lateral aspect of the leg.
C. Usually shallow.
D. The affected limb is usually warm.
E. More common in women.
ANSWER: B

36- All the following are true about laryngeal mask airway except

Dr. wessam alzaidat - General surgery


A. It sits in the oropharynx.
B. It is passed to its site under vision.
C. It has a cuff which is inflated to push the soft tissue away from the laryngeal inlet.
D. It does not pass through the vocal cords.
E. It does not fully protect against aspiration.
ANSWER: B

37- Hypercalcemia presents with all of the following conditions except


A. Abdominal pain.
B. Renal stones.
C. Psychological symptoms.
D. Carpopedal spasm.
E. Constipation.
ANSWER: D

38- In clinical practice, cryoprecipitate is commonly used to supply the patient


with coagulation factor
A. V
B. VII
C. VIII
D. IX
E. X
ANSWER: C

39- All of the following develop in hemorrhagic shock except


A. Inadequate tissue perfusion.
B. Anaerobic metabolism.
C. Lactic acidosis.
D. Hypokalemia.
E. Cell lysis.
ANSWER: D

40- A 22 years old male patient presented with an infected surgical wound. His
temperature is 38.2 degrees C, pulse 100 per minute, blood pressure 120/75,
respiratory rate 16/minute, WBCs 10.5 by 109 /L. His condition is best termed
A. Systemic inflammatory distress syndrome without sepsis.
B. Sepsis.
C. Sever sepsis.
D. Stage 1 septic shock.
E. Multiorgan dysfunction.
ANSWER: B

Dr. wessam alzaidat - General surgery


41- Which of the following organisms considered as a cause of opportunistic
infection?
A. Staphylococcus aureous
B. Streptococcus pneumonia
C. Pseudomonas aurogenosa
D. E. coli
E. Klebsiella pneumonia

ANSWER: C

42- A 30 years old male who was involved in a road traffic accident and
underwent laparotomy and splenectomy. In his second postoperative day he
developed fever. This fever is most likely due to
A. Atelectasis.
B. Wound infection.
C. Post-splenectomy sepsis.
D. Urinary tract infection.
E. Deep vein thrombosis.
ANSWER: A

43- Which of the following fistulae has a higher chance to heal spontaneously?
A. Low fistula in ano.
B. Postoperative colo-cutaneous fistula with daily output of 70cc at the 14th
day of diagnosis.
C. Biliary fistula with distal obstruction.
D. Ileal fistula caused by Crohn’s disease.
E. A fistula caused by a perforation in a colonic tumor which is not operated.
ANSWER: B

44- The best test to diagnose or exclude cancer in a 43 years old lady
presented with breast mass is
A. Mammography
B. Breast ultrasound
C. Chest CT scan
D. Fine needle aspiration cytology
E. True-cut biopsy

ANSWER: E

45- All the following arteries are parts of the blood supply of the thyroid gland

Dr. wessam alzaidat - General surgery


except
A. Thyroid ima
B. Superior thyroid arteries
C. Inferior thyroid arteries
D. Middle thyroid arteries
E. Branches of the thyro-cervical trunks

ANSWER: D

46- Which of the following is an example of surgical audit?


A. Reviewing the outcomes of thyroid surgery and comparing them with
standard outcomes in the region.
B. Reviewing the pathological results of thyroidectomy in a medical center
and identifying the commonest indication.
C. Prospective comparison between two modalities of treatment for a
specific surgical disease.
D. Comparing the results of a surgical procedure against sham surgery in
animals.
E. Retrospective reviewing of patients’ records with specific disease.
ANSWER: A

47- All the following are symptoms and signs of chronic unilateral iliac
artery obstruction except
A. Intermittent calf claudication.
B. Intermittent thigh claudication.
C. Absent femoral pulses.
D. Impotence.
E. Bruit over the iliac region.
ANSWER: D

48- All the following are better in laparoscopic surgery than open
laparotomy except
A. Less post-operative pain.
B. Better control of bleeding.
C. Better viewing of important structures in cholecystectomy.
D. Better exposure of subcostal regions.
E. Earlier return to work.
ANSWER: B

49- The circular end-to-end anastomosis (CEEA) stapler device is most

Dr. wessam alzaidat - General surgery


valuable in
A. Anastomosing the small bowel together.
B. Anastomosing the lower esophagus to a R-en-Y jejunal loop.
C. Anastomosing the small bowel to the transverse colon.
D. Anastomosing the right and left colon.
E. Closing a tracheoesophageal fistula.
ANSWER: B

50- All the following suggest the possibility of inhalational injury in burn
patients except
A. Swollen lips.
B. Change of voice.
C. Stridor.
D. Flam burn.
E. Burn happened in a closed space.
ANSWER: D

Dr. wessam alzaidat - General surgery


3rd year exam 6/2021 MOH
1- Following a fight, a 21-year-old male has an inability to dorsiflex his right foot (foot drop)
while losing sensation in the dorsum of his foot. The nerve affected is:
A. Medial plantar nerve
B. Lateral plantar nerve
C. Tibial nerve
D. Lateral cutaneous nerve of the calf
E. Common peroneal nerve
ANSWER: E

2- The organ most commonly found in a sliding hernia in women is:


A. Ovary and fallopian tube
B. Cecum
C. Small bowel
D. Spleen
E. Ureter
ANSWER: A

3- The vertebral level that corresponds to the nipple in men:


A. T1
B. T2
C. T3
D. T4
E. T5
ANSWER: D

4- A patient presents with a keloid scar to your clinic. All describe her condition except:
A. It is usually confined to the original scar
B. Can be treated with steroids
C. Can be treated with silicone injections
D. Can be treated with radiation (XRT)
E. Tend to form in upper extremities and sternum more commonly
ANSWER: A

5- The most important prognostic factor for breast cancer devoid of metastases is:
A. Node status
B. Size of the tumor
C. Tumor grade
D. Gender of the patient
E. Age of the patient
ANSWER: A

6- The predominant cell in the proliferative phase of wound healing is:


A. Polymorphonuclear cells
B. This phase is acellular
C. Platelets
D. Lymphocytes
E. Fibroblasts
ANSWER: E

Dr. wessam alzaidat - General surgery


7- A patient who is on chronic steroids, is about to undergo surgery. To ensure best possible
wound healing result, you:
A. Add high dose vitamin A before and after the operation
B. Add high dose vitamin D before and after the operation
C. Add high dose vitamin E before and after the operation
D. Add high dose vitamin K before and after the operation
E. Patient can be safely stopped at the time of surgery, regardless of indication
ANSWER: A

8- Hyperacute rejection following organ transplantation is most often due to:


A. Rh incompatibility
B. Macrophages
C. Previous sensitized T cells
D. ABO incompatibility
E. Preformed antibodies
ANSWER: D

9- 47-year-old woman presents to the emergency room after a motor vehicle accident, mildly
lethargic and confused, with normal airway and vitals. She complains of a headache, and
has a frontal scalp hematoma. After stabilizing her cervical spine and doing primary survey,
she becomes more lethargic, moving only to pain. Which of the following is the most likely
to be present?
A. Subdural hematoma
B. Extradural hematoma
C. Cerebral contusion
D. Intracerebral hematoma
E. Diffuse axonal injury
ANSWER: B

10- A 19-year-old girl sustained a posterior knee dislocation. Her distal pulses are absent. The
most appropriate next step is:
A. Start thrombolytic therapy immediately
B. Start Heparin immediately
C. Exploration of the vessels
D. Angiogram
E. Immediate reduction of the knee
ANSWER: E

11- Following a house fire, a 25-year-old male patient was brought to the emergency room
with extensive partial and full thickness burns. You note black carbon deposits around his
nostrils and oropharynx. Which of the following is an immediate priority?
A. Adequate analgesia
B. Aggressive fluid resuscitation
C. Immediate transfer to a burn unit
D. Secure the airway with higher possible need for intubation
E. Dressing changes to the burn wounds
ANSWER: D

12- There were 192,000 new cases diagnosed with lung cancer in 2019. This means:
A. Incidence

Dr. wessam alzaidat - General surgery


B. Prevalence
C. Mean
D. Mode
E. Median
ANSWER: A

13- All of the following statements are true except:


A. Lymph node metastasis is commoner in follicular cancer than in papillary cancer.
B. Papillary cancer is associated with BRAF mutation.
C. Distant metastasis is commoner in the follicular variety.
D. Hurthle cell carcinoma is a type of follicular cancer.
E. Papillary carcinoma is usually nonencapsulated and slow-growing.
ANSWER: A

14- All the following are true about hepatic venous drainage except
A. The right hepatic vein drains liver segments V, VI, VII and VIII.
B. The middle hepatic vein drains liver segments IV, V and VIII.
C. The left hepatic vein drains liver segment I.
D. The caudate lobe drains directly into the IVC.
E. The left and middle hepatic veins usually form a common trunk before entering the IVC.
ANSWER: C

15- All the following are true about bile acids and bile salts except
A. Bile salts are responsible for the digestion and absorption of lipids.
B. Bile salts are sodium and potassium salts of bile acids conjugated with amino acids.
C. The bile acids are derivatives of cholesterol.
D. Chenodeoxycholic acid is a secondary bile acid.
E. 90% to 95% of bile salts secreted into the intestine are reabsorbed at the terminal ilium.
ANSWER: D

16- All the following are true about liver hemangiomas except
A. They are the most common solid liver lesions.
B. More common in women.
C. Most are discovered incidentally.
D. Spontaneous rupture is common.
E. Resection can be accomplished mostly by enucleation
ANSWER: D

17- Which of the following is true about the pancreatic juice?


A. It is about 200 to 300cc per day in an average adult.
B. It is white.
C. It is alkaline.
D. It is hyperosmotic to the plasma.
E. It contains the active forms of proteases and lipases
ANSWER: C

18- Which of the following signs does Horner's syndrome include?

A. Ptosis.
B. Facial hyperhidrosis.

Dr. wessam alzaidat - General surgery


C. Diplopia
D. Exophthalmos.
E. Mydriasis.
Answer: A

19- Laparoscopic surgery may induce the following hemodynamic changes except:

A. Increased preload.
B. Increased cardiac output.
C. Increased pulmonary vascular resistance.
D. Increased systemic vascular resistance.
E. Decreased renal perfusion pressure.
Answer: B

20- Which of the following is not a limitation of minimal access surgery?


A. Technically more demanding
B. Loss of tactile feedback
C. Extraction of large specimens
D. Poor vision
E. Difficulty with haemostasis.
Answer: D

21- Hormones or peptides involved in satiety include:

A. Gastrin
B. Somatostatin
C. Glucagon
D. Ghrelin
E. Estrogen
Answer: D

22- Following insertion of a Veress needle, what is the initial maneuver to confirm
intraperitoneal placement?

A. Saline drop test


B. Aspiration of the needle
C. Flushing the needle
D. Measuring insufflation pressure
E. Starting high-flow insufflation
ANSWER: B

23- Which statement is FALSE regarding sigmoid volvulus?


A. Sigmoid volvulus is commonly seen in patients who are elderly, debilitated, chronically
constipated, or on psychotropic medications.
B. Mortality rates are lower when emergent decompression is performed before sigmoid
resection.
C. Nonoperative reduction of sigmoid volvulus is successful in 70%–80% of cases.
D. If the gangrenous bowel is seen during decompression, a rectal tube should be placed to
prevent recurrence.

Dr. wessam alzaidat - General surgery


E. Sigmoid resection within 2 to 3 days of decompression is the preferred definitive treatment.

ANSWER: D

24- Which of the following is false regarding colonic polyps?


A. Tubulovillous adenomas have a lower malignancy risk than villous adenomas.
B. Some hyperplastic polyps are premalignant.
C. The polyps in Peutz-Jeghers syndrome are hyperplastic.
D. hamartomatous polyps are not considered premalignant
E. In an adenomatous polyp, the risk of malignancy is related to its location in the GI
tract.
ANSWER E

25- Which of the following is true regarding diverticular diseases of the


lower GI tract?
A. They occur most commonly in the ascending colon.
B. The rectum can be affected.
C. Asymptomatic sigmoid or cecal diverticuli require surgical management
D. The vast majority of complications from diverticula occur at the sigmoid colon.
E. They are associated with a long, redundant colon.
ANSWER: D

26- A 35 years old male patient developed large amount of fresh bleeding per rectum 10
days post hemorrhoidectomy. The most probable cause for his bleeding is
A. Sloughing of a vessel wall.
B. Uncontrolled bleeding missed at the time of surgery.
C. Clot dislodgment.
D. Injury by defecation.
E. Vasodilation.
Answer: A

27- All the following are true about cryoprecipitate except


A. It is a supernatant precipitate of fresh frozen plasma.
B. It is rich in coagulation factor VII.
C. It is rich in fibrinogen.
D. Stored at -30 degrees centigrade.
E. Shelf life is 2 years.
Answer: B

28- All the following are true regarding platelets concentrate except

Dr. wessam alzaidat - General surgery


A- A unit is pooled usually from more than one donor.
B- It is kept on a special agitator.
C- It is stored at 4 degrees centigrade.
D- May transfer bacterial infections.
E- Shelf life is 5 days.
Answer: C

29- All of the following are recognized consequences of inadequate pain relief after
upper abdominal surgery EXCEPT:
A. Decreased functional residual capacity
B. Ileus
C. Deep vein thrombosis
D. Sodium and water retention
E. Reduced systemic vascular resistance
ANSWER: E

30- A 22 year old man in undergoing a rapid sequence induction prior to an


appendectomy. The anesthetist becomes concerned because the patient becomes
tachycardia and develops atrial fibrillation. His temperature is 39.5 oC, blood pressure
is slightly elevated. What is the most appropriate course of action?

A. Proceed to immediate laparotomy


B. Administration of broad spectrum intravenous antibiotics
C. Start infusion of inotropes
D. Administration of intravenous Dantrolene
E. Administration of Neostigmine
ANSWER: D

31- Bariatric surgery is used for obesity treatment but how much is the incidence of type 2
diabetes reduced in obese individuals after surgery?
A. 80-90%
B. No reduction in the incidence
C. Approximately 50 %
D. Approximately 40 %
E. Approximately 30 %
Answer: A

32- About laparoscopic sleeve gastrectomy , all are true except

A. A higher leak rate than laparoscopic RYGB


B. A lower mortality rate for super obese BMI > 60 kg/m2 undergoing duodenal
switch
C. A theoretical advantage over RYGB for iron absorption
D. Desire to eat increases
E. Reduction in hunger sensation
Answer: D

33- Unilateral bloody nipple discharge from one duct orifice is most commonly caused

Dr. wessam alzaidat - General surgery


by:

A. Paget's disease of the nipple.


B. Carcinoma in situ.
C. Inflammatory carcinoma.
D. Subareolar mastitis.
E. Intraductal papilloma.
Answer: E

34- Most common site of breast metastasis is:

A. Lung.
B. Brain.
C. Bone.
D. Adrenal.
E. Liver.
Answer: C

35- One of the following statements about cardiac myxoma is true:

A. This lesion is the most common primary cardiac tumor.


B. It is best diagnosed by cardiac catheterization and angiography.
C. The left ventricle is the most common site.
D. It has usually an extra cardiac growth pattern.
E. It has a multicentric origin in the chamber wall.
Answer: A

36- When is it mandatory to involve an independent mental capacity advocate?


A. When a patient requires a significant intervention, time permits and the patient
lacks capacity to decide for themselves
B. When a patient cannot decide on the best treatment
C. When parents disagree about treatment of a child
D. When two doctors disagree about the best treatment option
E. When a patient has lost trust in their clinician
ANSWER: A

37- A 40-year-old man who weighs 65 kg is being observed in the ICU. Twenty-four
hours postoperatively, he develops convulsions. His serum sodium is 118 mEq/L.
Appropriate management includes which of the following?
A. Administration of normal saline (0.9%)
B. Administration of hypertonic saline (3%)
C. Emergency hemodialysis
D. Administration of vasopressin
E. Administration of Lasix, 40 mg intravenously (IV)
ANSWER: B

Dr. wessam alzaidat - General surgery


38- A 42-year-old man with small-bowel fistula has been receiving TPN
with standard hypertonic glucose-amino acid solution for 3 weeks.
The patient is noticed to have scaly, hyperpigmented lesions over the
acral surfaces of elbows and knees, similar to enterohepatic
acrodermatitis. What is the most likely cause of the condition?

A. Copper deficiency
B. Essential fatty acid deficiency
C. Excess glucose calories
D. Hypomagnesemia
E. Zinc deficiency

ANSWER: E

39- Regarding successful day case surgery, one of the following is not
true;
A. minimal access techniques.
B. Good hemostasis.
C. Avoidance of unnecessary tissue handling or tension.
D. Provided with adequate analgesia.
E. Major procedures should be scheduled late on list.
ANSWER E

40- Postoperative suppressive treatment with thyroxin has a role in the


management of:
A. Follicular adenoma.
B. Papillary carcinoma.
C. Anaplastic carcinoma.
D. Medullary carcinoma.
E. Lymphoma.

Answer: B

41- Bone hunger syndrome can occur after total thyroidectomy for:
A. Papillary thyroid cancer.
B. Multinodular goiter.
C. Medullary thyroid cancer.
D. Graves` disease.
E. Follicular thyroid cancer.

Answer: D

42- The most common site of acute biliary pain (biliary colic) is
A. Subxiphoid.
B. Left hypochondrial.
C. Right flank.
D. Right scapular region.
E. Interscapular.

Dr. wessam alzaidat - General surgery


Answer: A

43- All the following are true about amylase in acute pancreatitis except
A. It starts to increase shortly after the onset of symptoms.
B. It peaks within several hours.
C. It remains elevated for 3 to 5 days in average.
D. Its serum levels in general correlate directly with the severity of disease.
E. The laboratory reading of it is commonly normal in acute pancreatitis caused by
hyperlipidemia.

Answer: D

44- All of the following are true regarding familial juvenile polyposis
EXCEPT ?
A. It is autosomal dominant.
B. The polyps are hamartomatous.
C. Bleeding is common symptom.
D. Once a polyp is detected, total proctocolectomy is recommended.
E. Intussusception is possible presentation.
ANSWER D

45- A 55-year-old male, who is noted to have diarrhea, flushing and


bronchoconstriction as well as right-sided cardiac valvular disease, is most likely to
have his primary carcinoid tumor in
A. Appendix
B. Bronchus
C. Rectum
D. Stomach
E. Ileum.
ANSWER E

46- Which of the following is a more common to cause massive colonic bleeding?
A. Cancer.
B. Ulcerative colitis.
C. Angiodysplasia.
D. Diverticulitis.
E. Granulomatous colitis.
ANSWER C

47- A 6 weeks old baby brought by his mother with umbilical discharge since birth. On
examination, it was not smelly and there was what looks like red spot inside the stump.
In this case the commonest cause will be:
A. Foreign body in the stump

Dr. wessam alzaidat - General surgery


B. Patent vietello intestinal duct
C. Patent urachus
D. Umbilical granuloma
E. Umbilical sepsis

ANSWER: D

48- A 2 months old baby presented with an umbilical hernia. The treatment is:
F. Observe
G. Herniotomy
H. Apply a truss
I. Inject sclerosing material
J. Truss for one year then operate
ANSWER: A

49- The following best describes the sequence of sensory


recovery in a healing skin graft:
K. Temperature, pain, light touch.
L. Pain, temperature, light touch.
M. Pain, light touch, temperature.
N. Light touch, pain, temperature.
O. Temperature, light touch, pain.
ANSWER C

50- One of the following statements about pulmonary hamartomas is true:


A. Hamartomas are benign chondromas.
B. Most are located in the conducting airways.
C. Wedge resection is curative.
D. A lobectomy is necessary to obtain draining hilar lymph nodes.
E. Hemoptysis is common.

Answer: C

Dr. wessam alzaidat - General surgery


Dr. wessam alzaidat - General surgery
4th year exam 6/2021 MOH
1- The peritoneal space located to the left and posterior to Foramen of Winslow is:
A. Rutherford Morison’s pouch (hepatorenal pouch)
B. Right subphrenic space
C. Right paracolic gutter
D. Lesser sac
E. Left spleno-renal space
ANSWER: D

2- Performing a PET scan can be difficult in one of the following situations:


A. Hypothyroidism
B. Hypertension
C. Hb A1c < 6%
D. Hb A1c > 8%
E. Hyperthyroidism
ANSWER: D

3- A 60-year-old man with unresectable lung cancer is undergoing radiation therapy


(XRT). The main target of XRT for malignancy treatment is:
A. DNA
B. RNA
C. Cell wall
D. Proteins
E. Cytoplasm
ANSWER: A

4- A 41-year-old large-surface-burn patient, is intubated in the ICU. After 10 hours of


aggressive hydration, escharotomy and fasciotomy, her urine output is becoming
scanty. After placing a central line, her CVP is 18. The most likely cause of her renal
failure is:
A. Inadequate fluid resuscitation
B. Cardiogenic shock
C. Sepsis
D. Pulmonary embolus
E. Myoglobinuria
ANSWER: E

5- One of the following injuries is associated with permanent damage to recurrent


laryngeal nerve during thyroid surgery:
A. Neuropraxia.
B. Axonotemesis.
C. Neurotemeisis.
D. Dendridotemisis.
E. Thermal injury
ANSWER: C

Dr. wessam alzaidat - General surgery


6- For a patient with a thyroid nodule, the following ultrasonic feature has the lowest
potential for malignancy:
A. Hypo-echogenicity.
B. Micro calcifications.
C. Spongiform appearance.
D. 40% of its size is a cyst.
E. 2 cm in diameter.
ANSWER: C

7- All of the followings are true regarding neuroendocrine tumors of the pancreas
except:
A. Insulinomas are the commonest functioning form.
B. Diarrhea is a common presentation in gastrinomas.
C. Peptic ulcers in gastrinomas are usually small and solitary.
D. VIPomas present with diarrhea.
E. They occur in 40–60 per cent of MEN-1 patients.
ANSWER: C

8- In breast carcinoma, one of the following statements is false:


A. Ductal carcinoma is the most common variant.
B. Lobular carcinoma occurs in 15 per cent.
C. There may be a combination of lobular and ductal features.
D. Colloid, medullary and tubular carcinomas carry a poor prognosis.
E. Paget’s disease is a superficial manifestation of an underlying breast carcinoma.
ANSWER: D

9- All the following are true regarding magnetic resonance imaging (MRI) for evaluation
of breast abnormalities except
A. It is useful for finding the primary breast lesion in patients with positive axillary
nodes and negative breast examination and mammogram.
B. It is more accurate than mammography in diagnosing invasive lobular cancer.
C. It is more helpful than mammography in assessing tumor extent particularly in older
women.
D. Its sensitivity in detecting invasive cancer is greater than 90%.
E. Its use as a screening tool is not settled.
ANSWER: C

10- Which of the following is a cause of presinusoidal portal hypertension?


A. Chronic viral hepatitis.
B. Alcohol abuse.
C. Autoimmune hepatitis.
D. Budd-Chiari syndrome
E. Schistosomiasis.
ANSWER: E

11- All the following are true about gallbladder cancer except

Dr. wessam alzaidat - General surgery


A. It is more common in females.
B. It carries bad prognosis.
C. Most are associated with gallbladder stones.
D. Most are squamous cell carcinomas.
E. Porcelain gallbladder carries relatively high incidence of the disease.
ANSWER: D

12- All the following are indications of splenectomy except


A. Hereditary spherocytosis who require frequent transfusion.
B. Pyruvate kinase deficiency with sever splenomegaly.
C. Glucose-6-phosphate dehydrogenase deficiency.
D. Sickle cell anemia with recurrent sequestration crises.
E. Idiopathic thrombocytopenia purpura refractory to steroid therapy
ANSWER: C

13- The most common form of testicular tumor in infants and children is:
A. Yolk sac tumor
B. Choriocarcinoma
C. Teratocarcinoma
D. Embryonal cell carcinoma
E. Seminoma
Answer: A

14- All the following are complications associated with pneumoperitoneum except
A. Hyperthermia
B. Acidosis
C. Cardiac arrhythmias
D. Gas embolism
E. Reduced cardiac return
Answer: A

15- During laparoscopic surgery, pneumoperitoneum usually results in a fall in cardiac


output when intra-abdominal pressure exceeds
A. 10 mmHg
B. 20 mmHg
C. 30 mmHg
D. 40 mmHg
E. 50 mmHg
Answer: B

16- Which of the following therapies is proven to reduce mortality and morbidity in
bleeding peptic ulcers?
A. Endoscopic procedures
B. H2 antagonists
C. Proton pump inhibitors
D. Octreotide
E. Antacids

Dr. wessam alzaidat - General surgery


Answer: A

17- Which of the following statements about gastric polyps is true?


A. Like their colonic counterparts, gastric epithelial polyps are common tumors.
B. They are analogous to colorectal polyps in natural history.
C. Endoscopy can uniformly predict the histology of a polyp based on location and
appearance.
D. In a given patient, multiple polyps are generally of a multiple histologic type.
E. Gastric adenomatous polyps greater than 2 cm. in diameter should be excised
because of the risk of malignant transformation.

Answer: E

18- What is the definition of overweight?


A. BMI > 25 kg/m2
B. BMI = 25 kg/m2
C. BMI 25-29.9 kg/m2
D. BMI 25-31 kg/m2
E. BMI > 30 kg/m2
Answer: C

19- A 3-year-old girl presents to the ED with crampy abdominal pain for 24 hours. The
pain is increasing in frequency and duration. Ultrasound demonstrates a target sign. The
most common pathologic lead point is:
A. Appendix
B. Small bowel polyp
C. Ectopic pancreatic tissue
D. Meckel diverticulum
E. Intraluminal hematoma
ANSWER: D

20- Electromagnetic interference is a concern for patients with pacemakers when using
energy devices. Of the following, which device would not generate any electromagnetic
interference?
A. Monopolar electrosurgical device.
B. Monopolar endoscopic device
C. Radiofrequency ablation (RFA)
D. Ultrasonic device
E. Bipolar electrosurgical device
ANSWER: D

21- A 32-year-old female patient is about to undergo an elective repair of a

Dr. wessam alzaidat - General surgery


paraumbilical hernia. Which of the following skin preparation agents would be most
appropriate to use?
A. 2% aqueous chlorhexidine
B. 2% alcoholic chlorhexidine
C. 0.5% alcoholic chlorhexidine
D. Aqueous povidone-iodine solution
E. Isopropyl alcohol
ANSWER: B

22- Common causes of colorectal anastomotic breakdown include all of the following
except:
A. Poor blood supply to the bowel edges
B. Short rectal stump
C. Emergency operation
D. Hand-sewn anastomosis
E. Poor technique
ANSWER D

23- An 85-year-old male patient with a history of chronic constipation presents with
acute severe colicky abdominal pain and absolute constipation. Plain abdominal
film shows a grossly dilated oval of large bowel arising from the left lower quadrant. A
diagnosis of sigmoid volvulus is made. The next step in management is:
A. Laparotomy
B. Sigmoidoscopy with flatus tube insertion
C. Sigmoid colectomy with colostomy
D. Barium swallow
E. Computed tomography
ANSWER B

24- A 22-year-old man in undergoing a rapid sequence induction prior to an


appendectomy. The anesthetist becomes concerned because the patient becomes
tachycardia and develops atrial fibrillation. His temperature is 39.5 C, blood
pressure is slightly elevated. What is the most appropriate course of action?

A. Proceed to immediate laparotomy


B. Administration of broad spectrum intravenous antibiotics

Dr. wessam alzaidat - General surgery


C. Start infusion of inotropes
D. Administration of intravenous Dantrolene
E. Administration of Neostigmine
ANSWER: D

25- About laparoscopic sleeve gastrectomy, all are true except

A. A higher leak rate than laparoscopic roux-en- Y gastric bypass (RYGB)


B. A higher risk of gastroesophageal reflux than RYGB
C. A theoretical advantage over RYGB for iron absorption
D. Desire to eat increases
E. Early satiety happens
Answer: D

26- Unilateral bloody nipple discharge from one duct orifice is most commonly caused
by:

A. Paget's disease of the nipple.


B. Carcinoma in situ.
C. Inflammatory carcinoma.
D. Subareolar mastitis.
E. Intraductal papilloma.
Answer: E

27- A 49-year-old lady with breast cancer is identified as having a 1mm foci of
malignant cells in a sentinel node. What is the term best used to describe this finding?

A. Isolated tumor cells


B. N1 disease
C. Micro metastasis
D. N2 disease
E. n3 disease
ANSWER: C

28- A 40-year-old man who weighs 65 kg is being observed in the ICU. Twenty-four
hours postoperatively, he develops convulsions. His serum sodium is 118 mEq/L.
Appropriate management includes which of the following?
A. Administration of normal saline (0.9%)
B. Administration of hypertonic saline (3%)
C. Emergency hemodialysis
D. Administration of vasopressin
E. Administration of Lasix, 40 mg intravenously (IV)
ANSWER: B

29- A 42-year-old man with small-bowel fistula has been receiving TPN with standard
hypertonic glucose-amino acid solution for 3 weeks. The patient is noticed to have

Dr. wessam alzaidat - General surgery


scaly, hyperpigmented lesions over the acral surfaces of elbows and knees, similar
to enterohepatic acrodermatitis. What is the most likely cause of the condition?

A. Copper deficiency
B. Essential fatty acid deficiency
C. Excess glucose calories
D. Hypomagnesemia
E. Zinc deficiency

ANSWER: E

30- In septic shock, which of the following is true?


A. The mortality rate is between 10% and 20%.
B. Gram-negative organisms are involved exclusively.
C. The majority of patients are elderly.
D. The most common source of infection is the alimentary tract.
E. Two or more organisms are responsible in most cases.
ANSWER: C

31- Bone hunger syndrome can occur after total thyroidectomy for:
A. Papillary thyroid cancer.
B. Multinodular goiter.
C. Medullary thyroid cancer.
D. Graves` disease.
E. Follicular thyroid cancer.

Answer: D

32- Multiple Endocrine Neoplasia type 2 (MEN 2) syndrome is associated with a germ
line mutations in:
A. The p53 tumor suppressor gene.
B. The H-ras gene.
C. The N-myc gene.
D. The RET proto-oncogene.
E. The BRAF proto-oncogene.

Answer: D

33- In primary hyperparathyroidism; the following test has an importance in


anticipating post-surgery bone hunger syndrome:
A. Alkaline phosphatase.
B. Erythrocyte sedimentation rate (ESR).
C. C reactive protein.
D. Preoperative blood calcium level.
E. Preoperative blood phosphate level.

Answer: A

Dr. wessam alzaidat - General surgery


34- All the following are true about amylase in acute pancreatitis except
A. It starts to increase shortly after the onset of symptoms.
B. It peaks within several hours.
C. It remains elevated for 3 to 5 days in average.
D. Its serum levels in general correlate directly with the severity of disease.
E. The laboratory reading of it is commonly normal in acute pancreatitis caused by
hyperlipidemia.
Answer: D

35- 57A 45 years old female who has liver cirrhosis, she has mild ascites, no
encephalopathy but she has grade 2 esophageal varices. Her hemoglobin is 10
gm/dl, platelets count 70000 per cubic mm, INR 2, serum creatinine 2mg/dl, serum
albumin 3 g/dl, and serum bilirubin 4 mg/dl. What is his Child-Turcotte-Pugh
score?
A. 6.
B. 8.
C. 10.
D. 12.
E. 14.

Answer: C

36- All of the following are true regarding familial juvenile polyposis except
A. It is autosomal dominant.
B. The polyps are hamartomatous.
C. Bleeding is common symptom.
D. Once a polyp is detected, total proctocolectomy is recommended.
E. Intussusception is possible presentation.
ANSWER D

37- In which portion of the gastrointestinal tract are proteins mostly absorbed?
A. Stomach.
B. Duodenum.
C. Jejunum.
D. Ileum.
E. Colon.
ANSWER: C

38- In neoplasia, cells progress from normal to cancerous. One of the following is true:
A. Hyperplasia in the absence of atypia or dysplasia, has minimal or no risk of cancer
B. Atypia changes are irreversible
C. Metaplasia describes cells that have altered size, shape & organization
D. All dysplastic tissue progress to carcinoma
E. Hamartoma carries the highest risk of progression to cancer
Answer: A

Dr. wessam alzaidat - General surgery


39- A 2 months old baby presented with an umbilical hernia. The treatment is:
A. Observe
B. Herniotomy
C. Apply a truss
D. Inject sclerosing material
E. Truss for one year then operate
ANSWER: A

40- A 6-day-old baby was born prematurely at 33 weeks. He has been suffering from
respiratory distress syndrome and has been receiving ventilatory support on
Neonatal ICU. He has developed abdominal distension and is increasingly septic.
Ultrasound of the abdomen shows free fluid and evidence of small bowel dilatation.
His blood pressure has remained labile despite inotropic support. What is the best
course of action?
A. Laparoscopy
B. Laparotomy
C. Contrast enema
D. Upper GI contrast study
E. MRI abdomen
ANSWER: B

41- The following best describes the sequence of sensory recovery in a healing skin
graft:
A. Temperature, pain, light touch.
B. Pain, temperature, light touch.
C. Pain, light touch, temperature.
D. Light touch, pain, temperature.
E. Temperature, light touch, pain.
ANSWER C

42- One of the following statements about pulmonary hamartomas is true:


A. Hamartomas are benign chondromas.
B. Most are located in the conducting airways.
C. Wedge resection is curative.
D. A lobectomy is necessary to obtain draining hilar lymph nodes.
E. Hemoptysis is common.
Answer: C

43- To prevent hyperacute rejection, one of the following is true:


A. Ensure HLA matching is present
B. Give FK 506 (Prograf)
C. Ensure ABO blood group compatability
D. Give Prednisone
E. Give Azathioprine

Dr. wessam alzaidat - General surgery


Answer: C

44- A 32-year-old woman is brought to the emergency room with obvious head and
extremity injuries. Primary survey shows she is apneic. Which method is a definitive
airway that is best provided immediately?
A. Orotracheal intubation
B. Nasotracheal intubation
C. Cricothyroidotomy
D. Tracheostomy
E. Laryngeal mask airway
Answer: A

45- A 23-year-old man presents to the emergency room with a stab wound to his anterior
border of the sternocleidomastoid muscle, 1 cm above the cricoid cartilage with
penetration to the platysma. His vitals and oxygen saturation are normal, with no
other physical findings. Which of the following is considered best management?
A. Discharge the patient home, as everything seems normal
B. Observe for 6 hours, if patient drinks well, with no respiratory distress then discharge
C. Observe without testing for 24 hours. If no further complaints, then discharge
D. Perform a CT-angiogram. If no carotid injury, then can be discharged
E. Formal neck exploration or perform upper endoscopy with esophagogram and
angiogram
Answer: E

46- You prescribed silver sulfadiazine ointment (e.g. Flamazine) to a patient who
sustained a third-degree burn to his dorsal right upper extremity. One of the
following is a potential side effect that can occur from this ointment:
A. Hyponatremia
B. Hypocalcemia
C. Methemoglobinemia
D. Hyperchloremia acidosis
E. Neutropenia
Answer: E

47- A 24-year-old man was involved in a motor vehicle accident. His car caught fire with
evidence of carbonaceous sputum, edematous oropharynx, tachypnea, and stridor.
The patient is hemodynamically stable with oxygen saturation of 97%. Before
attempting intubation, which of the following maneuvers is necessary?
A. Avoid oxygenation with his inhalational injury
B. Axial stabilization of his cervical spine
C. Administer paralyzing agents
D. Administer morphine for sedation
E. Immediate needle cricothyroidotomy
Answer: B

48- Which of the following is not a cause of gangrene?


A. Buergers disease.
B. Infection.
C. frostbite.
D. Chronic venous insufficiency.

Dr. wessam alzaidat - General surgery


E. intra-arterial drug injection.
ANSWER D

49- When vein grafts are used to bridge intra-arterial defects: one of the following is
true?
A. The vein wall thins due to the increased luminal pressure.
B. Graft length decreases by 25-30% long term.
C. The graft stretches by 20-30%
D. The vein wall thickens significantly.
E. There is no ingrowth of smooth muscle cells.
ANSWER D

50- All the following are true about arterial aneurysm except
A. The definition is increase in the vessel diameter more than 50%
B. False aneurysm has a wall consist of a single fibrous layer.
C. Mycotic aneurysm caused by fungal infection.
D. False aneurysm is commonly a sequelly of trauma.
E. Abdominal aortic aneurysm is the most common type of large vessel aneurysm.
ANSWER: C

Dr. wessam alzaidat - General surgery


5th year exam 6/2021 MOH
1- Positron Emission Tomography (PET) scan detects:
A. Glucose 6 phosphate molecules
B. Tumor specific proteins
C. Fluorodeoxyglucose molecules
D. Glycogen molecules
E. Necrotic tumor products

ANSWER: C

2- The most important prognostic factor for sarcomas devoid of metastases is:
A. Node status
B. Size of the tumor
C. Tumor grade
D. Gender of the patient
E. Age of the patient

ANSWER: C

3- The most common malignancy following transplantation is:


A. Lung cancer
B. Prostate cancer
C. Breast cancer
D. Hepatocellular cancer
E. Skin cancer

ANSWER: E

4- Parkland formula is used to calculate fluid requirements of burn patients. For a 70-kg
adult with a 40% second degree burn, the total fluid requirements in the first eight hours is:
A. 2.8 liters
B. 4.2 liters
C. 5.6 liters
D. 7.6 liters
E. 12.8 liters

ANSWER: C

5- The best way to diagnose burn wound sepsis is:


A. Wound swab culture
B. Blood culture
C. Examination
D. Biopsy of the wound
E. Measuring C-Reactive Protein level

ANSWER: D

Dr. wessam alzaidat - General surgery


6- One of the following statements regarding thyroglobulin is true:
A. It is part of normal thyroid function tests.
B. It is secreted only by cancerous cells.
C. Autoantibodies does not interfere with its levels.
D. It is a sensitive marker of recurrence of thyroid cancer.
E. Radioiodine treatment affects its concentration.

ANSWER: D

7- The least indicated surgical technique among the following nowadays is:
A. Hemithyroidectomy.
B. Subtotal thyroidectomy.
C. Near-total thyroidectomy.
D. Total thyroidectomy.
E. Total thyroidectomy and central neck dissection.

ANSWER: B

8- One of the following structures is not an anatomical relation to the submandibular


salivary gland:
A. The Mylohyoid muscle.
B. The facial artery.
C. The inferior alveolar nerve.
D. The lingual nerve.
E. The hypoglossal nerve.

ANSWER: C

9- Regarding primary lung tumor, all of the following statements are correct except:
A. Lifetime cigarette smoking is a major risk factor for bronchial carcinoma.
B. Small-cell lung cancer is less common, metastases early and is less amenable to
surgery compared with non-small-cell cancer.
C. Finger clubbing and hypertrophic pulmonary osteoarthropathy are usually incidental
findings and not due to primary lung cancer.
D. The appropriate treatment strategy is dependent on tumor type, tumor stage, and
the general fitness and lung function of the patient.
E. Late survival has a direct relationship with the tumor stage at the time of treatment.
ANSWER: C

10- All the following are true about hepatic adenomas except
A. More common in females.
B. Associated with contraceptive pills.
C. Risk of rupture and bleeding is very low.
D. It carries a risk of malignant transformation.
E. Surgical resection is indicated.

Dr. wessam alzaidat - General surgery


ANSWER: C

11- Which of the following is one of the Ranson’s prognostic signs of acute gallstone
pancreatitis?
A. Age more than 70 years.
B. WBC more than 16000 per cubic mm.
C. Blood glucose more than 200 mg per dL.
D. Serum LDH more than 350 IU per L.
E. Blood urea nitrogen elevation more than 5 mg per dL within the initial 48 hours.
ANSWER: A

12- All the following are true about Insulinomas except


A. It is the most common functional pancreatic neuroendocrine neoplasm.
B. The serum insulin level is elevated with low serum glucose.
C. Serum C-peptide level is elevated.
D. Insulinomas are much more common to be located in the body and tail of the
pancreas in comparison to the head of the organ.
E. Most are benign.
ANSWER: D

13- An epidural hematoma:


A. Is usually arterial in origin.
B. Is 5% accompanied by a skull fracture.
C. Should be suspected only in comatose patients.
D. Can be diagnosed from a brain CT scan.
E. Is usually venous in origin’
Answer: A

14- Which of the following is an absolute contraindication to performing laparoscopic


surgery?
A. Pregnancy
B. BMI > 40
C. Previous abdominal surgery
D. Having symptomatic chronic obstructed pulmonary disease (COPD)
E. Presence of an uncorrected coagulopathy
Answer: E

15- All of the following measures have been recommended for control of acid secretion
in patients with Zollinger-Ellison syndrome except:
A. Antrectomy.
B. Highly selective vagotomy.
C. Total gastrectomy.
D. Vagotomy and pyloroplasty.
E. Medical therapy with omeprazole.
Answer: A

Dr. wessam alzaidat - General surgery


16- Common sites of GIST in descending order as per incidence are
A. Ileum–duodenum–rectum–stomach
B. Stomach–ileum–duodenum–rectum
C. Rectum–ileum–duodenum–stomach
D. Duodenum– stomach–ileum–rectum
E. Ileum-rectum-stomach-duodenum
Answer: B

17- One specific problem that may arise with persistent vomiting after any of the
bariatric operations is Wernicke’s encephalopathy, which can be treated with
parenteral:
A. Vitamin B12
B. Omeprazole
C. Thiamine (vitamin B1)
D. Ascorbic acid (vitamin C)
E. Scopolamine
Answer: C

18- A full-term baby is born with drooling, coughing, and cyanosis after the first feeding,
but these resolve quickly and spontaneously. The next step in management should be:
A. Immediate intubation
B. Placement of orogastric tube
C. Two-view abdominal x-ray
D. Two-view chest x-ray
E. Upper gastrointestinal (UGI) contrast series
ANSWER: B

19- A 2-year-old child presents with an abdominal mass, “raccoon eyes,” and “blueberry
muffin” skin lesions. This most likely represents:
A. Rhabdomyosarcoma
B. Neuroblastoma
C. Wilms tumor
D. Hepatoblastoma
E. Teratoma
ANSWER: B

20- After completion of surgery, an anticholinesterase is administered to reverse the


neuromuscular blockade. Which of the following muscles would be expected to recover
first?
A. Diaphragm
B. Adductor pollicis

Dr. wessam alzaidat - General surgery


C. Ocular muscles
D. Pharyngeal
E. Quadriceps femoris.
ANSWER: A

21- Which of the following statements regarding focused abdominal sonography for
trauma (FAST) is true?
A. It can reliably evaluate the retroperitoneum.
B. It can quickly detect the presence of pericardial
fluid or a pleural effusion.
C. It is useful in detecting a cardiac contusion.
D. It is considered a replacement for computed
tomography (CT).
E. It can reliably detect diaphragmatic injuries.
ANSWER: B

22- According to the Joint Commission, what is the leading cause of sentinel events
(medical errors)?
A. Training errors
B. Systems errors
C. Communication errors
D. Cognitive errors
E. Medical administration errors
ANSWER: C

23- The mucous membrane lining the upper half of the anal canal is
A. lined with stratified squamous epithelium.
B. Drained by the inferior rectal vein
C. Drained into the superficial inguinal lymph nodes
D. Sensitive to touch and to pain
E. Sensitive to stretch
ANSWER E

24- One of the following statements is true regarding gynecomastia:

A. If the disease is unilateral, it is unlikely drug-related.


B. The standard surgical treatment is subcutaneous mastectomy.
C. The presence of gynecomastia is often associated with the subsequent development of

Dr. wessam alzaidat - General surgery


breast cancer.
D. A formal endocrine evaluation is indicated in most juvenile patients with
gynecomastia.
E. Pathologic causes of estrogen deficiency or testosterone excess are associated with
gynecomastia.

Answer: B

25- Most common site of breast cancer metastasis is:

A. Lung.
B. Brain.
C. Bone.
D. Adrenal.
E. Liver.
Answer: C

26- Compared with sporadic breast


cancers, which of the features listed below is least commonly associated with the
breast cancers found in women with a BRCA 1 mutation?

A. High mitotic activity


B. Medullary type
C. Histologically grade 3
D. Predominant lymphocytic
infiltrate
E. Strong estrogen receptor
positivity
ANSWER: E

27- Regarding successful day case surgery, one of the following is not true;
A. minimal access techniques.
B. Good hemostasis.
C. Avoidance of unnecessary tissue handling or tension.
D. Provided with adequate analgesia.
E. Major procedures should be scheduled late on list.
ANSWER E

28- Adrenocortical carcinoma:


A. Is suspected in asymptomatic patients with adrenal tumors larger than 3 cm on CT.
B. Most often is diagnosed early in its course when disease is confined to the adrenal
gland.
C. Is differentiated pathologically from benign adrenocortical adenoma by tumor
necrosis, hemorrhage, and cellular features of large hyperchromatic nuclei and more
than 20 mitoses per high-power field.
D. Should be resected only if disease is localized to the adrenal gland; otherwise

Dr. wessam alzaidat - General surgery


treatment with mitotane is indicated.
E. Carries a very poor prognosis.

Answer: E

29- Low pitch sound after thyroidectomy is usually attributed to malfunction of:
A. Omohyoid muscle.
B. Sternohyoid muscle.
C. Cricothyroid muscle.
D. Thyrohyoid muscle.
E. Sternothyroid muscle.

Answer: C

30- The most sensitive test for pheochromocytoma is:


A. Plasma free metanephrines.
B. 24 h urinary catecholamines.
C. 24 h urinary Vanillylmandelic acid (VMA).
D. Clonidine suppression test.
E. Plasma catecholamines.
Answer: A

31- Surgery for thyroglossal cyst usually includes excision of:


A. Part of Cricoids cartilage.
B. Part of Hyoid bone.
C. Part of Thyroid cartilage.
D. Part of overlying skin.
E. Part of Tongue muscles.
Answer: B

32- One of the following is part of Model for End-stage Liver Disease score
A. Serum albumin.
B. Hematocrit.
C. Platelet count.
D. Serum creatinine.
E. Ascites.

Answer: D

33- All the following are true about Transjugular Intrahepatic Portosystemic Shunt (TIPS)
except
A. Its main indication is to stop esophageal variceal hemorrhage not responded to
medical treatment.
B. It doesn’t affect possible future liver transplant plan.
C. In improves hepatic encephalopathy.
D. It may induce renal failure.

Dr. wessam alzaidat - General surgery


E. Shunt thrombosis rate is high.

Answer: C

34- All the following are common findings in patients with Budd-Chiari syndrome except
A. Occlusion of the three main hepatic veins.
B. Centrilobular necrosis in liver biopsy.
C. Atrophy of Caudate lobe.
D. Decreased liver perfusion via the portal vein.
E. Hepatomegaly.

Answer: D

35- All the following are true about cholangiocarcinoma except


A. They are usually adenocarcinomas.
B. Nodular type is the most common type.
C. Most of them are located intrahepatic.
D. Most cases present with painless jaundice.
E. The tumor marker CA 19-9 can be elevated but it is nonspecific.

Answer: C

36- In which portion of the gastrointestinal tract are proteins mostly absorbed?
A. Stomach.
B. Duodenum.
C. Jejunum.
D. Ileum.
E. Colon.
ANSWER C

37- A 45-year-old man with a 20-year history of quiescent ulcerative colitis undergoes
surveillance endoscopy and is found to have a focus of low-grade dysplasia on random
colonic biopsy in the transverse colon. The most appropriate next step in management
is:
A. Transverse colectomy.
B. Extended right hemicolectomy.
C. Total proctocolectomy with end ileostomy.
D. Total proctocolectomy with immediate J-pouch construction.
E. Transverse colectomy and left hemicolectomy.
ANSWER D

38- A patient with malignant mass inquires about tumor biology. One of the following
is true
A. Most malignant cells generally arise from multiple cell origin, and undergo
malignant transformation to form a malignant clone
B. Malignant cells proliferate slower than normal cells, but the rate of

Dr. wessam alzaidat - General surgery


proliferation becomes faster as the tumor size increases
C. Malignant cells are characterized by reversion to more primitive forms, with
cellular pleomorphism and loss of contact inhibition
D. Tumor doubling time is constant among different types and require 1 year at
most to detect from onset of malignant transformation
E. Anaplasia is the mildest form of malignancy
Answer: C

39- Neoplastic cells have the ability to spread. One of the following is true:
A. Lymph node metastases permeate the sinusoids of the node and later spread
throughout the subcapsular space
B. Carcinoma in situ has no detectable invasion beyond the basement membrane
C. Lymphatic involvement is common with sarcomas
D. The metastatic process of malignancy is highly efficient, as once they shed
into the bloodstream, they produce metastasis almost always
E. Epithelial neoplasm spread only through hematogenous route
Answer: B

40- A 3-year-old boy is referred to the clinic with a scrotal swelling. On examination,
the mass does not transilluminate and it is impossible to palpate normal cord above
it. What is the most likely diagnosis?
F. Hydrocele
G. Direct inguinal hernia
H. Indirect inguinal hernia
I. Femoral hernia
J. Lymphatic malformation
ANSWER: C

41- Which of the following will increase the length of a scar the most?
A. 30-degree Z-plasty.
B. 60-degree Z-plasty.
C. W-plasty.
D. M-plasty.
E. 10-degree Z-plasty.

ANSWER: B

42- Twenty-four hours after colon resection, urine output in a 70-year-old man is 10
mL/h. Blood chemistry analysis reveals sodium, 138 mEq/L; potassium, 6 mEq/L;
chloride, 100 mEq/L; bicarbonate, 14 mEq/L. His metabolic abnormality is
characterized by which of the following?
A. Abdominal distension
B. Peaked T waves
C. Narrow QRS complex
D. Cardiac arrest in systole
E. J wave or Osborne wave

Dr. wessam alzaidat - General surgery


ANSWER: B

43- A 10-year-old boy with history of prolonged bleeding after minor injury is
scheduled for inguinal hernia surgery. The bleeding time, PT, and fibrinogen are
normal. What would be the most helpful investigation?
A. Fibrinolysis (euglobulin clot lysis time)
B. Platelet count
C. Thrombin time
D. Partial thromboplastin time (PTT)
E. Factor VII assay
ANSWER: D

44- from a hematologic viewpoint, the loss of which immune function performed by the
spleen makes patients susceptible to overwhelming post-splenectomy infection
(OPSI)?
A. Loss of IgA.
B. Loss of IgM.
C. Loss of IgG.
D. Loss of IgE.
E. Loss of IgD.
ANSWER B

45- In regards to rejection, one of the following is true:


A. Hyperacute rejection is a cell-mediated immune response
B. Acute rejection occurs secondary to preformed antibodies, as with ABO blood
group incompatibility
C. Chronic rejection never occurs against minor histocompatibility antigens
D. Chronic rejection occurs within the first month, and is easily treated with
immunosuppressive therapy
E. Acute rejection typically is cell-mediated and characterized by graft
infiltration with lymphocytes and mononuclear cells
Answer: E

46- A 23-year-old man presents to the emergency room with a stab wound to his
anterior border of the sternocleidomastoid muscle, 1 cm above the cricoid cartilage
with penetration to the platysma. His vitals and oxygen saturation are normal, with
no other physical findings. Which of the following is considered best management?
A. Discharge the patient home, as everything seems normal
B. Observe for 6 hours, if patient drinks well, with no respiratory distress then discharge
C. Observe without testing for 24 hours. If no further complaints, then discharge
D. Perform a CT-angiogram. If no carotid injury, then can be discharged
E. Formal neck exploration or perform upper endoscopy with esophagogram and
angiogram
Answer: E

47- One of the following is not a risk factor for gastric cancer:
A. Pernicious anemia
B. Helicobacter Pylori

Dr. wessam alzaidat - General surgery


C. Partial gastrectomy
D. Blood group O
E. Dried fish
ANSWER: D

48- The most important step in management of Zenker’s diverticulum of the esophagus
is:
A. Esophagectomy
B. Excision of the diverticulum
C. Division of the cricopharyngeus muscle
D. Observation
E. Division of the superior laryngeal constrictor muscles
ANSWER: C

49- A 25-year-old male presents with symptoms of obstructed defecation. On


examination he is found to have a small full thickness external rectal prolapse. The
most appropriate management for the following patient is
A. Resectional rectopexy
B. Laparoscopic ventral mesh rectopexy
C. Stapled transanal resectional rectopexy
D. Anterior resection
E. Banding of prolapse
ANSWER: C

50- All the following regarding abdominal aortic aneurysm are true except
A. It is the most common type of large vessel aneurysm.
B. The vast majority are associated with atheromatous degeneration.
C. The vast majority starts above the origin of renal arteries.
D. Most remain asymptomatic until rupture occurs.
E. Aortoenteric fistula is a rare presentation.
ANSWER: C

Dr. wessam alzaidat - General surgery


Dr. wessam alzaidat - General surgery
5th year exam MOH
1.In thyroid surgery the avascular space important in identification superior
thyroid vessels:
Cricothyroid space.

2.Which of the following is true about cardiac tamponade in trauma patient?


Less than 100 cc of blood in the pericardium can cause tamponade.

3.Patient with fatty liver and HCC in the Rt lobe of the liver, Rt hepatectomy is
the only surgical option. plan to resect 65 % of the liver. Best to do?
Portal vein embolization followed by resection.

4.All the following mediastinal masses are usually found in the anterior
mediastinum except.
Neurogenic tumor

5.The most important step in treating zencker diverticulum is:


Division of the cricopharyngeous muscles.

6.Sorafenib :
Hepatocellular carcinoma.

DR.WBZ

Dr. wessam alzaidat - General surgery


7.a 10-year-old with a cycling accident . CT scan :crush injury to the distal
pancreas with pancreatic duct disruption . management?
Laparotomy and distal pancreatectomy.

8.In colon cancer one is incorrect:


If at operation hepatic Mets found, biopsy should be done.

9.A 57-year-old lady has a 1.5 cm speculated, centrally dense non palpable
mass, radiological guided true-cut biopsy showed benign parenchyma. the
most appropriate recommendation for her is:
Wire –localized excisional biopsy.

10.Female underwent sleeve gastrectomy 1 year go, presented with fatigue,


SOB on exertion, with palpitation. The most likely cause of her presentation is:
Iron deficiency anemia.

11.Arterial embolization may be used in treatment except:


Gastroesophageal varices.

12.Which is correct in breast CA?


Medullary breast cancer has better prognosis than the other IDC.

13.In FAP one of the following statements is incorrect:


A. More than 80% occur with positive familial history.
B. The majority will become malignant.

14.indications for hyperbaric oxygen therapy in wound and ulcer management


except:

Dr. wessam alzaidat - General surgery


DM ulcer Wigner 2

15.Cyclosporine side effect ,except :


Hypokalemia

16.Umbilical hernia in children except :


Supraumbilical hernias often close spontaneously

17.m.c malignancy in children age less than 2 years :


Neuroblastoma

18.Saline suppression test:


Conn disease

19.A case of rigid abdomen and increased End tidal co2 on induction (
malignant hyperthermia) :
which drug help in this case ?
Dantrolene

20.about parathyroid , one is incorrect :


superior parathyroid gland are less constant

21.tumor marker associated with papillary thyroid cancer:


BRAF

Dr. wessam alzaidat - General surgery


22.P53 gene mutation found in all except:
wilm’s tumor
neuroblastoma
breast cancer
soft tissue sarcoma
one more option

23.a case of rectal tumor , 4 cm from the anal verge with internal sphincter
involvement , management ?
APR

24.about Gallbladder CA , one is true :


G.B.S is the most important risk factor

25.about cholangiocarcinoma , one is incorrect :


klatskin tumor usually needs hepatic resection

26.rectal prolapse in children :


constipation should be treated

27.post operative patient has atelectatsis (can’t remember the exact scenario )
but the question was about which method should be used to avoid
endotracheal intubation :
nonrebreather mask
tent mask

Dr. wessam alzaidat - General surgery


venturi mask

28.Chest trauma with loss of full thickness part of the chest wall , management
:
occlusive dressing

29.About alginate for wound all true except :

30.Odd ratio:

31.Most common type of kidney stone in post resection in crhon pt


calcium oxalate

32.About lymphangitis all true except :


Most common organism is pseudomonas

33.About AAA one is wrong :


Endovascular repair has lower mortality rate than open repair
DR.WBZ
34.A case of isolated gastric varices due to pancreatitis , management :
Splenectomy

35.Baux score:
Age

36.RTA, laparotomy, Mx?

Dr. wessam alzaidat - General surgery


tranexamic acid

37.hypospadias, incorrect:
mainly to decrease sexual dysfunction

38: torsion?
should be managed in the first two hours ??!!

39.breast reconstruction:
latissimus dorsi flap

40.post splenectomy, blood changes:


reticulocyte

41.secretin effect, except:


increase gastrin

42.nasotracheal intubation is most suitable in :


penetrating neck injury

43.post gastric sleeve pt , to avoid complication of refeeding syndrome , give ?


Thiamine
44.a case of trauma intubated patient with rigid abdomen , your next step ?
go to the OR upon your decision

45.about pyloric stenosis one is false :


age from 6wks - 13 wks

Dr. wessam alzaidat - General surgery


46.about anal fissure :
GTN glyceryl nitrate is successful in TX

47.in which types of hiatal hernia the Gastroesophageal sphincter is found


above the diaphragm :
type 1 and 3

48.the interval which differentiate the temporary from permanent RLN injury :
6 months

49.all are causes of colovesical fistula except :


amebic colitis

Dr. wessam alzaidat - General surgery


10. Which is the most commonly cultured hospital acquired
organism in patients with aspiration pneumonia?
(A) Streptococcus pneumoniae
(B) Staphylococcus aureus
(C) Anaerobic species
(D) Pseudomonas aeruginosa
(E) Haemophilus influenza

ANSWER D

11. What is the most appropriate single agent for empiric


coverage of the above patient?
(A) Metronidazole

Dr. wessam alzaidat - General surgery


(B) Clindamycin
(C) Piperacillin-tazobactam
(D) Vancomycin
(E) First-generation penicillin

ANSWER C

postoperative fluid management


-Decreased urine output is a physiologic
response to surgical stress

Which of the following statements regarding


management of dermatofibrosarcoma protuberans
is true?

E. Imatinib can be used for locally advanced


and metastatic disease.

A 37-year-old woman undergoes a femoral


sentinel node biopsy for a 1.7-mm nonulcerated
melanoma of the thigh. Pathology on the single
node reveals a micrometastasis. She inquires
about a completion groin node dissection. She
should be told that this procedure

B. improves disease-free survival.

Dr. wessam alzaidat - General surgery


A 61-year-old man presents with the arm lesion shown
in figure 7.1. A biopsy is read as Merkel cell carcinoma.
Physical examination is otherwise unremarkable. A
metastatic workup is negative. What is the
recommended treatment?

D. Wide local excision and postoperative radiation

radiation therapy reduces the local relapse rate


significantly and may improve overall survival. Thus, adjuvant
radiation therapy to the primary site after resection is the
standard of care
no benefit of adjuvant chemotherapy is seen, and it is usually
reserved for metastatic disease

Dr. wessam alzaidat - General surgery


5th year mcq MOH 2021
1-Lung hamartoma: wedge resection

2-False about gastrinoma: small single ulcer

3-False about rectal anastmosis: hand sewin is a risk factor

4-3 yo with target sign: meckles

5-Sensation of graft skin: pain then touch then temperature

6-False about breast ca: medullary and tubular types have worse
prognosis

7-Perisinsoidal portal hypertension: schistosomiosis

8-Sigmoid volvulus next step: segmiodoscopy with rectal tube

9-False about pancreatitis: amylase level correlate with severity

10-Space posterior and to the left of winslow foramen is: lesser sac

11-Skin preparation best: 2% alcohol based chlorhexidine

12-Zinc deficiency: acrodermatitis at flexure regions

13-Stab wound neck zone 2, penetrating platysma, and asymptomatic:


observe patient for 24 hour then discharge.

14-Not an indication for splenectomy: G6PD

15-False about gallbladder ca: squamous is most common

16-Patient with na level 118 and seizure next step: hypertonic saline

17-Premature with NEC and liable bp despite vasopressor: laparatomy

18-Trauma pateint with head and axial injry, immidiate definitive airway
is: orotracheal intubation

19-True about sepsis: most common in elderly

Dr. wessam alzaidat - General surgery


20-False about Juvenile polyp: colon resection once polyos develop in
colon

21-Burn patient, fasciotomy done, 10 hours later he is aneuric, CVP 18,


most likely cause: myoglobinuria

22-Patient with minimal ascitis, no encephalopathy, inr 2, creatine 3,


albumin3, bilirubin 4, his child-pugh score is: 10

23-Irreversable nerve injury in: neurotmesis

24-False about mri in breast: better than mammography in elderly

25-SLNB with 1 mm node invasion is: micrometastasis

26-Electrocautery device not associated with magnetic field interference


is: us devices

27-Predictor of hungry bone syndrome after parthyroidectomy : alkaline


phosphatase level

28-Risk for hungry bone syndrome: graves disease

29-Limitation of PET scan in : hyperglycemia

30-Men 2 : ret mutation

31-The most common testicular tumor in children is: yolk sac tumor

32-Best modality to reduce mortality in Bleeding peptic ulcer is:


endoscopy

33-About gastric polyps: adenomatous polyp larger than 2 cm require


resection

34-Protein absorption in: jejunum

35-Umbilical hernia in 3 month infant treatment: observation

36-About patholgy: hyperplasia without atypia or dysplasia have low or


minimal risk for progression to cancer

37-Radiotherap target is : DNA

Dr. wessam alzaidat - General surgery


38-Trauma rta and burn, next step before intubation: axial stabalizatiin
of spine

39-False about sleeve gastrectomy: desire to eat increase

40-Not complication of laparoscopic pneumoperitonium: hyperthermia

41-Decrease cardiac output after laparascopic insuflation: occur when


intraabdominal pressure 20mmhg

42-False about AAA: mycotic aneurysm caused by fungal

43-Does not cause gangarene: venous insufficiency

44-After rapid sequence intubation, Sudden increase in co2 and


hyperthermia: treat with dantrolene

46-Low risk of malignancy in thyroid ultrasound: spongiforum


appearance

47-Changes in arterial vein graft: thickining in the wall

48-Silverin se: neutropenia

49-overweight bmi: 25-29.9

50- single duct, blood discharge: intraductal papilloma

Dr. wessam alzaidat - General surgery


site infection:
Board 2021 A. 3 days.
1.During thyroid surgery, the following B. 12 -14 days.
avascular space has a crucial role in C. 6- 7 days.
identification and control of superior D. 14- 21 days.
thyroid vessels: E. 22- 28 days.
A. Thyrohyoid space.
B. Sternothyroid space. ANSWER: B
C. Sternohyoid space.
D. Cricothyroid space. 6. Which is incorrect regarding
E. Suprathyroid space. inflammatory bowel disease?
A. The risk of developing ulcerative colitis
ANSWER:D more in nonsmoker than smoker.
B. Patients with CD are more prone to
2.One of the following is the maximum cancer than UC.
amount of time that extracorporeal circulation C. Effectiveness of colonoscopic
can be tolerated before significant risk for surveillance in cancer is controversial in
physiologic injury and metabolic defect occur? IBD.
A. 2 to 4 hrs. D. Toxic megacolon occurs in CD more
B. 6 to 8 hrs. than UC.
C. 10 to 12 hrs. E. Cobblestone appearance on bowel was
D. 14 to 18 hrs. characteristic of CD.
E. 18 to 20 hrs.
ANSWER:B
ANSWER: B
7. The serum level of the tumor marker
3.Which of the following is true about CA15-3 is best used in which of the
cardiac tamponade in trauma patient? following clinical situation?
A. It is more frequent after blunt chest A. Screening of breast cancer.
trauma. B. Monitoring patient with advanced
B. Less than 10 cc of blood in the breast cancer for recurrence.
pericardium can cause tamponade. C. Determining prognosis for early breast
C. Becks triad usually present. cancer.
D. Cardiac output increase. D. Staging of breast cancer.
E. Pericardiocentesis is successful in E. Diagnosis of breast cancer.
decompressing tamponade in about
25% of cases. ANSWER:B
ANSWER: C
8. All the following statement are true
4.Submandibular lymph nodes are considered? regarding blood supply of graft except?
A. Level 1a. A. Skin blood supply comes from muscles
B. Level 1b. and fascial perforating vessels.
C. Level 1c. DR.WBZ
D. Level 2. B. Direct cutaneous vessels can also
E. Level 3. contribute to the blood supply of the
ANSWER:B skin.
C. It requires receiving area to be
5.Which is the average additional length of stay vascularized so that ingress of
in hospital for patient that develop a surgical capillaries into the graft can occur and

Dr. wessam alzaidat - General surgery


revascularized. upper posteromedial aspect of the arm
D. A full thickness graft has the whole after breast surgery and axillary
dermis attached with fat trimmed off. dissection is usually due to:
E. A composite graft is a full thickness A. Lymphatic fibrosis.
graft to which other structure such as B. Long thoracic nerve resection.
hair may be added by suturing on. C. Intercostal brachial nerve cut.
D. Axillary vein thrombosis.
ANSWER:E E. Thoracodorsal nerve injury.

9. In vascular anastomosis the suture ANSWER:C


material used should be all except.
A. Non absorbable. 13. Which of the following is not true of
B. Elastic. diverticulitis?
C. Non elastic. A. Occur in 10 25% of patient’s
D. Mono filament. diverticulosis.
E. Smooth. B. Mostly occur in the Lt side of the colon.
C. Urinary frequency, pyuria, and dysuria
ANSWER: B is not uncommon.
D. Sigmoidoscopy is the best diagnostic
10. The most significant prognostic factor tool at time of presentation.
in patient with node positive stage 3 E. Mild cases can be treated as outpatient
melanoma is? with oral Amoclan plus clavulanic acid
A. Size of involved node. and for seven days.
B. Number of involved LN.
C. Tumor thickness. ANSWER:D
D. Tumor ulceration.
E. Patient age. 14. Which of the following is not cause of
gangrene?
ANSWER: B A. Burger disease.
B. Infection.
11. A 65-year-old male with previous C. Intra-arterial drug injection.
hepatosteatosis develop an HCC in the D. Forestbite.
Rt hemi-liver. A Rt hemi-hepatectomy E. Deep vein thrombosis.
is the only potential surgical option. His ANSWER:E
surgeon plan to resect 65 % of the liver. 15. In the esophageal cancer, all the
what is the best strategy? following is true except:
A. Proceed with surgery without A. T1b lesion can carry a 30-50%
embolization. involvement of lymph nodes.
B. Palliation alone. B. Longitudinal spread is common due to
C. Hepatic artery embolization followed the nature of lymphatic within the
by resection. esophageal wall.
D. Down staging with chemotherapy C. Skip lesion can occur due to
followed by resection. longitudinal lymphatic spread.
E. Portal vein embolization followed by D. The level of the Carina defines the
resection. preferential spread of lymphatic.
E. Adenocarcinoma respond well to
ANSWER:E radiation therapy.

12. Hypoesthesia and numbness of the ANSWER:E

Dr. wessam alzaidat - General surgery


E. Upper respiratory tract infection.
16. A 37-year-old female, who previously
underwent laparoscopic Roux-en-Y ANSWER:C
gastric bypass present to your office
with severe recurrent colicky abdominal 19. The proper duration of antibiotics for a
pain without vomiting. she is ventilator associated pneumonia caused
hemodynamically stable with free by Klebsiella pneumoniae is:
ultrasound and prominent gastric A. 8 days.
remnant on CT scan examination, your B. 15 days.
next step in management is: C. Until the x ray has cleared.
A. Diagnostic laparoscopy to r/o internal D. Until fever has resolved, and white
hernia blood cell count has normalized.
B. HIDA scan to r/o acalculous E. Until the patient is liberated from the
cholecystitis. ventilator.
C. Upper endoscopy to r/o marginal ulcer.
D. Continue to observe without any ANSWER:A
intervention.
E. Blood work to assess vitamin or 20. Which of the following interventions
mineral deficiency. improves survival in a patient with
moderate to severe acute respiratory
ANSWER:A distress syndrome?
A. High frequency oscillatory ventilation,
17. One of the following is true while early physical therapy, and
operating on patient with known gastric bronchoscopy.
adenocarcinoma: B. High tidal volume mechanical
A. Subtotal or near total gastrectomy carry ventilation, early neuromuscular
the same quality of life outcomes as blockade, and inhaled nitric oxide.
total gastrectomy. C. High tidal volume mechanical
B. If curvature resection is intended a 5 cm ventilation, recruitment maneuvers, and
margin of microscopically free normal bronchoscopy.
tissue is adequate. DR.WBZ
C. D2 lymphadenectomy improves
survival in managing patient with D. Low tidal volume mechanical
gastric adenocarcinoma. ventilation, early neuromuscular
D. Splenectomy is indicated of station 10 blockade, and prone positioning.
lymph nodes is grossly enlarged. E. Low tidal volume mechanical
E. Single liver metastasis does no ventilation, high dose steroids, and
necessitate preoperative neoadjuvant inhaled nitric oxide.
therapy.
ANSWER:D
ANSWER:C
21. A 72-year-old woman is recovering
from a partial gastrectomy. On
18. What is the most common healthcare
postoperative day 3, she is agitated and
acquired infection (HAI) among
confused. Postoperative delirium after
hospitalized patients?
major abdominal surgery in older
A. Ventilator associated pneumonia.
patients is associated with
B. Surgical site infection.
A. Increased length of hospital stays.
C. Urinary tract infection.
B. Increased intensive care unit admission
D. Genital tract infection.
rates.
Dr. wessam alzaidat - General surgery
C. A return to operating room. diaphragmatic crura in the parietal
D. No change in long term cognitive pleura.
performance.
E. Wound dehiscence. ANSWER:E

ANSWER:A 25. One of the growth factors that


participate in the process of wound
22. A 100 kg woman is admitted with healing.it has function in chemotaxis of
cholangitis. After endoscopic retrograde fibroblast and monocytes, in fibroblast
cholangiopancreatography clears her mitogenesis and stimulation of
common bile duct of obstructing bile angiogenesis and collagen synthesis and
duct stones, she is admitted to the mediator of tissue remodeling, what is
intensive care unit for persistent the factor?
hypotension (blood pressure 80/60 mm A. Epidermal growth factor.
Hg). The recommended fluid infusion B. Fibroblast growth factor.
for the first 3 hours of resuscitation is: C. Interleukin 1.
A. 1000 mL of crystalloid. D. Interleukin 6.
B. 2000 mL of crystalloid. E. Platelet derived growth factor.
C. 3000 mL of crystalloid.
D. 500 mL 5% albumin. ANSWER:E
E. 500 mL of 6% hydroxyethyl starch.
26. All the following mediastinal masses
ANSWER:C are usually found in the anterior
mediastinum except.
23. All the following are early A. Thymoma.
complications after the repair of B. Thyroid mass.
tracheoesophageal fistula repair, C. Lymphoma.
except? D. Teratoma.
A. Anastomotic leak. E. Ganglioneuroma.
B. Esophageal stricture. ANSWER:E
C. Recurrent fistula.
D. GERD. 27. The most important step in treating a
E. Disruption of the anastomosis. pulsion upper esophageal diverticulum
is:
ANSWER:D A. Resection of diverticulum.
B. Diverticulopexy.
24. When performing laparoscopic Nissen C. Esophagectomy.
fundoplication, the patient become D. Division of the superior laryngeal
markedly hypotensive, hypoxic, and constrictor muscles.
difficult to ventilate, the E. Division of the cricopharyngeus
anesthesiologist notes that the patient muscles.
breath sound are now diminished on the
left. What of the following should not ANSWER:E
be considered?
A. Placing an angio-catheter in the left 28. A 41-year-old male with a body mass
chest. index of 47 kg/m2 is visiting your office
B. Placement of a left tube thoracotomy. for bariatric surgery consolation. it is
C. Deflation of the abdomen. essential to complete of the following
D. Discontinuing nitrous ventilation. prior to any surgical intervention
E. Dissecting through the left except.
Dr. wessam alzaidat - General surgery
A. A complete weight history.
B. STGP–BANO questioner. ANSWER:C
C. Loss of 10 % of total body weight in
preparation for surgery. 32. Sorafenib is an approved targeted
D. Upper endoscopy. therapy in the treatment of?
E. Comprehensive chemistry panel. A. Breast cancer.
B. Malignant melanoma.
ANSWER:D C. Colorectal cancer.
D. Pancreatic cancer.
29. All the following are considered higher E. Hepatocellular carcinoma.
risk requiring more frequent monitoring
in patient who have Barret’s esophagus ANSWER:E
without dysphagia except 33. a 10-year-old boy is brought to the
A. Male gender. emergency room after a cycling
B. Age 53. accident where he thrown against the
C. Truncal obesity. handlebars. CT scan is performed and
D. Presence of cagA strain of H. pylori on demonstrate a crush injury to the distal
natural biopsy. pancreas with disruption of the
E. Family history of esophageal pancreatic duct. how should this be
adenocarcinoma. managed?
A. Laparotomy and ductal repair.
ANSWER:D B. Laparotomy and distal pancreatectomy.
C. Conservative management with
30. Sleeve gastrectomy patients are best octreotide.
counseled on all the following risks D. Laparotomy and
except? pancreaticoduodenectomy.
A. Increase de novo GERD. E. Radiological drainage and octreotide.
B. Increased concern on Barret’s ANSWER:B
esophagus and need for surveillance 34. what is the distance between the upper
endoscopy. esophageal sphincter and esophageal
C. Risk of vitamin and mineral deficiency narrowing caused by the aortic arch in
and need for supplements. an average adult male?
D. Risk of gallstones formation with A. 10 cm.
weight loss. B. 15 cm.
E. Risk of protein calorie malnutrition. C. 25 cm.
D. 30 cm.
ANSWER:E E. 40 cm.

31. Post anesthesia scoring for day case ANSWER:A


surgery are true except.
A. Total score more than 9 consider the 35. In large bowel cancer one of the
patient fit for discharge. following is false:
B. Surgical bleeding considers part of this A. Through preoperative assessment
scoring. should be done with colonoscopy US
C. Vital signs, pain, oral intake, and and spiral CT.
voiding included in the score. B. Resection is done if the patient has liver
D. Severe pain, nausea and vomiting metastasis.
require treatment has adverse effect on C. If at operation, hepatic metastases are
the score. found, biopsy should be done.
E. Orientation and steady gait good sign. D. Hepatic resection for metastasis should
Dr. wessam alzaidat - General surgery
be considered as a staged procedure. C. Short term follow-up in 4 to 6 months.
E. Over 95 percent of colonic carcinoma D. Wire –localized excisional biopsy.
can be respected. E. Tamoxifen.

ANSWER:C ANSWER:D

36. A 40-year-old man who weight 65 kg is 39. A 50-year-old male patient with
being observed in the ICU. twenty-four epigastric pain, unrelieved by PPI,
post operatively. he develops undergoes an upper endoscopy which
convulsion. his serum sodium is 118 shows a mass found in the stomach
meq/L. appropriate management mucosa only. Biopsy shows normal
includes which of the following? lymphoproliferative tissue, the next
A. Administration of normal saline 0,9 % appropriate therapy is:
at rate 1 ml/kg/hr. A. Triple therapy for helicobacter pylori.
B. Administration of hypertonic saline 3% B. Chemotherapy alone.
at rate of 0,28 ml/kg/hr. C. Chemotherapy with radiotherapy.
C. Emergency hemodialysis. D. Partial gastrectomy with
D. Administration of hypertonic saline 3% lymphadenectomy
at rate 1ml/kg/hr. E. Total gastrectomy with
E. Administration of Lasix 40 mg lymphadenectomy.
intravenously.
ANSWER:A
ANSWER:B
40. All the following mediastinal masses
37. All the following statements regarding are usually found in the anterior
propofol are true except? mediastinum except:
A. Induction and sedation doses for A. Thymoma.
children are higher than those for adult. B. Lymphoma.
B. In lower dose it used for conscious and C. Thyroid mass.
sedation. D. Teratoma.
C. Propofol dose suppress the respiratory E. Ganglioneuroma.
system.
D. The incidence of postoperative nausea ANSWER:E
and vomiting is increased with propofol
based anesthesia. 41. The incidence of lymphedema after
E. Propofol does not have analgesic axillary node dissection (level 1 and 2)
properties. is around:
A. 5%.
ANSWER:D B. 25%.
C. 50%.
38. A 57-year-old lady has a 1.5 cm
D. 75%.
speculated, centrally dense non palpable
E. 90%.
mass, radiological guided true-cut
biopsy showed benign parenchyma. the
ANSWER:B
most appropriate recommendation for
her is:
42. A 28-year-old female patient who
A. Routine screening mammography in 1
previously underwent a sleeve
year.
gastrectomy 1 year go, presented with
B. Additional margin with contrast
fatigue, SOB on exertion, with
enhanced MRI.
Dr. wessam alzaidat - General surgery
palpitation. The most likely and the
commonest case of her presentation is: ANSWER:A
A. Thyrotoxicosis.
B. Vitamin B12 deficiency. 46. The following are true about TIPS
C. Folic acid deficiency. except:
D. Iron deficiency anemia. A. Its main indication is to stop variceal
E. Pulmonary embolism. hemorrhage not responded to medical
treatment.
ANSWER:D B. It's does not affect possible future liver
transplant plan.
43. A previously healthy patient presented C. It improves hepatic encephalopathy.
with peritonitis secondary to perforated D. It may induce renal failure.
diverticulitis after adequate source E. Shunt thrombosis rate is high.
control she would receive 4 days of:
A. Gentamycin. ANSWER:C
B. ___
C. Clindamycin. 47. Epithelization by secondary intention in
D. Fluconazole. healing of a skin wound reaching the
E. Piperacillin-tazobactam. dermis or deeper is due to:
ANSWER:E A. Meet _____ of dermal ______ cells.
B. Migration of the basal cells of the edge
44. All the following are true about the of the wound _____.
anatomy of the rectum, except: C. Migration of the endothelial cells from
A. In adult it is about 12-16 cm in length. the newly formed vessels in the wound
B. It contains 3 valves. bed.
C. The presacral fascia separate the rectum D. ______ stem cells in the bone marrow
from the presacral venous plexus. brought to the wound by blood vessels.
D. The Waldeye’s fascia separate the E. Cells from stratum spongeum
rectum from the prostate and seminal underwent differentiation.
vessels in male.
E. The lateral rectal ligaments support the ANSWER:B
lower rectum.
48. A 69-year-old female S/P Rt . The
ANSWER:D procedure is complicated by a nerve
injury proximal to posterior belly of the
45. One of the following is correct digastric muscle, identified by
regarding the breast cancer local following the superior root of the Ansa
recurrence: Cervicalis. One of the following will be
A. The axillary nodal status is major factor found on the physical examination:
in predicting local recurrence. A. She would not have pain pick sensation
B. Regional LN recurrence is common in the right side of the tongue.
after ALND. B. Her tongue would deviate to the right
C. Recurrence after breast conserving when stung.
surgery is not negative prognostic C. She would have no taste on the right
factor. side of the tongue.
D. The treatment of chest wall recurrence D. She would have persistent bradycardia
is XRT and chemotherapy. postoperatively.
E. Most patients with local recurrence due E. The Rt mouth angle would deviate to
to local infection and bleeding. the right side.

Dr. wessam alzaidat - General surgery


A. Mucinous breast cancer is common
ANSWER:B variant of invasive breast cancer.
B. Medullary breast cancer has better
49. A 43-year-old male who had a stab prognosis than the other IDC.
wound in the right chest with massive C. The presence of DCIS within the
hemo-pneumothorax. After initial invasive cancer adversely affects the
resuscitation you decide to put a more prognosis.
durable wide-bore venous access. the D. Invasive ductal cancer has high
best route for the access is: incidence of bilateral cancer.
Must not be a central line. E. Apocrine breast cancer is low grade
tumor with _______.

50. Arterial embolization may be used in ANSWER:B


treatment of all conditions except:
A. Acute hemorrhage. 54. A 54-year-old male develop back and
B. Tumor resection. hip pain, followed in several days by
C. Arteriovenous malformation. sciatica. On examination he has
D. Hypersplenism. weakness in the right quadriceps and
E. Gastroesophageal varices. absence of the Rt knee and both ankles
reflexes, the rest of the leg has good
ANSWER:E strength, the most likely diagnosis is:
A. Herniated disc at L5-S1 with S1
51. Where is most cholangiocarcinoma compression.
located? B. Herniated disc at L4-L5 with L5
A. Left common hepatic duct. compression.
B. Between the upper border of pancreas C. Herniated disc at L3-L4 with L4
and the ampulla of Vater. compression.
C. Right common hepatic duct. D. Herniated disc at L2-L3 with L3
D. True intrahepatic bile ducts. compression.
E. Perihilar region. E. Proximal diabetic neuropathy.

ANSWER:E ANSWER:E

52. All of the following are true about


55. If you suspect parathyroid carcinoma
Vagus near except:
during parathyroidectomy for primary
A. The Vagus located in the floor of the
hyperparathyroidism, the best surgical
4th cerebral ventricle.
approach provided no that no
B. It travels in the neck in the carotid
suspicious lymph nodes will be:
sheath.
A. Excision of the enlarged parathyroid
C. In the mediastinum it gives the RLN.
only.
D. It divides into Rt and Lt branches
B. Excision of the enlarged parathyroid
around the thoracic esophagus.
with adjacent thyroid lobe.
E. Near the GE junction, the anterior
C. Excision of the enlarged parathyroid
Vagus branch to the liver.
with adjacent thyroid lobe and
prophylactic CND.
ANSWER:D
D. Excision of the enlarged parathyroid
with adjacent thyroid lobe and
53. Which of the following statements is
prophylactic CND and ipsilateral LND.
correct?
E. Excision of the enlarged parathyroid
Dr. wessam alzaidat - General surgery
with adjacent thyroid lobe and left upper quarter of the abdomen.
prophylactic CND and bilateral LND.
ANSWER:A
ANSWER:B
59. Which of the following chemotherapies
agent is an anti-metabolic
A. Methotrexate.
56. All of the following are accepted B. Cholosancil.
indication for hyperbaric oxygen C. Cyclophosphamide.
therapy in wound and ulcer D. Melphalan.
management except:
A. Clostridium myonecrosis
B. Crashed injury. ANSWER:A
C. Radiation induced soft tissue necrosis.
60. In FAP one of the following statements
D. DM ulcer Wigner 3.
is incorrect:
E. Refractory osteomyelitis.
A. It is inherited as mendelian dominant
condition.
ANSWER:D
B. More than 10% occur with positive
familial history.
57. Which of the following is the most
C. The majority will become malignant.
common variation in blood supply of
D. The condition usually manifest by the
the upper GI tract:
age of 15.
A. A posterior gastric artery presents.
E. Family members should be offered
B. A right gastric artery arising from the
genetic testing in early teen.
common hepatic artery.
C. A left gastric artery arising from the
ANSWER:B
common hepatic artery.
D. A left gastric vein entering the splenic
61. Percutaneous tracheostomy:
vein.
A. Has higher complication rate than
E. The inferior mesenteric vein entering
surgical tracheostomy.
the superior mesenteric vein.
B. It is safe without bronchoscopy
guidance.
ANSWER:E C. It is safest when performed by surgeon.
D. It is contraindicated in morbidity
58. Which of the following case scenario is obesity.
more likely to be managed non- E. It is contraindicated in patients with
operatively: recent cervical spine surgery.
A. A low energy bullet through the right
upper region entered in the liver in a
hemodynamically stable patient. ANSWER:B
B. A low energy bullet through the pelvis
62. A 55-year-old man is undergoing
in a hemodynamically stable patient.
Whipple procedure for carcinoma of the
C. A bullet seems to have an extra-
pancreas, which of the factors listed can
peritoneal trajectory in a
carries the greatest risks for the
hemodynamically stable patient with
development of post-operative
abdomen rigidity.
pancreatic fistula:
D. A bullet with retro-peritoneal trajectory.
A. Not receiving steroids post-operative.
E. A low energy bullet with an inlet in the
B. Soft pancreatic tissue in the pancreatic
left 6th intracostal space asserted in the
remnant.
Dr. wessam alzaidat - General surgery
C. High serum bilirubin prior to biliary A. Subclavian vein.
decompression pre-operatively. B. Internal jugular vein.
D. Placement of internal catheter than C. Femoral vein.
external pancreatic catheter. D. Long saphenous vein.
E. Pre-operative chemoradiotherapy. E. Basilica vein.
ANSWER:C DR.WBZ
ANSWER:B
66. The incidence of lymphedema after
63. All the following are true about human axillary node dissection (level 1 and 2)?
leukocyte antigen except? A. 5%.
A. Class 1 HLA antigen are expressed by B. 25%.
all nucleated cells. C. 50%.
B. Class 2 HLA antigen are expressed by D. 75%.
antigen presenting cells. E. 90%.
C. Their principal function is to present
fragment of foreign proteins to B ANSWER:B
lymphocyte. 67. A 28-year-old female who previously
D. They play a crucial role on transplant underwent sleeve gastrectomy 1 year
recipient. ago. Present with fatigue, shortness of
E. They can trigger rejection of a graft. breath on excretion with palpitation, the
ANSWER:C most likely and commonest to cause her
presentation is:
64. A 73-year-old lady has long standing A. Thyrotoxicosis.
CML and massive splenomegaly. B. Thamin b 12 deficiency.
decision is made to proceed with C. Folic acid deficiency.
splenectomy. what is the best operative D. Iron deficiency anemia.
strategy? E. Pulmonary embolus.
A. Splenectomy performed via a rooftop
incision with early division of the ANSWER:D
splenic artery. 68. Regarding lymphangitis, which of the
B. Laparoscopic splenectomy with early following incorrect?
division of the splenic vein. A. Lymphangitis is usually caused by
C. Splenectomy performed via a midline Psedomonas species.
incision with early division of the B. Lymphangitis is indicated by red
splenic artery. streaking in the limb.
D. Splenectomy performed via left C. Lymphangitis may predispose to further
subcostal incision with early splenic attacks.
artery. D. Lymphangitis is common in patient
E. Splenectomy performed via a midline with lymphedema.
incision with early division of the E. Lymphangitis may often result in
splenic vein. bacteremia.
ANSWER:C ANSWER:A
65. A 43-year-old male who had a stab 69. Which of the following is true
wound in the Rt chest with massive regarding anal condition?
hemopneumothorax. After initial A. Anal fissure usually occurs on anterior
resuscitation you decided to put a more wall regularly and may fissure outside
durable wide bore venous access. the of this area rises suspicion for other
best route for the condition is: pathology.
Dr. wessam alzaidat - General surgery
B. Glycerin trinitrate ointment has been E. Amino acid excess.
used with success on the fissure.
ANSWER:B
C. Perianal abscess is usually located in
the natal cleft in the midline. 75. HYPERCALIMIA, EXEPT?
D. Ischiorectal abscess are usually obvious 76. BURN.
on examination of the perineum with 77. BURN.
fluctuant abscess lateral to anus. 78. ETHICS.
E. Anal fissures are commonly secondary 79. STAT.
to Crohn’s disease.
ANSWER:B BEST OF LUCK
DR.Wessam Bassam ALZaidat
70. Currarino triad or syndrome is:
A. Sacral agenesis, anorecta
malformation, presacral teratoma.

71. Vascular anastomosis thread:


A. Polyproline 5/0.

72. All the following are false about silver


sulfadiazine except?
A. It has a wide range of antimicrobial
activity.
B. It is not used to treat infected burn
wound.
C. It has soothing quality and decrease
pain.
D. It is significantly absorbed
systemically.
E. It destroys skin graft.
ANSWER:E
73. Which of the following is part of Beaux
score?
A. Patient age.
B. Depth of burn.
C. Chronic medical illness.
D. Associated limb injuries.
E. Circumferential burns.
ANSWER:A
74. Patient after TPN present with scaly
cubital fossa rash and weakness, hair
loss most likely due to:
A. Linoleic acid deficiency.
B. Zinc deficiency.
C. Vitamin C deficiency.
D. Magnesium deficiency.
Dr. wessam alzaidat - General surgery
JRMS EXAMS
DR.WBZ
1. The right ureter in female, which is CORRECT?

Runs over the ovarian artery .AThe


uterine artery runs under the ureter in the pelvis .B
Is crossed by the right colic artery .CLies
beneath the third part of duodenum at its origin .DLies
beneath the bifurcation of the iliac vessels .E

ANSWER:C

2. Regarding gastric acid secretion, which is INCORRECT?

Cephalic phase of acid secretion is vagal dependent .A


Cephalic phase accounts for 60% of total acid secretion .BGastric
phase begins when food reaches the stomach .C Gastrin is the
main component of the gastric phase .D
Gastrin, histamine and acetylcholine stimulate parietal cells to secrete HCl .E

ANSWER:B

3. Which of the following concerning brachial plexus is CORRECT?

Long thoracic nerve originates from C6,C7 and C8 .ARadial


nerve arises from the medial cord of the plexus .B Median nerve
arises from the posterior cord of the plexus .CThoracodorsal nerve
arises from the medial cord .D
Musculocutaneous nerve arises from the lateral cord .E

ANSWER:E

4. Regarding the effect of aspirin on platelets, which is CORRECT?

Aspirin irreversibly induces thrombocytopenia .AEffect


of Aspirin lasts for 21 days .B
As furosemide, it inhibits ADP dependent platelets aggregation .C
Aspirin irreversibly inhibits platelets COX .D
Bleeding time is not affected .E

ANSWER:D

5. Massive blood transfusion, which is INCORRECT:

Hypercalcemia .A
Hyperkalemia .B
Hypothermia .C Platelets
dysfunction .D
Fibrinolysis .E

ANSWER:A

Dr. wessam alzaidat - General surgery


6. A 64-year-old woman undergoing radical hysterectomy under general anesthesia is transfused with 2U of
packed RBCs. A hemolytic transfusion reaction during anesthesia will be characterized by which of
the following?

Shaking chills and muscle spasms .A


Fever and oliguria .B
Hyperpyrexia and hypotension .C
Tachycardia and cyanosis .D
Bleeding and hypotension .E

ANSWER:E

Dr. wessam alzaidat - General surgery


7. In a 12-year-old boy who sustained severe head injury caused by a fall from the third floor of a
building, the syndrome of diabetes insipidus is characterized by which of the following?

Low serum sodium .AHigh


urinary specific gravity or osmolality .B
High serum osmolality .C
Low urine output .D
Expanded extracellular fluid volume .E

ANSWER:C

8. Which of the followings statements regarding intraductal papillary mucinous neoplasms of the
pancreas is true

The tubular variety of carcinoma arising in an IPMN indicates a better prognosis than the colloid .A
type
Main-duct IPMN have greater malignant potential than do the branch duct type .B
IPMN is more common in men than in women .CCystic
lesions larger than 3 cm should undergo surgical resection .DBranch-duct
IPMN should undergo resection regardless of size .E

ANSWER:B

9. A 64-year-old male is to undergo an elective laparotomy procedure. The proposed wound is


considered as “clean-contaminated.” This term implies an infection rate of which of the following?

1% .A
2% .B
9% .C
15% .D
30% .E

ANSWER:C

10. Galactorrhea, a milky discharge from the nipple in non-pregnant women, is most likely to be
associated with which of the following?

Fibroadenoma .A
Tubular adenoma .B
Pituitary adenoma .C
Hyperparathyroidism .DBreast
abscess .E

ANSWER:C

11. A 50-year-old man is involved in a major motor vehicle collision and suffers multiple trauma. He is
admitted to the intensive care unit. After 2 days of hospital admission he bleeds massively from the
stomach. What is the probable cause?

Dr. wessam alzaidat - General surgery


Gastric ulcer .A
Duodenal ulcer .B Hiatal
hernia .C Mallory-Weiss tear
.D
Erosive gastritis .E

ANSWER:E

Dr. wessam alzaidat - General surgery


12. The best option for repair of a blunt popliteal artery injury is the use of

The ipsilateral greater saphenous vein .A


The contralateral greater saphenous vein .B
A polytetrafluroethylene graft .C
Darcon graft .DA
basilic vein .E

ANSWER:B

13. Which of the following treatments is most appropriate for central diabetes insipidus

Normal saline infusion .A


Vasopressin .B
Fluid rerstriction .C
Demeclocycline .D
Hydrocortisone .E

ANSWER:B

14. The most common finding related to a cranial nerve injury after carotid endarterectomy consist of

Hoarseness .A
Tongue deviation .B
Lower lip drooping .C
Impaired swallowing .D
Shoulder droop .E

ANSWER:B

15. Which of the following statements is true regarding the pathophysiology of peptic ulcer disease

High acid secretion is associated with type 4 gastric ulcers .AType


2 peptic ulcers are unlikely to be associated with helicobacter pylori .B
Gastrin hyper secretion leads to increased parietal cells production of acid .C
Non-steroidal anti-inflammatory drugs increased acid secretion .DMost
acid is produced in the antrum of the stomach .E

ANSWER:C

16. Which of the followings is a result of sleeve gastrectomy compared with roux-en-y gastric bypass for
treating morbid obesity

Lower leak rate .A


Equivalent rate of type 2 diabetes mellitus resolution .B
Greater durability after 5 years .C
Equivalent excess weight loss after 2 years .D Lower
incidence of reflux postoperatively .E
ANSWER:B

17. In preparation for a pancreaticdounectomy , which of the following variants of hepatic artery
anatomy is most likely to be identified during CT scan imaging
Dr. wessam alzaidat - General surgery
Common hepatic artery arising from the left gastric artery .A Left
hepatic artery arising from the superior mesenteric artery .B
Right hepatic artery arising from the superior mesenteric artery .C
Left hepatic artery arising from gastrodoudenal artery .DRight
hepatic artery arising from the aorta .E

ANSWER:C

Dr. wessam alzaidat - General surgery


18. Which of the following statements is true regarding necrotizing pancreatitis

Contrast enhanced CT is the imaging modality of choice to determine the extent of necrosis .A
Enteral nutrition should be avoided for 10-14 days .BBroad
–spectrum antibiotics should be given .C
Pancreatic necrosis mandates immediate surgical intervention .D
Minimally invasive and percutaneous approaches to pancreatic necrosis is contraindicated .E

ANSWER:A

19. A 24 YEAR old man sustains stab wound along the left costal margin in the midclavicular line. He is
thermodynamically normal .a chest x-ray is unremarkable, and his focused assessment with sonographyfor
trauma exam is normal. Which of the following is the next step in his management

Observation and repeat CXR in 6 hours .A


Diagnostic peritoneal lavage .B
CT angiogram of the chest and abdomen .C
Diagnostic laparoscopy .D
Laparotomy .E

ANSWER:D

20. The interstitial fluid is generally poor while the plasma is generally rich in

Hydrogen ions .A
Sodium ions .B
Protein .C
Carbohydrates .D
Chloride Ions .E

ANSWER:C

21. Twenty-four hours after colon resection, urine output in a 70-year-old man is 10 mL/h. Blood
chemistry analysis reveals sodium, 138 mEq/L; potassium, 6 mEq/L; chloride, 100 mEq/L; bicarbonate, 14
mEq/L. His metabolic abnormality is characterized by which of the following?

Abdominal distension .A
Peaked T waves .B
Narrow QRS complex .C
Cardiac arrest in systole .D J
wave or Osborne wave .E

ANSWER:B

22. A 60-year-old woman who underwent a mastectomy for breast cancer 2 years earlier presents tothe
emergency department with headache, backache, and frequent vomiting. She is extremely thirsty
and stuporous. Which test is most likely to identify the cause?

Dr. wessam alzaidat - General surgery


CT scan of the head .AX-ray
of spine .B
Serum sodium determination .C
Serum calcium determination .D
Serum glucose determination .E

ANSWER:D

Dr. wessam alzaidat - General surgery


23. Which neoplasm is not characteristically associated with SIADH?

Mesothelioma .A
Bronchogenic carcinoma .B
Breast .C
Prostate .D
Bladder .E

ANSWER:C

24. Which is not true with regards to hypomagnesaemia?

it is nearly always seen in renal failure .A


it causes nausea, vomiting, loss of deep tendon reflexes, drowsiness and hypotension .B
it causes hypocalcaemia .Cif an
overdose is taken, Calcium should be given as it is direct antagonist of magnesium .D
it causes widening of the QRS .E

ANSWER:A

25. Before excision of a thyroglossal duct cyst, which of the following tests is recommended?

Fine needle aspiration .A


radioiodine scanning .B
MRI .C
Laryngoscope .D
Neck ultrasound .E

ANSWER:E

26. A 42 year old woman presents with flushing, nausea, vomiting, and weight loss, upper endoscopy
shows a near obstructing tumor of the antrum, biopsies are positive for carcinoid tumor, a CT scan shows
several lesions in both lobes of the liver consistent with metastatic disease .which of the flowing
surgical procedures should be planned?

Total gastrectomy with regional lymphadenectomy .ATotal


gatrectomy with resection of the hepatic metastasis .B
Antrectomy with gastrodoudenostomy .C
Antrectomy with cholecystectomy .D
Wedge resection with vagotomy .E

ANSWER:D

DR.WBZ

27. A 56 YEAR OLD man with child-Pugh B cirrhosis with hepatitis C is found to have 6 cm mass in
segment two and three with arterial enhancement and venous washout on CT scan, his alpha- fetoprotein
Dr. wessam alzaidat - General surgery
level is 450 ng/ml, an indocyanine green clearance is obtained to assess his suitability forresection .the
minimal percentage clearance at 15 minutes after injection of ICG needed to proceed
with surgery is

50% .A
60% .B
70% .C
80% .D
90% .E

ANSWER:D

Dr. wessam alzaidat - General surgery


28. During a modified radical neck dissection, the spinal accessory nerve is visualized

Cephalad to the greater auricular nerve on posterior surface of the sternocleidomastoid muscle .A
Anterior to the carotid sheath at the level of the cricoid cartilage .BAt the
posterior junction of the omohyoid and SCM muscles. .C
Caudal to the great auricular nerve on the posterior surface of the SCM .D
Posterior to the carotid sheath at the level of the cricoid cartilage .E

ANSWER:A

29. A 24 Year old man presents with 4-months history of postprandial nausea and abdominal pain,
vomiting and 6-kg weight loss, a CT scan obtained and showed SMA syndrome ,despite conservative
treatment the patient does not improve .after rehydration and electrolytes correction ,surgical
management should be

Release of ligament of treitz .A


Gastrojejunostomy .B
Doudenojejunostomy .C
Resection of the third and fourth portions of duodenum and doudenojejunostomy .D
Whipple procedure .E

ANSWER:C

30. Regarding Von-Willbrand disease, which of the following is INCORRECT?

VW disease is the most common congenital bleeding disorder .A


Is classified into three types .B
Type I does not respond to desmopressin therapy .C
Type II may respond to desmopressin therapy .D
Menorrahagia is common in female with VW disease .E

ANSWER:C

31. Which of the following pharmacologic interventions has not been shown to benefit patients with
acute variceal hemorrhage?

High-dose proton pump inhibitor .A


Somatostatin .B
Octreotide .C
Broad-spectrum antibiotic prophylaxis .D
Terlipressin .E

ANSWER:A

Dr. wessam alzaidat - General surgery


32. For which of the following patients is observation alone appropriate

A 53-year-old healthy man with a 8-cm nonfunctioning left adrenal mass .AA 46-
year-old man with a history of hypertension and unexplained hypokalemia. .B
A 32-year-old woman with elevated serum metanephrines and urinary VMA, metanephrines, and .C
an asymptomatic 2-cm right adrenal mass.
A 66-year-old man with a history of malignant melanoma on the leg at age 50, who presents with .D
a newly diagnosed 4-cm right adrenal mass.
A 44-year-old woman with an 8-cm left adrenal mass that appears to be a myolipoma based on .E
CT.

ANSWER:E

Dr. wessam alzaidat - General surgery


33. Which of the following is associated with the greatest risk of developing breast cancer in a woman?

Age greater than 40 .AFirst-


degree relative with breast cancer .B
Prior benign breast biopsy .C
Mutation of the BRCA1 gene .D
History of LCIS on biopsy .E

ANSWER:D

34. A 24-year-old medical student reports that she has noted numbness and tingling involving the fifth
finger. Which of the following nerve is most likely the site of irritation?

Median nerve .ARadial


nerve .B
Ulnar nerve .C
Lateral cutaneous nerve .DLong
thoracic nerve .E

ANSWER:C

35. Which of the following anatomic distributions is most common for Crohn disease?

Stomach and duodenum .A


Colon .B
Ileum .C
Esophagus .D
Terminal ileum and right colon .E

ANSWER:E

36. Which of the following factors contributes to polymicrobial diabetic foot infections?

Hyperglycemia .A
Patient age .B Age of
ulcer .C
Prior antibiotics treatment .D
Neuropathic ulcers .E

ANSWER:D

37. Which of the following is the most common cause of gastrointestinal tract fistulas?

Appendicitis .A
Dr. wessam alzaidat - General surgery
Ulcerative colitis .BCrohn
disease .C
Diverticulitis .D
Colon cancer .E

ANSWER:D

38. Which of the following is true of Hesselbach’s triangle?

This was named after a German actor. .A


The borders are the lateral border of rectus muscle, inguinal ligament, and inferior epigastric .B
vessels.
Indirect inguinal hernias are found here. .C
Femoral hernias are found lateral to the triangle. .D
The borders are the linea Alba, femoral vessels, and inferior epigastric vessels. .E

ANSWER:B

Dr. wessam alzaidat - General surgery


39. Which of the following is true of chronic pain after inguinal hernia repairs?

The most common cause of this problem is hernia recurrence. .A


Chronic pain has become a rare occurrence with the introduction of mesh implantation during .B
repairs.
Young adults and women are the least susceptible to this problem. .C
Causes of this condition include hernia recurrence, mesh-related reactions, nerve irritation, and .D
infections.
This problem can be virtually eliminated when hernia repairs are performed by expert surgeons. .E

ANSWER:D

40. Which of the following is the most common metabolic complication of primary
hyperparathyroidism?

Kidney stones .A
Osteoporosis .B
Pancreatitis .C
Gout .D
Hyperthyroidism. .E

ANSWER:A

41. Which of the following is not a condition associated with splenectomy?

OPSS (overwhelming post-splenectomy sepsis) .A


Increased risk of bacterial infections .B
Howell-Jolly bodies and Heinz bodies seen on peripheral smear .C
Thrombocytosis .D
Leukocytopenia. .E

ANSWER:E

42. Impingement of which of the following nerve roots can cause decreased strength in big toe dorsi-
flexion?

L5 .A
L3 .B
L2 .C
S1 .D
L4 .E

ANSWER:A

43. In which segment of the intestine is ligation of the inferior mesenteric artery during aortic aneurysm
repair most likely to produce ischemia?

Transverse colon .ALow


rectum .B
Dr. wessam alzaidat - General surgery
Splenic flexure of the colon .C
Right colon .DDistal
small bowel .E

ANSWER:C

Dr. wessam alzaidat - General surgery


44. Which of the following is most accurate regarding patients who develop fever during the
postoperative period?

They should receive broad-spectrum antibiotics until the fevers resolve .AThey
require no specific treatments because fever is an expected host response to surgical stress. .BThe
patients require immediate re-operation to address the source of fever. .C
Thorough searches for fever sources are required. Presumptive antibiotics can be given if .D
patients exhibit physiologic signs of sepsis or if the patients are immunocompromised.
High doses of corticosteroids should be prescribed to blunt the physiologic responses to .E
infection.

ANSWER:D

45. Which of the following statements is most accurate regarding secondary peritonitis?

Treatment with appropriate antimicrobial regimen will successfully resolve this process. .A
Successful treatment cannot be accomplished with antimicrobial therapy alone. .B
Resection of the segment of the GI tract source producing the peritonitis will successfully address .C
this process.
Antimicrobial therapy is not useful for secondary peritonitis, and treatment will only lead to the .D
selection of resistant microbial species.
Fungal species are the most common pathogen isolated in secondary peritonitis. .E

ANSWER:B

46. With regards to cytokines, which of the following alternatives is correct?

IL-1 is secreted mainly by lymphocytes and mediates inflammation. .AIL-10


is involved in cell division and activation. .B
IL-8 is secreted by macrophages and promotes chemotaxis. .C
IL-2 is a major inhibitor of cell division. .DTNF-
alpha is produced by T cells and is associated with a rise of immature neutrophils in the blood .E
circulation.

ANSWER:C

47. Amino acids are absorbed through the small –intestinal mucosa and are:

Stored in moderate quantities in the liver until needed for protein synthesis. .A
Metabolized through cytochrome p-450. .B
Processed and transported by apolipoproteins. .C
Used for gluconeogenesis under some conditions. .D
Used directly for brain metabolic needs. .E

ANSWER:D

48. Which of the following statements is true concerning the role of antibiotics in wound care?

Systemic antibiotics are indicated for open wound. .ABacterial


resistance can occur with systemic antibiotic but not topical antibiotics. .B

Dr. wessam alzaidat - General surgery


An indication for systemic administration of antibiotics is a granulation tissue bacterial count in .C
excess 0f 105 organisms per gram of tissue on a quantitative analysis.
Silver sulfadiazine is useful only for the management of burns. .D
Topical ointment should be avoided to minimize wound infection. .E

ANSWER:C

Dr. wessam alzaidat - General surgery


49. Activation of the classical complement pathway is caused by which of the following stimuli?

Mannose. .A
Microbial cell surfaces. .B
Interleukin-2. .C
Antigen-antibody complexes. .D
Plasma lectin. .E

ANSWER:D

50. Lymphatic metastases from the Right testis drain primarily into:

Inguinal nodes .AIliac


nodes .B
Inter-aorto-caval nodes .C
Obturator nodes .DPara
aortic nodes .E

ANSWER:C

51. Following admission to the emergency department, a 26-year-old woman with severe menorrhagia
states that both her father and sister have a bleeding disorder. The hemostatic disorder transmitted by
autosomal-dominant mode is which of the following?

Factor X deficiency .A
Von Willebrand’s disease .B
Factor VIII deficiency (true hemophilia) .CFactor
IX deficiency (Christmas disease) .DFactor V
deficiency (parahemophilia) .E

ANSWER:B

52. Above this level, spinal cord injury might results in neurogenic shock

T8 .A
C4 .B
T10 .C
C6 .D
T6 .E

ANSWER:E

53. Pancreatic acini secrete enzymes that play an important role in the digestion of proteins. What is
the most potent stimulant of pancreatic acinar cells?

Acetylcholine .A
Cholecystokinin (CCK) .B
Dr. wessam alzaidat - General surgery
Peptide YY .C
Secretin .D
Trypsinogen .E

ANSWER:B

Dr. wessam alzaidat - General surgery


54. Which one of the following statements about duodenal atresia and stenosis in the newborn is NOT
true?

Prenatal detection of duodenal atresia has remained constant over the past three decades. .A
A mucosal web with a normal muscular wall is the most common duodenal abnormality. .BThe
double-bubble sign on plain films is the classic x-Ray finding. .CDown’s
syndrome is identified in up to 25% of infants. .D
Cardiac abnormalities are commonly associated with duodenal atresia. .E

ANSWER:A

55. During the third trimester of pregnancy, the most common changes in renal function are:

Elevated BUN; decreased creatinine .A


Elevated BUN; elevated creatinine .B
Decreased BUN; decreased creatinine .C
Decreased BUN; elevated creatinine .D
Unchanged BUN and creatinine .E

ANSWER:C

56. A 45-year-old male with a known history of alcoholism is admitted with acute pancreatitis. His
serum calcium is 7 mg/dL. Management is based upon which of the following?

One-fourth of calcium in serum is ionized .A


Alkalosis increases the ionized calcium component .B
Hypocalcemia may cause polyuria and polydypsia .C
Determination of serum albumin is necessary .D
Treatment should involve intravenous administration of calcium chloride .E

ANSWER:D

57. Which of the following trace element deficiency is associated with glucose intolerance and
peripheral neuropathy?

Copper .AIron
.B Flourine .C
Chromium .D
Selenium .E

ANSWER:D

58. Which of the following statement is true about the role of macrophages in wound healing?

Macrophages are the dominant cell type during the inflammatory phase of wound healing. .A
Macrophages are not essential for wound healing. .BThe
macrophages role in wound healing is limited to phagocytosis. .C
Muscle cells are source of a number of humoral factors essential for wound healing. .D
Dr. wessam alzaidat - General surgery
Immunoglobulin G predominantly is produced by macrophages. .E

ANSWER:A

Dr. wessam alzaidat - General surgery


59. Product of platelet degranulation include:

Tumor necrosis factor-alpha. .A


Interlleukin-1 .B
Transforming growth factor-beta. .C
Insulin .DATP
.E

ANSWERC

60. Which of the following patients has the highest risk of developing colorectal cancer?

A 46-year-old man whose younger brother is just diagnosed with colon cancer. .A
A 46-year-old woman with BRCA1 mutation. .BA 46-
year-old man with 12-year history of ulcerative colitis. .C
A 46-year-old man who had two1-cm adenomatous polyps removed 1 year ago.4 .D
A 46-year-old man with familial adenomatous polyposis (FAP) syndrome. .E

ANSWER:E

61. A solitary lung nodule with a popcorn pattern of calcification is most likely

A primary lung cancer .AA


metastatic lesion .B
An old tuberculosis lesion .C
Histoplasmosis .D
Hamartoma. .E

ANSWER:E

62. Which of the following is not a risk factor for wound infection?

Prolonged operative time .A


Prolonged preoperative hospitalization .B Shaving
the skin the night before surgery .C
Patient’s having upper respiratory tract infection .D
Surgeon’s hand scrub for 5 instead of 10 minutes .E

ANSWER:E

63. An indication for '' damage control'' laparotomy include which of the following?

Disruption of the colon that cannot be repaired primarily .A


Use of type-specific blood transfusion. .B
Dr. wessam alzaidat - General surgery
Hyperglycemia. .C
Severe parenchymal injury that can be controlled with packing. .D
Anemia. .E

ANSWER:D

64. Which of the following is a recognized angiogenic factor?

Tumor necrosis factor .A


interleukin-2 .B
Vascular endothelial growth factor .C
interleukin-8 .D
Angiostatin. .E

ANSWER:C

Dr. wessam alzaidat - General surgery


65. The diagnosis of SIRS may include all of the following except:

Blood pressure 86/40 .A


Tempreture 35.6c .BHeart
rate of 103 beats/minute .C
Paco2 of 27mmHg .DWBC
of 15.5x10³/umm. .E

ANSWER:A

66. All of the following are mechanisms by which vasodilators improve cardiac function in acute
congestive heart failure except:

Increase stroke volume .ADecrease


ventricular filling pressure .B
Increase ventricular preload .C
Decrease end-diastolic volume .D
Decrease afterload .E

ANSWER:C

67. Which of the following statement is true concerning the surgical anatomy of the esophagus?

Surgical exposure of the cervical esophagus is best gained through the right side of the neck. .A
Spontaneous esophageal perforation tends to be associated with leakage into the chest. .B
Access to the entire thoracic esophagus can be obtained only through the left side of the chest. .C
The lower esophageal sphincter can be recognized distinctly by means of inspection of the .D
gastroesophageal junction.
The lower esophageal sphincter is a zone of low pressure 3-5cm long at the lower end of the .E
esophagus.

ANSWER:B

68. Regarding vitamins and wound healing which of the following is CORRECT:

Vitamin A potentiate the effect of steroids .A


Vitamin E has anti-inflammatory properties .B
Zinc deficiency has no effect in wound healing .C
Vitamin C is not essential for wound healing .D
Catabolic bio products of arginine can retard wound healing .E

ANSWER:B

69. The initial phase of wound healing is best characterized by only one of the following:

Begins 5-6 hours after the wound event .A


Bradykinin causes vasoconstriction, inhibiting the migration of neutrophils to the wound .B

Dr. wessam alzaidat - General surgery


Neutrophils trigger fibroblast response to start the following phase .C
Complement components together with platelet derived growth factor act as chemotactic .D
factors to stimulate migration of neutrophils to the wound
Monocytes differentiate into neutrophils 24 hours after the injury .E

ANSWER:D

Dr. wessam alzaidat - General surgery


70. A 43-year-old teacher underwent left parotidectomy. Upon awakening from surgery, paralysis of the
left lower lip was observed. This complication was most likely due to injury to which of the
following:

Parotid duct .AFacial


nerve - temporal branch .B
Facial nerve - cervical branch .C
Facial nerve - main trunk .D
Platysma muscle .E

ANSWER:C

71. One of the following is not reducing host resistance to infection?

Obesity. .A
Heart failure. .B
Steroid. .C
Malnutrition. .D
Uremia. .E

ANSWER:B

72. Who is forming the right superior sagittal fissure on the visceral surface of the liver?

Fossa of the falciform ligament .AFossa


of the round ligament .B
Gallbladder fossa .C
Fossa of the venous ligament .DFossa
of the inferior vena cava .E

ANSWER:C

73. Which of the following statements regarding the role of collagen in wound healing is TRUE?

Collagen synthesis in the initial phase of injury is the sole responsibility of endothelial cells. .A
Net collagen content increases for up to 6 week after injury. .B
At 3 weeks after injury, more than 50% of the tensile strength of the wound has been restored. .C
Tensile strength of the wound increases gradually for up to 2 years after injury; however, it .D
generally reaches a level of about 100% of that of uninjured tissue.
Tensile strength is the force necessary to reopen a wound .E

ANSWER:B

74. Which of the following statements regarding the pathogenesis of appendicitis is FALSE?

The anti-mesenteric border has the poorest blood supply and is usually the site of the perforation. .A
Fecaliths are commonly responsible for appendicitis in children. .B

Dr. wessam alzaidat - General surgery


Viral or bacterial infections can precede an episode of appendicitis. .C
Obstruction of venous outflow and then arterial inflow results in gangrene. .D Obstruction of
the lumen may occur as a result of inspissated stool or a foreign body. .E

ANSWER:B

Dr. wessam alzaidat - General surgery


75. All is true about cervical vertebrae EXCEPT.

Atlas (C1) has three ossification centers one for each lateral mass and one for vertebral body. .A
Atlas (C1) lateral masses fuse to body at age 7 years. .BAxis
(C2) develops from five ossification centers. .C
Axis (C2) subdental (basilar) synchondrosis does not fuse until 5 to 7 years of age. .D
Atlantoaxial (C1-2) articulation has a minimal role in neck rotation .E

ANSWER:E

76. In von Hipple- Lindau syndrome which of the following is incorrect.

It is Autosomal dominance with variable preponderance .A


Hemangioblastoma of the cerebellum is possible association .B
Kidney Tumor is against but cystic kidney goes with the diagnosis .C
Pancreatic and renal cysts may be present .D
Detailed eyes examination is very essential to rule out serious retinal angioma .E

ANSWER:C(ADRENAL MASS NOT KIDNEY)>>>>(VHL)


HAEMINGIOBLASTOMA CAN ASS WITH PHEOCHROMCYTOMA

77. The most common electrocardiogram (EKG) finding in patients with pulmonary emboli is.

Peaked T wave. .AAn S1-


Q3-T3 pattern .B
Rightward shift of the QRS axis. .CSinus
tachycardia (ST) and T wave changes. .D
Tachycardia .E

ANSWER:E

78. The following lab results are found in diabetes insipidus except.

Urine output > 250 ml/h. .A


Normal adrenal function. .B
Urine osmolality >200 mosm/h. .C
Normal levels of serum sodium or increased. .D
Specific gravity 1.001-1.005. .E

ANSWER:C

79. Extubation criteria for a ventilated patient include all except.

Patient on IMV or SIMV. .A


Peak inspiratory pressure < 40 mm Hg. .B
Pa O2 > 55 mm Hg. .C
Dr. wessam alzaidat - General surgery
pH > 7.35 .D
Respiratory rate < 30 per minute. .E

ANSWER:A

80. Possible side effects of steroids include all except.

Hypertension and hypokalemic alkalosis. .A


Psychosis. .B
Pancreatitis. .C
Hypoglycemia. .D
Cataract. .E

ANSWER:D

Dr. wessam alzaidat - General surgery


81. Concerning base skull fracture involving the anterior fossa all of the following are true except.

Epistaxis. .ACSF
rhinorrhea. .BBlindness.
.C Raccoon eyes. .D
Battle sign. .E

ANSWER:E

82. The absolute indication for surgical repair of depressed skull fracture is.

Overlying the sagittal sinus. .AOver


an eloquent area with no neurological deficits. .B
Compound dirty fractures. .C
Sharp angle of fracture. .D
Underlying thin non-compressing sub dural hematoma. .E

ANSWER:C

83. Which of the followings regarding bariatric procedures is false?

Gastric band is a restrictive procedure .ARYGB


is restrictive and malabsorptive procedure .B
Sleeve gastrectomy has the same effect of RYGB on resolution of hypertension .C
Iron deficiency is a life-long risk following RYGB .D
Sleeve gastrectomy has good effect on resolution of GERD in obese patients .E

ANSWER:E

84. Concerning bile salts, which of the following is correct?

Secondary bile salts are formed in the liver from cholesterol .APrimary
bile salts are absorbed in terminal ileum by passive diffusion .B
Primary bile salts are water soluble .C
Secondary bile salts are absorbed in colon by active absorption .D
Secondary bile salts are conjugated with glycine .E

ANSWER:C

85. Which of the following clinical conditions is indicated by the presence of serum antibodies against
hepatitis B surface antigen (anti-HBs) and serum anti-HBc with negative hepatitis B surface antigen?

Active, acute infection with hepatitis B virus .A


Normal response to vaccination with the hepatitis B vaccine .B
Chronic active hepatitis secondary to hepatitis B virus .C
Dr. wessam alzaidat - General surgery
Recovery following acute hepatitis B .D
Asymptomatic chronic carrier of hepatitis B virus .E

ANSWER:D

86. Regarding pancreatic anatomy, which of the following is correct?

The uncinate process is located anterior to the superior mesenteric artery .AThe
blood supply of the pancreas is partly derived from the inferior mesenteric artery .B
The body of the pancreas lies to the right of the SMA .C
In 60% of cases, the ducts of Wirsung and Santorini empty into the duodenum independently .D
The splenic artery usually runs posterior to the pancreatic body .E

ANSWER:D

Dr. wessam alzaidat - General surgery


87. A 46-year-old woman presents with a solitary 1.5 cm breast mass. A needle biopsy is consist with the diagnosis
of sclerosing adenosis. Which of the following is true:

It is most common in women younger than 45 year old .A


This findings increases the risk of developing breast cancer .B
Is not associated with hormone replacement therapy .CIt is
associated with an increased BMI .D
This lesion can be followed with ultrasound .E

ANSWER:B

89. A 40 year old man presents with a thyroid mass, physical examination also reveals raised lesions on the
tongue and a large head circumference, family history is positive for thyroid problems in multiple relatives, as wellas
breast, uterine and colon cancer, FNA of the thyroid mass reveals malignant papillary cells, the most likely
inherited syndrome is

Multiple endocrine neoplasia type 2 .A


Carney complex .B Von
Hipple- Lindau syndrome .C Familial
adenomatous polyposis .D
Cowden syndrome .E

ANSWER:E

90. During a modified Radical neck dissection, the spinal accessory nerve is visualized:

Cephalad to the greater auricular nerve on posterior surface of the sternocleidomastoid muscle .A
Anterior to the carotid sheath at the level of the carotid cartilage .BAt
the posterior junction of the omohyoid and the sternocleidomastoid muscle .C
Caudal to the greater auricular nerve on posterior surface of the sternocleidomastoid muscle .D
Posterior to the carotid sheath at the level of the carotid cartilage .E

ANSWER:A

91. The most common site of an ectopic parathyroid gland in a patient with persistent or recurrent
hyperparathyroidism is:

Paraesophageal .A
Mediastinal .B
Intrathymic .C
Intrathyroidal .D
In the carotid sheath .E

ANSWER:D

DR.WBZ
92. Which of the following is the best parameter for monitoring septic shock?
Central venous pressure (CVP) .A
Urine output .B
Serum lactate .C
Mental status changes .D
Vasopressor requirement .E

ANSWER:C
93. Hypothyroidism can be associated with all of the following pharmacologic therapies except:

Lithium .A
Amiodarone .B Interleukin-2

Dr. wessam alzaidat - General surgery


.C
Propylthiouracil (PTU) .D
Cimetidine .E

ANSWER:E

Dr. wessam alzaidat - General surgery


94. A 12 year-old boy is seen with gynecomastia. His breast size has been enlarging for 2 months. His parents are
concerned about cancer. The best step in management at this time would be:

Tamoxifen .ABreast
ultrasound .BMammogram
.C
Danazol .D
Observation .E

ANSWER:E

95. Dopamine is frequently used drug in critically ill patients because

At high doses it increases splanchnic flow .A


At high doses it increases coronary flow .B
At low doses it decreases heart rate .CAt
low doses it lowers peripheral resistance .DIt inhibits
catecholamine release .E

ANSWER:B

96. Which of the following statement is true concerning the postoperative fluid management in a surgical patient?

Intravenous fluid should be administrated to maintain a urine output of 0.1 mL/Kg/hour .AUrine
output of less than 0.5 ml/kg/hour is associated with increased hospital day .B
Decreased urine output is a physiologic response to surgical stress .C
Fluid overload reduces postoperative morbidity .D
Enhanced recovery after surgery pathways improve outcomes by fluid loading .E

ANSWER:C

97. A 35-year-old thin man quadriplegic has a chronic tracheostomy and is hospitalized for sepsis, he developed
ventilator associated pneumonia and abrupt intratracheal bleeding. Which of the following would be an
appropriate initial step in the management of this problem?

Removal of the tracheostomy tube and digital compression. .A


Overinflating of the balloon cuff on the tracheostomy. .B
Replacement of tracheostomy with an endotracheal tube. .C
Transport to the operating room immediately .D
Perform CT- angiography scan of the neck immediately .E

ANSWER:B

98. Which of the following is false regarding the factor V Leiden mutation?

Both heterozygous and homozygous mutations are associated with increased rates of VTE. .ACarriers
with provoked DVTs are generally managed with the same therapeutic regimen as non-carriers. .B
The hypercoagulable state associated with the mutation is readily detectable on standard clinical .C
coagulation assays as a shortened prothrombin time (PT).
Although the structural mutation is on factor V, it impacts the functional activity of activated protein C. .D
Roughly 5% of the American population is heterozygous, with the highest prevalence in Caucasians. .E

ANSWER:C

99. Seven years after her initial anti reflex surgery ARS, a patient undergoes a reoperation for recurrence of
symptoms. During the reoperation, what is the most likely finding?

Dr. wessam alzaidat - General surgery


Disrupted wrap .ALoose
wrap .B
Herniated wrap .C
Slipped wrap .D
Stricture .E

ANSWER:C

Dr. wessam alzaidat - General surgery


100. Which is the following statement is true regarding marginal ulceration after gastric bypass?

Ulcers most commonly present as painless upper GI bleeding .A


Smoking is a common risk factor .B
Ninety present of marginal ulceration after gastric bypass are caused by helicobacter pylori .CNon
absorbable suture use for the gastrojejunostomy will reduce the risk of marginal ulceration after .D
gastric bypass
The majority of patient presents within 3 months of index gastric bypass .E

ANSWER:B

101. 70 year-old man with cecal carcinoma undergoing right hemicolectomy through transverse incision. The
procedure is difficult and the incision extended medially by divided rectus sheath. Brisk arterial hemorrhage
ensues. From which of the damaged vessel originate

External iliac artery .A


Common iliac artery .B
Femoral artery .C Internal iliac
artery .D Superior vesical artery
.E

ANSWER:A

102. A patient undergoes a contrast radiograph of the small bowel for evaluation of intestinal obstruction. The
average transit time from the duodenum to the cecum is

30 min .A
1 h .B
3 h .C
5 h .D
7 h .E

ANSWER:B

103. A 38-year-old woman presents with RUQ. Imaging with CT shows a 3-cm mass in segment 7 of the liver that iswell
demarcated with a central fibrotic area. This central scar does not enhance in the arterial phase of the CT scan. Workup
shows no evidence of cirrhosis, hepatitis and a normal AFP level, and slightly elevated neurotensin.
What is the diagnosis?

FNH .A
Hepatic adenoma .B
Fibrolamellar carcinoma .C
Hepatoma .D
Hemangioma .E

ANSWER:C

104. A 59-year-old female presents to the ED with intermittent nausea, early satiety and bloating. She has a 12 year
history of type 2 diabetes mellitus, she underwent laparoscopic cholecystectomy 6 year ago, and a clinical diagnosis
of gastroparesis is made. Which of the following long-term treatment strategies should be initiated?

Domperidone .A
Dr. wessam alzaidat - General surgery
Metoclopramide .B
Erythromycin .C
Small low-fat, frequent meals per day .D
An implantable neurostimulation .E

ANSWER:D

Dr. wessam alzaidat - General surgery


Dr. wessam alzaidat - General surgery
105. A 42 year-old women presents with flushing, nausea, vomiting and weight loss. Upper endoscopy shows anear
obstructing tumor of the antrum. Biopsies are positive for carcinoid tumor. CT scan shows several lesions in

Dr. wessam alzaidat - General surgery


both lobes of the liver consistent with metastatic disease. Which of the following surgical procedure should be
planned.

Total gastrectomy with regional lymphadenectomy .ATotal


gastrectomy with resection of the hepatic metastases .B
Antrectomy with gastroduodenostomy .C
Antrectomy with cholecystectomy .D
Wedge resection with vagotomy .E

ANSWER:D ‫مكرر‬

106. All of the following regarding pancreatic polypeptide (PP) are true. Except

Proximal pancreaticomy is associated with grater PP deficiency then distal .A


Patient with PP producing tumors present with severe hypoglycemia .B
PP regulates expression of the hepatic insulin receptor gene .CPP
deficiency correlates with severity of chronic pancreatitis .D
It is secreted by F cells .E

ANSWER:B

107. Which of the following conditions is associated with hypernatremia?

Adrenal insufficiency .A Tumor


lysis syndrome .B Marked
hyperglycemia .C
Stevens–Johnson syndrome .D
Excessive loop diuretic administration .E

ANSWER:D

108. In the workup on a patient for possible appendicitis, CT scanning should be performed:

Before consulting the surgeon, by the emergency physician .A


In patients with equivocal physical findings .B
Routinely, in all patients with right lower quadrant pain .CWith
equal frequency in men and women .D
Never .E

ANSWER:B

109. Overwhelming post-splenectomy infection (OPSI)

Occurs more frequently after resection for trauma than hematologic disease .A
Occurs with equal frequency in children and adults .B
Is most frequently caused by Streptococcus pneumoniae .C
Usually occurs within 2 years after splenectomy .D
Generally has an identifiable site of infection .E

ANSWER:C

DR.WBZ
110. Radiation enteritis

Usually presents with perforation .AIs


caused by thrombosis of mucosal vessels .B Occurs

Dr. wessam alzaidat - General surgery


after 3,000 cGy of abdominal radiation .C
Routinely requires operative therapy .D
Is likely in patients who have undergone laparotomy .E

ANSWER:E

111. In surgical patients, adrenal insufficiency, which of the following statements is true?

Is generally due to pituitary failure .A


Presents with unexplained perioperative hypotension .B
Cause hypokalemia .CCan be
successfully treated with prednisolone alone .DResponds to
catecholamine infusion .E

ANSWER:B

Dr. wessam alzaidat - General surgery


112. With regard to tuberculous enteritis, which of the following statements is incorrect?

The common site is the cervical lymph node .A


Systemic symptoms precede cervical lymph node enlargement .BAn
abnormal chest X-ray is common in patients with cervical TB lymphadenitis .C
If FNA biopsy is non-diagnostic, incisional biopsy is preferred over excisional biopsy .D
TB lymphadenitis is not contagious .E

ANSWER:A

113. All of the following statements about carcinoma of the gallbladder are correct. Except :

The neoplasm usually starts in the cystic duct and neck of the gallbladder. .A
It is found more commonly in women than men. .BIt is
associated with the presence of gallstones in > 85% of cases. .C
Prognosis is generally poor with < 1 year survival with local invasion. .D
Chemotherapy and radiotherapy do not alter disease progression. .E

ANSWER:A

114. The most reliable measurement for diagnosing malnutrition is a

Serum albumin of 2.6 g/mL .ARatio of


exchangeable sodium (Na) to exchangeable potassium (K) of 0.9 .B
Respiratory quotient on indirect calorimetry of 1.0 .C
Daily calorie intake that meets < 40% of total energy needs .D
Loss of 7% of total body weight .E

ANSWER:D

115. Wound dehiscence

Occurs after 6% of laparotomies .AIs most


commonly due to suture breakage .B
Increases in frequency with advancing age .C
Is not increased by corticosteroid therapy .DIs
unrelated to wound infection .E

ANSWER:C

116. The hemodynamic value characteristic of septic shock is

Cardiac index, 2.8 L/min/m2 .A


Systemic vascular resistance index, 350 dynes cm-5 sec/m2 .B
Oxygen consumption, 135 mL/min/m2 .C
Oxygen delivery, 700 mL/min/m2 .D
Pulmonary capillary wedge pressure, 6.0 mm Hg .E

ANSWER:B

DR.WBZ
117. Decompression for abdominal compartment syndrome should be performed

If the urine output falls to 30 mL/hour .AWhen


the patient's respiratory rate increases to 24 breaths per minute .B
Based purely on physical findings .C
Dr. wessam alzaidat - General surgery
When bladder pressure exceeds 35 mm Hg .D
If the patient becomes hypoxemic .E

ANSWER:D

118. In laparoscopic surgery, the vessel most likely to be punctured during trochar placement is the:

Aorta .A
Right common iliac artery .B
Vena cava .C Left
common iliac artery .D Right
common iliac vein .E

ANSWER:B

Dr. wessam alzaidat - General surgery


119. A walled-off pelvic abscess secondary to sigmoid diverticulitis is Hinchey stage

I .A
II .B
III .C
IV .D
V .E

ANSWER:B

120. About chronic pancreatitis all are true, except?

Parlington Rochelle operation is longitudanal Pancreaticojejunostomy .A


Whipple Pancreaticoduodenectomy removes 60% of pancreatic parenchyma and has 25% incidence of .B
Diabetes
Frey's operation is Duodenal Presereving Pancreatectomy .C
Beger's operation is Duodenal Preserving Pancreatectomy .DIs a
different disease entity from acute pancreatitis .E

ANSWER:B

121. Which of the following about Pancreatic Ascites is not true

Conservative treatment effective in only 1/4th of patients .A


ERCP should be done before surgery .B
It is exudative .C
Metaplastic cells are present .D
Pancreatic fluid has high amylase and high albumin .E

ANSWER:A

122. Tetanus all are true except?

Is due to an infection with a gram-negative spore forming rod .A


The organism produces a powerful exotoxin .BThe
toxin prevents the release of inhibitory neurotransmitter .C
Clostridium tetani is sensitive to penicillin .D
Risus sardonicus is the typical facial spasm .E

ANSWER:A

123. Regarding gas gangrene

It is due to Clostridium botulinum infection .A


Clostridial species are gram-negative spore forming anaerobes .B The
clinical features are due to the release of protein endotoxin .C
Gas is invariably present in the muscle compartments .D
antibiotics alone are the main stay of treatment .E

ANSWER:D

124. The initial goal of therapy for acute toxic cholangitis is to:

Dr. wessam alzaidat - General surgery


Prevent cholangiovenous reflux by decompressing the duct system. .A
Remove the obstructing stone, if one is present. .B
Alleviate jaundice and prevent permanent liver damage. .C
Prevent the development of gallstone pancreatitis. .D
Prepare the patient for urgent cholangiography. .E

ANSWER:A

Dr. wessam alzaidat - General surgery


125. Which of the following statements about chronic pancreatitis is/are correct?

Chronic pancreatitis is the inevitable result after repeated episodes of acute pancreatitis. .A
Patients with chronic pancreatitis commonly present with jaundice, pruritus, and fever. .B Mesenteric
angiography is useful in the evaluation of many patients with chronic pancreatitis. .CTotal
pancreatectomy usually offers the best outcome in patients with chronic pancreatitis. .D
For patients with disabling chronic pancreatitis and a dilated pancreatic duct with associated stricture .E
formation, a longitudinal pancreaticojejunostomy (Peustow procedure) is an appropriate surgical option.

ANSWER:E

126. Standard supportive measures for patients with mild pancreatitis includes which of the following:

Intravenous fluid and electrolyte therapy. .A


Withholding of analgesics to allow serial abdominal examinations. .B
Subcutaneous octreotide therapy. .C
Nasogastric decompression. .D
Prophylactic antibiotics. .E

ANSWER:A

127. A 65-year-old man presents with obstructive jaundice. The patient’s workup begins with a CT scan. Which of
the following statement(s) is NOT true concerning his diagnosis?

A CT scan demonstrating intrahepatic biliary obstruction with a decompressed gallbladder and a non- .Adilated
extrahepatic biliary tree will be consistent with a Klatskin tumor
The presence of biliary obstruction seen on the CT scan requires further evaluation with invasive .B
cholangiography either percutaneously or endoscopically
Percutaneous transhepatic cholangiography would be the preferred technique for a suspected proximal .C
cholangiocarcinoma in that it will better visualize the proximal extent of the tumor in the biliary tree
The placement of a transhepatic biliary catheter can prove useful in surgical management of proximal bile .D
duct cancers
There is little role for cholangiography in the evaluation of a patient with suspected cholangiocarcinoma .E

ANSWER:E

128. Which of the following is not considered a risk factor for gall bladder cancer?

Gall stones>3cm in size. .A


Multiple small gall stones. .B
Porcelain gall bladder .C
Biliary salmonella typhi infection. .DGall
bladder adenoma. .E

ANSWER:B

129. The following Nyhus classification of hernias is correct except for:

Recurrent direct inguinal hernia—Type IVa. .AIndirect


inguinal hernia with a normal internal inguinal ring—Type I. .B
Femoral hernia—Type IIIc. .C
Direct inguinal hernia—Type IIIa. .D
Indirect inguinal hernia with destruction of the transversalis fascia of Hesselbach's triangle—Type II. .E
ANSWER:E
130. Regarding Hailar Cholangiocarcinoma which is INCORRECT:

Typically presents as stricture of the proximal hepatic ducts .AOften

Dr. wessam alzaidat - General surgery


involving the periductal lymphatic with frequent LN metastases .B
10% of patients with Hailar Cholangiocarcinoma have Primary Sclerosing Cholangitis PSC .C
In setting of PSC, Hailar Cholangiocarcinoma is usually unicentric .D
Resection is the only chance for cure .E

ANSWER:D

Dr. wessam alzaidat - General surgery


131. A patient with an open wound inquiries about the best option to provide the best aesthetic result. One of the
following is TRUE:

Nothing can be done, continuing dressing changes and preventing wound dehydration will result in the best .A
result Applying
a vacuum dressing will almost result in a scar-less wound .B
Placing retention sutures is the best option .C
Placement of silver dressing will produce the smallest scar .D
Delayed primary closure or skin graft will be the best result .E

ANSWER:E

132. What is the most common form of renal sarcoma

Liposarcoma .A
Rhabdosarcoma .B
Fibrosarcoma .C
Leiomyosarcoma .D
Angiosarcoma .E

ANSWER:D

133. A ten year old boy has a perineal “butterfly" hematoma fallowing a straddle injury. The suggest rupture of
the:

Tunica albogenea .A
Corpus spongiosum .B
Corpus cavernosum .C
Posterior urethra .DColle's
fascia .E

ANSWER:B

134. A 21 year old male is brought to the trauma room after sustaining a superficial stab wound to the left
shoulder. He reports that he received his full series of shots as a child and received his last tetanus booster shot
when he was 15. What should he receive for his tetanus prophylaxis?

Amoxicillin 500-mg PO TID .A


Nothing .B
Tetanus immune globulin (TIG) 250 units IM .C
Tetanus toxoid (dT) 0.5 ml IM .D
Tetanus toxoid (dT) 0.5 ml IM and TIG 250 units IM .E

ANSWER:D

135. Which one of the following statements about duodenal atresia and stenosis in the newborn is NOT true?

Prenatal detection of duodenal atresia has remained constant over the past three decades. .A
A mucosal web with a normal muscular wall is the most common duodenal abnormality. .BThe
double-bubble sign on plain films is the classic x-Ray finding. .C
Down’s syndrome is identified in up to 25% of infants. .D
Cardiac abnormalities are commonly associated with duodenal atresia. .E
ANSWER:A

136. The most important factor in the development of ulcers in a spinal cord injury patient is:

Malnutrition. .ASeptic
Dr. wessam alzaidat - General surgery
episodes. .B
Anemia. .C
Local pressure. .D
Diminished sensation .E

ANSWER:D

Dr. wessam alzaidat - General surgery


137. Pancreatic acini secrete enzymes that play an important role in the digestion of proteins. What is the most
potent stimulant of pancreatic acinar cells?

Acetylcholine .A
Cholecystokinin (CCK) .B
Peptide YY .C
Secretin .D
Trypsinogen .E

ANSWER:B

138. Which among the following statements concerning primary hyperaldosteronism is INCORRECT?

It results from an adenoma of the adrenal cortex in the majority of cases .A


It leads to sodium retention .BIt
may result in hypertension .C
It may suppress plasma renin activity .D
It leads to metabolic acidosis .E

ANSWER:E

139. Calcium metabolism: All are true, except:

Its main function is neuromuscular stability .A


Asymptomatic hypercalcemia may occur with hypoproteinemia .B
Metastatic breast cancer can be a cause of hypercalcemia .C
Calcium is given routinely in massive blood transfusion .D
Corticosteroids decrease resorption of calcium from bone .E

ANSWER:D

140. Crystalloid solutions: All of the following types of intravenous fluids are crystalloids, except:

Normal saline .A
Dextran 70 .B
Hypertonic saline .C
Ringer lactate .D
5% dextrose water .E

ANSWER:B

141. Hypernatremia (postoperative): All are true, except:

May indicate a deficit of total body water .A


May be caused by a high protein intake .B
May diagnose extra cellular fluid volume depletion .CA
common cause is excessive extrarenal water loss .D
Replacement of lost water with isotonic salt solutions can produce hypernatremia .E

ANSWER:B

142. The injection site that is associated with the highest risk for local anesthetic systemic toxicity is

Intercostal. .A
Caudal. .B
Epidural. .C Brachial

Dr. wessam alzaidat - General surgery


plexus. .DSubcutaneous.
.E

ANSWER:A

143. O2 saturation in normal adult venous blood is

27% .A
40% .B
75% .C
90% .D
100% .E

ANSWER:C

Dr. wessam alzaidat - General surgery


144. Compensation to acute hemorrhage includes

Baroreceptor stretch. .A
Cerebral vasodilatation. .B
Decreased catecholamines release. .C
Decreased circulating angiotensin II. .D
Decreased aldosterone secretion. .E

ANSWER:B

145. Regarding cerebral blood flow, which one is correct?

Is 5% of the cardiac output .AIs 10%


of the cardiac output .B
Is 15% of the cardiac output .C
Is 20% of the cardiac output .DIs 25%
of the cardiac output .E

ANSWER:C

146. The ulnar nerve arises from

The lateral cord of the brachial plexus containing fibers from the C6 and C7 nerve roots. .A
The medial cord of the brachial plexus containing fibers from the C8 and T1 nerve roots. .B
The posterior cord of the brachial plexus containing fibers of the C5 and C6 nerve roots. .CThe
lateral trunk of the brachial plexus containing fibers from C7 through T1. .DThe C5
through C7 nerve roots immediately before the upper trunk .E

ANSWER:B

147. Which of the following describes the inheritance pattern of hemophilia A?

Autosomal dominant inheritance .A


Autosomal recessive inheritance .B
Sex-linked dominant .C
Sex-linked recessive .D
Sporadic .E

ANSWER:D

148. The mechanism of action of the fluoroquinolone class of antibiotics, such as ciprofloxacin is

Inhibiting of cell wall synthesis .A


Selectively targeting bacterial topoisomerases (inhibit DNA replication) .B
Inhibiting RNA synthesis .C
Inhibiting ribosomal translation .D
Poisoning cytochrome C system .E

ANSWER:B

DR.WBZ………..
149. Which of the following statements describes the effect of parathyroid hormone (PTH) on bone?

PTH enhances bone formation when administered intermittently. .A


PTH enhances bone formation when present continuously. .BPTH
results in bone resorption when administered intermittently. .C
Dr. wessam alzaidat - General surgery
PTH results in decreased osteoblast number and activity when administered intermittently. .DPTH
impairs osteoclast function when administered intermittently. .E

ANSWER:A

150. Concerning massive blood transfusion, all are true EXCEPT:

The term implies a single transfusion more than 2500 ml transfusion over 24 hours. .A
Warning of the blood decrease the incidence of cardiac arrest. .BIf
diffuse bleeding occurs platelet and coagulation screening should be done and deficits corrected by fresh .C
frozen plasma.
The physiologic sequences of cirtrate toxicity are very significant. .D
The increased potassium content of the stored blood does not provide clinical effect unless the patient has .E
severe oligurea.

ANSWER:D

151. A 6 month old boy was born with hypospadias. This condition is due to failure in the development of which
of the following?

Urogenital fold. .A
Mullerian system. .B
Genital tubercle. .C
Urachus. .D
Vitelline duct. .E

ANSWER:A

Dr. wessam alzaidat - General surgery


152. The pelvic floor is formed by all of the following muscles except

Pubococcygeus. .A
Coccygeus. .B
Piriformis. .C
Puborectalis. .D
Iliococcygeus. .E

ANSWER:C

153. With regard to breast carcinoma in men, which is TRUE?

It is detected most commonly in men 60- 70 years old .A


Gynecomastia is a risk factor .BIt is
associated with a mutation in BRCA 1 gene .C
The prognosis is worse stage for stage than women .D
Sentinel lymph node biopsy is contraindicated .E

ANSWER:A

154. Which of the following is NOT an indication for post mastectomy radiotherapy?

T3 tumors .A
Multicentric DCIS larger than 6 cm .B
Four or more positive axillary lymph nodes .C
Inflammatory breast cancer .D
Extranodal extension .E

ANSWER:B

155. As part of her workup for anemia, a 60 year old female undergoes esophagogastroduodenoscopy (EGD),
which reveals a 5-cm mass along the greater curvature of the stomach. Biopsy of the mass reveals spindle cells and
C-kit expression. A computed tomography (CT) scan shows no others abnormalities. The most appropriate
next step for treatment is:

Endoscopic submucosal resection .ATotal


gastrectomy with lymph node resection .B Treatment
with imatinib followed by total gastrectomy .C
Wedges resection of gastric mass followed by treatment with imatinib .D
Wedges resection of the gastric mass .E

ANSWER:D

Dr. wessam alzaidat - General surgery


156. Which of the following is the most likely cause of ischemic orchitis following inguinal hernia repair?

Ligation of testicular artery .A


Complete excision of a large scrotal hernia sac .B
Inadvertent torsion of the testicular cord during the repair .C
Reconstruction of an internal ring that cause compression of the testicular cord .D
Reconstruction of an external ring that cause compression of the testicular cord .E

ANSWER:B

157. A 56 years old man is admitted after recurrent episodes of confusion and loss of consciousness. His serum
glucose is 45 mg/dL, raising the suspicion of insulinoma. Which of the following tests are NOT necessary for
confirmation of the diagnosis?

Pancreas protocol abdominal computed tomography (CT) .A


Serum C peptide level .B
Serum insulin level .C Serum
proinsulin level .DSerum
sulfonylurea .E

ANSWER:A

158. A 16 year old female is admitted to the hospital after suffering a 36% 2nd and 3d degree burns to the faceand
torso after being involved in a house fire. On the 2nd day of hospitalization, her monitoring arterial blood gas
shows metabolic acidosis. What is most likely the cause of her lab abnormality?

Respiratory rate of 14 .A
Silver nitrate .BSilver
sulfadiazine .C
Mafenide acetate .D
Respiratory rate of 22 .E

ANSWER:D

159. A 4 year old girl is brought into the emergency department from preschool after being bitten by a classmate.
Culture of the wound taken prior to irrigation are most likely to reveal which bacteria:

Eikenella corrodens .A
Fusobacterium species .B
Pasteurella multocida .C
Prevotella species .D
Staphylococcus aureus .E

ANSWER:E

160. A 45 year old woman is undergoing colectomy for ulcerative colitis. She has not taken prednisone for 4
months (she was taking 20 mg/day).What is most appropriate for her perioperative steroid management?

No steroids .A
50 mg hydrocortisone intraoperative, then 20 mg prednisone daily with taper .B320
mg hydrocortisone intraoperative, then 20 mg prednisone daily with taper .C500
mg hydrocortisone intraoperative, then 20 mg prednisone daily with taper .DNo
intraoperative steroids,20 mg prednisone daily with taper .E

ANSWER:A

161. What is TRUE of basal cell carcinoma?

Dr. wessam alzaidat - General surgery


It may show a flat ulcer. .A
It may metastasize to lymph nodes. .BIt
may metastasize to remote skin areas. .C
It is found exclusively in the head and neck. .DIt is
best treated by topical 5-FU. .E

ANSWER:A

162. Strawberry gallbladder is a gross description given to:

Adenmyomatosis .A
Porcelain gallbladder .B
Cholesterolosis .C
Acalculous cholecystitis .D
Gangrenous cholecysitis .E

ANSWER:C

163. Which of the following is the most common anterior mediastinal mass?

Teratoma .A
Thymoma .B
Shwanoma .C
Esophageal cysts .D
Bronchogenic cysts .E

ANSWER:B

164. All of the following are true regarding scar prognosis, EXCEPT:-

A scar usually looks its worst bet 4weeks and 4 months after injury. .A
Suture materials will determine the final scar. .B
Suture placement and removal affect the final scar. .C
Immobilization is important in healing. .D
Bruising, infection and improper orientation affect the final scar. .E

ANSWER:B

Dr. wessam alzaidat - General surgery


165. A 44-year-old male presents with a single gunshot wound to the abdomen. He is hemodynamically stable. On
exploration, his injuries are found to be limited to four small bowel injuries 8 cm apart, each with destruction of 20%
of the bowel wall, and a through-and through injury to the ascending colon with destruction of 30% of the
bowel wall. How should these injuries be managed?

Resection and anastomosis of the small bowel injuries and primary repair of the colon injury .A
Primary repair of both the small bowel and colon injuries .B
Primary repair of the small bowel injuries, primary repair of the colon injury, and creation of a diverting .C
ileostomy
Resection of the small bowel injuries and exteriorization of the colon injury as a colostomy .D
Resection and anastomosis of all injuries .E

ANSWER:A

166. Cecal diverticula are different from sigmoid diverticula in that

Sigmoid diverticula are true diverticula while cecal diverticula are false diverticula. .ACecal
diverticulitis is usually distinguishable from cancer. .B
Cecal diverticula are considered congenital in origin. .C
Asymptomatic cecal diverticula found on barium enema or colonoscopy should be treated operatively .D
because of the high incidence of complications.
Feculent peritonitis from perforation of a cecal diverticulum may be treated with resection and .E

ANSWER:C

167. The main cause of death in burn patients is:

Shock .AAcid-
base imbalance .B
Electrolytes imbalance .C
Hypoxia .D
Infection .E

ANSWER:E

168. Which of the following statements about hypertrophic pyloric stenosis in the newborn is NOT true?

There is familial predisposition. .A


Hypochloremia and hypokalemia may be present. .B
The pylorus can frequently be palpated on physical examination. .C
Emesis on feeding is common in the early postoperative period. .D
It is best diagnosed by an upper gastrointestinal study .E

ANSWER:E

Dr. wessam alzaidat - General surgery


169. All the following muscles are supplied by the trigeminal nerve EXCEPT:

Tensor velipalatini. .ATensor


tympani. .B
Masseter. .C
Posterior belly of digastric. .D
Mylohyoid .E

ANSWER:D

170. Which of the following techniques is the most useful in evaluating invasion of the bladder wall by transitional
cell carcinoma?

IVU .A
Nephrestogram .B
CT .C
MRI .D
Cystourethrogram .E

ANSWER:D

171. Hypophosphatemia increases the risk of the development of each of the following, EXCEPT?

Respiratory failure. .ATissue


anoxia due to decreased hemoglobin .B
Encephalopathy .C
Hypothyroidism .D
Hemolysis .E

ANSWER:D

171. Placement of an inferior vena cava (IVC) filter in patients with proximal lower extremity deep-vein
thrombosis (DVT) is indicated in each of the following scenarios EXCEPT:

Known large esophageal varices .A


Pulmonary embolus despite therapeutic anticoagulation .BHigh-
risk sonographic appearance of the proximal DVT .C
Recurrent unprovoked DVT .D
Severe congestive heart failure .E

ANSWER:D

172. Eight hours after an infant exhibits excessive drooling and mild respiratory distress. An abdominal radiograph
shows complete lack of air in the gastrointestinal tract. Which is the most likely diagnosis?

Hirschsprungâs disease. .A
Tracheoesophageal fistula H type. .B
Pyloric atresia. .C
Choanal atresia. .D
Esophageal atresia without tracheoesophageal fistula .E

ANSWER:E

Dr. wessam alzaidat - General surgery


173. All of the following are true about surgical nutrition EXCEPT:

Measurements of visceral and somatic protein status are biochemical indices used to evaluate nutritional .A
Status
Albumin levels that are less than 3.5 g/dL have been correlated with poor surgical outcome. C. Owing to .B
its shorter half-life (8-9 d) and smaller body pool size, albumin makes a better nutritional marker of
visceral protein status than does transferin.
In the situation of maintenance, energy needs are estimated at 25-35 kcal/kg, and protein needs are .C
estimated at 1.2-1.9 g protein/kg Height
and weight measurements are probably the most important set of vital signs in nutritional .D
assessment.
Dr. wessam alzaidat - General surgery
Dr. wessam alzaidat - General surgery
173. In which of the following scenarios is surgical excision recommended if it is found on core needle biopsy?

Fibroadenoma .A
Sclerosing adenosis .B
Fat necrosis .C
Pseudoangiomatous stromal hyperplasia .D
Atypical ductal hyperplasia .E

ANSWER:E

175. The decreased insulin to glucagon ratio seen during simple starvation allows:

Increase lipogenesis .A
Increase lipolysis .B
Increase protein synthesis .C
Increase glycogen production .D
Decrease lipolysis .E

ANSWER:B

176. Which of the following treatments have been proven to reduce the duration of postoperative ileus in the
clinical setting?

Epidural local anesthetics .A


Postoperative nasogastric (NG) decompression of the GI tract .B
Aggressive pain control with intravenous narcotics .C
Erythromycin administration .D
Starting total parenteral nutrition immediately after surgery .E

ANSWER:A

177. For which of the following patients is non-operative management most appropriate?

A 65-year-old otherwise healthy woman with a 5.2 cm AAA .A


A 96-year-old man with severe dementia and a 6-cm AAA. The patient has been a long-term resident of a .B
chronic care facility due to inability to care for himself secondary to his dementia
A 60-year-old man with a 5.3 cm AAA who presents with unexplained back pain over the past 24 hours. No .C
other causes of back pain can be identified
A 63-year-old man with 5.4 cm AAA and new onset of painful ecchymosis in the tips of several toes in both .D
feet
A 63-year-old man with a known AAA that is under surveillance. His aneurysm has grown from 4.5 to 5.4 cm .E
over the past 6 months.

ANSWER:B
178. Which of the following describes an advantage of endoscopic banding over TIPS?

Banding is associated with lower rates of post-procedural encephalopathy .A


Banding is more effective in the treatment of hepatic vein thrombosis .B
Banding procedures can be repeated as needed .C
Banding procedures causes less perihepatic scarring in patients who have liver transplant .D
Banding is associated with lower rate of infectious complications. .E

ANSWER:A
179. Which of the following statements regarding adrenalectomy for a pheochromocytoma is most accurate?

It results in blood pressure improvement, but blood pressure rarely normalizes .A It


corrects hypertension only in patients with benign disease .B

Dr. wessam alzaidat - General surgery


It may lead to profound intraoperative hypotension .C
It should be reserved for patients with hypertension refractory to drug therapy .D
Preoperative management is not needed if laparoscopic surgery is planned. .E

ANSWER:C

180. Which of the following is the most appropriate for a patient suspected of having chronic anal fissure based
on clinical history?

Obtain a barium enema, followed by a colonoscopy .A


Anorectal examination under sedation, anoscopy, and proctoscopy .B
Anal biopsy, anoscopy in the office, and barium enema .C
Anorectal examination in the office with sedation, anal biopsy, fissurectomy .D
Prescribe stool bulking agents. .E

ANSWER:B

Dr. wessam alzaidat - General surgery


181. By which of the following modalities is the staging of thymomas primarily determined?

Surgical resection .A
Immunohistochemistry assay .B
MRI evaluation .CCT
scan evaluation .D
Mediastinoscopy .E

ANSWER:A

182. Which of the following statements regarding prophylactic mastectomy is most accurate?

Prophylactic mastectomy is an acceptable treatment option .A


Prophylactic mastectomy is rarely indicated because chemoprevention, diagnostic, and surveillance .B
strategies are sufficient to identify and treat cancers at a treatable stage
Unilateral mastectomy is preferable over bilateral mastectomies .C
Prophylactic mastectomy should only be done if the patient will be able to undergo immediate .D
reconstruction procedure
Prophylactic mastectomy is only indicated for women who fail chemoprevention. .E

ANSWER:A

183. Which of the following does not need to be instituted for a patient following carotid endarterectomy?

Aspirin .A
Glycemic control in diabetics .B
Warfarin to target an INR of 2 to 3 .C
Statin .D
Smoking cessation. .E

ANSWER:C

184. Which of the following patients has the greatest likelihood of developing carpal tunnel syndrome?

A 45-year-old woman with diabetes insipidus .A


A 45-year-old woman with hypothyroidism .B
A 45-year-old woman with Addisonâs disease .CA 45-
year-old woman with hypertension .D
E. A 45-year-old woman with fibromyalgia .E

ANSWER:B

185. Which of the following radiographic abnormalities seen on CXR is suggestive of traumatic rupture of the
aorta (TRA)?

Pneumomediastinum .ASternal
fracture .B
Enlarged cardiac silhouette .C
Widen mediastinum .D
Right-sided diaphragmatic rupture. .E

ANSWER:D

Dr. wessam alzaidat - General surgery


186. Which of the following is the most appropriate treatment for a 40-year-old man with a T3N1 carcinoma of
the ascending colon?

Preoperative chemoradiation therapy followed by right hemicolectomy .A


Right colectomy and postoperative adjuvant therapy with oxaliplatin, 5-FU, and leucovorin .B
Endoscopic removal of the tumor followed by chemoradiation therapy .C
hemicolectomy and postoperative raditation and tamoxifen therapy .D
Definitive treatment with six cycles of FOLFOX4 and remove the colon only if symptoms develop' .E

ANSWER:B

187. Which of the following are the most common indications for liver transplantation in adults and children,
respectively?

Autoimmune hepatitis and inborn errors of metabolism .A


Malignant neoplasms and bilia .B
Cholestatic cirrhosis and viral hepatitis .C
Non-cholestatic cirrhosis and biliary atresia .D
Fulminant liver failure .E

ANSWER:D

188. Which of the following statements regarding surgery for Crohn disease is true?
Repeat operations are needed for 25% of the patients who require one operation .A
Surgical resection often cures patients with Crohn disease .B
Medical refractory disease is the most common indication for surgical treatments .C
Surgical therapy rarely improves the patient’s quality of life .D
Surgical treatments should be avoided at all costs in this patient population. .E

ANSWER:C

189. A 69-year-old man presents with confusion, abdominal pain, shaking chills, a temperature of 34°C (95°F), and
jaundice. An abdominal radiograph demonstrates air in the biliary tree and normal bowel gas pattern. Which of
the following is the most likely diagnosis?

Acute pancreatitis .AAcute


cholecystitis .B Acute
viral hepatitis .C
Gallstone ileus .D
Acute cholangitis .E

ANSWER:E

190. Hemostatic resuscitation means giving PRBCs/FFP/platelets in what ratio:

1:2:1 .A
1:3:2 .B
1:1:1 .C
2:1:1 .D
3:2:1 .E

ANSWER:C

191. Regarding Fluid resuscitation, all of the following are true except

Both lactated Ringer's solution and Normal saline are considered isotpnic solution. .A
Hypertonic saline ( 7.5%) has been used as a treatment modality in patients with closed head injuries .B
Dextrans have been used, in association with hypertonic saline, to help maintain intravascular volume. .C
Dr. wessam alzaidat - General surgery
Hetastarch has a limited role in massive resuscitation because of the associated coagulopathy and .D
hyperchloremic acidosis.
Albumin it is typically available as either a 5% solution or 1% solution. .E

ANSWER:E

192. Which of the following ventilation modes is more likely to cause increased auto-PEEP?

Pressure support ventilation .AAirway


pressure release ventilation .BAssist
control ventilation .C
Pressure-controlled inverse ratio ventilation .D
Continuous positive pressure ventilation. .E

ANSWER:D

193. The formula for oxygen extraction ratio is:

DO2/VO2 .A
VO2/DO2 .B
1/VO2 .C
COx oxygen content .DMAP-
CVP/CO. .E

ANSWER:B

Dr. wessam alzaidat - General surgery


194. Cardiac output increase with all of the following except

An increase in stroke volume .AAn


increase in dp/dt .B An
increase in LVEDV. .C
An increase in pulmonary venous pressure. .D
An increase in aortic pressure. .E

ANSWER:E

195. Regarding Trials and research which is true?

Ethical committee approval is invariably required for audit projects .A


The Declaration of Helsinki deals with the safeguarding of patients undergoing clinical trials .B
Consent of patients is not needed if a clear benefit from the control treatment is known .C
the cohort study of subjects is the gold standard .DDouble
blinding means neither the doctor nor the statistician are aware of treatment groups. .E

ANSWER:B

196. What is the most common cause of lower gastrointestinal (GI) bleeding (LGIB) in children?

Anal fissure .A
Hemorrhoids .B
Henoch-Schonlein purpura .C
Food allergy .D
Meckel’s diverticulum .E

ANSWER:A

197. Which of the following events occurs in the proliferative phase of wound healing?

Histamine release .A
Collagen cross linking .B
Thromboxane release .C
Phagocytosis .D
Collagen synthesis .E

ANSWER:B

Dr. wessam alzaidat - General surgery


198. Which of the following would be expected to cause a decrease in extracellular fluid potassium concentration
(hypokalemia) at least in part by stimulating potassium uptake into the cells?

β-adrenergic blockade .A Insulin


deficiency .B Strenuous
exercise .C
Aldosterone deficiency (Addison’s disease) .D
Metabolic alkalosis .E

ANSWER:E

199. Which of the following is the most common cause of acute respiratory distress syndrome (ARDS)?

Sepsis .A
Near drowning .B
Multiple blood transfusions .C
Multiple blunt trauma .D
Pancreatitis .E

ANSWER:A

200. Which of the following is useful as a sensitive screening test for clinically significant complications of blunt
cardiac injury?

Creatine kinase, MB isoenzyme (CK-MB) .A


Troponin I .B
Troponin T .C
EKG .D
Exercise stress test .E

ANSWER:D

201. In a patient with peripheral vascular disease, the principle source of distal emboli is the:

Aorta .A
Femoral artery .B
Heart .C
Iliac artery .D
Popliteal artery .E

ANSWER:C

202. The manifestations of hyperphosphatemia are related to its effects on:

Sodium .A
Potassium .B
Magnesium .C
Calcium .D
Chloride .E

ANSWER:D

203. Which of the following is the most important factor in determining the risk of rupture of an abdominal aortic
aneurysm (AAA)?

Age of the patient .A


Hypertension .B
Dr. wessam alzaidat - General surgery
Size of the aneurysm .C
Location of the aneurysm .D
Male gender .E

ANSWER:C

204. Regarding urethral injuries, all are true except:

Prostatic hematoma may be present. .A


Perineal hematoma may be seen. .B
Fracture pelvis may be the cause. .C
Descending urethrogram is the investigation of choice. .D May be
treated with suprapubic urinary bladder catheterization. .E

ANSWER:D

205. Which of the following is a common cause of compartment syndrome?

Femur fracture .A
Metacarpal fracture .B
Radius fracture .C
Tibia fracture .D
Metatarsal fracture .E

ANSWER:D

206. All of the following are clinical pictures in a patient with hypovolmic shock. Except.

Increased heart rate .A


Wide pulse pressure .B
Decreased urine output .C
Flat neck vein .DPale
and cold extremities. .E

ANSWER:B

Dr. wessam alzaidat - General surgery


207. The most common cause of intestinal obstruction between 3 months and 6 years of age is

Cystic fibrosis. .A
Hirschsprung’s disease .B
Intussusceptions .C
Meckel’s diverticulum. .DNatal
adhesions .E

ANSWER:C

208. The majority of esophageal perforations are:

Spontaneous. .A
Iatrogenic. .B
Traumatic. .C
Caused by caustic ingestion. .D
Idiopathic .E

ANSWER:B

209. the most common histologic type of malignant tumor of the head and neck is

adenocarcinoma .A
Squamous cell carcinoma .B
Melanoma .C
Sarcoma .D
Follicular neoplasm .E

ANSWER:B

210. Regarding choledochal cysts, which of the following is correct?

The most common presentation in children is RUQ abdominal pain and fevers. .AThe
most common type of choledochal cyst is confined to the intrahepatic biliary tree. .BCaroli’s
disease is defined as dilatation of the intra- and extrahepatic biliary tree. .C
There are two histologic types of choledochal cyst: glandular and fibrotic. .D
They are benign and do not progress to cholangiocarcinoma. .E

ANSWER:D

Dr. wessam alzaidat - General surgery


211. What is the appropriate treatment for biliary dyskinesia?

Fat-free diet and decrease weight .AERCP


with sphincteroplasty .B
Cholecystectomy .C
Prokinetic agents .D
Nothing by mouth, antibiotics, and cholecystectomy .E

ANSWER:C

212. Which of the following statements concerning HCC diagnosis and management is true?

Enlarged portal lymph nodes are a contraindication to extirpative surgery unless proven benign by tissue .A
biopsy.
An image-guided biopsy is performed prior to treatment for primary HCC to avoid treating benign .B
regenerative nodules.
A CT scan can anatomically define an abnormal nodule but provides no information on hepatic function. .C
HCC will demonstrate greater enhancement than surrounding liver on the arterial phase of a four-phase .D
liver CT.
An MRI can sometimes provide information not provided with a CT scan and does not require intravenous .E
contrast.

ANSWER:D

213. Which of the following is true regarding focal nodular hyperplasia (FNH)?

It usually causes symptoms. .A


It can be distinguished from a hepatic adenoma on magnetic resonance imaging (MRI) with a proper .B
protocol.
It is associated with a risk of spontaneous rupture and bleeding. .CIt
undergoes malignant transformation in 10% of cases. .DIt is
associated with the use of oral contraceptives. .E

ANSWER:B

214. Which of the following is true regarding burn hypermetabolism?

The patient’s homeostatic thermostat remains unchanged. .AThere


are decreased levels of circulating catecholamines. .B Patients prefer to
set the ambient room temperature to 65°F. .C
There is protein catabolism in skeletal muscles .D
The excision of necrotic tissue and the treatment of infection may increase hypermetabolism. .E

ANSWER:D

215. All of the following are reasonable treatment options for metastatic esophageal adenocarcinoma except

Radiation therapy .A
Endoscopic stenting .B Gastrostomy
tube placement .C
Photodynamic therapy .D
Dr. wessam alzaidat - General surgery
Esophagectomy .E

ANSWER:E

216. A study is proposed in which 50 patients with avascular necrosis (AVN) of the Hip are compared with 20
patients without AVN in terms of their Steroid Drug intake dose. This is an example of what type of study?

Case–control .A
Cohort .BCross-
sectional .CRandomized
.D
Longitudinal .E

ANSWER:A

Dr. wessam alzaidat - General surgery


217. The physis with the highest growth rate (in mm per year) is located at:

Proximal humerus .A
Distal femur. .B
Distal tibia .CDistal
radius .D Distal humeus
.E

ANSWER:B

218. Concerning the vertebral column and the vertebra, one is true:

Cervical and lumbar lordoses are primary curves present at birth. .AThe
posterior column is formed by the posterior longitudinal ligament and the neural arch. .B
The pedicles fuse laterally to form the spinous processes. .C
Transverse process arises from the lateral aspect of the vertebral bodies. .D
The pars interarticularis is the part of the lamina between the superior and inferior articular facets. .E

ANSWER:E

219. Regarding the femoral nerve, one is true:

Derived from the anterior divisions of L2-L4. .A


Contained within the femoral sheath. .B
Has no branches above the inguinal ligament. .C
The nerve to Vastus Mediallis runs in the Adductor Canal. .D
Supplies sensation to the lateral aspect of the foot. .E

ANSWER:D

220. Regarding morphine, all true except:

Stimulates edinger wistphal nucleus .A


Causes constipation .B
Causes respiratory depression .C
Increases gastric emptying .D
May cause histamine release .E

ANSWER:D

Dr. wessam alzaidat - General surgery


JMC BOARD 8 2022

1: screening of patient with lynch

Upper endocscopy every 10 yrs and lower every 3 yrs

Starting form age of 35 colonscopy and upper every three years

2: critically ill patient best way of feeding

Nasoduodenal

Enteral

Jejunal

Parenteral

3: best treatment of pancreatic divisium

Write answer duodojejunostomy

4: most common nerve injuried in mandibular gland surgery

5: best nerve graft used is :

Sural nerve

6: question about blood vessel injured in abdominal trauma and which one you can’t ligate

7: pt presented with neck trauma zone 2 expanding hematoma how to treat

First intubate in er then explore in or

8: scenario abut treatment of thyroid papillary carcinoma 4 cm near midline with no lymph nodes involved

9: during tracheostomy procedure performed in 2,3,4 ring of trachea which arterial injury

10: cricothyroidectomy procedure

11: basal cell carcinoma in check fully excised what is next

Observation

Dr. wessam alzaidat - General surgery


Follow up

Reevaluate after one yr

Council

12: squamous cell carcinoma in the lower limb

Question was about treatment, biopsy ,lymph node mets and causes

13: melanoma

14: when you close cleft closure lip

15: parotid gland all except stones

16: blood supply of pancreas from splenic artery

17: which type of intraductal papilloma associated with increase risk of malignancy

18: rectal carcinoma located 10 cm from anal verge involving mesocolon the best treatment

19: fixed tumor located 4 cm from anus treatment option

20: endoscopic treatment acceptable for esophageal cancer

21: invasive ductal carcinoma with pregnancy first trimester what is the treatment

22: split skin graft versus full thickness graft

More infection rates

More resistant

23: contraindications of breast conservative surgery all except

Recurrent mass , irradiation ,small breast ,

24: branchial cyst fistula :

Bilateral

Open in the pharynx

Located in the lower third of sternocleidomastoid

25: pleomorphic adenoma features all except : capsulated

26:question about ileostomy

27: most common cause of increase morbidity in surgical patient

Uti, pulmonary, ssi

28: abdominal aortic aneurysm associated with what:

One of options is infected aneurysm

29: first sign of retroperitoneum fibrosis is ureteric entrapment

Dr. wessam alzaidat - General surgery


30: in necrotizing enterocolitis the fisrt sign indicated bolwel ischemia

31: treatment of uncomplicated meconium iles

32: hernia repair about procedure all true except

33: melanoma prognostic features: depth

34: most important prognostic factors in liver mets: depth, number, size, remaining liver not involved

35: bismuth carolitte classification

36: nyhus classification

37: skin lesion with jaundice: pyoderma gangrenosum

38: which organ least affected by hemorrghic shock: skeletal muscle, cardiac muscle,liver , kidney

39: negative intermittent pressure in all except ischemia

40: ileal or colon conduct used for urinary prosthesis the most common abnormality is :

Increase uric acid, hypochloremia metabolic acidosis

41: complex fistula management: seton

42: pt is diagnosed as case of ca head of pancreas he lost 15kg in the last 6 months which procedure is best
for pt : pylorus preserving pancreaticoduodenectomy

43: carcioid stomach , mass submucosal and cd117 the best treatment

44: d2 lymph node in stomach carcinoma :

Group 10 s splenic group

45: zenker diverticulum relation to which muscle

46: question about multiple myeloma and protrubrans

47: post-transplant cmv chest infection what is the treatment: geniclover , acyclovir

48: most common nerve injury in parotidectomy surgery : inferior auricular nerve

Cleft palate treatment-1

1mo -6mo-12mo-24mo

2-laparoscopic gas used all except:

Methan,hilum,carpon oxid,nitros oxid,argan

Dr. wessam alzaidat - General surgery


3-about tips all except

Decresed encephalopathy

4- pt trauma m.ribs # parodoxial movment no hemathorax ot pneumothorax stable next step

a-intubation and hihe presser

b-pain manegment with closed observation

5- all of the following proinflamtare except

6-4cm papillary thyroid carcinoma treatment

a-rt lopectomy

b-rt lopectomy+ lnd dissection

c-total thyroidectomy

d-total with central dissection

e- total with 4,2 leveal lymph node dissection

7-aboute glutamin

8-nhys classification hernia all except

9-bismuth classification of bile duct injury

10-pt jaundice and appearance erythema and itching:

pyoderma gangrenosum

11-trauma trachea injury 2-3-4 stap wound exposure intra operation:

a-post.wall trachea

b-imm aorta

12- petersen hernia

13-aboute op hernia type all true except

14-salevary gland

15-nerve graft:

Sural nerve

16_pediatric singe of lapratomy about entracolitis:

Erthyma abdominal wall

17-secand most common death after transfision blood:

a-infection

b_abo inco

c-lung inj (tral)

18- cryoprecipitate all contant highe except:

Dr. wessam alzaidat - General surgery


8,11,fiprogen..

19-gastrectomy best assessment:

a-d2 all surgery

b- 10 lymph node spleen art dissected

20-panceratic blood supply branches splenic arty all except

21-short half life :7

22-hyperphosphatemy all except

a-hyperparathyrodism

b-hyperthyroism

c-malignat hyperthermia

23-psedohypothyridism???

hyperlipademia...‫الجواب اعتقد كان‬

24-mots threating bleeding frome liver

a-branches vines

25-most noscomia infection death:

a-uti

b- pneumonia

c-abd abscess

26-aboute budd chiari syndrome first sing:

Aching -a
Jaundice -b
Abdominal pain -c
Esophegal varesis -d
27-aboute marjonlis ulcer burn:
28-basal cell carcinoma:
29-melanoma 2cm treatment
30-sarcoma
31-retropertonial sarcoma first singe:
Uretar obstraction
32-jajunim and ilum diff:
33-cross match transfison
34-fistula treatment anal condition
35-sponge correct all except:
Longe time operation
36-bariatric surgery:
a-Afferent syndrome
b-fistula
36- trauma colon through and through
Praimer repiear with drain -A

Dr. wessam alzaidat - General surgery


Praimer repaier with antbiotic 14 day -B

37-oncolgy :

Hnsp,lych syndrome

38-abdominal vascular trauma

39-ilum colon urine

40- burn critical ill for nutration

a-nasogastric

b-parentral

c-nesjujenim

41-pediartic burn t 40,wbc 5000:

Toxic syndrome shock -a

42- PUD pathophesiolgy


43-lung mass
30 day ‫ اقل من‬-a
Spuneclated -b

43- aboute laprascopic changes all increased except:

a-SVR

B-HR

C-CARDIC OUTPUT

44-about STSG AND FTSGs

45-screen colon cancer pt 21 y/s ass/w famile hx ca colon father:

46- pt CD117 +,MASS 1CM mangment:

47-warfarine ,inr 3

Dr. wessam alzaidat - General surgery


Board Questions 8-2022

1. Screening of patient with Lynch syndrome (hereditary non-polyposis colorectal cancer)


a. Upper endoscopy every 10 yrs and lower every 3 yrs
b. Starting form age of 35 colonoscopy and upper every three years
 Patients with known Lynch syndrome (HNPCC) - Colonoscopy starting at age 20-25 or 10 years
before primary relative got cancer, repeat at 1–2-year intervals, colon cancers are more
commonly right sided (as opposed to left sided in sporadic cancer), Upper endoscopy
screening is also recommended starting at age 50.
 Patients with a known family history of FAP (relatives who are APC positive) - begin flexible
sigmoidoscopy at puberty (at age 10 to 12 years) and perform annually until 20; since all
affected patients will have rectosigmoid polyps or until the colon and rectum are removed. If
negative at 20, perform colonoscopy every 10 years. Relatives who are APC mutation negative
can wait until age 50 for screening because they are considered to have the same risk as the
normal population.

2. Lynch syndrome genetic mutations  DNA mismatch repair gene (correct errors of DNA replication)
(hMSH2 60%, hMLH1 30%, MSH6 7-10%, hPMS1 <1% & PMS2)

3. screen colon cancer pt 21 y/s, ass/w family hx ca colon father:


 Patients with a 1st degree relative with CRC or polyps ≥ 60 years old (or two first- or second-degree
relatives of any age) - begin at age 40, or 10 years before the earliest diagnosed age, and repeat
at 10-year intervals
 Patients with a 1st degree relative with CRC or polyps ≤ 60 years old (or two first-degree relatives
of any age) - begin at age 40, or 10 years before the earliest diagnosed age, and repeat at 5-year
intervals

4. Burn - Critically ill patient best way of feeding


a. Nasoduodenal
b. Enteral
c. Jejunal
d. Parenteral
 Early enteral feeding (< 24-48 hours) reduces ICU stay and improves survival in critically
ill non-surgical patients on mechanical ventilation or multiple pressers (eg pancreatitis,
sepsis, ARDS, head injury, burns)

5. Most common nosocomial infections to cause death


a. UTI
b. Pneumonia *
c. Abdominal abscess
 Pneumonia was the most frequent nosocomial infection related to death.
 Most Common Cause of Nosocomial Infection – UTI
 M.C nosocomial infection causing death is RTI-pneumonia

6. Best treatment of Pancreas divisum

Dr. wessam alzaidat - General surgery


 If symptomatic - ERCP with sphincterotomy of minor papilla (Santorini duct opening),
open sphincteroplasty if that fails.

7. Annular pancreas  duodeno-jejunostomy (Pancreas is not resected)

8. Most common nerve injured in Submandibular gland surgery  marginal mandibular nerve
9. Most common nerve injury in parotidectomy surgery: inferior auricular nerve
10. Parotid gland all except stones
11. Salivary gland (You have three major types of salivary glands, including your sublingual,
submandibular and parotid)
 Submandibular gland dissection 
 MC nerve injury - marginal mandibular nerve (mental nerve comes off this)
 Sx's: droop at corner of mouth, chin numbness
 Parotid gland dissection 
 MC nerve injury - greater auricular nerve (numbness over lower ear)

12. All of the following are gases used to provide pneumoperitoneum in laparoscopic surgery
except?
a. Methane
b. Carbon dioxide
c. Helium
d. Nitrous oxide
e. Argon
 ANSWER: A (Methane is flammable)

13. Best nerve graft used is:


a. Sural nerve
 The sural nerve is the preferred graft because it is easily accessible and leaves the patient
with few side effects.

14. Question about blood vessel injured in abdominal trauma and which one you can’t ligate
 Celiac or SMA

15. Neck trauma zone 2, expanding hematoma how to treat


a. First intubate in ER then explore in OR ***

16. Treatment of thyroid papillary carcinoma 4 cm near midline with no lymph nodes involved
a. rt lobectomy
b. rt lobectomy+ LN dissection
c. total thyroidectomy
d. total with central dissection
e. total with 4,2 level lymph node dissection
131
 Tumor >1cm  Total thyroidectomy + post-op I
If with enlarged LNs  plus MRND

17. During tracheostomy procedure performed in 2,3,4 ring of trachea which arterial injury
a. Innominate artery

18. Basal cell carcinoma in cheek, fully excised what is next


a. Observation
b. Follow up

Dr. wessam alzaidat - General surgery


c. Reevaluate after one yr. ***
d. Council
 Surgical excision should include 4 mm margins for small, primary BCC on cosmetically sensitive
areas, and 10 mm margins otherwise.
 routine annual follow-up that includes full-body skin examinations.

19. Squamous cell carcinoma in the lower limb


Question was about treatment, biopsy, lymph node mets and causes
 appears most frequently on sun-exposed areas
 Overlying erythema, papulonodular with crust and ulceration; usually red-brown
 May have surrounding induration and satellite nodules
 Metastases risk: Melanoma > Squamous cell CA > Basal cell CA
 Can develop in post-XRT areas or in old burn scars (Marjolin ulcer: arise in areas of previous burns)
 Risk factors: -
fair skin; light-colored eyes; prior actinic keratosis; xeroderma pigmentosum; Bowen’s disease,
atrophic epidermis and exposure to chlorophenols, nitrates, arsenicals, and hydrocarbons (coal tar).
Other risk factors associated with the disease are chronic excessive sun exposure, immunosuppression,
HPV, previous XRT, previous skin CA, previous trauma, and burns.
 Risk factors for metastasis - poorly differentiated, greater depth, recurrent lesions, immunosuppression
 Tx: 0.5cm margins usual (2cm margins for Marjolin’s ulcers and penile/vulvar areas)
 Can treat high risk with Mohs surgery (margin mapping using conservative slices; not used for
melanoma) when trying to minimize area of resection (ie lesions on face)
 Regional lymphadenectomy for clinically positive nodes (palpable).
 XRT & chemotherapy - may be of limited benefit for inoperable disease, metastases or
neuro/lymphatic/vessel invasion
 Indications for prophylactic lymph node dissection (any below, dissect appropriate lymph node basin):
1) > 4 mm deep
2) > 2 cm in circumference
3) parotid basin lesions (high metastasis rate to parotid gland) - all need superficial parotidectomy

20. Melanoma 2cm treatment


Recommended Surgical Margins for Melanoma Excision
Melanoma Thickness (Depth) (mm) Clinical Excision Margin (cm)
In situ 0.5cm
Thin (≤ 1mm) 1cm
Intermediate (1-2mm) 1-2cm
Thick (> 2mm) or with satellite lesions 2cm
 melanoma prognostic features: depth

21. Second most common cause of death after blood transfusion:


a. Bacterial infection / Sepsis ****
b. ABO incompatible blood
c. Transfusion-related acute lung injury (TRALI) – mcc
d. hemolytic transfusion reactions

22. Cleft palate treatment


a. 1m
b. 6m
c. 12m
d. 24m
 Cleft lip (primary palate) – involves lip, alveolus, or both
Repair at 10 weeks – 3months, 10 lb, Hgb 10. Repair nasal deformities at same time
Dr. wessam alzaidat - General surgery
May be associated with poor feeding
 Cleft palate (secondary palate) – involves hard and soft palates; may affect speech and
swallowing if not closed soon enough; may affect maxillofacial growth if closed too early
→ repair at 12 months

23. Blood supply of pancreas from splenic artery

24. pancreatic blood supply branches splenic artery all except inferior pancreaticoduodenal
artery is a branch of the SMA

25. pancreatic intraductal papillary mucinous neoplasm (IPMN)


 main duct neoplasm mostly malignant

26. Cryoprecipitate contains a high concentration of all except:


contains fibrinogen (factor I), factor 8 (VIII), factor 13 (XIII), von Willebrand factor, and
fibronectin.

27. Factor VII has the shortest half-life of all of the clotting factors, estimated to be 6 hours.

28. Which type of intraductal papilloma associated with increased risk of malignancy
diffuse papillomatosis
Spontaneous discharge
age >54 years
mass size >1 cm

29. Invasive ductal breast carcinoma with pregnancy first trimester what is the treatment
 MRM [+ SLNBx Metastases to lymph nodes occur in approximately 75% of pregnant] (not Total
Mastectomy)
 no hormonal or radiation therapy at any time during the pregnancy
 chemotherapy can be used after the 1st trimester

30. Treatment of inflammatory breast cancer  chemo, then MRM, then chemo-XRT
31. Contraindications of breast conservative surgery all except
 Recurrent mass, irradiation, small breast, multifocal lesion

32. Rectal carcinoma located 10 cm from anal verge involving mesocolon the best treatment
 APR + Post-op chemo-XRT
33. Fixed tumor located 4 cm from anus treatment option
 Lesions < 5 cm - Tx: WLE (need 0.5 cm margin)
 Lesions > 5 cm, if involving the anal sphincter, if positive nodes, or if recurrence 
Tx: chemo-XRT (5-FU & Cisplatin; trying to preserve the anal sphincter here and avoid APR)
Need inguinal lymph node dissection if clinically positive

34. patient CD117 + submucosal mass management


GASTROINTESTINAL STROMAL TUMORS (GISTS)
 Tx: Complete resection with 1-cm margins (Wedge resection);
no nodal dissection (tumors spread hematogenously, do not metastasize to lymph nodes)
 Effectively inhibited by tyrosine kinase inhibitors such as Imatinib (Gleevec), if malignant.
 Currently, imatinib is recommended for unresectable, metastatic, or recurrent lesions.
Adjuvant therapy should continue for a total of 3 years.
Dr. wessam alzaidat - General surgery
 GISTs are resistant to conventional chemotherapy and radiation therapy.
 The most useful indicators of survival and the risk of metastasis include
the size of the tumor at presentation, the mitotic index, location within the GI tract, and the absence
of
tumor rupture.

35. D2 lymphadenectomy in stomach carcinoma:


a. Group 10 is splenic group
 Stations 1, 2, and 7 to 12 are commonly removed with D2 lymphadenectomy
gastrectomy.

36. Pt underwent sleeve gastrectomy with leak  can be converted to gastric bypass

37. Endoscopic treatment acceptable for esophageal cancer

38. Split skin graft versus full thickness graft


 More infection rates
 More resistant
Full-thickness grafts more resistant to infection than split-thickness grafts
Infection-related graft loss was more commonly encountered in vascular ulcers and burn wounds,
and the most common cause was Pseudomonas aeruginosa.

39. Branchial cyst fistula:


 Bilateral
 Open in the pharynx
 Located in the lower third of sternocleidomastoid

40. Pleomorphic adenoma features all except: capsulated


 Often present as a painless mass
 Pleomorphic adenoma (mixed tumor) – #1 benign tumor overall of the salivary glands
 Malignant degeneration in 5%
 Tx: superficial parotidectomy
 If malignant degeneration, need total parotidectomy

41. question about ileostomy

42. 28: abdominal aortic aneurysm associated with what:


 One of options is infected aneurysm

43. first sign of retroperitoneum fibrosis is ureteric entrapment

44. In necrotizing enterocolitis, the first sign indicated bowel ischemia


Pediatric sign for laparotomy in necrotizing enterocolitis:
a. Erythema abdominal wall
 Indications for operation: Pneumoperitoneum (free air), peritonitis, clinical deterioration
(persistent metabolic Acidosis - persistent severe thrombocytopenia (< 100) despite
transfusion), abdominal wall erythema (indicates dead bowel beneath) → resect dead bowel
and bring up ostomies

45. Treatment of uncomplicated meconium ileus


Management:
NGT, Empiric antibiotics & fluid resuscitation important (osmotic load with gastrografin enema will draw
Dr. wessam alzaidat - General surgery
fluid, can cause hypotension)
Gastrografin enema Q12hr for several days (effective in 80%); can also make the diagnosis & potentially
treat the patient. Can also use N-acetylcysteine enema
Surgery is indicated if the obstruction does not respond to the Gastrografin enema or if complications
arise such as perforation or peritonitis.

46. Hernia repair about procedure all true except:

47. Primary determinant of resection in metastatic liver disease


(most important prognostic factors in liver mets)
a. Size
b. Number
c. Rest size after resection
d. liver function after resection
Answer: D

48. most threating bleeding from liver  Portal vein


post hepatic resection bleeding mostly from  hepatic veins (Must do embolization pre op to
reduce bleeding)

49. Bismuth classification of bile duct injuries and strictures


 Type 1 injury  ≥ 2 cm of the common hepatic duct is preserved below the bifurcation.
 Type 2 injury  < 2 cm remains. ***
 Type 3 injury reaches the bifurcation with preservation of continuity between the right
and
left ducts.
 Type 4 injury involves destruction of the hepatic duct confluence with separation of the
right and left hepatic ducts.
 Type 5 injury involves a separate inserting right sectoral duct with or without injury to
the common hepatic duct.

50. The following Nyhus classification of hernias is correct except for:


A. Recurrent direct inguinal hernia—Type IVa.
B. Indirect inguinal hernia with a normal internal inguinal ring—Type I.
C. Femoral hernia—Type IIIc.
D. Direct inguinal hernia—Type IIIa.
E. Indirect inguinal hernia with destruction of the transversalis fascia of Hesselbach's triangle—Type II.
Answer: E

51. Skin lesion (erythema and itching) with jaundice  pyoderma gangrenosum

52. Which organ least affected by hemorrhagic shock: skeletal muscle, cardiac muscle, liver, kidney

53. Intermittent negative pressure wound therapy (VAC) in all except ischemic ulcer

54. ileal or colon conduit used for urinary prosthesis the most common abnormality is:
 Increase uric acid, hypochloremia metabolic acidosis
https://www.ncbi.nlm.nih.gov/pmc/articles/PMC3822347/

Dr. wessam alzaidat - General surgery


55. Complex anal fistula management: seton

56. Hyperphosphatemia all except


a. Hyperparathyroidism ****
b. Hyperthyroidism
c. Malignant hyperthermia

57. pt is diagnosed as case of ca head of pancreas he lost 15kg in the last 6 months which procedure is best
for pt : CT - EUS Bx  pylorus preserving pancreaticoduodenectomy (Whipple procedure)
[EUS should be done to all patients]

58. Zenker’s diverticulum relation to which muscle  a pulsion diverticulum found posteriorly (MC)
between inferior fibers of the inferior pharyngeal constrictors & superior border of the cricopharyngeus
muscle (Killian's triangle) Tx: Cricopharyngeal myotomy

59. question about multiple myeloma & Dermatofibrosarcoma protuberans

60. post-transplant CMV chest infection what is the treatment: Ganciclovir #1 , acyclovir

61. All of the following proinflammatory except Prostaglandin

62. About TIPS all except


a. Decrease encephalopathy
63. Q. All the following are true about Transjugular Intrahepatic Portosystemic Shunt (TIPS) except
a. Its main indication is to stop esophageal variceal hemorrhage not responded to medical
Dr. wessam alzaidat - General surgery
treatment.
b. It doesn’t affect possible future liver transplant plan.
c. It improves hepatic encephalopathy.
d. It may induce renal failure.
e. Shunt thrombosis rate is high.
Answer: C
 TIPS – used for prolonged bleeding, progression of coagulopathy, visceral hypoperfusion, or refractory ascites
 Allows antegrade flow from portal vein to the IVC
 Complication of TIPS – development of encephalopathy
 Transjugular intrahepatic portosystemic shunt (TIPS). A catheter is passed into the hepatic vein via the jugular vein. A needle, inserted
through the catheter, is passed from the hepatic vein through the liver tissue into a major portal vein branch. The liver tract is
dilated with an angioplasty balloon catheter, and the tract is kept open after deployment of an expandable metal stent.

64. Trauma multiple ribs # paradoxical movement, no hemothorax or pneumothorax, stable next step
a. intubation and presser
b. pain management with closed observation ***
 Initial treatment of flail chest and pulmonary contusion includes administration of
humidified oxygen, adequate ventilation, and cautious fluid resuscitation.
 In the absence of systemic hypotension, the administration of crystalloid intravenous
solutions should be carefully controlled to prevent volume overload, which can further
compromise the patient’s respiratory status.
 Patients with significant hypoxia (i.e., PaO2 < 60 mm Hg [8.6 kPa] or SaO2 < 90%) on
room air may require intubation and ventilation within the first hour after injury.
Associated medical conditions, such as chronic obstructive pulmonary disease and renal
failure, increase the likelihood of requiring early intubation and mechanical ventilation.
 Definitive treatment of flail chest and pulmonary contusion involves ensuring adequate
oxygenation, administering fluids judiciously, and providing analgesia to improve
ventilation. The plan for definitive management may change with time and patient
response, warranting careful monitoring and reevaluation of the patient. (ATLS)
65. About Glutamine …
Glutamine is the principal energy source for enterocytes (small bowel cells)

66. Energy source for colonocytes?


A – Glutamate
B – short fatty acid
C – long fatty acid
D – Amino acid
Answer: B
Short-chain fatty acids (e.g., acetate, propionate & butyrate = butyric acid) - main nutrient of
colonocytes

67. Petersen hernia  Risk of SBO is significantly higher with a retrocolic versus antecolic approach.

68. The most common presenting clinical sign in Budd-Chiari syndrome is:
a-Ascites.
b-Esophageal varices.
c-Jaundice.
d-Encephalopathy.
e-Hemorrhoids.
Answer: A

69. Pseudohyponatremia  hyperlipidemia

Dr. wessam alzaidat - General surgery


70. Pt underwent gastric bypass, complain abdominal pain & distension, CT shows dilated bowel
loops with air in rectum Blind loop syndrome treated with antibiotics

71. Pt underwent gastrectomy, complain abdominal pain + bilious vomit, pain decrease with vomit
 Afferent-loop obstruction

72. In comparing laparoscopic gastrostomy versus percutaneous endoscopic gastrostomy (PEG) tubes, which of the
following statements is true?
A. Complication rates are similar in adults.
B. Early pneumoperitoneum is seen only after laparoscopic gastrostomy tube placement.
C. Laparoscopic gastrostomy tube placement is associated with more complications in children younger than 5 years
of age.
D. Previous fundoplication is a contraindication to PEG tube placement.
E. Mortality is higher after PEG tube placement.
ANSWER: A

73. absolute contraindications of gastrostomy  Ascites

74. PUD pathophysiology (gastric ulcer)


Gastrin secretion from partial cells

75. Desmoid tumors - benign but locally very invasive


Anterior abdominal wall (most common location)
High risk of local recurrences; no distant spread (do not metastasize)

76. High anion gap acidosis are most often due to ketoacidosis, lactic acidosis, chronic kidney disease, or
certain toxic ingestions.
Normal anion gap acidosis are most often due to gastrointestinal or renal HCO3− loss.

77. crossmatch testing in organ transplantation  a crossmatch involves placing recipient serum
(potentially containing donor-specific anti-HLA antibodies) onto donor lymphocytes (containing HLA
antigens)

78. The most reliable indicator of successful endotracheal intubation is:


(a) chest x-ray
(b) end-tidal CO2
(c) cord visualization
(d) chest auscultation
(e) pulse oximetry
Answer: B

79. Eight hours after treatment for a scald injury, an infant has a temperature of 40°C and a white
blood
count of 5,000/mm3, The burn wound is clean. The most likely diagnosis is:
(a) Cytomegalovirus infection
(b) clostridial wound infection
(c) toxic shock syndrome
(d) Pseudomonas wound infection
(e) Pneumocystis pneumoniae
Answer: Toxic shock syndrome

80. Sponge correct all except:

Dr. wessam alzaidat - General surgery


a. Emergency,
b. multiple specialties,
c. Thoracic surgery **
d. obese patient,
e. Long time operation (nurse team change)

Q81. which occur in mycotic aortic aneurysm (MAA)  Fever

Q82. Percent of carbohydrates in enteral feeding  50%

Q83.about marjolin ulcer burn: SSC

Q84.jajunim and ileum difference:

Q85. Trauma colon through and through


A- Primary repair with drain
B- Primary repair with antibiotic 14 day

Q86.lung mass
a- ‫اقل من‬30 day
b- Spuneclated

Q87.warfarin ,INR 3

Dr. wessam alzaidat - General surgery


Jordanian board of surgery part 2 22/9/2020
1. Best management for Acalculous cholecystitis. cholecystostomy.
2. Complete digestion of proteins occurs in: 1. Duodenum 2. Ileum 3. Duodenum
and ileum 4. Stomach
3. Early manifestation of lidocaine toxicity … restless?
4. Nutrition Not used in SMA syndrome. PEG (gastrostomy)
5. Not poor prognostic factor in colon cancer with liver metastasis … tumor size
more than 5 cm.
6. Most common organ in sliding rt sided hernia in females … cecum or fallopian
tube.
7. Management used in septic shock > non selective beta blockers
8. Anorectal angel formed by >> puborectalis
9. Intestinal bypass one is false: it’s a more common bariatric procedure
10.Management of chronic pancreatitis: longitudinal choledochotomy and
longitudinal pancreatojejunostomy.
11.During lap chole and transection for 2 mm bile duct would be best managed
by > ligation of the duct
12.The procedure that carouse decrease mortality and morbidity in bleeding
peptic ulcer disease is 1. Band ligation 2. IVF 3. Octreotide 4. PPI (I answered
this)
13.Electrocautery one is true: injury is more with monopolar than bipolar.
14.19 y/o female found to have 9*10 cm pancreatic tumor with solid component
your Dx would be >> solid pseudopapillary neoplasm
15.Gastrinoma ulcer true except: ulcers usually small and solitary.
16.Sister Mary joseph nodule: Mets from intra-abdominal malignancy
17.Patient with head trauma was alert then became confused Dx>> epidural
hematoma.
18.Stomach cancer one is true: if your find a polyp more than 2 cm you should
resect it due to high risk of malignancy
19.Cross match is: mixing the recipient’s plasma with the donor's red cells
20.Febrile non-hemolytic transfusion reaction (FNHTR) is caused by antibodies
directed against donor leukocytes and HLA antigens
21.Contrast nephropathy true except: occur 24-48 hours after exposure, peak at
5-7 days
22.Limitation of laparoscopic surgery all except: poor vision
23. Character of co2 pneumoperitoneum: high solubility in blood
24. C.I FOR BARIATRIC SURGERY: RECENT MI WITH 2 STENS (FROM SABISTON
MCQs)
Dr. wessam alzaidat - General surgery
25.About thyroid cancer one is true: Thyroglobulin is used for follow up
differentiated cancer
26.Liver abscess vs amebic abscess: serological test is used to dx amebic abscess.
27.Injury to the nerve that lie with superior thyroid artery would result in: loss of
high pitch sounds.
28.Refeeding syndrome true except: prominent hypercalcemia
29.2 y/o male missed rt testes, on lap. Exploration blind ended testicular vessels
your next step: close and no further work-up.
30.Duodenal hematoma true: should start on TPN until resolution of the
hematoma.
31.Frey’s syndrome: gustatory sweating.
32.Post splenectomy complications one is true: leukocytosis within 5 hours
(correct answer) thrombocytosis in 3 months / absence of Howell jolly bodies.
33.True cut leads for CABG except: acute myocardial infarction.
34.Blood supply for Axial groin flap: superficial circumflex iliac artery.
35.Hodgkin’s lymphoma subtypes except: leukocyte-lymphocyte type.
36.Post splenectomy the immunity of which IG is affected: IgM
37.Thyroglossal duct cyst: you remove part of the hyoid bone.
38.Radiotherapy affect: DNA, RNA, cell wall
39.Sensitivity: ability to detect a disease.
40.A study in which the doctor knows and the patient doesn’t know about the
treatment is considered: Single blind randomized study
41.Duplex u/s for DVT except: can differentiate between old and recent
thrombosis.
42.Physician can treat all efficiently except: cerebral contusion (you only try to
prevent secondary brain injury, epidural hematoma, hyponatremia,
hypovolemia
43.Patient with dysphagia, weight loss and salivation your Dx: cricopharyngeal
dysfunction
44.GIST true except: symptomatic means more aggressive tumor. / never
metastasize to the lungs
45.D2 resection involves: celiac LNs other options SMA/ porta hepatis
46.Peri-umbilical innervation> T10
47.Patient with trauma to the knee, posterior knee dislocation and absent pulse,
after reduction has pulse but weak >> next step angiogram (I answered this) /
heparin immediately, exploration
48.All are true except in breast cancer: medullary, colloid, tubular variants have
poor prognosis
49.Level 2 axillar LNs: posterior to P. Minor
Dr. wessam alzaidat - General surgery
50.Essential cell in wound healing>>> macrophage
51.Patient presented with wound after a while has decrease in size (Wound
contracture) this is caused by >>> myofibroblasts
52.Phyllodes tumor except>> should do lymph node dissection.
53.Breast MRI except >> better sensitivity than mammogram especially in older
women.
54.Post mastectomy radiation in all of the following except: DCIS more than 6cm
55.All involved in colon cancer except>> RET protooncogene.
56.Patient has F.B ingested 2 years ago now present with the F.B located in the rt
lower quadrant and multiple small liver cysts (abscess) management >>
removal of the FB and IV Abx
57.Management of patient with gastric non-MALT lymphoma>>> chemotherapy,
radiation, surgery.
58.Diaphragmatic injury one is true: more common on left side
59.Stage IV colon cancer (patient with liver Mets resected and CEA < 200) 5 yr.
survival is>> 25%
60.PTH assay should be done >> 10 minutes after resection.
61.Good prognostic variant of papillary cancer>>> follicular variant
62.The term plunging ranula refers to which clinical entity? (baily)
A A malignant congenital salivary mass arising from the submandibular gland
B A benign salivary mass involving the parotid and submandibular glands
C A mucous retention cyst originating from the sublingual glands, limited by
the mylohyoid muscle
D A mucous retention cyst originating from the submandibular and
sublingual glands which perforates the mylohyoid muscle to enter the neck
E A midline neck mass which moves on tongue protrusion.
63.Follow up for 2 cm adrenal adenoma should be done in >> 1 year.
64.Bisthmus classification of cholangiocarcinoma for Rt hepatic duct involvement
>> bisthmus 3A
65.Patient with circumferential burn to the right upper limb escharotomy done
but still has weak distal pulse >> do fasciotomy
66.Patient with 40% and inhalational injury still has hypotension, next step >>
increase fluid intake (percentage of burn is underestimated in inhalational
injury)
67.MCC of Rectovaginal fistula>> obstetric injury.
68.All increase in septic shock except >> SVR
69.type of kidney stones in crohn’s disease>> calcium oxalate
70.patient on steroids and will have surgery>> give vitamin A
Dr. wessam alzaidat - General surgery
71.patient with no splenic artery >> completely replaced Common hepatic artery
72.patient with burn received 30 units of PRBCs and still hypotensive . most likely
cause is >> cardiac tamponade (other options hypokalemia, hypocalcemia,
hyponatremia)
73.deep cervical LNs dissection in thyroid cancer involves LNs of which levels (I &
II, III & IV, IV, I) i answered III & IV

Dr. wessam alzaidat - General surgery


JRMS exams
2020
1- Which one of the following concerning fibrolameller variant of hepatocellular carcinoma is
true:

A. Is associated with hepatitis B infection.

B. Is strongly associated with oral contraceptives.

C. Occur mainly in cirrhotic liver.

D. Associated with high level of AFP.

E. Associated with better prognosis compared to the classical hepatocellular carcinoma.

Answer:E

2-Regarding Gallbladder cancer which of the following is true:

A. The most common cause of gallbladder cancer is diffuse calcification (porcelain


gallbladder).

B. Most of the gallbladder cancers are squamous carcinomas.

C. Most gallbladder cancers are suspected clinically and radiologicaly prior to


cholecystectomy.

D. Epithelial dysplasia is considered a precursor lesion for gallbladder cancers.

E. Mostly arises from gallbladder polyps less than 10 mm.

Answer:D

3- Which cause of cirrhosis is less likely to develop HCC (hepatocellular carcinoma):

A. HBV.

B. HCV.

C. Autoimmune diseases.

D. Hemochromatosis.

E. Obesity.

Dr. wessam alzaidat - General surgery


Answer:C

4- The most common cause of death in patient with cholangiocarcinoma is:

A. Distant metastasis.

B. Septic complications.

C. Variceal hemorrhage.

D. Thromboembolic events.

E. Hepatic encephalopathy.

Answer:B

5- The half-life of alpha-fetoprotein is:

A. 6 hours.

B. 24 hours.

C. 5 days.

D. 14 days.

E. 21 days.

Answer:C

Dr. wessam alzaidat - General surgery


6- In patients with cirrhotic liver disease, the level of alpha-fetoprotein indicative of malignancy:

A. 10 ng/ml

B. 50 ng/ml

C. 100 ng/ml

D. Above 400 ng/ml

E. Above 1000 ng/ml

Answer:D

7- Concerning the tumor marker CA19-9, all of the following is true except:

A. Can be elevated in cholangitis.

B. Is elevated in colorectal carcinomas.

C. Usually elevated in cholangiocarcinoma.

D. Usually elevated in pancreatic cancer.

E. 5-10 % of the population cannot produce CA19-9 even in the presence of biliary or
pancreatic diseases.

Answer:B

8- Which of the following is exposed on the midplane of the liver after right hemihepatectomy:

A. Posterior branch of portal vein.

B. Anterior branch of portal vein.

C. Middle hepatic vein.

D. Left hepatic vein.

E. Right hepatic vein.

Answer:C

Dr. wessam alzaidat - General surgery


9- Which of the following treatments is associated with the best outcome in patients with
gallbladder cancer:

A. Surgical resection.

B. Systemic Chemotherapy.

C. Radiofrequency ablation.

D. External beam radiotherapy.

E. Systemic Chemotherapy followed by radiotherapy.

Answer:A

10- Which of the following treatments is recommended for T1 gallbladder cancer

A. Cholecystectomy and CBD excision and hepaticojejunostomy.

B. Cholecystectomy alone.

C. Cholecystectomy followed by systemic chemotherapy

D. Cholecystectomy and external beam radiotherapy.

E. Cholecystectomy and resection of segment 4b and segment 5 of the liver.

Answer:B

11- Which of the following is not part of the child-Pugh scoring system:

A. Serum bilirubin.

B. Serum albumin.

C. Prothrombin activity.

D. Platelets count.

E. Ascites.

Answer:D

Dr. wessam alzaidat - General surgery


12- Which of the following is the best therapy for a single 4 cm hepatocellular carcinoma in a
patient with uncontrollable ascites and recurrent variceal bleeding:

A. Liver resection.

B. Ethanol injection.

C. Liver transplantation.

D. Chemotherapy.

E. Radiofrequency ablation.

Answer:C

13- Which of the following is correct concerning hepatic blood supply:

A. 25% of the hepatic blood supply reach the liver via the portal vein.

B. 25% of the hepatic blood supply reach the liver via the hepatic artery.

C. 75% of the hepatic blood supply reach the liver via the hepatic artery.

D. 20% of the population have a replaced right hepatic artery arising directly from the
aorta.

E. The caudate lobe of the liver supplied by the gastroduodenal artery.

Answer:B

14- the most common site of cholangiocarcinoma:

A. Distal common bile duct.

B. Proximal common bile duct.

C. Hepatic duct confluence.

D. Intrahepatic biliary ducts.

E. Isolated left hepatic duct.

Answer:C

Dr. wessam alzaidat - General surgery


15- Regarding hepatic pyogenic abscess, all true except:

A. Commonly caused by gram negative bacteria.

B. Ascending cholangitis and portal phlebitis are the most common causes.

C. Fever and abdominal pain are the most common presenting symptoms

D. CT scan shows Low attenuating well defined lesions on unenhanced CT scan.

E. Surgical treatment is the initial modality of treatment.

Answer:E

16- the most common type of choledochal cyst is:

A. Choledochal cyst type-I

B. Choledochal cyst type-II

C. Choledochal cyst type-III

D. Choledochal cyst type-IV

E. Choledochal cyst type-V

Answer:A

17- Regarding Echinococcal disease, all are true except:

A. Is an endemic zoonosis acquired from ingestion of echinococcus granulosus eggs from


infected animals.

B. The liver is the most common site of hydatid cyst.

C. The right lobe of the liver is affected in 60-85% of cases.

D. The mainstay of treatment is drug therapy (Albendazole).

E. The hydatid cyst disease may be presented as obstructive jaundice.

Answer:D

Dr. wessam alzaidat - General surgery


18- Concerning gangrenous cholecystitis, the most common site of perforation is:

A. The gallbladder neck.

B. The body of the gallbladder.

C. The areas were the gallbladder stone stuck.

D. The Hartman’s pouch.

E. The gallbladder fundus.

Answer:E

19- Regarding Mirizzi syndrome all of the following is true except:

A. Defined as biliary obstruction secondary to cholecystitis.

B. The conversion rate to open cholecystectomy and complications are similar to those
for laparoscopic cholecystectomy for acute cholecystitis.

C. The impacted stone may erode to the adjacent structures causing cholecystoenteric
fistula.

D. Signs and symptoms of Mirizzi syndrome mimics those with acute cholecystitis in
addition to elevated bilirubin al alkaline phosphatase.

E. Open cholecystectomy is the gold standard for treatment when this condition identified
preoperatively.

Answer:B

20- One of the following is an absolute contraindication for laparoscopic cholecystectomy:

A. Pregnancy.

B. Previous upper abdominal surgery.

C. Suspicion of carcinoma.

D. Morbid obesity.

E. Liver cirrhosis.

Answer:C

Dr. wessam alzaidat - General surgery


21- The most common complication post ERCP is :

A. Bleeding.

B. Pancreatitis.

C. Cholangitis.

D. Bleeding.

E. Duodenal perforation.

Answer:B

22- The most common indication for laparoscopic cholecystectomy is :

A. Biliary colic.

B. Acute cholecystitis.

C. Pancreatitis.

D. Gallbladder empyema.

E. Hydropes of the gallbladder.

Answer:A

23- One of the following is an indication for laparoscopic cholecystectomy for a patient with an
asymptomatic gallbladder stone:

A. Previous bariatric surgery.

B. Diabetic patient.

C. Children with gall bladder stone without hemoglobinopathy or hemolytic anemia.

D. Kidney transplant patients

E. High risk group for gallbladder cancer.

Answer:E

Dr. wessam alzaidat - General surgery


24- All of the followings are part of Ranson’s criteria at the initial 48 hours of admission except:

A. Increased BUN > 8 mg/dl.

B. PaO2 < 60 mm Hg.

C. Calcium < 8 mg/dl.

D. LDH > 350 IU/L.

E. Fluid sequestration > 6 liters.

Answer:D

25- Concerning primary sclerosing cholangitis (PSC) all of the following are true except:

A. More common among men in their fourth decade of life.

B. PSC is an insidious and progressive disease that frequently leads to end-stage liver
disease and liver failure.

C. May be associated with ulcerative colitis.

D. PSC is a chronic cholestatic liver disease involve the intrahepatic bile ducts sparing the
extrahepatic bile ducts.

E. 15% to 40% of patients are asymptomatic at the time of diagnosis.

Answer:D

Dr. wessam alzaidat - General surgery


26.Which of the following does not supply to the colon:

A. Inferior mesenteric artery.


B. The Marginal artery of Drummond.
C. The middle colic artery.
D. The superior mesenteric artery.
E. The coeliac axis.

Answer:E

27.Which of the following is not correct about the ileostomy:

A. It can have two lumens.


B. It is flush to the skin.
C. It is most commonly sited in the right iliac fossa.
D. It can have single lumen.
E. It can defunction the remaining colon.

Answer:B

28.Which of the following is INCORRECT regarding the boundaries of the inguinal canal :

A. Inguinal canal forms the floor.


B. Conjoint tendon forms the posterior wall.
C. Lacunar ligament forms the posterior wall.
D. External oblique aponeurosis form the anterior wall.
E. Iliopubic tract forms the floor.

Answer:C

29.55 year old male, complaining of episodic upper abdominal pain , greasy stool that is difficult
toflush, he is also found to have diabetes , serum trypsin was low and the level of serum
amylase and lipase were normal, whats the most likely diagnosis :

A. Acute pancreatitis.
B. Cystic fibrosis.
C. Chronic pancreatitis.
D. Pancreatic cancer.
E. Small bowel carcinoid.

Answer:C

Dr. wessam alzaidat - General surgery


30.A 50 year old presents with vague persistent abdominal pain. CT scan of the abdomen reveals
aduodenal diverticulum. Which of the statements is true regarding duodenal diverticulum.

A. The overall incidence of duodenal diverticulum is 7-20% and about 10% of these require
surgery.
B. Most of the duodenal diverticula are symtomatic and perforation is the most common
complication.
C. Surgery should be avoided and carried out only in case of complications like
perforation, uncontrooled hemorrahge, persistent biliary or pancreatic symptoms.
D. Juxta vaterian diverticulum is the easiest to manage.
E. it’s the most common diverticulum of the small bowel.

Answer:C

31.Helicobacter Pylori (H.Pylori) is a known cause of peptic ulcer disease. It was discovered
inAustralia in 1987. Which of the following statements is not true regarding it:
A. Its infectivity is highest in developed world.
B. Person to person transmission is common
C. It is seen in populations with low socio economic status
D. H. Pylori is a gram negative microaerophilic bacteria.
E. It is a disease associated with poor sanitation, overcrowding countries.

Answer:A

32.Ideal treatment of alkaline reflux gastritis after after Billroth I and Billroth II gastrectmy is

A. Conversion of Billroth I gastrectomy to Billroth II gastrectomy.


B. Roux en Y gastrojejunostomy.
C. Total gastrectomy.
D. Conservative management.
E. Hepaticojejunostomy to divert bile from the stomach.

Answer:B

33.Treatment for bleeding duodenal diverticulum is


A. Diverticulectomy
B. Diverticulopexy
C. Diverticulization
D. Subtotal diverticulectomy
E. Whipple procedure.

Answer:A

34.Which of the following is not true about Pneumatosis intestinalis of small intestine:
A. It is seen equally and males and females.

Dr. wessam alzaidat - General surgery


B. Most common location is subserosa in the jejunum.
C. Operative Procedures are required in most of the cases.
D. It is associated with COPD and immunodeficiency states.
E. On plain X-ray they appear as radiolucent areas in the bowel wall which have to be
distinguished from luminal intestinal gas.

Answer:C

35.What is not true about blind loop syndrome?


A. It manifest as diarrhea, steatorrhea, weight loss, abdominal pain.
B. Megaloblastic anemia is commonly seen.
C. Surgery is almost always required to correct small bowel syndrome
D. Broad spectrum antibiotics are the treatment of choice
E. Associated with deficiencies of the fat-soluble vitamins (A, D, E, and K).

Answer:C

36.What is not true about the immune mechanism in the small intestine?
A. Intestine contains more than 70% of IgA producing cells in the body.
B. Secretory IgA inhibits the adherence of bacteria to epithelial cells and prevents their
colonization and multiplication.
C. Ig A is produced by T-lymphocytes in the lamina propria.
D. Approximately 60% of the lymphoid cells are T cells.
E. Secretory IgA neutralizes bacterial toxins and viral activity and blocks the absorption of
antigens from the gut.

Answer:C

37.Which of the following is the most common cause of death in Crohn's disease of small
intestine
A. Malignancy
B. Sepsis
C. Electrolyte Disorders
D. Thromboembolic Phenomenon
E. Toxic mega colon.

Answer:A

38.What is the first step in investigating a 26 year old male with solitary thyroid nodule 1 cm in
sizein the right lobe of the thyroid?
A. Radio Isotope scan.
B. Thyroid function test (T3,T4, TSH).
C. US guided FNABx.
D. Clinical examination every 6 months.
E. Right hemithyroidectomy.

Answer:B

Dr. wessam alzaidat - General surgery


39.Which of the following is not true for Hashimoto thyroiditis?
A. It is an autoimmune disease caused by CD4 cells with specificity to thyroid antigens
B. Commonly presents as hypothyroidism
C. Surgery is required in almost all cases
D. Hashimoto thyroiditis can progress to lymphoma of thyroid
E. Hashimoto's thyroiditis more common in women

Answer:C

40.Regarding overwhelming post-splenectomy infection (OPSI), all of the following are true
except:

A. Is a rare but rapidly fatal infection occurring in individuals following splenectomy.


B. Are caused by encapsulated organisms including Streptococcus pneumonia.
C. Most infections occur after the second year of splenectomy.
D. Occur in 1-2% of patient following splenectomy.
E. Carries a 50% mortality risk.

Answer:C

Dr. wessam alzaidat - General surgery


JRMS RANDOM MCQS

<1>For correction of pre-existing dehydration: All are true, Except:


<A>Bowel losses come from the extra-cellular fluid
<B>Pure water losses are from the total body water
<C>Protein containing fluid is lost from the plasma
<D>The only hypotonic solutions in the body are saliva and sweat
<E>Losses from extracellular compartment are much better tolerated than those
from intracellular compartment

ANSWER:E

<2>The time for platelet transfusion during splenectomy for idiopathic


thrombocytopenic purpura is:
<A>On making the incision
<B>After ligation of the splenic artery
<C>On induction of anesthesia
<D>After removal of the spleen
<E>One hour before surgery

ANSWER:B

<3>Total parenteral nutrition (TPN): All are true, Except:


<A>It should be given through a central vein
<B>The main source of calories has to be of carbohydrates
<C>Average calorie requirement is 30-40 K Calorie per kg body weight
<D>Average protein requirement is 3 grams per Kg body weight
<E>Commonest metabolic complication is hyperosmolar non-ketotic hyperglycemia

ANSWER:D

<4>Burst abdomen: All the followings are true, Except:


<A>Overt wound dehiscence follows removal of the skin sutures.
<B>Occult wound dehiscence occurs with disruption of the musculoaponeurotic
layers beneath intact skin sutures.
<C>It is a herniation of bowel through a partial or complete defect in the
musculoaponeurotic closure.
<D>Wound dehiscence is at least twice as common in women as in men
<E>Wound dehiscence is more common in patients over the age of 60

ANSWER:D

<5>Pain in the postoperative period: All the followings are causes, Except:
<A>Stitch abscess
Dr. wessam alzaidat - General surgery
<B>Granuloma
<C>Incisional hernia
<D>Neuroma
<E>Obliteration of the dead space

ANSWER:E

DR.WBZ
<6>Contraindications to major elective surgery requiring general anesthesia: all are
true, Except:
<A>Myocardial infarction 2 months ago
<B>Preoperative serum potassium of 2.6 mmol/liter in a patient on diuretic therapy
<C>Previous mitral valve replacement
<D>Resolving upper respiratory tract infection
<E>Unsuspected glycosuria on routine ward urine testing

ANSWER:C

Dr. wessam alzaidat - General surgery


<7>Discharge criteria following ambulatory surgery include all the following,
Except:
<A>Ability to eat solid food
<B>Stable vital signs
<C>Ability to ambulate
<D>Ability to have protective airway reflex
<E>Adequate pain control

ANSWER:A

<8>Hidradenitis suppurativa, all are true, Except:


<A>Characterized by recurrent abscess/sinus formation
<B>Is a disease of apocrine sweat glands
<C>Young adult are the usual victims of this disease
<D>Axilla is the commonest site affected
<E>Best treated by antibiotics for a long time

ANSWER:E

<9>Lignocaine, all are true, Except:


<A>Reduce cell membrane permeability to sodium ions
<B>Acts on small unmyelinated C fibers before large myelinated A fibers.
<C>Has a duration of action of greater than 12 hours
<D>20 ml of 1% lignocaine is a safe dose in a 70 kg man
<E>Addition of adrenaline allows the dose used to be safely doubled

ANSWER:C

<10>Tetanus, all are true, Except:


<A>Is due to an infection with a gram-negative spore forming rod
<B>The organism produces a powerful exotoxin
<C>The toxin prevents the release of inhibitory neurotransmitter
<D>Clostridium tetani is sensitive to penicillin
<E>Risus sardonicus is the typical facial spasm

ANSWER:A

<11>A 67-year-old patient is severely injured in an automobile accident. His pelvis


is crushed and his right femur fractured by the dashboard of the car. Extraction
from the vehicle is a long and difficult procedure. An intravenous infusion of
lactated Ringer's solution is begun at the scene of the accident, and additional
intravenous fluid is administered upon arrival in the emergency room to raise his
central venous pressure (CVP) to 10 cm. H 2O is given. His blood pressure rises
from 70/40 to 130/80 mm. Hg during the fluid resuscitation but no urine output is
Dr. wessam alzaidat - General surgery
observed. The initial step to evaluate the patient's anuria should be:
<A>Furosemide (Lasix), 100 mg. as an intravenous bolus.
<B>Mannitol, 12.5 gm, as an intravenous bolus.
<C>Lactated Ringer's solution as an intravenous bolus to raise the CVP to 15 cm. H
2O.
<D>Computed tomography of the abdomen and pelvis with contrast.
<E>Digital rectal exam
ANSWER:E
DR.WBZ
<12>Abdominal trauma, all are true, Except:
<A>Solid organ injury is the commonest cause of internal bleeding
<B>Peritoneal lavage is indicated for patients with equivocal physical sign
<C>Peritoneal lavage is considered positive when RBCs count in the lavage fluid
is 1000 RBC/ml
<D>Renal injuries are not always associated with hematuria
<E>Computed tomographic scanning is preferred in the pediatric age group in
stable patients for diagnosis

ANSWER:C

Dr. wessam alzaidat - General surgery


<13>Diaphragmatic injuries, all are true, Except:
<A>They are more commonly associated with chest than abdominal injuries
<B>Road traffic accidents are the commonest cause
<C>It is commoner on the left side
<D>Unrecognized may present later with incarcerated hernia
<E>Associated intra abdominal injuries should be excluded

ANSWER:A

<14>Hemorrhagic shock, what initial attention in blood pressure is seen?


<A>Increase in systolic pressure
<B>Decrease in systolic pressure
<C>Increase in diastolic pressure
<D>Decrease in diastolic pressure
<E>Increase in pulse pressure

ANSWER:C

<15>Parotid Pleomorphic adenoma: All are true, Except:


<A>It is the most common parotid gland tumor and it accounts for 70-80% of all
benign parotid tumors
<B>The majority (90%) arise in the superficial lobe lateral to the facial nerve
<C>It is painless slowly growing mass
<D>It is usually solitary and well demarcated
<E>It is more common in men

ANSWER:E

<16>40-year-old male undergoes total thyroidectomy for follicular thyroid


carcinoma. 48 hours later he develops circumoral numbness, followed by
laryngospasm and then generalized seizure. Of the following, which One is the first
priority?
<A>Proceed to operating room to explore the operative site
<B>Administer 10ml of 10% calcium gluconate slowly intravenously
<C>Obtain a serum magnesium measurement and administer intravenous
magnesium chloride
<D>Obtain a CT scan of the head to evaluate the possibility of brain metastasis
<E>Immediate endotracheal intubation

ANSWER:B

Dr. wessam alzaidat - General surgery


<17>Primary hyperparathyroidism, most patients have elevations of all the
following, Except:
<A>Total serum calcium
<B>Ionized serum calcium
<C>Serum alkaline phosphates
<D>Urinary CAMP
<E>Urinary calcium

ANSWER:C

Dr. wessam alzaidat - General surgery


<18>Thyroglossal cyst: all are true, Except:
<A>Usually lies in the midline just below the hyoid bone.
<B>Moves with tongue protrusion
<C>Connected to the foramen ovale
<D>Frequent aspiration is not indicated for treatment.
<E>Thyroid follicles may be found in around 40% of cases

ANSWER:C

<19>Thyroid cancer, all are true, Except:


<A>Total thyroidectomy is the optimal operation for differentiated tumors
<B>Postoperative thyroglobulin is a sensitive indicator for recurrence or metastasis
<C>Anaplastic carcinoma affects elderly females more than males
<D>Differentiated tumors are not sensitive to TSH stimulation
<E>Postoperative radioiodine scan is needed to detect any distant metastasis

ANSWER:D

<20>Breast cancer, the commonest histological type is:


<A>Tubular carcinoma
<B>Carcinoma of no special type(NST)
<C>Medullary carcinoma
<D>Carcinosarcoma
<E>Infiltrating lobular carcinoma

ANSWER:B

<21>Idiopathic granulomatous mastitis (IGM), all are true, Except:


<A>Is characterized by the presence of chronic granulomatous lobulitis in the
absence of an obvious etiology
<B>Clinically, patients present with a hard lump that mimics carcinoma, which may
lead to nipple retraction and sinus formation.
<C>It is linked to lactation and was reported to occur during pregnancy and
postpartum period.
<D>Associations with alpha-1-antitrypsin deficiency and hyperprolactinemia have
been reported.
<E>Conservative treatment with estrogen is the mainstay of treatment

ANSWER:E

<22>All the following viruses are associated with human tumors, Except:
<A>Epstein-Barr viruses
<B>Hepatitis B Virus
<C>Adeno-viruses

Dr. wessam alzaidat - General surgery


<D>Human immuno-deficiency viruses
<E>Human papilloma viruses

ANSWER:C

DR,WBZ

<23>Hamartomas, One of the following statements is True:


<A>There is proliferation of abnormal cells in a tissue
<B>It has malignant potential
<C>It has a normal number of a cell type not usually found in that tissue
<D>It has an increased number of a cell type normally found in that tissue
<E>It has a pluripotent germ cell element

ANSWER:D

Dr. wessam alzaidat - General surgery


<24>Malignant melanoma, all are true, Except:
<A>They are usually pigmented, but a small number lack pigmentation
<B>It usually presents with history of a change in a pigmented lesion
<C>At least 5 centimeters clear margin is needed to establish adequate excision
<D>Prophylactic lymph node dissection is recommended for intermediate stage
lesions (Clarks‟ level III & IV)
<E>Immunotherapy has a rule in treatment of advanced stages

ANSWER:C

<25>Femoral hernia: all are true, Except:


<A>Strangulates more commonly than inguinal hernias
<B>More common in elderly female
<C>During repair the femoral vein is found on its lateral side
<D>May be confused by a saphena varix
<E>Reaching the bottom of scrotum called complete hernia

ANSWER:E

<26>Inguinal hernia (Direct): One of the followings is True:


<A>Never occurs in women
<B>Frequently strangulates
<C>Is generally the most difficult groin hernia to reduce
<D>Is associated with a patent processus vaginalis
<E>Is not seen in children

ANSWER:E

<27>Rectus sheath hematoma, all are true, Except:


<A>Bleeding into rectus sheath may simulate acute surgical abdomen
<B>It may follow direct trauma to the abdominal wall in the elderly
<C>Spontaneous rectus sheath hematoma may occur in infection diseases
<D>It can be a manifestation of over treatment with anticoagulants
<E>It‟s main treatment is exploration of the hematoma and evacuation of blood clots

ANSWER:E

<28>Structures which pass through diaphragm: One is True:


<A>Inferior vena cava pierces the central tendon to the right of median plane at
level of 10th thoracic vertebra
<B>Esophagus passes through oval hiatus at level of 12th thoracic vertebra
<C>Thoracic duct passes through the aortic hiatus
<D>The Right vagal trunk passes Right to the inferior vena cava
<E>Branches of the Right gastric artery pass though the esophageal hiatus

Dr. wessam alzaidat - General surgery


ANSWER:C

DR.WBZ
<29>Mesenteric cyst, all are true, Except:
<A>They are most often due to congenital lymphatic spaces that gradually fill
with lymph
<B>They usually present as abdominal masses accompanied by pain, nausea, or
vomiting
<C>They usually can be diagnosed by physical examination and have
characteristic lateral mobility
<D>They are best treated by marsupialization
<E>Omental cysts are frequently asymptomatic unless they undergo torsion

ANSWER:D

Dr. wessam alzaidat - General surgery


<30>Peritonitis, all are true, Except:
<A>Primary peritonitis is usually caused by single organism
<B>A defect in the gastrointestinal tract is the commonest cause of secondary
peritonitis
<C>The main goal of surgery in secondary peritonitis is to control the source of
contamination
<D>Antibiotic against anaerobic organism is usually enough to cover the causative
organisms
<E>Ultrasound guided drainage is used for localized from of abscess

ANSWER:C

<31>An 18-year-old woman presents with abdominal pain and weight loss.
Examination reveals a soft, moveable epigastric mass. Endoscopy demonstrates a
large mass of black hair in the stomach. Which of the following is the most
appropriate therapy?
<A>Endoscopic extraction in one session
<B>Oral administration of papain
<C>Psychiatric consultation
<D>Gastrostomy and removal
<E>Gastrojejunostomy

ANSWER:D

<32>Bleeding from gastrointestinal tract: The following conditions may present


with hematemesis, Except:
<A>Esophageal varices
<B>Chronic peptic ulcer
<C>Carcinoma of stomach
<D>Malory-weiss syndrome
<E>Colonic diverticulosis

ANSWER:E

<33>Dumping syndrome, all are true, Except:


<A>Symptoms can be controlled with a somatostatin analog
<B>Diarrhea is always part of the dumping syndrome
<C>Flushing and tachycardia are common features of the syndrome
<D>Separating solids and liquids in the patient's oral intake alleviates some of the
symptoms
<E>Early postoperative dumping after vagotomy often resolves spontaneously

ANSWER:B

Dr. wessam alzaidat - General surgery


<34>Gastric cancer, One of the followings is True:
<A>Distal gastric cancers are becoming more common
<B>Intestinal-type gastric tumors resemble colon carcinomas and have a better
prognosis than diffuse type
<C>Early gastric cancers are confined to the mucosa and are lymph node negative
<D>Squamous cell carcinoma constitute more than 50% of gastric carcinoma
<E>Broders' histologic grading system correlates well with survival: patients with
grade IV tumors have 5-year survival rates around 65%

ANSWER:B

<34>Gastric lymphoma (primary diffuse large B-cell), One of the following is True:
<A>Bleeding is the most common presentation
<B>Mucosal biopsies establish the diagnosis
<C>Primary therapy is surgical excision
<D>Constitutes 40% of all gastric malignancies
<E>Long-term survival is equivalents to adenocarcinoma

ANSWER:C

Dr. wessam alzaidat - General surgery


<35>Lymph node mass palpable in „Troisier's sign‟ is:
<A>Epigastric
<B>Left supraclavicular
<C>Left axillary
<D>Left inguinal
<E>Left submandibular

ANSWER:B

<36>Vascular compression of the duodenum, One of the following statements


about the anatomic basis for this syndrome is True:
<A>The duodenum is obstructed in its first third before it crosses over the
lumbar vertebral column.
<B>Structures crossing beneath the superior mesenteric artery include the
duodenum, the uncinate process of the pancreas, and the left renal vein.
<C>Hyperextension of the back allows the angle of origin of the superior
mesenteric artery to widen, lessening the obstruction of the duodenum.
<D>Patients are at significant risk for vascular compression of the duodenum if the
angle between the takeoff of the superior mesenteric artery and the aorta is less than
45 degrees.
<E>Arteriographic studies show a typical area of extrinsic compression and
narrowing of the arterial lumen due to duodenal pressure.

ANSWER:B

<37>Blind loop syndrome, all are true, Except:


<A>Is due to stasis
<B>Causes bacterial overgrowth
<C>Is more commonly caused surgically
<D>May cause steatorrhea
<E>Causes macrocytic anemia due to folic acid deficiency

ANSWER:E

<38>Cardinal symptoms of intestinal obstruction are the following, Except:


<A>Nausea
<B>Abdominal pain
<C>Abdominal distention
<D>Constipation
<E>Vomiting

ANSWER:A

Dr. wessam alzaidat - General surgery


<39>Crohn‟s disease, may have all the following associated conditions, Except:
<A>Ankylosing spondylitis
<B>Primary sclerosing cholangitis
<C>Small bowel lymphoma
<D>Uveitis
<E>Erythyma nodosum

ANSWER:C

Dr. wessam alzaidat - General surgery


<40>Intestinal obstruction, One of the following is mostly suggestive of
strangulation:
<A>Intermittent abdominal pain
<B>Abdominal distension
<C>Rebound tenderness
<D>Exaggerated bowel sounds
<E>Multiple air fluid levels on standing x-ray of abdomen

ANSWER:C

<41>Meckel's diverticulum, most often occurs in the:


<A>Proximal jejunum
<B>Distal jejunum
<C>Proximal ileum
<D>Distal ileum
<E>Sigmoid colon

ANSWER:D

<42>Paralytic ileus, the following drugs can be the cause, Except:


<A>Morphine
<B>Neostigmine
<C>Propantheline
<D>Phenothiazine
<E>Ganglion blocking agent

ANSWER:B

<43>Small bowel fistulas, factors that prevent its spontaneous closure include
all the followings, Except:
<A>High output of more than 500 ml/24hrs
<B>Long tract more than 2.5 cm
<C>Undrained abscess
<D>Active granulomatous disease
<E>Disruption of more than 50% of bowel circumference

ANSWER:B

<44>Carcinoid of the appendix: All are true, Except:


<A>Although the vast majority of appendiceal carcinoids behave in a benign
fashion, they are considered malignant because they all have the potential for
invasion, metastasis, and production of physiologically active substances
<B>Carcinoids occur in about 0.5% of appendectomy specimens
<C>Carcinoids account for about 85% of appendiceal malignancies

Dr. wessam alzaidat - General surgery


<D>About 10% of all carcinoid tumors originate in the appendix
<E>They are most common in young adult women

ANSWER:D

<45>A six-year old child presented with generalized peritonitis secondary to


perforated appendicitis. The most appropriate management is:
<A>Take blood culture and start antibiotics after culture sensitivity result
<B>Rush the child to theatre for immediate surgical intervention
<C>Initial conservative management with nasogastric tube and antibiotics for 48
hours then reassessment
<D>Initial fluid resuscitation, antibiotics, then surgical exploration
<E>Ultrasound scan to confirm diagnosis

ANSWER:D

DR.WBZ

Dr. wessam alzaidat - General surgery


<46>Nonspecific mesenteric lymphadenitis, all are true, Except:
<A>Most commonly occur below 18 years of age
<B>Shifting tenderness may be present in the abdomen
<C>Is complicated by formation of intra abdominal abscesses
<D>Can be accurately diagnosed only at laparotomy
<E>May be associated with cervical lymphadenopathy

ANSWER:C

<47>Basal tone of the anal canal measured by anal manometry study is mainly
contributed by:
<A>External sphincter alone
<B>Internal sphincter alone
<C>Internal and external sphincter
<D>Longitudinal muscle layer
<E>Hemorrhoidal cushions

ANSWER:B

<48>Colonic angiodysplasia, all are true, Except:


<A>More common in the cecum and ascending colon
<B>May be associated with aortic valve disease
<C>It is a common cause of massive lower GI bleeding
<D>It is a congenital disease
<E>The lesions are small and may not bee seen through colonoscopy

ANSWER:D

<49>Colonic motility, all are true, Except:


<A>Mass contractions occurs at varying intervals through the entire colon's length
<B> “Antiperistaltic” contractions occur in the descending colon
<C>The rectum can accommodate stool by receptive relaxation
<D>Colon contents in the descending colon are propelled by tonic contractions.
<E>Defecation involves both sensory and motor pathways

ANSWER:B

<50>Colonic polyps, One of the followings is True:


<A>Juvenile rectal polyps are adenomatous polyps
<B>Metaplastic polyps are premalignant
<C>The risk of malignancy is higher in tubular than villous adenomas
<D>Villous adenomas occasionally cause hyperkalemia
<E>All patients with untreated familial adenomatous polyposis will eventually
Dr. wessam alzaidat - General surgery
develop colorectal carcinoma

ANSWER:E

<51>Colorectal cancer (CRC): the least possible presentation is:


<A>Constipation
<B>Constipation alternating with diarrhea
<C>Abdominal mass
<D>Massive lower GI bleeding
<E>Large bowel obstruction (Abdominal distension)
<F>Inherited CRC is responsible for the minority of CRC cases

ANSWER:D

Dr. wessam alzaidat - General surgery


<52>Colostomy, all are true, Except:
<A>It is an anastomosis between skin and colon
<B>It may be temporary or permanent
<C>One of its complications is prolapse of the colon through it
<D>It may be constructed in left iliac fossa
<E>Patients with colostomy need water and electrolyte replacement by
intravenous route

ANSWER:E

<53>Diverticular disease, all are true, Except:


<A>The presence of pneumaturia indicates that the rectum is involved
<B>Initial management of acute diverticulitis is non surgical
<C>Peridiverticular pus collection needs drainage
<D>Most of the cases are asymptomatic
<E>A solitary diverticulum in the cecum may be a true diverticula

ANSWER:A

<54>Fissure-in-ano, all are true, Except:


<A>There is manometric evidence of sustained hypertonicity of the internal anal
sphincter in patients with chronic anal fissure
<B>Over 90% of anal fissures are located in the posterior midline
<C>Successful treatments for chronic anal fissure produce a significant reduction
in resting anal tone and a significant increase in anodermal blood flow
<D>Nitric oxide has been identified as the chemical messenger of the intrinsic non-
adrenergic, non-cholinergic pathway mediating relaxation of the internal anal
sphincter
<E>Warm baths have been shown to increase the resting anal pressure

ANSWER:E

<55>Inflammatory bowel diseases, all are true, Except:


<A>Total proctocolectomy and pouch formation is the ideal operation for
ulcerative colitis
<B>Crohn's disease is a contraindication for pouch formation
<C>Temporary ileostomy is not a must for pouch formation operation
<D>Pouchitis necessitates removal of the pouch
<E>Pouch formation is not always technically possible

ANSWER:D

<56>Solitary rectal ulcer, all are true, Except:


<A>It is confined to the rectum

Dr. wessam alzaidat - General surgery


<B>May be polypoid
<C>Most patients have rectal bleeding as the presenting symptom
<D>The lesion may be multiple
<E>Defecography frequently revealed an occult rectal prolapse

ANSWER:A

<57>The incidence of anastomotic leakage is greatest in:


<A>Large bowel anastomoses
<B>Gastrojejunal anastomoses
<C>Ileoileal anastomoses
<D>Ileo-jejunal anastomosis

Dr. wessam alzaidat - General surgery


<E>Esophago- jejunal anastomoses

ANSWER:A

<58>The best intraoperative management of a 3 cm non bleeding liver laceration


during laparotomy for an injured patient is one of the followings:
<A>Suture the laceration
<B>Mesh closure of the laceration
<C>Peritoneal Toileting and close abdomen
<D>Use binding glue
<E>Resect the laceration and approximate the edges well

ANSWER:C

<59>Cholangiocarcinoma (CCA): all are true, Except:


<A>Cholangiocarcinomas tend to grow slowly and to infiltrate the walls of the
ducts, dissecting along tissue planes.
<B>Local extension occurs into the liver, porta hepatis, and regional lymph nodes
of the celiac and pancreaticoduodenal chains.
<C>May be complicated by cholangitis that requires immediate antibiotic
intervention and aggressive biliary drainage
<D>In the US: Each year, approximately 2500 cases occur, compared to 5000 cases
of gallbladder cancer and 15,000 cases of hepatocellular cancer. Average incidence
is 1 case per 100,000 persons per year.
<E>Carcinoembryonic antigen (CEA) and CA 19-9 are not elevated in CCA

ANSWER:E

<60>Gallbladder cancer: all are true, Except:


<A>Majority occurs in female
<B>Associated with stones in 90% cases
<C>Presently features are not typical
<D>Five year survival is 50%
<E>Chronic infection by Salmonella typhi has been associated with an increased
risk of gallbladder cancer.

ANSWER:D

<61>Gallstones in diabetes patients, which of the following statements is correct?


<A>Gallstones occur with the same frequency in diabetes patients as in the healthy
population.
<B>The presence of gallstones, regardless of the presence of symptoms, is an
indication for cholecystectomy in a diabetes patient.
<C>Diabetes patients with gallstones and chronic biliary pain should be managed
Dr. wessam alzaidat - General surgery
conservatively with chemical dissolution and/or lithotripsy because of severe
complicating medical conditions and a high operative risk.
<D>The presence of diabetes and gallstones places the patient at high risk for
pancreatic cancer.
<E>Diabetes patients with symptomatic gallstones should have prompt elective
cholecystectomy, to avoid the complications of acute cholecystitis and gallbladder
necrosis
ANSWER:E

<62>Hydatid cyst: One of the following statements is True:


<A>The human being gets infected by ingesting the contaminated sheep meat
<B>Is best treated by albendazole
<C>Heavy calcification is characteristic of lung lesions
<D>Can be treated by laparoscopic aspiration
<E>Camels are occasionally the definitive hosts

ANSWER:D

Dr. wessam alzaidat - General surgery


Dr. wessam alzaidat - General surgery
DR.WBZ

Dr. wessam alzaidat - General surgery


<63>Laparoscopic access for abdominal surgery, as compared with laparotomy,
results in which of the following?
<A>A greater increase in the catecholamine response as compared to open
procedures, owing to the stress of the pneumoperitoneum
<B>A more pronounced suppression of immune function due to impairment of
T-cell function as determined by skin antigen testing
<C>A reduction in the rate of postoperative ileus
<D>A threefold increase in the rate of deep venous thrombosis
because of
pneumoperitoneum-induced venous stasis
<E>An increase in wound infection rates related to laparoscopic methods of
specimen removal

ANSWER:C

<64>Liver malignancies (Metastatic): A 58-year-old male undergoes resection of a


Dukes C2 colon cancer via right hemicolectomy. Three years postoperatively, rising
CEA levels prompt evaluation including abdominal computed tomography. Two
lesions, each measuring 2 cm, are noted in the right hepatic lobe. No other
abnormalities are noted. A right hepatic lobectomy is performed without
complication. Which of the following most closely approximates anticipated 5-year
survival?
<A>85–90%
<B>65–70%
<C>45–50%
<D>25–30%
<E>5-10%

ANSWER:D

<65>Post-cholecystectomy pain, potential causes include all the following, Except:


<A>Common bile duct stone
<B>Traumatic stricture
<C>Biliary dyskinesia
<D>Cystic duct remnant
<E>Irritable bowel syndrome

ANSWER:E

<66>Whipple's triad is diagnostic of:


<A>Carcinoma thyroid
<B>Insulinoma
<C>Renal cell carcinoma
<D>Lymphoma
<E>Pheochromocytoma
Dr. wessam alzaidat - General surgery
ANSWER:B
<67>Acute pancreatitis secondary to hyper triglyceridemia, all are true, Except:
<A>Clinically significant pancreatitis usually does not occur until a person's serum
triglyceride level reaches 1000 mg/dL
<B>It is associated with type I and type V hyperlipidemia
<C>While somewhat controversial, most authorities believe that the association is
caused by the underlying derangement in lipid metabolism rather than by
pancreatitis causing hyperlipidemia
<D>This type of pancreatitis tends to be more severe than alcohol- or gallstone-
induced disease
<E>In order to unmask a true elevation in serum amylase, the serum should be
acidified

ANSWER:E

Dr. wessam alzaidat - General surgery


<68>Acute pancreatitis, the followings are uncommon physical findings, which
are associated with severe necrotizing pancreatitis, Except:
<A>Cullen sign
<B>Grey-Turner sign
<C>Trousseau sign
<D>Erythematous skin nodules
<E>Purtscher retinopathy

ANSWER:C

<69>Insulinoma: all are true, Except:


<A>Causes hypoglycemia which is not constant
<B>Tumor is difficult to be detected
<C>Patient may be treated in mental hospitals
<D>Can be confused with epilepsy
<E>Is characterized by loss of appetite

ANSWER:E

<70>Following splenectomy, there is increased susceptibility to infection with all,


Except:
<A>Strep. Pneumoniae
<B>Haemophilus influenzae
<C>Malaria
<D> E.coli
<E>Meningococcal infections

ANSWER:D

<71>Overwhelming post-splenectomy infection, all are true, Except:


<A>Is usually due to unencapsulated bacterial infection
<B>Streptococcus pneumonia is the commonest etiological agent
<C>Despite aggressive therapy it can have a mortality of over 50%
<D>The risk of infection can be reduced with pneumococcal and Haemophilus
vaccination
<E>Penicillin antibiotic prophylaxis should be considered in all children

ANSWER:A

<72>Splenic artery aneurysm, all are true, Except:


<A>Females are affected more than males
Dr. wessam alzaidat - General surgery
<B>Generally it is single and situated in the main trunk of the artery
<C>Multiple aneurysms may be a sequence of intraabdominal sepsis
<D>Spontaneous rupture may occur in late pregnancy
<E>Treatment of choice is selective embolization

ANSWER:E

<73>Hyperaldosteronism (Conn's syndrome), all are true, Except

<A>Is due to primary hyperaldosteronism


<B>Can result in hypokalemia
<C>Can result in a metabolic alkalosis
<D>Is most often due to an adrenal adenoma
<E>Can cause severe hypotension

ANSWER:E

Dr. wessam alzaidat - General surgery


<74>Pheochromocytoma: All are true, Except:
<A>In 90% of cases it is caused by solitary adrenal origin
<B>It is found in 5% of patients investigated for hypertension
<C>It is usually discovered early because of the catecholamines effect
<D>10% of cases are malignant
<E>It mostly secretes noradrenalin

ANSWER:B

<75>A needle passed through the right ninth intercostal space in mid axillary
line can pass through all the following, Except:
<A>External oblique muscle
<B>Pleural cavity
<C>Diaphragm
<D>Serratus anterior muscle
<E>Costo diaphragmatic recess

ANSWER:D

<76>Thymus gland, all are true, Except:


<A>Lies within the anterior mediastinum
<B>Develops from the fourth pharyngeal pouch
<C>Decreases in size with age
<D>Is made up of cells of endodermal origin
<E>Descends anterior to the brachiocephalic vein

ANSWER:B

<77>After an acute myocardial infarction, elective operation should be postponed


for at least:
<A>1 month
<B>3 months
<C>6 months
<D>1 year
<E>2 years

ANSWER:C

<78>Congenital abnormalities, all are true, Except:


<A>Omphalocele represents a defect in the abdominal wall lateral to the umbilical
Dr. wessam alzaidat - General surgery
cord.
<B>The herniated viscera associated with omphaloceles are usually covered with a
membranous sac.
<C>An umbilical polyp is a small excrescence of omphalomesenteric duct mucosa
that is retained in the umbilicus.
<D>Meckel's diverticulum results when the intestinal end of the omphalomesenteric
duct persists and represents a true diverticulum.
<E>In gastroschisis no sac is present to cover the herniated intestine

ANSWER:A

Dr. wessam alzaidat - General surgery


<79>Congenital diaphragmatic hernia (Bochdalek), One of the following is True:
<A>Presentation can be delayed after the first year of life
<B>Severity of the condition is related to the amount of viscera in the chest
<C>The defect occurs in the anterolateral part of the diaphragm
<D>More common on the right than the left side
<E>Occurs due to incomplete extension of septum transversum to the thoracic wall

ANSWER:A

<80>Gastroschisis and Exomphalos: One of the followings is True:


<A>A gastroschisis has a sac
<B>Gastroschisis is associated with major congenital abnormalities
<C>The postoperative mortality of surgery for gastroschisis approaches 50%
<D>Both conditions may be diagnosed prenatally with ultrasound
<E>Both conditions require delivery by caesarian section

ANSWER:D

<81>Hirschsprung's disease, all are true, Except:


<A>Often presents with neonatal large bowel obstruction
<B>Results from absence of ganglion cells in both the Meissner's and Auberbach's
plexus
<C>A contrast-study will show contraction of the aganglionic segment
<D>The diagnosis can be confirmed by histological evidence of reduced
acetylcholinesterase in the aganglionic segment
<E>Early treatment may involve rectal irrigation or an emergency colostomy

ANSWER:D

<82>Wilms' tumor, the most common manifestation is:


<A>Abdominal mass
<B>Loss of weight
<C>Hematuria
<D>Persistence crying of the child
<E>Pathologic fracture of bone

ANSWER:A

<83>Skin cancer: The most common Is:


<A>Squamous cell carcinoma (SCC)
<B>Basal cell carcinoma (BCC)
<C>Malignant melanoma (MM)
<D>Adenocarcinoma
<E>Sarcoma

Dr. wessam alzaidat - General surgery


ANSWER:B

DR.WBZ
<84>Split Skin Graft (SSG) differs from Full Thickness Graft (FTG) in that:
<A>SSG contains no dermal elements
<B>SSG provides more flexibility and strength
<C>SSG develop deeper pigmentation following transfer
<D>SSG is more acceptable in appearance
<E>SSG fails more readily if recipient bed conditions are sub optimal

ANSWER:C

Dr. wessam alzaidat - General surgery


<85>Burn: Partial Thickness Burn (PTB): All are true, Except:
<A>Usually heals without grafting

Dr. wessam alzaidat - General surgery


<B>May deteriorate into FTB (Full Thickness Burn)
<C>May cause severe physiologic derangement of the patient
<D>The characteristic sign of PTB is blister formation
<E>Usually heals with severe scarring if not excised and grafted

ANSWER:E

<86>A 55-year-old woman gives a history of tiredness, aching, and a feeling of


heaviness in the left lower leg for the past 3 months. These symptoms are relieved
by leg elevation. She is also awakened frequently by calf and foot cramping, which
is relieved by leg elevation, walking, or massage. On physical examination there
are superficial varicosities, nonpitting edema, and a slightly painful, 2 cm. diameter
superficial ulcer 5 cm. above and behind the left medial malleolus. What is the most
appropriate diagnosis?
<A>Isolated symptomatic varicose veins
<B> Superficial lymphatic obstruction
<C>Deep venous insufficiency
<D>Arterial insufficiency
<E>Incompetent perforating veins

ANSWER:C

<87>Abdominal aortic aneurysm (AAA): The incidence is highest among patients


with which of the followings:
<A>Femoral aneurysm
<B>Thoracic aortic aneurysm
<C>Bilateral popliteal aneurysm
<D>Aortoiliac occlusive disease
<E>Subclavian artery aneurysm

ANSWER:C

<88>Arteriovenous fistula (congenital of the lower limb): You would suspect it if


all the following are present, Except:
<A>Unusual varicosities were present in an enlarged limb
<B>Painful ulcer present with normal foot pulses
<C>Occlusion of the femoral artery leads to tachycardia
<D>The pulse pressure is great
<E>Machinery murmur is heard

ANSWER:C

<89>Carotid boy tumor: All are true, Except:


<A>It is usually unilateral

Dr. wessam alzaidat - General surgery


<B>Usually painful
<C>Arteriography is a useful diagnostic tool
<D>Characteristically the tumor moves from side to side and not vertically
<E>Radiotherapy has no effect on the tumor

ANSWER:B

<90>Fat embolism, all are true, Except:


<A>It is relatively common but only rarely causes symptoms
<B>Fat particles are present in the pulmonary bed in 90% of long bone fracture
<C>Petechiae of the limbs
<D>Fat droplets in sputum and urine
<E>For well established cases positive pressure ventilation is an essential line of
treatment

ANSWER:C

Dr. wessam alzaidat - General surgery


DR.WBZ

Dr. wessam alzaidat - General surgery


<91>Leriche's syndrome: results from obstruction of:
<A>Common iliac artery
<B>Internal iliac artery
<C>Femoral artery
<D>Aorto-iliac junction
<E>Axillary artery

ANSWER:D

<92>Lymphedema (Secondary): includes by all of the followings, Except:


<A>Lymphogranuloma
<B>Surgery
<C>Radiation therapy
<D>Milroy's disease
<E>Trauma

ANSWER:D

<93>Volkmann's ischemic contracture occurs most commonly:


<A>In adults
<B>After radial artery cannulation
<C>In collagen vascular disease
<D>Following supracondylar fracture
<E>In thoracic outlet compression syndrome

ANSWER:D

<94>White clot syndrome: One of the following is True:


<A>Presence of massive thrombocytopenia
<B>Seen in DVT of calf veins
<C>Finding of effort-vein thrombosis
<D>Treated by heparin
<E>Thrombectomy is the treatment of disease

ANSWER:A

<95>Achalasia: One of the following statements is True:


<A>In most Jordanian cases the cause is a parasitic infestation by Trypanosoma
cruzi
<B>Chest pain and hematemesis are the usual symptoms
<C>Distal-third esophageal adenocarcinomas may occur in as many as 20% of
patients within 10 years of diagnosis
<D>Manometry demonstrates failure of LES relaxation on swallowing
<E>Endoscopic botulinum toxin injection of the LES, provide highly effective
Dr. wessam alzaidat - General surgery
curative therapy

ANSWER:D

DR.WBZ

<96>Assessment of gastroesophageal reflux disease by manometry: All are true,


Except:
<A>Absent or extremely low LES pressures have predictive value in identifying
more severe reflux
<B>Peristaltic dysfunction increases with increasing severity of esophagitis
<C>With established reflux disease the upper esophageal sphincter is hypertensive
<D>Esophageal functional changes are worst in patients with a circumferential
columnar-lined esophagus
<E>Absence of peristalsis may be associated with more severe forms of reflux
disease

ANSWER:C

DR.WBZ

Dr. wessam alzaidat - General surgery


<97>Esophageal leiomyomas: One of the followings is True:
<A>The majority are diagnosed after they cause dysphagia and chest pain
<B>Biopsy is indicated at the time of esophagoscopy, to rule out carcinoma
<C>Full-thickness elliptical excision of the esophageal wall is the preferred
surgical approach
<D>Endoscopic ultrasonography is a reliable means of following leiomyomas
conservatively
<E>Recurrence of resected leiomyomas is minimized by wide local excision

ANSWER:D

<98>Esophageal varices: Which of the following treatments for bleeding varices


best preserves portal blood flow to the liver?
<A>Side-to-side portocaval shunt
<B>Transjugular intrahepatic portosystemic shunt (TIPS)
<C>Distal splenorenal shunt (DSRS)
<D>Interposition mesocaval shunt with 16-mm Dacron graft
<E>Autologous internal jugular vein interposition mesocaval shunt

ANSWER:C

<99>When a stricture is present in association with gastroesophageal reflux, each


of the following is an acceptable repair for reflux control, Except one. Identify the
poorest repair:
<A>Intrathoracic total fundoplication
<B>Lengthening gastroplasty with total fundoplication
<C>Total fundoplication
<D>Lengthening gastroplasty with partial fundoplication
<E>Partial fundoplication

ANSWER:E

Dr. wessam alzaidat - General surgery


JUH General Surgery Exam
Senior exam, 14th May 2020

GI:
1- Patient post laparoscopic gastric bypass with retrocolic anastomosis, presented after 5 years
with intestinal obstruction, what is the most likely cause:
- Internal hernia*
- adhesions
- jejunojejunal intussusception at site of anastomosis

2- Patient post laparoscopic sleeve gastrectomy, presented with nausea and vomiting 2 weeks
after surgery, gastrografin swallow showed hourglass deformity, what is next in management:
- Convert to RYGB

3- Patient post laparoscopic gastric bypass discharged on day 1, came to ER on day 5 post op
complaining of abdominal pain, diarrhea, fever, tachycardia, what would you do:
- do abdominopelvic CT with IV and oral contrast
- send to OR for diagnostic laparoscopy

4- Difference between TEP and TAPP in hernia repair, TEP has:


- less risk of visceral injury
- less adhesions
- less cost
- shorter hospital stay

5- Pt underwent right inguinal hernia repair with mesh 4 years ago via open approach, presented
with recurrent bulge at site of inguinal area, he has also undergone lap chole, retropubic
prostatectomy. What is best for management:
- TAPP repair
- open repair
- TEP repair
- observation, watchful waiting

6- Pt underwent laparoscopic ventral hernia repair, presented with recurrence 2 months later.
What is the most likely cause:
- failure of mesh fixation
- inadequate facial adhesiolysis
7- Pt underwent primary colonic anastomosis post colonic resection, air leak test done at end of
procedure showed large defect at site of anastomosis, what would you do:
- primary repair of anastomosis
- primary repair with diversion stoma

Dr. wessam alzaidat - General surgery


- insertion of drain at site of anastomosis
8- Question about causes of colonic anastomotic failure
9- Solitary rectal ulcer, true:
- treated with Sucralfate enema
- associated in 80% on cases with rectosigmoid intussusception

10- Siewert type 3 adenocarcinoma in the cardia of stomach requires how much resection:
- total gastrectomy with possible esophagectomy
11- Question about common Bile duct injury management
12- Pyogenic liver abscess, false:
- most common nowadays occurs in females with endocarditis
13- Splenic abscess is associated with adhesions with which of the following:
- left hemidiaphragm
- stomach
- tail of pancreas*
14- Question about Fenestrated cholecystectomy vs Reconstituting cholecystectomy
15- About component separation in ventral hernia repair, false:
- violation of semilunar line is associated with iatrogenic spigelian hernia
- incision done 0.5-1 cm lateral to semilunar line
16- Extended right hemicolectomy for management of splenic flexure tumor, false:
- anastomosis between distal ileum and sigmoid
- ligation of right colic, middle colic and left colic
- no difference between it and left hemicolectomy
17- Pancreatic duct injury diagnosis, false:
- can be diagnosed with intra operation visualization of pancreases without performing
pancreatecography
18- Question about CBD stricture:
- ERCP to visualize porta hepatis mass
19- Question about Gastric cancer, diffuse intestinal type
20- Question about Gastric cancer, hereditary type
21- What would decrease incidence of parastomal hernia:
- application of mesh
- stoma brought up from rectus muscle
- suturing of serosa to peritoneum
22- Pt presented with small bowel obstruction, and pain radiating to inner thigh, diagnosis:
- Obturator hernia
- femoral hernia
- indirect inguinal hernia
23- Pt with Ascites refractory to medical management presented with mildly symptomatic
umbilical hernia, what would you do:
- arrange for peritoneal tapping just before umbilical hernia repair
- repair with mesh
- performing TIPS before surgery for treatment of ascites
24- Question about Cholestatic jaundice
25- 60-year-old male, undergoing lap chole for symptomatic GBS, when you entered the abdomen
you noticed an incidental meckel's diverticulum, what would you do:
- observation
- segmental resection
- diverticulectomy
26- Question about Antegrade colonic irrigation
27- Perianal fistula, trans sphincteric management:
- cutting seton

Dr. wessam alzaidat - General surgery


- non cutting seton
- advancement flap
- PLIS
28- Question about Chronic mesenteric ischemia
29- Cholesterol GB stones, false:
- accumulation of cholesterol in the smooth muscle of gallbladder result in motility disorder
30- Cecal volvulus, true:
- colonoscopic deflation is successful in majority of cases
31- Question about presentation of Acute thrombotic mesenteric ischemia
symptoms with percentages!
32- Loss of abdominal domain, false:
- forms a secondary abdominal cavity
- The diameter of the hernia defect is generally greater than 10 cm
- 40% or greater of the abdominal contents are located outside of the abdominal cavity
- volume of sac/volume of abdominal cavity >25%
- associated with higher incidence of visceral injury
33- Gallbladder polyp, true:
- when dx suspected pre op lap chole is not a valid option
34- About gastric outlet obstruction palliative management surgical gastrojujnosotmy vs stent
35- Pt with FAP presented with mass on abdomen, diagnosis:
- desmoid
36- Difference between Lap vs open hemicolectomy, true:
- faster recovery of ileus
- decreased hospital stay
- lower wound infection
- lower adhesion
37- OPSI, false
38- Rise in ETCO2 and airway pressure while dissecting hernia sac in lap paraesophageal hernia
repair, what is your management:
- increase minute ventilation
- deflate capnothorax with chest tube
39- 33-year-old female admitted with Active colitis to medical floor not responding to medical
treatment on day 3, all false in management except:
- it is safe to continue on IV steroids
- rescue surgery should not be discussed yet
- stoma nurse should not visit patient yet
40- Question about Biliary pancreatitis:
- lap chole indicated in same admission
41- Obstructing sigmoid cancer, all valid option for management except:
- loop ileostomy
- transverse colostomy
- Hartman colostomy
- resection with primary anastomosis
42- Fistulizing perianal crohns disease
43- Primary sclerosing cholangitis, false:
- ERCP shows distal stenosis with proximal dilatation
44- Colonic pseudopolyp
45- Definitive diagnosis of TB peritonitis:
- culture of peritoneal fluid
- diagnostic lap and peritoneal biopsy
- percutaneous peritoneal biopsy
46- Clostridium Diff manamgement

Dr. wessam alzaidat - General surgery


Critical care and trauma:
47- Which of the following decrease mortality in patients with ARDS:
- prone ventilation
- low tidal volume ventilation
48- Facial trauma, false:
- subconjunctival hemorrhage indicate basal skull fracture*
- periauricular ecchymosis indicate basal skull fracture
- hemotympanum indicate basal skull fracture
- impaired lateral gaze indicate lateral orbital trauma
49- Retroperitoneal hematoma, true:
zones … penetrating and blunt trauma … mx
50- Neck penetrating injury zone 2, stable pt with active pulsatile bleeding, mx:
- wound exploration at ER
- wound exploration at OR
- angiography
- bronchoscopy and esophagoscopy
51- Splenic trauma requiring surgical management:
- active extravasation of contrast in CT
- associated liver trauma grade 2
52- Indicator of adequate perfusion on CPB:
- perfusion pressure
- mixed venous saturation
- urine output
53- Indication of tranexamic acid in trauma patient:
- any trauma patient with significant bleeding within 6 hours of injury
54- Pt has right sided chest tube due to hemothorax post RTA, developed sudden collapse, jugular
venous distension and tachycardia with desaturation, tracheal midline, normal heart sounds,
chest tube functioning, what is diagnosis:
- massive PE
- cardiac tamponade
- tension pneumothorax
- massive hemothorax
55- Question about Oxygen dissociation curve

CVT:
56- Pt post small cell lung cancer lobectomy, in floor developed nonfunctioning chest tube and
stopped auscultation, what would be the cause:
- clogged tube with fibrin
- resolution of pneumothorax
- full lung expansion
- tension pneumothorax
57- All cause chylothorax except:
- mediastinitis
- central venous line insertion
- trauma
58- At the recovery room, you where called to check pt 1 hour post Right CEA showing signs of left
stroke. What would be the cause:
- cardiac embolus
- technical error*
- hyperperfusion
Dr. wessam alzaidat - General surgery
- watershed stroke
59- Post TEVAR (thoracic endovascular aortic repair) with spinal shock and hypotension,
management:
- dexamethasone
- IV resuscitation and elevation of BP
- return to OR
60- Cause of decrease BP after removal of aortic clamp in open AAA repair:
- decrease vascular resisitance
61- Sexual impotence post AAA bilateral iliac graft repair:
- extensive dissection at left common iliac vessels*
- extensive dissection at right common iliac vessels
- extensive dissection at inferior end of abdominal aorta opposite sacrum
62- To visualize internal carotid artery you have to do all except:
- mobilization of hypoglossal nerve
- removal of styloid process
- cut occipital artery
- cut posterior belly of digastric muscle
63- CEAP classification that defines healed venous ulcer:
- C5
64- Reperfusion injury indicating fasciotomy can be predicted by:
- history of ischemia more that 6hrs
65- Dobutamine, false:
- decrease heart oxygen requirement
- work of B1 and B2
- low dose is defined as 5 /hr
66- Weaning from ventilator, false:
- continued requirement of vasopressor prevent weaning
- no clear consensus on criteria for weaning
- pt must be able to initiate breathing
- defined as complete independency from mechanical ventilator
67- In 60-year-old male requiring bioprosthetic valve replacement of aortic valve, what is the
benefit over mechanical valve:
- less paravalvular leak
- less requirement for anticoagulation
- suitable for valve in valve replacement
- Less risk of infection
Breast:
68- All the following are examples of displacement of tissue flaps, except:
- Reduction mastopexy lumpectomy*
- Batwing mastopexy lumpectomy
- Daunt mastopexy lumpectomy
- Latissimus dorsi muscle flap
- TRAM flap
69- About Types of IDC, false:
- mucinous and tubular are the most common
70- Next management for 2.3 cm breast cancer with negative SLNBx, ER PR positive after
undergoing lumpectomy:
- radiotherapy and hormonal therapy only
Plastic:
71- True about lower Lip SCC 1.5 cm:
Dr. wessam alzaidat - General surgery
- least safety margin is 4 mm
- defect may be closed with Abbe flap
72- True about Melanoma:
- 1.2 cm requires SLNBx
73- True about melanoma:
- stage 3 a carries better prognosis than stage 2c
74- Phosphorus burn management is irrigation with:
- normal saline*
- mineral oil
- calcium gluconate
75- Question about oral cancer …
76- Female lady with breast cancer right sided at upper outer quadrant, size 2.3 cm, 1.5 cm away
from nipple, pathology IDC Her-2 negative, with negative axilla. Came to your clinic requesting
bilateral nipple sparing mastectomy;
- contraindicated due to tumor location
77- Post CABG in which LIMA was used, pt came with discharge at sternal wound site and fever, CT
done which showed Sternal sinus, after debridement of sternal wound and removal of sternal
wire, what is next in management:
- RIGHT pec major muscle flap
- LEFT rectus muscle flap
- negative wound therapy
- omental flap

pediatrics
78- True about hirshprung disease:
- more common in females
- usually present with delayed passage of meconium more than 24 hours
- majority of cases involve area from anorecal line upwards
79- Pyloric stenosis, false:
- perforation at site of myotomy requires closure of defect and re-opening 90-180 degrees
80- Wandering spleen, true:
- surgical treatment include fixation in extraperitoneal space
- usually present with painless abdominal mass
- low likelihood of splenic torsion and infarction
81- About Wilms tumor, true:
- majority of cases are unilateral unifocal
82- All the following don’t indicate non-intentional pediatrics trauma except:
- bruises at different stages of healing
- thoracic cage trauma
- when the care giver blame siblings and friends for trauma
- contact burn with regular shapes
83- All the following true about empyema except:
- majority treated with antibiotics and chest tube drainage in stage 1
- caused by pseudomonas
84- Radiological feature of meconium ileus:
- bubbly or soap appearance
- diffusely dilated colon with no air fluid levels
85- About Nissen fundoplication in pediatrics, false:
- indicated in pts with GERD and un-explained apneic spills
- for pts with neurological defect on gastric feeding to decrease risk of aspiration

Dr. wessam alzaidat - General surgery


Endocrine:
86- All the following structures are encountered during left lap adrenalectomy, except:
- 4th part of duodenum
- colon
- stomach
- tail of pancrease
- left kidney
87- What define Lateral neck dissection LN:
- dissection of level 2,3,4
- dissection of level 2,3,4,5
- dissection of level 1,2,3,4
88- Which of the following could be an Indication for thymectomy:
- hyperparathyroidism due to two parathyroid adenoma
- gravis
- goiter with retrosternal extension
89- PTC micropapillary next step in mx after thyroidectomy
- do nothing
- arrange for neck dissection
- radioactive iodine
90- False about PTC:
- positive LN mets in central neck dissection in low risk pts is a poor prognosis
91- All the following about Parotid gland anatomy is true, except:
- facial nerve runs deep to its vessels*
- deep lobe accounts for 10-15% on gland
- invested with deep fascia
92- False about Salivary gland anatomy
93- In a B.K.Amputation, what is the length of tibia bone cut from tibial plate:
10-12 cm*
14-16 cm
94- All the following are worrisome features of thyroid nodule on ultrasound imaging except:
- honeycomb *
- echogenic halo
- microcalcification
- solid component
95- Pt with conns syndrome, found to have bilateral adrenal adenoma, what would you do to aid in
localizing of functioning adenoma:
- bilateral venous sampling
Biostat:
96- You are provided with a summary of a study, asked to calculate Incidence proportion rate
= new cases / person-time
97- A relative risk of 12 means that:
- there is an increased risk of disease with exposure to risk factor
- there is a decreased risk of disease with exposure to risk factor
- no significant association

Total of 100 questions, time 2 hours


The END

Dr. wessam alzaidat - General surgery


Dr. wessam alzaidat - General surgery
Mcqs for residents exam 2
1. Which of the following is true of crush syndrome?
a. Is also referred to as traumatic squeeze
b. The urine is clear
c. The serum calcium level is usually elevated
d. Sensory and motor function is preserved in compressed limb
e. Serum creatinine, potassium and phosphate are elevated
Answer E
2.Which of the following incomplete spinal cord injury syndromes has the most potential
for recovery:
A. Anterior cord syndrome
B. Central cord syndrome
C. Brown-Sequard syndrome
D. Posterior cord syndrome
E. Pyramidal syndrome
Answer C
3. Which of the following inflammatory mediators has been most closely associated with
the magnitude of the inflammatory response to blunt trauma and with the development of
multiple organ dysfunction syndrome (MODS)?
a. Interleukin-1 (IL-1).
b. Beta human chorionic gonadotrophin (ß-HCG).
c. Tumour necrosis factor beta (TGF-ß).
d. Tumour necrosis factor alpha (TNF-a).
e. Interleukin-6 (IL-6).
Answer E
4. Which of the following does NOT lead to a thrombotic tendency?
A. Factor V Leiden
B. Antithrombin deficiency
C. Protein C and S deficiency
D. Heparininduced thrombocytopenia
E. Anti-Protein Z deficiency
Answer E
5. What is the risk of HIV seroconversion from a contaminated needlestick?
A 0.1%
B 0.3%
C 0.5%
D 1%
E 3%
Answer B

Dr. wessam alzaidat - General surgery


6. Regarding gastric acid secretion, which is INCORRECT?
A. Cephalic phase of acid secretion is vagal dependent
B. Cephalic phase accounts for 60% of total acid secretion
C. Gastric phase begins when food reaches the stomach
D. Gastrin is the main component of the gastric phase
E. Gastrin, histamine and acetylcholine stimulate parieta
Answer B

7. Regarding Von-Willbrand disease, which of the following is INCORRECT?


A. VW disease is the most common congenital bleeding disorder
B. Is classified into three types
C. Type I does not respond to desmopressin therapy
D. Type II may respond to desmopressin therapy
E. Menorrahagia is common in female with VW disease
Answer C
8. Regarding the effect of aspirin on platelets, which is CORRECT?
A. Aspirin irreversibly induces thrombocytopenia
B. Effect of Aspirin lasts for 21 days
C. As furosemide, it inhibits ADP dependent platelets aggregation
D. Aspirin irreversibly inhibits platelets COX
E. Bleeding time is not affected
Answer D

9. The abdominal part of esophagus receives its blood supply from:


A. Direct branch from the aorta
B. Branches from the bronchial artery
C. Inferior phrenic artery
D. Right gastric artery
E. Right gastroepiploic artery
Answer C
10. The most potent stimulus for gastrin release is:
A. Acetylcholine
B. Histamine
C. Somatostastin
D. Hydrochloric acid
E. Luminal peptides and amino acids
Answer E

Dr. wessam alzaidat - General surgery


11. All of the following hemodynamic parameters occur during CO2 pneumoperitoneum,
EXCEPT:
A. Decrease in pulmonary vascular resistance
B. Decrease in cardiac preload
C. Increase in systemic vascular resistance
D. Increase in heart rate
E. Decrease in venous return
Answer A
12. All of the following are signs of portal hypertension EXCEPT:
A. Ascites
B. Esophageal varices
C. Thrombocytosis
D. Rectal varices
E. Caput medusa
Answer C
13. The most common geriatric cervical fracture is:
A. Jefferson fracture
B. Hangman fracture
C. Clay shoveler’s fracture
D. Odontoid fracture
E. Tear drop fracture
Answer D
14. According to pack – year quantification of cigarette smoking if the patient smoking 16
cigarettes per day for 10 years that means:
A. 1 Pack year
B. 4 pack year.
C. 8 pack year
D. 16 pack year
E. 32 pack year
Answer C
15. All are major criteria for fat embolism except
A. Axillary or subcojunctival petechiae
B. Hypoxemia
C. CNS depression
D. Pulmonary edema
E. Urinary fat globules
Answer E
16.Regarding acute osteomyelitis, the most common site involved is:
A. Upper and femur
B. Upper tibia
C. distal femur
D. Pelvis bone

Dr. wessam alzaidat - General surgery


E. Forearm.
Answer C
17.The P value refers to:
A. The chance of the study demonstrating a significant association when, in reality, there
is no significant association.
B. The chance of the study demonstrating no significant association when, in reality,
there is a significant association.
C. The strength of the association between two variables.
D. The power of the study to demonstrate valid findings.
E. The chance of the study demonstrating a significant association when, in reality, there
is a significant association.
Answer D
18.Which of the following is NOT absolute contraindication for contrasted MRI?
A. Programmable shunt valve
B. Pregnancy.
C. Placement of a vascular stent, coil or filter within the past 6 weeks.
D. Swann-Ganz catheter (pulmonary artery catheter)
E. Cardiac pacemakers, implanted neurostimulators, cochlear implants, infusion.
Answer B
19. A pelvic hematoma might damage:
A. Lateral femoral cutaneous nerve
B. Femoral nerve
C. Obturator nerve
D. Sciatic nerve
E. Iliohypogastric nerve.
Answer B
20.What is the main role of selective arterial embolization
A. Substantial reduction of tumor vascularity , making possible and safer tumor excision
B. Reducing tumor mass
C. Visualization of the angiographic pattern of the disease
D. Reduce epidural bleeding
E. Visualize the surrounding tissue
Answer A
21.The intracranial tumor most likely to be encountered in a middle-aged man with the
acquired immunodeficiency syndrome (AIDS) is
A. Glioblastoma multiforme.
B. Lymphoma.
C. Meningioma.
D. Oligodendroglioma.
E. Ependymoma.
Answer B

Dr. wessam alzaidat - General surgery


22. Which of the following statements is true regarding peripheral nerve injuries?
A. Neuropraxia is temporary loss of function without axonal injury; structure damage
occur in most cases.
B. Axonotmesis is disruption of the axon and axon sheath associated with traumatic
injury.
C. Neurotmesis is disruption of the axon with preservation of the axon sheath which
usually preserves sensory and motor function.
D. Electromyography (EMG) is useful in the early assessment of nerve injuries.
E. Regeneration in a peripheral nerve occurs at a rate of 1 mm/day, so improvement may
not be obvious for many months.
Answer E
23.The following relate to the half-life of different coagulation factors except:
A. Half-life of Factor VII is less than 5 hours.
B. Half-life of Factor V is around 25 hours.
C. Half-life of Factor VIII is around 10 hours.
D. Half-life of Factor IX is around 25 hours.
E. Half-life of Factor XI is around 45 hours.
Answer B
28. Prothrombin Time (PT) is prolonged in:
A. Severe thrombocytopenia
B. Hemophilia A
C. Christmas Disease
D. Von Willebrand Disease
E. Over anticoagulation with Warfarin.
Answer E
29.Once thawed, fresh frozen plasma must be transfused within:
A. 4 hours
B. 8 hours
C. 12 hours
D. 24 hours
E. 1 hour
Answer D

THE END

DR.WBZ best of luck

Dr. wessam alzaidat - General surgery


MCQS JORDAN BOARD

ANSWER AT THE END

1.which of the following amino acids is a critical nutrient supporting the immune
function:
a) Taurine.
b) Branched chain amino acids (BCAA).
c) Alanine.
d) Arginine.
e) Tyrosine.

2. The optimal initial treatment of aspiration pneumonia is:


a) Bronchoalveolar lavage with antacids.
b) Bronchoalveolar lavage with saline.
c) Parenteral corticosteroids.
d) Maintenance of oxygenation.
e) Early prophylactic antibiotics.

3. All are true about ultrasound, EXCEPT:


a) Can detect a lesion less than 1 cm in the liver.
b) Is useful about differentiating a thyroid cyst.
c) It helps in the diagnosis of acute appendicitis.
d) Its not the best modality to visualize the pancreas.
e) Investigation of choice in GB stones.

4. Which statement is correct regarding neurapraxia:


a) The injury involves the axon without loss of continuity of nerve sheath.
b) Momentary loss of conduction through the nerve without any organic
lesion.
c) Partial or complete division of the nerve.
d) Spontaneous recovery is impossible.
e) Wallerian degeneration occurs.

Dr. wessam alzaidat - General surgery


5. The most important factor in the development of pressure ulcers in a spinal
cord injury patient is:
a) Malnutrition.
b) Septic episodes.
c) Anemia.
d) Local pressure.
e) Diminished sensation.

6. The earliest sign of Volkmann's ischemic contracture is:


a) Absence of the radial pulse.
b) Pain on passive finger extension.
c) Loss of voluntary finger motion.
d) Absent sensation in the median nerve distribution.
e) Pallor of the nail beds.

7. Regarding the oxygen dissociation curve, one is TRUE:


a) Is shifted to the left by increase 2,3-DPG.
b) Is sigmoid shape for both hemoglobin and myoglobin.
c) Is shifted to the left when one ascends to high altitude.
d) Is shifted to the right by an increase in the PH.
e) Is ideally 50% saturated at a PO2 of 28mmHg.

8. Regarding insulin, all are true EXCEPT:


a) It is a polypeptide composed of two amino acid chains.
b) Its secretion is affected by glucagon.
c) It stimulates gluconeogenesis.
d) It has little or no effect on the uptake of glucose in the brain.
e) If lacking, it may be a cause for fatty liver.

9. Which of the following statements regarding hemorrhoids is NOT TRUE:


a) Hemorrhoids are specialized "cushions" present in every one to aid
continence.
b) External hemorrhoids are covered by skin, while internal hemorrhoids are
lined by mucosa.
c) Pain is often associated with uncomplicated hemorrhoids.
d) Haemorrhoidectomy is reserved for third and fourth degree hemorrhoids.
e) They are classified and treated according to the degree of their symptoms.

Dr. wessam alzaidat - General surgery


10. Infection affects wound healing by all the following mechanisms EXCEPT:
a) Prolonging oedema.
b) Decreasing tissue PO2.
c) Increasing collagenolysis.
d) Decreasing the inflammatory phase.
e) Increasing the inflammatory phase.

11. All the followings may be linked to the etiology of malignant melanoma
EXCEPT:
a) Fair complexion.
b) Intermittent sun exposure.
c) Positive family history.
d) Junctional nevus.
e) Dysplastic nevus.

12. The most reliable indicator of the adequacy of burn resuscitation is:
a) Central venous pressure.
b) Pulmonary capillary wedge pressure.
c) Urine output.
d) Blood pressure and heart rate.
e) Mental status.

13. The most common site for squamous cell carcinoma of the lip is the:
a) Upper lip midline.
b) Upper lip laterally.
c) Lower lip midline.
d) Lower lip laterally.
e) Oral commissure.

14. As the amount of total body fat increase, the total body water:
a) Increases.
b) Decrease.
c) Unchanged.
d) Depends on patient's age.
e) Depends on patient's sex.
15. One week of starvation results in a loss of approximately 100 gm of nitrogen
(13.8 gm N/ day). This is associated with protein loss of:
a) 625 gm.
b) 100 gm.
c) 1000 gm.
d) 50 gm.
e) 16 gm.
Dr. wessam alzaidat - General surgery
16.The HIGHEST potassium content is in:
a)Saliva.
b)Gastric juice.
c)Bile.
d)Ileal.
e- Duodenal

17.During vessel wall injury, the initial step in clot formation is:

a)Local mast cells release of adenosine diphosphate (ADP).


b)Fibrin polymerization.
c)Plasminogen activation.
d)Platelet adherence to sub endothelial collagen.
f)None of the above.
16. Postoperative staphylococcal wound infection usually detected in:
a) 1st to 2nd day.
b) 3rd to 4th day.
c) 5th to 8th day.
d) 9th to 14th day.
e) After the 14th day.
17. A palpable radial pulse indicates a systolic blood pressure greater than:
a) 40 mmHg.
b) 60 mmHg.
c) 80 mmHg.
d) 100 mmHg.
e) 120 mmHg.
18. What is the first parameter to change in early hypovolemic shock?
a) Systolic blood pressure.
b) Pulse rate.
c) Diastolic blood pressure.
d) Respiratory rate.
e) Level of consciousness.
19. A previously healthy 18 years old man involved in automobile accident. He lost
consciousness, but regains it and appeared normal. An hour later he became
somnolent and has weakness on his right side, with dilated pupil on the left
side. Most likely he has:
a) Acute epidural hematoma.
b) Acute subdural hematoma.
c) Subarachnoid hemorrhage.
d) Intraventricular hemorrhage.
e) Brain concussion.

Dr. wessam alzaidat - General surgery


20. Concerning factors that increase the risk of pulmonary embolism, all are
correct EXCEPT:
a) Superficial phlebitis one year ago.
b) Estrogen therapy.
c) Obesity.
d) Pregnancy.
e) Deep venous thrombosis one year ago.

21. The TRUE statement regarding small intestine is:


a) The entire small intestine is intraperitoneal.
b) The jejunum is longer in length, larger in diameter, and thinner-walled than
ileum.
c) The muscularis, the muscle layer, provide strength for placement of sutures
for creation of a bowel anastomosis.
d) Peyer’s patches are most prominent in the distal ileum
e) The marginal artery of Drummond provides blood supply to the duodenum.

22. A 40 years old female had a subtotal thyroidectomy, several hours later she
complained of difficulty in breathing and she had stridor and a markedly
swollen, tense neck wound the first step in the management of this patient
should be to:
a) Intubate with an endotracheal tube.
b) Perform a tracheostomy.
c) Control the bleeding site in the operating room.
d) Open the wound and evacuate the hematoma at bed side
e) Aspirate the hematoma.

23. Patients with acute pancreatitis who are at increased risk to develop
complications include those with each of the following EXCEPT:
a) Ransons score> 3.
b) An APACHI II score >12.
c) Poorly enhancing areas of 50% of the pancreas on a dynamic CT scan.
d) Amylase values more than eight times upper limits of normal.
e) Partial obliteration of the lesser sac and acute fluid collections near the
splenic hilum and inferior to pancreatic head.

Dr. wessam alzaidat - General surgery


24. Regarding acute suppurative ( toxic ) cholangitis, one is TRUE :
a) Third generation cephalosporin are the antibiotic of choice for patients with
renal impairment.
b) Urgent decompression of the biliary tree is indicated.
c) Emergency surgical decompression is indicated for patients with known
cholilithiasis.
d) The catheter for percutaneous drainage should be placed through the
common duct into the duodenum.
e) Nasobiliary tube decompression is an effective form of drainage for
obstructing lesion in the upper third of the common duct.
25. The most common presenting symptom in colorectal carcinoma in patients
under age of 40 is:
a) Abdominal pain.
b) Weight loss.
c) Back pain.
d) Rectal bleeding.
e) Abdominal distension.

28-A 24 years old man has appendectomy, histopathological report notes acute
inflammation with a 1.2 cm carcinoid tumor on the mid of appendix. This patient
should have:

f) No further treatment.
g) Chemotherapy.
h) Right hemicolectomy.
i) Serial urinary 5- hydroxyindole acetic levels.
j) Regional radiation therapy.
29- Gastric ulcers are:
A- Malignant when located in the greater curvature.

B- More common in women.

C- Most commonly complicated by perforation.

D- Associated with hypersecretion of acid in over 90% of cases.

E- Not associated with Helicobacter pylori.

30- Which is the most commonly cultured hospital acquired organism in critical

Dr. wessam alzaidat - General surgery


care patients with aspiration pneumonia?

a- Streptococcus pneumoniae

b- Staphylococcus aureus

c- Anaerobic species

d- Pseudomonas aeruginosa

e- Haemophilus influenzae

31- Which of the following will immediately delay or cancel an elective surgical
case if not obtained appropriately preoperatively?

a- CBC

b-Urinanalysis

c- CXR

d- Informed consent

e- ECG

32- The most common cause of nipple discharge is

a- carcinoma

b- fibrocystic disease

c- intraductal papilloma

d- duct-ectasia

e- trauma

33- ALL OF THE FOLLOWING ARE SINGS OF ATROPINE POISONING EXCEPT

a) dry mouth
b) red face
c) small pupils
d) tachycardia
e) drowsiness

Dr. wessam alzaidat - General surgery


34- PAIN IS CARRIED TO THE BRAIN BY THE FOLLOWING TRACT

a) spino cerebellar tract


b) spino thalamic tract
c) cortico- spino tract
d) reticular tract
e) olvospinal tract
35- A CETYLE CHOLINE IS THE CHEMICAL TRANSMITTER IN

a) all pre and post ganglionic sympathetic system


b) all pre and post ganglionic parasy pathetic system
c) all post ganglionic sympathetic system
d) in adrenal medulla
e) all pre ganglionic sympathetic system

36- ALL OF THE FOLLOWING AFFECTING NERVE CONDUCTION


EXCEPT:

a) myelination of the nerve


b) nerve fiber diameter
c) hypoxia
d) autonomic nervous system
e) inter nodal distance

37- THE MOST HELPFULL CLINICAL SIGN FOR SPINAL CORD TUMOUR AT T10 LEVEL IS:

a) exaggerated of knee jerk


b) anesthesia up to umbilicus level
c) urine retention
d) anesthesia up to hypogastrium
e) dissociated sensory loss

38- THE EARLY TREATMENT OF CERVICAL SPINES INJURY INCLUDES ALL OF THE FOLLWING
EXEPT:

a) immobilization by neck collar


b) the patient should be covered
c) lateral spine x-ray while patient on stretcher
d) change the patient position to take off his clothes in E.R
e) treatment of spinal shock
39- THE MOST IMPORTANT FUNCTION OF INTERVERTEBRAL DISC IS:

a) Preserve spinal curvature


Dr. wessam alzaidat - General surgery
b) Protects spinal cord
c) Shock absorber
d) Transmit weight of the body to the lower limbs
e) Has no significant function on adult age
40 – BLOOD SUPPLY OF SPINAL CORD IS USUALLY COME FROM :

a) vertebral arteries only


b) ant , post spinal arteries and radicular arteries
c) vertebral arteries and abdominal aorta
d) vertebral arteries and ascending cervical arteries only
e) radicular arteries from the aorta only
41- FRACTURE OF THE L 4 VERTEBRAE WITH LOWER LIMBS WEAKNESS IS DUE TO :

a) spinal cord contusion at that level

b) only dura injury

c) dura and nerves injury

d) spinal cord , dura and nerves injury

e) spinal concussion without any nerve injury

42 – BARO RECEPTORS IS LOCATED IN :

a) skin

b) wall of the great vessels in the neck

c) heart

d) in the brain at the medulla level

e) in the vessels of the brain

43- THE MOST SERIOUS COMPLICATION WHICH AFFECT THE SEQUALE OF HEAD INJURY IS
DUE TO:

a) brain infection and brain abscess

b) brain hypoxia

Dr. wessam alzaidat - General surgery


c) secondary bleeding

d) decrease intracranial pressure

e) CSF leakage

44-A LADY IN CHILDBEARING AGE WITH REPEATED PREGNANCES AND HAS POOR INTAKE OF
ANIMAL DIET IS PRONE TO:

a)Osteoporosis

b)Osteomalacia

c)Rickety bone

d)Vitamin B deficiency

e)Compressed spinal fractures


45- ALL OF THE FOLLOWING STRUCTURES PASSING UNDER THE FLEXOR RETINACULUM
EXCEPT:

a) Flexor Digitorum longus

b)Flexor Digitorum brevis

c)Median nerve

d)Flexor indices

e)Palmaris longus
46- THE ORIGION OF SYMPATHETIC SYSTEM FROM THE SPINAL CORD IS:

a)Cranio cervical outflow

b)Thoraco lumbar outflow


c)Cranio sacral outflow
d)Only lumbar outflow
e)Whole spinal cord
47-METABOLIC ACIDOSIS MAY BE BROUGHT BY:

a) loss of CO2 by increase ventilation


b) retention of CO2 by respiratory obstruction
c) persistent vomiting
d) absorption of excessive amount of Na HCO3
e) excessive exercise

Dr. wessam alzaidat - General surgery


48-THE MOST POTENT STIMULANT OF RESPIRATION WOULD BE:

a) a two fold increase in PCO2 of inspired air


b) a two fold increase in PO2 of inspired air
c) a 50% decrease in PCO2 of inspired air
d) a 50% decrease in PO2 of inspired air
e) a & d are equally potent stimuli
49-The psoas major muscle

a. flexes the thigh at the hip.


b. extends the thigh at the hip.
c. adducts the thigh at the hip.
d. abducts the thigh at the hip.
e. assists in full contraction of the diaphragm

50-The renal arteries typically branch from the abdominal aorta a the
level of the:

a. 12th thoracic vertebral body.


b. first lumbar vertebral body.
c. second lumbar vertebral body.
d. third lumbar vertebral body.
e. fourth lumbar vertebral body.

51-A 20–year–old man is undergoing retroperitoneal dissection for a


testicular germ cell tumor. The inferior mesenteric artery is divided
during reflection of the intestines to expose the retroperitoneum. This
can be expected to result in:

a. ischemia of the descending colon.


b. ischemia of the sigmoid colon.
c. ischemia of the rectum.
d. ischemia of the splenic flexture
e. none of the above.

52-The cremaster muscle is innervated by:

a. the ilioinguinal nerve.


b. the iliohypogastric nerve.
c. the obturator nerve.
d. the genital branch of the genitofemoral nerve.
e. the femoral branch of the genitofemoral nerve.

Dr. wessam alzaidat - General surgery


53-As one proceeds outward from the adrenal medulla, the three
separate functional layers of the adrenal cortex are, in correct order:

a. the zona reticularis, zona fasciculata, then zona glomerulosa.


b. the zona fasciculata, zona reticularis, then zona glomerulosa.
c. the zona glomerulosa, zona fasciculata, then zona reticularis.
d. the zona glomerulosa, zona reticularis, then zona fasciculata.
e. the zona reticularis, zona glomerulosa, then zona fasciculata.

54-Proceeding from posterior to anterior, the structures encountered in


the renal hilum are, in correct order:

a. the renal artery, renal vein, and renal pelvis.


b. the renal pelvis, renal artery, and renal vein.
c. the renal pelvis, renal vein, and renal artery.
d. the renal vein, renal artery, and renal pelvis.
e. the renal artery, renal pelvis, and renal vein.

55-During inguinal hernia repair in a male patient, the ilioinguinal nerve


is injured in the canal, which will most likely produce:

a. anesthesia over the dorsum of the penis.


b. anesthesia over the pubis and scrotum, and loss of cremasteric
contraction.
c. anesthesia over the pubis and anterior scrotum only.
d. anesthesia over the anterior and medial thigh.
e. anesthesia over the pubis only.

56-Hematuria is distinguished from hemoglobinuria or myoglobinuria


by:

a. dipstick testing.
b. the simultaneous presence of significant leukocytes.
c. microscopic presence of erythrocytes.
d. examination of serum.
e. evaluation of hematocrit.

57-Glucose will be detected in the urine when the serum level is above:

a. 75 mg/dl.
b. 100 mg/dl.
c. 150 mg/dl.
Dr. wessam alzaidat - General surgery
d. 180 mg/dl.
e. 225 mg/dl.

58-What proportion of the cardiac output is delivered to the kidney?

a. 5%
b. 20%
c. 50%
d. 85%
e. 100%

59-The predominant buffering system in humans is:

a. bicarbonate.
b. titratable acids.
c. ammonium (NH4+).
d. sodium lactate
e. phosphate

60-After a 7–hour–long, complex urethral reconstruction performed in


the extended lithotomy position, the patient has severe thigh and
buttock pain. The creatine phosphokinase levels are dramatically
elevated. To prevent ARF, the next step should be:

a. dopamine infusion.
b. plasmapheresis.
c. dobutamine infusion.
d. forced alkaline diuresis.
e. percutaneous nephrostomy.

61-The renal structure at greatest risk for ischemic injury is the:

a. vasa recta.
b. cortical collecting duct.
c. juxtaglomerular apparatus.
d. medullary thick ascending loop of Henle.
e. distal convoluted tubule.

Dr. wessam alzaidat - General surgery


62-Acute pyelonephritis is best diagnosed by:

a. chills, fever, and flank pain.


b. bacteriuria and pyuria.
c. focal scar in renal cortex.
d. delayed renal function.
e. vesicourethral reflux.

63-The most reliable early clinical indicator of septicaemia is:

a. chills.
b. fever.
c. hyperventilation.
d. lethargy.
e. change in mental status.

64-Which of the following is true regarding testosterone?

a. Testosterone is synthesized by the Sertoli cells of the testes.


b. Testosterone is synthesized by the Leydig cells of the testes.
c. Testosterone is a direct precursor of pregnenolone.
d. 5α–Reductase is an enzyme that converts DHT into testosterone.
e. Aromatase converts estrogens into testosterone

65-Calcium is maximally absorbed in which portion of the


gastrointestinal tract?

a. Stomach
b. Jejunum
c. Jejunum and proximal ileum
d. Ileum
e. Ascending colon

66- the most common type of congenital diaphragmatic hernia is caused by:

a. a defect in the central tendon


b. eventration of the diaphragm in the fetus
c. a defect through the space of Larry
d. a defect through the pleuroperitoneal fold.
e. all of the above.

67- The calorie-nitrogen ratio for infant should be maintained at:

a. 75:1

Dr. wessam alzaidat - General surgery


b. 100:1

c. 50:1

d. 150:1

e. 25:1

68- A patient has undergone an ileal resection . Which of the following


conditions would he be least likely to develop?

a- alopecia

b- megaloblastic anemia

c- nephrolithiasis

d- cholelithiasis

e- steatorrhea

69.With regard to Hirschsprung’s disease ( aganglionosis), all the following are


true except:

a- It is more common in males.


B. It may be complicated by enterocolitis.

C. Barium enema study may be normal.

D. It is best diagnosed by full thickness rectal biopsy.

E .Surgery can’t be accomplished without colostomy.

70. In regard to malignant hyperthermia, one is true.

A. It triggered by Depolarizing muscle relaxing agents


B. Non polarizing muscle agents are contra indicated .
C. Dantroline must be given after induction as prophylaxis.
D. Manitole and fluid support are not useful to prevent renal failure.
E. Alkalosis is the main metabolic disturbance.
71. Incompatible blood transfusion during surgery under general anesthesia,
which of the following is NOT true.

A. Unexplained bleeding.
Dr. wessam alzaidat - General surgery
B. Hematuria may be present.
C. Skin rash may be seen
D. Hypertension is well know problem in spite transfusion
E. Volume support is mandatory.
72. The most common cause of death in Paediatric age group is :

A. Infection
B. Trauma.
C. Malignancy.
D. Congenital anomalies.
E. Malnutrition.
73-In the management of pyloric stenosis, which of the following is true.

A. Acidosis should be corrected first.


B. Chloride level is normal in spite alkalosis.
C. Hypokalaemia should be corrected.
D. Usually there is compensatory hypocarbia.
E. Paradoxical aciduria is not a feature.

74 - The ligamentum teres represents an obliterated

a- Ductus venosus

b- Ductus arteriosus

c- Internal iliac artery

d- Umbilical vein

e- porta hepatis

75- Tributaries of the portal vein include all of the the following except:

a- Superior mesenteric

b- Para umbilical

c- uterine

d- pyloric

e- lienal ( splenic )

Dr. wessam alzaidat - General surgery


76- The following statements are true regarding the liver anatomy except :

a- Is attached to the diaphragm and anterior abdominal wall by the Falciform


ligament

b- Is totally covered by peritoneum

c- Drains by hepatic veins into the inferior vena cava

d- Has a lymph drainage to both the mediastinal and porta hepatic nodes

e- Is directly related to the right suprarenal gland

77- An animal is in negative nitrogen balance when :

a- The intake exceeds output

b- New tissue is being synthesized

c-Output exceeds intake

d- Intake is equal to output

e- The urine is nitrogen – free

78- Bilirubin secreted into the intestine is subjected to enzymatic degradation , the
final product being :

a- Biliverdin

d- Bilirubinogen

c- Urobilinogen

d- Stercobilin

e- Mesobilirubinogen

79- In mammals , Norepinephrine is synthesized from :

a- Pyruvate

b- Arginine

c- Catechol

d- Tyrosine

Dr. wessam alzaidat - General surgery


e- Tryptamine

80- Tissue graft within the same individual is known as :

a- Allograft

b- Isograft

c- Autograft

d- Xenograft

e- Non of the above

81- The most effective means of sterilization by heat is :

a- steam

b- Boiling

c- Hot air

d- Fractional sterilization

e- Steam under pressure

82- One of the following statements is true regarding Exotoxins:

a- Heat stable

b- Lipopolysaccharide in nature

c- part of cell wall of becteria

d- protein in nature

e- less potent than endotoxins

83- Gall stones associated with pernicious anemia are most likely to be :

a- pure cholesterol

b- Calcium bilirubinate

c- Calcium carbornte

d- Mixed gall stones

Dr. wessam alzaidat - General surgery


e- combined gall stones

84- In familial polyposis coli, polyps tend to appear most frequently :

a- At birth

b- During the first decade

c-during the second and third decades of life

d- During the fourth decades of life

e- After age of 40

DR.WBZ

85- The substance involved with protein synthesis

a- DNA

b- RNA

c- Adenine

d- Guanine

e- Cytosine

86- In surgical skin wound 80% of tensile strength of unwounded skin is restored
by :

a- 1 week

b-2 weeks

c- 4 weeks

d- 12 weeks

e- 24 weeks

Dr. wessam alzaidat - General surgery


87- Calcitonin sometimes is elaborated by which one of the following tumors :

a-Medullary carcinoma of the breast

b- Medullary carcinoma of the thyroid

c- Giant cell carcinoma a of pancrease

d- Carcinoid tumor of appendix

e- Ganglioneuromas

88- Normal " prothrombin time " as measured by usual clinical tests depends on
normal levels of all of the following factors excepts :

a- Factor II

b- Factor V

c- Factor VII

d- Factor VIII

e- Factor X

89- Side effects of large doses of cortisone include all of the following except :

a- Hirsutism and osteoporosis

b- Mental changes

c- Recurrence or appearance of new peptic ulcers

d- Increased susceptibility to infection

e- Depletion of serum sodium

90- The average daily loss of water through the lungs and skin (
insensible water loss ) is approximately :

a- 10 ml

b- 100ml

Dr. wessam alzaidat - General surgery


c- 1000ml

d- 2000ml

e- Any of these depending on circumstances

91- The most common cause of gastric outlet obstruction in adults is;

A- Adenocarcinoma of the stomach.

B- Hypertrophic pyloric stenosis.

C- Duodenal stenosis secondary to peptic ulceration.

D- Bezoar.

E- Gastric lymphoma.

92- About colorectal carcinoma associated with Crohn’s disease, one is TRUE:

A- It is usually common in women.

B- The frequency of carcinoma is similar in patients with extensive, long


standing ulcerative colitis.

C- The right colon is involved in over 70% of patients.

D- The mean age of patients with colorectal carcinoma is 35.

E- The prognosis is the same for Ca associated with Crohn's dis and Ca in
colon without Crohn's dis,

93- Regarding pancreatic carcinoma all are true Except:-

A- 90% are ductal adenocarcinomas

B- Less than 20% occur in the head of the gland

Dr. wessam alzaidat - General surgery


C- The usual presentation is with pain, weight loss and obstructive jaundice

D- Ultrasound has a sensitivity of 80-90% in the detection of the tumour

E- Less than 20% of patients are suitable for curative surgery

94- Heparin all are true Except:-

A-Is a heterogeneous mixture of sulphated polypeptides

B- Potentiates the actions of antithrombin III

C- Has a half life of 90 minutes.

D-Can be reversed by protamine sulphate

E-Can induce an idiosyncratic thrombocytopenia

95- All of the following affect Gastrin release EXCEPT:

a. antral acidification.

b. antral alkalization.

c. carbohydrates in antrum.

d. gastric distension.

e. somatostatin release.

96- What portion of the colon absorb the majority of fluid?

a. ascending colon

b. transverse colon

c. descending colon

d. sigmoid colon

e. rectum

97- The concentration of which electrolyte in pancreatic secretion increases as the

Dr. wessam alzaidat - General surgery


rate of secretion increases?

a. sodium

b. potassium

c. chloride

d. bicarbonate

e. calcium

98- All of the following promote LES relaxation EXCEPT:

a. atropine

b. nitric oxide

c. cholecystokinin (CCK)

d. gastric distention

e. pharyngeal stimulation

99- Intraabdominal adhesions following abdominal surgery have been associated


with all of the following EXCEPT:

a. small bowel obstruction

b. infertility

c. chronic pelvic pain

d. intestinal malabsorption

e. increased risk for enterotomy on subsequent laparatomy

100- Which of the following is true regarding small bowel carcinoid tumors?

a. most are biochemically atypical tumors lacking the enzyme dopa decarboxylase

b. they are most common GI carcinoid tumors

c. regional lymph node involvement is common in tumors less than 1 cm in size

d. diagnosis is frequently made in patients prior to surgery

e. among carcinoid tumors, they are associated with the lowest rate of second
Dr. wessam alzaidat - General surgery
primary malignancy

Dr. wessam alzaidat - General surgery


Dr. wessam alzaidat - General surgery
KEY 1 C

2 D
27 D

28 A

29 C

30 D

31 D

32 C

33 C

34 B

35 B

36 D

37 B

38 D

39 C

40 B

41 C

42 B

43 B

44 B

45 E

46 B

47 E

48 A

49 A

Dr. wessam alzaidat - General surgery


50 C

51 E

52 D

53 A

54 B

55 C

56 C

57 D

58 B

59 A

60 D

61 D

62 A

63 C

64 B

65 C

66 D

67 D

68 A

69 E

70 A

71 D

72 B

73 C

Dr. wessam alzaidat - General surgery


74 D

75 C

76 B

77 C

78 D

79 D

80 C

81 E

82 D

83 B

84 C

85 B

86 D

87 B

88 D

89 E

90 C

91 A

92 B

93 B

94 A

95 C

96 A

97 D

Dr. wessam alzaidat - General surgery


98 A

99 D

100 C

Dr. wessam alzaidat - General surgery


PAST BOARDS MCQs
1.Regarding Peutz Jeghers syndrome all are true, except:
a. Is an autosomal dominant condition.
b. Often presents with anaemia in childhood.
c. Is characterised by circumoral mucocutaneous pigmented lesions.
d. Is associated with adenomatous polyps of the small intestine.
e. Malignant change occurs in 2-3% of polyps.

ANSWER:D

2. Regarding peptic ulceration all are true, except:


a. H. pylori is a gram-negative bacillus.
b. Duodenal is more common than gastric ulceration.
c. Zollinger-Ellison syndrome is associated with gastrin hyposecretion.
d. H2-blockers will heal 85-95% of duodenal ulcers in 8 weeks.
e. Triple therapy can eradicate H. pylori in 80% of patients in one week.

ANSWER:C

3.Regarding head Injuries all are true, except:


a. More than 1 million people are seen in UK hospitals each year with head injuries.
b. Skull X-rays can not exclude an intracerebral haematoma.
c. Raised intracranial pressure is associated with an increase in cerebral perfusion.
d. Cushing's response consists of a rise in blood pressure and fall in heart rate.
e. Pupillary dilatation usually occurs on the same side of the intracerebral
haematoma.

ANSWER:C

4. Regarding stones in the gallbladder, one is correct:


a. Pigment stones are the most common.
b. Pigment stones are due to increased excretion of polymerised conjugated bilirubin.
c. Are not a risk factor for the development of gallbladder carcinoma.
d. 90% of gallstones are radio-opaque.
e. A mucocele of the gallbladder is mostly caused by a stone impacted in Hartmann's
pouch.
ANSWER:E
5.Regarding stones in the common bile duct, all are true, except:
a. Are found in 10 to 15% of patients undergoing cholecystectomy (Without pre-op
ERCP).
b. Can present with Charcot's Triad.
c. Are suggested by a bile duct diameter >8mm on ultrasound.
d. ERCP, sphincterotomy and balloon clearance is now the treatment of choice.
Dr. wessam alzaidat - General surgery
e. If removed by exploration of the common bile duct the T-tube can be removed
after 3days.

ANSWER:E 10 DAYS

6. Regarding crystalloid solutions, all are true, except:


a. Normal saline contains 154 mmol sodium and 154 mmol chloride.
b. 3 litres of dextrose 1/5 th saline in a day will provide 90 mmol of sodium.
c. 2 grams of potassium chloride is equal to 57 mmol of the salt.
d. Hartmann's solution contains calcium and bicarbonate.
e. The daily maintenance potassium requirement of a 40 Kg woman is about 40
mmol.

ANSWER:C

7. Regarding colloid solutions, all are true, except:


a. Human albumin has a molecular weight of 69 kDa.
b. Albumin has a half life in the circulation of about 15 days.
c. Gelatins (e.g. Haemaccel®) are polysaccharides with a MW of about 35 kDa.
d. Dextrans reduce platelet aggregation and can induce anaphylaxis.
e. 6% Hydroxylethyl Starch (HES) is a synthetic polysaccharide derived from
amylopectin.

ANSWER:C

8. Regarding central parenteral nutrition, all are true, except:


a. Is a hypo-osmolar solution.
b. Typically contains 14-16g nitrogen as L-amino acids.
c. Typically contains about 250g glucose.
d. Is associated with metabolic disturbances in about 5% of patients.
e. Can induce derangement of liver function tests.

ANSWER:A

9.Regarding solitary thyroid nodules, all are true, except:


a. Are more prevalent in women.
b. In the adult population, less than 10% are malignant.
c. More than 50% of scintigraphically cold nodules are malignant.
d. The risk of a hot nodule being malignant is negligible.
e. Should not be surgically removed in all patients.

ANSWER:C

10. Regarding the femoral canal, one is correct:


a. Lies lateral to the femoral vein.
Dr. wessam alzaidat - General surgery
b. Has no relation to the inguinal ligament.
c. Has the lacunar ligament as its lateral border.
d. Has the pectineal ligament as its posterior border.
e. Contains the lymph node of Lund.

ANSWER:D

11.Regarding the pathology of ulcerative colitis, one is true:


a. Shows full thickness bowel inflammation.
b. The rectum is almost always not involved.
c. 10% of patients have terminal ileal disease.
d. Enterocutaneous or intestinal fistulae are common.
e. The serosa is usually affected.

ANSWER:C

12. Regarding benign breast disease, all are true, except:


a. Cyclical mastalgia is the commonest reason for referral to the breast clinic.
b. Fibroadenomas are derived from the breast lobule.
c. Lactational breast abscesses are usually due to Staph aureus.
d. Duct ectasia is less common in smokers.
e. Atypical lobular hyperplasia is associated with an increased risk of breast cancer.

ANSWER:D

Dr. wessam alzaidat - General surgery


13.Regarding infantile hypertrophic pyloric stenosis, one is true:
a. Occurs with a male : female ratio of 4:1.
b. Sons of affected mothers have a 80% risk of developing the lesion.
c. Invariably presents between six and eights months of age.
d. Typically presents with bile stained projectile vomiting.
e. Surgical treatment is by Heller's cardiomyotomy.

ANSWER:A

14. Regarding oesophageal atresia , all are true, except:


a. Is often associated with a distal tracheo-oesophageal fistula.
b. Polyhydramnios is often present late in pregnancy.
c. 50% have other associated congenital abnormalities.
d. Contrast x-ray studies are necessary to confirm the diagnosis.
e. Post-operatively over 15% develop oesophageal strictures.

ANSWER:D

15.All the following are features of Fallot's Tetralogy, except:


a. Atrial septal defect.
b. Pulmonary stenosis.
c. Right ventricular hypertrophy.
d. A right to left cardiac shunt.
e. Cyanotic attacks during feeding and crying.

ANSWER:A

16.Regarding phaeochromocytomas, one is correct:


a. Are neuroendocrine tumours of the adrenal cortex.
b. Are benign in 10% of cases.
c. Occur as part of the MEN type I syndrome.
d. Can be imaged with a meta-iodobenzylguanidine scan.
e. Require only beta-blockage drug prior to surgery.

ANSWER:D

17.Regarding the functions of a tracheostomy all are true, except:


a. Bypasses an upper airway obstruction.
b. Increases the anatomical dead space.
c. Decreases airway resistance.
d. Protects against aspiration.
e. Allows frequent airway suction.
ANSWER:B
DR.WBZ
18. Regarding hepatitis B, all are true, except:
Dr. wessam alzaidat - General surgery
a. It is due to a DNA virus.
b. It has an incubation period of 2 to 6 days.
c. It can be transmitted via blood products.
d. About 1% of the UK population are hepatitis B surface antigen positive.
e. About 10% of infected patients become chronic carriers.

ANSWER:B

19.The best surgical modality to treat recurrent primary spontaneous


pneumothorax in a 20 year old man is:
a.bullectomy.
b.pleurectomy.
c.bullectomy and pleurectomy.
d.pleurodesis.
e.decortication.

ANSWER:C

20-Regarding the anatomy of diaphragm, all are true except:


a.The hiatal opening is situated at the level of the tenth thoracic vertebra.
b.The inferior vena cava passes through the right side of diaphragm at the level of
12th thoracic vertebra.
c.The principle blood supply of the diaphragm is derived directly from the aorta.
d.The left diaphragm is lower than the right diaphragm in a normal adult.
e.Nerve supply to the diaphragm is via phrenic nerve which arises from 3rd,4th&5th
cervical nerve roots.

ANSWER:B

21.Which one of the following diagnostic methods is the best to diagnose


pneumothorax?
a.Standing CXR, expiratory film.
b.Standing CXR, inspiratory film.
c.Supine CXR, expiratory film.
d.Supine CXR, inspiratory film.
e.Lateral CXR.

ANSWER:A

22.Regarding chest trauma, all are true, except:


a.Rib fracture in a child is usually a marker of severe injury.
b.Thoracostomy tube should be inserted prior to CXR in any patient with suspected
pneumothorax associated with hypotension.
c.A patient with flail chest, should be considered for ventilatory support if his
Dr. wessam alzaidat - General surgery
respiratory rate is above 35/min.
d.Exploratory thoracotomy should be done if the amount of blood production through
the chest tube is more than 200 cc hourly for 4 hours.
e.Chest tube should be clamped during the transport of the patient.

ANSWER:E

23. The best site to insert a chest tube is:


a.2nd intercostal space, anterior axillary line.
b.3rd intercostal space,posterior axillary line.
c.4th intercostal space ,posterior axillary line.
d.5th intercostal space, anterior axillary line.
e.7th intercostal space, mid axillary line.

ANSWER:D

24. The best non invasive diagnostic procedure to diagnose diaphragmatic injury
after trauma is:
a.Chest xray.
b.Chest CT scan.
c.Chest MRI.
d.Chest ultrasound.
e.Barium meal.

ANSWER:C

25.Regarding the anatomy of the tracheobronchial tree and the lung, all are
true, except:
a.The right lung is larger than the left lung.
b.The major (oblique fissure) separates the lower lobe from the upper and middle
lobe.
c.There are 8 segments in the right lung.
d.The right main bronchus is shorter and wider than the left main bronchus.
e.The posterior wall of the trachea is membranous.

ANSWER:E
26. All the followings are true data about the anatomy and the physiology of
lower oesophageal sphincter, except:
a.In adults, it measures 3 to 5 cm in length.
b.In normal individuals, manometry demonstrates pressure of 5 mm Hg within the
lower oesophageal sphincter.
c.The lower oesophageal sphincter remains contracted at rest.
d.It is not a true sphincter.
e.It normally relaxes with the approach of the primary peristaltic wave.

Dr. wessam alzaidat - General surgery


ANSWER:B

27.Which of the following is probably a posterior mediastinal mass rather than


an anterior one?
a.Thyroid tumour.
b.Neurogenic tumour.
c.Thymic carcinoma.
d.Teratoma.
e.Lymphoma.

ANSWER:B

28. Regarding thoracic duct,one is correct:


a.Is the main collecting vessel of the lymphatic system and is far smaller than the
right terminal lymphatic duct.
b.Originates from the cysterna chyle at the level of 4th lumbar vertebra.
c.Originates in the left side.
d.Ascends to enter the chest through the inferior vena cava hiatus.
e.Lies on the anterior surface of the vertebral column behind the esophagus and
between the aorta and the azygous vein.

ANSWER:E

29. The maximum safe dose of local anesthetic administered subcutaneously in a


70 kg man is:
a.10 to 20 mL of 1% lidocaine.
b.40 to 50 mL of 2% lidocaine with epinephrine.
c.40 to 50 mL of 1% lidocaine with epinephrine.
d.40 to 50 mL of 1% bupivacaine.
e.40 to 50 mL of 1% lidocaine without epinephrine.

ANSWER:C
30.Which of the following vessels of the lower limb is least likely to be occluded
by atherosclerosis?
a.Anterior tibial.
b.Peroneal.
c.Proximal posterior tibial.
d.Distal posterior tibial.
e.Dorsalis pedis.

ANSWER:C

31. After thoracotomy, a woman has a loss of sensation of the nipple areolar
complex,injury to which intercostal nerve is responsible for this?
Dr. wessam alzaidat - General surgery
a.Second.
b.Third.
c.Fourth.
d.Fifth.
e.Sixth.

ANSWER:C

32. Oxygen delivery can be increased by increasing all of the followings, except:
a.Hemoglobin.
b.Atmospheric pressure.
c.Cardiac output.
d.Inspired oxygen concentration.
e.Oxygen extraction.

ANSWER:E

33. Which of the followings is the best abdominal site to assess the bowel sounds
using the stethoscope?
a.Right upper quadrant.
b.Right lower quadrant.
c.Left upper quadrant.
d.Left lower quadrant.
e.Periumbilical.

ANSWER:B

34. Brain stem death is diagnosed in the absence of these brain stem reflexes,
except:
a- Corneal reflexes.
b- Pupillary reflexes.
c- Doll's eye movements.
d- Caloric response.
e- Apnoeic test.

ANSWER:C

35. The followings are indications for admission to hospital after a minor head
injury, except:

Dr. wessam alzaidat - General surgery


a- reduced level of consciousness.
b- Concussion without skull fracture.
c- Clinical or radiological evidence of skull fracture.
d- Focal neurological dysfunction.
e- Difficulty in assessing the patient.

ANSWER:B

36. In Extradural haematomas all the followings occur, except:


a- One in ten occur in the frontal region.
b- Less than half exhibit a lucid interval.
c- 20% occur in the temporal area.
d- The first symptom is often progressive paroxysmal headache.
e- Nearly always associated with vault skull fracture.

ANSWER:C

37. The source of bleeding in the extradural space are the followings,
except:
a- Diploic vessels of the skull vault.
b- The cerebral veins.
c- The middle meningeal vessels.
d- The dural sinuses.
e- The dural arterioles.

ANSWER:B

38.The following regimens are used to treat raised intracranial


pressure (ICP), except:
a- Hypoventilation .
b- Mannitol 20% solution, 1g mannitol/kg body weight.
c- Drainage of CSF.
d- Barbiturates.
e- Surgical decompression.

ANSWER:A

39. Cerebrospinal fluid (CSF) characterizes by the followings ,except:


a. Circulates at about 0.5 ml/min.
b. Secreted at about 500 ml/24 hours.
c. Secreted in the lateral,third,and fourth ventricles.
Dr. wessam alzaidat - General surgery
d. Secreted predominantly by active secretion.
e. Absorbed by the choroid villi.

ANSWER:E

40. Definitive diagnosis of Pulmonary embolism is made by one of the


followings:
a- ECG (EKG).
b- Chest X-ray.
c- VQ scan.
d- ABG.
e- Pulmonary angiography.

ANSWER:E

41.In the body metabolism, 10g of protein, would produce approximately:


a. 20 kcal.
b.41 kcal.
c.410 kcal.
d.4100 kcal.
c. 900 kcal.

ANSWER:B

42. The Glasgow coma scale is dependant upon all of the followings, except:
a.Response to speech.
b.Response to pain.
c.Response of the pupils.
d.Motor response.
e.Response of the patient.

ANSWER:C

43. Which one of the following spinal segments causes signs of wasting of the
intrinsic muscles of the hand?
a- C5.
b- C7/8.
c- T1.
d- L5.
e- S1.

ANSWER:C

Dr. wessam alzaidat - General surgery


44. The methods by which bacteria gain access into the brain include all the
followings, except:
a- Direct spread from the surrounding tissues.
b- Haematogenous.
c- Via implantation during trauma.
d- Lymphatics.
e- During surgery.

ANSWER:D

45. All the followings are premalignant lesions of the skin, except:
a. Actinic keratosis.
b. Bowen’s disease.
c. Seborrheic keratosis.
d. Dysplastic nevi.
e. Nevus sebaceous.

ANSWER:C

46. The commonest cause of death after major burns in modern burn units is:
a) Inadequate resuscitation.
b) Pneumonia.
c) Burn wound sepsis.
d) Renal failure.
e) Bacterial endocarditis.

ANSWER:B

47. Complications of mandibular fractures include all the followings, except:


a. Malocclusion.
b. Lower lip paraesthesia.
c. Trismus.
d. Ankylosis of the temperomandibular joint.
e. Paralysis of the lower lip.

ANSWER:E

48. An ulnar nerve injury at the elbow would result in all the followings, except:
a. Clawing of the ring and little fingers.
b. Inability to abduct the little finger.
c. Inability to flex the ring and little fingers.
d. Inabilility to adduct the thumb.
e. Radial deviation of the wrist in flexion.

Dr. wessam alzaidat - General surgery


ANSWER:C

49. The most important factor which decide the efficacy of laser treatment for a
particular condition is:
a. Age of patient.
b. Depth of lesion.
c. Energy Fluence (Power density x Duration of treatment).
d. Spot size.
e. Wavelength as decided by the lasing medium.

ANSWER:E

50. A blow-out fracture of the orbit is by definition:


a. A fracture that results in rupture of the globe.
b. A fracture that results in disruption of the orbital rim.
c. A fracture of the thin walls of the orbit allowing herniation of the orbital contents.
d. A fracture that results from an explosion.
e. A fracture that includes bones adjacent to the orbit.

ANSWER:C
51. Which of the followings is considered the primary blood supply to the skin?
a. Fascial plexus.
b. Subcutaneous plexus.
c. Subdermal plexus.
d. Dermal plexus.
e. Subepidermal plexus.

ANSWER:C

52. Principles of skin wound repair include all of the followings, except:
a. Obliteration of the dead space.
b. Eversion of the skin edges.
c. Tension free closure.
d. Layered tissue closure.
e. Use of nylon simple sutures at the skin edge.

ANSWER:E

53. The most reliable indicator of the adequacy of burn resuscitation is:
a. Central venous pressure.
b. Pulmonary capillary wedge pressure.
c. Urine output.
d. Blood pressure and heart rate.
e. Mental status.

Dr. wessam alzaidat - General surgery


ANSWER:C

54. The most common site for squamous cell carcinoma of the lip is the:
a. Upper lip midline.
b. Upper lip laterally.
c. Lower lip midline.
d. Lower lip laterally.
e. Oral commissure.

ANSWER:D

55. Which of the following cell types is essential for normal wound healing?
a. Leukocytes.
b. Monocytes.
c. Platelets.
d. Erythrocytes.
e. All of the above.

ANSWER:B

56. The most important factor in the development of ulcers in a spinal cord
injury patient is:
a. Malnutrition.
b. Septic episodes.
c. Anemia.
d. Local pressure.
e. Diminished sensation.

ANSWER:D

57. Which vitamin D metabolite stimulate intestinal calcium absorption?

a. 1,25–dihydroxyvitamin D.
b. 1,25–dihydroxyvitamin D1.
c. 1,25–dihydroxyvitamin D2.
d. 1,25–dihydroxyvitamin D3.
e. 1,25–dihydroxyvitamin D4.

ANSWER:D

58. Urinary stones generally do not pass spontaneously if they are larger than?

a. 2 mm.
Dr. wessam alzaidat - General surgery
b. 3 mm.
c. 4 mm.
d. 7 mm.
e. 8 mm.

ANSWER:C

59.Which of the following studies replace intravenous urography in the initial


evaluation of patients with renal colic?

a. Kidney, ureter and bladder study.


b. Renal tomography.
c. Ultrasonography.
d. CTScan.
e. MRI.

ANSWER:D

60. Initial hormonal evaluation of an infertile man with a sperm count of 5


million sperm/ml should include assays of testosterone level and:

a. Prolactin.
b. Follicle–stimulating hormone (FSH).
c. FSH, luteinizing hormone (LH) and prolactin.
d. FSH and thyroid function.
e. Sex steroid hormone binding globulin (SHBG).

ANSWER:B

61. Where does benign prostatic hyperplasia (BPH) originate?

a. In the transition zone.


b. In the peripheral zone.
c. In the periurethral glands.
d. In the transition zone and periurethral zone.
e. In the whole organ.

ANSWER:D

62. It is advisable in a man with BPH and a slightly elevated creatinine level to
perform a(an):

a. Transurethral resection of the prostate (TURP)


b. Intravenous pyelography.
c. Renal sonography.
Dr. wessam alzaidat - General surgery
d. Urodynamic study.
e. kidney, ureter and bladder X-ray.

ANSWER:C
63.A 20 year old man has undergone a retroperitoneal dissection for a testicular
germ cell tumor. The inferior mesenteric artery has been divided during
reflection of the intestines to expose the retroperitoneum. This can be expected
to result in:

a. Ischemia of the descending colon.


b. Ischemia of the sigmoid colon.
c. Ischemia of the rectum.
d. Ischemia of the transverse colon.
e. None of the above.

ANSWER:E

64. The predominant buffering system in humans is:

a. Bicarbonate.
b. Titratable acids.
c. Ammonium (NH4+).
d. Urea.
e. Phosphate.

ANSWER:A

65. Terminal hematuria (at the end of the urinary stream) is usually due to:

a. Bladder neck or prostatic inflammation.


b. Bladder cancer.
c. kidney stones.
d. Ureteric calculi.
e. Ureteral stricture disease.

ANSWER:A

66. Pneumaturia may be due to all of the followings, except:

a. Diverticulitis.
b. Colon cancer.
c. Recent urinary tract instrumentation.
d. Inflammatory bowel disease.
e. Ectopic ureter.

Dr. wessam alzaidat - General surgery


ANSWER:E
67. As one proceeds outward from the adrenal medulla, the three separate
functional layers of the adrenal cortex are, in correct order:

a. The zona reticularis, zona fasciculata, then zona glomerulosa.


b. The zona fasciculata, zona reticularis, then zona glomerulosa.
c. The zona glomerulosa, zona fasciculata, then zona reticularis.
d. The zona glomerulosa, zona reticularis, then zona fasciculata.
e. The zona reticularis, zona glomerulosa, then zona fasciculata.

ANSWER:A

68. What proportion of the cardiac output is delivered to the kidney?

a. 5%.
b. 20%.
c. 50%.
d. 85%.
e. 100%.

ANSWER:B

69. During surgical dissection, the ureter can be identified as it enters the pelvis:

a. At the aortic bifurcation.


b. Crossing the superior border of the sacrum.
c. Crossing the common iliac artery at the branching of the internal iliac artery.
d. Crossing the uterine artery.
e. At the internal inguinal ring.

ANSWER:C

70. All the followings are consequences of major haemorrhage, except:


a- Rise in serum osmolality.
b- Rise in intracellular osmotic pressure.
c- Decrease in plasma oncotic pressure .
d- Peak albumin synthesis in 6 hrs.
e- Rise in blood sugar.

ANSWER:B

71. Signs of cardiac tamponade may include all of the followings, except:
a- Agitation and confusion.
b- Hypotension.
c- Congested neck veins.
Dr. wessam alzaidat - General surgery
d- Muffled heart sounds.
e- Deviated trachea .

ANSWER:A

72. About achalasia, one is not true :

a- It is the most common primary esophageal disorder.


b- Manometry reveals non relaxation of the lower esophageal sphincter at rest.
c- Acceptable treatment includes Endoscopic myotomy.
d- It is most often associated with hyperperistalsis of the esophageal body.
e- The intraluminal pressure is elevated.

ANSWER:D

73. All are causes of metabolic acidosis, except:


a- Untreated diabetes mellitus.
b- Duodenal Ulcer.
c- Uremia.
d- Shock.
e- Liver failure .

ANSWER:B

74. All are recognized complications of acute pancreatitis, except:


a- Adult respiratory distress syndrome .
b- Tetany.
c- Acute renal failure .
d- Disseminated intravascular coagulation .
e- Hypercalcaemia .

ANSWER:E

75. The following body fluids have high risk of HIV transmission, except:
a- Semen .
b- Blood.
c- Urine.
d- Breast milk.
e- Fresh frozen plasma.

ANSWER:D

76. Factors which contribute to wound dehiscence include all the followings,
except:
a- Old age.
Dr. wessam alzaidat - General surgery
b- Coughing.
c- Hypoproteinemea.
d- Anaemia.
e- Malignancy.

ANSWER:D

77. The most common extracranial solid tumour in children is:


a. Wilm's tumour.
b. Neuroblastoma.
c. Hepatoblastoma.
d. Hepatocellular carcinpma.
e. Seminoma.

ANSWER:B

78. In intussusception, all are true, except:


a. Idiopathic causes are the most common in the infantile age group.
b. Ileo-colic intussusception are seen most commonly.
c. Abdominal colic and vomiting present in most of the cases.
d. Bloody stool is an indication for open surgical reduction.
e. In older children, a pathological leading point must be excluded.

ANSWER:D

79. Regarding inguinal hernia in children, all are true, except:


a. Is more common in pre-term babies.
b. Is more common in males.
c. Testicular atrophy might be a long term complication.
d. Complications are more common in pre-terms than in term babies.
e. Contralateral groin exploration is routinely done to exclude bilateral hernias.
ANSWER:E DR.WBZ
80. One of the followings is true regarding undescended testes:
a. The incidence is 1 in 10 000 boys.
b. The most common location is in the femoral region.
c. Is associated with high rate of infertility.
d. Orchidopexy is indicated at the age of five years.
e. Bilateral cases are more frequent than unilateral cases.

ANSWER:C

81. The most common indication for splenectomy in children is:


a. Splenic injury in blunt abdominal trauma.
b. Sarcoidosis.
c. Idiopathic thrombocytopenic purpura ( ITP ).
Dr. wessam alzaidat - General surgery
d. Hereditary spherocytosis.
e. Thalassemia.

ANSWER:D

82. Recurrent UTI in children mostly raise the suspicion of:


a. Urethral diverticulum.
b. Vesico ureteric reflux.
c. Ureteric duplication.
d. Renal ectopia.
e. Neurogenic rectum.

ANSWER:B

83. The absolute contraindication for circumcision in children is:


a. Hypospadias.
b. Blood dyscrasia.
c. Recurrent UTI.
d. Para-phimosis.
e. Phimosis.

ANSWER:A

84. A child weighs 15 Kg, his fluid requirement is:


a. 50 ml per hour.
b. 50 ml per kg per hour.
c. 1000 ml per 24 hours.
d. 10 ml per kg per hour.
e. 150 ml per hour.
ANSWER:A
85. Biliary Atresia is definitely diagnosed by:
a. HIDA scan.
b. Abdominal ultrasound.
c. Liver function test.
d. Abdominal C.T scan with i.v contrast.
e. Intra-operative cholangiogram.

ANSWER:E

86. Two years old infant previously healthy, presented with sudden onset of
respiratory distress, Choking and transient cyanosis. The most likely diagnoses
is:
a. F.B aspiration.
b. Pneumothorax.
c. Pneumonia.
Dr. wessam alzaidat - General surgery
d. Hemothorax.
e. Hemo-pneumothorax.

ANSWER:A

87. All are causes of bile-stained vomiting in newborns, except:


a. Sepsis.
b. Duodenal atresia.
c. Meconium ileus.
d. Complicated intestinal malrotation.
e. Esophageal atresia.

ANSWER:E

88. All are normal blood tests in the term newborn at day one of age, except:
a. Blood glucose 45mg-60mg / dl.
b. WBC 20. 000 to 30.000.
c. Creatinine up to 1.3mg%.
d. PCV 55%- 65%.
e. ESR 60mm/first hour.

ANSWER:E

89. The most common cause of death in old children is:


a. Malignancy.
b. Congenital heart disease.
c. Trauma.
d. Renal Anomalies
e. CNS abnormalities.

ANSWER:C

90. Causes of abdominal calcification on x-ray include all of the followings,


except:
a. Faecolith.
b. Wilm's tumour.
c. Neuroblastoma.
d. Nephrocalcinosis.
e. Calcified lymph nodes.

ANSWER:A

91. Which of the followings is the most important stimulus for triggering the
endocrine response to injury?
a. Tissue acidosis.
Dr. wessam alzaidat - General surgery
b. Local wound factors.
c. Afferent nerve stimuli from the injured area.
d. Hypovolemia.
e. Temperature changes.

ANSWER:C

92. Which of the following substances is elevated during the acute response to
injury?

a. Glucagon.
b. Insulin.
c. Thyroid stimulatring hormone (TSH).
d. Prolactin.
e. Calcitonin.

ANSWER:A

93. With regard to protein loss after injury, which of the following statements is
true?
a. It results from impaired protein synthesis.
b. It occurs primarily from catabolism of skeletal muscle.
c. It occurs primarily from acute renal failure.
d. It occurs primarily from the site of injury.
e. It can be prevented by total parenteral nutrition.
ANSWER:B

94. Normal wound healing is accelerated by one of the followings?


a. Ascorbic acid.
b. Vitamin A.
c. Zinc.
d. Increased local oxygen tension.
e. Frequent dressing.

ANSWER:C

95. All of the followings are true regarding perforated appendicitis, except:
a. Higher rate in children.
b. Higher rate in old patients.
c. Usually due to delay in presentation and diagnosis.
d. Appendicular mass is felt in more than 50% of the cases.
e. The commonest complication is wound infection.

ANSWER:D

Dr. wessam alzaidat - General surgery


96. With regard to blunt abdominal trauma, the commonest intra abdominal
organ which can be affected is?
a. Urinary bladder.
b. Large intestine.
c. Pancreas.
d. Stomach.
e. Spleen.

ANSWER:E

97. One of the followings is not a cause of hypokalemia?


a. Inadequate oral intake.
b. Potassium-free intravenous fluids.
c. High output nasogastric tube.
d. Blood transfusion.
e. Massive vomiting and diarrhea.

ANSWER:D

98. With regard to wound classification, penetrating abdominal trauma is?


a. Clean wound.
b. Clean/contaminated wound.
c. Contaminated wound.
d. Dirty wound.
e. Not classified.

ANSWER: (C) OR D

99. The most common cause of hypercalcemia in hospitalized patients is?


a. Malignancy.
b. Over oral intake.
c. Bone fracture.
d. Blood transfusion.
e. Parathyroid adenoma.

ANSWER:A

100. One of the following is not a cause of dilutional hyponatremia?


a. Excessive oral water intake.
b. Iatrogenic (intravenous) excess of free water administration.
c. Postoperative increase of antidiuretic hormone secretion.
d. Decreased oral intake of sodium.
e. Some drugs which cause water retention.

ANSWER:D
Dr. wessam alzaidat - General surgery
PAST BOARDS MCQs

1-To maintain a normal hydrogen balance, total daily excretion of H+ should equal
the daily: -
a) Fixed acid production plus fixed acid ingestion.
b) HCO3- excretion
c) Pseudomonas aeruginosa.
d) S.Fecales.
e) Staph. Aureus.

ANSWER:A

2- A 45-year-old obese man undergoes subtotal gastrectomy for an antral carcinoma.


On the fifth postoperative day, non-foul-smelling serosanguineous drainage appears
from the wound. All the following may have been contributing factors EXCEPT
a) malnutrition
b) atelectasis
c) anemia
d) poor technique
e) ascites

ANSWER:C

Dr. wessam alzaidat - General surgery


3- A 32-year-old man is admitted to a hospital where he undergoes exploratory
surgery for a gunshot wound of the left colon. Subsequently, he is maintained on
intravenous hyperalimentation; his vital signs are stable, and his urine output
averages 250 mL/h. On the fourteenth hospital day, he is reexplored under general
anesthesia for drainage of a left subphrenic abscess. Parenteral nutrition is continued
during surgery. In the recovery room 6 h later, the patient is stable and his urine
output is brisk, but he remains comatose with spontaneous respirations.
The most likely cause for the patient's delayed emergence from anesthesia is
a) hypoglycemia
b) nonketonic, hyperosmolar coma
c) partial curarization
d) volume overload
e) hypoventilation and hypoxemia

ANSWER:B
4- A patient with a non obstructing carcinoma of the sigmoid colon is being prepared
for elective resection. To minimize the risk of postoperative infectious complications,
your planning should include:

a) A single preoperative parenteral dose of antibiotic effective against aerobes and


anaerobes
b) Avoidance of oral antibiotics to prevent emergence of Clostridium difficile
c) Postoperative administration for 2–4 days of parenteral antibiotics effective against
aerobes and anaerobes
d) Postoperative administration for 5–7 days of parenteral antibiotics effective against
aerobes and anaerobes
e) Operative time less than 5 h.

ANSWER:C

5- Regarding Parathyroid cancer, which one of the following statements is true


a) Is a frequent cause of hyperparathyroidism
b) Most patients present with hypercalcemia and a palpable mass
c) The histopathological criteria to differentiate a parathyroid adenoma and
parathyroid carcinoma are well defined
d) It is associated with multiple endocrine neoplasia type 1
e) Localizing studies are mandatory before neck exploration.

ANSWER:B

6- The rational for Endoscopic management of cystic duct leak after laparoscopic
cholecystectomy is: -
a) Covering the cystic duct-common bile duct junction with plastic stent to prevent
further leak.
b) Increase in the transpapillary pressure gradient by papillotomy.
Dr. wessam alzaidat - General surgery
c) Promoting flow and reducing the amount of bile flowing through the leak by
placing a plastic stent and also performing a papillotomy.
d) The reduction in the transpapillary pressure gradient by insertion of a temporary
stent.
e) Promoting flow and reducing the amount of bile flowing through the leak by
placing a naso- biliary drain.

ANSWER:D

7- If tacks/staples are placed to affix the mesh during laparoscopic inguinal hernia
repair, they should NOT be placed:-
a) Into Cooper's ligament.
b) Inferior and lateral to the internal inguinal ring.
c) Superior to the inguinal ligament In to the posterior aspect of the rectus muscle.
d) Superior and medial to the internal inguinal ring.
e) At the pubic bone.

ANSWER:B

8- What are the early signs and symptoms of gastrojejunostomy leak in obese
patients?
a) Fever.
b) Tachycardia
c) Peritonitis.
d) Oliguria.
e) Hypotension.

ANSWER:B

9- A 33-year-old man underwent laparoscopic Roux-en-Y gastric bypass 15 months


prior to his presentation to an emergency room for sudden onset of abdominal pain.
He has no other health problems. The pain extends "like a band" across his mid-
abdomen and causes him to bend over. Abdominal exam demonstrates minimal
tenderness in the left upper quadrant of the abdomen and no signs of peritonitis. Lab
tests including complete blood count and comprehensive chemistry panel are normal.
What is the most likely cause for his pain?
a) Anastomotic chronic stricture.
b) Internal herniation.
c) Marginal ulcer.
d) Late dumping syndrome.
e) Gastrointestinal leak.

ANSWER:B
Dr. wessam alzaidat - General surgery
10- The most common complication of PEG ( Percuatnious Endoscopic Gastrostomy)
placement is?
a) Bleeding
b) Infection
c) Peritonitis
d) Gastro-colic fistula
e) Tube dislodgment

ANSWER:B

11- About colorectal carcinoma associated with Crohn’s disease, one is TRUE:
a) It is usually common in women.
b) The frequency of carcinoma is similar in patients with extensive, long-standing
ulcerative colitis.
c) The right colon is involved in over 70% of patients.
d) The mean age of patients with colorectal carcinoma is 35.
e) The occurrence of carcinoma is unrelated to the duration of Crohn’s disease.

ANSWER:B

12- Prolonged absence of enteral nutrition during critical illness causes all of the
following changes in the small intestine EXCEPT:
a) Decrease villous height and cellular mass.
b) Reduction in the production of brush border enzymes.
c) Decrease nutrition absorption.
d) Increased translation in gut bacteria.
e) Irreversible changes in mucosal absorption.

ANSWER:E

Dr.WBZ

13- Two yrs after resection of a T2N0M0 colon cancer, a chest X-ray reveals a
solitary peripheral pulmonary mass, chest and abdominal CT confirms an isolated
pulmonary lesion without other abnormalities. Which of the following statements is
true?
a) Staging with PET is not indicated
b) Percutaneous biopsy is not indicated because the risk of intrathoracic spread.
c) A normal serum carcinoembryonic antigen (CEA) level excludes metastatic
disease.
d) Resection is indicated.
e) Primary treatment should be Cytotoxic chemotherapy rather than resection.

Dr. wessam alzaidat - General surgery


ANSWER:D
14- A 35 year old female patient with BMI of 48 who complains of obesity, What
comorbidities might you expect NOT to improve with weight loss?
a) Type II diabetes.
b) Hypertension.
c) Gastroesophageal reflux.
d) Cholesterol.
e) COPD.

ANSWER:E

15- A smooth, rubbery 3 cm lesion is removed from the breast of a 35-year-old


woman with a preoperative diagnosis of fibroadenoma. Histologically, this lesion is
found to be a phyllode tumor. Appropriate management at this time is
a) observation only.
b) reexcision of the area with a 1 cm margin.
c) total (simple) mastectomy.
d) total mastectomy with axillary dissection.
e) tamoxifen therapy.

ANSWER:B

16- Which is true regarding operations for morbid obesity?


a) Bacterial overgrowth in the bypassed segment is a complication of jejunoileal
bypass
b) Dumping symptoms are greater following a vertical banded gastroplasty than
after a Roux-en-Y gastric bypass
c) Following Roux-en-Y gastric bypass, the patient may be permitted to resume
normal eating habits
d) Long-term weight loss is not sustained after a jejunoileal bypass operation
e) Ulceration in the bypassed antrum has been a problem after Roux-en-Y gastric
bypass operations

ANSWER:A

17- Colorectal pseudo-obstruction has been associated with all of the following
EXCEPT
a) excess parasympathetic tone
b) malignant infiltration of the celiac plexus
c) neuroleptic medications
d) opiate usage
e) severe metabolic illness

ANSWER:A

Dr. wessam alzaidat - General surgery


18- During brief endoscopic abdominal procedures,the largest reserve of body buffers
to absorb CO2 is found in
a) bone
b) kidney
c) liver
d) lung
e) straited muscle

ANSWER:A

19- Abdominal insufflation to an intraabdominal pressure of 15 mmHg produces all


of the following effects EXCEPT
a) decreased cardiac output
b) decreased glomerular filtration rate
c) increased left heart filling pressure
d) increased peak inspiratory pressure
e) increased right heart filling pressure

ANSWER:A

20- Regarding surgery for ulcerative colitis all are true EXCEPT:-
a). 30% patients with total colitis will require surgery within 5 years.
b). Panproctocolectomy and pouch formation is appropriate as an elective operation.
c). Pouches can be fashioned as 'S' 'J' or 'W' loops.
d). Over 90% patients with a pouch have perfect continence.
e). With a pouch the mean stool frequency is about 6 times per day.

ANSWER:D

21- Six weeks after undergoing a laparoscopic Roux-en-Y gastric bypass, a 42-year-
old man develops intolerance of solids and most liquids. Pre-operatively, he weighed
135 kg and he had a BMI of 45 kg/m2. His current weight is 114 kg and his BMI is
38 kg/m2. Consumption of a three-ounce meal leads to dysphagia and vomiting. He is
referred for an upper endoscopy. The endoscopic can’t go through the gastro-
jejunostomy. Which of the following is the most appropriate therapy?
a) Nasogastric decompression of the gastric pouch and proximal small intestine.
b) Take down of the gastric bypass with small bowel resection and redo-
gastrojejunostomy.
c) Aggressive fluid resuscitation with normal saline.
d) Dilation of the gastrojejunostomy anastomosis followed by observation and
supportive care.
e) Administrations of a proton pump inhibitor and antibiotic therapy for possible
Helicobacter pylori with repeat endoscopy in six weeks

ANSWER:D
Dr. wessam alzaidat - General surgery
22- After insertion of the veress needle, ways to assume correct position of the needle
include:
a) Aspirate with a syringe to see what returns.
b) Inject 5cc saline and try to immediately get the fluid back.
c) Leave a drop of saline on the hub of the needle and see if it flows freely into the
abdomen.
d) Hook up the insufflator and see if low pressure flow results.
e) All of the above methods can, and usually should, be used.

ANSWER:E

23-Concerning base skull fracture of the anterior fossa all of the following is true
except:
a) Epistaxis.
b) CSF rhinorrhea.
c) Blindness.
d) Raccon eyes.
e) Battle sign.

ANSWER:E

24- One of the followings is true regarding Undescended testes:


a) Incidence is 1 in 10 000 boys
b) Most common location is the femoral region
c) Is associated with high rate of infertility
d) Orchidopexy is indicated at five years age
e) Increased risk of malignancy

ANSWER:E

25- Persistent, conjugated hyprebilirubinaemia may be caused by all of the


followings, EXCEPT:-
a) Alpha-1- antitrypsin deficiency
b) Hypothyroidism
c) Hemolytic disease
d) Cytomegalovirus infection
e) Cystic fibrosis

ANSWER:C

26- Neoadjuvant combined chemotherapy and radiation therapy in rectal carcinoma:


a) Is not effective in down staging tumors and should not be considered in patients
with fixed lesion.
b) Has been proven to be effective in patients with tumors less than 3 cm in diameter.
Dr. wessam alzaidat - General surgery
c) Should be followed by definitive operation in eight to ten weeks.
d) Is well tolerated by most patients but often result in higher operative morbidity.
e) Decrease local recurrence rate when compared with postoperative radiation
therapy.

ANSWER:E

27- About gastric lymphoma one is TRUE:


a) Gastrointestinal bleeding is the most common symptom.
b) Endoscopic biopsy establishes the diagnosis in all cases.
c) Primary therapy is surgery.
d) Primary therapy is radiation.
e) The long term survival is similar to the adenocarcinoma.

ANSWER:C

28- In patients with carotid artery disease:-


a) A bruit is a reliable sign of the degree of stenosis.
b) Atheroma is most commonly seen in the external carotid artery.
c) An embolic event often results in an ipsilateral hemiplegia.
d) Prophylactic aspirin reduces the risk of a stroke.
e) Surgery is of proven benefit in those with asymptomatic stenoses.

ANSWER:D

29- Which of the following statements regarding malignant hyperthermia is TRUE?


a) It is inherited as an autosomal recessive trait.
b) It is more common in the elderly than in children.
c) Patients with a family history of malignant hyperthermia cannot safely undergo
elective surgery if general anesthetics are to be used.
d) Termination of the procedure and intravenous use of danazol are the recommended
treatment for patients experiencing an acute intraoperative episode of malignant
hyperthermia.
e) The pathogenesis is related to intramuscular calcium transport

ANSWER:E

30- Oxygen delivery is dependent on all of the following EXEPT:-


a) Cardiac output
b) Hemoglobin
c) Oxygen saturation
d) Arterial oxygen pressure
e) Metabolic acidosis

ANSWER:E
Dr. wessam alzaidat - General surgery
31. The major indications for performing orchidopexy.
a) To enhance fertility.
b) To prevent the likelihood of development of cancer.
c) To repair a concomitant hernia.
d) To reduce the likelihood of torsion, trauma, and pain.
e) For psychological effect and cosmesis.

ANSWER:A OR D
(B) Not to prevent but to early detect and dx early malignancy

32. Clinical features of blunt trauma of children, All of the following are correct
EXCEPT:

a) The most common cause of death and disability following severe trauma in
children, is related to CNS injury.
b) Thoracic injuries in children are second to head injury as a cause of mortality.
c) Massive hemothorax is the most common indication for urgent thoracotomy in the
injured child.
d) CT scan of the abdomen is superior to both U/S and Isotope scan, as it provides
more information and clearer images.
e) Flail chest is primarily a problem of children under 12 years of age.

Answer:E
33. All the following are manifestations of hepatocellular failure EXCEPT:
a) Gynecomastia
b) Polycythemia
c) Hypoalbuminemia
d) Secondary hyperaldosteronism
e) Increased sensitivity to morphine

Answer:B

34.Radiotherapy plays an important role in the treatment of the following tumors


except
a) Rectal carcinoma
b) Wilms tumor
c) Gastric carcinoma
d) Medulloblastoma (intracranial)
e) Hodgkin’s diseas

Answer:C

35. In patient receiving assisted ventilation with positive end expiratory pressure
Dr. wessam alzaidat - General surgery
(PEEP).
The sudden occurrence of hypotension most likely caused by
a) Hypovolemia
b) Acute congestive cardiac failure
c) Haemothorax
d) Massive atelactasis
e) Tension pneumothorax

Answer:E

36. The most reliable indicator of the adequacy of burn resuscitation is.
a) Central venous pressure.
b) Pulmonary capillary wedge pressure.
c) Urine output.
d) Blood pressure and heart rate.
e) Mental status

Answer:C

37.Infection affects wound healing by all the following mechanisms except:


a) Prolonging oedema.
b) Decreasing tissue PO2.
c) Increasing collagenolysis.
d) Decreasing the inflammatory phase.
e) Increasing the inflammatory phase.

Answer:D

38. In females with UTI the best way to collect urine for culture is
a) Mid stream urine
b) Suprapubic aspiration
c) Initial stream
d) By catheterization
e) By cleaning the area and any stream

Answer:D

39. When a renal mass identified by I.V.P., features suggestive of malignancy include
all except:
a) Calcifications within the mass
b) Increase tissue density
c) Irregularity of the margin
d) Decrease tissue density
e) Distortion of the collecting system

Dr. wessam alzaidat - General surgery


Answer:D

40. Contraction of bladder as a whole generally requires stimulation by


a) Sympathetic nerve
b) Parasympathetic nerve
c) Somatic nerve
d) It contracts spontaneously
e) Sympathetic and parasympathetic

Answer:B

41. Microscopic BPH describes


a) Enlarged” prostate
b) a proliferative process of the stromal and epithelial elements of the prostate
c) a proliferative process of the stromal elements
d) a proliferative process of the epithelial elements
e) prostate size should be more than 40 gm
Answer:B
42. Chronic scrotal pain is most often due to
a) Testicular torsion
b) Trauma
c) cryptochidism
d) hydrocele
e) orchitis

Answer:D

43. The most common complication after TURP is


a) Failure to void
b) Hemorrhage requiring transfusion
c) Clot retention
d) UTI
e) TURP syndrome

Answer:A

44. What is the most sensitive test for identifying residual freagments after PCNL
a) Nephrestomytubogram
b) MRI
c) Ultrasonography
d) Noncontrast CT
e) CT with contrast

Answer:D

Dr. wessam alzaidat - General surgery


45. The standered method of urinary tract reconstruction during renal transpantationis
a) ureteropyeloplasty
b) ureteroureterostomy
c) ureteroneocystostomy
d) vesicopyeloplasty
e) cutaneous ureterostomy

Answer:C

46. In electrolyte disorders that occur when jejunum is used for urinary intestinal
diversion, one is NOT happened:
a) hyponatremia
b) hyperchloremia
c) hyperkalemia
d) azotemia
e) acidosis

Answer:B

47. The landmark in retroperitoneoscopy:


a) Psoas muscle
b) Renal artery
c) Renal vein
d) Ureter
e) Transversalis muscle

Answer:A

48. In Renal Trauma, one is true:


a) The degree of hematuria and the severity of the renal injury correlate consistently
b) All blunt trauma patients with gross hematuria should undergo renal imaging.
c) Renal image is not mandatory in penetrating injuries with microscopic hematuria
d) The preferred imaging study for renal trauma is non-contrast-enhanced CT
e) patients with microscopic hematuria and hemodynamically stable should undergo
renal imaging

Answer:B

49. Definitive diagnosis of a ruptured bladder is made by performing a (n):


a) IVP.
b) Ultrasound of the abdomen.
c) CT of the abdomen.
d) CT cystogram.
e) VCU.

Dr. wessam alzaidat - General surgery


Answer:D

50. Adequate urine output for adult postoperative surgical patients is greater than
a) 35 ml|hr regardless of body size
b) 50 ml|hr regardless of body size
c) 0,5 ml|kg|hr
d) 1,0 ml|kg|hr
e) 1,5 ml|kg|hr

Answer:C

51. For pediatric patients with appendicitis, which of the following statements is
NOT true?
a) The rate for misdiagnosis is highest in children under 3 years old.
b) CT scan is less accurate in children than adults.
c) CT scan may be used to rule out peri-appendiceal abscess.
d) Urinary sepsis is the commonest misdiagnosis.
e) WBC count does not effectively differentiate perforated from nonperforated
appendicitis.

Answer:D

52. A 38-year-old man has a painless, nontender mass in his left neck that moves with
swallowing. Fine-needle aspiration shows medullary carcinoma. The right side
appears normal. The best course of action would be
a) Left thyroid lobectomy
b) Subtotal right lobectomy and left lobectomy
c) Total thyriodectomy
d) Total thyroidectomy with central neck dissection
e) Total thyroidectomu with radical neck dissection

Answer:D

53. Bacteremia in patients with biliary tract sepsis is most likely due to
a) Bacteroides Fragilis
b) Enterobacter
c) Escherichia coli
d) Enterococcus
e) Coagulase-negative Staphylococcus

Answer:C

54. A 24-year old man undergoing laparotomy for symptomatic Crohn’s disease has a
2-cm stricture of the mid-ileum without any evidence of disease elsewhere in the
gastrointestinal tract. The best surgical option for this lesion would be
Dr. wessam alzaidat - General surgery
a) Resection with primary anastomosis
b) Heineke-Mikulicz strictureplasty
c) Heineke-Mikulicz strictureplasty with biopsy
d) Mechanical dilatation
e) Isoperistaltic side-to-side strictureplasty

Answer:A Or C

55. Incompatible blood transfusion during surgery under general anesthesia, which
of the following is NOT true.
a) Unexplained bleeding.
b) Hematuria may be present.
c) Skin rash may be seen
d) Hypertension is well know problem in spite transfusion
e) Volume support is mandatory.

Answer:D

56. In regard to ventilation in pediatric age group, which of the following is the most
deleterious?
a) CPAP.
b) High volume ( Volutrauma).
c) High pressure ( Barotrauma)
d) PEEP.
e) High ventilatory rate.

Answer:B

57. Regarding urethral injuries, all are true except:


a) Prostatic hematoma may be present.
b) Perineal hematoma may be seen.
c) Fracture pelvis may be the cause.
d) Descending urethrogram is the investigation of choice.
e) May be treated with suprapubic urinary bladder catheterization.

Answer:D
58. Regarding overwhelming post splenectomy sepsis, which of the following is true.
a) Pneumococcal vaccination is not protective.
b) It is common after traumatic splenectomy.
c) Mortality rate is low.
d) It is common after splenectomy for blood diseases.
e) It is common after 10 years post surgery.

Answer:D
59. Regarding abdominal trauma in paediatric age group, which of the following is
Dr. wessam alzaidat - General surgery
true:

a) Clinical examination, stabilization and resuscitation are not of priority.


b) Ultrasound is the diagnostic tool of choice.
c) Blood transfusion should be started immediately.
d) Peritoneal lavage is contra-indicated.
e) Splenectomy is not a routine management for all types of splenic injuries

Answer:E

60. With regard to Hirschsprung’s disease ( aganglionosis), all the following are true
except:
a) It is more common in males.
b) It may be complicated by enterocolitis.
c) Barium enema study may be normal.
d) It is best diagnosed by full thickness rectal biopsy.
e) Surgery can’t be accomplished without colostomy.

Answer:E

61. Which of the followings is not a complication of esophageal atresia and


tracheoesophageal fistula repair?:
a) Esophageal stricture.
b) Anastomotic leakage.
c) Gastroesophageal reflux.
d) Recurrent fistula.
e) Massive variceal bleeding.

Answer:E

62. The most common condition, which needs liver transplantation in pediatric age
group, is:
a) Metabolic liver disease.
b) Malignant liver tumors.
c) Biliary atresia.
d) Choledochal cyst.
e) Viral hepatitis.

Answer:C

63. The least blood loss during burn wound excision occurs when escharectomy is
performed:
a) During the first 48 hours.
b) On days 3-5 post-burn.
Dr. wessam alzaidat - General surgery
c) On days 6-10 post-burn.
d) After day 10 post-burn.
e) When eschar is infected.

Answer:A

64. Contraindications to breast reconstruction after mastectomy include:


a) Diabetes Mellitus.
b) Age over 60 years.
c) Obesity.
d) Bilateral breast cancer.
e) None of the above.

Answer:E

65. All the following muscles are supplied by the trigeminal nerve EXCEPT:
a) Tensor veli palatini.
b) Tensor tympani.
c) Masseter.
d) Posterior belly of digastric.
e) Mylohyoid.

Answer:D

66. The advantages of early excision and graft for full-thickness burns in the first 48
hours post-burn include all of the following EXCEPT:
a) Less blood loss.
b) Less pain.
c) Improved cosmetic result.
d) Decreased hospital stay.
e) Ease of differentiation between partial and full-thickness burns.

Answer:E

67. All of the following statements are correct concerning Corticosteroids EXCEPT:
a) Inhibit fibroblast migration into the wound.
b) Inhibit the activity of prolylhydroxylase.
c) Prevent collagen deposition.
d) Inhibit wound contracture.
e) Vitamin A restores the harmful effects of steroids.

Answer:C*

68. The burn that is difficult to estimate in percentage is the:


a) Electrical burn
Dr. wessam alzaidat - General surgery
b) Sun burn
c) Scald burn
d) Direct flame burn
e) Chemical burn

Answer:A

69. All of the following are signs of burn inhalation injury EXCEPT:
a) Erythema of oral mucosa
b) Coughing
c) Black sputum
d) Red colored burn
e) Burn of nasal hair

Answer:D

70. A blow-out fracture of the orbit is by definition:


a) A fracture that results in rupture of the globe.
b) A fracture that results in disruption of the orbital rim.
c) A fracture of the thin walls of the orbit allowing herniation of orbital
contents.
d) A fracture that results from an explosion.
e) A fracture that includes bones adjacent to the orbit.

Answer:C

71. A patient who has a penetrating wound to the extremity has the arteriogram
shows less than 5mm intimal defects and psudoaneurysm. Management of this injury
should be
a) immediate operation
b) Anticoagulant for 6 wks.
c) Observation
d) Elective repair
e) Antiplatelet therapy for 6 wks.
Answer:C DR.WBZ 72. Which
of the following statements about carotid endarterectomy is NOT true?
a) Patients with a symptomatic critical stenosis benefit from carotid
endarterectomy
b) The risk of contralateral stroke is reduced after carotid endarterectomy
c) Operative mortality is less than 2%
d) It prolongs survival when compared with patients treated medically
e) It should be accompanied by antiplatelet medication

Answer:D

Dr. wessam alzaidat - General surgery


73. Which of the following is not associated with left-sided portal hypertension
a) Gastric varices
b) Abdominal pain
c) Splenomegally
d) Hepatomegally
e) Chronic pancreatitis

Answer:D

74. A 15 years old boy complains of excessive sweating of his hands. They are cold,
clammy, and damp to the touch. Perspiration dripping from his hands forms a small
puddle during the examination. The remainder of medical history is unremarkable.
Which of the following statements is TRUE
a) the result of sympathectomy for this disorder are generally good
b) the sweat glands involved here are apocrine
c) if not treated, this will likely be permanent
d) topical aluminum salts create ulceration
e) a short course of methotrexate is ofen helpful

Answer:A

75. Which of the following statements about blunt tracheobronchial trauma is true
a) Repair should include a prophylactic tracheostomy
b) Delayed repair is preferred
c) Aneasthesia is safer if the endotracheal tube can be passed beyond the site of
injury
d) Massive hemoptysis is common at presentation
e) Bronchoscopy is hazardous and should be avoided

Answer:C

76. Regarding the functions of a tracheostomy all are true, except:


a) Bypasses an upper airway obstruction.
b) Increases the anatomical dead space.
c) Decreases airway resistance.
d) Protects against aspiration.
e) Allows frequent airway suction.

Answer:B

77. With regard to protein loss after injury, which of the following statements is true?
a) It results from impaired protein synthesis.
b) It occurs primarily from catabolism of skeletal muscle.
c) It occurs primarily from acute renal failure.
d) It occurs primarily from the site of injury.
Dr. wessam alzaidat - General surgery
e) It can be prevented by total parenteral nutrition.

Answer:B

78. Which of the followings is the most important stimulus for triggering the
endocrine response to injury?
a) Tissue acidosis.
b) Local wound factors.
c) Afferent nerve stimuli from the injured area.
d) Hypovolemia.
e) Temperature changes.

Answer:C

79. All of the followings are true regarding perforated appendicitis, except:
a) Higher rate in children.
b) Higher rate in old patients.
c) Usually due to delay in presentation and diagnosis.
d) Appendicular mass is felt in more than 50% of the cases.
e) The commonest complication is wound infection.

Answer:D
80. Factors which contribute to wound dehiscence include all the followings, except:
a) Old age.
b) Coughing.
c) Hypoproteinemea.
d) Anaemia.
e) Malignancy.

Answer:D

81.The lesions in familial adenomatous polyposis with the smallest malignant


potential are:

a) Gastric polyps
b) Duodenal polyps
c) Abdominal desmoids
d) Brain tumors
e) Ileal polyps

Answer:A

82. Regarding hernias All true except


a) One portion of the bowel wall is herniated , but not entire lumen is
called Richter hernia
Dr. wessam alzaidat - General surgery
b) Spigelian hernia is Associated with the semilunar line
c) Femoral hernias are more common in females than males
d) Paradoudenal hernia is an internal hernia where the superior
mesenteric vein forms part of its boundaries
e) Obturator hernia Causes anterior thigh pain with walking

Answer:D

83. All of the following are consequences of obstructive jaundice except


a) Wound healing is impaired
b) The function of kupffer cells is increased
c) There is a decrease in the absorption of fat soluble vitamins
d) There is an increased risk of renal failure
e) There is an increased chance of the bile being infected in
malignant obstruction
Answer:B

84. In patients with upper gastrointestinal haemorrhage due to a peptic


ulcer All true except
a) A visible vessel in the ulcer bed at endoscopy is a risk factor
for rebleeding
b) An older patient would be expected to have a higher chance
of rebleeding
c) An adherent clot seen at endoscopy reduces the risk of
rebleeding
d) H2 antagonists do not reduce the bleeding rate
e) A gastric ulcer is more likely to bleed than a duodenal ulcer

Answer:C

85.In the management of trauma patient all true except


a) Mouth to mouth breathing provides a maximum inspired
oxygen concentration of 10%
b) Criciod pressure should be not be used to aid endotrachial
intubation in vomiting patients.
c) Nasogastric intubation is contraindicated in patients who are
suspected to have fracture of the base of skull
d) Needle cricothyroidotomy aids effective ventilation for up to
30 minutes
e) Insertion of (oropharyngial airway) should be attempted in
only on unconscious patients

Answer:A

86. Local anesthetic agents


Dr. wessam alzaidat - General surgery
a) Should not be given intravenously
b) Lignocaine (xylocaine) is four times more potent than
bupivacaine
c) The maximum safe dose for Lignocaine is 3mg/kg and up to
7mg/kg if mixed with adrenaline
d) Can be used safely in the distal limbs even if mixed with
adrenaline
e) Bupivacaine is less cardiotoxic than lignocaine

Answer:C

87. Regarding anal fissures, all are true except:


a) 10% occur in the posterior midline
b) Multiple fissures suggests the diagnosis of TB or crohn's
disease
c) 50% of the fissures heal with the use of bulking agent
d) Sphinterotomy has a success rate of over 90%
e) Sphinterotomy is associated with minor fecal incontinence in
over 15% of patients

Answer:A

88. All of the following are indications for surgery in patients with
spontaneous pnuemothorax except;
a) Scuba divers
b) Individuals living in remote area
c) Airline pilots
d) Age more than 60 years
e) Previous contralateral pneumothorax

Answer:D

89. Low molecular weight Heparin as its primary inhibitory effect on one of
the following:
a. Factor II.
b. Factor IXa.
c. Antithrombin III.
d. Factor Xa.
e. Factor XIIa.

Answer:D
Dr. wessam alzaidat - General surgery
90. The O2 dissociation curve is shifted to the right by :
a) Decreased CO2 tension
b) Increased CO2 tension
c) Increased PH
d) Increased N2 tension
e) Decreased N2 tension
Answer:B
91. The Glasgow coma scale is the dependant upon all of the following
except
a) response to speech
b) respnse to pain
c) response of the pupils
d) motor response
e) response of the patient

Answer:C

92. In traumatic subarachnoid haemorrhage all of the following are correct


except .
a) is commoner than aneurysmal haemorrage
b) is commoner than subarachnoid haemorrhage associated with AVM.
c) may cause vasospasm and cerebral infarction
d) is often associated with frontal skull fracture
e) is more common in elderly people

Answer:D

93. The adverse prognostic factors for the development of acute subdural
hematoma involve all of the following except
a) old age
b) young age
c) chronic alcoholism
d) skull fracture
e) temporal agenesis

Answer:B

94. Extradural hematoma has the following characters except


a) is more common than acute subdural hematoma
b) is more often associated with vault skull fracture
c) is less often associated with primary brain injury
d) is more like to expand
e) is more likely to be arterial in origin

Dr. wessam alzaidat - General surgery


Answer:A

95. The following are indication for admission to the hospital after a minor
head injury except
a) reduce level of consiousness
b) concussion without skull fracture
c) clinical and radiological evidence of skull frature
d) focal neurological dysfunction
e) difficulty in assessing the patient

Answer:B

96. Which anastamosis location has the highest leakage rate?


a) Jejunojejunostomy after a Roux-en-Y reconstruction
b) Pancreaticojejunostomy
c) Esophagojejunostomy
d) Rectosigmoid anastamosis
e) Gastrojejunostomy

Answer:B

97. Bilibrubin
a) Is conjugated to glucuronic acid in the gallbladder
b) Is transported in hepatic sinusoidal blood bound to albumin
c) When conjugated, is secreted into bile by passive diffusion
d) Is converted to urobilinogen by jejunal enterocytes
e) Is produced predominantly by early phase (< 3 days)
erythrocyte heme breakdown

Answer:B
98.Exclusion criteria for laparoscopic colectomy for diverticulitis include all of
the following EXCEPT
a) Multiple areas of colonic involvement
b) Purulent peritonitis
c) BMI > 30 kg/m2
d) Free peritoneal perforation
e) Multilocular abscess

Answer:C

Dr. wessam alzaidat - General surgery


99. 48 hours after performing a laparoscopic cholecystectomy, a surgeon
realizes that one of the clips was placed across the common bile duct. The
correct approach would be

a) Drainage via percutaneous transhepatic cholangiogram


b) Operative removal of the clips
c) Choledochoduodenostomy
d) Loop choledochojejunostomy
e) Roux-en-Y choledochojejunostomy

Answer:E

100. The ilioinguinal nerve

a) Is a branch of the femoral nerve


b) Has a motor component
c) Can be injured easily at the internal spermatic ring
d) Provides sensation for the penis and upper scrotum
e) Runs along the hypogastric vein

Answer:D

Dr. wessam alzaidat - General surgery


MCQs for Residents exam
1.Hyperglycemia in a surgical patient receiving TPN may best be managed by

a. Oral hypoglycemic drugs

b Decreasing the dextrose load and doubling the amount of fat

c. Adding regulalar insulin to the TPN

e. Increasing concentration of protein and carbohydrate calories and decreasing


that of lipids

ANSWER:C

2.which amino acids can be metabolized outside the liver and are a local source of
energy For muscle?

a. Leucine, isoleucine, valine


b. Alanine, argenine, lysine.
c. Ethionone, glutamine, lysine.
d. Phenyalanine,tyrosine,histidine.
e. None of the above.

ANSWER:A

3. The gastrointestinal tract can secrete and absorb how much water in the
form of gastric juices per day in 70-kg adult male?
a. 1-2 L/day
b. 4-5 L/day
c. 6-7 L/day
d. 8-10 L/day
e. 50 L/day
ANSWER:D

4. The protein –sparing effect of glucose administration begin to be


manifested after administration of how much glucose?
a. 1 L of 5% dextrose in water (DƽW)
b. 2 L of (DƽW)
c. 3 L of(DƽW)

Dr. wessam alzaidat - General surgery


d. 4 L of (DƽW)
e. 5 of (DƽW).

ANSWER:B

5. TNF-α release :
a. can be effectively blocked by anti-TNF-α antibodies to halt systemic
inflammatory syndrome ( SIRS).
b. Doesn't have any beneficial effects in the early phases of the
inflammatory response.
c. Is primarily from leukocytes
d. Promotes polymorphonuclear (PMN) cell adherence and further
cytokines release.
e. Is always deleterious.

ANSWER:D

6. you suspect that a patient has ARF secondary to hypovolemia.


All of the following are appropriate initial treatment except:
a. check the hemoglobin level
b. Give intravenous boluses.
c. Start vasopressor infusion to keep mean arterial pressure greater
than 65 mm Hg
d. Calculate the fractional excretion of sodium
e. Rule out causes of outflow obstruction.

ANSWER:C

7. Which one of the following suggest an acute adrenal crisis:


a. Random cortisole level of 34 mcg/dl
b. Hypothermia
c. Hyperglycemia
d. Hypokalemia
e. Increase cortisole of 5 mcg/dl after stimulation with cosyntropin

ANSWER:E

Dr. wessam alzaidat - General surgery


8. The syndrome of multi-organ-failure (MOF):

a. Involve sequential insults that lead to systemic hyperinflammation


b. Require the documentation of active infection
c. Has decreased in incidence over the past decade
d. Require diagnosis within 3 days of the systemic insult
e. Demonstrate consistent improvement after blood transfusion.

ANSWER:A

9. The systemic parameters in the definition of SIRS include all of the following
except:
a. Temperature lower than 36ºC
b. Respiratory rate greater than 20 breath/mint
c. Paco2 less than 32 mm Hg
d. Systolic blood pressure lower than 90 mm Hg
e. Heart rate greater than 90 beats/mint.

ANSWER:D

10. Strategies that have been suggested to decrease the risk for
postoperative
Pulmonary complications include all of the following except:

a. Routine nasogastric tube decompression


b. Lung expansion maneuvers
c. Preoperative smoking cessation.
d. Postoperative epidural anesthesis
e. use of intraoperative short-acting neuromuscular blocking agents.

ANSWER:A

11. All of the following are true concerning the sympathetic nervous system
except:
a. Circulating epinephrine is produced mainly in the adrenal glands and
secreted as a hormone.

Dr. wessam alzaidat - General surgery


b. Most circulating norepinephrine is derived from synaptic nerve clefts.
C. Activation of the sympathetic nervous system result in vasoconstriction,
Tachycardia, and tachypnea.
d. Norepinephrine acts as neurotransmitter.
e. Up to 5% of norepinephrine and 15% of dopamine are produced by the
enteric
nervous system.

ANSWER:E

12. Acute repiratory syndrome (ARDS), histologic examination of an alveolar


biopsy
In the first 24 hours would demonstrate.
a. influx of protein-rich leukocyte.
b. Preservation of type II pneumocytes.
c. Bacterial colonization.
d. Alveolar hemorrhage.
e. High level of collagen and fibronectin.

ANSWER:A

13. which of the following statements regarding the inflammatory phase of the
wound healing.
a. it last up to 24 hours after the injury is incurred.
b. Initial vasodilation is followed by subsequent vasoconstriction.
c. Bradykinin causes vasoconstriction, which inhibits migration of neutrophils
to the healing wound.
d. The complement component C5a and platelet factor attract neutrophil to
the wound.

e. the presence of neutrophil in the wound is essential for normal wound


healing.

ANSWER:D

14. Which of the following statements is true about growth factors.

Dr. wessam alzaidat - General surgery


a. Epidermal growth factor (EGF) stimulates the production of collagen.
b. Vascular endothelial growth factor (VEGF) and PDGF both stimulate
angiogesis
by binding to a common receptor.
c. Fibroblast growth factor (FGF) stimulate wound contracture.
d. Transforming growth factor-B (TGF-B) is stored in endothelial cells.
e. Tumor necrosis factorα (TNF-α) inhibits angiogenesis.

ANSWER:B

15. Which of the following statements regarding scar revision is true?


a. scar maturation refers to the change in size of the wound in the first 1-2
months
b. scar revision should have been performed in the first 3 months after injury
to minimize fibrosis.
c. revision should be performed earlier in children than in adult
d. It corrects undesirable pigmentation
e. Scar revision should be delayed approximately 1 year to allow maturation.
ANSWER:E

16. Which of the following statements regarding wound healing is true?


a. Vitamin A is needed for hydroxylation of lysine and proline.
b. High doses of vitamin C improve wound healing
c. Vitamin E is involved in the stimulation of fibroplasias, collagen cross-linking.
d. Zinc deficiency results in delayed early wound healing.
e. Iron deficiency had been linked to defects in long-term wound healing.

ANSWER:D

17 . Which portion of the cell cycle in actively dividing cells is most sensitive to
Ionizing radiation?
a. S phase
b. M phase
c. G1 phase
d. G2 phase
e. All phases are equally radiosensitive.

Dr. wessam alzaidat - General surgery


ANSWER:B

18. Which of the following statements regarding chromosome is not true?


a. The nucleus contains the entire cellular DNA
b. Histones compact and organize the DNA strands.
c. interactions between DNA and proteins expose specific genes and control
their expression.
d. During mitosis, the spindle apparatus attaches to the chromosome at
centromere.
e. Telomeres maintain chromosomal length replication cycles .

ANSWER:A

19. In DNA replication, what type of mutation is specifically associated with


generation
Of a stop codon?.
a. Point mutation
b. Missense mutation.
c. Nonsense mutation
d. Frameshift mutation
e. Neutral mutation.

ANSWER:C

20. Which of the following condition is associated with an isolated prothrombin


time
(PT) prolongation.

a. Von willebrand disease.


b. Factor VIII deficiency ( hemophilia).
c. Common pathway deficiency ( factor II, V, X, and fibrinogen).
d. Therapeutic anticoagulation with warfain
e. Therapeutic anticoagulation with heparin.

ANSWER:D

Dr. wessam alzaidat - General surgery


21. Regarding complication of blood transfusion, which is true?
a. Febrile reaction is rare.
b. Gram-positive organisms are the most common contaminants of stored
blood.
c. Screening for minor antigens should be repeated every week when multiple
transfusions are given.
d. A small amount (more than 0.1 ml air) is well tolerated.
e. Malaria, chagas disease,( HTL-V-1 virus,) ( AIDS) and hepatitis can be
transmitted
by blood transfusion.

ANSWER:E

22. which of the following statements is true regarding the stored blood?
a. Packed RBC stored in additive solution (AS-3) and kept at 4ºC are suitable
for transfusion for 3 months.
b. Platelets in banked blood retain their functions for 3 days.
C. Factors II, VII, IX, and XI are stable at 4ºC.
d. A decrease in RBC oxygen affinity occurs during storage as a result of
a decrease in 2,3-DPG levels.
e. There a significant rate of hemolysis in stored blood.
ANSWER:C

23. Which of the following statements regarding daily fluid balance is incorrect?
a. Daily water intake is 2000-2500ml.
b. Average stool loss, 1000ml.
c. Average insensible loss, 600ml.
d. Average urine volume, 800ml-1500ml.
e. Average increase in insensible loss in febrile patient, 250 ml/day for each
degree
of fever.
ANSWER:B

24. Which of the following statements concerning Na +concentration of various


fluids

Dr. wessam alzaidat - General surgery


Is incorrect.
a. pancreatic secretion, 140 mEq/L
b. Sweat, 40 mEq/L
C. Gasric secretion, 50 mEq/L
d. Saliva, 100mEq/L
e. Ileostomy output, 125 mEq/L.

ANSWER:D

25. Which of the following condition is not associated with hypernatremia?


a. Diabetes insipidus
b. Tumor lysis syndrome.
c. Steven-Johnson syndrome.
d. Primary hypodepsia.
e. Enterocutaneous fistula.
ANSWER:B

26. Which of the following clinical signs or symptoms is not associated with
serum Na+
Concentration below 125 mEq/L?
a. Headache.
b. Hallucination.
c. Bradycardia
d. Hypoventilation
e. Hyperthermia

ANSWER:E

27. Which of the following (ECG) findings is not associated with hyperkalemia?
a. Peaked T waves
b. Prolong PR interval
c. Loss of P wave
d. Narrowing of QRS complex.
e. T waves higher than R waves in more than one level.

ANSWER:D

Dr. wessam alzaidat - General surgery


28. Which of the following is least useful in the immediate treatment of
hyperkalemia?
a. Calcium salts.
b. NaHco3
c. Potassium-binding resins.
d. Glucose and insulin
e. Hemodylesis
ANSWER:C DR.WBZ

29. With regard to hypokalemia, which of the following statements is not true?.
a. K+ and H+ are exchanged for Na+ in renal tubule.
b. Respiratory acidosis is associated with increase renal K+ loss.
c. hypokalemia can cause decrease deep tendon reflexes.
d. Flattened T waves and a prolonged QT interval are associated with
hypokalemia.
e. Intravenous K+ administration should not exceed 40 to 60 mEq/hour.

ANSWER:B

30. Which of the following clinical scenario is not associated with acute
hypocalcemia?
a. Fluid resuscitation from shock.
b. Rapid infusion of blood products.
c. Improper administration of phosphates.
d. Vitamin D-deficient diets.
e. Acute pancreatic.

ANSWER:D

31. with regard to acid-base buffering. Which of the following is false?


a. The major extracellular buffer is Hco3
b. Intracellular PH and extracellular PH are usually the same.
c. The major intracellular buffer consists of protein and phosphate salts.
d. H+ cannot directly pass though cell membrane.
e. Treating acidosis with Hco3 infusion can cause cell death.

Dr. wessam alzaidat - General surgery


ANSWER:B

32. Which of the following statement is not correct with regard to cardiac
output(CO)
a. CO alone is not an indicator of myocardial contractility.
b. Ventricular end-Diastolic volume (EDV),vascular resistance, and myocardial
contractility determine stroke volume (SV).
c. Arterial blood pressure alone is an accurate indicator of CO.
d. CO varies directly with pulse rate of up to 160 beats/min in sinus rhyhtm,
after
which it decrease.
e. Atrial contraction contributes up to 30% of EDV.

ANSWER:C

33. With regards to ventilator mechanics, which of the following statements is


false?.
a. The work of breathing at rest consume 2% of total oxygen consumption.
b. COPD is associated with increase in the work breathing as a result of increase
inspiratory work.
c. The work of breathing may increase to 50% of total oxygen consumption in
postoperative patients.
d. Airway pressure reflects the compliance of the chest wall and diaphragm
as well as that of lungs.
e. Comliance is measured as the change in volume divided by change in
pressure.
ANSWER:B

34. With regard to Multi organ Failure(MOF). Which of the following statement
is false?.
a. Sepsis is the major risk
b. Injury to the microvascular endothelium is uniformly present.
c. Neutrophil-mediated injury is dependent on adherence to the microvascular
endothelium .
d. There is a bimodal pattern to the development of MOF.
e. An increase in the gastrointestinal barrier is often present.

Dr. wessam alzaidat - General surgery


ANSWER:E

35. Regarding pelvic , fracture which is true?


a. mortality rate is 30% if hypotension present.
b.A pubic symphysis diastasis of 3 cm double the volume of potential pelvic
space
for hematoma.
c. It is imperative to make large surgical incision from xiphoid to symphysis to
allow
better visualization of the pelvis.
d. The most common cause of death in pelvic fracture is overwhelming sepsis.
e. The external iliac artery is commonly involved in severe pelvic fracture.
ANSWER:B DR.WBZ
36. Select the correct statement regarding flail chest.
a. It occur when three or more adjacent ribs are fractured.
b. work of breathing is increased secondary to paradoxical chest wall
infection.
c. Patients with flail chest should be aggressively resuscitated because of
probable
development of pulmonary contusion.
d. Patients with this condition should be prophylactically intubated secondary
to a high likelihood of respiratory failure.
e. If a patient doesn't require mechanical ventilation , it is important to avoid
use of positive end-expiratory pressure.

ANSWER:B

37. A 25 year old male, sustained flame burn, on his right arm
circumferentially,
Bilateral on his legs, and on perineum. What the approximate % TBSA?
a. 28%.
b. 36%
c. 46%
d. 64%
e. 52%

Dr. wessam alzaidat - General surgery


ANSWER:C

38. Which of the following is a potential premalignant precursor of melanoma?


a. Keratoacanthoma
b. Actinic keratosis
c. Seborrheic keratosis
d. dysplastic nevus
e. Bowen disease.

ANSWER:D

39. The preferred diagnostic biopsy method for pigmented skin lesion is:
a. Punch biopsy
b. Incisional biopsy
c. Shave biopsy
d. Excisional biopsy
e. Excision with 0.5 –cm margin.

ANSWER:D

40. The most common histologic type of melanoma is :


a. Superficial spreading
b. Nodular
c. Lentigo
d. Acral lentiginous
e. Desmoplastic

ANSWER:A

41. Which of the following statements is false regarding direct inguinal hernias?
a. The most likely cause is destruction of connective tissue as a result of
physical
stress.
b. Direct hernia should be repaired promptly because of the risk of
incarceration.

Dr. wessam alzaidat - General surgery


c. A direct hernia may be a sliding involving apportion of the bladder wall.
d. A direct hernia may pass through the external ring.
e. An indirect hernia may present as well.

ANSWER:B

42. Inguinal hernia:


a. Are best treated with laparoscopic technique.
b. Should always be repaired
c. Benefit from the use of mesh in reducing recurrence.
d. Are associated with a high incidence of strangulation.
e. Are rare in infants.

ANSWER:C

43.Which of the following is not a characteristic of medullary breast cancer


a. Lymphocystic infiltrate.
b. Benign appearance on ultrasound.
c. High rate of lymph node metastasis
d. statistically better than average prognosis
e. Usually manifest as a palpable mass.

ANSWER:C

44. Which of the following is not an indication for postmastectomy


radiotherapy?
a. T3 Tumors
b. Multicentric DCIS larger than 6 cm
c. inflammatory breast cancer.
d. four or more positive axillary lymph nodes.
e. Gross extranodal extension.

ANSWER:B

45. with regars to phyllodes tumor of the breast, which statement is not true?
a. It is histologically characterized by epithelial cystlike spaces.

Dr. wessam alzaidat - General surgery


b. Examination reveals a firm, moble, well circumscribed mass.
c. 10% to 15% are malignant
d. The benign version can grow aggressively and recur locally.
e. It commonly metastasizes to lymph nodes.

ANSWER:E

46. Regarding molecular marker in breast cancer, which of the following


Statements is correct?
a. Carrier of BRCA2 mutations are more likely to have triple –negative cancer.
b. HER2-positive cancers are unlikely to respond to treatment with
trastuzumab.
c. ER-positive/HER2-negative patient should be treated by endocrine therapy
d. All breast cancer are sensitive to endocrine therapy.
e. Triple –negative patients have an excellent prognosis.
ANSWER:C

47. Which of the following is associated with appearance of invasive lobular


carcinoma
On mammography?
a. Discrete bilateral mass
b. partially cystic appearance
c. Indistinct mass with poorly defined border.
d. mass with microcalcification.
e. Branching pleomrphic calcification.

ANSWER:C

48. what is percentage of false negative rate for sentinel lymph node
biopsy(SLNB)?
a. 1%
b.8%
c. 15%
d. 20%
e. 25%

Dr. wessam alzaidat - General surgery


ANSWER:B

49. With regard to thyroid anatomy, which of the following statements is


incorrect?
a. The inferior thyroid artery arise directly from the external carotid artery.
b. the thyroid ima artery arises directly from the aorta in 3% of cases.
c. The ligament of berry is located near the entry point of RLN.
d. venous drainage of the thyroid gland is via superior, middle, and inferior
branches.
e. The superior and middle thyroid vein drain into the jugular vein.

ANSWER:A

50.Which of the following statements regarding Hurthle cell carcinoma is


incorrect?
a. It represent a subtype of follicular carcinoma
b. Hurthle cell carcinoma account for 3% of all thyroid cancer.
c. It is more likely to be multifocal
d. It demonstrate poor radioactive iodine.
e. Lymph node dissection is indicated for all patients.

ANSWER: E

51.Crohn's disease:
A. Is caused by Mycobacterium paratuberculosis.
B. Is more common in Asians than in Jews.
C. Tends to occur in families.
D. Is less frequent in temperate climates than in tropical ones.
E. Is improved by smoking.
ANSWER:c

52.A 15-year-old boy is admitted with a history and physical findings consistent with
appendicitis. Which finding is most likely to be positive?

A.Pelvic crepitus

B.Iliopsoas sign

C.Murphy sign

Dr. wessam alzaidat - General surgery


D.Flank ecchymosis

E.Periumbilical ecchymosis

ANSWER: b

53.A 50-year-old man is admitted with massive bright red rectal bleeding. He
recently had a barium enema that demonstrated no diverticular or space-occupying
lesion. Nasogastric suction reveals no blood but does produce yellow bile. The
patient continues to bleed. What is the next diagnostic step?

A. Repeat barium enema

B. Colonoscopy

C. Upper gastrointestinal series

D. Mesenteric angiography

E. Small bowel follow-through with barium

ANSWER:D

54.Recurrence after operation for Crohn's disease:


A. Occurs after operations for ileal Crohn's but not colonic Crohn's.
B. Is usually found just proximal to an enteric anastomosis.
C. Rarely requires reoperation.
D. Occurs in 1% of patients at risk per year during the first 10 years after the
operation.
E. Is prevented by maintenance therapy with corticosteroids.
ANSWER: (B )

55.A 15-year-old boy awakens with sudden onset of right lower quadrant and scrotal
tenderness accompanied by nausea and vomiting. Which of the following is the most
appropriate diagnosis and represents a surgical emergency?

A.Acute prostatitis

B.Acute epididymitis

C.Torsion of the testicle

D.Acute appendicitis

E .Gastroenteritis

Dr. wessam alzaidat - General surgery


ANSWER (C )

56.Factors that decrease collagen synthesis include all of the following except:
A. Protein depletion.
B. Infection.
C. Anemia.
D. Advanced age.
E. Hypoxia.

ANSWER.(C)

57.A 47-year-old woman presents with dysphagia to both solids and liquids equally.
She has experienced a 10-kg weight loss over the last several months. A barium
swallow reveals a birdlike narrowing in the distal esophagus. What is the underlying
cause of her symptoms?

A.Disorganized, strong nonperistaltic contractions in the esophagus

B.Failure of the lower esophageal sphincter to relax

C.Hiatal hernia

D.Barrett's esophagus

E.Esophageal stricture secondary to untreated gastroesophageal reflux

ANSWER (B)

58.A 45-year-old male executive is seen because he is vomiting bright red blood.
There are no previous symptoms. The man admits to one drink a week and has no
other significant history. In the hospital, he bleeds five units of blood before
endoscopy. What is the most likely diagnosis?

A.Gastritis

B.Duodenal ulcer

C.Esophagitis

D.Mallory-Weiss tear

E.Esophageal varices

ANSWER (B)

59.45-year-old man is seen in the emergency department after vomiting bright red

Dr. wessam alzaidat - General surgery


blood. He has no previous symptoms. He drinks one alcoholic beverage a day.What
is the most reliable method for locating the lesion responsible for the bleeding?

A.Upper gastrointestinal series

B.Exploratory laparotomy

C.Upper endoscopy

D.Arteriography

E.Radionuclide scanning

ANSWER (c)

60.After several hours in the hospital, he begins to have recurrent bleeding. He is


transferred to a critical care bed and is persistently hypotensive despite trasnfusion of
nine units of packed red blood cells. Which is the most appropriate next step in
management of this patient?

A.Upper endoscopy with attempt at cauterization of bleeding

B.Transport to the interventional radiology unit to identify and embolize bleeding


source

C.Placement of a Blakemore tube to temporarily tamponade bleeding and to allow for


stabilization of blood pressure

D.Laparotomy to control bleeding

E.Infusion of vasopressin and additional units of blood

ANSWER (D)

61.A 25-year-old man is admitted with a history of sudden onset of severe


midepigastric abdominal pain. Upright chest radiograph reveals free intraperitoneal
air. What is the therapy for this patient?

A.Upper endoscopy

B.Barium swallow

C.Gastrografin swallow

D.Observation

E.Laparotomy

Dr. wessam alzaidat - General surgery


ANSWER (E)

62. Concerning severe pancreatitis:

A.Hypocalcaemia is the most common metabolic problem

B. Coagulopathy is usually the first organ system failure to manifest itself

C.Failure of two organ systems is associated with 90% mortality

D. Solid, infected pancreatic necrosis will often respond to intravenous antibiotics

E. Positive end expiratory pressure (PEEP) may be useful in managing respiratory


failure

ANSWER (E)

63.A 55-year-old female patient is evaluated for new onset of diabetes mellitus. Her
medical history is largely unremarkable. Her physical examination is unrevealing
except for the presence of an erythematous skin rash. Her further evaluation should
include an investigation of the possibility of which of the following?

A.Insulinoma

B.Glucagonoma

C.Gastrinoma

D.Carcinoid tumor

E.Pancreatic cholera

ANSWER (B)

DR.WBZ

64.A 60-year-old female patient has a workup for episodic symptoms of palpitations,
nervousness, and bizarre behavior, all of which tend to occur during fasting states.
Biochemically, she is diagnosed as having an insulinoma. What is the best choice for
localizing this tumor?

A.CT scan

B.MRI

C.Selective arteriography

Dr. wessam alzaidat - General surgery


D.Percutaneous catheterization of the portal vein with selective venous sampling

E.Surgical exploration and intraoperative ultrasound

ANSWER (E)

65. A 55-year-old woman with progressive but episodic muscle weakness is


diagnosed as having myasthenia gravis. Her chest radiograph is normal and reveals
no evidence of mediastinal mass or tumor. What is the most definitive treatment that
can be offered this patient?

A.Prednisone

B.Neostigmine

C.Thymectomy

D.Plasmapheresis

E.Atropine

ANSWER (c)

66.A 50-year-old hypertensive man has definitive biochemical evidence of a


pheochromocytoma. Computed tomography (CT) scan and magnetic resonance
imaging (MRI) do not reveal any abnormalities, and m-iodobenzylguanidine
scanning is not readily available. What should be the next step in the management of
this patient?

A.Abdominal exploration

B.Continued clinical observation

C.Mediastinoscopy

D.Selective venous sampling

E.Mediastinal exploration

ANSWER (D)

67.The hemodynamic value characteristic of septic shock is


A.Cardiac index, 2.8 L/min/m2
B. Systemic vascular resistance index, 350 dynes cm-5 sec/m2
C.Oxygen consumption, 135 mL/min/m2
D. Oxygen delivery, 700 mL/min/m2

Dr. wessam alzaidat - General surgery


E. Pulmonary capillary wedge pressure, 6.0 mm Hg

ANSWER (c)

DR.WBZ

Dr. wessam alzaidat - General surgery


MOH 5th year 6 2022

1.complete rectal prolapse in the adult:

a. more common in males


b. is an intussusception of the rectum
c. continence is usually maintained
d. conservative mangmment is usually successful
e. is a complication of IBS

ANSWES:B

2.one true about pulmonary hamartomas:

a.hamartomas are benign chondromas


b.most are located in conducting airways
c.wedge resection is curative
d.lobectomy is necessary to obtain draining hilar lymph nodes
e.hemoptysis is common

ANSWES:C

3.all causes of hyperphosphatemia except:


a. renal failure
b. hyperparathyroidism
c.hyperthyrodism
d. tumor laysis syndrome
e.malignent hyperthermia

ANSWES:E

4.true about communication between staff in OR except:


a.The traditional surgical hierarchy usually aide better communication…..
b.Cultural and lingual barriers between OR personnel may affect communication
c.Surgical sponge count should not be done during critical portions of……
d.Communication bw the surgeon and pathologist is vulnerable by messenger such
as a nurse
e.Poor communication is an important cause of wrong site surgery

ANSWER:B ??

5.During neck exploration for symptomatic primary hyperparathyroidism in a 65 yrs

Dr. wessam alzaidat - General surgery


right inf. parathyroid glands are identified
They are confirmed by frozen sect… left inf. parathyroid cannot be identified.despite
careful examination of the left lobe
Which of the following is the most likely location for this missing inferior
parathyroide:
a.Carotid sheath
b.Posterior to the inferior thyroid artery
c.intrathyroidal
d.Thymus
e.Retroesophagel

ANSWER: D or C,, mostly C because in the in Q mentioned during intra op

6. A 23 yo man presents to the ER with a stab wound to his ante 1 cm above the
cricoid cartilage with penetration to the platysma
His vitals and oxygenation normal physical findings. Which of the following is
considered a proper management?
a.Discharge the patient home
b.Observe for 6 hours, if patient drinks well. with no respiratory distress then
discharge him
c.Observe for 24 hours. If no further complaints discharge him
d.Perform a CT-angiogram. If normal discharge him
e.Formal neck exploration

ANSWER:d or e

7. An 18 yo patient presents with a 12-day history of abdominal pain and pyrxia and
the abdomen is soft with no rebound. A mass in the right iliac fossa is palpable
the diagnosis of an appendicular mass with an associated abscess. The patient
admitted for observation and conservative management. After 2 days the mass is
same. Temp. remains raised. The next stage in management is
a.Continue antibiotics for further 14 days
b.Proceed to appendicectomy
c.Percutaneous drainage
d.Right hemicolectomy
e. Laparoscopic drainage

ANSWER:c

8.which form of skin cancer is ass with the highest risk of simultaneous internal
malignancy:
a.Basal Cel Carcinoma
Dr. wessam alzaidat - General surgery
b.Squamous Cell Carcinoma
c.Melanoma
d.Extra-mammary Paget's disease
e.Bowen's disease

ANSWER:C

9. Which of the following regarding intra-ductal papillary mucinous neoplasms true:


a.The tubular variety of carcinoma arising in an IPMN indicates a better prognosis
b. Main-duct IPMN have greater malignant potential than do the branch duct type
c. IPMN is twice more common in men than in women.
d.Cystic lesions larger than 3 cm should undergo annual follow up.
e.Branch-duct IPMN should undergo resection regardless of size.

ANSWE:B

Dr.WBZ

10. regarding preoperative medications management are true except:


a.warfarin should be stopped 3 to 5 days before surgery and replaced with heparin.
b.pt should take B blocker on day of surgery as usual.
c.pt should take thyroxine on day of surgery as usual.
d.pt should take angiotensin converting enzyme inhibitors on day of surgery as
usual.
e.no need to stop aspirin if the surgery is not associated with high risk of blood
loss.

ANSWER:E

11. woman undergoes laparoscopic sleeve gastrectomy for morbid obesity. She
return for operative evaluation and is tolerating liquids and has lost 10 lbs.
in 14 days. Review of final report greater curvature with 9 mitoses/hpf. What is the
next best step in management:
a.Imatinib Therapy
b.Advance diet, follow up in 2 months
c.CT chest/abdomen/pelvis with IV and PO contrast
d.PET/CT
e.FOLFOX chemotherapy

ANSWER:A
Dr. wessam alzaidat - General surgery
12. Multiple options for tx of pressure sores exist. Which of the following options is
not suitatable:
a.Primary closure
b.Local wound care
c.skin grafts
d.Musculocutaneous reconstruction
e.Fasciocutaneous reconstruction

ANSWER:A

13.Which types of surgeries is less likely to be an indication for a day-case surgery?


a.Open cholecystectomy.
b.Adult inguinal hernia repair.
c.Hemithyroidectomy.
d.Sleeve gastrectomy for obesity.
e.Excisional biopsy of a breast mass

ANSWER:A

14. A 40-years old woman has a 2 cm area of pleomorphic calcification in the upper
outer quadrant of the left breast bx demonstrates intermediate
to high grade DCIS. She is interested in breast conservation. What is the width to
reduce her risk of local recurrence?
a.No ink on tumor.
b.1 mm
c.2 mm
d.5 mm
e.10 mm

ANSWER:C

15.Which of the following molecularly targeted therapies is directed against the


HER2 gene?
a.Cetuximab
b.suntinib
c.trastuzumab
d.Temsitolimus
e.Tamoxifen

ANSWER:C

Dr. wessam alzaidat - General surgery


16.Which of the following is true regarding the development of cancer in Crohn's
disease?
a.There is a higher rate of a Crohn's associated malignancy in a fistula tract
compared to a stricture
b.The risk of colon cancer in Crohn's disease is no greater than the general
population
c.Surveillance colonoscopy is not indicated in a patient with an ileosigmoid fistula.
d.Colon cancer in the setting of Crohn's disease should be managed similar to the
general population
e.Subtotal colectomy with ileorectal anastomosis has better outcomes in Crohn's
disease than general population

ANSWER>D

17. 34-year-old woman presents with recurrent pancreatitis. She has no history of
gallstones or alcohol use.
Cholangiopancreatography shows separate dorsal and ventral pancreatic ducts.
What is the next best step
a.Sphincterotomy and stenting of minor papilla
b.Sphincterotomy and stenting of major papilla
c.Frey Procedure
d.Puestow Procedure
e.Supportive Care

ANSWER:A

18. Following a hypotensive shock which lasted for hours, which of the following
tissues can withstand ischemia least
a.Skeletal muscle
b.Small Intestinal Epithelium
c.Retina
d.Myocardium
e.Hippocampus

ANSWER:C

19. The clinical picture of gallstone ileus includes all the following except?
a.Air in the biliary tree
b. Small bowel obstruction
c.stone at the site of obstruction
d.Pale stools
e.ass bouts of cholangitis
Dr. wessam alzaidat - General surgery
ANSWER:E

20. All of the following statements regarding blood pressure control in a Pt with
pheochromocytoma ……. adrenalectomy are true except:
a.Alpha blocker should be started preoperatively to control blood pressure
b.Beta blocker should be started before alpha blocker preoperatively
c.Pt should show signs of orthostatic hypotension preoperatively to make sure that
that bp controlled
d.After adrenalectomy the patient might need vasopressors to support blood
pressure.
e.After adrenalectomy blood sugar should be checked regularly

ANSWER:B

21. The most important treatment in anaphylaxis is


a.Iv fluids
b.Steroids
c.Antihistamines
d.Epinephrine
e.B2 agonist

ANSWER:A

22. After a modited radical mastectomy, a 45-year-old woman reports new-onset


weakness in the ipsilateral arm on a cord to adjust the curtains in her home. On
examination, she has difficulty when attempting t her ann. What is the best
explanation for her deficits?
a.intentional transection of the intercostobrachial nerve
b.Application of surgical clips across the long thoracic nerve
c.Inadvertent transection of the thoracodorsal nerve
d.Complete cautery injury to the supraclavicular nerve
e.Retractor injury to the medial pectoral nerve

ANSWER:C

23.Regarding penetrating abdominal wound, which of the following is correct?


a.Injuries to the bowel may be primarily repaired if less than 75% of the bowel
circumferential
b.The most common organ injured is the spleen
Dr. wessam alzaidat - General surgery
c.negative FAST examination usually rule out injury
d.Hemodynamically stable patients with no evidence of peritonitis on exam may
undergo evaluation ct scan of the abdomen
e.For unstable Pt with penetrating abdominal injuries diagnostic laparoscopy should
be injuries can be identified quickly

ANSWER:d

24. Which of the following is true about Bochdalek diaphragmatic hernia?


a.The defect in the diaphragm located anterolateral.
b.The anomaly is located on the right side in most cases
c.The abdominal cavity is small in these patients
d.Associated anomalies are rare.
e.Because the lung is deflated in utero, prenatal ultrasound is insensitive for
diagnosis

ANSWER:C

25.best describes the sequence of sensory recovery in a healing skin graft:


a.Temperature, pain, light touch
b.Pain, temperature, light touch
c.Pain, light touch, temperature.
d.light touch, pain, temperature.
e.Temperature, light touch, pain

ANSWER:C (PTT)

DR.WBZ…………..

26.regarding Pt with unresectable metastatic colon cancer and an asymptomatic or


minimally symptomatic g the following are true EXCEPT:
a.The principal treatment is chemotherapy
b.Resection of the primary colon cancer prior to the development of symptoms
could prevent morbity outcomes
c.Resection is not recommended for mild obstructing symptoms
d.Resection is not recommended for anemia
e.No Survival benefit with primary tumor resection in asymptomatic patients

ANSWER:C ??

27. A 22 yo female with history of a gun-shot wound to the abdomen requiring


multiple bowel resections has been in(TPN) for short gut syndrome. She presents for
Dr. wessam alzaidat - General surgery
a clinic follow-up stating that her hair fell out, and she has developed multiple
bruises over her arms and legs. In addition, she has a diffuse scaly rash Which of the
following nutrients or trace elements likely deficient in?
a.Copper
b.Vitamin C
c.Linoleic acid
d.Zinc
e.Selenium
ANSWER:C

28. 50-year-old male undergoes a resection of a large refropertoneal


leiomyosarcoma. There is an estimated blood .. cc. At the next day, the pt is found
to be anemic and is given 2 units of blood. Halfway through the first unit developed
chills and his temperature inc. from 37° to 39 C. Which of the following is true in
regard to this condition?
a.The transfusion does not need to be stopped
b.This occurs more commonly when given packed red blood cells vs platelets
c.Filtration is more effective than leukocyte washing in preventing this condition
d.Aspirin is more effective than acetaminophen in treating this condition
e.Pre-transfusion administration of acetaminophen and diphenhydramine is the
most effective to do

ANSWER:c
29.which of the following measures have been recommend for control of acid
secretion in patients with Zollinger-Ellison except??
a.Antrectomy
b.Highly selective vagotomy.
c.Total gastrectomy
d.Vagotomy and pyloroplasty.
e.Medical therapy with PPI .
ANSWER:A DR.WBZ
30.After significant blunt trauma, a patient arrives to the ER with GCS 4/15…
saturation is poor at 89% after applying a non-rebreather mask. His eyes are bruised
and blood come from his ears. The method of airway management that should be
avoided:
a.Oropharyngeal intubation
b.Laryngeal mask airway
c.Cricothyroidotomy is oropharyngeal is unsuccessful
d.Nasopharyngeal intubation
e.Oropharyngeal intubation with bronchoscopy guidance

ANSWER:D
Dr. wessam alzaidat - General surgery
31. Which of the following is a cyanotic congenital heart
a.Patent ductus arteriosus
b.Tetralogy of Fallot
c.Ventricular septal defect
d. Atrial septai detect
e.Coarctation of the aorta

ANWER:B

32. Regarding pleomorphic adenoma of parofid gland all the following are true
except
a.Parotid gland is the most commonly involved
b.Malignant transtormation does not occur
c.Also called mixed tumor
d.Slow growing & lobular
e .Superficial parotidectomy

ANSWER:B

33. Parietal cells of the stomach secrete one of the following:

b.Gastrin
c.Intrinsic factor
d.Secretin
e.Cholecystokinin

ANWER:C

34. 36-year-old male is brought to the ER due to severe retrosteral chest pain and
dysphagia, which started after multiple episodes of forceful vomiting after binge
drinking the night before He is found to have crepitus in his neck and hamman sign
physical exam. A CT scan shows diffuse mediastinal and cervical air What is the most
likely site of perforation?
a.Esophageal perforation at the cnicopharyngeus
b.Perforation of the left lateral lower esophagus, 5cm above the GE junction
c.Tracheal disruption at the level of the cricoid cartilage
d.Bronchial disruption at the takeoff of the right main stem bronchus
e.Perforation at the right lower thoracic esophagus just proximal to the LES

Answer:B

Dr. wessam alzaidat - General surgery


35. about SIADH all the following is true except?
a.it can occur after head inj
b. it can caused by NSAID
c.1st lin Tx infusion of normal saline.
d. furosemide can be used to increase free water loss
e. Pt is euovolmic and hyponatremic.

Answer: C

36.which hernia type vulnerable most to strangulate: or type of internal hernia can
strangulate?
..
..

37. patient with a large fungating mass in the antrum of the stomach, he undergoes
r decompression of the stomach and enteral feeding. Enteral feeds should include
this amino acid for enterocytes:

Glutamine

38. about cholangiocarcinoma except:


a.y adenocarcinomas
b. …..the most common type
c. ..e located intrahepatic
d. present with painless jaundice
e . tumor marker CA 19-9 can elevated but not specific

Answer:

39.prognostic factor for sarcoma without meatstases:

40. about the lower esophageal sphincter


a. anatomical landmarks
b…musculature is located at the lesser curvature of GE junction
c…Table to occur if the intra-abdominal part of esophagus is more
d….les contract (close) when the gastric fundus is distended with gas and liquid
e… meals increase the risk of GERD

ANSWER:

41.pt predisposed to hypotension hyperkalemia and renal failure the dx of this


condtion??
Dr. wessam alzaidat - General surgery
42.case about conservative breast cancer
a. immediate reconstruction is better
b.cancers are considered indications for immediate reconstruction.
c.delay in adjuvant therapy alter immediate reconstruction in the event of
postoperative
d.breast tolerates radiation therapy well.
e.In adanced diseases, reconstruction may be obligatory to cover the chest wall.
ANSWER:
DR.WBZ
43. a37 yo female with PR/ER and HER –ve…radiology report show a 1.9 cm invasive
ductal cancer with the closet margin measuring 1 mm. one of two stasis with no
extra capsular extension, what would you advice the patient regarding her need :
a.mastectomy is indicated.
b.lumpectomv cavity is necessary
c.underergo completion Axillary dissection.
d.lumpectomy cavity and completion axillary dissection
e.undergo modified radical mastectomy

Answer:

44. drugs are associated with cardiac toxicity?


Answer:doxorubicin

45. nerves are at risk of jury during submandibular staladenectomy EXCEPT


a.branch of facial nerve.
b.hypoglossal nerve.
c.laryngeal nerve.
d.mandibullar branch of facial nerve.
e.???

Answer:c

46. abdominal colectomy with ileanal pouch reconstruction for familial


adenomatous polyposis, a 24-y/o develope a firm palpable mass at his leostomy site
scar that is nontender. Ultrasound study reveals a densely lid 8-om mass within the
soft tissue of the abdominal wall. The most likely diagnosis is

Desmoid tumor

47. hemothorax, all of the following statements are true EXCEPT:


a.thoroctomy is indicated if bleeding is more than 1 liter in the first 4 hours
Dr. wessam alzaidat - General surgery
b.thoroctomy is indicated in clotted nemothorax.
c.bleeding is from the intercostal vessels in most of the cases.
d.Is Self-limiting in most of the cases
e.can occur without a rib fracture.

Answer:c

48. statements is true about invasive breast cancer?


a. non special type (NST) ductal carcinoma accounts for about half of cases.
b. carry better prognosis
c.is produces extensive fibrosis.
d. disease of the nipple usually is associated with invasive cancer.
e.carcinoma is a frequent phenotype of BRCA1 hereditary breast cancer.

Answer: A

Dr. wessam alzaidat - General surgery


Boards Part one
1.Which of the following does not pass through the inguinal canal?
Round ligament
Vas deferens
Genital branch of genitofemoral nerve
Broad ligament
Testicular artery

ANSWER:D

2.Four days after an Ivor Lewis esophagectomy, you start enteral feeding through the jejunostomy
feeding tube. The long chain fatty acids contained in the feeds:
Are not found in chylomicrons
Are only synthesized in the body
Enter the circulation via lymphatics
Enter the circulation via the portal system
Are not absorbed at all

ANSWER:C

3.A 65-year-old woman, presents with a tender mass below and medial to the pubic tubercle. One Of
the following is not a boundary to this mass:
Femoral artery
Femoral vein
Inguinal ligament
Pectineal ligament
Lacunar ligament

ANSWER:A

4.Following a fight, a 21-year-old male has an inability to dorsiflex his right foot (foot drop) while
losing sensation in the dorsum of his foot. The nerve affected is:
Medial plantar nerve
Lateral plantar nerve
Tibial nerve
Lateral cutaneous nerve of the calf
Common peroneal nerve

ANSWER:E

5.The organ most commonly found in a sliding hernia in women is:


Ovary and fallopian tube
Cecum
Small bowel
Spleen
Ureter

ANSWER:A

6.The vertebral level that corresponds to the nipple in men:


T1

Dr. wessam alzaidat - General surgery


T2
T3
T4
T5

ANSWER:D

7.The peritoneal space located to the left and posterior to Foramen of Winslow is:
Rutherford Morison’s pouch (hepatorenal pouch)
Right subphrenic space
Right paracolic gutter
Lesser sac
Left spleno-renal space

ANSWER:D

8. Within 24 hours of a surgical wound, the predominant cell type is:


Polymorphonuclear cells
Macrophages
Lymphocytes
Fibroblasts
The wound is acellular.

ANSWER:A

9.The most important cell to affect wound healing among the following is:
Polymorphonuclear cells
Macrophages
Lymphocytes
Platelets
Mast cells

ANSWER:B

10.The predominant cell in the proliferative phase of wound healing is:


Polymorphonuclear cells
This phase is acellular
Platelets
Lymphocytes
Fibroblasts

ANSWER:E

11.A patient who is chronic steroids, is about to undergo surgery. To ensure best possible wound
healing result, you:
Add high dose vitamin A before and after the operation
Add high dose vitamin D before and after the operation
Add high dose vitamin E before and after the operation
Add high dose vitamin K before and after the operation
Patient can be safely stopped at the time of surgery, regardless of indication

ANSWER:A

Dr. wessam alzaidat - General surgery


12.A patient who previously sustained a large open gluteal laceration, comes to your clinic with a
smaller wound to your surprise. This is explained by one of the following:
Lymphocytes
Macrophages
Fibroblasts
Myofibroblasts
PMNs

ANSWER:D

13.A patient presents with a keloid scar to your clinic. All describe her condition except:
It is usually confined to the original scar
Can be treated with steroids
Can be treated with silicone injections
Can be treated with radiation (XRT)
Tend to form in upper extremities and sternum more commonly

ANSWER:A

14.A 60-year-old man with unresectable lung cancer is undergoing radiation therapy (XRT). The main
target of XRT for malignancy treatment is:
DNA
RNA
Cell wall
Proteins
Cytoplasm

ANSWER:A

15.Positron Emission Tomography (PET) scan detects:


Glucose 6 phosphate molecules
Tumor specific proteins
Fluorodeoxyglucose molecules
Glycogen molecules
Necrotic tumor products

ANSWER:C

16.Performing a PET scan can be difficult in one of the following situations:


Hypothyroidism
Hypertension
Hb A1c < 6%
Hb A1c > 8%
Hyperthyroidism

ANSWER:D

17.The most important prognostic factor for sarcomas devoid of metastases is:
Node status
Size of the tumor
Tumor grade
Gender of the patient

Dr. wessam alzaidat - General surgery


Age of the patient

ANSWER:C

18.The most important prognostic factor for breast cancer devoid of metastases is:
Node status
Size of the tumor
Tumor grade
Gender of the patient
Age of the patient

ANSWER:A

19.Hyperacute rejection following organ transplantation is most often due to:


Rh incompatibility
Macrophages
Previous sensitized T cells
ABO incompatibility
Preformed antibodies

ANSWER:D

20.The most common malignancy following transplantation is:


Lung cancer
Prostate cancer
Breast cancer
Hepatocellular cancer
Skin cancer

ANSWER:E

21.A patient arrived to the emergency department after a head injury at work. After full primary
survey, he has no verbal response, he opens his eyes when you rub his sternum, moans
(incomprehensible sound) without any movement to his arms and legs. His Glasgow Coma Scale
(GCS) is:
2/15
3/15
5/15
7/15
12/15

22.47-year-old woman presents to the emergency room after a motor vehicle accident, mildly lethargic
and confused, with normal airway and vitals. She complains of a headache, and has a frontal scalp
hematoma. After stabilizing her cervical spine and doing primary survey, she becomes more lethargic,
moving only to pain. Which of the following is the most likely to be present?
Subdural hematoma
Extradural hematoma
Cerebral contusion
Intracerebral hematoma

Dr. wessam alzaidat - General surgery


Diffuse axonal injury

ANSWER:B

23.A 19-year-old girl sustained a posterior knee dislocation. Her distal pulses are absent. The most
appropriate next step is:
Start thrombolytic therapy immediately
Start Heparin immediately
Exploration of the vessels
Angiogram
Immediate reduction of the knee

ANSWER:E

24.Following a house fire, a 25-year-old male patient was brought to the emergency room with
extensive partial and full thickness burns. You note black carbon deposits around his nostrils and
oropharynx. Which of the following is an immediate priority?
Adequate analgesia
Aggressive fluid resuscitation
Immediate transfer to a burn unit
Secure the airway with higher possible need for intubation
Dressing changes to the burn wounds

ANSWER:D

25.A 50-year-old man sustains a flame burn involving the entire left upper extremity, entire anterior
and posterior trunk, genital area, and half of the left lower extremity. Please estimate the total
surface area that is burned:
64%
55%
43%
37%
28%
26.Parkland formula is used to calculate fluid requirements of burn patients. For a 70-kg adult with a 40%
second degree burn, the total fluid requirements in the first eight hours is:
2.8 liters
4.2 liters
6.4 liters
7.6 liters
12.8 liters

ANSWER:C

27.A 41-year-old large-surface-burn patient, is intubated in the ICU. After 10 hours of aggressive
hydration, escharotomy and fasciotomy, her urine output is becoming scanty. After placing a central
line, her CVP is 18. The most likely cause of her renal failure is:
Inadequate fluid resuscitation
Cardiogenic shock
Sepsis
Pulmonary embolus
Myoglobinuria

ANSWER:E
Dr. wessam alzaidat - General surgery
28.The best way to diagnose burn wound sepsis is:
Wound swab culture
Blood culture
Examination
Biopsy of the wound
Measuring C-Reactive Protein level

ANSWER:D
Burn=Biopsy B B not culture

29.In chemical burns, the most essential initial step in limiting the extent of injury is:
A)Thorough irrigation of the injured area with water or oil according to type of chemical causing
injury
B)Early debridement without irrigation
C)Applying acid if chemical causing burn was alkaline or vice versa
D)Immediate coverage of the injury site to decrease reaction of chemical with air oxygen
E)Applying a tourniquet to injury site to prevent spread of chemicals systemically

ANSWER:A

30.There were 192,000 new cases diagnosed with lung cancer in 2019. This means:
Incidence
Prevalence
Mean
Mode
Median

ANSWER:A

31.Specificity of a test reflects:


Ability to detect the disease
Rejecting the null hypothesis when it is true
Accepting the null hypothesis when it is false
Ability to say that no disease is present
Lower specificity means a more reliable test

ANSWER:D

32.Sensitivity of a test reflects:


Ability to detect the disease
Rejecting the null hypothesis when it is true
Accepting the null hypothesis when it is false
Ability to say that no disease is present
Higher sensitivity always means a more specific test

ANSWER:A

33.Type II error of a test reflects:


Ability to detect the disease
Rejecting the null hypothesis when it is true
Accepting the null hypothesis when it is false

Dr. wessam alzaidat - General surgery


Ability to say that no disease is present
Type II error is minor and does not affect research validity

ANSWER:C

34.One of the following statements regarding thyroglobulin is true:


It is part of normal thyroid function tests.
It is secreted only by cancerous cells.
Autoantibodies does not interfere with its levels.
It is a sensitive marker of recurrence of thyroid cancer.
Radioiodine treatment affects its concentration.

ANSWER:D

35.All of the following statements are true except:


Lymph node metastasis is commoner in follicular cancer than in papillary cancer.
Papillary cancer is associated with BRAF mutation.
Distant metastasis is commoner in the follicular variety.
Hurthle cell carcinoma is a type of follicular cancer.
Papillary carcinoma is usually nonencapsulated and slow-growing

ANSWER:A

36.One of the following injuries is associated with permanent damage to recurrent laryngeal nerve
during thyroid surgery:
Neuropraxia.
Axonotemesis.
Neurotemeisis.
Dendridotemisis.
Thermal injury.

ANSWER:C

37.The least indicated surgical technique among the following nowadays is:
Hemithyroidectomy.
Subtotal thyroidectomy.
Near-total thyroidectomy.
Total thyroidectomy.
Total thyroidectomy and central neck dissection.

ANSWER:B

38.For a patient with a thyroid nodule, the following ultrasonic feature has the lowest potential for
malignancy:
Hypo-echogenicity.
Microcalcifications.
Spongiform appearance.
40% of its size is a cyst.
2 cm in diameter.

ANSWER:C

39.One of the following structures is not an anatomical relation to the submandibular salivary gland:

Dr. wessam alzaidat - General surgery


The Mylohyoid muscle.
The facial artery.
The inferior alveolar nerve.
The lingual nerve.
The hypoglossal nerve.

ANSWER:C

40. The term plunging ranula refers to which clinical entity:


A malignant congenital salivary mass arising from the submandibular gland.
A benign salivary mass involving the parotid and submandibular glands.
A serous cyst originating from the sublingual glands that is potentially malignant.
A mucous retention cyst originating from the submandibular and sublingual glands that reaches the
neck.
A midline neck mass which moves on tongue protrusion.

ANSWER:D

41.All of the followings are true regarding neuroendocrine tumors of the pancreas except:
Insulinomas are the commonest form.
Diarrhoea is a common presentation in gastrinomas.
Peptic ulcers in gastrinomas are usually small and solitary.
VIPomas present with diarrhoea.
They occur in 40–60 per cent of MEN-1 patients.

ANSWER:C

42.With regard to thyroid anatomy, One following statements is incorrect:


The inferior thyroid artery arises directly from the external carotid artery.
The thyroid ima artery arises directly from the aorta in 1% to 4% of patients.
The ligament of Berry is located near the entry point of the RLN.
Venous drainage of the thyroid gland is via the superior, middle, and inferior branches.
The superior and middle thyroid veins drain into the jugular vein.

ANSWER:A
43. In breast carcinoma, one of the following statements is false:
Ductal carcinoma is the most common variant.
Lobular carcinoma occurs in 15 per cent.
There may be a combination of lobular and ductal features.
Colloid, medullary and tubular carcinomas carry a poor prognosis.
Paget’s disease is a superficial manifestation of an underlying breast carcinoma.

ANSWER:D

44.Which of the following is not true regarding magnetic resonance imaging (MRI) for evaluation of
breast abnormalities:
It is useful for finding the primary breast lesion in patients with positive axillary nodes and negative
breast examination and mammogram.
It is more accurate than mammography in diagnosing invasive lobular cancer.
It is more helpful than mammography in assessing tumor extent particularly in older women.
Its sensitivity in detecting invasive cancer is greater than 90%.
Its use as a screening tool is not settled.

Dr. wessam alzaidat - General surgery


ANSWER:C

45.Regarding primary lung tumor, all of the following statements are correct except:
Lifetime cigarette smoking is a major risk factor for bronchial carcinoma.
Small-cell lung cancer is less common, metastasises early and is less amenable to surgery compared
with non-small-cell cancer.
Finger clubbing and hypertrophic pulmonary osteoarthropathy are usually incidental findings and
not due to primary lung cancer.
The appropriate treatment strategy is dependent on tumour type, tumour stage, and the general
fitness and lung function of the patient.
Late survival has a direct relationship with the tumour stage at the time of treatment.

ANSWER:C

46.All the following are true about the liver except


It’s weight is about 2% of the body weight.
It is surrounded by a fibrous sheath called Glisson’s capsule.
The round ligament is the remnant of the obliterated umbilical artery.
The falciform ligament anchors the liver to the anterior abdominal wall.
The ligamentum venosum is the obliterated ductus venosus.

ANSWER:C

47.All the following are true about hepatic venous drainage except
The right hepatic vein drains liver segments V, VI, VII and VIII.
The middle hepatic vein drains liver segments IV, V and VIII.
The left hepatic vein drains liver segments I, II and III.
The caudate lobe drains directly into the IVC.
The left and middle hepatic veins usually form a common trunk before entering the IVC.

ANSWER:C

48.All the following are true about bile acids and bile salts except
Bile salts are responsible for the digestion and absorption of lipids.
Bile salts are sodium and potassium salts of bile acids conjugated with amino acids.
The bile acids are derivatives of cholesterol.
Chenodeoxycholic acid is a secondary bile acid.
90% to 95% of bile salts secreted into the intestine are reabsorbed at the terminal ilium.

ANSWER:D

49.Which of the following statements is true about the liver enzymes ALT and AST?
ALT is more specific for liver disease than AST.
Both participate in glycolysis.
They are usually severely elevated in cases of liver cirrhosis.
In alcoholic liver disease, an ALT:AST ratio more than 2 is common.
In fulminant hepatitis, ALT and AST levels are usually moderately elevated.

ANSWER:A
50.All the following are true about Gilbert’s syndrome except
It is characterized by decrease activity of glucuronyltransferase.
It causes increase in conjugated bilirubin level.

Dr. wessam alzaidat - General surgery


It is a benign condition.
Presents with jaundice during stress conditions or fasting.
No treatment is required for this syndrome.

ANSWER:B

51.Which of the following is a cause of presinusoidal portal hypertension?


Chronic viral hepatitis.
Alcohol abuse.
Autoimmune hepatitis.
Budd-Chiari syndrome
Schistosomiasis.

ANSWER:E

52.All the following are true about liver hemangiomas except


They are the most common solid liver lesions.
More common in women.
Most are discovered incidentally.
Spontaneous rupture is common.
Resection can be accomplished mostly by enucleation.

ANSWER:D

53.All the following are true about hepatic adenomas except


More common in females.
Associated with contraceptive pills.
Risk of rupture and bleeding is very low.
It carries a risk of malignant transformation.
Surgical resection is indicated.
ANSWER:C
54.The gallbladder wall contains all the following layers except
Mucosa.
Submucosa.
Muscularis.
Subserosa.
Serosa.

ANSWER:B

55.All the following about the vascular system of the gallbladder are true except
The cystic artery is usually a branch of the right hepatic artery.
When the cystic artery reaches the neck of the gallbladder it divides into anterior and posterior
branches.
Usually, a single cystic vein drains the gallbladder to the portal vein.
Frequently, a visible lymph node overlies the cystic artery near its end.
The nerves of the gallbladder come through the celiac plexus.

ANSWER:C

56.Which of the following is true regarding gallstones?


Most persons with gallstones have symptoms related to them.
Most gallstones are pure cholesterol stones.

Dr. wessam alzaidat - General surgery


Pure cholesterol stones are usually large and single.
Most gallbladder stones are radiopaque.
Black stones are usually formed secondary to bacterial infection caused by bile stasis.

ANSWER:C

57.The best treatment for acute calculous cholecystitis is


Early laparoscopic cholecystectomy within 3 days from the onset of symptoms.
Early open cholecystectomy within 3 days from the onset of symptoms.
Interval cholecystectomy 6 weeks after initial medical treatment.
Cholecystostomy.
Intravenous antibiotics.

ANSWER:A

58.All the following are true about gallbladder cancer except


It is more common in females.
It carries bad prognosis.
Most are associated with gallbladder stones.
Most are squamous cell carcinomas.
Porcelain gallbladder carries relatively high incidence of the disease.

ANSWER:D

59.Which of the following statements is true?


The inferior mesenteric vein often joins the splenic vein near its junction with the superior
mesenteric-portal vein.
The superior mesenteric artery runs to the right and parallel to the superior mesenteric vein.
The pancreas is wrapped by peritoneum.
All the arteries that supply the pancreas are branches of the celiac trunk.
The body of the pancreas constitutes about 65% of the size of the pancreas.

ANSWER:A

60.Which of the following is true about the pancreatic juice?


It is about 200 to 300cc per day in an average adult.
It is white.
It is alkaline.
It is hyperosmotic to the plasma.
It contains the active forms of proteases and lipases.

ANSWER:C

61.Which of the following is one of the Ranson’s prognostic signs of acute gallstone pancreatitis?
Age more than 70 years.
WBC more than 16000 per cubic mm.
Blood glucose more than 200 mg per dL.
Serum LDH more than 350 IU per L.
Blood urea nitrogen elevation more than 5 mg per dL within the initial 48 hours.

ANSWER:A

62.All the following are true about insulinomas except

Dr. wessam alzaidat - General surgery


It is the most common functional pancreatic neuroendocrine neoplasm.
The serum insulin level is elevated with low serum glucose.
Serum C-peptide level is elevated.
Insulinomas are much more common to be located in the body and tail of the pancreas in
comparison to the head of the organ.
Most are benign.

ANSWER:D

63.All the following are true about the spleen except


It is mesodermal in origin.
Most accessory spleens present in the hilum of the spleen.
The gastrosplenic ligament contains the short gastric vessels.
In most people the splenic artery divides far from the spleen and many long vessels enter the spleen
over three fourth of its medial surface.
The white pulp constitutes about 75% of the spleen.

ANSWER:E

64.All the following are indications of splenectomy except


Hereditary spherocytosis who require frequent transfusion.
Pyruvate kinase deficiency with sever splenomegaly.
Glucose-6-phosphate dehydrogenase deficiency.
Sickle cell anemia with recurrent sequestration crises.
Idiopathic thrombocytopenia purpura refractory to steroid therapy
ANSWER:C
65.Which of the following statements is true?
1 milliequivalent calcium ion equals 2 millimoles.
Normal serum osmolality is between 250 and 270 milliosmoles.
The intracellular fluid ionic concentration is more than the interstitial fluid.
Sodium is the main intracellular cation.
Proteins in the plasma is more than proteins intracellular.

ANSWER:C

66.Which of the following body fluids contains the highest concentration of bicarbonate?
Stomach.
Small intestine.
Colon.
Pancreas.
Bile.

ANSWER:D

67.All the following are manifestations of hypercalcemia except


Abdominal pain.
Hypertension.
Polyurea.
Hyperreflexia.
Arrhythmia.

ANSWER:D

Dr. wessam alzaidat - General surgery


68.Which of the following has the highest effect in increasing nitrogen wasting?
Major burn.
Skeletal trauma.
Severe sepsis.
Elective surgery.
Starvation.

ANSWER:A

69.All the following are true about platelets except


They don’t have nucleus.
Up to 30% may be sequestered in the spleen.
They have an average life span of 3 weeks.
They play an integral role in the formation of the hemostatic plug.
Heparin does not interfere with platelets aggregation.

ANSWER:C
70.All the following are true about von Willebrand’s disease except
It is the most common congenital bleeding disorder.
Patients presented with easy bruising and mucosal bleeding.
It affects platelets’ function.
All patients respond to desmopressin.
Patients may require von Willebrand’s factor concentrates.
ANSWER:D
71.Febrile nonhemolytic transfusion reaction is caused by
Preformed cytokines or host antibodies to donor lymphocytes.
Infusion of contaminated blood.
Transfusion of large volume of blood.
Anti-HLA antibodies in the transfused blood.
Transfusion of ABO incompatible blood.

ANSWER:A

72.The most common form of testicular tumour in infants and children is:
Yolk sac tumour
Choriocarcinoma
Teratocarcinoma
Embryonal cell carcinoma
Seminoma

ANSWER: A

73.Which of the following techniques is NOT used for removing the Foley catheter?
Overfilling of the balloon.
Cross-section οf the balloon filling lumen.
Balloon perforation through the filling lumen.
Transrectal ultrasound-guided balloon perforation
Mild continuous traction.

ANSWER:E

74.The most important factor in the formation of uric acid calculi is:
Concentration of uric acid in the urine

Dr. wessam alzaidat - General surgery


Volume of urine
Limited solubility of uric acid in acidic solutions
Excessive dietary intake of purines
Presence of symptomatic gout

Answer: C

75.The actions of parathyroid hormone (PTH) on the kidney are to:


Increase calcium and phosphorus absorption
Decrease calcium and phosphorus absorption
Increase calcium and decrease phosphorus absorption
Increase phosphorus and decrease calcium absorption
Promote production of active Vitamin D

Answer: E

76.The intracranial tumor most likely to be encountered in a middle-aged man with the acquired
immunodeficiency syndrome (AIDS) is:
Glioblastoma multiforme.
Ependymoma.
Meningioma.
Oligodendroglioma.
Lymphoma.

Answer: E

77.The evaluation of a comatose patient with a head injury begins with:


The cardiovascular system.
Pupillary reflexes.
Establishment of an airway.
Computed tomography (CT) of the brain.
History.

Answer: C

78.An epidural hematoma:


Is usually arterial in origin.
Is 5% accompanied by a skull fracture.
Should be suspected only in comatose patients.
Can be diagnosed from a brain CT scan.
Is usually venous in origin’

Answer: A

79.Which of the following signs does Horner's syndrome include?

Ptosis.
Facial hyperhidrosis.
diplopia
Exophthalmos.
Mydriasis.

Answer: A

Dr. wessam alzaidat - General surgery


80.Laparoscopic surgery may induce the following haemodynamic changes except:

Increased preload.
Increased cardiac output.
Increased pulmonary vascular resistance.
Increased systemic vascular resistance.
Decreased renal perfusion pressure.

Answer: B

81.Which of the following is an absolute contraindication to performing laparoscopic surgery?


Pregnancy
BMI > 40
previous abdominal surgery
having symptomatic chronic obstructed pulmonary disease (COPD)
presence of an uncorrected coagulopathy
Answer: E

82.Which of the following not limitations of minimal access surgery?


Technically more demanding
Loss of tactile feedback
Extraction of large specimens
Poor vision
Difficulty with haemostasis.

Answer: D

83.Which of the following complications not associated with pneumoperitoneum?

Hyperthermia
Acidosis
Cardiac arrhythmias
Gas embolism
Reduced cardiac return

Answer: A

84.During laparoscopic surgery, pneumoperitoneum usually results in a fall in cardiac output when
intra-abdominal pressure exceeds
10 mmHg
20 mmHg
30 mmHg
40 mmHg
50 mmHg

Answer: B

85.Which of the following cells produces HCL?


Beta cells
Oxyntic cells
Chief cells
Alpha cells
Surface mucus cells
Dr. wessam alzaidat - General surgery
Answer: B

86.The gastric juice contains


Trypsin, pepsin, lipase
Pepsin, lipase, rennin
Pepsin, amylase, trypsin
Trypsin, pepsin, rennin
Amylase, lipase, pepsin

Answer: B

87.Secretion of Gastric juice is stimulated by


Gastrin
Cholecystokinin
Enterogastrin
Pepsin
lipase

Answer: A

88.posterior superior iliac spine lies at:

L5 level
S1 level
S2 level
S3 level
L4 level

Answer: c

89.Dermatome of xiphoidal region is:

T5
T6
T7
T8
T9

Answer: B

90.The role of the parietal cells is to:

Secretes hydrogen ions and chloride ions into lumen and are responsible for maintaining the
acidic pH.
Alkaline mucous for shielding the epithelium from hydrochloric acid.
Secretes histamine which stimulates gastric secretion
Secretes gastric lipase.
Secretes amylase

Answer: A

91.Which tissue layer provides for primary digestive motility?


Dr. wessam alzaidat - General surgery
Mucosa
Submucosa
Mesentery (a fold of tissue that a aches organs to the body wall)
Muscularis Mucosa
serosa

Answer: D

92.In which region of the GI tract is the longitudinal muscle of the muscularis arranged into distinct
longitudinal bundles?

Duodenum
Jejunum
Ileum
Colon
stomach

Answer: D

93.Which of the following therapies is proven to reduce mortality and morbidity in bleeding peptic
ulcers?
Endoscopic procedures
H2 antagonists
Proton pump inhibitors
Octreotide
Antacids

Answer: A

94.Which is the most common cause of oesophageal perforation?


Iatrogenic causes
Trauma
Boerhaave’s syndrome
Foreign body
Tumour

Answer: A

95.Which of the following Foreign Bodies needs to be removed as an emergency?

50 cent coin in the stomach


50 cent coin in the oesophagus
lithium button battery in stomach
lithium button battery in oesophagus
5 grams of cocaine in condom sitting in small intestine

Answer: D

96.Which is the most common cause of peptic ulcer disease of the following?

Smoking
NSAID’s
Zollinger-Ellison syndrome
Dr. wessam alzaidat - General surgery
Ethanol excess
Family hx of PUD

Answer: B

97.Which is the most common complication of Peptic ulcer disease?

Perforation
Gastric outlet obstruction
Penetration
Haemorrhage
All are uncommon occurring in less than 5% of patients.

Answer: D

98.All of the following measures have been recommended for control of acid secretion in patients
with Zollinger-Ellison syndrome except:

Antrectomy.
Highly selective vagotomy.
Total gastrectomy.
Vagotomy and pyloroplasty.
Medical therapy with Prilosec (omeprazole).

Answer: A

99.Which of the following statements about gastric polyps is true?

Like their colonic counterparts, gastric epithelial polyps are common tumors.
They are analogous to colorectal polyps in natural history.
Endoscopy can uniformly predict the histology of a polyp based on location and
appearance.
In a given patient, multiple polyps are generally of a multiple histologic type.
Gastric adenomatous polyps greater than 2 cm. in diameter should be excised because of the risk of
malignant transformation.

Answer: E

100.Common sites of GIST in descending order as per incidence are


Ileum–duodenum–rectum–stomach
Stomach–ileum–duodenum–rectum
Rectum–ileum–duodenum–stomach
Duodenum– stomach–ileum–rectum
Ileum-rectum-stomach-duodenum

Answer: B

DR.WBZ

Dr. wessam alzaidat - General surgery


BEST WISHES

Dr. wessam alzaidat - General surgery


N0 PAIN NO GAIN
1.What is the definition of overweight?

BMI > 25 kg/m2


BMI = 25 kg/m2
BMI 25-29.9 kg/m2
BMI 25-30 kg/m2
BMI > 30 kg/m2

Answer: C

2.Hormones or peptides involved in satiety include:

Gastrin
Somatostatin
Glucagon
Ghrelin
Estrogen

Answer: D

3.One specific problem that may arise with persistent vomiting after any of the bariatric operations is
Wernicke’s encephalopathy, which can be treated with parenteral:

Vitamin B12
Omeprazole
Thiamine (vitamin B1)
Ascorbic acid (vitamin C)
Scopolamine
Answer: C

4. A full-term baby is born with drooling, coughing, and cyanosis after the first feeding, but
these resolve quickly and spontaneously. The next step in management should be:

Immediate intubation

Placement of orogastric tube

Two-view abdominal x-ray

Two-view chest x-ray

Upper gastrointestinal (UGI) contrast series

ANSWER:B
5. A patient is diagnosed with pyloric stenosis after 3 days of nonbilious emesis. This patient’s
electrolyte and acid/base balance will result in:

Respiratory alkalosis

Dr. wessam alzaidat - General surgery


Hyperkalemia

Aciduria

Hyperchloremia

Hyponatremia

ANSWER:C
6. A 30-kg child has an estimated hourly fluid requirement of:

40 mL/hr

50 mL/hr

60 mL/hr

70 mL/hr

80 mL/hr

ANSWER:D
7. An 8-year-old boy is involved in a bike crash. He is hemodynamically stable. On examination, he
has ecchymosis on the left upper quadrant. A focused assessment with sonography for trauma (FAST)
scan demonstrates free fluid in the pelvis. The next most appropriate step in management is:

CT scan of abdomen and pelvis

Admission to the intensive care unit (ICU) with serial CBC and exams

Two-view abdominal x-rays

Diagnostic laparoscopy

Transfusion of 10 mL/kg of packed RBCs

ANSWER:A
8. A 3-year-old girl presents to the ED with crampy abdominal pain for 24 hours. The pain is
increasing in frequency and duration. Ultrasound demonstrates a target sign. The most common
pathologic lead point is:

Appendix

Small bowel polyp

Ectopic pancreatic tissue

Meckel diverticulum

Intraluminal hematoma

ANSWER:D

9. A 2-year-old child presents with an abdominal mass, “raccoon eyes,” and “blueberry muffin” skin
lesions. This most likely represents:

Dr. wessam alzaidat - General surgery


Rhabdomyosarcoma

Neuroblastoma

Wilms tumor

Hepatoblastoma

Teratoma

ANSWER:B
10. Which of the following postoperative complications has not been shown to correlate with The
American Society of Anesthesiologists’ (ASA) classification of physical status?

Mortality

Operative duration

Intraoperative blood loss

Wound infection

Anastomotic leak

ANSWER:E
11. After completion of surgery, an anticholinesterase is administered to reverse the neuromuscular
blockade. Which of the following muscles would be expected to recover first?

Diaphragm

Adductor pollicis

Ocular muscles

Pharyngeal

Quadriceps femoris.

ANSWER:A
12. With regard to bipolar and monopolar devices:

Both use direct current (DC).

Both use alternating current (AC).

Bipolar uses AC, while monopolar uses DC.

Bipolar uses DC, while monopolar uses AC.

Neither requires an electrosurgical unit (ESU). Coagulation

ANSWER:B
13. What is the first step to take if there is an intraoperative fire?

Remove all burning drapes from the patient

Dr. wessam alzaidat - General surgery


Turn off insufflation gas

Turn off all anesthetic gases

Extinguish the fire

Initiate Code Red Alert per hospital protocol

ANSWER:C
14. Electromagnetic interference is a concern for patients with pacemakers when using energy
devices. Of the following, which device would not generate any electromagnetic interference?

Monopolar electrosurgical device

Monopolar endoscopic device

Radiofrequency ablation (RFA)

Ultrasonic device

Bipolar electrosurgical device

ANSWER:D

15. Following insertion of a Veress needle, what is the initial maneuver to confirm intraperitoneal
placement?

Saline drop test

Aspiration of the needle

Flushing the needle

Measuring insufflation pressure

Starting high-flow insufflation

ANSWER:B
16. A 32 year old female patient is about to undergo an elective repair of a paraumbilical hernia.
Which of the following skin preparation agents would be most appropriate to use?

2% aqueous chlorhexidine

2% alcoholic chlorhexidine

0.5% alcoholic chlorhexidine

Aqueous povidone-iodine solution

Isopropyl alcohol

ANSWER:B
17. Which of the following has been shown to reduce the incidence of acquired central venous line
sepsis?

Administration of intravenous (IV) vancomycin prophylaxis at the time of line


insertion

Dr. wessam alzaidat - General surgery


Utilizing the femoral vein site of insertion

Using a multiple lumen catheter

Daily review and removal of the catheter at the earliest opportunity

Changing the lines every seven days

ANSWER:D

18. Which of the following statements regarding focused abdominal sonography for trauma (FAST) is
true?

It can reliably evaluate the retroperitoneum.

It can quickly detect the presence of pericardial fluid or a pleural effusion.

It is useful in detecting a cardiac contusion.

It is considered a replacement for computed tomography (CT).

It can reliably detect diaphragmatic injuries.

ANSWER:B

19. Which of the following is not a sonographic characteristic of an inflamed gallbladder?

Gallbladder distention

Pericholecystic fluid

Wall thickness of 1 mm

Sonographic Murphy’s sign

Gallstones

ANSWER:C
20. Which of the following findings is consistent with perforated acute appendicitis?

External diameter > 4 cm

Compressible cecum

Asymmetric appendiceal thickening

Fluid within the lumen of the appendix

Appendicolith

ANSWER:C

Dr. wessam alzaidat - General surgery


21. During laparoscopic cholecystectomy, the so-called “Mickey Mouse” view of the hepatoduodenal
ligament can be obtained with laparoscopic ultrasound to help delineate important anatomy. Which
of the following lists all of the structures seen when the proper view is obtained?

Portal vein, common hepatic duct, and inferior vena cava (IVC)

Portal vein, common hepatic duct, and hepatic artery

Portal vein, common hepatic duct, and cystic artery

Portal vein, common bile duct, and hepatic artery

Portal vein, common bile duct, and cystic artery

ANSWER:C
22. Residents are expected to develop certain skills and habits so that they can meet all of the
following goals to satisfy the criteria of practice-based learning and improvement of core
competency except:

Use an evidence-based approach to patient care

Participate in mortality and morbidity conferences that evaluate and analyze


patient care outcomes

Use information technology to optimize learning

Locate, appraise, and assimilate evidence from scientific studies related to


their patients’ health problems

Promote the participation of their patients in a clinical trial

ANSWER:E
23. Residents are expected to accomplish which of the following to meet the systems-based practice
core competency?

Understand the nuances of the health care delivery systems

Coordinate their patient’s care even outside their clinical specialty

Ensure good patient outcomes regardless of cost awareness

Participate in identifying system errors

Understand the roles of other specialists only for the purposes of patient care

ANSWER:D
24. According to the Joint Commission, what is the leading cause of sentinel events (medical errors)?

Training errors

Systems errors

Communication errors

Cognitive errors

Medical administration errors

Dr. wessam alzaidat - General surgery


ANSWER:D

25. Which statement is FALSE regarding sigmoid volvulus?


Sigmoid volvulus is commonly seen in patients who are elderly, debilitated,
chronically constipated, or on psychotropic medications.

Mortality rates are lower when emergent decompression is performed before


sigmoid resection.

Nonoperative reduction of sigmoid volvulus is successful in 70%–80% of cases.

If the gangrenous bowel is seen during decompression, a rectal tube should be


placed to prevent recurrence.

Sigmoid resection within 2 to 3 days of decompression is the preferred


definitive treatment.

ANSWER: D

26. Which of the following is the least appropriate operative strategy for a patient presenting
with sigmoid diverticulitis and localized peritonitis?
Sigmoid colectomy with primary anastomosis

Sigmoid colectomy with primary anastomosis and diverting ileostomy

Sigmoid colectomy with end colostomy and rectal stump

Drainage and ileostomy without resection

Total colectomy

ANSWER: E

27. Common causes of colorectal anastomotic breakdown include all of the


following except:
Poor blood supply to the bowel edges

Short rectal stump

Emergency operation

Hand-sewn anastomosis

Poor technique

ANSWER D

Dr. wessam alzaidat - General surgery


28. Which of the following is false regarding colonic polyps?
Tubulovillous adenomas have a lower malignancy risk than villous
adenomas.
Some hyperplastic polyps are premalignant.
The polyps in Peutz-Jeghers syndrome are hyperplastic.
hamartomatous polyps are not considered premalignant
In an adenomatous polyp, the risk of malignancy is related to its location
in the GI tract.
ANSWER E

29. Which of the following is true regarding diverticular diseases of the lower GI
tract?
They occur most commonly in the ascending colon.
The rectum can be affected.
Asymptomatic sigmoid or cecal diverticuli require surgical management
95% of complications from diverticula occur at the sigmoid colon.
They are associated with a long, redundant colon.
ANSWER D

30. An 85-year-old male patient with a history of chronic constipation presents with
acute severe colicky abdominal pain and absolute constipation. Plain abdominal
film shows a grossly dilated oval of large bowel arising from the left lower quadrant. A
diagnosis of sigmoid volvulus is made. The next step in management is:
Laparotomy
Sigmoidoscopy with flatus tube insertion
Sigmoid colectomy with colostomy
Barium swallow
Computed tomography
ANSWER B

31. The mucous membrane lining the upper half of the anal canal is ;

Dr. wessam alzaidat - General surgery


lined with stratfied squamous epithelium.
Drained by the inferior rectal vein
Drained into the superfacial inguinal lymph nodes
Sensitive to touch and to pain
Sensitive to stretch
ANSWER E

32. Regarding successful day case surgery, one of the following is not true;

Associated with excessive blood loss or fluid shifts

The patient should be reasonably ambulant afterwards

Avoidance of unnecessary tissue handling or tension.

Very low risk of serious postoperative bleeding

Patients should not travel home by public transport.

ANSWER A

Dr. wessam alzaidat - General surgery


Boards part one final

1. In the first 48 hours after surgical trauma, the serum levels of all the following increase
except
A. Growth hormone.
B. Adrenaline.
C. Insulin.
D. Interleukin 1.
E. Interleukin 6.
Answer: C

2. A 35 years old male patient developed large amount of fresh bleeding per rectum 10 days
post hemorrhoidectomy. The most probable cause for his bleeding is
A. Sloughing of a vessel wall.
B. Uncontrolled bleeding missed at the time of surgery.
C. Clot dislodgment.
D. Injury by defecation.
E. Vasodilation.

Answer: A

3. All the following are true about cryoprecipitate except


A. It is a supernatant precipitate of fresh frozen plasma.
B. It is rich in coagulation factor VII.
C. It is rich in fibrinogen.
D. Stored at -30 degrees centigrade.
E. Shelf life is 2 years.

Answer: B

4. All the following are true regarding platelets concentrate except


A- A unit is pooled usually from more than one donor.
B- It is kept on a special agitator.
C- It is stored at 4 degrees centigrade.
D- May transfer bacterial infections.
E- Shelf life is 5 days.

Answer: C

5. In response to reversible hypovolemic shock, all of the following indices decrease except
A- Cardiac output.
B- Systemic vascular resistance.
C- Blood volume in venous capacitance.
D- Central venous pressure.
E- Mixed venous oxygen saturation.
Dr. wessam alzaidat - General surgery
Answer: B

6. All of the following are recognized consequences of inadequate pain relief after upper
abdominal surgery EXCEPT:
A. Decreased functional residual capacity
B. Ileus
C. Deep vein thrombosis
D. Sodium and water retention
E. Reduced systemic vascular resistance

ANSWER: E

7. A 22 year old man in undergoing a rapid sequence induction prior to an appendectomy.


The anesthetist becomes concerned because the patient becomes tachycardia and develops
atrial fibrillation. His temperature is 39.5 oC, blood pressure is slightly elevated. What is
the most appropriate course of action?

A. Proceed to immediate laparotomy


B. Administration of broad spectrum intravenous antibiotics
C. Start infusion of inotropes
D. Administration of intravenous Dantrolene
E. Administration of Neostigmine

ANSWER: D

8. bariatric surgery is used for obesity treatment but how much is the incidence of type 2
diabetes reduced in obese individuals after surgery?
a. 80-90%
b. No reduction in the incidence
c. Approximately 50 %
d. Approximately 40 %
e. Approximately 30 %

Answer: A

9. Hormones or peptides involved in satiety include:

A. Gastrin
B. Somatostatin
C. Glucagon
D. Ghrelin
E. Estrogen

Answer: D

10. Currently accepted guidelines from the National Institutes of Health for preoperative
selection of patients for weight reduction surgery include all the following except:

A. Patients with Prader-Willi syndrome


Dr. wessam alzaidat - General surgery
B. BMI > 35 kg/ m2 with associated medical comorbidity worsened by obesity
C. Failed Medical therapy
D. Psychiatrically stable
E. Motivated patient

Answer: A

11. Laparoscopic sleeve gastrectomy ,which one is not true ?

A. A higher leak rate than laparoscopic RYGB


B. A lower mortality rate for super obese BMI > 60 kg/m2 undergoing duodenal
switch
C. A theoretical advantage over RYGB for iron absorption
D. Desire to eat increases
E. Reduction in hunger sensation

Answer: D

12. One specific problem that may arise with persistent vomiting after any of the bariatric
operations is Wernicke’s encephalopathy, which can be treated with parenteral:

A. Vitamin B12
B. Omeprazole
C. Thiamine (vitamin B1)
D. Ascorbic acid (vitamin C)
E. Scopolamine

ANSWER: C

13. Unilateral bloody nipple discharge from one duct orifice is most commonly caused by:

A. Paget's disease of the nipple.


B. Carcinoma in situ.
C. Inflammatory carcinoma.
D. Subareolar mastitis.
E. Intraductal papilloma.

Answer: E

14. One of the following statements is true regarding gynecomastia:

A. If the disease is unilateral, it is unlikely drug-related.


B. The standard surgical treatment is subcutaneous mastectomy.

Dr. wessam alzaidat - General surgery


C. The presence of gynecomastia is often associated with the subsequent development of breast
cancer.
D. A formal endocrine evaluation is indicated in most juvenile patients with gynecomastia.
E. Pathologic causes of estrogen deficiency or testosterone excess are associated with
gynecomastia.

Answer: B

15. . Most common site of breast metastasis is:

A. Lung.
B. Brain.
C. Bone.
D. Adrenal.
E. Liver.

Answer: C

16. A 49 year old lady with breast cancer


is identified as having a 1mm foci of malignant cells in a sentinel node. What is the term
best used to describe this finding?

A. Isolated tumor cells


B. N1 disease
C. micrometastasis
D. N2 disease
E. n3 disease
ANSWER: C
17. Compared with sporadic breast cancers,
which of the features listed below is least commonly associated with the breast cancers
found in women with a BRCA 1 mutation?

A. High mitotic activity


B. Medullary type
C. Histologically grade 3
D. Predominant lymphocytic infiltrate
E. Strong estrogen receptor positivity
ANSWER: E

18. One of the following statements about cardiac myxoma is true:

A. This lesion is the most common primary cardiac tumor.


B. It is best diagnosed by cardiac catheterization and angiography.
C. The left ventricle is the most common site.
Dr. wessam alzaidat - General surgery
D. It has usually an extra cardiac growth pattern.
E. It has a multicentric origin in the chamber wall.

Answer: A

19. When is it mandatory to involve an independent mental capacity advocate?


A. When a patient requires a significant intervention, time permits and the patient lacks
capacity to decide for themselves
B. When a patient cannot decide on the best treatment
C. When parents disagree about treatment of a child
D. When two doctors disagree about the best treatment option
E. When a patient has lost trust in their clinician
ANSWER: A
20. A 40-year-old man who weighs 65 kg is being observed in the ICU. Twenty-four hours
postoperatively, he develops convulsions. His serum sodium is 118 mEq/L. Appropriate
management includes which of the following?
A. Administration of normal saline (0.9%)
B. Administration of hypertonic saline (3%)
C. Emergency hemodialysis
D. Administration of vasopressin
E. Administration of Lasix, 40 mg intravenously (IV)
ANSWER: B

21. A 42-year-old man with small-bowel fistula has been receiving TPN with standard
hypertonic glucose-amino acid solution for 3 weeks. The patient is noticed to have scaly,
hyperpigmented lesions over the acral surfaces of elbows and knees, similar to
enterohepatic acrodermatitis. What is the most likely cause of the condition?

A. Copper deficiency
B. Essential fatty acid deficiency
C. Excess glucose calories
D. Hypomagnesemia
E. Zinc deficiency

ANSWER: E

22. After undergoing subtotal gastrectomy for carcinoma of the stomach, a 64-year-old
woman is receiving peripheral parenteral nutrition. To increase calories by the peripheral
route, what should be prescribed?

A. D5W in normal saline


B. Multivitamin infusion
Dr. wessam alzaidat - General surgery
C. D25W (25% dextrose in water)
D. Soybean oil ( intra lipid)
E. Lactulose

ANSWER: D

23. In septic shock, which of the following is true?


A. The mortality rate is between 10% and 20%.
B. Gram-negative organisms are involved exclusively.
C. The majority of patients are elderly.
D. The most common source of infection is the alimentary tract.
E. Two or more organisms are responsible in most cases.

ANSWER: C
24. Regarding successful day case surgery, one of the following is not true;
A. minimal access techniques.
B. Good hemostasis.
C. Avoidance of unnecessary tissue handling or tension.
D. Provided with adequate analgesia.
E. Major procedures should be scheduled late on list.
ANSWER E
25. Postoperative suppressive treatment with thyroxin has a role in the management of:
A. Follicular adenoma.
B. Papillary carcinoma.
C. Anaplastic carcinoma.
D. Medullary carcinoma.
E. Lymphoma.

Answer: B

26. Bone hunger syndrome can occur after total thyroidectomy for:
A. Papillary thyroid cancer.
B. Multinodular goiter.
C. Medullary thyroid cancer.
D. Graves` disease.
E. Follicular thyroid cancer.

Answer: D

27. Adrenocortical carcinoma:


A. Is suspected in asymptomatic patients with adrenal tumors larger than 3 cm on CT.
B. Most often is diagnosed early in its course when disease is confined to the adrenal gland.
C. Is differentiated pathologically from benign adrenocortical adenoma by tumor necrosis,
hemorrhage, and cellular features of large hyperchromatic nuclei and more than 20 mitoses
Dr. wessam alzaidat - General surgery
per high-power field.
D. Should be resected only if disease is localized to the adrenal gland; otherwise treatment with
mitotane is indicated.
E. Carries a very poor prognosis.

Answer: E

28. Multiple Endocrine Neoplasia type 2 (MEN 2) syndrome is associated with a germ line
mutations in:
A. The p53 tumor suppressor gene.
B. The H-ras gene.
C. The N-myc gene.
D. The RET proto-oncogene.
E. The BRAF proto-oncogene.

Answer: D

29. In primary hyperparathyroidism; the following test has an importance in anticipating post
surgery bone hunger syndrome:
A. Alkaline phosphatase.
B. Erythrocyte sedimentation rate (ESR).
C. C reactive protein.
D. Preoperative blood calcium level.
E. Preoperative blood phosphate level.

Answer: A

30. Low pitch sound after thyroidectomy is usually attributed to malfunction of:
A. Omohyoid muscle.
B. Sternohyoid muscle.
C. Cricothyroid muscle.
D. Thyrohyoid muscle.
E. Sternothyroid muscle.

Answer : C

31. The most sensitive test for pheochromocytoma is:


A. Plasma free metanephrines.
B. 24 h urinary catecholamines.
C. 24 h urinary Vanillylmandelic acid (VMA).
D. Clonidine suppression test.
E. Plasma catecholamines.

Answer: A

32. Surgery for thyroglossal cyst usually includes excision of:


A. Part of Cricoids cartilage.
B. Part of Hyoid bone.
C. Part of Thyroid cartilage.
Dr. wessam alzaidat - General surgery
D. Part of overlying skin.
E. Part of Tongue muscles.

Answer: B

33. . All of the followings are suggestive of the diagnosis of sebaceous cyst except:
A. Presence of a punctum.
B. Smooth surface.
C. Soft consistency.
D. Attachment to skin.
E. Slippery edges.

Answer: E

34. All of the following features of cervical lymph nodes indicate a poor prognosis except
A. Morphology
B. laterality
C. level
D. time of presentation
E. extracapsular extension

ANSWER:D
35. All the following are true about gall bladder except
A. It is lined by highly folded tall columnar epithelium.
B. It has two muscle layers, the muscularis mucosa and muscularis propria.
C. The cystic artery is usually a branch from the right hepatic artery.
D. Cystic artery divides into anterior and posterior divisions.
E. Venous drainage is usually through small cystic veins that enter directly into the liver.

Answer: B
36. The most common site of acute biliary pain (biliary colic) is
A. Subxiphoid.
B. Left hypochondrial.
C. Right flank.
D. Right scapular region.
E. Interscapular.

Answer: A

37. All the following are true except


A. The common hepatic artery usually gives rise to the gastroduodenal artery.
B. The gastroduodenal artery gives rise to the superior pancreaticoduodenal artery.
C. The splenic artery gives rise to the inferior pancreaticoduodenal artery.
D. The inferior pancreatic artery usually arises from the superior mesenteric artery.
Dr. wessam alzaidat - General surgery
E. The inferior mesenteric vein usually drains to the splenic vein.

Answer: C

38. Which of the following enzymes is excreted from the pancreas in its active form?
A. Amylase.
B. Trypsin.
C. Chymotrypsin.
D. Elastase.
E. Phospholipase A2.

Answer: A

39. All the following are true about amylase in acute pancreatitis except
A. It starts to increase shortly after the onset of symptoms.
B. It peaks within several hours.
C. It remains elevated for 3 to 5 days in average.
D. Its serum levels in general correlate directly with the severity of disease.
E. The laboratory reading of it is commonly normal in acute pancreatitis caused by
hyperlipidemia.

Answer: D
40. 57A 45 years old female who has liver cirrhosis, she has mild ascites, no encephalopathy
but she has grade 2 esophageal varices. Her hemoglobin is 10 gm/dl, platelets count 70000
per cubic mm, INR 2, serum creatinine 2mg/dl, serum albumin 3 g/dl, and serum bilirubin
4 mg/dl. What is his Child-Turcotte-Pugh score?
A. 6.
B. 8.
C. 10.
D. 12.
E. 14.
Answer: C
41. One of the following is part of Model for End-stage Liver Disease score
A. Serum albumin.
B. Hematocrit.
C. Platelet count.
D. Serum creatinine.
E. Ascites.
Answer: D

42. All the following are true about Transjugular Intrahepatic Portosystemic Shunt (TIPS)

Dr. wessam alzaidat - General surgery


except
A. Its main indication is to stop esophageal variceal hemorrhage not responded to medical
treatment.
B. It doesn’t affect possible future liver transplant plan.
C. In improves hepatic encephalopathy.
D. It may induce renal failure.
E. Shunt thrombosis rate is high.

Answer: C

43. All the following are common findings in patients with Budd-Chiari syndrome except
A. Occlusion of the three main hepatic veins.
B. Centrilobular necrosis in liver biopsy.
C. Atrophy of Caudate lobe.
D. Decreased liver perfusion via the portal vein.
E. Hepatomegaly.

Answer: D

44. All the following are true about cholangiocarcinoma except


A. They are usually adenocarcinomas.
B. Nodular type is the most common type.
C. Most of them are located intrahepatic.
D. Most cases present with painless jaundice.
E. The tumor marker CA 19-9 can be elevated but it is nonspecific.

Answer: C

45. All of the following are true regarding familial juvenile polyposis EXCEPT ?
A. It is autosomal dominant.
B. The polyps are hamartomatous.
C. Bleeding is common symptom.
D. Once a polyp is detected, total proctocolectomy is recommended.
E. Intussusception is possible presentation.
ANSWER D

46. A 55-year-old male, who is noted to have diarrhea, flushing and bronchoconstriction as
well as right-sided cardiac valvular disease, is most likely to have his primary carcinoid
tumor in ?
A. Appendix
B. Bronchus
C. Rectum
D. Stomach
E. Ileum.
ANSWER E
47. Which of the following is a more common to cause massive colonic bleeding?
Dr. wessam alzaidat - General surgery
A. Cancer.
B. Ulcerative colitis.
C. Angiodysplasia.
D. Diverticulitis.
E. Granulomatous colitis.
ANSWER C
48. In which portion of the gastrointestinal tract are proteins mostly absorbed?
A. Stomach.
B. Duodenum.
C. Jejunum.
D. Ileum.
E. Colon.
ANSWER C
49. Mass movement is the characteristic motility pattern of which portion of the
gastrointestinal tract?
A. Stomach.
B. Duodenum.
C. Jejunum.
D. Ileum.
E. Colon.
ANSWER E
50. Which of the following has the highest risk for Colo-rectal carcinoma?
A. Ulcerative colitis.
B. Crohns disease.
C. Familial polyposis coli.
D. Diverticulosis.
E. Juvenile polyps.
ANSWER C
51. A 45-year-old man with a 20-year history of quiescent ulcerative colitis undergoes
surveillance endoscopy and is found to have a focus of low-grade dysplasia on random
colonic biopsy in the transverse colon. The most appropriate next step in management is:
A. Transverse colectomy.
B. Extended right hemicolectomy.
C. Total proctocolectomy with end ileostomy.
D. Total proctocolectomy with immediate J-pouch construction.
E. Transverse colectomy and lt hemicolectomy.
ANSWER D
52. A 36-year-old man developed neck and left arm pain. He noted paresthesia in the left
index and long fingers. He was found to have weakness of the left triceps muscle and a
diminished left triceps jerk. His left-sided disc herniation is most likely to be at:

A. C3–C4.
B. C4–C5.

Dr. wessam alzaidat - General surgery


C. C5–C6.
D. C6–C7.
E. C7–T1.

Answer: D

53. Which of the following signs does Horner's syndrome include?

A. Ptosis.
B. Facial hyperhidrosis.
C. diplopia
D. Exophthalmos.
E. Mydriasis.
Answer: A
54. In neoplasia, cells progress from normal to cancerous. One of the following is true:
A. Hyperplasia in the absence of atypia or dysplasia, has minimal or norisk of cancer
B. Atypia changes are irreversible
C. Metaplasia describes cells that have altered size, shape & organization
D. All dysplastic tissue progress to carcinoma
E. Hamartoma carries the highest risk of progression to cancer

Answer: A
55. A patient with malignant mass inquires about tumor biology. One of the following is true
A. Most malignant cells generally arise from multiple (not single) cells, and undergo
malignant transformation to form a malignant clone
B. Malignant cells proliferate slower than normal cells, but the rate of proliferation
becomes faster as the tumor size increases
C. Malignant cells are characterized by reversion to more primitive forms, with
cellular pleomorphism and loss of contact inhibition
D. Tumor doubling time is constant among different types and require 1 year at most
to detect from onset of malignant transformation
E. Anaplasia is the mildest form of malignancy

Answer: C
56. Neoplastic cells have the ability to spread. One of the following is true:
A. Lymph node metastases permeate the sinusoids of the node and later spread
throughout the subcapsular space
B. Carcinoma in situ has no detectable invasion beyond the basement membrane
C. Lymphatic involvement is common with sarcomas
D. The metastatic process of malignancy is highly efficient, as once they shed into the
bloodstream, they produce metastasis almost always
E. Epithelial neoplasm spread only through hematogenous route

Answer: B
57. With regards to tumor markers, one of the following is true:
A. Lactic dehydrogenase (LDH) has little value in lymphoma
Dr. wessam alzaidat - General surgery
B. Prostate-specific antigen can only be elevated in prostate cancer
C. Alkaline phosphatase is a useful tumor marker
D. Ca-153 is extremely useful in the diagnosis of ovarian cancer
E. Ca-19.9 can be elevated in pancreatic cancer

Answer: E
58. A 6 weeks old baby brought by his mother with umbilical discharge since birth. On
examination, it was not smelly and there was what looks like red spot inside the stump. In
this case the commonest cause will be:
A. Foreign body in the stump
B. Patent vietello intestinal duct
C. Patent urachus
D. Umbilical granuloma
E. Umbilical sepsis

ANSWER: D

59. A 2 months old baby presented with an umbilical hernia. The treatment is:
A. Observe
B. Herniotomy
C. Apply a truss
D. Inject sclerosing material
E. Truss for one year then operate
ANSWER: A
60. The commonest age that an inguinal hernia can strangulate is:
A. Up to one year of age
B. 1-2 years
C. 5 years
D. 10 years
E. Teenagers
ANSWER: A
61. A 3 year old boy is referred to the clinic with a scrotal swelling. On examination, the
mass does not transilluminate and it is impossible to palpate normal cord above it. What is
the most likely diagnosis?
A. Hydrocele
B. Direct inguinal hernia
C. Indirect inguinal hernia
D. Femoral hernia
E. Lymphatic malformation
ANSWER: C

62. A 6 day old baby was born prematurely at 33 weeks. He has been suffering from
respiratory distress syndrome and has been receiving ventilatory support on Neonatal ICU.
Dr. wessam alzaidat - General surgery
He has developed abdominal distension and is increasingly septic. Ultrasound of the
abdomen shows free fluid and evidence of small bowel dilatation. His blood pressure has
remained labile despite inotropic support. What is the best course of action?
A. Laparoscopy
B. Laparotomy
C. Contrast enema
D. Upper GI contrast study
E. MRI abdomen
ANSWER: B
63. The following best describes the sequence of sensory recovery in a healing skin graft:
A. Temperature, pain, light touch.
B. Pain, temperature, light touch.
C. Pain, light touch, temperature.
D. Light touch, pain, temperature.
E. Temperature, light touch, pain.
ANSWER C
64. Which of the following will increase the length of a scar the most

A. 30-degree Z-plasty.
B. 60-degree Z-plasty.
C. W-plasty.
D. M-plasty.
E. 10-degree Z-plasty.

ANSWER:B

65. Twenty-four hours after colon resection, urine output in a 70-year-old man is 10 mL/h.
Blood chemistry analysis reveals sodium, 138 mEq/L; potassium, 6 mEq/L; chloride, 100
mEq/L; bicarbonate, 14 mEq/L. His metabolic abnormality is characterized by which of
the following?
A. Abdominal distension
B. Peaked T waves
C. Narrow QRS complex
D. Cardiac arrest in systole
E. J wave or Osborne wave
ANSWER: B

66. A24-year-old woman is scheduled for an elective cholecystectomy. The best method of
identifying a potential bleeder is which of the following?
A. Platelet count
B. Acomplete history and physical examination
C. Bleeding time
D. Lee-White clotting time

Dr. wessam alzaidat - General surgery


E. Prothrombin time (PT)
ANSWER: B

67. A 10-year-old boy with history of prolonged bleeding after minor injury is scheduled for
inguinal hernia surgery . The bleeding time, PT, and fibrinogen are normal. What would
be the most helpful investigation?
A. Fibrinolysis (euglobulin clot lysis time)
B. Platelet count
C. Thrombin time
D. Partial thromboplastin time (PTT)
E. Factor VII assay
ANSWER: D

68. A 60-year-old woman who underwent a mastectomy for breast cancer 2 years earlier
presents to the emergency department with headache, backache, and frequent vomiting.
She is extremely thirsty and stuporous. Which test is most likely to identify the cause?
A. CT scan of the head
B. X-ray of spine
C. Serum sodium determination
D. Serum calcium determination
E. Serum glucose determination
ANSWER: D
69. One is true regarding role of FDG-PET in malignant melanoma:
A. FDG PET/CT is the modality of choice for detection of brain metastases.
B. FDG PET imaging is of limited use in patients with stage melanoma (I-II) disease.
C. FDG PET has a very high sensitivity for the detection of sentinel lymph node
metastases.
D. PET scan results have no significant impact on patient management.
E. FDG PET imaging has shown excellent results for the evaluation of uveal
melanoma.

ANSWER: B
70. One of the following is true:
A. Increased FDG uptake in a skeletal lesion is pathognomonic for malignant
neoplasms.
B. Low grade osteosarcomas show high FDG uptake.
C. Low grade soft-tissue sarcomas show high FDG uptake.
D. A decrease in the tumor to non-tumor uptake ratio of greater than 10% after
chemotherapy correlate well with extensive histologic tumor necrosis.
E. FDG PET imaging is superior to bone scintigraphy for evaluation of tumor
response to therapy as bone healing.

Dr. wessam alzaidat - General surgery


ANSWER: E
71. The median survival of metastatic colorectal cancer exceeds 20 months in newer
population studies. Median survival means:
A. The maximum survival time a patient can reach
B. The minimum survival time a patient can reach
C. The highest number of people achieving similar survival time
D. The middle point of survival data for which the data below and above that time are
equal
E. One standard deviation from the average
Answer: D
72. In a study, the age of the population studied was expressed in mean with standard
deviation as (40 years ± 9 years). If 95% of the data fall between 2 standard deviations
from the mean, then 95% of the patients age fall between:
A. 31-49 years
B. 22-58 years
C. 35.5-44.5 years
D. 13-67 years
E. Cannot tell from the data given

Answer: B
73. Which bacterial genus produces the most potent exotoxins?
A. Staphylococcus.
B. Pseudomonas.
C. Klebsiella.
D. Streptococcus.
E. Serratia.
ANSWER D

74. from a hematologic viewpoint, the loss of which immune function performed by the
spleen makes patients susceptible to overwhelming post-splenectomy infection (OPSI)?
A. Loss of IgA.
B. Loss of IgM.
C. Loss of IgG.
D. Loss of IgE.
E. Loss of IgD.
ANSWER B
75. Which of the sutures listed below would be most suitable for suturing the femoral artery
following an embolectomy?
A. 5/0 polyglactin
B. 5/0 polypropylene
C. 5/0 silk
D. 5/0 polyglyconate
E. 5/0 polyester

Dr. wessam alzaidat - General surgery


ANSWER: B
76. In contrast to non-small cell lung cancer (NSCLC), Small cell lung cancer (SCLC) is
characterized by:

A. Greater response rate to chemotherapy.


B. Inability to achieve surgical cure.
C. Less frequent association with paraneoplastic syndromes at the time of diagnosis.
D. Lower likelihood of metastases present at the time of diagnosis.
E. Slower growth.

Answer: A

77. One of the following statements about pulmonary hamartomas is true:


A. Hamartomas are benign chondromas.
B. Most are located in the conducting airways.
C. Wedge resection is curative.
D. A lobectomy is necessary to obtain draining hilar lymph nodes.
E. Hemoptysis is common.

Answer: C

78. To prevent hyperacute rejection, one of the following is true:


A. Ensure HLA matching is present
B. Give FK 506 (Prograf)
C. Ensure ABO blood group compatability
D. Give Prednisone
E. Give Azathioprine

Answer: C

79. In regards to rejection, one of the following is true:


A. Hyperacute rejection is a cell-mediated immune response
B. Acute rejection occurs secondary to preformed antibodies, as with ABO blood group
incompatibility
C. Chronic rejection never occurs against minor histocompatibility antigens
D. Chronic rejection occurs within the first month, and is easily treated with
immunosuppressive therapy
E. Acute rejection typically is cell-mediated and characterized by graft infiltration with
lymphocytes and mononuclear cells

Answer: E
80. A 32-year-old woman is brought to the emergency room with obvious head and extremity
injuries. Primary survey shows she is apneic. Which method is a definitive airway that is
best provided immediately?
A. Orotracheal intubation
B. Nasotracheal intubation
C. Cricothyroidotomy
D. Tracheostomy
E. Laryngeal mask airway
Dr. wessam alzaidat - General surgery
Answer: A
81. A 23-year-old man presents to the emergency room with a stab wound to his anterior
border of the sternocleidomastoid muscle, 1 cm above the cricoid cartilage with
penetration to the platysma. His vitals and oxygen saturation are normal, with no other
physical findings. Which of the following is considered best management?
A. Discharge the patient home, as everything seems normal
B. Observe for 6 hours, if patient drinks well, with no respiratory distress then discharge
C. Observe without testing for 24 hours. If no further complaints, then discharge
D. Perform a CT-angiogram. If no carotid injury then can be discharged
E. Formal neck exploration or perform pan endoscopy with esophagogram and angiogram

Answer: E

82. You prescribedsilver sulfadiazine ointment (e.g. Flamazine)to a patient who sustained a
third-degree burn to his dorsal right upper extremity.Oneof the following is a potential
side effect that can occur from this ointment:
A. Hyponatremia
B. Hypocalcemia
C. Methemoglobinemia
D. Hyperchloremia acidosis
E. Neutropenia

Answer: E
83. A 26-year-old man with 45% burn, admitted in the burn unit for the past 2 weeks, started
developing fever, with tachycardia. You suspected burn wound sepsis. One of the
following is true:

A. Burn sepsis can be prevented by prophylactic systemic antibiotics


B. Systemic antibiotics should never be started before wound culture & sensitivity
results are available
C. A positive wound culture mandates starting systemic antibiotics even if sepsis is not
present
D. Anaerobic organisms are the commonest to cause sepsis in burn patients
E. Culture is best obtained by tissue biopsy

Answer: E
84. A 24-year-old man was involved in a motor vehicle accident. His car caught fire with
evidence of carbonaceous sputum, edematous oropharynx, tachypnea, and stridor. The
patient is hemodynamically stable with oxygen saturation of 97%.Before attempting
intubation, which of the following maneuvers is necessary?
A. Avoid oxygenation with his inhalational injury
B. Axial stabilization of his cervical spine
C. Administer paralyzing agents
D. Administer morphine for sedation
E. Immediate needle cricothyroidotomy

Dr. wessam alzaidat - General surgery


Answer: B
85. After significant blunt trauma, a patient arrives to the emergency room with Glasgow
Coma Scale (GCS) of 4/15. His oxygen saturation is poor at 89% after applying a non-
rebreather mask. His eyes are bruised and blood-stained fluid is coming out of his ears.
The method of airway management that is best avoided:
A. Oropharyngeal intubation
B. Laryngeal mask airway
C. Cricothyroidotomy is oropharyngeal is unsuccessful
D. Nasopharyngeal intubation
E. Oropharyngeal intubation with bronchoscopy guidance

Answer: D

86. Which tissue layer provides for primary digestive motility?

A. Mucosa
B. Submucosa
C. Mesentery (a fold of tissue that a aches organs to the body wall)
D. Muscularis Mucosa
E. serosa

Answer: D
87. The "gold standard" for the identification of the kidney causing abnormal renin secretion
is:
A. Single-dose Captopril tests
B. Differential renal vein renin determination
C. IVU
D. Captopril renography
E. Angiography

Answer: B

88. All of the following are indications for partial nephrectomy in patients with renal
carcinoma EXCEPT:
A. Bilateral renal cell carcinomas
B. Localized, unilateral renal cell carcinoma in patients with renal insufficiency
C. Localized renal cell carcinoma and a contralateral kidney involved with renal
calculi
D. Localized renal cell carcinoma of any size and a normal contralateral kidney
E. Renal cell carcinoma in a solitary kidney

Answer: D

89. Which of the following is not a cause of gangrene?


A. Buergers disease.
B. Infection.
C. frostbite.
D. Chronic venous insufficiency.
Dr. wessam alzaidat - General surgery
E. intra-arterial drug injection.
ANSWER D

90. One of the following regarding deep venous thrombosis is not true?
A. Doppler ultrasound imaging is useful for detecting occlusions of major veins.
B. Post thrombotic syndrome results in venous valvular incompetence and subsequent
venous hypertension.
C. Vena cava filters can be placed in pregnant women.
D. Early clot thrombolysis is associated with improved long-term outcomes.
E. Doppler ultrasound imaging can differentiate between old and new thrombi.
ANSWER E
91. The initial phase of wound healing is best characterized by only one of the following:
A. Begins 5-6 hours after the wound event
B. Bradykinin causes vasoconstriction, inhibiting the migration of neutrophils to the
wound
C. Neutrophils trigger fibroblast response to start the following phase
D. Complement components together with Platelet derived growth factor act as
chemotactic factors to stimulate migration of neutrophils to the wound
E. Monocytes differentiate into neutrophils 24 hours after the injury

Answer: D

92. A 7-year-old boy sustained a cut on his forehead that requires suturing. The parents ask
you how the wound stopped bleeding although it was not sutured. Which of the following
is the explanation for the finding?
A. Initial bleeding cause vessels to vasoconstrict, release tissue thromboplastic factors
and activating the coagulation and complement cascade
B. Without pressure by the parents, the bleeding would have never stopped.
C. Kinins and prostaglandins maintain the vasoconstriction to aid in stopping the
bleeding
D. Epithelialization occurs within minutes to aid in restoring integrity of bleeding
vessels
E. Thrombin & Fibrinogen inhibit platelets & monocytes as vasoconstriction is enough
to stop bleeding.

Answer: A
93. When vein grafts are used to bridge intra-arterial defects: one of the following is true?
A. The vein wall thins due to the increased luminal pressure.
B. Graft length decreases by 25-30% long term.
C. The graft stretches by 20-30%
D. The vein wall thickens significantly.
E. There is no ingrowth of smooth muscle cells.

ANSWER D

Dr. wessam alzaidat - General surgery


94. One of the following is not a risk factor for gastric cancer:
A. Pernicious anemia
B. Helicobacter Pylori
C. Partial gastrectomy
D. Blood group O
E. Dried fish

ANSWER: D
95. The lower esophageal sphincter pressure at rest and during a swallow is:
A. 70 mmHg and 15 mmHg
B. 70 mmHg and 15 mmHg
C. 15 mmHg and 0 mmHg
D. 70 mmHg and 15 mmHg
E. 0 mmHg and 15 mmHg

ANSWER: C
96. Parietal cells of the stomach secrete is one of the following:
A. Pepsinogen
B. Gastrin
C. HCL and Intrinsic factor
D. Secretin
E. Cholecystokinin

ANSWER: C
97. Which of the following structures results in severe peptic ulcer disease not amenable to
conventional medical treatment?
A. Zolliner-Ellison syndrome
B. Plummer-Vinson syndrome
C. Hereditary Non-Polyposis Colon Cancer (HNPCC)
D. CREST syndrome
E. Gardiner’s syndrome

ANSWER: A
98. Intrinsic factor binds one of the following:
A. Copper
B. Vitamin B6
C. Folic acid
D. Iron
E. Vitamin B12

ANSWER: E

99. The most important step in management of Zenker’s diveticulum of the esophagus is:
A. Esophagectomy
B. Excision of the diverticulum
C. Division of the cricopharyngeus muscle
D. Observation

Dr. wessam alzaidat - General surgery


E. Division of the superior laryngeal constrictor muscles

ANSWER: C
100. A 25 year old male presents with symptoms of obstructed defecation. On
examination he is found to have a small full thickness external rectal prolapse. The most
appropriate management for the following patient is
A. Resectional rectopexy
B. Laparoscopic ventral mesh rectopexy
C. Stapled transanal resectional rectopexy
D. Anterior resection
E. Banding of prolapse

ANSWER: C

Dr. wessam alzaidat - General surgery


PAST BOARDS MCQs
1- Arterial injuries that can be safely ligated in the unstable
trauma patient include all except :

a- SMA
b- radial art
c- inferior mesentric art
d- internal iliac artery
e- celiac art

Answer:A

2-in which of the following venous injuries is ligation most


tolerated :
a- infra renal vena cava
b- supra renal vena cava
c- common femoral vein
d- popliteal vein
e- innominate vein

Answer:A

3-following blunt abdominal trauma , mandatory exploration is


indicated for any non expanding hematoma identified on CT –
scan in which of the following areas :

a- Rt . perinephric
b- mid line infra mesocolic
c- lateral pelvic area
d- retro hepatic
e- lt. perinephric

Answer:B

Dr. wessam alzaidat - General surgery


4-which structure is the most sensitive to hypoxia?

a-skin
b- bone
c-non myelinated nerve
d- myelinated nerve
e- skeletal muscle

Answer:C

5-Pulmonary artery catheter is used to measure which variable


directly :-

a-cardiac index
b- systemic vascular resistance
c- mixed- venous oxygen saturation
d- left ventricular end diastolic index
e- pulmonary vascular resistance index

Answer:B

6-An example of anti-inflammatory cytokine is:


a-IL-2
b-IFN-y
c-Lymphotoxin-alpha
d-IL-4
e-TNF-alpha

Answer:D

7-Trendelenburg position considered a viable treatment option


for which category of shock?

a-Cardiogenic
b-Neurogenic
c-Hypovolemic
d-Septic
e-Cardiac compressive

Dr. wessam alzaidat - General surgery


Answer:B

8-All of the following symptoms are seen with


hypermagnesemia except:

a-Tachypnea
b-Depression of reflexes
c-Arrythmias
d-hypotension
e-Central nervous system depression

Answer:A

9-The most likely cell of origin for gastrointestinal stromal


tumor(GIST):

a-Interstitial cells of Cajal


b-Kulchitsky cells
c-Myofibroblasts
d-G-cells
e-Paneth cells

Answer:A

10-Which of the following regarding inflammatory bowel


disease is true:
a-Bloody diarrhea is more common in crohns disease than in
ulcerative colitis.
b-Associated extra intestinal disease is less common in
ulcerative colitis than in crohns disease
c-Crohns disease frequently involves the entire colon
d-Crohns is more commonly associated with primary sclerosing
cholangitis than is ulcerative colitis.
e-Surgical intervention for ulcerative colitis is limited to
management of complications of the primary disease process.

Answer:B
Dr. wessam alzaidat - General surgery
11-Each of the following is associated with familial polyposis
syndrome except:

a-Gardners syndrome
b-Turcots syndrome
c-MEN type 2b
d-Peutz-Jeghers syndrome
e-Juvenile polyposis syndrome.

Answer:C

12-Surgical exploration for patient with primary


hyperparathyroidism reveals all four glands to be:
enlarged.What is the most appropriate way to manage?

a-Closure with localization study


b-Incision biopsy of all glands
c-Subtotal parathyroidectomy
d-Excision of the largest enlarged gland
e-Selective venous PTH sampling.

Answer:C

13-The most common clinical manifestation of MEN 1 is:

a-Hypoglycemia.
b-Renal stones
c-Galactorrhea
d-Osteoporosis
e-Peptic ulcer disease.

Answer:E

14-In MEN 1, the most common pancreaticodoudenal tumors


are
:
a-Gastrinomas
Dr. wessam alzaidat - General surgery
b-Non functioning tumors
c-Insulinomas.
d-Somatostatinomas.
e-VIPomas.

Answer:B

15-Cholecystokinin:
a-Relaxes the sphincter of oddi.
b-Inhibits gastric acid secretion.
c-Inhibits gall bladder contraction.
d-Causes mesenteric vasodilatation.
e-Inhibits pancreatic exocrine secretions.

Answer:A

16-Rectus sheath hematomas


a-Can be caused by coughing.
b-Are rarely associated with anticoagulative therapy.
c-Usually occur at the semicircular line of Douglas at the entry
site of superior epigastric artery into the rectus sheath.
d-Are infrequently palpable on physical examination.
e-Usually require operative drainage.

Answer:A

17-Most primary and metastatic tumors to liver derive nearly all


their vascular inflow from branches of:
A-Portal vein.
b-Collateral circulation.
c-Hepatic artery.
d-Celiac axis.
e-Unnamed branches of aorta.

Answer:C

18-Which of the following is not a complication of Caroli


Dr. wessam alzaidat - General surgery
disease:-
a-Biliary stone formation.
b-Recurrent cholangitis.
c-Septicemia.
d-Cholangiocarcinoma.
e-Renal disorders.

Answer:E

19-Which of the following is not considered a risk factor for gall


bladder cancer?
a-Gall stones>3cm in size.
b-Multiple small gall stones.
c-porcelain gall bladder
d-Biliary salmonella typhi infection.
e-Gall bladder adenoma.

Answer:B

20-Which is true about surgical anatomy of pancreas:-


a-The head lies over the first lumber vertebra.
b-The accessory duct enters the second part of duodenum.
c-The superior mesenteric vessels pass behind the uncinate
process.
d-The SMA lies to the right of the superior mesenteric vein.
e-The superior mesenteric vein join the portal vein behind the
head of pancreas.

Answer:D B

21-Most common indication for splenectomy for a red cell


enzymatic defect is:
a-Hereditary spherocytosis.
b-G6PD.
c-Private kinase deficiency.
d-Hereditary high red blood cell phosphatidyl choline anemia.
e-Cold-agglutinin syndrome.
Answer:C

Dr. wessam alzaidat - General surgery


22-Which of the following is true of retroperitoneal soft tissue
sarcomas:

a-The most common histological cellular type is malignant


fibrous histiosarcoma.
b-Pre-operative biopsy is contraindicated because of tumor
seeding risk.
c-Resection of contiguous organs that are not involve with
tumor is indicated to obtain adequate margins.
d-Death usually results from distant metastases.
e-Lung is the most common site of distant metastases.

Answer:C

23-Which of the following structures form the medial border of


the (triangle of doom) to be avoided in laparascopic inguinal
herniorrhaphy?

a-Inferior epigastric vessels.


b-Vas deferens.
c-Spermatic vessels.
d-Iliac vessels.
e-Lateral femoral cutaneous nerve.

Answer:B

24-A 45 year old man who has been HIV positive for 15 years
has a painful 3-cm neck mass. A 2 week course of antibiotics
does not change the mass. Biopsy will most reveal:
a-Hodgkin’s lymphoma.
b-B cell lymphoma.
c- T cell lymphoma.
d- Metastatic lung carcinoma.
e- Kaposi’s sarcoma.

Answer:B
25-Risk factors for male breast cancer include:-
Dr. wessam alzaidat - General surgery
a-Gynecomastia
b-BRCA 1 gene mutation
c- BRCA 2 gene mutation
d- Anabolic abuse.
e- Cannabis abuse.

Answer:B

26-A 2-cm gastric ulcer in the antrum of the stomach is


associated with all of the following except:
a-Helicobacter pylori.
b-Increased acid secretion.
c- Malignancy.
d-nonsteroidal anti-inflammatory drugs.
e-Atrophic gastritis.

Answer:B

27-The most common presenting clinical sign in Budd-Chiari


syndrome is:
a-Ascitis.
b-Esophageal varices.
c-Jaundice.
d-Encephalopathy.
e-Hemorroids.

Answer:A

28-Percutaneous image guided drainage of intra abdominal


abscesses is LEAST successful in which of the following sites?
a-Subphrenic.
b-Hepatic.
c-Peripancreatic.
d-Diverticular.
e-Appendiceal.

Answer:C
29-In the workup on a patient for possible appendicitis, CT
Dr. wessam alzaidat - General surgery
scanning should be performed:
a) Before consulting the surgeon, by the emergency physician
b) In patients with equivocal physical findings
c) Routinely, in all patients with right lower quadrant pain
d) With equal frequency in men and women
e) Never

Answer:b

30-The gastric mucosal cell that secretes intrinsic factor is the


a) G cell
b) Parietal cell
c) D cell
d) Enterochromaffin-like cell
e) Chief cell

Answer:b

31-Hypertrophic pyloric stenosis is likeliest to occur in a(n)


a) Firstborn child
b) African-American infant
c) Child 6 to 9 months of age
d) Female infant
e) Infant born prematurely

Answer:a

32-Overwhelming postsplenectomy infection (OPSI) :


a) Occurs more frequently after resection for trauma than
hematologic disease
b) Occurs with equal frequency in children and adults
c) Is most frequently caused by Streptococcus pneumoniae
d) Usually occurs within 2 years after splenectomy
e) Generally has an identifiable site of infection

Answer:c
33-The characteristic feature of Crohn's colitis that best

Dr. wessam alzaidat - General surgery


distinguishes the clinical entity from ulcerative colitis is
a) Perianal disease
b) Rectal bleeding
c) Risk of malignancy
d) Obstructive symptoms
e) Pseudopolyps

Answer:a

34-Type 1 gastric ulcers are


a) At the incisura
b) Along the greater curvature
c) Prepyloric
d) Associated with simultaneous duodenal ulcers
e) Close to the esophagogastric junction

Answer:a

35-Radiation enteritis
a) Usually presents with perforation
b) Is caused by thrombosis of mucosal vessels
c) Occurs after 3,000 cGy of abdominal radiation
d) Routinely requires operative therapy
e) Is likely in patients who have undergone laparotomy

Answer:e Occur after >4500 cGy

36-The small bowel tumor with the greatest propensity for


bleeding is
a) Carcinoid
b) Lymphoma
c) Adenocarcinoma
d) Hamartoma
e) Leiomyoma

Answer:e
37-Gastric stump cancers (gastric adenocarcinomas in patients

Dr. wessam alzaidat - General surgery


who underwent gastric resection)
a)Are the consequence of Helicobacter pylori infection
b)Are generally of the polypoid type
c)Have a 5-year survival rate of 40%
d)Occur within 5 years after gastrectomy
e)Are caused by bile reflux

Answer:e

38-A grade III laceration of the spleen is characterized by


a)Intraparenchymal hematoma > 5 cm
b)Capsular tear 1-3 cm deep
c)Hilar vessel disruption
d)Subcapsular hematoma involving 10%-50% of surface area
e)Active hemorrhage

Answer:a

39-The spleen filters all of the following particles/cells


EXCEPT
a. Malformed erythrocytes
b. T lymphocytes
c. Malarial parasites
d. Streptococcus pneumoniae
e. Platelets

Answer:b

40-Acute appendicitis
a.Occurs most commonly in the second and third decades of life
b.Can be cured readily by antibiotics
c.Is most commonly caused by a fecalith
d.Carries an overall mortality rate of 7%
e.Induces leukocytosis in 90% of patients

Answer:A
41-The stomach bed does not include :

Dr. wessam alzaidat - General surgery


a.Splenic artery.
b.Celiac trunk.
c.Transverse mesocolon.
d.Left adrenal gland.
e.Neck of the pancreas.

Answer:E

42-which of the following is the strongest of all other risk


factors in the development of Gastric carcinoma :
a.Helicobacter pylori.
b.Atrophic gastritis.
c.Blood group A.
d.Pernicious anemia.
e.Low socioeconomic class.

Answer:B

43-whilst performing a small bowel resection for strictures


following crohn”s disease, you realize that on inspection, there
are marked differences between jejunal and ileal anatomy. Such
differences include all the following Except:

a.Wider lumen in the jejunum.


b.Less lymphatics in the jejunal mesentry compared to ileal.
c.More prominent and multiple arcades of vessels in the ileum.
d.Thicker wall of the jejunum.
e.Thicker and more fat-laden mesentry increasing towards the
ileum.

Answer:e

44-A 54 years old woman is referred to your surgical team with

Dr. wessam alzaidat - General surgery


a diagnosis of small bowel obstruction. Which one of the
following clinical signs would you look for in trying to identify
the commonest cause of this condition :

a.surgery scar.
b.Lump in the groin above & medial to the pubic tubercle.
c.Lump in the groin below & lateral to the pubic tubercle.
d.Cachexia & nodule at the umbilicus.
e.Circumoral pigmentation & a family history of previous
obstruction.

Answer:A

45-True statements regarding appendiceal neoplasms include


which of the following?

a.Carcinoid tumors of the tip of the appendix less than 1.5 cm


are adequately treated by simple appendectomy
b.Appendiceal carcinoma is associated with secondary tumors of
the GI tract in up to 60% of patients
c.Survival following right colectomy for a Dukes’ stage C
appendiceal carcinoma is markedly better than that for a
similarly staged colon cancer at 5 years
d.Mucinous cystadenocarcinoma of the appendix is adequately
treated by simple appendectomy, even in patients with rupture
and mucinous ascites
e.Up to 50% of patients with appendiceal carcinoma have
metastatic disease, with the liver as the most common site of
spread

Answer:A

46-All of the following statements about carcinoma of the


gallbladder are correct Except :

a.The neoplasm usually starts in the cystic duct and neck of the
gallbladder.

Dr. wessam alzaidat - General surgery


b.It is found more commonly in women than men.
c.It is associated with the presence of gallstones in > 85% of
cases.
d.Prognosis is generally poor with < 1 year survival with local
invasion.
e.Chemotherapy and radiotherapy do not alter disease
progression.

Answer:A

47-All of he following statements are true regarding diverticular


disease Except :

a.It is found more commonly in the developed world.


b.Surgical treatment is usually unnecessary in acute
uncomplicated cases.
c.Diverticulae are more commonly found in the descending
colon.
d.Perforation and fistula formation can result from an attack of
acute diverticulitis.
e.Resolution of the diverticulae can occur with high fiber diet
and adequate hydration.

Answer:E

48-All of the following arteries are branches of the superior


mesenteric artery, except:
a.Ileocolic
b.Replaced left hepatic
c.Inferior pancreaticoduodenal
d.Jejunal
e.Replaced right hepatic

Answer:b

49-Mesenteric cysts
a.Occur primarily in the mesocolon

Dr. wessam alzaidat - General surgery


b.Are discovered mainly in children
c.Can contain chyle or serous fluid
d.Require total enucleation for a cure
e.Are neoplastic

Answer:c

50-Decompression for abdominal compartment syndrome


should be performed
a.If the urine output falls to 30 mL/hour
b.When the patient's respiratory rate increases to 24 breaths per
minute
c.Based purely on physical findings
d.When bladder pressure exceeds 35 mm Hg
e.If the patient becomes hypoxemic

Answer:d

51-In laparoscopic surgery, the vessel most likely to be


punctured during trochar placement is the
a.Aorta
b.Right common iliac artery
c.Vena cava
d.Left common iliac artery
e.Right common iliac vein

Answer:b

52-A walled-off pelvic abscess secondary to sigmoid


diverticulitis is Hinchey stage
a.I
b.II
c.III
d.IV
e.V
Answer:b
53-Following resolution of acute appendicitis, an interval
appendectomy
Dr. wessam alzaidat - General surgery
a.In 90% of interval appendectomies and no pathologic
abnormality.
b.Always reveals luminal occlusion of the appendix
c.Should be performed because of the high incidence of
recurrent appendicitis
d.Is not indicated in patients older than 40 years
e.Needs to be performed open because of the fibrosis induced by
appendicitis

Answer:a

54-True statements concerning the diagnosis and management


of retroperitoneal fibrosis include all of the following except:

a.Most patients present with dull, non-colicky back, flank, or


abdominal pain
b.Evidence of impaired renal function with an elevated blood
urea nitrogen is common
c.The diagnosis is most commonly suggested by intravenous
pyelography although contrast studies with CT scan or MRI are
useful in further defining the disease
d.Most patients will need operative intervention.
e.The prognosis for nonmalignant retroperitoneal fibrosis is
grim with progression of disease until death occurring in most
patients

Answer:E

55-Lynch Syndrome all are true except?

a) It is due to mutation in MMR gene


b) Associated with APC gene
c) It has Autosomal Dominant Inheritence
d) Adenomas in patients with Lynch syndrome display high
grade dysplasia than adenoma in patients with sporadic
colorectal cancer
e) It occurs predominantly on right side, has increased incidence

Dr. wessam alzaidat - General surgery


of synchronous and metachronous disease.

Answer:B

56-About chronic pancreatitis all are true except?


a) Parlington Rochelle operation is longitudanal
Pancreaticojejunostomy
b) Whipple Pancreaticoduodenectomy removes 60% of
pancreatic parenchyma and has 25% incidence of Diabetes
c) Frey's operation is Duodenal Presereving Pancreatectomy
d) Beger's operation is Duodenal Preserving Pancreatectomy
e) is a different disease entity from acute pancreatitis

Answer:E

57-Which of the folowing about Pancreatic Ascites is not true?


a) Conservative treatment effective in only 1/4th of patients
b) ERCP should be done before surgery
c) It is exudative
d) Metaplastic cells are present
e) Pancreatic fluid has high amylase and high albumin

Answer:A

58-Tetanus all are true except?


a. Is due to an infection with a gram-negative spore forming rod
b. The organism produces a powerful exotoxin
c. The toxin prevents the release of inhibitory neurotransmitter
d. Clostridium tetani is sensitive to penicillin
e. Risus sardonicus is the typical facial spasm

Answer:A

59-Regarding gas gangrene


a. It is due to Clostridium botulinum infection
b. Clostridial species are gram-negative spore forming
Dr. wessam alzaidat - General surgery
anaerobes
c. The clinical features are due to the release of protein
endotoxin
d. Gas is invariably present in the muscle compartments
e. antibiotics alone are the main stay of treatment

Answer:D

60-A 34-year-old morbidly obese diabetic woman underwent a


gastric bypass about 12 hours ago. The operation was
technically difficult but finally went well. You are called
because she now has a temperature of 99.2‫ ؛‬F, pulse of 134, and
some pain in her incision and her back. She looks well; the
incision is clean; and her examination is otherwise negative. A
bolus of 500 ml. of dextrose/lactated Ringer's did not change her
vital signs, except that her pulse rise to 140 without an increase
in urine output. Your next step should be:

a.Another bolus of crystalloids.


b.Posterioanterior and lateral chest films.
c.Obtain white cell count, differential count, and electrolytes
d.Call the operating room and warn them that you need to re-
explore for a leak.
e.Increase her pain medication.

Answer: D

61-All of the following statements about the embryology of


Meckel's diverticulum are true except:

a.Meckel's diverticulum usually arises from the ileum within 60


cm. of the ileocecal valve.
b.Meckel's diverticulum results from the failure of the
vitelointestinal duct to obliterate.
c.The incidence of Meckel's diverticulum in the general
population is 5%.
d.Meckel's diverticulum is a true diverticulum possessing all

Dr. wessam alzaidat - General surgery


layers of the intestinal wall.
e.Gastric mucosa is the most common ectopic tissue found
within a Meckel's diverticulum.

Answer: C

62-A 43-year-old woman presents with complaints of anal pain


and spotting of blood with defecation. Physical examination
reveals a 2 3 cm area of ulceration within the anal canal. The
remainder of the physical examination is normal. Incisional
biopsy is positive for squamous cell carcinoma. Appropriate
management includes which of the following?

a.Abdominoperineal resection
b.Wide local excision, skin grafting, proximal diverting
colostomy
c.Primary radiation therapy
d.Local excision and primary closure
e.chemotherapy

Answer: C

63-The initial goal of therapy for acute toxic cholangitis is to:


a.Prevent septicemia by decompressing the duct system.
b.Remove the obstructing stone, if one is present.
c.Alleviate jaundice and prevent permanent liver damage.
d.Prevent the development of gallstone pancreatitis.
e. Prepare the patient for urgent cholangiography.

Answer: A

64-The clinical picture of gallstone ileus includes all of the


following Except?
a.Air in the biliary tree.
b.Small bowel obstruction.
c.A stone at the site of obstruction.
d.Acholic stools.

Dr. wessam alzaidat - General surgery


e.Associated bouts of cholangitis.

Answer: D

65- The following Nyhus classification of hernias is correct


except for:
A. Recurrent direct inguinal hernia—Type IVa.
B. Indirect inguinal hernia with a normal internal inguinal
ring—Type I.
C. Femoral hernia—Type IIIc.
D. Direct inguinal hernia—Type IIIa.
E. Indirect inguinal hernia with destruction of the transversalis
fascia of Hesselbach's triangle—Type II.

Answer: E BOARD 8 2022

66- The following statements about the repair of inguinal


hernias are true except:
A. The conjoined tendon is sutured to Cooper's ligament in the
Bassini hernia repair.
B. The McVay repair is a suitable option for the repair of
femoral hernias.
C. The Shouldice repair involves a multilayer, imbricated repair
of the floor of the inguinal canal.
D. The Lichtenstein repair is accomplished by prosthetic mesh
repair of the inguinal canal floor in a tension-free manner.
E. The laparoscopic transabdominal preperitoneal (TAPP) and
totally extraperitoneal approach (TEPA) repairs are based on the
preperitoneal repairs of Cheattle, Henry, Nyhus, and Stoppa.

Answer: A

67-Which of the following statements is true concerning the


diagnosis and management of epigastric hernias?
A. A large peritoneal sac containing abdominal viscera is
common

Dr. wessam alzaidat - General surgery


B. At the time of surgical repair, a careful search for other
defects should be performed
C. Recurrent epigastric hernias after simple closure is
uncommon
D. Patients with symptoms of a painful midline abdominal mass
frequently will contain incarcerated small bowel

Answer: b

68- Chylous ascites is the accumulation of chyle within the


peritoneal cavity. Which of the following statement(s) is/are true
concerning chylous ascites?

a.The cisterna chyli lies at the anterior surface of the first and
second lumbar vertebrae and receives lymphatic fluid from the
mesenteric lymphatics
b.Chylous ascites is most commonly associated with abdominal
lymphoma
c.Paracentesis and analysis of chylous fluid typically reveals
elevated triglycerides, protein, and leukocyte levels. The
cytology is seldom positive despite the presence of malignancy.
d.Treatment of chylous ascites with dietary manipulation will be
successful in most cases
e.The mortality rate in adults with chylous ascites is in excess of
50%

Answer:D

69- A 48-year-old woman maintained on Warfarin for a history


of cardiac valvular replacement and a history of recent upper
respiratory infection presents with severe abdominal pain
exacerbated by movement. Her physical examination shows
tenderness in the right paramedian area with voluntary guarding
but no peritoneal signs. Which statement is true concerning the
diagnosis and management of this patient.

a.Urgent laparotomy should be performed because of concern

Dr. wessam alzaidat - General surgery


for arterial mesenteric embolus
b.The correct diagnosis could likely be made by CT scan and
operation avoided
c.The status of her anticoagulation should be checked and if her
prothrombin time is excessively prolonged,surgery is necessary
d.If untreated, hemodynamic instability is common
e.The usual cause is severe trauma.

Answer: b

70-True statements concerning the diagnosis and management


of retroperitoneal fibrosis include:

a.Most patients present with dull, non-colicky back, flank, or


abdominal pain
b.Evidence of impaired renal function with an elevated blood
urea nitrogen is common
c.The diagnosis is most commonly suggested by intravenous
pyelography although contrast studies with CT scan or MRI are
useful in further defining the disease
d.Most patients can be managed nonoperatively
The prognosis for nonmalignant retroperitoneal fibrosis is grim
e.with progression of disease until death occurring in most
patients

Answer: a, b, c

71- In biliary atresia:


a. Medical treatment can be curative.
b. Operative success for bile drainage improved if surgery
delayed after 3 months.
c. its rarely progress to cirrhosis.
d. indirect bilirubin level above 2 mg\dl is suggestive of the
diagnosis.
e. liver biopsy shows inflammation and proliferation of the bile
ducts.

Dr. wessam alzaidat - General surgery


Answer: E

72- The best of the following to diagnose Hirschsprung’s


disease is:
a. Aganglionosis of dilated bowel segment.
b. Presence of hypertrophied nerve trunks in rectal biopsy
specimen.
c. Barium enema examination.
d. Failure to pass meconium in the first 24 hours.
e. Relaxation of internal sphincter during anorectal manometry
study.

Answer:B

73- A infant with intussusception:


a- A pathologic lead points is usually identified.
b- The site of origin is usually in the small bowel.
c- Triad of colicks, red currant Jelly stool and palpable
abdominal mass presents in less than 50% of cases.
d- Surgical resection is mostly required.
e- Peaks around 4 years of age.

Answer:C

74-10 years boy presented to ER with abdominal pain after


minor trivial blunt Lt lower chest wall trauma, on exam: the HR
130,BP 70/40, the chest exam was unremarkable, the abdomen
distended and tender all over and absent bowel sounds. His
mother reported history of cervical lymphadenopathy with Flu
like illness 2 weeks ago. Most likely this child’s scenario
resulted from:
a- Mesenteric lymphadenitis.
b- Acute non obstructive appendicitis.
c- Meckle’s diverticulitis.
d- Splenic injury.
e- Fractured rib.

Answer:D
Dr. wessam alzaidat - General surgery
75- 3 years old girl noticed by her mother to have a lump around
her labia that’s firm and tender which can’t be pushed back in
and the physician appreciated tenderness in Rt lower quadrant,
she most likely have:
a- Acute appendicitis
b- Inguinal lymphadenopathy.
c- Incarcerated ovary in indirect inguinal hernial sac.
d- She experienced sexual abuse.
e- Bartholin gland abscess.

Answer:C

76- A 5 year old child who suffered from vomiting for 2 days,
when presented he was lethargic, tachypnic and febrile and his
abdominal exam revealed generalized tenderness and rigidity
without localization, the next step:
a- Order an abdominal CT scan with oral contrast and IV
contrast.
b- Rule out intussusception by Barium enema.
c- Start broad spectrum IV antibiotics and observe.
d- Diagnostic lap. and accordingly.
e- Consider rigid esophagoscopy.

Answer is: D

77- A 6 years old girl with anterior neck mass with overlying
redness which rises when she sticks out her tongue, the mom
reports same presentation 2 months earlier which improved with
oral antibiotic but persisting lump, which of the following is
true:
A- Prepare for surgery to incise and drain the presumed abscess.
B- Excise the body of hyoid bone during surgery to avoid
recurrence.
C- Start on IV steroids to decrease the edema.
D- Tracheostomy is mandatory.
E- Secure the airway with endotracheal tube.

Dr. wessam alzaidat - General surgery


Answer: B

78- On the 30th day of a healthy looking newborn, he started to


have persistent strongly projectile non-bilious vomiting after
feeding for 2 days, you suspect hypertrophic pyloric stenosis,
which of the following is true:
a- An impalpable abdominal mass rules out the above diagnosis.
b- A double bubble sign on X-ray film.
c- Immediate Pyloromyotomy must be done to resume feeding
soon.
d- Rule out other important congenital anomalies (VACTERL).
e- Ultrasonography is superior to upper GI series.

Answer is: E

79- All the following are associated with Tracheoesophageal


fistula except:
a- Congenital heart disease.
b- Hemivertebrae.
c- Omphalocele.
d- Horseshoe kidney.
e- Imperforate anus.

Answer: C

80- The best management of irreducible inguinal hernia in a 2


months old infant is:
a- Reduction if successful and follow up till the age of one year.
b- Reduction if successful and surgery after 2-5 days.
c- Immediate surgical management.
d- Consider mesh repair to prevent recurrence.
e- Reduction if successful and surgery after 1 month.

Answer: B

Dr. wessam alzaidat - General surgery


Dr. wessam alzaidat - General surgery
RANDOM BOARDS MCQs
<1>Hypercalcemia (acute): The first line of management Is:
A.Fluid volume correction
B.Furosemide
C.Steroids
D.Hemodialysis
E.Mithramycin

Answer:A

<2>All the followings are hazards of bicarbonate therapy, Except:


A.Shifts the hemoglobin dissociation curve to the left
B.Paradoxical acidosis
C.Hypernatremia and hyperosmolality
D.Decreases the plasma level of ionized calcium
E.Activates the simultaneously administered catecholamines

Answer:E

<3> Blood transfusion fatal reactions, the most common cause Is:
<A>An allergic reaction
<B>An anaphylactoid reaction
<C>A clerical error
<D>An acute bacterial infection transmitted in blood
<E>Hypokalemia

Answer:C

<4> Humoral immunity, which type of cell is NOT involved?


<A>Plasma cell
<B>B cell
<C>Memory cell
<D>Killer T cell
<E>Th2 cell

Answer:D

<5>Total parenteral nutrition (TPN): Deficiency of which of the


following mineral occurs in patient on TPN:
<A>Chromium
<B>Manganese
<C>Zinc
<D>All of the above
<E>None of the above

Answer:D

<6>Diathermy, all are true, Except:


<A>Works by using a direct current
<B>The frequencies used range from 400kHz to 10MHz
<C>A cutting effect is produced by using a continuous output
<D>A coagulation effect is produced by using a pulsed output
<E>Maximal current density and heating effect is seen at the point of least cross-sectional area

Dr. wessam alzaidat - General surgery


Answer:A

<7>Immunoglobulins, all are true, Except:


<A>Rhesus antibodies are of IgG class
<B>IgG is the 1st response to invading organisms
<C>IgA has the highest concentration in human secretion
<D>IgM is the best complement activator
<E>IgE mediates allergy by binding antigen on the membrane of the mast cell

Answer:C

<8> 'Keratoderma blenorrhagicum' is seen in:


<A>Crohn's disease
<B>Ulcerative colitis
<C>Reiters' disease
<D>Gonorrohea
<E>Diverticulosis

Answer:C

<9>Tetanus, all are true, Except:


<A>It is a wound infection
<B>Low grade fever is present
<C>The shorter the incubation period the worse is the prognosis
<D>It may be mistaken for acute abdomen
<E>It is mainly a carpopedal spasm

Answer:E

<10>After hospitalization for a parasternal knife wound, all of the following may be anticipated,
Except:
<A>Distended neck veins
<B>Hypotension
<C>Pulsus paradoxus
<D>Increased heart sounds
<E>Pleural effusion

Answer:D

<11>The most appropriate method to diagnose small bowel injury in a conscious trauma patient with
seatbelt injury is:
<A>Diagnostic peritoneal lavage
<B>Ultrasound
<C>Computed tomography scan
<D>Serial abdominal examination
<E>Plain abdominal film

Answer:D

<12>Parotid Pleomorphic adenoma: All are true, Except:


<A>It is the most common parotid gland tumor and it accounts for 70-80% of all benign parotid
tumors
<B>Malignant transformation is seen in 30 % of cases and is usually associated with tumors that have
been present for 1-2 years
<C>Pain does not indicate that a neoplasm is malignant; however, in patients with known malignancy,
pain is a poor prognostic sign
Dr. wessam alzaidat - General surgery
<D>Facial weakness is sign of malignancy, and indicates a poor prognosis
<E>It is more common in women

Answer:B

<13>Tongue ulcers, all are true, Except:


<A>Dental ulcers commonly occur at the side of the tongue
<B>Syphilitic ulcers occur typically in the midline
<C>Malignant ulcers are usually adenocarcinoma type
<D>Tuberculous ulcers occur on the dorsum
<E>If the ulcer does not heal promptly when the apparent cause is removed, it must be biopsied

Answer:C

<14>Cystic swelling appearing in front of the upper part of sternomastoid muscle in a young adult,
suggests:
<A>Carotid body tumor
<B>Cystic hygroma
<C>Tuberculous lymph node
<D>Ranula
<E>Branchial cyst

Answer:A

<15>Lateral aberrant thyroid tissue, the treatment is:


<A>Reassurance
<B>Thyroxin
<C>Radical surgery
<D>Radioactive iodine
<E>External beam radiotherapy

Answer:C

<16>Medullary thyroid carcinoma, all are true, Except:


<A>Is a tumor of the parafollicular C cells
<B>Produce calcitonin as the principle hormone
<C>10% of cases are sporadic
<D>Can occur as part of the MEN type II syndrome
<E>Total thyroidectomy is the surgical treatment of choice

Answer:C

<17>Thyroglossal duct cyst (TGDC): Other than the history and physical exam, which of the following
tests is considered an essential feature of the preoperative evaluation of a patient with a suspected this
condition?
<A>Cervical ultrasound
<B>Thyroid scan
<C>Serum T3 and T4 levels
<D>Needle aspiration
<E>None of the above

Answer:A
<18>Breast conservation surgery, all are associated with a lower rate of recurrence, Except:
<A>Low proportion of in-situ carcinoma
<B>Tumor size less than 4 cm
Dr. wessam alzaidat - General surgery
<C>Use of radiotherapy
<D>Younger age at presentation
<E>Use of adjuvant systemic treatment

Answer:D

<19>Tuberculous Mastitis, all are true, Except:


<A>It usually affects women in the reproductive age group
<B>AIDS represents a predisposing factor
<C>Demonstration of a caseating granuloma is diagnostic
<D>The treatment of choice is wide surgical resection with anti-tubercular drugs
<E>Polymerase chain reaction (PCR) on excised tissue has no diagnostic advantage

Answer:E

<20>All the following malignancies are associated with chromosomal deletions, Except:
<A>Retinoblastoma
<B>Squamous cell carcinoma of the lung
<C>Colorectal carcinoma
<D>Multiple endocrine neoplasia type 2-A
<E>Medullary carcinoma of thyroid

Answer:B

<21>Bone metastases, all are true, Except:


<A>Less than 5% of patients with malignant disease develop bone metastases
<B>10% of patients with bone metastases develop a pathological fracture
<C>Breast cancer is the commonest cause of bone metastases in women
<D>Radiological changes are seen late in the disease process
<E>Prostate cancer can cause osteosclerotic lesions

Answer:A

<22>Chemotherapy is least effective in:


<A>Breast cancer of no special type (NOS)
<B>Duke’s C colonic carcinoma
<C>Seminomas
<D>Thigh fibrosarcoma
<E>Pediatric rhabdomyosarcoma

Answer:D

<23>Malignant melanoma, all are true, Except:


<A>Lentigo maligna melanoma is a superficial lesion with a good prognosis
<B>Acral lentiginous melanomas most commonly occur on the dorsum of then hand
<C>The Breslow thickness is a good prognostic factor
<D>Sentinel lymph node biopsy is a useful means of assessing lymph node status
<E>Isolated limb perfusion with cytotoxic agents palliates recurrent limb disease

Answer:A

<24>Sliding hernia, all are true, Except:


<A>The anterior wall of the hernial sac is formed by a viscus
<B>It is due to slipping of parietal peritoneum on the underlying cellular tissue
<C>Usually large in size
<D>More on the left side
Dr. wessam alzaidat - General surgery
<E>Occasionally the large intestine is strangulated

Answer:A

<25>Peritoneum, all are true, Except:


<A>It has a surface area of 10m2
<B>Peritoneal fluid has a protein concentration of less than 3g\dl
<C>Normal peritoneal fluid WBC count is less than 3x 109/liter
<D>Peritoneal fluid has no fibrinogen clotting system
<E>Peritoneal fluid circulates via the diaphragmatic lymphatics

Answer:A SURFACE 2 M2 = SKIN SURFACE,,,Has fibrinolaytic activity when inj produce iflamm,
exudate.

<26>A 34-year-old morbidly obese diabetic woman underwent a gastric bypass about 12 hours ago.
The operation was technically difficult but finally went well. You are called because she now has a
temperature of 99.2º F, pulse of 134, and some pain in her incision and her back. She looks well; the
incision is clean; and her examination is otherwise negative. A bolus of 500 ml. of dextrose/lactated
Ringer's did not change her vital signs, except that her pulse rise to 140 without an increase in urine
output. Your next step should be:
<A>Another bolus of crystalloids
<B>Posteroanterior and lateral chest films
<C>Obtain white cell count, differential count, and electrolyte values
<D>Call the operating room and warn them that you need to re-explore for a leak
<E>Increase her pain medication

Answer:D

<27>Bleeding from upper gastrointestinal tract, all are true, Except:


<A>Acute gastrointestinal (GI) bleeding is a potentially life-threatening
abdominal emergency that remains a common cause of hospitalization
<B>The incidence of upper gastrointestinal bleeding (UGIB) is approximately
100 cases per 100,000 population per year
<C>Bleeding from the upper GI tract is approximately 4 times as common as
Bleeding from the lower GI tract
<D>Mortality rate from UGIB is 1% overall
<E>An aging patient population with an increasing prevalence of associated medical comorbidities has
kept the mortality figures largely unchanged for the past 30 years, despite technologic advances in
endoscopy and other minimally invasive procedures

Answer:D

<28>Chronic duodenal ulcer is associated with:


<A>Hyperthyroidism
<B>Hyperparathyroidism
<C>Insulinoma
<D>Phaeochromocytoma
<E>Cholelithiasis

Answer:B

<29>Gastric leiomyomas, all are true, Except:


<A>They are the most common type of gastric tumor of the stomach at autopsy
<B>The leiomyoblastoma cell type reflects malignant transformation of gastric leiomyomas.
<C>A conservative surgical approach is indicated for their resection since regional lymphadenectomy
has not been proved reliable even when they turn out to be malignant.
Dr. wessam alzaidat - General surgery
<D>Severe hemorrhage may occur from deep ulcerations overlying the intramural tumor.
<E>Gastric leiomyomas are not encapsulated, even though on section they appear to be well-
circumscribed.

Answer:B

<30>Gastric polyps, One of the following is True:


<A>Like their colonic counterparts, gastric epithelial polyps are common tumors.
<B>They are analogous to colorectal polyps in natural history.
<C>Endoscopy can uniformly predict the histology of a polyp based on location and appearance.
<D>In a given patient, multiple polyps are generally of multiple histologic types.
<E>Gastric adenomatous polyps greater than 2 cm. in diameter should be excised because of the risk
of malignant transformation.

Answer:E

<31>Gastrin-releasing peptide (GRP), all are true, Except:


<A>In species other than man and dog, GRP is commonly referred to as bombesin.
<B>GRP serves as a neurotransmitter.
<C>GRP inhibits pancreatic secretion when given intravenously.
<D>GRP stimulates gastric acid secretion when given intravenously.
<E>GRP is released in response to cholinergic stimulation of the parietal cells to stimulate release of
gastrin.

Answer:C

<32>The most effective therapy for morbid obesity, in terms of weight control, is:
<A>Intensive dieting with behavior modification.
<B>A multidrug protocol with fenfluramine, phenylpropanolamine, and mazindol.
<C>A gastric bypass with a 40-ml. pouch, a 10- to 20-cm. Roux-en-Y gastroenterostomy.
<D>A gastric bypass with a 15-ml. pouch, a 40- to 60-cm. Roux-en-Y gastroenterostomy.
<E>Daily exercise with strong emphasis on utilizing all four limbs.

Answer:D

<33>Vascular compression of the duodenum in adults, One of the following has been used successfully
in its treatment:
<A>Subtotal gastrectomy and Roux-en-Y gastrojejunostomy
<B>Total parenteral nutrition
<C>Division of the ligament of Treitz and duodenal mobilization
<D>Percutaneous endoscopic gastrostomy
<E>Duodenojejunostomy

Answer:E

<34>Zollinger-Ellison syndrome, treatment of choice is


<A>Total gastrectomy
<B>Ranitidine
<C>Partial gastrectomy
<D>Vagotomy with gastric drainage
<E+>Proton-pump inhibitors (PPIs)

Answer:E

<35>The optimal management of traumatic duodenal hematoma is:


<A>Angiography and embolization
Dr. wessam alzaidat - General surgery
<B>Laparotomy and evacuation
<C>Laparotomy and gastrojejunostomy
<D>Observation
<E>Aspiration under U/S

Answer:D

<36>Crohn’s disease, all are common complications, Except:


<A>Recurrent intestinal obstruction
<B>Intestinal stricture
<C>Perianal disease
<D>Entero-enteric fistula
<E>Free perforation in to the peritoneal cavity

Answer:E

<37>Intestinal bypass, all are true, Except:


<A>The operation produced weight loss similar to that of the gastric bypass.
<B>The operation produced severe metabolic disturbances, including hypocalcemia, increased bile
salts and glycine synthesis.
<C>Bacterial overgrowth in the bypassed segment led to liver failure.
<D>The operation demonstrated that an adult human could survive with 40 to 50 cm. of small
intestine.
<E>Is the treatment of choice for morbid obesity

Answer:E

<38>Malabsorption, simple tests include all the following, Except:


<A>Microscopic examination.
<B>D-xylose absorption.
<C>A 72-hour stool collection for fats.
<D>Small bowel x-ray series.
<E>Enteroscopy

Answer:E

<39>Mesenteric ischemia, all are true, Except:


<A>Characterized by severe pain out of proportion of the physical findings
<B>The presence of peritoneal irritation signs is a grave situation
<C>Non occlusive visceral ischemia has an excellent prognosis
<D>The most common site for acute arterial obstruction is the origin of the superior mesenteric artery
<E>Angiography is the standard investigation for acute arterial occlusion

Answer:C

<40>Perforation in enteric fever, the commonest site is:


<A>Jejunum
<B>Cecum
<C>Terminal ileum
<D>Ascending colon
<E>Stomach

Answer:C

<41>Short bowel syndrome, all are true, Except:


<A>Might be seen in Crohn’s disease
Dr. wessam alzaidat - General surgery
<B>Up to 70% of small bowel can be safely resected if the terminal ileum and ileocecal valve are left
intact
<C>Massive small bowel resection leads to gastric hypo-secretion and increased bowel PH
<D>Unabsorbed fats are irritating to colon, increasing diarrhea and steatorrhea
<E>Fluid and electrolyte problems are due to shortened transit time and diarrhea

Answer:C

<42>The lamina propria between the intestinal epithelium and the muscularis mucosae contains all
the followings, Except:
<A>Blood and lymph vessels.
<B>Nerve fibers.
<C>Enterochromaffin cells.
<D>Macrophages.
<E>Connective tissue.

Answer:C

<43>Carcinoid of the appendix, One of the followings is True:


<A>Carcinoids of appendix most often occur at the base
<B>Nearly 50% are multiple
<C>Appendiceal carcinoids are often malignant
<D>30% of appendiceal carcinoids present with the classic carcinoid syndrome
<E>If regional lymph nodes are enlarged right hemicolectomy is indicated

Answer:E

<44>Anorectal functional testing, all are true, Except:


<A>Anorectal manometry is often performed through open-tipped multilumen catheters perfused
with fluid.
<B>Anorectal manometry can not differentiate between segmental and global defects of the anal
sphincter in patients with incontinence.
<C>Electromyography can demonstrate persistent contraction of the pubis rectalis muscle during
defecation, which is diagnostic of paradoxical pelvic floor contraction.
<D>Measurement of sensory thresholds may reveal insensitivity in patients with chronic constipation.
<E>Catheters are connected to a transducer and register internal and external sphincter pressures and
presence of the anorectal inhibitory reflex.

Answer:B

<45>Colon physiology, all are true, Except:


<A>Colonic recycling of urea is accomplished by the splitting of urea by bacterial ureases.
<B>Fermentation by colonic bacteria may rescue malabsorbed carbohydrates.
<C>The preferred fuel of the colonic epithelium is glucose.
<D>The colonic epithelium utilizes n-butyrate for the absorption of sodium and water
<E>Insoluble fibers create bulk in the stool.

Answer:C

<46>Colorectal cancer, adjuvant chemotherapy: One of the following recommendations is True:


<A>Patients with Stage I or Dukes A and B1 disease should receive adjuvant treatment for 1 year with
levamisole combined with 5-FU.
<B>Patients with Stage III or Dukes C disease should receive adjuvant treatment for 1 year with
levamisole combined with 5-FU.
<C>There is no role for adjuvant therapy for colon cancer at any stage.
<D>Adjuvant chemotherapy is active in colon cancer only when combined with radiotherapy.
Dr. wessam alzaidat - General surgery
<E>5-Fluorouracil (5-FU) achieves an 80% to 90% response in patients with advanced disease.

Answer:B

<47>Diverticular disease, all are true, Except:


<A>It is more common in Western Europe than in Africa
<B>A low-fiber diet may predispose to development of diverticulosis
<C>It involves sigmoid colon in more than 90% of patients
<D>Sixty percent develop diverticulitis sometime during their lifetime
<E>It is the most common cause of massive lower gastrointestinal hemorrhage

Answer:D

<48>Familial adenomatous polyposis (FAP), One of the followings is True:


<A>Inherited in an autosomal dominant manner, this genetic defect is of variable penetrance, some
patients having only a few polyps whereas others develop thousands.
<B>The phenotypic expression of the disease depends mostly on the genotype
<C>Appropriate surgical therapy includes total abdominal colectomy with ileorectal anastomosis and
ileoanal pull-through with rectal mucosectomy
<D>Panproctocolectomy with ileostomy is not appropriate therapy for this disease
<E>Pharmacologic management of this disease may be appropriate in some instances

Answer:C

<49>Fistula in Ano, all are true, Except:


<A>Intersphincteric is the commonest type
<B>Usually preceded by a perianal abscess
<C>Fistulas secondary to other disease are more common than primary ones
<D>Best treated by unroofing of the fistula tract
<E>Surgery may end up with anal incontinence

Answer:C

<50>Perianal abscess, all are true, Except:


<A>Usually caused by mixed microbes
<B>Commonly starts as anal gland infection
<C>Recurrent and multiple could be caused by inflammatory bowel disease
<D>Antibiotic administration may treat abscess in its early stage
<E>Improperly drained abscess can lead to fistula formation

Answer:D

DR.WBZ
<51>Pouchitis can frequently complicate the ileal pouch/and anastomosis procedure, with regard to
this condition, One statement is True:
<A>It occurs with equal frequency in patients with familial polyposis and ulcerative colitis
<B>It is found more frequently in patients with capacious S-shaped pouches than in those with J-
pouches
<C>Most patients are treated successfully with oral metronidazole
<D>The responsible pathogen is usually Bacteroides
<E>Recurrent persistent pouchitis invariably necessitates pouch excision

Answer:C

<52>Rectal injuries, all are true, Except:


Dr. wessam alzaidat - General surgery
<A>The most common cause is iatrogenic
<B>A diverting colostomy is usually needed
<C>Repair of the defect should be done
<D>The role of the distal rectal washout is controversial
<E>Drainage of the retroperitoneal space is not mandatory

Answer:C

<53>Technique of anterior resection, all are true, Except:


<A>Mobilization of the splenic flexure is done on the middle colic artery
<B>Division of the inferior mesenteric artery at its origin is not an oncological need
<C>Division of the inferior mesenteric vein at the lower end of the pancreas is needed from an
oncological point of view
<D>The pelvis must be drained
<E>Circular staplers have the same anastomotic leak rate as hand sewn ones

Answer:C

<54>The presence of generalized gastrointestinal polyposis with alopecia, atrophy of finger nails,
cutaneous pigmentations, characterizes which syndrome:
<A>Cronkhite-Canada syndrome
<B>Gardner's syndrome
<C>Peutz-Jeghers syndrome
<D>Meigs syndrome
<E>Mallory-Weiss syndrome

Answer:A
C-C S DERMATOLGICAL TRIAD :1 ALOPECIA 2 HYPERPIGMENTATION 3 ATROPHY FO=INGER NAIL

<55>Ulcerative colitis (Toxic): Initial management should include all the followings, Except:
<A>Broad-spectrum antibiotics.
<B>Large intravenous doses of corticosteroids
<C>Intravenous fluid and electrolyte resuscitation.
<D>Opioid antidiarrheals
<E>Total parenteral nutrition to improve nutritional status

Answer:D

<56>Cecal diverticular: One of the following is True:


<A>Cecal diverticuli are usually multiple
<B>Preoperative diagnosis of cecal diverticulitis is possible is 80% of cases
<C>Simple diverticullectomy with closure of defect is the procedure of choice in localized cecal
diverticulitis
<D>Removal of the appendix is contraindicated if the intraoperative diagnosis is cecal diverticulitis
<E>Cecal diverticula are usually acquired mucosal herniation through muscularis propria of cecum

Answer:C

<57>A 45-year-old man is scheduled to undergo laparoscopic cholecystectomy for symptomatic


cholelithiasis. He has a long midline scar extending from the xiphoid process to the umbilicus due to
open splenectomy for traumatic splenic rupture 30 years ago. What are the alternatives for initial
trocar placement?
<A>Extend the vertical incision with careful peritoneal dissection
<B>Insert the initial trocar high in the epigastric region
<C>Insert the initial trocar in the right anterior axillary line!
<D>All of the above
Dr. wessam alzaidat - General surgery
<E>The position of the initial trocar need not be changed

Answer:D

<58>Cholangiocarcinoma (CCA): all are true, Except:


<A>A strong relationship exists between CCA and primary sclerosing cholangitis (PSC)
<B>Certain chemical exposures have been implicated in the development of CCA, primarily among
workers in the aircraft, rubber, and wood finishing industries
<C>CCA occasionally has developed years after administration of the radiopaque medium thorium
dioxide (ie, Thorotrast)
<D>Congenital diseases of the biliary tree, including choledochal cysts and Caroli disease, have been
associated with CCA
<E>Alpha1-antitrypsin deficiency is associated with gallbladder cancer but not CCA

Answer:E
<59>Choledochal cysts, all are true, Except:
<A>The morbidities associated with choledochal cysts are age dependent
<B>The most worrisome complication of choledochal cysts is cholangiocarcinoma
<C>Choledochal cysts are more prevalent in males
<D>Most patients with choledochal cysts have some clinical manifestation of the disease in childhood
<E>Todani Type I choledochal cysts are the most common and represent 80-90% of the lesions.

Answer:C

<60>Dye used for intravenous cholangiography is:


<A>Iapanoic acid
<B>Meglumine
<C>Na diatrizoate
<D>Urograffin
<E>Giemsa

Answer:B

<61>Hepatic amebiasis: One of the following is True:


<A>The infection is most commonly transmitted by person-to-person contact rather than by
contaminated food and water
<B>An increasing incidence is seen among homosexual men
<C>Active disease is present when trophozoites or cyst are found in the stool
<D>The abscesses tend to rupture intraperitoneally when found in the left lobe
<E>Diagnosis is usually made by aspirating protozoans from abscess

Answer:B

<62>Hydatid disease of liver: Casoni's test is positive in:


<A>35%
<B>50%
<C>65%
<D>95%
<E>100%

Answer:C

<62>Treatment of Carbon dioxide embolus should include all the followings, Except:
<A>Cardiopulmonary resuscitation if necessary
<B>Evacuate the pneumoperitoneum
<C>Place patient in head-down, left lateral decubitus position
Dr. wessam alzaidat - General surgery
<D>Place an emergent central venous line
<E>Stick a needle into the right chest

Answer:E

<63>Liver neoplasms (Benign): One of the following is True:


<A>Hepatic adenomas (HAs) are usually multiple and can reach a diameter of 30 cm
<B>Unlike patients with adenomas, patients with focal nodular hyperplasia (FNH) are usually
asymptomatic
<C>Hemorrhage, necrosis, and malignant change are more common in FNH than adenomas
<D>FNH derives its blood supply peripherally, where as adenomas derive blood supply centrally
<E>Adenomas blend with the surround normal hepatic tissue, whereas FNH appears distinct

Answer:B

<64>Yesterday, a 38-year-old woman underwent a laparoscopic cholecystectomy for cholelithiasis


and was discharged home 8 hours after surgery. She returns this morning complaining of worsening
abdominal pain. The oral narcotics that the patient was prescribed are ineffective in controlling the
pain. The patient's temperature is 38.3 C. Laboratory studies reveal an elevated white blood cell count.
Abdominal ultrasonography shows a large subhepatic fluid collection. The fluid is percutaneously
aspirated and reveals enteric contents.
What step should be taken next in the management of this patient? Please choose the single most
appropriate answer to the question:
<A>Immediate laparptomy
<B>Observation in the hospital until pain improves
<C>Intravenous antibiotics and close observation
<D>ERCP
<E> PTC

Answer:A

<65>Acute pancreatitis, all are true, Except:


<A>The initiating event may be anything that injures the acinar cell and impairs the secretion of
zymogen granules, such as alcohol use, gallstones, and certain drugs.
<B>Can develop when ductal cell injury leads to delayed or absent enzymatic secretion, such as with
the CFTR gene mutation
<C>Lysosomal and zymogen granule compartments fuse, enabling activation of trypsinogen to trypsin
<D>Intracellular somatostatin triggers the entire zymogen activation cascade
<E> Secretory vesicles are extruded across the basolateral membrane into the interstitium, where
molecular fragments act as chemoattractants for inflammatory cells
Answer:D
<66>Cancer of the exocrine pancreas, all are true, Except:
<A>It is usually an adenocarcinoma of duct cell origin
<B>Most common site is the head
<C>The majority are multicentric
<D>It includes mucinous and adenosquamous varieties
<E>CEA is usually elevated but not always

Answer:C

<67>Following a motor vehicle accident, a truck driver complains of severe abdominal pain. The
amylase level is markedly increased to 800 u. pancreatic trauma is suspected. Pancreatic trauma:
<A>Is usually an isolated single-organ injury
<B>Often requires a portocaval shunt operation
<C>May easily be overlooked at operation
<D>Is proven by the elevated amylase level
Dr. wessam alzaidat - General surgery
<E>Blunt trauma is about three times more common than penetrating trauma and is associated with a
higher mortality rate.

Answer:C

<68>Mucin secreting tumors of the pancreas (Intraductal papillary mucinous tumors (IPMT) of the
pancreas): all are true, Except:
<A>It is more common in males
<B>Prolonged symptoms of pancreatitis should prompt a work up of this tumor
<C>The diagnosis should be suspected when there is mucin extravasation or plug on ERCP
<D+>Conservative management (no surgery) is enough since the tumor is benign and indolent
<E>The neoplasms were typically more than 5 cm in diameter

Answer:D

<69>Normal functions of the adult spleen include al of the following, Except:


<A>Hematopoiesis
<B>Erythrocyte processing and repair
<C>Production of opsonins
<D>Destruction of aged and damaged erythrocytes
<E>Clearance of circulating bacteria

Answer:A

<70>Overwhelming post-splenectomy sepsis:


<A>Commonly occurs after splenectomy for trauma
<B>Does not occur if accessory spleens are present
<C>Can be fatal within hours of onset
<D>Is most common in elderly patients
<E>Most fatal cases occur 10 to 15 years after splenectomy

Answer:C

<71>Hyperaldosteronism (Conn's syndrome), all are true, Except:


<A>The commonest cause is hyperplasia of the zona glomerulosa of the adrenal cortex
<B>Its incidence is more in female
<C>It constitutes 1% of patients investigated for hypertension
<D>There is increase in urinary potassium
<E>There is increase in blood sodium

Answer:A

<72>Pheochromocytoma, all are true, Except:


<A>Urinary catecholamine estimation is 90% sensitive for its diagnosis
<B>Iodocholesterol isotope scan is used for its localization
<C>It can be associated with multiple endocrine neoplasia type 2
<D>10% of cases are caused by extra-adrenal source
<E>Adrenalectomy is the treatment of choice

Answer:B

<73>Adult respiratory distress syndrome (ARDS): all are true, Except:


<A>Its name is misleading, because children, as well as adults, may be affected
<B>The basic fault is a breakdown of the endothelial-alveolar barrier (alveolo-
capillary junction)
<C>The lungs as a whole become very "stiff," and it becomes much harder for
Dr. wessam alzaidat - General surgery
the patient to breathe
<D>Can be caused by pancreatitis
<E>The pulmonary artery occlusion pressure is more than 20 mm Hg

Answer:E

<74>The most common tumor in the superior mediastinum is:


<A>Pericardial cyst
<B>Ectopic parathyroid gland
<C>Thymoma
<D>Aortic arch aneurysm
<E>Retrosternal extension of goiter

Answer:E

<75>Thoracic outlet syndrome, all are true, Except:


<A>Is more common in men than women
<B>Neurological symptoms are more common than arterial or venous symptoms
<C>Symptoms are often worsened by carrying weights or lifting the arms above head height
<D>Diagnostic tests include the Roo's and Adson's maneuver
<E>Onset of symptoms is precipitated by trauma in about one-third of patients

Answer:A

<76>Ischemic heart disease: All the followings are known risk factors, Except:
<A>Hyperthyroidism
<B>Family history of coronary artery disease
<C>Smoking tobacco
<D>Hyperlipidemia
<E>Diabetes mellitus

Answer:A

<77>Patent ductus arteriosus (PDA): In a premature infant with hyaline membrane disease and
inability to be weaned from mechanical ventilation, which of the following would suggest
hemodynamically significant (PDA)?
<A>Systolic murmur
<B>Hyperactive precordium with bounding peripheral pulses
<C>Jaundice
<D>Diminished femoral pulses
<E>Diastolic murmur

Answer:B

<78>Penetrating cardiac injury: The most useful incision in the operating room is:
<A>Left anterior thoracotomy
<B>Right anterior thoracotomy
<C>Bilateral anterior thoracotomy
<D>Median sternotomy
<E>Subxyphoid

Answer:D

<79>Biliary atresia, the most accurate test for diagnose is:


<A>HIDA scan
<B>Abdominal ultrasound
Dr. wessam alzaidat - General surgery
<C>Liver biopsy
<D>CT scan
<E>PTC (percutaneous transhepatic cholangiogram)

Answer:C

<80>Esophageal atresia (Isolated) without a tracheoesophageal fistula: the most appropriate initial
management is:
<A>Emergency exploration and anastomosis
<B>Cervical esophagostomy and gastrostomy
<C>Gastrostomy only
<D>Immediate esophageal replacement
<E>Feeding jejunostomy

Answer:B

<81>Intussusception, all are true, Except:


<A>Is most common in children from 6 to 12 years
<B>Presents with colicky abdominal pain, rectal bleeding and an abdominal mass
<C>10% present with diarrhea and vomiting suggestive of gastroenteritis
<D>If no shock or peritonitis, hydrostatic reduction can be attempted
<E>A Meckel's diverticulum can induce an intussusception

Answer:A

<82>Massive upper gastrointestinal bleeding in children, the most common cause is:
<A>Peptic ulcer disease
<B>Esophago-gastric varices
<C>Gastro-esophageal reflux
<D>Congenital arterio-venous malformation
<E>Esophageal atresia

Answer:B DR.WBZ
<83>Burn: Full Thickness Burn (FTB): One of the following is True:
<A>Blister formation is characteristic sign
<B>Sensation usually is intact
<C>Capillary refill time is less than 3 seconds
<D>Bleeding on pin prick
<E>Destruction of both layers of epidermis and dermis

Answer:E

<84>Cleft lip abnormality is caused by:


<A>Failure of fusion of palatal shelves
<B>Failure of fusion of medial nasal process with frontonasal process
<C>Failure of fusion of medial nasal process with the maxillary process
<D>Failure of fusion between both medial and lateral nasal processes
<E>Failure of fusion maxillary process with the mandibular process

Answer:C

<85>Hemangiomas: Which of the following would be helpful in distinguishing a deep hemangioma


from AV-malformation:
<A>Computerized tomography (CT scan) with contrast injection
<B>Clinical examination X-ray investigation
Dr. wessam alzaidat - General surgery
<C>Arteriography
<D>Ultra sound scan (USS)
<E>X-ray investigation

Answer:A

<86>Number of muscles inserted on the index finger:


<A>Three
<B>Four
<C>Five
<D>Six
<E>Seven

Answer:E

<87>A disease identified by the presence of Donovan bodies and caused by an organism similar to
Klebsiella pneumoniae is:
<A>Chancroid
<B>Granuloma inguinale
<C>Lymphogranuloma venereum
<D>Malakoplakia
<E>Bacterial vaginosis

Answer:B

<88>A 50 year old male patient presented to the emergency room with acute, ischemic leg pain found
to have a history of painful calf on walking for the last 3 years. This patient could have one of the
following, Except:
<A>Thromboangiitis obliterans (Buerger's disease)
<B>Acute arterial thrombosis
<C>Acute arterio-arterial emboli
<D>Acute embolus originated from the heart
<E>Acute femoral artery thrombosis following diagnostic arteriogram

Answer:A

<89>A previously healthy 16-year-old high school student begins to notice pain in his right calf during
football practice after running two laps around the track. The pain is relieved immediately when he
stops running. The most likely diagnosis is:
<A>Arterial embolus
<B>Muscle cramps
<C>Popliteal entrapment syndrome
<D>Popliteal aneurysm
<E>Deep venous thrombosis

Answer:C

<90>Axillary-subclavian vein thrombosis is associated with all the following, Except:


<A>Pancoast syndrome
<B>Parenteral nutrition
<C>Cervical rib
<D>Cervical discopathy
<E>Congestive cardiac failure

Answer:D

Dr. wessam alzaidat - General surgery


<91>Arteriovenous fistula (congenital of the lower limb): You would suspect it if all the following are
present, Except:
<A>Unusual varicosities were present in an enlarged limb
<B>Painful ulcer present with normal foot pulses
<C>Occlusion of the femoral artery leads to tachycardia
<D>The pulse pressure is great
<E>Machinery murmur is heard

Answer:C

<92>Chronic venous disease: One of the following is True:


<A>Bicuspid valves in perforating veins normally direct flow from the deep to the superficial venous
system
<B>The normal decrease in distal venous pressure with exercise exceeds 60%
<C>Following exercise, the distal venous pressure usually returns to the resting level within 10
seconds
<D>Brownish skin discoloration of chronic venous disease is due to excess melanocyte activity
<E>If the feet and toes are involved, venous insufficiency is more likely than lymphedema

Answer:B

<93>Leriche's syndrome: Symptoms or sings of atherosclerotic occlusive disease of the bifurcation of


the abdominal aorta (Leriche's syndrome) include One of the followings:
<A>Claudication of the buttock and thigh
<B>Causalgia of the lower leg
<C>Retrograde ejaculation
<D>Gangrene of the feet
<E>Dependent rubor of the feet

Answer:A

<94>Renal fibromuscular dysplasia: The single most characteristic finding is:


<A>Complete occlusion of the renal artery
<B>Renal artery aneurysms
<C>Area of narrowing alternating with area of dilatation in the main renal arteries
<D>Involvement of the renal artery origin
<E>Arterio-venous malformation in the kidney

Answer:C

<95>Vascular grafts: all are true, Except:


<A>PTFE grafts are more porous than Dacron
<B>Vein grafts have better long-term patency
<C>Graft failure less than a month after surgery is usually due to a surgical error
<D>Graft failure within the first year is often due to neo-intimal hyperplasia
<E>Graft failure beyond one year is due to progression of the underlying disease

Answer:A

<96>All are indications for surgical reconstruction of the esophagus, Except:


<A>Continuing requirement for frequent dilation of an extensive esophageal stricture for a minimum
of 2 years
<B>Failure or refusal of the patient to comply with a treatment regimen of regular dilation
<C>Development of a fistula between the esophagus and tracheobronchial tree
<D>Iatrogenic perforation of the esophagus during attempted dilation
<E>Very tight stricture not amenable for dilatation
Dr. wessam alzaidat - General surgery
Answer:D

<97>Esophageal cancer: One of the followings about its surgical treatment is True:
<A>The finding of severe dysplasia in association with Barrett's mucosa is an indication for an
antireflux operation to prevent subsequent development of carcinoma
<B>The number of lymph nodes resected with the tumor determines survival
<C>The morbidity and mortality rates for cervical esophagogastric anastomotic leak are substantially
less than those associated with intrathoracic esophagogastric anastomotic leak
<D>The leading complications of intrathoracic esophagogastric anastomosis are bleeding and wound
infection
<E>Transhiatal esophagectomy without thoracotomy achieves better long-term survival than
transthoracic esophagectomy

Answer:C

<98>oesophageal reflux disease (GERD): which of the following tests is the most sensitive for its
detection?
<A>Barium swallow
<B>Manometry
<C>24 hours PH-monitoring
<D>Acid perfusion (Bernstein) test
<E>Standard antireflux test

Answer:C

<99>The best management for a 48-hour-old distal esophageal perforation Is:


<A>Antibiotics and drainage
<B>Division of the esophagus and exclusion of the perforation
<C>Primary repair with buttressing
<D>Resection with cervical esophagostomy, gastrostomy, and jejunostomy
<E>T-tube fistula and drainage Answer:C

Dr. wessam alzaidat - General surgery


Residents exam Mcqs
1-tube thoracostomy is still associated with a 25% complication rate.
Perhaps it is this high complication rate, that reinforces the concept that chest
tube placement should be avoided if less than 300 mL of blood (small
hemothorax) is in the pleural space.

Periprocedural antibiotics for placement of tube thoracostomy do not reduce the


incidence of empyema or pneumonia and are not routinely indicated when a
chest tube is placed for traumatic hemothorax.

2-Andexanet alpha is used to reverse the


direct oral anticoagulant (DOAC) rivaroxaban.

3-Congenital adrenal hyperplasia (CAH) is most commonly


caused by a deficiency of which of the following enzymes?

21-Hydroxylase

4.The most common endogenous cause of Cushing syndrome is:


A. Adrenocortical carcinoma (ACC)
B. ACTH-hypersecreting pituitary adenoma
C. Cortisol-hypersecreting adrenal adenoma
D. Ectopic ACTH-producing tumor
E. Adrenal hyperplasia

ANSWER:B
COMMENTS: Overall, the most common cause of Cushing syndrome
is the exogenous use of corticosteroids. Among the endogenous
types of Cushing syndrome, an ACTH-hypersecreting
pituitary adenoma is the most common (70%). Adrenal adenomas
and ectopic ACTH-producing tumors each account for 10% of the
endogenous causes of Cushing syndrome.

adrenal mass Size is the most important criteria to help diagnose malignancy.
predictor of survival.
Adequacy of resection is the most important (adrenal mass)

 Mitotane has
adrenolytic activity and can be used for the treatment of metastatic
or unresectable disease.(pheoch)

Dr. wessam alzaidat - General surgery


5.The most common indication for revision surgery after
gastric bypass is:

Failure to lose weight

Patients may experience worsening of their GERD symptoms after sleeve


gastrectomy

6.Which of the following complications is increased in sleeve


gastrectomy compared with Roux-en-Y gastric bypass?

-Late anastomotic leak

sleeve gastrectomy can be converted into a Roux-en-Y gastric bypass if greater


weight loss is desired.

7.Advantages of a laparoscopic Roux-en-Y gastric bypass over


the open approach include all of the following except:
A. Decreased postoperative pneumonia rate
B. Decreased deep venous thrombosis (DVT) and PE rate
C. Decreased wound infection rate
D. Decreased incisional hernia rate
E. Decreased internal hernia rate

regarding gastric bypass The gastric pouch is completely divided from the distal
part of the stomach.

8.Anemia in the burn patient is frequently secondary to:

- Increased erythrocyte fragility

-Epithelial cell stromal GIST is the most common cell type arising in the
Stomach

- cellular spindle type is the next most common.

-The glomus tumor type is seen only in the stomach.

9.Type III collagen is the most common type of collagen present in granulation
tissue

Dr. wessam alzaidat - General surgery


10.During thyroid surgery, the following avascular space has a crucial role in
identification and control of superior thyroid vessels:

Cricothyroid space.

11.Which of the following is true about cardiac tamponade in trauma patient?

Less than 100 cc of blood in the pericardium can cause tamponade.

12.A 65-year-old male with previous hepatosteatosis develop an HCC in the Rt


hemi-liver. A Rt hemi-hepatectomy is the only potential surgical option. His
surgeon plan to resect 65 % of the liver. what is the best strategy?

Portal vein embolization followed by resection.

13.All the following mediastinal masses are usually found in the anterior
mediastinum except.

Ganglioneuroma

14.The most important step in treating a pulsion upper esophageal diverticulum


is:

Division of the cricopharyngeous muscles.

15.Sorafenib is an approved targeted therapy in the treatment of?

Hepatocellular carcinoma.

16.a 10-year-old boy is brought to the emergency room after a cycling accident
where he thrown against the handlebars. CT scan is performed and demonstrate
a crush injury to the distal pancreas with disruption of the pancreatic duct. how
should this be managed?

Laparotomy and distal pancreatectomy.

17.In large bowel cancer one of the following is false:

Dr. wessam alzaidat - General surgery


If at operation, hepatic metastases are found, biopsy should be done.

18.A 57-year-old lady has a 1.5 cm speculated, centrally dense non palpable
mass, radiological guided true-cut biopsy showed benign parenchyma. the most
appropriate recommendation for her is:

Wire –localized excisional biopsy.

19.Female underwent sleeve gastrectomy 1 year go, presented with fatigue,


SOB on exertion, with palpitation. The most likely cause of her presentation is:

Iron deficiency anemia.

20.Arterial embolization may be used in treatment except:

Gastroesophageal varices.

21.Which is correct?

Medullary breast cancer has better prognosis than the other IDC.

22.In FAP one of the following statements is incorrect:


A. More than 80% occur with positive familial history.
B. The majority will become malignant.

23.indications for hyperbaric oxygen therapy in wound and ulcer management


except:
DM ulcer Wigner 3.

24.Cyclosporine side effect ,except :


Hypokalemia

25.Umbilical hernia in children except :

Supraumbilical hernias often close spontaneously

26.m.c malignancy in children age less than 2 years :

Dr. wessam alzaidat - General surgery


Neuroblastoma

27.Saline suppression test:


Conn disease

28.A case of rigid abdomen and increased End tidal co2 on induction (
malignant hyperthermia) which drug help in this case ?

Dantrolene

29.about parathyroid , one is incorrect :

superior parathyroid gland are less constant

30.tumor marker associated with papillary thyroid cancer:


BRAF

31.P53 gene mutation found in all except:


wilm’s tumor
neuroblastoma
breast cancer
soft tissue sarcoma
one more option

32.a case of rectal tumor , 4 cm from the anal verge with internal sphincter
involvement , management ? APR

33.about Gallbladder CA , one is true :


G.B.S is the most important risk factor

34.about cholangiocarcinoma , one is incorrect :


klatskin tumor usually needs hepatic resection

35.rectal prolapse in children :


constipation should be treated

36.post operative patient has atelectatsis (can’t remember the exact scenario )
but the question was about which method should be used to avoid endotracheal
intubation :
nonrebreather mask
tent mask

Dr. wessam alzaidat - General surgery


venturi mask

37.Chest trauma with loss of full thickness part of the chest wall , management :
occlusive dressing

38.About alginate for wound all true except :

39.Odd ratio:

40.Most common type of kidney stone in post resection in crhon pt


Ca oxalate

41.About lymphangitis all true except :


Most common organism is pseudomonas

42.About AAA one is wrong :


Endovascular repair has lower mortality rate than open repair

43.A case of isolated gastric varices due to pancreatitis , management :


splenectomy

Dr. wessam alzaidat - General surgery


RANDOM SURGERY MCQS (UNIVERITIES
HOPITALS)

1. Regarding contrast study for intestinal obstruction: 6. Rightward shift of oxyhemoglobin dissociationcurve
(a) Gastrografin is preferred to barium for studying occurs with:
distal small bowel (a) hypothermia
(b) Gastrografin has no therapeutic potential (b) acidosis
(c) Gastrografin is less hazardous than barium if (c) decrease in 2,3-diphosphoglycerate
aspiration occurs (d) hypocapnia
(d) Gastrografin can cause serious fluid shift (e) methemoglobinemia
(e) barium can convert partial small bowel obstruc-
tion into complete obstruction 7. The most common site of gastrointestinal lym-
phoma is:
2. An absolute contraindication to breast-conserving (a) small intestine
surgery for breast cancer is: (b) stomach
(a) large tumor (c) colon
(b) tumor of high grade (d) duodenum
(c) early pregnancy (e) appendix
(d) retroareolar tumor
(e) clinical axillary nodes 8. Meckel’s diverticulum:
(a) is a false diverticulum
3. The most common indication for surgery in chronic
(b) is asymptomatic in most cases
pancreatitis is:
(c) commonly presents as gastrointestinal bleeding
(a) jaundice
in adults
(b) pain
(d) commonly presents with intestinal obstruction
(c) pseudocyst
in children
(d) gastric outlet obstruction
(e) is found in approximately 5% to 10% of people
(e) endocrine deficiency

4. The most common cause of spontaneous intestinal 9. Biliary-enteric fistula most commonly connects:
fistula is: (a) gallbladder and ileum
(a) radiation injury (b) gallbladder and duodenum
(b) malignancy (c) common bile duct and jejunum
(c) Crohn’s disease (d) gallbladder and jejunum
(d) ulcerative colitis (e) common bile duct and ileum
(e) diverticular disease
10. Spontaneous closure is least likely in fistulae origi-
5. Gastrointestinal stromal tumors (GIST): nating from:
(a) occur most commonly in the duodenum (a) colon
(b) are almost always malignant (b) esophagus
(c) can be treated adequately with enucleation (c) pancreas
(d) are often radioresistant (d) stomach
(e) spread mainly via the lymphatics (e) small intestine

Dr. wessam alzaidat - General surgery


406 Part XVI. Surgery Review Questions and Answers
11. Gastrointestinal diverticula do not occur in: 18. Malignant small bowel neoplasms most commonly
(a) cecum present with:
(b) duodenum (a) weight loss
(c) rectum (b) abdominal pain
(d) jejunum (c) gastrointestinal bleeding
(e) ileum (d) jaundice
(e) intestinal perforation
12. The hepatic caudate lobe: 19. Small bowel obstruction in an elderly female
(a) drains directly into the inferior vena cava without external hernia or previous surgery is most
(b) represents segment IV likely caused by:
(c) is supplied by the left portal vein only (a) small bowel neoplasm
(d) is supplied by the right portal vein only (b) volvulus
(e) lies to the right of the inferior vena cava (c) gallstone ileus
(d) abdominal abscess
13. von Willebrand’s disease: (e) obturator hernia
(a) is an autosomal dominant disorder
(b) results in prolonged prothrombin time 20. During cell cycle, DNA replication occurs in:
(c) is associated with normal bleeding time (a) G1 phase
(d) is due to decreased hepatic synthesis of von (b) G2 phase
Willebrand’s factor (c) S phase
(e) is typically associated with joint bleeding (d) M phase

21. Li-Fraumeni syndrome shows increased incidenceof:


14. Which of the following is consistent with syndrome (a) colon cancer
of inappropriate antidiuretic hormone (SIADH)? (b) ovarian cancer
(a) hypovolemia (c) lung cancer
(b) increased urine sodium (d) breast cancer
(c) hypernatremia (e) pancreatic cancer
(d) plasma hyperosmolarity
(e) excessive diuresis 22. The best operative approach to choledochal cyst is:
(a) cystoduodenostomy
(b) cystojejunostomy
15. Normal anion gap acidosis is associated with:
(c) Roux-en-Y cystojejunostomy
(a) ketoacidosis
(d) Cyst excision and hepaticojejunostomy
(b) lactic acidosis
(c) salicylate poisoning 23. Leiomyoma of the esophagus:
(d) severe diarrhea (a) commonly presents with dysphagia
(e) uremic acidosis (b) is more common in females
(c) is usually multiple
16. Benign small bowel tumors most commonly present (d) is usually diagnosed with endoscopic biopsy
as: (e) is usually located in the lower one-third of the
(a) small bowel obstruction esophagus
(b) gastrointestinal bleeding
(c) weight loss 24. Hemangioma of the liver:
(d) incidental finding on laparotomy (a) is the most common benign hepatic tumor
(e) intestinal perforation (b) is diagnosed with percutaneous needle biopsy
(c) is associated with -fetoprotein level
(d) should be resected as soon as diagnosed
17. The diagnostic test of choice for suspected acute
sigmoid diverticulitis is: 25. Phosphorus:
(a) barium enema (a) is a major extracellular anion
(b) Gastrografin enema (b) is passively absorbed from the gastrointestinal
(c) computed tomography scan of the abdomen tract
and pelvis (c) deficiency may result in insulin resistance
(d) abdominal ultrasound (d) deficiency is rare in hospitalized patients
(e) colonoscopy
174. Surgery Review Questions 407
26. Hypermagnesemia is a complication of: 33. Intussusception in adults:
(a) extensive burns (a) is idiopathic in most cases
(b) acute pancreatitis (b) usually requires resection
(c) oliguric renal failure (c) is often successfully treated with hydrostatic
(d) resection of the terminal ileum reduction
(e) diuretic therapy (d) is seldom recurrent

27. The optimal management of traumatic duodenal 34. Optimal treatment for an ileosigmoid fistula in
hematoma is: Crohn’s disease is:
(a) angiography and embolization (a) closure of the fistula
(b) laparotomy and evacuation (b) proximal ileostomy
(c) laparotomy and gastrojejunostomy (c) proximal ileostomy and closure of fistula
(d) observation (d) ileocecectomy and sigmoid colectomy
(e) ileocecectomy and closure of the sigmoid
28. Hairy cell leukemia: defect
(a) can be cured with splenectomy
(b) is an aggressive form of leukemia 35. Obturator hernia:
(c) death is usually related to infectious com- (a) is most common in older women with
plications cachexia
(d) is a T-cell leukemia (b) is associated with pain in the sciatic nerve
distribution
29. Overwhelming postsplenectomy sepsis: (c) is repaired through a transverse incision in the
(a) commonly occurs after splenectomy for trauma upper medial thigh
(b) does not occur if accessory spleens are present (d) is rarely strangulated
(c) can be fatal within hours of onset
36. Gastrointestinal bleeding in Mallory-Weiss
(d) is most common in elderly patients
syndrome:
(e) most fatal cases occur 10 to 15 years after
(a) is occult in the vast majority of cases
splenectomy
(b) can be controlled with balloon tamponade
(c) stops with nonoperative management in most
30. The development of thrombocytopenia and arterial
cases
thrombosis with heparin requires:
(d) is caused by mucosal tear in the lower one-third
(a) continuation of heparin and platelet trans-
of the esophagus
fusion
(b) continuation of heparin and thrombolysis 37. The risk of overwhelming postsplenectomy sepsis is
(c) doubling the heparin dosage highest for patients requiring splenectomy for:
(d) changing the route of heparin administration (a) thalassemia
(e) discontinuation of heparin (b) trauma
(c) immune thrombocytopenic purpura
31. A trauma patient has a Glasgow Coma Scale score (d) hereditary spherocytosis
of 13, blood pressure 80/40; widened mediastinum (e) acquired hemolytic anemia
on chest x-ray, and bloody peritoneal tap. The next
step in management is: 38. After mastectomy, winging of the scapula results
(a) obtain head computed tomography scan from injury to:
(b) perform thoracotomy (a) the medial pectoral nerve
(c) perform arch angiography (b) the lateral pectoral nerve
(d) monitor intracranial pressure (c) the long thoracic nerve
(e) perform laparotomy (d) the thoracodorsal nerve
(e) the intercostal-brachial nerve
32. Insulinomas:
(a) are often multiple 39. von Willebrand’s disease:
(b) are mostly benign (a) is the most common congenital bleeding
(c) are a common component of multiple endo- disorder
crine neoplasia type 1 (MEN 1) (b) commonly results in hemarthrosis
(d) are commonly located in the head of the (c) affects males only
pancreas (d) results in prolonged prothrombin time

Dr. wessam alzaidat - General surgery


408 Part XVI. Surgery Review Questions and Answers
40. Disc lesion between L4 and L5 will lead to: 47. The most common cause of hypercalcemic crisis is:
(a) reduced knee jerk (a) sarcoidosis
(b) reduced ankle jerk (b) primary hyperparathyroidism
(c) weakness of foot dorsiflexion (c) secondary hyperparathyroidism
(d) reduced sensation on the small toe (d) malignancy
(e) renal failure
41. Kidney transplant recipients are at increased risk
for:
(a) epidermoid skin cancer 48. A sudden onset of glucose intolerance in patients
(b) lung cancer receiving total parenteral nutrition often indicates:
(c) colon cancer (a) diabetes mellitus
(d) breast cancer (b) sepsis
(e) uterine cancer (c) hypophosphatemia
(d) adrenal insufficiency
42. The optimum management of medullary thyroid (e) zinc insufficiency
carcinoma in multiple endocrine neoplasia type 2
(MEN 2) is:
(a) thyroid lobectomy 49. Pectus excavatum:
(b) thyroid lobectomy and cervical lymphadenec- (a) is usually associated with respiratory dysfunc-
tomy tion
(c) radioactive iodine (b) is usually associated with dysphagia
(d) radiotherapy (c) is often associated with cardiac dysfunction
(e) total thyroidectomy  radical neck dissection (d) cosmesis is usually the indication for surgery
43. Optimum management for a 2-cm mass in the head of
the pancreas with hypoglycemia and high insulin 50. Sacrococcygeal teratoma:
levels is: (a) is usually malignant
(a) total pancreatectomy (b) is more common in males
(b) Whipple resection (c) requires complete excision of the coccyx
(c) local excision (d) diagnosis is ruled out if calcification is absent on
(d) streptozotocin administration radiography
44. Which of the following is not a risk factor for wound
infection? 51. Dry, scaly, pruritic rash on the trunk and extremities
(a) prolonged operative time of a patient receiving total parenteral nutrition is
(b) prolonged preoperative hospitalization caused by:
(c) shaving the skin the night before surgery (a) zinc deficiency
(d) patient’s having upper respiratory tract infec- (b) vitamin A deficiency
tion (c) vitamin C deficiency
(e) surgeon’s hand scrub for 5 instead of 10 minutes (d) free fatty acid deficiency
45. On the second day after abdominal aortic aneurysm
repair, the patient passes grossly bloody stool. The 52. The main cause of postoperative death in children
next step is: with chronic diaphragmatic hernia is:
(a) immediate exploratory laparotomy (a) increased intra-abdominal pressure
(b) sigmoidoscopy (b) persistent lung collapse
(c) computed tomography scan of the abdomen (c) patent ductus arteriosus
with intravenous contrast (d) abnormal pulmonary microvasculature
(d) barium enema
(e) aortogram
53. A neonate has bilious vomiting and a double-
46. Spontaneous pneumothorax: bubble sign on plain x-ray. The most appropriate
(a) is more common in young females operation is:
(b) is typically postexertional (a) division of annular pancreas
(c) is recurrent in at least 30% of cases (b) gastroenterostomy
(d) often requires thoracotomy in the first episode (c) duodenoduodenostomy
(e) is often associated with severe persistent pain (d) duodenal resection
174. Surgery Review Questions 409
54. The most common source of bacteria in wound 62. Pulmonary hilar adenopathy with noncaseating
infection after groin hernia repair is: granuloma is consistent with:
(a) the patient’s skin (a) Hodgkin’s disease
(b) the patient’s nasopharynx (b) lymphoma
(c) operating room air (c) sarcoid
(d) surgical instruments (d) tuberculosis
(e) operating room staff (e) metastatic carcinoma

55. Enterocytes’ energy requirements are provided by: 63. A solitary lung nodule with a popcorn pattern of
(a) arginine calcification is most likely:
(b) alanine (a) a primary lung cancer
(c) glutamine (b) a metastatic lesion
(d) glycine (c) an old tuberculosis lesion
(d) histoplasmosis
56. In critical illness, immune function can be enhanced (e) a hamartoma
by:
(a) arginine 64. The most common presentation of gastric lym-
(b) glutamine phoma is:
(c) alanine (a) abdominal pain
(d) glycine (b) weight loss
(c) upper gastrointestinal bleeding
57. Intra-aortic balloon pump: (d) gastric perforation
(a) increases pulmonary wedge pressure (e) nausea and vomiting
(b) increases afterload
(c) increases diastolic pressure 65. The protective immune function of immunoglobu-
(d) increases duration of systole lin A is mediated through:
(e) decreases duration of diastole (a) inhibition of bacterial adherence to epithelial
cells
58. Popliteal artery aneurysms: (b) activation of complement
(a) are less common than femoral aneurysm (c) opsonization
(b) are often bilateral (d) direct destruction of microorganisms
(c) are more common in females
(d) seldom result in limb ischemia 66. The most potent stimulus for antidiuretic hormone
secretion is:
59. The most common cause of pneumaturia is: (a) hypovolemia
(a) Crohn’s disease (b) hyponatremia
(b) ulcerative colitis (c) hyperkalemia
(c) malignancy (d) raised serum osmolarity
(d) radiation enteritis
(e) diverticular disease 67. Which of the following distinguishes adrenal insuf-
ficiency from sepsis?
60. The most common benign hepatic lesion is: (a) hypotension
(a) hemangioma (b) fever
(b) adenoma (c) tachycardia
(c) focal nodular hyperplasia (d) altered mental status
(d) hamartoma (e) hypoglycemia

61. The most common etiologic factor for hepatocellu- 68. For penetrating chest injury, thoracotomy is indi-
lar carcinoma worldwide is: cated if:
(a) hepatitis C virus (a) initial chest tube output is 500 cc
(b) hepatitis B virus (b) initial chest tube output is 1,000 cc
(c) alcoholic cirrhosis (c) initial chest tube output is 1,500 cc
(d) aflatoxin ingestion (d) initial chest tube output is 2,000 cc
(e) schistosomiasis (e) persistent chest tube output of 100 cc/hr

Dr. wessam alzaidat - General surgery


410 Part XVI. Surgery Review Questions and Answers
69. Which cell type is essential for wound healing? 76. Post-transplant lymphoproliferative disorders are
(a) neutrophil related to:
(b) macrophage (a) cytomegalovirus
(c) fibroblast (b) Epstein-Barr virus
(d) lymphocyte (c) human immunodeficiency virus (HIV)
(e) endothelial (d) herpesvirus
(e) hepatitis B virus
70. The treatment for osteosarcoma of the distal femur
is: 77. Which of the following requires surgical drainage?
(a) above-knee amputation (a) amebic liver abscess
(b) chemotherapy followed by above-knee ampu- (b) peridiverticular abscess
tation (c) appendiceal abscess
(c) chemoradiation (d) pancreatic abscess
(d) chemotherapy alone (e) subphrenic abscess
(e) chemotherapy and limb-sparing surgery
78. The main complication of topical silver nitrate is:
(a) metabolic acidosis
71. Abnormal bleeding with normal prothrombin time (b) metabolic alkalosis
occurs with: (c) hyperkalemia
(a) heparin overdose (d) hypocalcemia
(b) cirrhosis (e) hyponatremia
(c) hemophilia
(d) von Willebrand’s disease
79. The adverse effects of steroids on wound healing
can be reversed with:
72. The most significant prognostic factor for soft tissue (a) vitamin C
sarcoma is: (b) vitamin A
(a) site (c) copper
(b) size (d) vitamin D
(c) grade (e) vitamin E
(d) cell type
80. The most common source of infection in burn
73. After reduction of posterior knee dislocation, the patients is:
patient should undergo: (a) burn wound
(a) observation (b) urinary tract infection
(b) discharge (c) pneumonia
(c) splinting (d) thrombophlebitis
(d) angiogram (e) endocarditis
(e) internal fixation
81. The most common cause of death after kidney
transplantation is:
74. Metabolic acidosis is a complication of topical appli- (a) operative technical complications
cation of: (b) atherosclerotic complications
(a) sodium mafenide (c) infection
(b) silver nitrate (d) cancer
(c) silver sulfadiazine (e) graft rejection
(d) Betadine
(e) bacitracin
82. Which of the following is least appropriate when
evaluating a 14-year-old girl with a breast lump?
75. Stored blood is deficient in: (a) ultrasound
(a) factor II (b) clinical follow-up
(b) factor VII (c) mammography
(c) factor VIII (d) fine-needle aspiration
(d) factor IX (e) excisional biopsy
(e) factor XI
174. Surgery Review Questions 411
83. An absolute contraindication to renal trans- 90. Coumadin-induced skin necrosis is due to:
plantation is: (a) protein S deficiency
(a) chronic osteomyelitis (b) protein C deficiency
(b) diabetes mellitus (c) antithrombin III deficiency
(c) age 55 years (d) disseminated intravascular coagulation
(d) lung cancer treated 10 years ago
(e) exertional angina 91. A patient receiving 1,800 cal/day in total parenteral
nutrition will require:
(a) 125 g protein/day
84. Changes occurring in stored blood include:
(b) 150 g protein/day
(a) increased H
(c) 200 g protein/day
(b) increased 2,3-diphosphoglycerate
(d) 250 g protein/day
(c) decreased red cell fragility
(e) 300 g protein/day
(d) decreased K
(e) increased Ca2
92. Apoptosis:
 (a) is an energy-dependent cell death
85. The most effective method of treating hyperkalemia (b) results in cell swelling
is: (c) is associated with an inflammatory response
(a) intravenous calcium gluconate (d) is usually toxin induced
(b) intravenous sodium bicarbonate (e) is indistinguishable from necrosis
(c) hemodialysis
(d) cation-exchange resin 93. Splenic artery aneurysm:
(e) intravenous glucose-insulin (a) is usually asymptomatic
(b) is usually a pseudoaneurysm
(c) is more common in young males
86. The most common cause of hypercalcemia in hos-
(d) rupture is seldom fatal
pitalized patients is:
(a) primary hyperparathyroidism
94. The site of action of aldosterone is:
(b) metastatic carcinoma
(a) proximal renal tubules
(c) sarcoidosis
(b) distal renal tubules
(d) immobility
(c) collecting ducts
(e) milk alkali syndrome
(d) glomeruli
(e) loop of Henle
87. Smoke inhalation is most reliably excluded by:
(a) absence of carbonaceous sputum 95. The site of action of antidiuretic hormone is:
(b) normal carboxyhemoglobin level (a) collecting ducts
(c) normal xenon-133 inhalation scan (b) glomeruli
(d) normal chest x-ray (c) proximal tubules
(e) normal flexible bronchoscopy (d) distal tubules
(e) loop of Henle
88. Which of the following ligands bind to cell surface
96. The common bile duct:
receptors?
(a) lies to the right of the hepatic artery
(a) steroids
(b) is posterior to the hepatic artery
(b) catecholamines
(c) lies to the right of the portal vein
(c) retinoids
(d) is posterior to the portal vein
(d) thyroid hormones
(e) lies to the left of the hepatic artery
(e) vitamin D
97. A respiratory quotient (RQ) of 1 indicates that the
89. von Willebrand’s disease: main source of fuel is:
(a) is autosomal dominant (a) fatty acid
(b) results in prolonged prothrombin time (b) protein
(c) results in thrombocytopenia (c) carbohydrate
(d) is associated with normal bleeding time (d) ketone
(e) is corrected by factor VIII therapy (e) glycerol

Dr. wessam alzaidat - General surgery


412 Part XVI. Surgery Review Questions and Answers
98. Magnesium: 105. DNA alkylation is the chemotherapeutic action of:
(a) deficiency can be accurately diagnosed with (a) cyclophosphamide
serum level measurement (b) vincristine
(b) hypomagnesemia is associated with hyper- (c) methotrexate
kalemia (d) doxorubicin
(c) hypomagnesemia is associated with neuromus-
cular excitability
106. Spontaneous perforation of the esophagus:
(d) hypomagnesemia is a complication of renal
(a) is usually seen in elderly females
failure
(b) is the most common cause of esophageal perfo-
(e) is a major extracellular cation
ration
99. The management of pancreatic pleural fistula is: (c) is usually located in the midesophagus
(a) distal pancreatectomy (d) is usually preceded by history of dysphagia
(b) pancreaticojejunostomy (e) typically presents with sudden onset
(c) cystogastrostomy
(d) tube thoracostomy 107. Fentanyl:
(e) thoracotomy and decortication (a) is normally found in the body
(b) is 100 times more potent than morphine
100. 48 hours after coronary artery bypass grafting, (c) has twice as long a duration of action as mor-
nausea with epigastric pain and tenderness are most phine
likely due to: (d) results in hypotension because of histamine
(a) perforated peptic ulcer release
(b) acute cholecystitis
(c) acute pancreatitis
(d) acute myocardial infarction 108. Popliteal artery aneurysms:
(e) mesenteric ischemia (a) are commonly complicated by rupture
(b) occur equally in both sexes
101. Hypotension develops after pneumoperitoneum (c) are not surgically treated unless size is 4 cm
and trocar placement for laparoscopic cholecystec- (d) are bilateral in more than 50% of cases
tomy. The next action is to:
(a) convert to open cholecystectomy 109. The lower esophageal sphincter (LES) tone:
(b) deflate the abdomen (a) is increased by gastrin
(c) give intravenous fluids (b) is decreased by metoclopramide
(d) place the patient in head down position (c) is increased by nicotine
(e) check for bowel injury (d) is increased by chocolate
102. Delayed primary wound closure:
(a) results in increased angiogenesis 110. Tachycardia is the main side effect of:
(b) results in decreased wound strength (a) fentanyl
(c) results in lower collagen content (b) succinylcholine
(d) results in a wider scar (c) morphine
(d) pancuronium
103. The most common cause of esophageal perforation
is:
111. Intralobar pulmonary sequestration often presents
(a) penetrating neck injury
with:
(b) iatrogenic
(a) chest pain
(c) spontaneous
(b) dyspnea
(d) foreign body
(c) recurrent pneumonia
(e) malignancy
(d) hemoptysis
104. Multicentricity is characteristic of:
(a) squamous cell carcinoma of the lung 112. Doxorubicin:
(b) small cell lung cancer (a) is an alkylating agent
(c) bronchoalveolar carcinoma (b) is an antimetabolite
(d) bronchial adenocarcinoma (c) is an alkaloid
(e) bronchial carcinoid (d) is an anthracycline antibiotic
174. Surgery Review Questions 413
113. A 45-year-old man has an 8  4 cm soft tissue mass 120. Arrest of cell cycle in the metaphase is the action
in his right thigh. The most appropriate method to of:
confirm the diagnosis of sarcoma is: (a) cyclophosphamide
(a) fine-needle aspiration (b) methotrexate
(b) core biopsy (c) doxorubicin
(c) local excision (d) vincristine
(d) incisional biopsy
(e) enucleation 121. After an acute myocardial infarction, elective oper-
ation should be postponed for at least:
114. Appropriate treatment of malignant hypothermia is (a) 1 month
intravenous: (b) 3 months
(a) morphine sulfate (c) 6 months
(b) dantrolene (d) 1 year
(c) benzodiazepines (e) 2 years
(d) KCl
122. Succinylcholine is contraindicated for:
(e) calcium gluconate
(a) patients with hepatitis
(b) emergency intubation of burn victims
115. Volkmann’s contracture is a complication of:
(c) celiotomy 2 weeks after spinal cord injury
(a) humeral head fracture
(d) parotidectomy
(b) femoral neck fracture
(e) thyroidectomy
(c) posterior dislocation of the knee
(d) supracondylar humeral fracture 123. Splenic artery aneurysm:
(e) Colles’ fracture (a) can present with double-rupture phenomenon
(b) is more common in males
116. Decreasing glucose and increasing fat in total par- (c) is seldom multiple
enteral nutrition will: (d) will continue to enlarge if left untreated
(a) increase respiratory quotient
(b) increase C02 production 124. One week after Coumadin therapy, a patient devel-
(c) decrease minute ventilation ops severe pain in the right leg with areas of skin
(d) delay weaning from mechanical ventilation necrosis. Appropriate action is:
(a) transfusion of fresh-frozen plasma
117. Acute appendicitis in pregnancy: (b) intravenous vitamin K therapy
(a) has a higher incidence of wound infection (c) wound debridement and antibiotics
(b) is least common in the third trimester (d) stopping Coumadin and starting heparin
(c) leukocytosis is important in making the (e) lumbar sympathectomy
diagnosis
125. Mixed venous saturation is increased in:
(d) initial observation is recommended when diag-
(a) hypovolemic shock
nosis is suspected
(b) septic shock
(c) cardiogenic shock
118. Eight hours after esophagogastroduodenoscopy
(d) neurogenic shock
(EGD), a patient complains of severe substernal
(e) anaphylactic shock
pain. Chest x-ray film shows left pleural effusion.
The next test is: 126. Paraneoplastic syndrome is most commonly associ-
(a) repeated EGD ated with:
(b) celiotomy (a) bronchial adenocarcinoma
(c) computed tomography scan of the chest (b) small cell lung cancer
(d) water-soluble esophagogram (c) bronchoalveolar carcinoma
(e) thoracotomy (d) bronchial carcinoid

119. The initial site of distant metastasis in breast 127. The endothelial cells are the source of:
cancer is: (a) factor II
(a) lungs (b) factor V
(b) liver (c) factor VII
(c) bones (d) factor VIII
(d) brain (e) factor X

Dr. wessam alzaidat - General surgery


414 Part XVI. Surgery Review Questions and Answers
128. Which of the following is likely to be multiple? 136. Pulmonary fibrosis is a complication of:
(a) gastrinoma (a) bleomycin
(b) insulinoma (b) cyclophosphamide
(c) somatostatinoma (c) tamoxifen
(d) vasoactive intestinal peptide–producing tumor (d) vincristine
(e) gluconoma (e) methotrexate

129. The mechanism of action of heparin is: 137. The best indicator of adequate resuscitation in
(a) direct inhibition of thrombin shock is:
(b) prevention of factor II synthesis (a) normal blood pressure
(c) inhibition of cyclo-oxygenase (b) normal pulse
(d) potentiation of antithrombin III action (c) adequate urine output
(e) improved mental status
130. Which of the following is an aminoester? (f) decreased lactate level
(a) tetracaine
(b) lidocaine 138. Adrenal incidentalomas:
(c) bupivacaine (a) are commonly malignant
(d) mepivacaine (b) require endocrine evaluation
(e) etidocaine (c) are commonly associated with hyperaldostero-
nism
131. Vitamin K: (d) are not resected unless they are 6 cm
(a) is required for factor VIII synthesis
(b) is water soluble 139. Peaked T-wave on electrocardiogram is a feature of:
(c) is absorbed in the proximal small bowel (a) hypernatremia
(d) requires bile salts for absorption (b) hypermagnesemia
(c) hyperkalemia
132. Hyponatremia is a complication of topical ap- (d) hypocalcemia
plication of: (e) hyponatremia
(a) bacitracin
(b) povidone-iodine 140. Distinction between prerenal and renal azotemia is
(c) silver sulfadiazine best made by:
(d) Sulfamylon (a) urine sodium level
(e) silver nitrate (b) serum sodium level
(c) urine creatinine level
133. Which of the following is an etiologic factor in (d) serum creatinine level
occlusive vascular diseases? (e) urine microscopy
(a) arginine
(b) glutamine 141. During prolonged starvation, the brain’s main
(c) methionine fuel is:
(d) homocystine (a) glucose
(e) xanthine (b) amino acids
(c) ketones
134. Cardiomyopathy is a complication of: (d) short-chain fatty acids
(a) methotrexate
(b) tamoxifen 142. Dobutamine:
(c) doxorubicin (a) has a major chronotropic action
(d) cyclophosphamide (b) decreases cardiac filling pressure
(e) vincristine (c) increases systemic vascular resistance
(d) directly increases renal blood flow
135. Spontaneous antitumor activity is a function of:
(a) macrophages 143. Epidermal growth factor stimulates:
(b) B lymphocytes (a) angiogenesis
(c) cytotoxic T cells (b) wound contraction
(d) helper T cells (c) fibroblast proliferation
(e) natural killer cells (d) epithelialization
174. Surgery Review Questions 415
144. Magnesium: 151. Headache, vomiting, and seizure may develop with
(a) is an intracellular ion rapid correction of:
(b) deficiency leads to hyporeflexia (a) hyponatremia
(c) depletion is common in shock states (b) hypernatremia
(d) is excreted mostly in stool (c) hypokalemia
(d) hyperkalemia
145. The most commonly injured organ in a patient with (e) hypercalcemia
seatbelt sign is:
152. The most common microbial agent transmitted by
(a) the liver
blood transfusion is:
(b) the spleen
(a) Cytomegalovirus
(c) the colon
(b) human immunodeficiency virus
(d) the pancreas
(c) hepatitis C virus
(e) the small intestine
(d) hepatitis B virus

146. The development of necrolytic erythematous rash in a 153. The most frequent major complication of blood
diabetic patient requires assessment of the serum transfusion is:
level of: (a) volume overload
(a) insulin (b) hemolytic reaction
(b) somatostatin (c) human immunodeficiency virus infection
(c) cortisol (d) hepatitis C
(d) gastrin (e) hepatitis B
(e) glucagon
154. adenosine diphosphate–induced platelet aggrega-
tion is inhibited by:
147. Appearance of a U-wave on electrocardiogram
(a) aspirin
occurs in:
(b) heparin
(a) hyperkalemia
(c) dipyridamole
(b) hypokalemia
(d) Coumadin
(c) hypermagnesemia
(d) hypomagnesemia
155. Malignant hyperthermia
(e) hypercalcemia
(a) is rare in children
(b) is an autosomal dominant disorder
148. Seizures can be associated with the use of: (c) results in respiratory alkalosis
(a) aztreonam (d) is prevented by intravenous calcium gluconate
(b) Flagyl
(c) clindamycin 156. Colonic perforation in a patient with acquired
(d) ciprofloxacillin immunodeficiency syndrome is most likely due to:
(e) imipenem-cilastatin (a) Clostridium difficile
(b) Cytomegalovirus
149. Nasotracheal intubation: (c) Bacteroides
(a) is contraindicated for suspected cervical spine (d) Salmonella typhi
injury (e) Escherichia coli
(b) is contraindicated for apneic patients
157. Which of the following results of thyroid fine-needle
(c) is usually successful on the first attempt
aspiration indicates surgical treatment?
(d) is less tolerated by patients than is endotracheal
(a) a nodule that disappeared on aspiration
intubation
(b) clumps of follicular cells
(c) anaplastic thyroid cells
150. The most important initial management of sus- (d) lymphoma
pected blunt myocardial injury is:
(a) electroencephalographic monitoring 158. Somatostatin:
(b) chest computed tomography scan (a) is produced by antral G cells
(c) assessment of cardiac enzymes (b) has a half-life of 30 minutes
(d) insertion of pulmonary artery catheter (c) inhibits gastric motility
(e) echocardiogram (d) increases portal blood flow

Dr. wessam alzaidat - General surgery


416 Part XVI. Surgery Review Questions and Answers
159. The most common site for accessory splenictissue 166. Esophageal diverticula:
is: (a) traction diverticula are false diverticula
(a) gastrosplenic ligament (b) pulsion diverticula are common in the
(b) gastrocolic ligament midesophagus
(c) splenic hilum (c) pulsion diverticula are usually associated with
(d) splenocolic ligament enlarged lymph nodes
(e) the pelvis (d) myotomy is always indicated for pulsion
diverticula
160. The most sensitive indicator of increased hemolysis
with hypersplenism is: 167. The primary fuel source for enterocytes is:
(a) reticulocyte count (a) short-chain fatty acids
(b) splenic enlargement (b) glucose
(c) bilirubin level (c) triglycerides
(d) hemoglobin level (d) lactulose
(e) haptoglobin level (e) glutamine

161. Howship-Romberg sign is characteristic of: 168. Infection caused by dog and cat bites is due to:
(a) femoral hernia (a) Pasteurella species
(b) Spigelian hernia (b) Mycobacterium
(c) obturator hernia (c) Staphylococcus aureus
(d) lumbar hernia (d) Actinomyces
(e) epigastric hernia (e) Candida

162. Which of the following is least indicated for evalu-


169. Which of the following is preserved in radical neck
ating adrenal incidentaloma?
dissection?
(a) magnetic resonance imaging
(a) internal jugular vein
(b) 24-hour urinary catecholamine testing
(b) sternomastoid muscle
(c) 24-hour urinary cortisol testing
(c) spinal accessory nerve
(d) low dexamethasone suppression test
(d) submandibular salivary gland
(e) fine-needle aspiration
(e) posterior belly of the digastric muscle

163. Lobular carcinoma in situ:


(a) is mostly found in premenopausal women 170. The organ most commonly involved in graft-versus-
(b) usually presents as a breast lump host reaction is:
(c) has characteristic calcification pattern on mam- (a) lungs
mography (b) kidneys
(d) is precancerous (c) heart
(e) associated cancer is lobular in nature (d) skin

164. Sequence of return of gastrointestinal motility after 171. The most characteristic of malignancy in a cystic
surgery is: pancreatic neoplasm is:
(a) intestine, stomach, colon (a) size 6 cm
(b) stomach, intestine, colon (b) wall calcification
(c) colon, intestine, stomach (c) multiple loculations
(d) colon, stomach, intestine (d) dense vascularity
(e) stomach, colon, intestine
172. Which of the following is contraindicated for a para-
165. The treatment of choice for Barrett’s esophagus plegic undergoing laparotomy?
with severe dysplasia is: (a) benzodiazepines
(a) follow-up endoscopy and biopsy (b) sodium thiopental
(b) esophagectomy (c) succinylcholine
(c) Nissen fundoplication (d) propofol
(d) Proton pump inhibitors (e) fentanyl
174. Surgery Review Questions 417
173. Which of the following is consistent with pleural 180. The most common retroperitoneal sarcoma is:
transudate? (a) liposarcoma
(a) red blood cells count of 1,000/mm3 (b) fibrosarcoma
(b) white blood cells count of 1,500/mm3 (c) leiomyosarcoma
(c) specific gravity of 1.120 (d) neurosarcoma
(d) protein concentration of 3.5 g/dL
181. Platelet dysfunction in uremia can be corrected
with:
174. Appropriate management of renal cell carcinoma
(a) fresh-frozen plasma
extending into the inferior vena cava is:
(b) cryoprecipitate
(a) radiotherapy
(c) desmopressin (DDAVP)
(b) chemotherapy
(d) factor VIII concentrate
(c) chemoradiation
(e) vitamin K
(d) radical nephrectomy and caval tumor extrac-
tion 182. Gynecomastia is a side effect of:
(e) radical nephrectomy, caval resection, and graft (a) ketoconazole
interposition (b) amphotericin B
(c) fluconazole
175. The gastroduodenal artery is a branch of the: (d) miconazole
(a) celiac axis
(b) right gastric artery 183. Optimum approach to inflammatory breast car-
(c) common hepatic artery cinoma is:
(d) right hepatic artery (a) total mastectomy
(e) left hepatic artery (b) modified radical mastectomy
(c) lumpectomy and radiotherapy
(d) chemotherapy
176. Inflammatory aortic aneurysm:
(a) repair is associated with a higher incidence of (e) chemotherapy, modified radical mastectomy,
graft infection and radiotherapy
(b) is more likely to rupture than is a noninflam-
184. A potent inhibitor of T cell proliferation is:
matory aneurysm
(a) transforming growth factor-
(c) leads to circumferential thickening of the aorta
(b) platelet-derived growth factor
(d) may present with abdominal pain in the absence
(c) epidermal growth factor
of rupture
(d) basic fibroblast growth factor

177. The initial management of a diabetic patient with 185. The most reliable indicator of successful endotra-
fever, plantar ulcer, and foot edema is: cheal intubation is:
(a) antibiotics and elevation (a) chest x-ray
(b) hyperbaric oxygen (b) end-tidal CO2
(c) exploration and ulcer debridement (c) cord visualization
(d) midtarsal amputation (d) chest auscultation
(e) femoropopliteal arterial bypass (e) pulse oximetry

186. The layer responsible for the strength of an intes-


178. Male breast cancer is associated with:
tinal anastomosis is the:
(a) BRCA-1 gene
(a) mucosa
(b) BRCA-2 gene
(b) submucosa
(c) APC gene
(c) muscularis propria
(d) p53 gene
(d) serosa

179. The diagnosis of inflammatory breast cancer is 187. Healing of the donor site for a split thickness skin
confirmed by: graft is accelerated by:
(a) mammography (a) transforming growth factor-
(b) fine-needle aspiration (b) recombinant human growth hormone
(c) ultrasound (c) epidermal growth factor
(d) skin biopsy (d) platelet-derived growth factor

Dr. wessam alzaidat - General surgery


418 Part XVI. Surgery Review Questions and Answers
188. Prevention of empyema in a patient with residual 195. The respiratory quotient in prolonged starvation is:
hemothorax and a chest tube in place is best (a) 1.0
achieved by: (b) 1.2
(a) intravenous third-generation cephalosporins (c) 0.8
(b) placement of a second chest tube (d) 0.7
(c) needle thoracentesis
(d) intravenous vancomycin 196. The most abundant amino acid in the body is:
(a) alanine
189. Popliteal artery entrapment: (b) valine
(a) is more common in females than males (c) leucine
(b) is diagnosed by passive dorsiflexion of the foot (d) glutamine
(c) results from compression by the medial head of (e) arginine
the gastrocnemius
(d) requires bilateral exploration in most cases 197. Maximum efficiency of glucose use in total par-
enteral nutrition occurs at the infusion rate of:
190. Cervical anastomosis after esophagectomy for (a) 4 mg/kg/min
cancer: (b) 5 mg/kg/min
(a) has a lower leak rate than thoracic anastomosis (c) 6 mg/kg/min
(b) leak is likely to heal spontaneously (d) 7 mg/kg/min
(c) has a higher long-term mortality rate than tho-
racic anastomosis 198. Refeeding syndrome is most commonly related to:
(a) hyponatremia
(d) has a lower operative mortality rate than tho-
racic anastomosis (b) hypocalcemia
(c) hypophosphatemia
191. Appropriate management of 3-cm squamous cell (d) hypokalemia
carcinoma of the anal canal is:
199. Eczematoid rash at intertriginous areas with pro-
(a) chemotherapy
longed total parenteral nutrition is caused by:
(b) abdominoperineal resection
(a) zinc deficiency
(c) local excision
(b) fatty acid deficiency
(d) radiotherapy  local excision
(c) copper deficiency
(e) chemotherapy  radiation
(d) magnesium deficiency
(e) niacin deficiency
192. Idiopathic thrombocytopenic purpura:
(a) is caused by antiplatelet immunoglobulin G 200. The volume of air moved with maximum exhalation
originating in the spleen after deep inhalation measures:
(b) is associated with splenomegaly (a) vital capacity
(c) is associated with prolonged prothrombin time (b) inspiratory reserve volume
(d) splenectomy is required for most pediatric cases (c) tidal volume
(e) occurs with a male to female ratio of 3 : 1 (d) residual volume
(e) total lung volume
193. The most common cause of death related to a
central venous catheter is: 201. Bleeding after adequate heparin reversal with pro-
(a) air embolism tamine is usually caused by:
(b) central vein perforation (a) protamine toxicity
(c) tension pneumothorax (b) heparin rebound
(d) catheter embolism (c) hypothermia
(e) catheter-related sepsis (d) thrombocytopenia
(e) factor VIII depletion
194. Postoperative morbidity after splenectomy for
hematologic diseases is highest for: 202. Ileocolic intussusception in children:
(a) idiopathic thrombocytopenic purpura (a) presents with rectal bleeding in 90% of cases
(b) hereditary spherocytosis (b) is commonly caused by an underlying pathology
(c) myeloid dysplasia (c) is ideally treated by operative reduction
(d) sickle cell anemia (d) is the most common cause of intestinal obstruc-
(e) thalassemia tion before the age of 3 years
174. Surgery Review Questions 419
203. Alopecia with prolonged total parenteral nutrition 210. Pulmonary sequestration:
may be caused by: (a) commonly occurs on the right side
(a) zinc deficiency (b) intralobar sequestration is supplied by the pul-
(b) magnesium deficiency monary artery
(c) vitamin A intoxication (c) extralobar sequestration is supplied by the
(d) essential fatty acids deficiency aorta
(e) selenium deficiency (d) intralobar sequestration drains into systemic
veins
204. A 1-cm carcinoid found in the midappendix after (e) extralobar sequestration drains into pulmonary
appendectomy requires: veins
(a) octreotide therapy
(b) right hemicolectomy 211. The most important diagnostic test for a thyroid
(c) streptozotocin therapy nodule is:
(d) no further action (a) ultrasound
(b) radioactive isotope scan
205. The most common complication of heparin reversal (c) thyroid function test
with protamine is: (d) fine-needle aspiration
(a) bradycardia (e) computed tomography scan
(b) hypotension
212. Pleomorphic parotid adenoma:
(c) thrombotic crisis
(a) requires core biopsy before resection
(d) thrombocytopenia
(b) is adequately treated with enucleation
(e) leukopenia
(c) commonly undergoes malignant transformation
(d) commonly results in facial palsy
206. The most common side effect of pancuronium is: (e) is the most common parotid neoplasm
(a) tachycardia
(b) hypotension 213. The mean arterial pressure is:
(c) hyperkalemia (a) diastolic pressure  1/2 pulse pressure
(d) hyperthermia (b) systolic pressure  pulse pressure
(e) renal insufficiency (c) systolic pressure  1/3 pulse pressure
(d) diastolic pressure  1/3 pulse pressure
207. The most common cause of massive lower
gastrointestinal bleeding in children is: 214. A contraindication to breast-conserving therapy is:
(a) anal fissure (a) age more than 70 years
(b) intussusception (b) coronary artery disease
(c) Meckel’s diverticulum (c) family history of breast cancer
(e) angiodysplasia (d) collagen vascular disease

215. Death from tension pneumothorax is caused by:


208. Fine-needle aspiration of bilateral upper cervical
(a) decreased venous return
lymphadenopathy shows squamous cell carcinoma.
(b) cardiac arrhythmia
No primary lesion is found on clinical examination.
The most likely source is: (c) acute hypoxia
(d) acute hypercapnia
(a) lungs
(b) esophagus 216. The most important element in the history of an
(c) tongue infant with vomiting is:
(d) tonsils (a) the frequency of vomiting
(e) nasopharynx (b) the amount of vomiting
(c) the presence of fever
209. Analysis of pleural effusion shows red blood cell (d) if vomiting is projectile
count of 500/mm3, white blood cell count of (e) if vomiting is bile stained
600/mm3, protein level of 1.5 g/dL, and specific
gravity of 1.010. The most likely diagnosis is: 217. The substrate for nitric oxide synthetase is:
(a) congestive heart failure (a) glutamine
(b) parapneumonic effusion (b) alanine
(c) hemothorax (c) L-arginine
(d) bronchogenic carcinoma (d) valine

Dr. wessam alzaidat - General surgery


420 Part XVI. Surgery Review Questions and Answers
218. The most frequent complication of giant gastric 226. The rate of axonal regeneration after nerve injury
ulcer is: is:
(a) gastric outlet obstruction (a) 1 cm/month
(b) perforation (b) 1 mm/day
(c) upper gastrointestinal bleeding (c) 1 mm/week
(d) gastroenteric fistula (d) 1 cm/week

219. Mucosal defense is provided by immunoglobulin:


(a) A 227. The optimal treatment for bleeding gastric varices
(b) G in chronic pancreatitis is:
(c) M (a) distal pancreatectomy
(d) D (b) splenorenal shunt
(e) E (c) portocaval shunt
(d) splenectomy
(e) transjugular intrahepatic portosystemic shunt
220. The most common cause of nipple discharge is:
procedure
(a) duct ectasia
(b) breast cancer
(c) intraductal papilloma 228. Electrical burn injury:
(d) pituitary adenoma (a) can be estimated by the extent of skin damage
(e) fibrocystic disease (b) typically affects the trunk more than the
extremities
221. The most common agent transmitted by blood (c) requires close cardiac monitoring
transfusion is: (d) is inversely related to tissue resistance
(a) human immunodeficiency virus
(b) hepatitis B virus
(c) hepatitis C virus 229. The highest rate of metastasis occurs in carcinoid
(d) Cytomegalovirus arising from:
(a) appendix
222. Advantage of full thickness over split thickness skin (b) bronchus
graft is: (c) ileum
(a) less wound contraction (d) stomach
(b) better take
(c) more resistance to infection
(d) better sensory function 230. The most common presentation of ductal carcinoma
in situ is:
223. Cyclosporin A inhibits the production of: (a) breast pain
(a) interleukin–1 (b) breast lump
(b) interleukin–2 (c) nipple discharge
(c) interleukin–6 (d) microcalcification
(d) tumor necrosis factor-
 231. The main source of fuel in sepsis is:
224. The main component of urinary stones complicat- (a) glucose
ing resection of terminal ileum is: (b) fatty acids
(a) urate (c) ketones
(b) oxalate (d) amino acids
(c) phosphate
(d) ammonium
232. A patient with electrical burn of the leg complains
225. A characteristic feature of toxic shock syndrome in of pain on passive movement of the foot. The pedal
children with burns is: pulses are diminished. The next step is:
(a) purulent wound drainage (a) escharotomy
(b) leukopenia (b) femoral angiogram
(c) hypothermia (c) leg elevation and intravenous heparin
(d) bradycardia (d) fasciotomy
174. Surgery Review Questions 421
233. The most common complication of blood transfu- 240. Hypothermic coagulopathy:
sion is: (a) is associated with clotting factor depletion
(a) hemolytic reaction (b) can be corrected with fresh-frozen plasma
(b) human immunodeficiency virus transmission transfusion
(c) allergic reaction (c) is associated with prolonged prothrombin time
(d) volume overload and partial thromboplastin time
(e) coagulopathy (d) is a complication of massive transfusion

234. The highest concentration of immunoglobulin A– 241. Secretin:


producing cells is in: (a) stimulates gastrin secretion
(a) bloodstream (b) stimulates pancreatic enzyme secretion
(b) oral cavity (c) inhibits intestinal motility
(c) bronchial tree (d) stimulates gastric acid secretion
(d) small intestine
(e) urogenital tract 242. The use of inverse ratio ventilation will:
(a) decrease auto-positive end-expiratory pressure
235. In the treatment of coagulopathy: (b) improve alveolar ventilation
(a) calcium should be routinely infused with (c) increase incidence of pneumonia
massive transfusion (d) decrease mean airway pressure
(b) desmopressin (DDAVP) stimulates the release
of factor VIII 243. The optimal management of esophageal leiomyoma
(c) von Willebrand’s disease can be treated with is:
factor VIII concentrate (a) Ivor-Lewis esophageal resection
(d) the effect of aspirin can be reversed by fresh- (b) transhiatal esophageal resection
frozen plasma (c) segmental esophageal resection
(d) esophagomyotomy and enucleation
(e) endoscopic resection
236. The most common source of metastatic small bowel
tumor is:
(a) lungs 244. Hand infection caused by a human bite is due to:
(b) melanoma (a) Staphylococcus aureus
(c) breast (b) Clostridium difficile
(d) soft tissue sarcoma (c) Herpes simplex
(d) Eikenella corrodens
(e) Candida species
237. Myeloid metaplasia:
(a) is a disease of young females 245. Gastric intrinsic factor is secreted from:
(b) is rarely associated with splenomegaly (a) parietal cells
(c) results in extramedullary hematopoiesis (b) chief cells
(d) results in increased bone marrow megakary- (c) antral G cells
ocytes (d) D cells

238. Which of the following is contraindicated in man- 246. 246 Which of the following directly induces
aging corrosive esophagitis? coagulation?
(a) gastric lavage (a) superglue
(b) esophagogastroduodenoscopy (b) oxidized cellulose
(c) corticosteroids (c) absorbable gelatin sponge
(d) tracheostomy (d) microfibrillar collagen (Avitene)

239. Which of the following is effective in treating refrac- 247. Which of the following inhibits gastric bicarbonate
tory Crohn’s fistula? secretion?
(a) total parenteral nutrition (a) prostaglandins
(b) prednisone (b) vagal stimulation
(c) infliximab (c) aspirin
(d) azathioprine (d) gastrin

Dr. wessam alzaidat - General surgery


422 Part XVI. Surgery Review Questions and Answers
248. A characteristic of prerenal azotemia is: 255. A colorectal tumor that invades through the mus-
(a) abnormal urine sediment cularis propria into the subserosa is a:
(b) fractional excretion of sodium value 1% (a) T1 lesion
(c) urine sodium level 40 mEq/L (b) T2 lesion
(d) blood urea nitrogen/serum creatinine level 10 (c) T3 lesion
(d) T4 lesion

249. At an operation for small bowel obstruction, cecal


volvulus is diagnosed. The cecum is viable. The pro- 256. Glutamine:
cedure of choice is: (a) is supplied in total parenteral nutrition
(a) cecopexy (b) increases intestinal cellularity
(b) tube cecostomy (c) is an essential amino acid
(c) right hemicolectomy (d) is a substrate for gluconeogenesis
(d) resection, ileostomy, and mucous fistula
257. Which of the following is a defect in the hemoglo-
bin chain that responds to splenectomy?
250. For a patient with a serum potassium level of (a) idiopathic thrombocytopenic purpura
7 mEq/dL and an absent P-wave on electrocardio- (b) hereditary spherocytosis
gram, the initial management is: (c) thalassemia
(a) intravenous Lasix (d) glucose-6-phosphate deficiency
(b) intravenous glucose/insulin
(c) Kayexalate enema
258. A central scar in a hepatic lesion is characteristic of:
(d) intravenous sodium bicarbonate
(a) focal nodular hyperplasia
(e) intravenous calcium gluconate
(b) hepatic adenoma
(c) hemangioma
251. Optimal management of mucosa-associated lym- (d) hamartoma
phoid tissue (MALT) lymphoma is: (e) hepatocellular carcinoma
(a) chemotherapy
(b) total gastrectomy 259. The initial step in management of a hypercalcemic
(c) chemoradiation crisis is intravenous:
(d) antibiotics (a) steroids
(b) calcitonin
(c) saline
252. Positive end-expiratory pressure therapy will result
(d) furosemide
in:
(e) mithramycin
(a) decrease in extravascular lung water
(b) increase in cardiac preload
(c) decrease in atrial natriuretic peptide 260. Hepatic focal nodular hyperplasia:
(d) decrease in functional residual capacity (a) usually occurs in women of reproductive age
(b) is related to oral contraceptive use
(c) presents with abdominal pain in most cases
253. At the initiation of swallowing, the pressure at the (d) carries the risk of spontaneous rupture
lower esophageal sphincter:
(a) remains unchanged 261. The most accurate measure of adequacy of nutri-
(b) decreases and then increases tional support is:
(c) increases and then decreases (a) serum albumin level
(d) increases and then returns to baseline (b) body weight
(c) triceps skinfold measurement
254. The serum sodium level in a 60-year-old man who (d) serum prealbumin level
weighs 70 kg is 125 mEq/L. His sodium deficit is:
(a) 130 mEq 262. Refractory hypokalemia can be caused by:
(b) 210 mEq (a) hypocalcemia
(c) 360 mEq (b) hyponatremia
(d) 520 mEq (c) hypophosphatemia
(e) 850 mEq (d) hypomagnesemia
174. Surgery Review Questions 423
263. A respiratory quotient of 1.2 indicates: 270. Perianal Crohn’s disease:
(a) lipogenesis (a) typically occurs late in the course of the disease
(b) ketogenesis (b) fistulas are usually multiple
(c) pure fat utilization (c) fecal diversion is usually curative
(d) carbohydrates are the source of fuel (d) lesions are typically located posteriorly
(e) proteins are the source of fuel (e) granulomas are seldom found on biopsy

264. Mallory-Weiss tear is located: 271. Appropriate management of chronic pancreatitis


(a) in the distal esophagus with pancreatic duct ectasia is:
(b) anteriorly across the gastroesophageal junction (a) pancreaticoduodenectomy
(c) posteriorly across the gastroesophageal junc- (b) distal pancreaticojejunostomy
tion (c) longitudinal pancreaticojejunostomy
(d) on the lesser curve of the cardia (d) near total pancreatectomy
(e) on the greater curve of the cardia (e) sphincteroplasty

265. The blood supply of the thoracic stomach used for 272. The most common cause of massive bleeding in
esophageal replacement depends on: chronic pancreatitis is:
(a) the left gastric artery (a) pseudoaneurysm
(b) the short gastric vessels (b) arteriovenous fistula
(c) the right gastroepiploic artery (c) mycotic aneurysm
(d) the left gastroepiploic artery (d) fibromuscular dysplasia

266. The most serious complication of gastric bypass pro- 273. Colonic distension in toxic megacolon is most
cedure is: prominent in the:
(a) hepatic failure (a) cecum
(b) anastomotic leak (b) ascending colon
(c) urolithiasis (c) transverse colon
(d) intestinal obstruction (d) descending colon
(e) hypocalcemia (e) sigmoid colon

267. Helicobacter pylori: 274. At the lung functional residual capacity:


(a) colonization is highest in childhood (a) chest wall exerts inward elastic recoil
(b) eradication does not influence ulcer recurrence (b) lungs exert outward elastic recoil
(c) is isolated in up to 90% of duodenal ulcer cases (c) alveolar pressure equals pleural pressure
(d) metronidazole achieves eradication as a single (d) lungs and chest wall exert equal and opposing
agent recoil

268. L5–S1 disc lesion will result in: 275. Which of the following inhibits intestinal motility?
(a) weak plantar flexion (a) gastrin
(b) weak dorsiflexion (b) cholecystokinin
(c) absent knee jerk (c) epinephrine
(d) lost sensation in the big toe (d) motilin
(e) serotonin

269. A patient with head injury opens his eyes and with-
draws his arm to pain. He is making incomprehen- 276. The most common site of ectopic pheochromocy-
sible sounds. His Glasgow Coma Scale score is: toma is:
(a) 12 (a) lower pole of the kidney
(b) 10 (b) para-aortic tissue
(c) 8 (c) mediastinum
(d) 6 (d) pelvis

Dr. wessam alzaidat - General surgery


424 Part XVI. Surgery Review Questions and Answers
277. Which of the following is poorly absorbed in 285. Which of the following is an inhibitor of wound
achlorhydria? contraction?
(a) proteins (a) glucocorticoids
(b) fats (b) d-penicillamine
(c) bile salts (c) colchicine
(d) vitamin D (d) aspirin
(e) vitamin B12
286. Fetal wound healing is characterized by:
278. The most common cause of hypophosphatemia in (a) increased angiogenesis
hospitalized patients is: (b) increased hyaluronic acid synthesis
(a) renal failure (c) increased inflammatory response
(b) sepsis (d) decreased collagen
(c) glucose overload
(d) diarrhea 287. Death from postoperative renal failure is most com-
monly due to:
279. The main fuel for colonocytes is: (a) myocardial infarction
(a) glutamine (b) bleeding
(b) short-chain fatty acids (c) sepsis
(c) alanine (d) liver failure
(d) glucose
(e) ketones 288. Regarding graft rejection:
(a) hyperacute rejection is antibody mediated
280. Pulmonary fibrosis is a complication of:
(b) hyperacute rejection is reversed with steroids
(a) doxorubicin
(c) acute rejection is B cell mediated
(b) methotrexate
(d) acute rejection occurs over month
(c) bleomycin
(d) cyclophosphamide
289. The most important prognostic variable for
(e) vincristine
melanoma is:
(a) gender
281. The most useful serum marker for cancer screening
(b) age
is:
(c) Clark’s level
(a) prostate-specific antigen
(d) Breslow’s thickness
(b) CA 19.9
(e) complexion
(c) -fetoprotein
(d) carcinoembryonic antigen
290. Follicular thyroid carcinoma:
282. The inferior parathyroid gland originates fromthe: (a) is the most common thyroid cancer
(a) first pharyngeal pouch (b) is readily diagnosed with fine-needle aspira-
(b) second pharyngeal pouch tion
(c) third pharyngeal pouch (c) spreads via hematogenous route
(d) fourth pharyngeal pouch (d) is commonly multifocal

283. Hypotension and decreased end-tidal CO2 during 291. Multiple endocrine neoplasia is associated with
laparoscopy are likely due to: germline mutation in:
(a) tension pneumothorax (a) p53 gene
(b) inferior vena cava compression (b) RET proto-oncogene
(c) CO2 embolism (c) N-myc gene
(d) anesthetic overdose (d) APC gene

284. The main fuel for most cancer cells is: 292. Optimal treatment of cloacogenic carcinoma of anal
(a) butyrate canal is:
(b) glutamine (a) local excision
(c) L-arginine (b) abdominoperineal resection
(d) glucose (c) chemotherapy alone
(e) ketones (d) radiotherapy alone
(e) chemoradiation
174. Surgery Review Questions 425
293. The most common hernia in women is: 301. On postoperative day 1, a patient develops a
(a) femoral hernia temperature of 104°F and foul-smelling wound
(b) obturator hernia drainage. The most likely isolate is:
(c) inguinal hernia (a) Gram-negative rods
(d) umbilical hernia (b) Gram-positive rods
(e) spigelian hernia (c) Gram-negative cocci
(d) Gram-positive cocci
294. Gastroschisis:
(a) is usually associated with other anomalies 302. A hemodynamic consequence of carbon dioxide
(b) is usually associated with chromosomal dis- pneumoperitoneum is:
orders (a) decrease in cardiac index
(c) is located on the left of the umbilical cord (b) decrease in systemic vascular resistance
(d) repair is followed by prolonged ileus (c) decrease in mean arterial pressure
(d) increase in cardiac preload
295. The most common visceral aneurysm is:
(a) celiac
(b) superior mesenteric artery 303. In the diabetic foot:
(c) splenic (a) atherosclerosis often involves the pedal arteries
(d) hepatic (b) foot sepsis is often polymicrobial
(c) ankle–brachial index accurately measures the
degree of ischemia
296. Gastroschisis is associated with an increased risk of:
(d) diabetic neuropathy involves only sensory
(a) hepatomegaly
nerves
(b) intestinal atresia
(c) microcephaly
(d) cardiac anomalies 304. Decreased hemoglobin affinity to oxygen at the
tissue level is caused by:
297. The cremaster muscle is derived from: (a) increased body temperature
(a) the external oblique muscle (b) decreased 2,3-diphosphoglycerate
(b) the internal oblique muscle (c) decreased pCO2
(c) the transversus abdominis muscle (d) increased pH
(d) the transversalis fascia
305. A patient with pelvic fracture is hypotensive and has
298. The time for platelet transfusion during splenec- grossly positive diagnostic peritoneal lavage. The
tomy for idiopathic thrombocytopenic purpura is: next step is:
(a) on making the incision (a) angiography and embolization
(b) after ligation of the splenic artery (b) computed tomography of the abdomen and
(c) on induction of anesthesia pelvis
(d) after removal of the spleen (c) celiotomy
(d) application of C-clamp
299. The most common congenital cardiac defect is: (e) application of pneumatic antishock garment
(a) atrial septal defect
(b) ventricular septal defect
(c) transposition of great vessels 306. Heparin:
(d) aortic coarticulation (a) prevents platelet aggregation
(b) prevents factor VII synthesis
(c) is a cyclo-oxygenase inhibitor
300. The most appropriate method to diagnose small (d) potentiates the action of antithrombin III
bowel injury in a conscious trauma patient with
seatbelt injury is:
(a) diagnostic peritoneal lavage 307. Cholecystokinin:
(b) ultrasound (a) stimulates the sphincter of Oddi
(c) computed tomography scan (b) is secreted by the antral mucosa
(d) serial abdominal examination (c) stimulates pancreatic enzyme secretion
(e) plain abdominal film (d) stimulates gastric emptying

Dr. wessam alzaidat - General surgery


426 Part XVI. Surgery Review Questions and Answers
308. The earliest and most specific sign of malignant 315. The lateral boundary of a femoral hernia is:
hyperthermia is: (a) the femoral nerve
(a) high fever (b) the femoral artery
(b) hypotension (c) the femoral vein
(c) increase in end tidal CO2 (d) the lacunar ligament
(d) tachycardia
(e) hypoxia
316. A characteristic of primary hyperaldosteronism is:
(a) hyperkalemia
309. The most frequent manifestation of blunt myocar- (b) hyper-reninism
dial contusion is: (c) hypertension
(a) atrioventricular block (d) hyperplasia of zona reticularis
(b) atrial flutter (e) hyperplasia of zona fasciculata
(c) premature ventricular contractions
(d) premature atrial contractions
(e) atrial fibrillation 317. Regarding the adrenal gland:
(a) the adrenal cortex does not have nerve supply
(b) the adrenal medulla is supplied by postgan-
310. The most commonly injured nerve under general glionic adrenergic fibers
anesthesia is: (c) the right adrenal vein drains into the renal vein
(a) radial nerve (d) the left adrenal vein drains into the inferior
(b) ulnar nerve vena cava
(c) median nerve
(d) brachial plexus
(e) common peroneal nerve 318. At an operation for appendicitis, the appendix is
found to be normal and the fallopian tube is found
to be thickened with surrounding purulent exudate.
311. Respiratory distress associated with goiter is most
The operative management should be:
commonly caused by:
(a) appendectomy
(a) recurrent laryngeal nerve palsy
(b) appendectomy and salpingectomy
(b) malignant tracheal invasion
(c) salpingectomy
(c) retrosternal goitrous extension
(d) no operative intervention
(d) hemorrhage in large goiter

312. Malignant hyperthermia: 319. In the preoperative preparation of pheochromocy-


(a) can be induced by local anesthetics toma, medications are given in the following order:
(b) can be induced by nondepolarizing muscle (a) diuretics and then -blockers
relaxants (b) -blockers and then -blockers
(c) can be induced by nitrous oxide (c) -blockers and then -blockers
(d) is related to disordered K metabolism (d) diuretics and then -blockers
(e) is more common in children than adults
320. Hyperinsulinism in a newborn is most likely caused
313. The most common cardiac anomaly found in adults by:
is: (a) nesidioblastosis
(a) atrial septal defect (b) glycogen storage disease
(b) ventricular septal defect (c) benign insulinoma
(c) transposition of great vessels (d) malignant insulinoma
(d) coarctation of the aorta
321. A patient with abdominal wall desmoid tumor
314. The most common complication of epidural anal- should be screened for:
gesia is: (a) lung cancer
(a) hypotension (b) colon polyps
(d) nausea (c) breast cancer
(c) respiratory depression (d) medullary thyroid carcinoma
(d) deep vein thrombosis (e) pancreatic cancer
174. Surgery Review Questions 427
322. Which of the following is a vasoconstrictor? 330. The colon secretes:
(a) procaine (a) water
(b) bupivacaine (b) sodium
(c) lidocaine (c) chloride
(d) cocaine (d) potassium

323. Paralytic ileus is a complication of: 331. Mild diabetes, skin rash, and glossitis are features of:
(a) cyclophosphamide (a) somatostatinoma
(b) vinca alkaloids (b) gastrinoma
(c) methotrexate (c) glucagonoma
(d) cisplatin (d) insulinoma
(e) doxorubicin
332. The most common intra-abdominal solid tumor in
children is:
324. Neutropenic enterocolitis is a complication of:
(a) nephroblastoma
(a) cytarabine
(b) neuroblastoma
(b) cyclophosphamide
(c) rhabdomyosarcoma
(c) doxorubicin
(d) fibrosarcoma
(d) cisplatin
333. Distinction between toxic epidermal necrolysis and
325. Severe peripheral neuropathy is a complication of: staphylococcal scalding skin syndrome is based on:
(a) vinca alkaloids (a) degree of erythema
(b) cyclophosphamide (b) bullae formation
(c) cytarabine (c) level of exfoliation
(d) cisplatin (d) response to steroids

326. Watery diarrhea, hypokalemia, hypochloremia, and 334. An indication for laparotomy in neonatal necrotiz-
acidosis are features of: ing enterocolitis is:
(a) insulinoma (a) distended bowel loops
(b) gastrinoma (b) thickened bowel wall
(c) vasoactive intestinal peptide–producing tumor (c) abdominal wall erythema
(d) glucagonoma (d) pneumatosis intestinalis

335. Regarding congenital diaphragmatic hernia:


327. Characteristics of somatostatinoma are:
(a) requires emergency operation if respiratory dis-
(a) mild diabetes, skin rash, glossitis
tress is present
(b) ulcer diathesis, diarrhea
(b) foramen of Bochdalek hernia is the most
(c) mild diabetes, diarrhea, gallstones
common type
(d) diarrhea, hypokalemia, hypochloremia
(c) foramen of Morgagni hernia presents with res-
piratory distress
328. An inhibitor of platelet aggregation is: (d) is rarely associated with underlying lung pathol-
(a) prostacyclin I ogy
(b) thromboxane A2
(c) adenosine diphosphate 336. Regarding neonatal Hirschsprung’s disease:
(d) serotonin (a) diagnosis is confirmed by barium enema
(e) von Willebrand’s factor (b) enterocolitis is the leading cause of death
(c) mainly affects females
329. Regarding gastrointestinal bleeding in children: (d) shows absent nerve trunks in the aganglionic
(a) anal fissure is the leading cause segments
(b) clear nasogastric aspirate rules out upper gas-
trointestinal bleeding 337. Prosthetic graft infection is most commonly due to:
(c) Meckel’s diverticulum is seldom the cause of (a) Staphylococcus epidermidis
massive bleeding (b) Staphylococcus aureus
(d) bleeding is the most common presentation of (c) Escherichia coli
intussusception (d) Streptococcus faecalis

Dr. wessam alzaidat - General surgery


428 Part XVI. Surgery Review Questions and Answers
338. Sinistral portal hypertension is most commonly due 345. The most common cause of small bowel obstruction
to: during pregnancy is:
(a) hypercoagulable states (a) incarcerated groin hernia
(b) schistosomiasis (b) adhesions
(c) alcoholism (c) gallstone ileus
(d) chronic pancreatitis (d) intestinal volvulus
(e) intussusception
339. Cervical sympathectomy is least likely to improve:
346. Which of the following is an analgesic?
(a) hyperhidrosis
(a) sodium thiopental
(b) scleroderma
(b) ketamine
(c) causalgia
(c) etomidate
(d) frostbite
(d) propofol

340. Management of deep vein thrombosis during 347. A contraindication to the use of ketamine is:
pregnancy is: (a) hypotension
(a) 10-day intravenous heparin and then Coumadin (b) head injury
until term (c) asthma
(b) 10-day intravenous heparin and then Coumadin (d) hypoventilation
for 6 months
(c) 10-day intravenous heparin and then prophy- 348. The cytokine directly responsible for hepatic acute
lactic subcutaneous heparin until term phase response is:
(d) 10-day intravenous heparin and then therapeu- (a) interleukin-1
tic subcutaneous heparin until term (b) interleukin-2
(c) interleukin-6
(d) tumor necrosis factor-
341. A pleural fluid pH 6.5:

(a) is normal
349. In multiple endocrine neoplasia type 1:
(b) indicates esophageal perforation
(a) almost all patients have parathyroid hyper-
(c) indicates pleural transudate
plasia
(d) indicates bacterial infection
(b) almost all patients have pancreatic endocrine
tumor
342. The most accurate method to diagnose traumatic (c) almost all patients have pituitary adenoma
aortic arch injury is: (d) all patients have hyperparathyroidism, pancre-
(a) upright chest x-ray atic, and pituitary lesions
(b) chest computed tomography
(c) magnetic resonance imaging 350. Postoperative cardiac events are most likely if pre-
(d) transesophageal echocardiogram operative electrocardiogram shows:
(a) ST-T wave changes
(b) bundle branch block
343. Lung resection is contraindicated if:
(c) left ventricular hypertrophy
(a) preoperative PO2 is 60
(d) Q-wave
(b) preoperative PCO2 is 50
(c) FEV1  1 liter
351. An indication for preoperative angiography for
(d) FEV1/VC is 75%
elective abdominal aortic aneurysm surgery is:
(e) MBC is 60%
(a) suspected contained rupture
(b) suspected inflammatory aneurysm
344. Immediately after intravenous (IV) injection of (c) aneurysm larger than 7 cm
5,000 U of heparin, the effect can be reversed with (d) history of claudication
IV:
(a) 10 mg protamine sulfate 352. An early feature of lidocaine toxicity is:
(b) 20 mg protamine sulfate (a) arrhythmia
(c) 30 mg protamine sulfate (b) muscle twitching
(d) 40 mg protamine sulfate (c) respiratory depression
(e) 50 mg protamine sulfate (d) hypotension
174. Surgery Review Questions 429
353. The superior pancreaticoduodenal artery is abranch 360. A 70-kg male with 50% body surface area second-
of: degree burn requires:358
(a) the celiac axis (a) 7,000 mL of lactated Ringer’s solution over the
(b) the superior mesenteric artery first 6 hours
(c) the hepatic artery (b) 7,000 mL of lactated Ringer’s solution over the
(d) the right gastric artery first 8 hours
(e) the gastroduodenal artery (c) 8,000 mL of lactated Ringer’s solution over the
first 8 hours
354. Which of the following pancreatic enzymes is se- (d) 10,000 mL of lactated Ringer’s solution over the
creted in an active form? first 8 hours
(a) lipase
(b) phospholipase A 361. A complication that enteral and parenteral feeding
(c) trypsin have in common is:
(d) elastase (a) increased incidence of sepsis
(b) intestinal villous atrophy
(c) elevated liver transaminases
355. A tracheostomy-related tracheoinnominate fistula
(d) hyperosmolar nonketotic coma
is best managed by:
(e) diarrhea
(a) division of innominate artery and ligation of
both ends 362. The most common organism isolated from bile is:
(b) division of innominate artery and vein graft (a) Escherichia coli
(c) division of innominate artery and polytetraflu- (b) Klebsiella
oroethylene graft (c) Staphylococcus aureus
(d) primary repair of the innominate artery (d) Bacteroides
(e) Proteus
356. Optimum calorie/nitrogen ratio for protein synthe-
sis is: 363. The protein loss equivalent to 100 g of negative
(a) 25–50 : 1 nitrogen balance is:
(b) 50–75 : 1 (a) 75 g
(c) 75–100 : 1 (b) 150 g
(d) 100–150 : 1 (c) 375 g
(d) 525 g
357. Aldosterone: (e) 625 g
(a) stimulates sodium resorption in proximal renal
tubules 364. The most common cause of cancer-related death in
(b) stimulates sodium resorption in distal renal females is:
tubules (a) breast cancer
(c) stimulates potassium resorption in proximal (b) colon cancer
renal tubules (c) ovarian cancer
(d) stimulates potassium resorption in distal renal (d) pancreatic cancer
tubules (e) lung cancer

365. The highest potassium concentration is found in:


358. Hypertension in cases of extra-adrenal pheochro- (a) saliva
mocytoma is caused by: (b) gastric secretion
(a) combined epinephrine and norepinephrine (c) bile
(b) pure epinephrine (d) small intestine
(c) pure norepinephrine (e) pancreatic secretion
(d) rennin, epinephrine, and norepinephrine
366. The risk of regional node metastases in 0.70 mmthick
359. The highest bicarbonate concentration is found in: melanoma is:
(a) saliva (a) 5%
(b) gastric secretions (b) 10%
(c) biliary secretion (c) 20%
(d) pancreatic secretion (d) 30%
(e) small intestine (e) 50%

Dr. wessam alzaidat - General surgery


430 Part XVI. Surgery Review Questions and Answers
367. The initial fluid bolus for an injured child is: 374. In the TNM staging, stage II colorectal cancer is:
(a) 10 mL/kg of lactated Ringer’s solution (a) T1, N1, M0
(b) 20 mL/kg of lactated Ringer’s solution (b) T2, N0, M0
(c) 30 mL/kg of lactated Ringer’s solution (c) T3, N1, M0
(d) 40 mL/kg of lactated Ringer’s solution (d) T4, N0, M0
(e) T3, N1, M1
368. Elevation of urinary 5-hydroxyindole acetic acid is
diagnostic of: 375. Postoperative radiation without chemotherapy:
(a) pheochromocytoma (a) improves survival in stage II colon cancer
(b) Cushing’s disease (b) improves local recurrence in stage II colon
(c) carcinoid syndrome cancer
(d) aldosteronoma (c) improves local recurrence in stage III rectal
cancer
369. Regarding hypertrophic pyloric stenosis: (d) improves local recurrence and survival in stage
(a) more common in males III rectal cancer
(b) most commonly presents in the first week of life
(c) vomit is typically bile stained 376. Risk of irreversible tissue damage in pressure sores
(d) diagnosis should be confirmed with upper gas- is highest with:
trointestinal contrast study (a) constant pressure of 50 mm Hg for 2 hours
(b) constant pressure of 70 mm Hg for 2 hours
370. A colon polyp with the highest malignant potential (c) constant pressure of 100 mm Hg for 30 minutes
is: (d) constant pressure of 150 mm Hg for 20 minutes
(a) 1 cm tubular adenoma
(b) 4 cm hyperplastic polyp 377. In caustic esophageal injury:
(c) 2 cm villous adenoma (a) upper endoscopy is contraindicated in the acute
(d) 2 cm tubulovillous adenoma phase
(e) 3 cm juvenile polyp (b) early induced emesis is helpful in minimizing
the period of mucosal contact
371. Gastric smooth muscle tumors present most com- (c) alkalis cause coagulative tissue necrosis
monly as: (d) acids cause severe gastric rather than eso-
(a) upper gastrointestinal hemorrhage phageal injury
(b) incidental finding on esophagogastroduode-
noscopy 378. Toxic epidermal necrolysis in children is most com-
(c) gastric outlet obstruction monly related to:
(d) intermittent epigastric pain (a) Pseudomonas aeruginosa
(e) weight loss (b) Escherichia coli
(c) Staphylococcus aureus
372. The desired minimum distal margin of resected (d) Streptococcus pneumoniae
rectal cancer is:
(a) 1 cm 379. The most essential step in management of
(b) 2 cm esophageal reflux and Barrett’s change is:
(c) 3 cm (a) Nissen fundoplication
(d) 4 cm (b) close endoscopic surveillance
(e) 5 cm (c) esophageal resection
(d) medical management of reflux disease
373. Regarding gastric adenocarcinoma:
(a) the intestinal type is more common in younger 380. A prominent ―v‖ wave in the right atrial venous
patients waveform indicates:
(b) the intestinal type is often associated with peri- (a) tricuspid valve regurgitation
toneal spread (b) atrial fibrillation
(c) the intestinal type has worse prognosis than (c) pulmonary embolism
diffuse type (d) atrial septal defect
(d) the intestinal type is often distal in location (e) atrial flutter
174. Surgery Review Questions 431
381. The mainstay of treatment of blunt carotid artery 388. The initial step in management of a 25-year-old
injuries is: male with a painless scrotal mass is:
(a) surgical exploration and vein patch graft (a) fine-needle aspiration and cytology
(b) endoluminal stenting (b) abdominal computed tomography scan
(c) catheter thrombolysis (c) observation and repeat examination in 2 weeks
(d) anticoagulation (d) scrotal ultrasound
(e) orchiectomy
382. Regarding the anatomy of the esophagus:
(a) the cervical esophagus lies to the right of the
midline 389. The most common anterior mediastinal tumor is:
(b) the thoracic esophagus is anterior to the aortic (a) thymoma
arch (b) retrosternal goiter
(c) the left vagus nerve passes posterior to the (c) lymphoma
esophagus (d) teratoma
(d) the cervical esophagus is supplied by the infe-
rior thyroid artery
(e) the abdominal esophagus is supplied by the 390. The hallmark of multiple endocrine neoplasia type
right gastric artery 2 syndromes is:
(a) hyperparathyroidism
383. The most common cause of early postoperative (b) pheochromocytoma
death in elective abdominal aortic aneurysm repair (c) medullary thyroid carcinoma
is: (d) pituitary adenoma
(a) acute renal tubular necrosis (e) neural gangliomas
(b) ischemic stroke
(c) acute myocardial infarction
(d) respiratory failure 391. The cranial nerve most commonly injured during
(e) exsanguination from anastomotic disruption carotid endarterectomy is:
(a) IX
384. The initial test in cases of suspected gastroe- (b) X
sophageal reflux should be: (c) XI
(a) Barium swallow (d) XII
(b) upper endoscopy
(c) gastric pH monitoring
(d) esophageal manometry 392. Inadvertent tissue extravasation of intravenous
dopamine is best managed with:
385. Which of the following organisms is slime- (a) topical steroids and elevation
producing? (b) local ice packs and elevation
(a) Candida albicans (c) local infiltration of nitroglycerin
(b) Staphylococcus aureus (d) local infiltration of phentolamine
(c) Staphylococcus epidermidis (e) local infiltration of 1% lidocaine
(d) Klebsiella pneumoniae
(e) Enterobacter aerogenes
393. Hürthle cell thyroid carcinoma:
386. An increased incidence of adenocarcinoma of the (a) is adequately treated with thyroid lobectomy
esophagus is associated with: (b) is seldom bilateral
(a) achalasia (c) often metastasizes to the cervical lymph nodes
(b) lye ingestion (d) is often related to previous neck radiation
(c) Barrett’s esophagus
(d) Plummer-Vinson syndrome
394. Which of the following contributes to the arterial
387. Insulinomas: supply of the thoracic esophagus?
(a) are usually benign (a) right gastric artery
(b) are usually multiple (b) bronchial artery
(c) are commonly located in the tail of the pancreas (c) pulmonary artery
(d) are less common than gastrinoma (d) innominate artery

Dr. wessam alzaidat - General surgery


432 Part XVI. Surgery Review Questions and Answers
395. Regarding postoperative myocardial infarction: 401. Hürthle cell thyroid carcinoma:
(a) it is often associated with chest pain (a) can be readily diagnosed with fine-needle
(b) ischemic injury is more common postopera- aspiration
tively rather than intraoperatively (b) is often associated with previous neck radiation
(c) T-wave changes are the most specific finding for (c) is more aggressive than follicular thyroid carci-
acute myocardial infarction noma
(d) it occurs most commonly within the first 48 (d) typically shows avid 131I uptake
hours after surgery
402. Burn-associated inhalation injury can be ex-cluded
396. The least appropriate test in the initial evaluation of if:
adrenal incidentaloma is: (a) the chest x-ray on admission is normal
(a) magnetic resonance imaging of the abdomen (b) there is no abnormal finding on flexible bron-
(b) fine-needle aspiration biopsy choscopy
(c) follow-up computed tomography of the abdo- (c) the arterial oxygen saturation is over 90%
men in 6 months (d) the FEV1/FVC is normal
(d) 24-hour urinary vanillylmandelic acid (VMA)
level 403. The lateral boundary of a left paraduodenal hernia
(e) serum potassium level is:
(a) the splenic artery
397. A 58-year-old female with a tubo-ovarian abscess (b) the left renal vein
should be treated with: (c) the superior mesenteric vein
(a) antibiotics and laparoscopic drainage (d) the inferior mesenteric artery
(b) antibiotics with hysterectomy and bilateral (e) the inferior mesenteric vein
salpingo-oophorectomy
(c) antibiotics and unilateral salpingo- 404. The carotid body:
oophorectomy (a) is a pressure receptor
(d) antibiotics and computed tomography–guided (b) is an osmoreceptor
drainage (c) is a subintimal structure
(e) antibiotics and observation (d) is located in the adventia
(e) is located in the media
398. Cannon ―a‖ waves on a right atrial waveform tracing
indicate: 405. Barrett’s esophagus:
(a) atrial flutter (a) is commonly congenital in origin
(b) atrial fibrillation (b) is associated with epidermoid carcinoma of the
(c) tricuspid regurgitation esophagus
(d) pulmonary embolism (c) is reversed by a successful antireflux surgery
(e) atrioventricular block (d) is an indication for life-long endoscopic
surveillance
399. Intralobar pulmonary sequestration:
(a) is supplied by the aorta 406. The most common causative agent in nosocomial
(b) is drained by the azygos venous system sinusitis in the ICU is:
(c) is commonly associated with diaphragmatic de- (a) Pseudomonas aeruginosa
fect (b) Staphylococcus aureus
(d) has a separate pleural covering (c) Staphylococcus epidermidis
(d) Streptococcus pneumoniae
400. Specific therapy for heparin-induced thrombocy-
topenia is: 407. The treatment of hyperacute kidney transplant
(a) low-molecular-weight heparin rejection is:
(b) Coumadin (a) administration of OKT-3
(c) dextran-60 (b) administration of massive doses of steroids
(d) aspirin (c) immediate transplant nephrectomy
(e) lepirudin (d) observation and dialysis
174. Surgery Review Questions 433
408. Blunt cardiac injury: 415. The indication for fine-needle aspiration biopsy of
(a) most commonly affects the left ventricle an adrenal incidentaloma is:
(b) most commonly results in ventricular arrhyth- (a) suspected pheochromocytoma
mias (b) suspected aldosteronoma
(c) can be reliably excluded if cardiac enzymes are (c) suspected adrenal carcinoma
normal (d) suspected adrenal metastases
(d) should be suspected if admission electrocardio-
gram is abnormal 416. Male breast cancer:
(a) has a peak incidence at the age of 40 years
409. A characteristic of Buerger’s disease is: (b) is typically hormonally dependent
(a) superficial migratory thrombophlebitis (c) typically presents with bloody nipple discharge
(b) female predominance (d) is seldom an indication for tamoxifen therapy
(c) severe involvement of the aortoiliac segment
(d) the disease is restricted to the lower extremities 417. A non-anion gap metabolic acidosis is associated
with:
410. Recurrence of thyroid carcinoma after definitive (a) diabetic ketoacidosis
treatment is best detected by: (b) hemorrhagic shock
(a) ultrasound of the neck (c) excessive sodium chloride administration
(b) thyroid-stimulating hormone level measure- (d) uremia
ment (e) ingestion of ethylene glycol
(c) thyroglobin serum measurement
(d) computed tomography scan of the neck 418. Inverse ratio ventilation:
(e) triiodothyronine/thyroxine measurement (a) decreases mean airway pressure and increases
intrathoracic pressure
(b) decreases mean airway pressure and decreases
411. The primary regulator of aldosterone secretion is:
intrathoracic pressure
(a) angiotensin II
(c) increases mean airway pressure and increases
(b) serum K level
intrathoracic pressure
(c) adrenocorticotropic hormone
(d) increases mean airway pressure and decreases
(d) prostaglandins
intrathoracic pressure

412. The optimum management of a T4 breast cancer is: 419. Duodenal atresia:
(a) modified radical mastectomy and radiation (a) is caused by intrauterine mesenteric vascular
(b) chemoradiation only accident
(c) simple mastectomy and radiation (b) commonly exhibits normal muscular wall with
(d) chemotherapy followed by mastectomy and a mucosal web
radiation (c) is seldom associated with normal passage of
(e) radiation therapy only meconium at birth
(d) is rarely associated with other congenital
413. The diagnosis of esophageal achalasia is confirmed anomalies
by:
(a) bird’s beak appearance on barium swallow 420. Angiotensin-converting enzyme is produced by:
(b) subatmospheric intraluminal esophageal pres- (a) type I pneumocytes
sure on manometry (b) type II pneumocytes
(c) endoscopic evidence of distal esophagitis (c) hepatocytes
(d) failure of lower esophageal sphincter relaxation (d) juxtaglomerular cells
on manometry (e) vascular endothelial cells
(e) hyperperistalsis of the body of the esophagus
421. Hereditary nonpolyposis colorectal cancer:
414. Factor VIII–related antigen is a marker of: (a) is an autosomal recessive disorder
(a) Kaposi’s sarcoma (b) is associated with higher incidence of ovarian
(b) melanoma cancer
(c) postmastectomy angiosarcoma (c) is associated with higher incidence of endome-
(d) desmoid tumors trial cancer
(e) Merkel cell carcinoma (d) is mostly left-side colon cancer

Dr. wessam alzaidat - General surgery


434 Part XVI. Surgery Review Questions and Answers
422. Auto-positive end-expiratory pressure in mechani- 429. Appropriate management of radiation mastitis is:
cal ventilation is most likely to develop with: (a) local heat application
(a) high rates and prolonged I : E ratio (b) danazol therapy
(b) high rates and decreased I : E ratio (c) therapeutic breast massage
(c) decreased rate and decreased I : E ratio (d) simple mastectomy
(d) decreased rate and prolonged I : E ratio (e) pentoxifylline therapy
430. Optimum therapy for effort thrombosis of the axil-
423. Initial management of T4 invasive lobular breast lary vein is:
carcinoma is: (a) therapeutic heparin followed by Coumadin for
(a) tamoxifen therapy 3 months
(b) modified radical mastectomy (b) therapeutic heparin followed by Coumadin for
(c) radiation therapy to the breast and ipsilateral 6 months
axilla (c) thrombolysis, anticoagulation, and possible first
(d) neoadjuvant chemotherapy rib resection
(d) thrombolysis, anticoagulation, and balloon an-
gioplasty
424. The fluid of chylothorax is composed of: (e) thrombectomy, anticoagulation, and stent place-
(a) pure fat ment
(b) fat and neutrophils 431. Drooling from the corner of the mouth after sub-
(c) fat and lymphocytes mandibular gland excision is due to:
(d) fat and macrophages (a) injury of the lingual nerve
(b) injury of the ansa cervicalis
(c) injury of the hypoglossal nerve
425. The most common cause of renovascular hyperten-
(d) injury of the marginal mandibular nerve
sion is:
(a) aneurysm of the renal artery 432. The component of blood transfusion responsible for
(b) fibromuscular hyperplasia immunosuppression is:
(c) renal artery calcinosis (a) red cells
(d) renal artery atheroma (b) immunoglobulins
(c) white blood cells
(d) platelets
426. Merkel cell carcinoma:
(a) is highly radiosensitive 433. In therapeutic immunosuppression, rapamycin:
(b) has the best prognosis when it occurs on the (a) blocks interleukin-1 production
trunk (b) blocks interleukin-1 action
(c) seldom spreads to regional lymph nodes (c) blocks interleukin-2 production
(d) is less likely than melanoma to recur after local (d) blocks interleukin-2 action
excision 434. Gallstones diagnosed during pregnancy:
(a) may resolve spontaneously after delivery
(b) are the most common cause of jaundice during
427. A complication of tacrolimus therapy is: pregnancy
(a) new-onset diabetes (c) are seldom the cause of recurrent acute chole-
(b) upper gastrointestinal bleeding cystitis during the same pregnancy
(c) thrombocytopenia (d) are most commonly pigment stones
(d) cardiac arrhythmia
(e) bronchospasm 435. Trauma cesarean section is indicated if the fetus is
viable and:
(a) the mother is unstable and cardiac arrest is anti-
428. Extralobar pulmonary sequestration: cipated
(a) typical presents with repeated pulmonary infec- (b) 5 minutes of cardiopulmonary resuscitation were
tions successful
(b) is drained by the pulmonary veins (c) 5 minutes of cardiopulmonary resuscitation were
(c) is supplied by the aorta unsuccessful
(d) is more common on the right side (d) cardiopulmonary resuscitation has just been
(e) is commonly connected with the bronchial tree initiated
174. Surgery Review Questions 435
436. The single most common abdominal operation for 443. An 80-year-old male is confused, lethargic, and has
the elderly is: tonic spasms. He has been receiving intravenous 5%
(a) adhesiolysis for small bowel obstruction dextrose in water in one-half normal saline for 3
(b) closure of perforated duodenal ulcer days after right hemicolectomy. The most likely
(c) resection of colon cancer cause is:
(d) cholecystectomy (a) hypokalemia
(e) appendectomy (b) hyperkalemia
(c) hypocalcemia
437. An age-related change in respiratory functions is: (d) hypernatremia
(a) increased total lung capacity (TLC) (e) hyponatremia
(b) decreased total lung capacity (TLC)
(c) decreased functional residual capacity (FRC) 444. A physiologic change in pregnancy is:
(d) increased residual volume (RV) (a) increased gallbladder contractility
(e) increased vital capacity (VC) (b) elevated alkaline phosphatase
(c) elevated alanine aminotransferase (AST)/
438. Fetal death during pregnancy is most commonly due aspartate aminotransferase (ALT)
to: (d) direct hyperbilirubinemia
(a) abruptio placentae (e) indirect hyperbilirubinemia
(b) subarachnoid hemorrhage
(c) penetrating fetal injury 445. In the elderly:
(d) maternal demise (a) the myocardium is oversensitive to the effect of
catecholamines
439. A physiologic change of aging is: (b) systolic function is more impaired than diastolic
(a) increased total body water function
(b) decreased antidiuretic hormone secretion (c) diastolic function is more impaired than systolic
(ADH) function
(c) decreased aldosterone secretion (d) systolic and diastolic functions are equally
(d) exaggerated thirst response impaired
(e) decreased atrial natriuretic peptide (ANP)
446. A Crohn’s disease–related internal fistula:
440. Which of the following is contraindicated during
(a) most commonly presents with florid sepsis
pregnancy for treating breast cancer?
(a) methotrexate (b) most commonly presents with Crohn’s disease
flare–up
(b) 5-fluorouracil
(c) doxorubicin (c) requires excision of both organs involved for
surgical treatment
(d) cyclophosphamide
(d) most commonly involves small bowel to small
bowel fistulation
441. In the elderly:
(e) requires surgical intervention as soon as the
(a) there is decreased insensible water loss
diagnosis is made
(b) there is increased antidiuretic hormone re-
sponse
(c) there is increased volume of distribution of 447. In inflammation, early ―rolling‖ of neutrophils on
water-soluble medications endothelium is a function of:
(d) there is decreased volume of distribution of (a) selectins
lipid-soluble medications (b) N-cahedrins
(c) immunoglobulin superfamily
442. During pregnancy: (d) integrins
(a) acute appendicitis is more common than in the (e) complement activation
nonpregnant state
(b) intestinal obstruction is most commonly caused 448. A paraesophageal hernia:
by adhesions (a) is a type I hiatal hernia
(c) laparoscopic cholecystectomy is contraindi- (b) is most common in men 50 years of age
cated (c) may present with chronic anemia
(d) leukocytosis is often indicative of a surgical (d) is often associated with reflux
abdominal pathology (e) requires surgery only if symptomatic

Dr. wessam alzaidat - General surgery


436 Part XVI. Surgery Review Questions and Answers
449. The most common cause of large bowel obstruction 456. Eight hours after treatment for a scald injury, an
is: infant has a temperature of 40°C and a white blood
(a) colorectal cancer count of 5,000/mm3. The burn wound is clean. The
(b) Crohn’s colitis most likely diagnosis is:
(c) diverticulitis (a) Cytomegalovirus infection
(d) adhesions (b) clostridial wound infection
(e) volvulus (c) toxic shock syndrome
(d) Pseudomonas wound infection
450. Hypophosphatemia is associated with: (e) Pneumocystis pneumoniae
(a) increased hemoglobin p50, respiratory failure,
and encephalopathy
(b) decreased hemoglobin p50, hemolysis, and res- 457. The medial boundary of the femoral canal is:
piratory failure (a) the femoral vein
(c) respiratory failure, hypothyroidism, and hemol- (b) the femoral artery
ysis (c) the lacunar ligament
(d) ataxia, cardiomyopathy, and hypothyroidism (d) the inguinal ligament
(e) the femoral nerve
451. Preoperative radiotherapy for rectal adenocarci-
noma:
(a) improves survival 458. Hemorrhagic dermal bullae are characteristic of
(b) increases postoperative morbidity wound infection caused by:
(c) down-stages tumor in up to 50% of cases (a) coagulase-negative Staphylococcus
(d) is less effective than postoperative radiation (b) Streptococcus pyogenes
(c) Clostridium perfringens
452. The principal side effect of ganciclovir therapy is: (d) Clostridium tetani
(a) bone marrow suppression (e) Eikenella
(b) elevated liver enzymes
(c) acute pancreatitis
459. A marker with prognostic significance in cases of
(d) acute renal failure
seminoma is:
(e) prolonged ileus
(a) -fetoprotein
453. In contrast to ulcerative colitis, Crohn’s disease: (b) human chorionic gonadotropin-
(a) is not associated with increased cancer risk (c) alkaline phosphatase of hepatic origin
(b) may spare the rectum (d) testosterone level
(c) is more commonly associated with sclerosing (e) serum lactate dehydrogenase
cholangitis
(d) is more commonly associated with toxic mega- 460. Collagen synthesis in the actively healing wound is
colon best assessed by:
454. The negative predictive value of a test is: (a) glutamine content
(a) the proportion of patients with the disease who (b) arginine content
have a positive test (c) hydroxyproline content
(b) the proportion of patients without the disease (d) alanine content
who have a negative test
(c) the proportion of patients without the disease 461. The main motor nerve supply to the urinary bladder
who have a positive test is:
(d) the proportion of patients with a positive test (a) the pelvic nerve
who have the disease (b) the hypogastric nerve
(e) the proportion of patients with negative test (c) the pudendal nerve
who do not have the disease (d) the presacral nerve
455. Radiation enteritis of the large bowel most com-
monly affects: 462. Compared with hepatitis B virus, hepatitis C virus:
(a) the cecum (a) is an RNA virus
(b) the splenic flexure (b) less commonly results in persistent viremia
(c) the sigmoid (c) can be prevented with effective vaccination
(d) the rectum (d) can be transmitted via the fecal-oral route
174. Surgery Review Questions 437
463. The positive predictive value of a test is: 470. The thoracodorsal nerve:
(a) the proportion of patients with the disease who (a) is purely motor
have a positive test (b) supplies motor innervation to the serratus ante-
(b) the proportion of persons without the disease rior muscle
who have a negative test (c) injury during axillary dissection will lead to
(c) the proportion of persons with a positive test winged scapula
who have the disease (d) injury will result in loss of sensation on the
(d) the proportion of persons with a negative test medial surface of the upper arm
who do not have the disease
471. Hypocalcemia is a complication of chemotherapy
464. The circulating level of which cytokine can be used with:
as prognostic marker in sepsis? (a) cyclophosphamide
(a) interleukin-1 (b) vincristine
(b) interleukin-2 (c) methotrexate
(c) interleukin-6 (d) mithramycin
(d) interleukin-8 (e) Adriamycin
(e) tumor necrosis factor-
 472. Male breast cancer most commonly presents with:
465. Nitric oxide: (a) mastodynia
(a) is a potent vasoconstrictor (b) bleeding per nipple
(b) prevents platelets aggregation (c) breast mass only
(c) requires cyclic adenosine monophosphate for (d) breast mass and bleeding per nipple
its actions (e) breast mass with ulceration
(d) is normally stored in endothelial cells
473. Which of the following is a clear indication of
466. The most common cause of facial nerve paralysis is: surgery for necrotizing enterocolitis?
(a) parotid surgery (a) generalized gas distension of the intestine
(b) faciomaxillary injury (b) bleeding rectum
(c) Bells’ palsy (c) pneumatosis intestinalis
(d) pleomorphic adenoma of the parotid gland (d) gas in the portal vein
(e) carcinoma of the parotid gland (e) pneumoperitoneum

467. Most renal absorption of sodium takes place in: 474. An infected dog bite is most readily treated with:
(a) the proximal tubules (a) gentamicin
(b) the loop of Henle (b) ampicillin
(c) the distal tubules (c) vancomycin
(d) the collecting ducts (d) clindamycin

468. Distinction between hemorrhagic and cardiogenic 475. During CO2 pneumoperitoneum:
shock can be based on: (a) the mean arterial pressure increases
(a) level of urinary sodium (b) the mean arterial pressure decreases
(b) ventricular filling pressures (c) the systemic vascular resistance increases
(c) system vascular resistance (d) the systemic vascular resistance decreases
(d) serum lactate level (e) the cardiac output increases
(e) mixed venous oxygen saturation
476. In diabetic foot ulcers:
469. Renin–angiotensin system is activated by: (a) neuropathy is restricted to somatic nerves
(a) hyponatremia (b) the heel is the most common location
(b) hypernatremia (c) motor neuropathy mostly affects toe flexors
(c) hypokalemia (d) cold, dry foot is a classic finding
(d) hypocalcemia (e) the ankle–brachial index is an accurate measure
(e) hypercalcemia of the degree of ischemia

Dr. wessam alzaidat - General surgery


438 Part XVI. Surgery Review Questions and Answers
477. A sign of early sepsis is: 485. The most common fatal infection in burn victims is:
(a) metabolic alkalosis (a) pneumonia
(b) metabolic acidosis (b) venous line–related sepsis
(c) respiratory alkalosis (c) burn wounds sepsis
(d) respiratory acidosis (d) urinary tract infection

478. In contrast to a keloid, a hypertrophic scar: 486. The cardiac ejection fraction:
(a) is more likely to be familial (a) is normally 45%
(b) may subside spontaneously (b) is an accurate indicator of cardiac dysfunction
(c) may develop in delayed fashion years after in the elderly
initial injury (c) is increased in mitral stenosis
(d) often extends beyond the limits of the original (d) is decreased in mitral incompetence
wound (e) is increased in ventricular septal defect
479. In transplant recipients, there is an increased inci- 487. After 3 years, vein graft failure in the lower extrem-
dence of: ities is mostly caused by:
(a) colon cancer (a) technical complications
(b) anal cancer (b) anastomotic aneurysm
(c) lung cancer (c) atherosclerosis of the graft
(d) breast cancer (d) intimal thickening
(e) prostate cancer
488. The predominant parathyroid pathology in multiple
480. The most common cause of small bowel bleeding in
endocrine neoplasia is:
adults is:
(a) a single parathyroid adenoma
(a) Crohn’s disease
(b) multiple parathyroid adenomas
(b) arteriovenous malformation
(c) parathyroid hyperplasia
(c) leiomyosarcoma
(d) parathyroid carcinoma
(d) Meckel’s diverticulum
(e) carcinoid neoplasms 489. In renal artery stenosis, angiotensin-converting
enzyme inhibitors will:
481. A type IV hiatal hernia:
(a) decrease glomerular filtration rate
(a) is a sliding hiatus hernia
(b) increase glomerular filtration rate
(b) is a traumatic diaphragmatic hernia
(c) result in dilation of the afferent arterioles
(c) is a hernia that contains the stomach
(d) result in constriction of the afferent arterioles
(d) is a hernia that contains parts of the intestine
(e) result in constriction of the efferent arterioles
and colon

482. Isosulfan blue injection for sentinel node biopsy 490. The appropriate agent for prophylaxis in elective
may result in: colectomy is:
(a) cardiac arrhythmia (a) Flagyl
(b) inaccurate pulse oximetry (b) imipenem
(c) malignant hyperthermia (c) vancomycin
(d) hypercapnia (d) cefotetan
(e) skin sloughing (e) cefazolin

483. The most common complication of gastric ulcer is: 491. Which of the following is a dopamine antagonist?
(a) malignant transformation (a) phentolamine
(b) perforation (b) propofol
(c) upper gastrointestinal bleeding (c) haloperidol
(d) gastric outlet obstruction (d) clonidine

484. The preferred neuromuscular-blocking agent in a 492. Peripheral neuropathy is the main side effect of:
liver failure patient is: (a) cyclophosphamide
(a) vecuronium (b) vincristine
(b) atracurium (c) methotrexate
(c) pancuronium (d) mithramycin
(d) pipecuronium (e) Adriamycin
174. Surgery Review Questions 439
493. In liver transplantation, biliary complications are 499. The optimum management for stage II thyroid lym-
most commonly related to: phoma is:
(a) adequacy of hepatic venous outflow (a) total thyroidectomy
(b) adequacy of portal venous flow (b) total thyroidectomy and radical neck dissection
(c) adequacy of hepatic arterial flow (c) total thyroidectomy and chemotherapy
(d) length of the donor’s common bile duct (d) chemoradiation
(e) cervical radiation
494. The optimum management of aspiration pneu-
monitis is: 500. The most reliable indicator of successful ventilatory
(a) endotracheal intubation and mechanical ven- weaning is:
tilation (a) PO2 100 with FiO2 of 40%
(b) endotracheal intubation, mechanical ventila- (b) PCO2 40 mm Hg
tion, and bronchial lavage (c) negative inspiratory force less (more negative)
(c) endotracheal intubation, ventilation, bronchial than 30 cm HO2
lavage, and steroids (d) f/Vt 100
(d) endotracheal intubation, ventilation, bronchial
lavage, and antibiotics 501. Chronic allograft rejection:
(a) can be prevented with adequate immun-
495. Malignant hyperthermia: osuppression
(a) is triggered by the stress of surgery or anesthe- (b) is the main cause of death after liver trans-
sia only plantation
(b) is a contraindication to future general anesthe- (c) is more common with liver than heart trans-
sia plantation
(c) can be prevented by perioperative calcium (d) is more common with kidney than liver trans-
channel blockers plantation
(d) is associated with intraoperative rise of end
tidal CO2
502. The most accurate test to determine the need for
neoadjuvant therapy in esophageal carcinoma is:
496. The long thoracic nerve:
(a) endoluminal ultrasound
(a) is purely motor
(b) chest computed tomography scan
(b) provides sensation to the medial wall of the
(c) bronchoscopy
axilla
(d) barium swallow
(c) provides sensation to the medial aspect of the
(e) esophagogastroduodenoscopy
upper arm
(d) supplies motor innervation to the latissimus
dorsi muscle 503. The most characteristic metabolic abnormality in
(e) its injury results in weakness of arm abduc- glucagonoma is:
tion (a) hypoglycemia
(b) hypoaminoacidemia
497. The likelihood of malignancy in cystic pancreatic (c) hypocholesterolemia
neoplasm is related to: (d) hypercholesterolemia
(a) high glycogen content
(b) mucin content 504. A local anesthetic that can be safely administered
(c) sunburst appearance on computed tomography with tetracaine allergy is:
scan (a) lidocaine
(d) previous history of pancreatitis (b) cocaine
(c) procaine
498. Colovesical fistula: (d) chloroprocaine
(a) is most commonly caused by colon cancer
(b) is more common in females than in males 505. Clonidine is:
(c) is readily diagnosed with barium enema in most (a) an 1-agonist
cases (b) an 1-antagonist
(d) requires segmental colectomy and partial cys- (c) an 2-agonist
tectomy for treatment (d) a 1-agonist
(e) most commonly presents with pneumaturia (e) a 2-agonist

Dr. wessam alzaidat - General surgery


440 Part XVI. Surgery Review Questions and Answers
506. A definitive diagnosis of inflammatory breast cancer 513. A pulmonary embolus is associated with:
is provided by: (a) decrease in pulmonary capillary wedge pressure
(a) unique mammographic appearance (b) decrease in mean pulmonary artery pressure
(b) finding tumor emboli in dermal lymphatics (c) decrease in central venous pressure
(c) finding extensive inflammatory cell infiltration (d) decrease in dead space/tidal volume ratio
of the tumor
(d) elevated white cell count, fever, axillary 514. Pyoderma gangrenosum is typically associated with:
lymphadenopathy (a) advanced colonic adenocarcinoma
(e) ultrasound appearance of cavitation (b) rectal carcinoid
(c) ulcerative colitis
(d) villous adenoma of the colon
507. Tissue loss almost always results from ligation of
the: 515. Popliteal artery aneurysms:
(a) popliteal artery (a) commonly present with rupture
(b) common femoral artery (b) seldom result in limb ischemia
(c) superficial femoral artery (c) are the most common peripheral arterial aneu-
(d) brachial artery rysms
(e) portal vein (d) require operation only if they result in em-
bolization
508. Male breast cancer: (e) are most commonly false aneurysms
(a) is associated with the BRCA-1 gene mutation
(b) can only be of ductal origin 516. After vigorous exercise, an athlete develops pain on
(c) is seldom hormone receptor positive dorsiflexion of the foot and decreased sensation in
(d) develops at a much younger age than female the first web space. Appropriate action should be:
breast cancer (a) color duplex scan and immediate hepariniza-
tion
(b) leg elevation, ice packs, and nonsteroidal anti-
509. Keloid formation has been associated with an
inflammatory medications
increased amount of:
(c) immediate fasciotomy
(a) transforming growth factor-
(d) immediate femoral arteriogram
(b) platelet-derived growth factor
(c) epidermal growth factor 517. The most common presentation of testicular cancer
(d) basic fibroblast growth factor is:
(e) tumor necrosis factor- (a) a painless scrotal mass
 (b) acute testicular pain
510. The anteroposterior anatomic relationship at the (c) a secondary hydrocele
renal hilum is: (d) gynecomastia
(a) vein—artery—ureter (e) retroperitoneal lymphadenopathy
(b) artery—vein—ureter
(c) ureter—artery—vein 518. Which of the following is most likely to cause a
(d) ureter—vein—artery false-positive fecal occult blood test?
(a) oral iron therapy
(b) Coumadin therapy
511. Endophthalmitis is characteristic of:
(c) aspirin therapy
(a) Escherichia coli sepsis
(d) nonsteroidal anti-inflammatory medications
(b) toxic shock syndrome
(e) dietary peroxidases
(c) systemic candidiasis
(d) facial necrotizing fasciitis 519. Aldosteronoma is associated with:
(a) hypertension, hypokalemia, high aldosterone
512. The most important determinant of survival of and high renin levels
retroperitoneal sarcoma is: (b) hypertension, hyperkalemia, high aldosterone
(a) the use of adjuvant chemotherapy and low renin levels
(b) the size of the primary tumor (c) hypertension, hyperkalemia, high aldosterone
(c) the histologic type of the primary and high renin levels
(d) the use of intraoperative radiotherapy (d) hypertension, hypokalemia, high aldosterone
(e) complete surgical resection and low renin levels
174. Surgery Review Questions 441
520. A PO2 of 90 torr, PCO2 of 28 torr, and pH of 7.16 527. Neurogenic bladder dysfunction with intact bladder
on room air are indicative of: sensation is associated with:
(a) hypovolemic shock (a) nerve injury with abdominoperineal resection
(b) alveolar hypoventilation (b) cauda equina lesion
(c) prolonged nasogastric suctioning (c) myelomeningocele
(d) hyperventilation (d) paraplegia
(e) cerebrovascular accident
521. Idiopathic thrombocytopenic purpura is most likely 528. Tumor necrosis factor- antagonist is useful in
to respond to splenectomy if: treating:
(a) the spleen is enlarged (a) disseminated intravascular coagulopathy
(b) the patient is a female (b) septic shock
(c) the disease is chronic (c) Crohn’s disease
(d) the disease responded to steroids (d) metastatic melanoma

522. A marker that would distinguish nonseminoma 529. An 83-year-old female presents with vomiting and
from seminoma is: abdominal distension. She complains of pain in the
(a) lactate dehydrogenase medial aspect of the right thigh, and a palpable lump
(b) alkaline phosphatases can be felt on the right side on rectal examination.
(c) human chorionic gonadotropin The appropriate action is:
(d) -fetoprotein (a) nasogastric tube, intravenous fluids, and
observation
(b) flexible sigmoidoscopy and drainage of the
523. Positive end-expiratory pressure: rectal mass
(a) increases cardiac output (c) urgent laparotomy
(b) decreases functional residual capacity (d) Gastrografin study with small bowel follow-
(c) increases right to left shunting through
(d) lowers PCO2 (e) right groin exploration and hernia repair
(e) decreases alveolar–arterial oxygen gradient
530. A normal-sized spleen is found in:
(a) idiopathic thrombocytopenic purpura
524. Estrogen therapy for postmenopausal women is (b) myelodysplasia
associated with: (c) Gaucher’s disease
(a) increased incidence of thrombophlebitis (d) thalassemia
(b) increased incidence of hepatic adenoma (e) schistosomiasis
(c) increased incidence of endometrial cancer
(d) increased incidence of breast cancer 531. Neurogenic bladder after extensive pelvic surgery is
characterized by:
525. A 10-year-old boy comes from a family of ―bleed- (a) painful urine retention
ers.‖ His coagulation profile shows: prolonged (b) large bladder capacity with overflow inconti-
bleeding time, normal prothrombin time, and pro- nence
longed partial thromboplastin time. His platelet (c) small residual volume and uninhibited bladder
count is 150,000/mm3. The most likely diagnosis is: contractions
(a) idiopathic thrombocytopenic purpura (d) autonomic dysreflexia
(b) thrombotic thrombocytopenic purpura
532. Regarding cytokines:
(c) hemophilia
(a) serum level is not related to the severity of
(d) Christmas disease
illness
(e) von Willebrand’s disease
(b) are stored intracellularly as preformed mole-
cules
526. A single organism is usually the causative agent in: (c) are produced by a limited number of specific
(a) pelvic inflammatory disease cells
(b) perforated diverticulitis (d) most commonly function in an endocrine
(c) acute cholecystitis fashion
(d) primary peritonitis (e) low serum levels are normally detected in heal-
(e) diabetic foot infections thy individuals

Dr. wessam alzaidat - General surgery


442 Part XVI. Surgery Review Questions and Answers
533. Hypothermic coagulopathy is characterized by: 538. The most common posterior mediastinal mass in
(a) normal prothrombin time and normal partial children is:
thromboplastin time (a) neuroblastoma
(b) normal prothrombin time and prolonged partial (b) teratoma
thromboplastin time (c) lymphoma
(c) prolonged prothrombin time and normal partial (d) pheochromocytoma
thromboplastin time
(d) depletion of factor VII
(e) depletion of factor VIII 539. In the absence of sepsis, glucose intolerance with
total parenteral nutrition may indicate:
(a) copper deficiency
534. A patient with diagnosed pseudomembranouscolitis
(b) zinc deficiency
is developing a worsening clinical picture and is taken
(c) magnesium deficiency
for urgent laparotomy. The appropriate sur- gical
(d) chromium deficiency
procedure should include:
(a) segmental colectomy and exteriorization of the
ends
540. The most common presentation of gastric cancer is:
(b) total abdominal colectomy
(a) hematemesis and melena
(c) diverting proximal colostomy
(b) acute abdominal pain due to perforation
(d) segmental colectomy and primary anastomosis
(c) vague abdominal pain and weight loss
(e) segmental colectomy, primary anastomosis, and
(d) an epigastric mass
diverting colostomy

535. The specificity of a test is: 541. In burn victims, the finding most indicative of
(a) the proportion of patients with the disease who inhalation injury is:
have a positive test (a) singed nasal hair
(b) the proportion of patients without the disease (b) soot around the mouth
who have a negative test (c) dyspnea
(c) the proportion of patients with a positive test (d) carbonaceous sputum
who have the disease
(d) the proportion of patients with a negative test
who do not have the disease 542. A child with anaphylactoid purpura develops an
acute colicky abdominal pain and bleeding per
rectum. The most likely diagnosis is:
536. A 45-year-old-male complains of severe chest pain
(a) perforated duodenal ulcer
after a diagnostic upper endoscopy. He has crepitus
(b) bleeding Meckel’s diverticulum
on palpation of his neck. The next step in manage-
(c) enterocolitis with perforation
ment should be:
(d) intussusception
(a) repeated upper endoscopy
(b) immediate endotracheal intubation
(c) Gastrografin swallow
543. The most reliable means of preoperative nutritional
(d) administration of aspirin and sublingual
assessment is:
nitrite
(a) clinical history of weight loss
(e) admission to ICU, administration of intra-
(b) serum albumin level
venous narcotics, and observation
(c) impaired cell-mediated immunity
(d) triceps skinfold measurement
537. Familial adenomatous polyposis:
(a) is caused by inactivation of a tumor suppressor
gene 544. In contrast to adults, fetal wound healing:
(b) has normal life expectancy after prophylactic (a) has a higher content of type III collagen
colectomy (b) has a higher level of transforming growth
(c) is associated with increased risk of right-side factor-
colon cancer (c) has an exaggerated inflammatory phase
(d) is inherited as an autosomal recessive trait (d) has much less hyaluronic acid content
174. Surgery Review Questions 443
545. The greatest amount of maintenance intravenous 551. The sensitivity of the test is:
fluids is required for: (a) the proportion of patients with the disease who
(a) 21-year-old male athlete have a positive test
(b) 55-year-old obese male office worker (b) the proportion of patients without the disease
(c) 21-year-old housewife who have a negative test
(d) 75-year-old female with recent weight loss (c) the proportion of patients with a positive test
who have the disease
(d) the proportion of patients with a negative test
546. The most accurate test to detect subclinical who do not have the disease
hypothyroidism is:
(a) radioactive iodine uptake 552. Regarding an amebic liver abscess:
(b) thyroid-stimulating hormone level (a) surgical drainage is usually required
(c) total thyroxine level (b) negative stool testing for amebiasis rules out the
(d) free thyroxine level disease
(c) it should be drained percutaneously under com-
puted tomography guidance
547. A 28-year-old male has a closed head injury, pul- (d) it is treated with metronidazole
monary contusion, grade III splenic injury, and
closed femoral shaft fracture. The ideal manage- 553. An infant has been having episodic coughing for 48
ment of his fracture is: hours. On examination, he is wheezing with
(a) external fixation decreased aeration of the left chest. A chest x-ray
(b) skeletal traction shows an overinflated left lung. The next step in
(c) intramedullary nailing within 24 hours of injury management should be:
(d) intramedullary nailing 1 week after the injury (a) insertion of left-side chest tube
(e) use of metal plates and screws (b) insertion of right-side chest tube
(c) endotracheal intubation
(d) administration of steroid inhaler and observa-
548. Accessory spleens are most commonly found in: tion
(a) idiopathic thrombocytopenic purpura (e) rigid bronchoscopy
(b) thrombotic thrombocytopenic purpura
(c) schistosomiasis 554. A 40-year-old female has a 4-cm hemangioma in the
(d) hereditary spherocytosis right lobe of the liver on computed tomography
scan. She is asymptomatic. Appropriate action
should be:
549. The effectiveness of prophylactic antibiotics in (a) fine-needle biopsy
surgery is mostly related to the: (b) arrangement for elective resection
(a) use of broad-spectrum agents (c) no further action
(b) continuation of antibiotics for 24 hours after (d) angiographic embolization
surgery
(c) timing of initial administration 555. A non-anion gap metabolic acidosis is associated
(d) use of two synergistic agents with:
(e) use of bactericidal agents (a) methane intoxication
(b) large amount of saline resuscitation
(c) diabetic ketoacidosis
550. A 6-year-old boy has a right undescended testicle. (d) cardiogenic shock
His parents should be advised that: (e) hemorrhagic shock
(a) right orchiectomy should be performed
(b) a course of endocrine treatment is advisable 556. The gold standard for the diagnosis of pelvic inflam-
(c) orchiopexy is performed to improve spermato- matory disease is:
genesis and prevent cancer (a) vaginal microbiology swab
(d) orchiopexy is performed but with no effect on (b) pelvic ultrasound
spermatogenesis or cancer prevention (c) laparoscopy
(e) the right testicle is probably atrophic and can be (d) pelvic computed tomography scan
left undisturbed (e) endometrial biopsy

Dr. wessam alzaidat - General surgery


444 Part XVI. Surgery Review Questions and Answers
557. A 60-year-old alcoholic male presents with severe 558. Mediastinal granulomas may be associated with:
chest pain after repeated vomiting. A chest x-ray (a) epiphrenic esophageal diverticulum
shows a small left pleural effusion. The next step in (b) Zenker’s diverticulum
management is: (c) esophageal traction diverticulum
(a) obtain cardiac enzymes and admit to coronary (d) achalasia
care unit
(b) insert nasogastric tube, administer intravenous
fluids, and observe
(c) insert nasogastric and left-side chest tube and
antibiotics
(d) administer Gastrografin swallow test
(e) perform upper endoscopy
175
Surgery Review Answers

1. d 40. c 79. b 118. e 157. b 196. d


2. c 41. a 80. c 119. c 158. c 197. d
3. b 42. e 81. c 120. d 159. c 198. c
4. c 43. c 82. c 121. c 160. a 199. a
5. d 44. e 83. a 122. c 161. c 200. a
6. b 45. b 84. a 123. a 162. e 201. b
7. b 46. c 85. c 124. d 163. a 202. d
8. b 47. d 86. b 125. b 164. a 203. d
9. b 48. b 87. e 126. b 165. b 204. d
10. d 49. d 88. b 127. d 166. d 205. b
11. c 50. c 89. a 128. a 167. e 206. a
12. a 51. d 90. b 129. d 168. a 207. c
13. a 52. d 91. a 130. a 169. c 208. e
14. b 53. c 92. a 131. d 170. d 209. a
15. d 54. a 93. a 132. e 171. b 210. c
16. a 55. c 94. b 133. d 172. c 211. d
17. c 56. a 95. a 134. c 173. a 212. e
18. a 57. c 96. a 135. e 174. d 213. d
19. c 58. b 97. c 136. a 175. c 214. d
20. c 59. e 98. c 137. e 176. d 215. a
21. d 60. a 99. d 138. b 177. c 216. e
22. d 61. b 100. c 139. c 178. b 217. c
23. e 62. c 101. b 140. a 179. d 218. b
24. a 63. e 102. a 141. c 180. a 219. a
25. c 64. a 103. b 142. b 181. c 220. c
26. c 65. a 104. c 143. d 182. a 221. d
27. d 66. a 105. a 144. a 183. e 222. a
28. c 67. e 106. e 145. e 184. a 223. b
29. c 68. c 107. b 146. e 185. b 224. b
30. e 69. b 108. d 147. b 186. b 225. b
31. e 70. e 109. a 148. e 187. c 226. b
32. b 71. b 110. d 149. b 188. b 227. d
33. b 72. c 111. c 150. a 189. c 228. c
34. e 73. d 112. d 151. a 190. b 229. c
35. a 74. a 113. d 152. a 191. e 230. d
36. c 75. c 114. b 153. d 192. a 231. d
37. a 76. b 115. d 154. c 193. b 232. d
38. c 77. d 116. c 155. b 194. c 233. c
39. a 78. e 117. b 156. b 195. d 234. d

Dr. wessam alzaidat - General surgery


446 Part XVI. Surgery Review Questions and Answers
235. b 289. d 343. b 397. b 451. b 505. c
236. b 290. c 344. e 398. e 452. a 506. c
237. c 291. b 345. b 399. a 453. b 507. a
238. a 292. e 346. b 400. e 454. e 508. b
239. c 293. c 347. b 401. c 455. d 509. a
240. d 294. d 348. c 402. b 456. c 510. a
241. c 295. c 349. a 403. e 457. c 511. c
242. b 296. b 350. d 404. d 458. b 512. e
243. d 297. b 351. d 405. d 459. e 513. a
244. d 298. b 352. b 406. a 460. c 514. c
245. a 299. b 353. e 407. c 461. a 515. c
246. d 300. d 354. a 408. d 462. a 516. c
247. c 301. b 355. a 409. a 463. c 517. a
248. b 302. a 356. d 410. c 464. c 518. e
249. c 303. b 357. b 411. a 465. b 519. d
250. e 304. a 358. c 412. d 466. c 520. a
251. d 305. c 359. d 413. d 467. a 521. d
252. c 306. d 360. b 414. c 468. b 522. d
253. b 307. c 361. d 415. d 469. a 523. e
254. b 308. c 362. a 416. b 470. a 524. c
255. c 309. c 363. e 417. c 471. d 525. e
256. b 310. b 364. e 418. c 472. c 526. d
257. c 311. c 365. a 419. b 473. e 527. e
258. a 312. e 366. a 420. e 474. b 528. c
259. c 313. a 367. b 421. c 475. c 529. c
260. a 314. c 368. c 422. a 476. c 530. a
261. d 315. c 369. a 423. d 477. d 531. b
262. d 316. c 370. c 424. c 478. b 532. a
263. a 317. a 371. b 425. d 479. b 533. a
264. d 318. a 372. b 426. a 480. b 534. b
265. c 319. b 373. d 427. a 481. d 535. b
266. b 320. a 374. d 428. c 482. b 536. c
267. c 321. b 375. c 429. e 483. b 537. a
268. a 322. d 376. b 430. c 484. b 538. a
269. c 323. b 377. d 431. d 485. a 539. d
270. b 324. a 378. c 432. c 486. e 540. c
271. c 325. d 379. b 433. d 487. c 541. d
272. a 326. c 380. a 434. a 488. c 542. d
273. c 327. c 381. d 435. c 489. a 543. a
274. d 328. a 382. d 436. d 490. d 544. a
275. c 329. a 383. c 437. d 491. c 545. a
276. b 330. d 384. b 438. d 492. b 546. b
277. e 331. c 385. c 439. c 493. c 547. c
278. c 332. b 386. c 440. a 494. a 548. d
279. b 333. c 387. a 441. b 495. d 549. c
280. c 334. c 388. d 442. b 496. a 550. d
281. a 335. b 389. a 443. e 497. b 551. a
282. c 336. b 390. c 444. b 498. e 552. d
283. c 337. a 391. b 445. c 499. e 553. e
284. b 338. d 392. d 446. d 500. d 554. c
285. c 339. b 393. c 447. a 501. d 555. b
286. b 340. d 394. b 448. c 502. a 556. c
287. c 341. b 395. b 449. a 503. b 557. d
288. a 342. d 396. b 450. b 504. a 558. c
Board part 2 MCQS

1- A 28-year-old woman, who is 18 weeks pregnant, presents to her


general practitioner(GP) with intermittent right upper quadrant pain and
dyspepsia. Her partner reports that she looked ‘yellow’ 1 week ago, but
she does not currently appear jaundiced. She is generally well, apyrexial,
and she is able to maintain normal dietary intake. A scan performed at
the GP confirms gallstones, without evidence of cholecystitis, and the
common bile duct measures 6 mm, within normal limits. Bilirubin,
amylase and liver function tests are all normal.
What is the most appropriate management?
A Dietary advice and nothing more for now
B Laparoscopic cholecystectomy on the next available list
C List and defer for laparoscopic cholecystectomy once the baby is
delivered
D Magnetic resonance cholangiopancreatography (MRCP)
E Open cholecystectomy

Answer: B

2- A 44-year-old woman, who underwent a right hemicolectomy for


caecal carcinoma, is identified on follow-up scan to have developed
metastatic liver lesions.
Which of the following statements best describes the selection of
patients for liver resection in the context of colorectal cancer
metastases?
A Metastases should not be resected if surgery is only feasible following
downstaging with chemotherapy
B Patients should only be considered if all lesions can be managed
synchronously
C Patients with recurrent metastases (following limited resection)
cannot be considered for further surgery
D Previous chemotherapy administration may limit the extent of surgery
offered
E Radiofrequency ablation may facilitate resection of liver lesions,
without significant associated risk

Answer: D

Dr. wessam alzaidat - General surgery


3- A 60-year-old man presents as a general surgical emergency with
severe pancreatitis. Which of the following statements best describes
the severity of acute severe pancreatitis?
A Body mass index correlates poorly with severity
B Severe pancreatitis is defined as organ(s) failure and/or local
pancreatic complications
C Severe pancreatitis is initially predicted from the Glasgow–Imrie score
≥3
D Severity is not judged on the same criteria as other organ scoring
systems
E Severity may only be scored at 48 hours

Answer:B

4- A 48-year-old woman presents to surgical outpatients with


intermittent dyspepsia and right upper quadrant pain. Seven years ago
she had a potentially curative mastectomy and right axillary clearance
for a high-grade, T3N1 carcinoma of the breast. She underwent adjuvant
chemotherapy and has remained well until now. The history is in
keeping with biliary colic, and an abdominal ultrasound
reveals three hypoechoic lesions within segments V and VI of the liver;
she also has gallstones.A CT subsequently confirms appearances
consistent with isolated metastases in the right lobe, without any other
intra-abdominal pathology. What is the most appropriate management
plan?
A Obtain a tissue diagnosis with core biopsy
B Obtain a tissue diagnosis with fine-needle aspiration cytology
C Refer to the hepatobiliary and pancreatic surgeons for a limited right
hepatectomy
D Refer to medical oncology for consideration of chemotherapy
E Watchful waiting

Answer:A

5- A 42-year-old woman sustains an iatrogenic bile duct injury during


laparoscopic cholecystectomy. Which of the following statements best
describes the management of injury to the biliary tree?
A A ductal T-tube and a subhepatic drain offer a temporising measure
before definitive management
B Bile in an abdominal drain 48 hours postcholecystectomy is within
normal acceptable limits
C Endoscopic retrograde cholangiogram is always diagnostic of the
degree of injury

Dr. wessam alzaidat - General surgery


D Low straight-forward common bile duct injuries recognised at
laparoscopiccholecystectomy should be repaired with primary suture
E Significant biliary injuries managed in hepatobiliary units normally fully
recover with minimal morbidity

Answer:A

6- Regarding lower extremity amputations, which of the following is


true?
(A) Higher energy expenditure is required for ambulation after below-
knee amputation (BKA) than after above-knee amputation (AKA).
(B) Lower extremity digital amputations should be performed through
the joint space to preserve smooth cartilaginous surfaces forgranulation.
(C)Dependent rubor at a proposed level of amputation is
acontraindication to amputation at that level.
(D) Guillotine amputation is inappropriate in the face of active infection.
(E) Diabetic patients who undergo amputation have up to a 10% risk of
contralateral amputation within 5 years.

Answer:C

7- In a newborn baby with type E tracheoesophageal fistula,


what surgical approach is indicated?
(A) Right thoracotomy
(B) Left thoracotomy
(C) Right cervical incision
(D) Midline laparotomy
(E) Endoscopic (endobronchial)

Answer:C

8- Which of the following is true regarding the management of


esophageal perforation?
A. The distal esophagus is best approached through the right chest.
B. Cervical perforations occur most commonly at the thoracic inlet.
C. Primary repair alone is sufficient in early perforation.
D. Diversion and wide drainage are best in high-risk patients.
E. Resection is required in perforation with achalasia

Answer:D

9- Which of the following is correct regarding inhalation injury in burn


patients?

Dr. wessam alzaidat - General surgery


A. The admission chest radiograph is useful for ruling out inhalation
injury on admission.
B. Supraglottic inhalation injury may necessitate intubation even if gas
exchange is initially unaffected.
C. With proper pulmonary toilet, pneumonia is an unusual
complication of smoke inhalation.
D. Smoke inhalation is basically just a subset of acute
respiratory distress syndrome (ARDS) seen in burn victims.
E. Daily bronchoscopy is mandatory to monitor the
evolution of inhalation injury

Answer:B

10. Which hormone is matched with the correct diagnostic/therapeutic


function?
A. Cholecystokinin (CCK)/treatment of esophageal variceal bleeding
B. Somatostatin/relief of spasm of the sphincter of Oddi
C. Gastrin/measurement of maximal gastric acid secretion
D. Glucagon/provocative test for gastrinoma
E. Secretin/stimulation of gallbladder contraction

Answer:C

11- A 45-year-old man requires surgery for an intractable duodenal


ulcer. Which operation best prevents ulcer recurrence?
A. Subtotal gastrectomy
B. Truncal vagotomy and pyloroplasty
C. Truncal vagotomy and antrectomy
D. Selective vagotomy
E. Highly selective vagotomy

Answer:C

12- A 35-year-old male underwent a laparoscopic appendectomy. On


final pathology, he was found to have a 1.4-cm carcinoid tumor in the
mid-appendix with direct extension to the mesoappendix, negative
margins, and no lymphovascular invasion. What is the best treatment
plan?
A. No further treatment needed
B. Ileocecectomy
C. Right hemicolectomy
D. Medical treatment with octreotide
E. Chemotherapy

Dr. wessam alzaidat - General surgery


Answer:C

13- A 39-year-old man with a history of mild, long-standing ulcerative


colitis controlled with sulfasalazine recently underwent routine
colonoscopy that showed a lesion in the sigmoid colon. Pathologic
evaluation reveals high-grade dysplasia. Which of the following is the
best management option?
A. Sigmoid colectomy, provided that the rectum is minimally involved
B. Proctocolectomy with ileal pouch–anal anastomosis(IPAA)
C. Total abdominal colectomy with ileorectal anastomosis(IRA)
D. Total proctocolectomy with Brooke ileostomy
E. Polypectomy and frequent surveillance colonoscopy to remove
further polyps

Answer:B

14- With regard to ischemic colitis, which of the following statements is


true?
A. The most common symptoms are lower abdominal pain and bright
red rectal bleeding.
B. Occlusion of the major mesenteric vessels is the cause of ischemia in
most cases.
C. The splenic flexure and hepatic flexure are the most vulnerable areas
of the colon.
D. Nonoperative management is not justified because of the significant
rates of perforation and peritonitis.
E. Griffith’s point at the rectosigmoid junction is the most vulnerable
area

Answer:A

15- Which of the following is the appropriate operation for a sigmoid


cancer that has not metastasized?
A. Segmental resection of the sigmoid with 5-cm margins and high
ligation of the sigmoid branch of the IMA
B. Resection of the entire sigmoid from the distal descending colon to
the rectosigmoid junction, sparing the main left colic artery
C. Resection of the sigmoid and the descending colon, with high ligation
of the IMA at its origin
D. Resection of the entire colon proximal to the lesion with IRA
E. Resection of the entire sigmoid with wide iliac lymph node dissection
and prophylactic oophorectomy.

Dr. wessam alzaidat - General surgery


Answer:B

16 - A 75-year-old man with hepatitis C cirrhosis presents for an annual


checkup. He denies any new complaints, including jaundice, abdominal
pain, ascites, gastrointestinal bleeding, or encephalopathy. Temperature
is 99.1°F, pulse 85 beats/min, blood pressure 109/78 mmHg, respiratory
rate 18 breaths/min, and saturation 98% on room air. What studies
should be ordered?
A. None
B. Ultrasound
C. α-Fetoprotein (AFP) level
D. Esophagogastroduodenoscopy (EGD)
E. CT sca
12) Bm

Answer:B

17- A 65-year-old man undergoes CEA for 75% stenosis of his left ICA. On
arrival at the intensive care unit, he develops aphasia. What is the next
step in management?
A. Serial neurologic examinations
B. Anticoagulation
C. CT head
D. No intervention required
E. Return to the operating room (OR)

Answer:E

18- The use of negative pressure wound therapy after burn wound
excision
A. increases acute kidney injury.
B. increases use of nursing care.
C. increases opioid use.
D. decreases total treatment costs.
E. increases rate of skin graft take

Answer:E
19- Flexible bronchoscopy in intubated patients on mechanical

Dr. wessam alzaidat - General surgery


ventilation increases
A. end expiratory lung volumes.
B. incidence of pneumothorax.
C. PaO2.
D. PaCO2.
E. tidal volume.

Answer:D
20- Which of the following parameters is required to characterize acute
respiratory distress syndrome based on the current definition of this
condition?
A. Pulmonary artery wedge pressure
B. Pulmonary vascular resistance
C. Mean airway pressure
D. Plateau pressure
E. Positive end expiratory pressure

Answer:E

21- Which of the following statements regarding the management of a


patient with necrotizing pancreatitis is true?
A. Mortality is greater than 30%.
B. Infected pancreatic necrosis increases mortality to 50%.
C. Open surgical necrosectomy remains the treatment of choice.
D. Intervention is reserved primarily for those with symptomatic clinical
deterioration.
E. Percutaneous and endoscopic drainage techniques are not
appropriate.

Answer:D

22- In patients with traumatic brain injury (head abbreviated injury score
>3), venous thromboembolus (VTE) prophylaxis with low molecular
weight heparin compared with
unfractionated heparin results in
A. increased mortality.
B. decreased bleeding complications.
C. increased unplanned returns to operating room.
D. increased incidence of heparin-induced thrombocytopenia.
E. decreased VTE rate.

Answer:E

Dr. wessam alzaidat - General surgery


23- A 72-year-old woman is recovering from a partial gastrectomy. On
postoperative day 3, she is agitated and confused. Postoperative
delirium after major abdominal
surgery in older patients is associated with
A. increased length of hospital stay.
B. increased intensive care unit admission rates.
C. a return to operating room.
D. no change in long-term cognitive performance.
E. wound dehiscence
Answer:A
23- A 52-year-old patient is critically ill in the intensive care unit 5 days
after urgent hepatic resection for bleeding hepatocellular cancer. He
suddenly develops bright red blood in one of the surgical drains.
Intravascular contrast for CT angiography is planned as a diagnostic
study to consider angioembolization of the source of bleeding. What
strategy is effective in reducing the risk of contrast associated acute
kidney injury?
A. Isotonic saline before and after
B. Sodium bicarbonate after
C. Sodium bicarbonate during contrast infusion
D. Hypertonic saline before and after
E. Albumin after

Answer:A

24- Which of the following factors has the highest risk for clinically
important gastrointestinal bleeding from stress ulcers in critically ill
patients?
A. Mechanical ventilation
B. Renal failure
C. Hepatic failure
D. Shock
E. Coagulopathy

Answer:A

25- A 7-year old girl is admitted after she was a restrained passenger in a
T-bone motor vehicle crash with impact on her side of the car. She is
tachycardic and has an acute abdomen. At surgery, she has an isolated
duodenal perforation in the second portion opposite the ampulla of
Vater involving 25% of the circumference. What is the best operative
plan?
A. Primary repair of the duodenum

Dr. wessam alzaidat - General surgery


B. Pyloric exclusion
C. Repair the duodenum, gastrostomy, jejunostomy, and duodenostomy
D. Duodenal diverticularization
E. Gastrojejunostomy, nasogastric decompression, nasojejunal feeding
tube

Answer:A

26- A 25-year-old female driver of an automobile sustained severe


multiple traumatic injuries. She had an altered sensorium and required
intubation. Etomidate was used for induction. A noted side effect of this
agent is associated with what electrolyte abnormality?
A. Hypomagnesemia
B. Hypokalemia
C. Hyperphosphatemia
D. Hyponatremia and hyperkalemia
E. Hypernatremia and hypokalemia

Answer:D

27- An. obese woman who underwent a transabdominal hysterectomy


10 years ago presents for an elective ventral hernia repair. She
undergoes hernia repair via a laparosc:opic approach. to which. she
responds well initially. However. she develops a recurrent bulge 2
months later and presents to your office for counseling. What is the
most common reason for recurrence after laparoscopic ventral hernia
repair?
A. Failure of suture material or tacks
B.Inadequate dissection of the fasdal defect
C.Separation of the mesh from the abdominal wall
D.Seroma formation
E- Unrecognized defect

Answer:C

28- You are a tumor researcher trying to find novel cancer treatment.
One of your projects involves constructing a tumor vaccine for breast
cancer. Such a vaccine would take dvantage of which of the following
aspects of the immune system to prevent cancer?
A.Activation of latent T cells ln the thymus
B. Adaptive immunity
C.Cytokine activation
D.Innate immunity

Dr. wessam alzaidat - General surgery


E.Type IV hypersensitivity reaction

Answer:B

29- C-reactive protein (CRP)


A. Is secreted in a circadian rhythm with higher levels in the morning
B. Increases after eating a large meal
C. Does not increase in response to stress in patients with liver failure
D. Is less sensitive than erythrocyte sedimentation rate as a marker in
ammation
E. its value is not concordant for measurement of inflammation and
follow up of treatment
Answer:C
30- A 42-year-old man receives a cadaveric renal transplantation and is
commenced on antirejection treatment, which includes cyclosporin.
What is the most common side effect of cyclosporin?
A Alopecia
B Hepatotoxicity
C Nephrotoxicity
D Pancreatitis
E Pancytopaenia

Answer:C

31- The spinal accessory nerve passes through the posterior cervical
triangle:
A. Superficial to the platysma muscle.
B. In the superficial cervical fascia.??
C. Between the investing fascia and the prevertebral fascia.??
D. Within the carotid sheath.
E. Deep to the prevertebral fascia.

Answer:B or C

32- Hypophosphatemia increases the risk of the development of each of


the following, EXCEPT?
A .Respiratory failure.
B .Tissue anoxia due to decreased hemoglobin
C .Encephalopathy
D .Hypothyroidism
E .Hemolysis

Answer:D

Dr. wessam alzaidat - General surgery


33- Cushing syndrome:
A. Most commonly iatrogenic

34- Most common breast ca type


A. Ductal

35- Post PCNL to detect retained stone


a) Non contrast CT

36- Best prognosis for endocrine ca


A. Papillary thyroid ca

37- Marfan with ascending dissection ?


1. Nitroglycerine and attempt resection

38- Lactating with breast tenderness and no fluctuation


A. Staph

39- LCIS
A. May occur in contralateral side

40- Late complication ok Kasai except


A. Leak

41- Breas ca with axillary lymph node and lung mets


a. T4N1M1

42- Wound contraction


A. fibroblast actin myofilamints

44- Diagnosis esophageal injury ?


E. Esophageoscopy

45- Q about all true except


A. glucagon increases intestinal motility

46- Endoleak
i. Type II from lumber

Dr. wessam alzaidat - General surgery


47- warfarin induced skin necrosis def:
A. Protien c

48- Enteral vs parenteral feed


A. bowel function
B. aspiration
C. duration of requirement

49- Fat embolism ?


b) Present with retinal petechiae

50-TPN:
hypokalemia

Dr. wessam alzaidat - General surgery


JRMS RESIDENTS MCQS

1. Most common site of malignancy that leads to metastasize to liver

A. Thyroid
B. RCC
C. Breast
D. Colon
E. Lung

2. Appropriate management of 3-cm squamous cell carcinoma of the anal canal is:

A. Chemotherapy
B. Abdominoperineal resection
C. Local excision
D. Radiotherapy + local excision
E. Chemotherapy + Radiotherapy (Nigro protocol)

3. A 37 year-old female presents with a palpable mass on physical exam . A


mammogram was obtained and her lesion was characterized as a BIRADS 0 your
next step:

A. An ultrasound
B. Excision
C. Breast MRI
D. Fine-needle biopsy
E. Repeat mammogram in 6 months

4. A 62-year-old female presents with scaly skin surrounding her nipple that has not
resolved.
A biopsy including skin was performed at this area. Pathology returns with large
vacuolated cells in the epithelium and no other atypical cells or cancerous cells are
present. Her treatment should be:

A. Modified radical mastectomy


B. Simple mastectomy
C. Steroid cream
D. Simple mastectomy with SLN biopsy
E. Wide excision of the skin involved

5. The deleterious effects of steroids on wound healing can be partiallyreversed


with:

A. vitamin C
B. vitamin A
C. copper
D. vitamin D
E. vitamin E
Dr. wessam alzaidat - General surgery
6. Hemostatic resuscitation means giving PRBC/FFP/Plat in what ratio:

A. 1:1:1
B. 1:2:1
C. 1:2:2
D. 2:1:2
E. 2:2:2

7. An 88 year old man with severe dementia present to ER from his nursing home
with constipation and abdomen distention for last 8 days. Past medical history is
non tender. Abdominal X ray markedly dilated large bowel with no visible air in
rectum. The next step in management is:

A. Urgent laparotomy for toxic megacolon


B. Decompression rectal tube
C. Neostigmine for pseudo-obstruction
D. Gastrografin enema
E. Loop colostomy for chronic obstruction

8. The following statements about the repair of inguinal hernias are true, except:

A. The conjoined tendon is sutured to Cooper's ligament in the Bassini hernia


repair.
B. The McVay repair is a suitable option for the repair of femoral hernias.
C. The Shouldice repair involves a multilayer, imbricated repair of the floor of the
inguinal canal.
D. The Lichtenstein repair is accomplished by prosthetic mesh repair of the inguinal
canal floor in a tension-free manner.
E. The laparoscopic transabdominal preperitoneal (TAPP) and totally
extraperitoneal approach (TEPA) repairs are based on the preperitoneal repairs
of Cheattle, Henry, Nyhus, and Stoppa.

9. The boundaries of the femoral triangle are the inguinal ligament, except:

A. Adductor longus.
B. Pectineus.
C. Sartorius.
D. Adductor brevis
E. Iliopsoas.

10. A 59 years old female newly dx of rectal ca 14 cm from anal verge. Histopathology
consistent with moderately differentiated adenocarcinoma. Staging shows tumor
confined to bowel wall without lymphadenopathy or Mets. Next step in
management:

A. Trananal excision
B. Chemoradiation
C. Resection with 5 cm distal margin
D. Resection with 2 cm distal margin
E. Resection with total mesorectal excision

Dr. wessam alzaidat - General surgery


11. By which of the following modalities is the staging thymomas primarily
determined:

A. Surgical excision
B. Immunohistochemistry assay
C. MRI evaluation
D. CT scan
E. Mediastinoscopy

12. A 55 year old women with BMI 41 presents to clinic with continued reflex
symptoms despite Nissen fundoplication and 2 revisions. During work-up, it is
evident that fundoplication has failed what is the best option for this patient:

A. Conversion to Toupet fundoplication


B. Laparoscopic revision of Nissen fundoplication
C. Esophageal dilatation
D. Tuncal vagotomy and gastric resection
E. Laparoscopic revision to gastric bypass

13. Bevacizumab

A. Is a monoclonal antibody that targets vascular endothelial growth factor (VEGF)


B. Is a monoclonal antibody that targets EGFR
C. Has no proven survival benefit in the metastatic setting
D. Is standard treatment for metastatic colorectal cancer in the UK

14. Which of the following patients is at an average risk of colorectal cancer?

A. 45-year-old female with an aunt and cousin with colorectal cancer


B. 35-year-old female with ulcerative colitis diagnosed at age 25
C. 75-year-old male with a history of recurrent diverticulitis
D. 60-year-old male whose grandfather was diagnosed with colon cancer at age 60
E. 25-year-old male whose older brother had hundreds of polyps on colonoscopy

15. A 38-year-old woman presents with RUQ. Imaging with CT shows a 3-cm mass in
segment 7 of the liver that is well demarcated with a central fibrotic area. This
central scar does not enhance in the arterial phase of the CT scan. Workup shows
no evidence of cirrhosis, hepatitis and a normal AFP level, and slightly elevated
neurotensin. What is the diagnosis?

A. FNH
B. Hepatic adenoma
C. Fibrolamellar carcinoma
D. Hepatoma
E. Hemangioma

Dr. wessam alzaidat - General surgery


16. The Howship-Romberg sign is associated with which type of hernia

A. Grynfeltt hernia
B. Richter hernia
C. Petit hernia
D. Littre hernia
E. Obturator hernia

17. A-33-year-old female is involved in a motor vehicle collision and found to have a
complete transaction of the pancreas at the level of the pancreatic neck:

A. Oversewing of the proximal segment and distal pancreatectomy


B. Drainage of the proximal and distal segments
C. Drainage of the proximal segment and oversewing of the distal segment
D. Oversewing of the proximal segment and drainage of the distal segment
E. Roux-en-Y pancreaticojejunostomy of the proximal pancreas and distal
pancreatectomy

18. A patient with acute pancreatitis deteriorates clinically and is reimaged. CT scan
shows findings of a fluid collection consistent with infected pancreatic necrosis
that is adherent to the posterior gastric wall and close to the pancreatic head.
What is the best management?

A. Endoscopic drainage
B. Cystogastrostomy
C. Roux-en-Y cystojejunostomy
D. Percutaneous drainage
E. ERCP

19. A 25-year-old female driver of an automobile sustained severe multiple traumatic


injuries. She had an altered sensorium and required intubation. Etomidate was
used for induction. A noted side effect of this agent is associated with what
electrolyte abnormality?

A. Hypomagnesemia
B. Hypokalemia
C. Hyperphosphatemia
D. Hyponatremia and hyperkalemia
E. Hypernatremia and hypokalemia

20. Which electrolyte and acid-base abnormality is present in a neonate with


intractable projectile vomiting from hypertrophic pyloric stenosis?

A. Hyperkalemic hypochloremic metabolic alkalosis


B. Hypokalemic hypochloremic metabolic acidosis
C. Hypokalemic hyperchloremic metabolic alkalosis
D. Hypokalemic hypochloremic metabolic alkalosis
E. Hypernatremic metabolic acidosis

Dr. wessam alzaidat - General surgery


21. Which of the following is depleted rapidly post-operative

A. Glycogen
B. Glucose
C. Ketone bodies
D. Protein
E. Short-chain fatty acid

22. Which of the following is not an indication to repair a splenic artery aneurysm?

A. Asymptomatic and size >2 cm


B. Pregnancy
C. Symptomatic and size = 1.5 cm
D. Symptomatic and size = 2 cm
E. Presence of collagen vascular disease

23. Which of the following indications for splenectomy poses the highest risk of post-
splenectomy sepsis?

A. Trauma
B. Idiopathic thrombocytopenic purpura (ITP)
C. Hereditary spherocytosis (HS)
D. Thalassemia major
E. Hereditary elliptocytosis

24. Which of the following is the best indication for splenectomy?

A. Sarcoidosis
B. Gaucher disease
C. Myelofibrosis
D. Hairy cell leukemia with neutropenia
E. Secondary hypersplenism in a cirrhotic patient

25. A patient present to ED with an amputation through the proximal phalanx of his
left index, the treatment of choice is:

A. Toe to finger transfer


B. Thener flap
C. Replantation
D. Revision amputation
E. Local flap

Dr. wessam alzaidat - General surgery


26. The best neuromuscular relaxant for intubate 33 year-old male with end stage
renal failure who candidate to perform lap cholecystectomy

A. Mivacurium
B. Pancuronium
C. Succinylcholine
D. Vecoronium
E. Atracronium

27. A 53 year old man presents with biopsy proven 4 cm buccal mucosa squamous cell
carcinoma of his left check, no lymph node palpable or visible on CT scan. The best
treatment well be:

A. Wide local excision with radical neck dissection


B. Local excision
C. Radiation therapy
D. Wide local excision with selective neck dissection
E. Wide local excision with modified radical neck dissection

28. 5 month old infant present with mix hyperbilirubinemia undergoes percutaneous
liver biopsy. MRCP and abdominal U/S that ultimately revealed the diagnosis of
biliary atresia. Which of the following is the best treatment:

A. CT guided drainage of biliary tree


B. Chocystostomy tube
C. Kasai portoenterostomy
D. Liver transplant
E. ERCP & stent

29. 16 year-old male is evaluated by his pediatrician for pubertal abnormal. The
physician describes the risk of malignancy of gonads with the patient. Which of the
following is associated with greatest risk of testicular cancer

A. Testicular trauma
B. Androgen insentivity
C. Noonen syndrome
D. Turner syndrome
E. XY gonadal dysgenesis

30. All of the following are consider risk factors for squamous cell carcinoma except:

A. Burns
B. Pilonidal abscess
C. Rhabdomyolysis
D. Irradiation
E. Venous stasis ulcer

Dr. wessam alzaidat - General surgery


31. A 47 year-old women is found to have cluster calcification on mammogram. Trucut
biopsy reveals pleomorphic lobular carcinoma in situ. The next step in
management is:

A. Observation with 6 months follow-up U/S


B. Observation with 6 months follow-up mammogram
C. Add aromatase inhibitors
D. Add trastuzumb
E. Wire localized partial mastectomy

32. Which of the following innervated by the external branch of the superior laryngeal
nerve

A. Posterior cricoarytenoid
B. Lateral cricoarytenoid
C. Thyroid arytenoid
D. Cricothyroid
E. Interarytenoid

33. 17 year-old male has 2 cm right cervical nodule. An FNA is done and cells stain
positive for calcitonin. Labs also reveal elevated calcium. What is the most likely
diagnosis?

A. Familial MTC
B. Follicular thyroid cancer
C. MEN 1
D. MEN 2a
E. MEN 2b

34. A 69 year-old man is seen in the office to discuss management options for his
abdominal aortic aneurism. His medical history includes coronary artery disease,
poorly controlled HTN, COPD, and he continues to smoke 1 pack per day. A
previous CT scan showed his aneurysm to be 4.5 cm 3 years ago. He was then lost
to follow-up until a recent CT performed in the ED found it to be 6.5 cm. which of
the following below would preclude endovascular repair?

A. Contained rupture
B. 1 cm infra-renal aortic neck
C. 3 cm diameter at iliac bifurcation
D. Growth of > 1cm over the past year
E. Multiple prior abdominal surgeries

Dr. wessam alzaidat - General surgery


35. A 73-year-old male present 2 years after aorta-bi-femoral bypass with a swollen,
tender mass in his groin. CT scan shows air and fluid around the left femoral
anastomosis that tract into the abdomen. The correct management of this

A. Culture-directed antibiotic treatment and close observation


B. Endovascular repair
C. Removal of aorta-bi-femoral bypass and replaced with Rifampin soaked Dacron
D. Removal of aorta-bi-femoral bypass followed by axillary-bi-femoral bypass
E. Removal of aorta-bi-femoral bypass followed by axillary-left-femoral bypass

36. Which of the following fracture is most closely associated with intra-abdominal
injuries?

A. Calcaneal fractures
B. Distal radius fracture
C. Chance fractures
D. Jefferson fracture
E. Femoral neck fracture

37. What must be done in all posterior knee dislocations if there no other sign of
trauma?

A. Application of a long leg splint


B. Application of a long leg cast
C. Determination of compartment pressure above and below the knee
D. Arteriogram to rule out vascular injury
E. Amputation

38. 70 year-old man with caceal carcinoma undergoing right hemicolectomy through
transverse incision. The procedure is difficult and the incision extended medially
by divided rectus sheath. Brisk arterial hemorrhage ensues. From which of the
damaged vessel originate?

A. External iliac artery


B. Femoral artery
C. Common iliac artery
D. Internal iliac artery
E. Superior vesical artery

39. Regarding vitamins and wound healing which of the following is correct:

A. Vitamin A potentiate the effect of steroid


B. Vitamin E has anti-inflammatory properties
C. Zinc deficiency has no effect in wound healing
D. Vitamin C not essential for wound healing
E. Catabolic bioproductus of arginine can retard wound healing

Dr. wessam alzaidat - General surgery


40. 55 year-old women has incidentally identified cystic lesion in the head of pancreas.
She no pervious history of pancreatitis, analysis of fluid revealed normal amylase
level, low CEA, negative mucin and scant cells. Mostly likely diagnosis

A. IPMN
B. Pancreatic pseudocyst
C. Mucinous cystic neoplasm
D. Pancreatic duplication
E. Serous cystic neoplasm

41. 33 year-old female known case GBS, admitted to hospital as a case of mild acute
pancreatitis (amylase 1400), 2 days after admission on her abdomen pain resolved
but her serum amylase remain elevated (amylase 850)LFT within normal limit and
her abdominal U/S shows cholelithiasis and normal CBD. The best next step:

A. Continue observation, repeat labs, and do laparoscopic cholecystectomy after


normalize
B. Discharge and schedule her for elective laparoscopic cholecystectomy after 6
weeks
C. Proceed with laparoscopic cholecystectomy
D. Do abdomen CT scan and give her ursodeoycholate to dissolve stones
E. Do ERCP for CBD stone prior to laparoscopic cholecystectomy

42. Compared to truncal vagotomy and pyloroplasty, proximal gastric vagotomy:

A. Decrease gastric empty


B. Impair antral pump mechanism
C. Increase dump syndrome
D. Maintain gastric empty
E. Reduce acid production

43. 55 year-old male stepped on a rusty nail in his garage. The area has surrounding
erythema. He reports that he received his full series of vaccination as child. He
received last tetanus booster shot when he was 20 years. What should be received
for tetanus prophylaxis:

A. Tetanus toxoid 0.5 ml IM


B. Tetanus immune globin 250 IU ml
C. Tetanus toxoid 0.5 ml IM with Tetanus immune globin 250 IU ml
D. Just antibiotics prophylaxis
E. No need for prophylaxis

Dr. wessam alzaidat - General surgery


44. After 20 years of active ulcerative colitis disease, the risk of cancer approximates

A. 10%
B. 20%
C. 50%
D. 75%
E. 90%

45. The incidence of Gall Bladder stone (GBS) in female above 40

A. 10%
B. 20%
C. 50%
D. 75%
E. 90%

46. How many times does the bile acid pool recycle daily?

A. Once
B. Two times
C. Three times
D. Five times
E. Ten times

47. How much is the daily basal secretion of hepatic bile?

A. 100 ml
B. 300 ml
C. 600 ml
D. 1500 ml
E. 2000 ml

48. In cirrhotic patients who are actively bleeding, the coagulopathy of end-stage liver
disease can be differentiated from DIC most readily by estimation of which of the
following factors?

A. Factor II
B. Factor IX
C. Factor VII
D. Factor VIII:C
E. Factor X

Dr. wessam alzaidat - General surgery


49. The most common presentation for appendiceal adenocarcinoma is:

A. Palpable abdominal mass


B. Acute appendicitis
C. Ascites
D. Incidental finding during unrelated abdominal surgery
E. Chronic anemia

50. A hypotensive patient had sustained multiple stab wounds to the abdomen. In the
operating room, a through-and-through laceration to the stomach and a
pancreatic and a low-grade splenic injury. How would you manage the gastric
perforations?

A. Gastric resection with primary closure


B. Primary closure of both wounds
C. A posterior gastrojejunostomy with primary repair of the anterior injury
D. Resection with Roux-en-Y reconstruction
E. Subtotal gastrectomy with gastrojejunostomy

51. Kidney transplant recipients are at increased risk for:

A. Epidermoid skin cancer


B. Lung cancer
C. Colon cancer
D. Breast cancer
E. Uterine cancer

52. A 60-year-old otherwise healthy woman presents to the emergency department


for acute onset of lower abdominal pain with associated nausea and emesis. Her
previous surgical history includes a total hysterectomy. On examination, she is
distended and peritonitic. Her vitals show tachycardia and hypotension, and lab
results show bicarb of 14 and lactic acid of 4.8. The emergency department orders
a plain film which show cecal volvulus. What is the optimal therapy for this
patient?

A. Subtotal colectomy with end ileostomy


B. Endoscopic decompression, placement of a rectal tube, and sigmoid colectomy
with primary anastomosis
C. Intraoperative decompression followed by subtotal colectomy
D. Laparotomy, detorsion, and cecostomy tube
E. Right hemicolectomy and primary anastomosis with or without diverting
ileostomy

Dr. wessam alzaidat - General surgery


53. Cecal diverticula are different from sigmoid diverticula in that:

A. Sigmoid diverticula are true diverticula while cecal diverticula are false
diverticula.
B. Cecal diverticulitis is usually distinguishable from cancer.
C. Cecal diverticula are considered congenital in origin.
D. Asymptomatic cecal diverticula found on barium enema or colonoscopy should
be treated operatively because of the high incidence of complications.
E. Feculent peritonitis from perforation of a cecal diverticulum may be treated with
resection and primary anastomosis in most cases.

54. A 35-year-old man with chronic constipation has noticed the passage of bright red
blood per rectum. Anoscopy demonstrates hemorrhoids. Which of the following is
true regarding the anatomy of the anal canal?

A. The dentate line lies above the columns of Morgagni.


B. Anal gland ducts are completely separate from the anal crypts.
C. The columns of Morgagni overlie the internal hemorrhoidal plexus.
D. The typical locations for internal hemorrhoids are right lateral and left anterior.
E. Anal glands are frequently found in the ischiorectal space.

55. Early gastric cancer is best defined as:

A. Lymph node negative


B. Limited to the mucosa
C. Limited to the mucosa and submucosa with negative nodes
D. Limited to the mucosa and submucosa regardless of nodes
E. In the muscularis propria but not the serosa

56. A 53-year-old woman with a history of a vagotomy and antrectomy with Billroth II
reconstruction for peptic ulcer disease presents with recurrent abdominal pain. An
esophagogastroduodenoscopy (EGD) demonstrates that ulcer and serum gastrin
levels are greater than 1000 pg/mL on three separate determinations (normal is
40-150). Which of the following is the best test for confirming a diagnosis of
gastrinoma?

A. A 24-hour urine gastrin level


B. A secretin stimulation test
C. A serum glucagon level
D. A 24-hour urine secretin level
E. A serum glucose to insulin ratio

Dr. wessam alzaidat - General surgery


57. A 62-year-old woman with a history of coronary artery disease presents with a
pancreatic head tumor and undergoes a pancreaticoduodenectomy.
Postoperatively, she develops a leak from the pancreaticojejunostomy
anastomosis and becomes septic. A Swan-Ganz catheter is placed, which
demonstrates an increased cardiac output and decreased systemic vascular
resistance. She also develops acute renal failure and oliguria. Which of the
following is an indication to start dopamine?

A. To increase splanchnic flow


B. To increase coronary flow
C. To decrease heart rate
D. To lower peripheral vascular resistance
E. To inhibit catecholamine release

58. A 55-year-old female undergoes liver resection for metastatic carcinoid tumor.
Upon induction, she becomes flushed, tachycardic to HR = 120 bpm, and
hypotensive with a systolic blood pressure of 70 mm Hg. Her end-tidal CO2 is
normal and she remains normothermic. What is the next best step in
management?

A. Abort the operation


B. Administer dexamethasone
C. Administer dantrolene
D. Administer octreotide
E. Administer Benadryl

59. Which of the following is the best test for prognosis and for monitoring treatment
response in carcinoid tumors?

A. Platelet serotonin levels


B. A 24-hour urinary 5-HIAA test
C. Serum chromogranin A levels
D. Serum serotonin levels
E. Neuron-specific enolase

60. A 15-year-old boy presents to the ED with severe left testicular pain that woke him
from sleep 3 hours ago. He denies scrotal trauma or recent infections. He had a
similar episode 6 months ago that resolved within minutes. Physical exam reveals
an enlarged, firm, and tender left testicle with abnormal lie. Stroking the left inner
thigh does not elicit elevation of the hemiscrotum. Manual elevation of the
scrotum does not relieve the pain. Which of the following is the best next step in
management?

A. Testicular ultrasound
B. Attempt left testicular detorsion in the ED followed by left testicular orchiopexy in
the OR
C. Attempt left testicular detorsion in the ED followed by bilateral orchiopexy in the
OR
D. Take to the OR to perform left testicular detorsion and bilateral orchiopexy
E. Take to the OR to perform a left orchiectomy

Dr. wessam alzaidat - General surgery


61. A 25 year-old man presents to the ER with complaints of vague chest discomfort.
Acute coronary syndromes have been ruled out and the patient is noted to have a
large anterior mediastinal mass. The alpha fetoprotein levels are markedly
elevated. What is the likely diagnosis?

A. Non-seminomomatous germ cell tumor


B. Seminoma
C. Thymoma
D. Lymphoma
E. Ectopic thyroid goiter

62. Von Willebrand disease (vWD) is characterized by:

A. Normal bleeding time


B. Lack of treatment response to DDAVP (desmopressin) infusion
C. Types 1 and 2, which demonstrate autosomal recessive inheritance
D. Defect in platelets adhesion
E. Prolonged INR

63. A 65-year-old woman presents with a 1-cm lesion with a pearly border on her
nose, and punch biopsy is consistent with a basal cell carcinoma. She is scheduled
to undergo Mohs surgery. Which of the following is a benefit of Mohs surgery over
wide local excision?

A. Mohs surgery results in a smaller cosmetic defect while obtaining negative


margins circumferentially.
B. Mohs surgery offers a shorter operating time.
C. Mohs surgery can be performed on many different types of skin cancers.
D. Mohs surgery results in less recurrence and metastases.
E. Mohs surgery does not depend on intraoperative evaluation of specimen
margins with frozen sections.

64. After a 3-hole esophagectomy with a cervical anastomosis, the patient begins to
develop leukocytosis, fever, and erythema of the wound. A barium swallow is
performed and there is an anastomotic leak noted. What is the most appropriate
management of this patient?

A. Open the wound and redo the anastomosis


B. Open the wound and widely drain the anastomosis
C. Endoluminal stent
D. NPO with oral antibiotics
E. NPO with IV antibiotics

Dr. wessam alzaidat - General surgery


65. Hyperthyroidism can be caused by all of the following except:

A. Graves' disease.
B. Plummer's disease.
C. Struma ovarii.
D. Hashimoto's disease.
E. Medullary carcinoma of the thyroid.

66. Strawberry gallbladder is a gross description given to:

A. Cholesterolosis
B. Adenomyomatosis
C. Porcelain gallbladder
D. Acalculous cholecystitis
E. Gangrenous cholecystitis

67. A 42-year-old man who weighs 60 kg is receiving 3 L of standard hypertonic 25%


glucoseamino acid solution. He has no history of smoking or bronchial asthma. In
the ICU, he is alert, afebrile, and hemodynamically stable, but he remains
intubated and attempts to wean him off the ventilator have been unsuccessful.
What is the most likely cause?

A. Copper deficiency
B. Excess fat calories
C. Excess glucose calories
D. Excess amino acids
E. Inadequate glucose calories

68. A patient with Crohn disease undergoes an ileocecectomy for perforation and
subsequently has a prolonged postoperative ileus. What is the most common
electrolyte abnormality associated with starting TPN?

A. Hypernatremia
B. Hypokalemia
C. Hypophosphatemia
D. Hypomagnesemia
E. Hypochloremia

69. The most common bacterial cause of proctitis is:

A. Chlamydia trachomatis
B. Neisseria gonorrhoeae
C. Treponema pallidum
D. Haemophilus ducreyi
E. Shigella species

Dr. wessam alzaidat - General surgery


70. A 52 year-old man presents to the emergency room with abdominal pain, weight
loss, and dehydration. He has had large volume diarrhea (>3 L/day) over the past
few weeks. Abdominal computed tomography (CT) reveals a small mass in the tail
of the pancreas, and a plasma vasoactive intestinal polypeptide (VIP) is sent.
Which of the following would be most consistent with a diagnosis of Verner-
Morrison syndrome?

A. Abdominal pain
B. Osmotic diarrhea
C. Stool osmotic gap of 300 mOsm/kg
D. VIP plums level of 600 pg/mL
E. Weight loss

71. Which of the following is a genetic mutation occur early in the development of
colorectal cancer?

A. Activation of adenomatous polyposis coli (APC)


B. CAG triplicate repeat sequence
C. Inactivation of K-ras
D. Activation of DCC/p53
E. hMLH1/hMSH2

72. All of the following are true regarding PP, except

A. Proximal pancreatectomy is associated with a greater PP deficiency than distal


pancreatectomy
B. Patients with PP-producing pancreatic tumors present with sever hypoglycemia
C. PP regulates expression of the hepatic insulin receptor gene
D. It is secreted by F cells
E. PP deficiency correlates with severity of chronic pancreatitis

73. Which of the following nerves is not at risk during a laparoscopic repair of an
inguinal hernia?

A. Pudendal nerve
B. Femoral branch of the genitofemoral nerve
C. Lateral cutaneous nerve of the thigh
D. Intermediate cutaneous branch of the anterior branch of the femoral nerve

74. Superficial lymphatics of the nipple and areola collect in a

A. Pectoral or external mammary group


B. Subscapular group
C. Lateral group
D. Central group
E. Rotter’s nodes, or interpectoral nodes

Dr. wessam alzaidat - General surgery


75. The clinical picture of gallstone ileus includes which of the following, one false?

A. Air in the biliary tree.


B. Small bowel obstruction.
C. A stone at the site of obstruction.
D. Acholic stools.
E. Associated bouts of cholangitis.

76. Which of the following anesthetic drug can cause metabolic acidosis:

A. Etomidate
B. Propofol
C. Fentanyl
D. Rocuronium
E. Ketamine

77. Colonic polyps, one of the followings is true:

A. Juvenile rectal polyps are adenomatous polyps


B. Metaplastic polyps are premalignant
C. The risk of malignancy is higher in tubular than villous adenomas
D. Villous adenomas occasionally cause hyperkalemia
E. All patients with untreated familial adenomatous polyposis will eventually
develop colorectal carcinoma

78. Which of the following types of polyps is associated with hypokalemia?

A. Villous adenoma
B. Hyperplastic polyp
C. Adenomatous polyp
D. Retention polyp
E. Pseudopolyp

79. A 62-year-old man is involved in a moving vehicle accident. He suffered significant


blunt trauma to the sternum during the accident. He has a systolic blood pressure
of 95 on arrival to the ER. His CVP is 15 and his CXR is normal. Which of the
following is the most likely cause of his hypotension?

A. Cardiac contusion
B. Spinal cord injury
C. Cardiogenic shock following an acute myocardial infarction
D. Intra-abdominal hemorrhage

80. A 60-year-old man with carcinoma of the esophagus is admitted with severe
malnutrition. Nutritional support is to be initiated. What should be his daily caloric
intake?

A. 1 kcal/kg body weight/day


B. 5 kcal/kg body weight/day
C. 15 kcal/kg body weight/day
D. 30 kcal/kg body weight/day
E. 100 kcal/kg body weight/day

Dr. wessam alzaidat - General surgery


81. What is the treatment of the Warthin's tumor?

A. Observation
B. Surgical enucleation
C. Total parotidectomy with resection of the fascial nerve
D. Total parotidectomy with postoperative radiation
E. Superficial parotidectomy

82. A 38-year-old man has a painless, non-tender mass in his left neck that moves with
swallowing. Fine-needle aspiration shows medullary carcinoma. The right side
appears normal. The best course of action would be

A. Left thyroid lobectomy


B. Subtotal right lobectomy and left lobectomy
C. Total thyroidectomy
D. Total thyroidectomy with central neck dissection
E. Total thyroidectomy with radical neck dissection

83. The earliest manifestations of hypocalcemia following thyroidectomy is

A. Laryngospasm
B. Carpopedal spasm
C. Circumoral numbness
D. Hyporeflexia
E. Bradycardia

84. A modified radical neck dissection Type I spares one of the following structures

A. Vagus nerve
B. Spinal accessory nerve
C. Jugular vein
D. Posterior triangle lymph nodes
E. Sternocleidomastoid muscle

85. You see a 4- year-old boy regarding an abdominal mass. His parents have noted
that he has been increasingly lethargic over the past several weeks. They think he
is pale, and he has recently started to take long afternoon naps. His appetite has
been poor and he has lost 4 pounds in the last 2 months. Upon physical
examination, the child is thin and pale. His lungs are clear with equal breath
sounds. His heart tones are normal. His abdomen is quite full. You feel a large
mass under the costal margin that firm and immobile. The most likely diagnosis

A. Lymphoma
B. Rhabdomyoscarcoma
C. Wilms tumor
D. Neuroblastoma
E. Renal cell carcinoma

Dr. wessam alzaidat - General surgery


86. During laparoscopic surgery, pneumoperitoneum usually results in a fall in
systemic blood pressure when intra-abdominal pressure exceeds

A. 5 mmHg
B. 10 mmHg
C. 20 mmHg
D. 30 mmHg
E. 40 mmHg

87. 22-year-old man undergoes an exploratory laparotomy after a gunshot wound to


the abdomen. The patient has multiple injuries, including a significant liver
laceration, multiple small-bowel and colon injuries, and an injury to the infra-
hepatic vena cava. The patient receives 35 units of packed RBCs, 15 L of crystalloid,
12 units of fresh-frozen plasma (FFP), and a 12 packs of platelets. The patient's
abdomen is packed closed and he is taken to the intensive care unit for further
resuscitation. Which of the following warrants a decompressive laparotomy?

A. Increased peak airway pressure


B. Increased cardiac output
C. Decreased systemic vascular resistance
D. Decreased plasma renin and aldosterone
E. Increased cerebral perfusion pressure

88. All of the following are true regarding Pneumatosis intestinalis, except?

A. In neonates, it is most commonly associated with necrotizing enterocolitis


B. It can lead to surgical pneumoperitoneum
C. In adults, secondary pneumatosis intestinalis is most often associated with COPD
D. Primary pneumatosis intestinalis usually requires surgical intervention
E. The diagnosis is readily made with plain radiography

89. Galactorrhea, a milky discharge from the nipple in non-pregnant women, is most
likely to be associated with which of the following?

A. Fibroadenoma
B. Tubular adenoma
C. Pituitary adenoma (often present with a bitemporal hemianopia).
D. Hyperparathyroidism
E. Breast abscess

90. An overdose of morphine is treated with

A. Naloxone I.V
B. Nalorphine I.V
C. Methadone I.V
D. Flumzenil
E. Steroids.

Dr. wessam alzaidat - General surgery


91. A 48-year-old woman maintained on Warfarin for a history of cardiac valvular
replacement and a history of recent upper respiratory infection presents with
severe abdominal pain exacerbated by movement. Her physical examination
shows tenderness in the right paramedian area with voluntary guarding but no
peritoneal signs. Which statement is true concerning the diagnosis and
management of this patient?

A. Urgent laparotomy should be performed because of concern for arterial


mesenteric embolus
B. The correct diagnosis could likely be made by CT scan and operation avoided
C. The status of her anticoagulation should be checked and if her prothrombin time
is excessively prolonged surgery is necessary
D. If untreated, hemodynamic instability is common
E. The usual cause is severe trauma.

Dr. wessam alzaidat - General surgery


Answers

1. D 40. E 79. C
2. E 41. C 80. D
3. C 42. D 81. E
4. B 43. A 82. D
5. B 44. A 83. C
6. A 45. A 84. B
7. D 46. E 85. D
8. A 47. C 86. C
9. D 48. D 87. A
10. C 49. B 88. D
11. A 50. B 89. C
12. E 51. A 90. A
13. A 52. E 91. B
14. C 53. C 92.
15. C 54. C 93.
16. E 55. D 94.
17. A 56. B 95.
18. D 57. B 96.
19. D 58. D 97.
20. D 59. C 98.
21. A 60. C 99.
22. E 61. A 100.
23. D 62. D
24. D 63. A
25. D 64. B
26. E 65. E
27. D 66. A
28. D 67. C
29. E 68. C
30. C 69. B
31. E 70. D
32. D 71. A
33. D 72. B
34. B 73. A
35. D 74. D
36. C 75. D
37. D 76. A
38. A 77. E
39. B 78. A

Dr. wessam alzaidat - General surgery


Summary of oncology medication &important points
Trastuzumab-
Her-2+
breast cancer Her-2+
Gastric CA Her-2+

Cetuximab - effective in wild type colon CA where is kras negative


but not effective in kras+
EGFR-colon CA
squamous cell head and neck

Imatinib is an oral tyrosine kinase inhibitor


GIST, ALL, CML; some leukemias

Erlotinib is an oral EGFR inhibitor


metastatic non-small cell
lung cancer with EGFR mutations.
unresectable pancreatic cancer.

DabRAFEnib is an oral kinase inhibitor


unresectable or metastatic melanoma with BRAF V600E
non-small-celllung cancer with BRAF

Melanoma
Drugs improve disease free- survival - Doxorubicin chemotherapy
Melanoma
Drugs improves survival for those with Mets:
)1(ipilimumab (antibody against cytotoxic T lymphocytic)
)2(vemurafenib or dabrafenib (selective BRAF kinase inhibitor)

Imiquimod, a nonspecific immune response modifier


superficial basal cell carcinomas

for GB ca after surgery give radio (fluourcil )


GB cancer more agrressive than cholangiocarcinoma

small bowel Cancer just surgical resection Alone !! nothing more

the commonest cause of splenic cyst worldwid is Hydatid disease

Canada syndrome- autoimmune IgG4 - Steroids - sporadic -


hamartomas - high malignancy risk

=Solitary juvenile polyps (sporadic)seem to have little propensity


for malignancy;

=juvenile polyposis( BMPR1A and SMAD4) familial


mucous lakes (pathology)
Rt colon
Hamartomas
Dr. wessam alzaidat - General surgery
up to 40% of patients with juvenile polyposis will develop a CRC.

PJS - STK11-gene on chromosome 19


in PJS Surgical treatment is limited to conservative
resection of the polyps through enterotomies or limited resection

sporadic colon CA is more frequent than familial

Lynch syndrome - adenomatous polyps

DUKE A confined to the wall


DUKE B penetration without LN
DUKE C LN
DUKE D METS

Lateral stalks contain middle rectal artery + pudendal n

periampullary carcinoid - Whipple

Arteriography: bleeding rate 0.5-1 mL/min. but best for pinpoint


the source
Tagged RBCs: bleeding rate 0.1 mL/min.
99mTc-SC: bleeding rate > 0.05-0.1 mL/min.

Morpheaform Basal cell carcinoma‫االسوء‬

The endoscopic appearances of Gastric lymphoma are vague and non


specific. Mucosal thickening is the Earliest sign

Diverticulosis - arterial bleed


angiodysplasia - venous bleed

for crohn's duodenal stricture (((((Do bypass )))not


stricturoplasty and not resection

FAP disease is transmitted


as an autosomal dominant trait, and therefore 50% of the offspring
of an afflicted individual will have the disease .
About 30%–40% of familial polyposis patients do not have a family
history of polyposis,
and these cases represent spontaneous mutations in the APC gene.

LN + rectal CA Chemotherapy and radiation followed by resection

The anal canal is the most common site of the development


of malignant melanoma of the alimentary tract.

Hx of FAP Female with intraperitoneal mass ? DEsmoid ? give


Dr. wessam alzaidat - General surgery
tamoxifine

Recurrence is Uncommon after surgical hemorrhoidectomy.

The QuantiFERON gold test is useful for diagnosing


both active and latent Mycobacterium tuberculosis infection.

The deep postanal space lies below the levator ani


muscles and posterior to the external sphincter muscles.

in Appendix The cause of the obstruction is usually:


lymphoid hyperplasia in younger patients
and fecaliths in adults.

Exemestane and ovarian suppression for 5 years


‫بدال ال‬Tamoxifine

perforated appendix case - undergone conservation with guided


draiange and abx
Adults who have not had a recent colonoscopy should undergo one
because up to
%5of patients have a cecal neoplasm.

abdominal pain during extension of the right thigh while lying on


his left
side -Retrocecal over the psoas muscle (Appendix)

The obturator sign is elicited


with passive external rotation of the flexed right hip. If
positive,
the obturator sign suggests that the inflamed tip is lying in the
pelvis.

Breast angiosarcoma ? mastectomy first Then adjuvent chemo

Carcinoid syndrome with liver Mets


Synchronous treatment with a right hemicolectomy and
radiofrequency ablation of the liver metastases is
appropriate.

when mucocele of the appendix is found at the time of surgery: Do


appendectomy

Histologically, mucoceles can be


categorized as a benign type, which is the result of occlusion of
the proximal lumen of the appendix, or a malignant type, which
is a variant of a mucous papillary adenocarcinoma. Treatment of
an appendiceal adenocarcinoma is right hemicolectomy.

Dr. wessam alzaidat - General surgery


adenocarcinoma of the appendix-
Early lesions confined to the mucosa or
submucosa (T0–T1) may be treated with a simple appendectomy as
long as there are clear margins.
Any more invasive lesions require a formal right hemicolectomy.
%55((((of patients present with metastatic disease at the time of
diagnosis))))

splenic abscess stable pt ? Splenectomy

CBD exploration - If a stone cannot be removed at surgery then


((((Do Bypass))) the chances of succeeding at ERCP are slim .
In this case, its probably best to ((bypass)) the distal bile duct
and a ((choledochoduodenostomy)) is the best way of achieving
this.

the highest risk of stroke if using the CHA2DS2-VASc risk


assessment tool? Age

Preoperative oral carbohydrate loading will ? reduce insulin


resistance.

cyclophosphamide is alkylating agent

chloarmbucil for polycythemia vera to prepare the pt for operation

Cross-matching should be done before Dextran administration.

every 1 Nitrogen gives 6.25 protein and 100 kcl carbs

hypoxic respiratory failure type 1


hypercapnic respiratory failure type 2

Atrial fibrillation is the most common postoperative cardiac


arrhythmia

delirium increased length of hospital stay.

which is the best predictor of


successful extubation? Rapid shallow breathing index (RSBI

most common site of heart Myxoma is Left Atrium

The 2,3-diphosphoglycerate level is (((((((increased)))))))))) in


pregnancy

.55Hyperthyroidism can be caused by all of the following except:


A. Graves' disease.
B. Plummer's disease.
C. Struma ovarii.
D. Hashimoto's disease.
(((((E.)))) Medullary carcinoma of the thyroid.

Dr. wessam alzaidat - General surgery


ERCP ERCP ERCP is The most definitive test for a lesion requiring
operative correction is demonstration of a
disrupted major pancreatic duct. ERCP ERCP ERCP

posterior knee dislocation - absent pulse ?? do reduction first

The Spigelman criteria


are used to grade duodenal polyposis seen in FAP .

Pseudomonas aerguinosa
is an opprotunistic infection bacteria
most common agent in aspiration pneumonia
give Tazo

FLam burn Does not cause inhalational injury

Small cell Lung Ca is more responsive to chemotherapy than NSCLC

follicular subtype of papillary thyroid Ca is the best prognosis

Ranula - mucus retention cyst from perforating submandibular and


sublingual gland through the mylohyoid muscle to the neck

mcc of HCC is Hep B


mcc of Liver transplant Hep C

de nove melanoma Nodular

branching microcalcification = comedo DCIS

P53 or any tumor supressor gene protect against neoplasm

IPMN suspected ?? Go for EUS

facial mass - histopathology showes biphasic apperance of the


lesion ?? Pleomorphic adenoma

Gastric CA without mets ? NO sleeve at all - do subtotal with D2


and roux y

NSAID use causes diaphragm disease (norrowing of small bowel )


pinhole bowel - recurrent obstruction

lynch syndrome patient which surgery to do -total colectomy with


ileorectal anastomosis

pregnant is at higher risk of sigmoid volvolus and treated with


Hartman

Dr. wessam alzaidat - General surgery


suspected Duodenal perforation Do gastrografin study

Glycin amino acid is found in all types of collagen

Dr. wessam alzaidat - General surgery


1. Which of the following is true of crush syndrome?

a. Is also referred to as traumatic squeeze


b. The urine is clear
c. The serum calcium level is usually elevated
d. Sensory and motor function is preserved in compressed limb
e. Serum creatinine, potassium and phosphate are elevated

Option 1

Which of the following incomplete spinal cord injury syndromes has the most potential for recovery:

A. Anterior cord syndrome


B. Central cord syndrome
C. Brown-Sequard syndrome
D. Posterior cord syndrome
E. Pyramidal syndrome

. Which of the following inflammatory mediators has been most closely associated with the magnitude of the
inflammatory response to blunt trauma and with the development of multiple organ dysfunction syndrome (MODS)?

a. Interleukin-1 (IL-1).
b. Beta human chorionic gonadotrophin (ß-HCG).
c. Tumour necrosis factor beta (TGF-ß).
d. Tumour necrosis factor alpha (TNF-a).
e. Interleukin-6 (IL-6).

4. Which of the following does NOT lead to a thrombotic tendency?

A. Factor V Leiden
B. Antithrombin deficiency
C. Protein C and S deficiency
D. Heparininduced thrombocytopenia
E. Anti-Protein Z deficiency

5. What is the risk of HIV seroconversion from a contaminated needlestick?

A 0.1%
B 0.3%
C 0.5%
D 1%
E 3%

6. Regarding gastric acid secretion, which is INCORRECT?

A. Cephalic phase of acid secretion is vagal dependent


B. Cephalic phase accounts for 60% of total acid secretion
C. Gastric phase begins when food reaches the stomach
D. Gastrin is the main component of the gastric phase
E. Gastrin, histamine and acetylcholine stimulate parieta

7. Regarding Von-Willbrand disease, which of the following is INCORRECT?

A. VW disease is the most common congenital bleeding disorder


B. Is classified into three types
C. Type I does not respond to desmopressin therapy
D. Type II may respond to desmopressin therapy
E. Menorrahagia is common in female with VW disease

8. Regarding the effect of aspirin on platelets, which is CORRECT?

A. Aspirin irreversibly induces thrombocytopenia


B. Effect of Aspirin lasts for 21 days
C. As furosemide, it inhibits ADP dependent platelets aggregation
D. Aspirin irreversibly inhibits platelets COX
E. Bleeding time is not affected

9. The abdominal part of esophagus receives its blood supply from:

A. Direct branch from the aorta


B. Branches from the bronchial artery
C. Inferior phrenic artery
D. Right gastric artery
E. Right gastroepiploic artery

Dr. wessam alzaidat - General surgery


10. The most potent stimulus for gastrin release is:

A. Acetylcholine
B. Histamine
C. Somatostastin
D. Hydrochloric acid
E. Luminal peptides and amino acids

11. All of the following hemodynamic parameters occur during CO2 pneumoperitoneum, EXCEPT:

A. Decrease in pulmonary vascular resistance


B. Decrease in cardiac preload
C. Increase in systemic vascular resistance
D. Increase in heart rate
E. Decrease in venous return

12. All of the following are signs of portal hypertension EXCEPT:

A. Ascites
B. Esophageal varices
C. Thrombocytosis
D. Rectal varices
E. Caput medusa

13. The most common geriatric cervical fracture is:

A. Jefferson fracture
B. Hangman fracture
C. Clay shoveler’s fracture
D. Odontoid fracture
E. Tear drop fracture

14. According to pack – year quantification of cigarette smoking if the patient smoking 16 cigarettes per day for 10
years that means:

A. 1 Pack year
B. 4 pack year
C. 8 pack year
D. 16 pack year
E. 32 pack year

15. All are major criteria for fat embolism except

A. Axillary or subcojunctival petechiae


B. Hypoxemia
C. CNS depression
D. Pulmonary edema
E. Urinary fat globules

Regarding acute osteomyelitis, the most common site involved is:

A. Upper and femur


B. Upper tibia
C. distal femur
D. Pelvis bone
E. Forearm.

The P value refers to:

A. The chance of the study demonstrating a significant association when, in reality, there is no significant association.
B. The chance of the study demonstrating no significant association when, in reality, there is a significant association .
C. The strength of the association between two variables.
D. The power of the study to demonstrate valid findings.
E. The chance of the study demonstrating a significant association when, in reality, there is a significant association.

.Which of the following is NOT absolute contraindication for contrasted MRI?

A. Programmable shunt valve


B. Pregnancy.
C. Placement of a vascular stent, coil or filter within the past 6 weeks.
D. Swann-Ganz catheter (pulmonary artery catheter)
E. Cardiac pacemakers, implanted neurostimulators, cochlear implants, infusion.

20. A pelvic hematoma might damage:

Dr. wessam alzaidat - General surgery


A. Lateral femoral cutaneous nerve
B. Femoral nerve
C. Obturator nerve
D. Sciatic nerve
E. Iliohypogastric nerve.

What is the main role of selective arterial embolization

A. Substantial reduction of tumor vascularity , making possible and safer tumor excision
B. Reducing tumor mass
C. Visualization of the angiographic pattern of the disease
D. Reduce epidural bleeding
E. Visualize the surrounding tissue

The intracranial tumor most likely to be encountered in a middle-aged man with the acquired immunodeficiency
syndrome (AIDS) is

A. Glioblastoma multiforme.
B. Lymphoma.
C. Meningioma.
D. Oligodendroglioma.
E. Ependymoma.

24. Which of the following statements is true regarding peripheral nerve injuries?

A. Neuropraxia is temporary loss of function without axonal injury; structure damage occur in most cases.
B. Axonotmesis is disruption of the axon and axon sheath associated with traumatic injury.
C. Neurotmesis is disruption of the axon with preservation of the axon sheath which usually preserves sensory and motor funct ion.
D. Electromyography (EMG) is useful in the early assessment of nerve injuries.
E. Regeneration in a peripheral nerve occurs at a rate of 1 mm/day, so improvement may not be obvious for many months .

The following relate to the half-life of different coagulation factors except:

A. Half-life of Factor VII is less than 5 hours.


B. Half-life of Factor V is around 25 hours.
C. Half-life of Factor VIII is around 10 hours.
D. Half-life of Factor IX is around 25 hours.
E. Half-life of Factor XI is around 45 hours.

28. Prothrombin Time (PT) is prolonged in:

A. Severe thrombocytopenia
B. Hemophilia A
C. Christmas Disease
D. Von Willebrand Disease
E. Over anticoagulation with Warfarin.

Once thawed, fresh frozen plasma must be transfused within:

A. 4 hours
B. 8 hours
C. 12 hours
D. 24 hours
E. 1 hour

Dr. wessam alzaidat - General surgery


1. A 60-year-old man presents for evaluation of a small right inguinal hernia. The hernia is easily
reducible and is not associated with untoward symptoms. It has not limited the patient's activities. The
patient would like to pursue nonoperative management for now. What is the most likely indication for
surgical management of the hernia in the future?
Cosmesis
Impotence
Pain
Strangulation
Bowel incarceration
Clear selection

2. A 54-year-old man presents with abdominal pain, elevated serum lipase, and a CT scan consistent
with acute necrotizing pancreatitis. Which of the following statements regarding the management of
necrotizing pancreatitis is true?
Immediate surgery is an independent predictor of poor outcomes.
. Management via the open-abdomen approach is preferred.
Anatomic resection and necrosectomy are associated with similar rates of postoperative exocrine and endocrine
insufficiency.
Percutaneous management cannot replace surgery.
The step-up approach is associated with more delay -related complications.

3. A 65-year-old man with painless jaundice undergoes a triple-phase pancreas protocol CT scan. An ill-
defined hypodense mass is seen in the pancreas phase in the pancreatic head. The mass extends to the
superior mesenteric artery with less than 180° abutment and has contact along the superior
mesenteric vein confluence with an irregularity of the vein. He is otherwise healthy. What is the next
step in his management?

Neoadjuvant chemoradiation therapy and restaging CT scan


Pancreaticoduodenectomy
Palliative biliary and enteric bypass
Percutaneous transhepatic biliary drainage
Pancreaticoduodenectomy with en bloc resection of superior mesenteric artery and superior mesenteric vein
with reconstruction

4. During the initial open repair of an 8-cm ventral hernia, the lowest risk of recurrence is with
primary repair with nonabsorbable suture.
sublay mesh repair with synthetic mesh.
sublay mesh repair with biologic mesh.
underlay mesh repair with synthetic mesh.
onlay mesh repair with synthetic mesh.

5. Which of the following is the most appropriate indication for hyperthermic intraperitoneal
chemotherapy in patients with metastatic gastrointestinal adenocarcinoma?
Intrahepatic metastasis
Biliary obstruction
Retroperitoneal lymphadenopathy
A fixed tumor shelf on the pelvis
Diffuse peritoneal tumor seeding

6. A 55-year-old healthy woman presents to the emergency department with lower abdominal pain. A
CT scan of the abdomen/pelvis shows a 2.5-cm mass in the pancreatic tail. Ultrasound-guided biopsy of
the pancreatic mass shows a low-grade neuroendocrine tumor. What is the recommended treatment?

Enucleation of the mass


Total pancreatectomy
Dr. wessam alzaidat - General surgery
Observation with repeat imaging in 6 months
Distal pancreatectomy with splenectomy
Octreotide

7. A 52-year-old man has chronic hepatitis B without evidence of cirrhosis. What is the most
appropriate way to screen for hepatocellular carcinoma?

Liver function tests


Alfa-fetoprotein
Ultrasound
CT scan
MRI

8. A 65-year-old woman undergoes a laparoscopic cholecystectomy for cholecystitis. Her final


pathology shows an adenocarcinoma invasive into the subserosal soft tissue, less than 1 mm from the
cauterized margin. A CT scan shows no residual or metastatic disease. The appropriate next step is

port-site excision.
chemoradiation.
resection of liver segments IVb/V with portal lymphadenectomy.
common bile duct resection with portal lymphadenectomy.
observation.

9. which of the following statements is true and can be used to counsel patients with thyroid disease
concerning the use of intraoperative nerve monitoring (IONM)?
Operative time is reduced.
Operative cost is reduced.
Injurv to the recurrent laryngeal nerve during the initial operation is decreased.
Recurrent laryngeal nerve injury is decreased in thyroid reoperative surgery.
Vagus nerve stimulation is a substitute for actual identification of the recurrent laryngeal nerve at surgery.

10. Which of the following statements is true regarding postoperative fluid management?
Intravenous fluids should be administered to maintain a urine output of mL/kg/hour.
Urine output of less than 0.5 mL/kg/hour is associated with increased hospital stay.
Decreased urine output is a physiologic response to surgical stress.
Fluid overload reduces postoperative morbidity.
Enhanced recovery after surgery pathways improve outcomes by fluid loading.

11. A 48-year-old woman with a history of deep venous thrombosis treated with rivaroxaban presents
with 12 hours of right upper quadrant pain. Physical examination, laboratory values, and ultrasound
are consistent with acute cholecystitis. Laparoscopic cholecystectomy is planned. Given her normal
renal function coagulation profile when will her coagulation profile normalize?
6 hours
12 hours
36 hours
72 hours
96 hours
12. A previously healthy patient presents with peritonitis secondary to perforated diverticulitis. After
adequate source control, she should receive 4 days of
2 points
gentamicin.
tigecycline.
clindamycin.
fluconazole.
piperacillin-tazobactam.

Dr. wessam alzaidat - General surgery


13-Which of the following should be monitored postoperatively in a patient with von Willebrand
disease who has undergone an elective abdominal operation?
Platelet count
Fibrinogen level
Von Willebrand Factor activity
Thromboelastography
Factor VIII activity
14. Heparin, all are true except:
A. Is a heterogeneous mixture of sulphated polypeptides
B. Potentiates the actions of antithrombin III
C. Has a half life of 90 min
D. Can be reversed by protamine sulphate
E. Can induce an idiosyncratic thrombocytopenia

15. Regarding malignant melanoma


A. The incidence is doubling every 10 years
B. Superficial spreading melanoma is the commonest clinico-pathological type
C. Tumour thickness is the most important prognostic indicator
D. Regional lymphadenectomy provides survival benefit in patients with 'intremediatethick' tumours
E. Acrallentiginoustumours are usually found on the trunk and proximal limbs
Clear selection
16. Regarding benign breast disease, which is false?
A. Cyclical mastalgia is the commonest reason for referral to the breast clinic
B. Fibroadenomas are derived from the breast lobule
C. Lactational breast abscesses are usually due to Staph aureus
D. Duct ectasia is less common in smokers
E. Atypical lobular hyperplasia is associated with an increased risk of breast cancer
17. Regarding Calcium, all of the following cause hypercalcemia except:
A. Hyperparathyroidism
B. Sarcoidosis
C. Pancreatitis
D. Thyrotoxicosis
E. Familial hypocalciurichypercalcemia
18. Staphylococcus aureus produces all of the following except:
A. Cell wall peptidoglycan
B. Enterotoxin
C. Epiderolytic toxin
D. Neuroexotoxin
E. Protease
19. The decreased insulin to glucagon ratio seen during simple starvation allows:
2 points
A. Increase lipogenesis
B. Increase lipolysis
C. Increase protein synthesis
D. Increase glycogen production
E. Decrease lipolysis
Clear selection
20. Which of the following disturbances is associated with tumor lysis syndrome?
A. Hypocalcemia
B. Hypouricemia
C. Hypokalemia
D. Hypomagnesemia
E. Hypophosphatemia

Dr. wessam alzaidat - General surgery


21. Regarding pancreatic anatomy, which of the following is correct?
A. The uncinate process is located anterior to the superior mesenteric artery
B. The blood supply of the pancreas is partly derived from the inferior mesenteric artery
C. The body of the pancreas lies to the right of the SMA
D. In 60% of cases, the ducts of Wirsung and Santorini empty into the duodenum independently
E. The splenic artery usually runs posterior to the pancreatic body

22. All of the following are relative contraindications to thrombolytic therapy except:
A. Pregnancy
B. Recent surgery (less than 10 days)
C. Recent trauma
D. Recent CVA(less than 2 months)
E. Liver disease
23. Concerning gangrenous cholecystitis, the most common site of perforation is:
A. The gallbladder neck.
B. The body of the gallbladder.
C. The areas were the gallbladder stone stuck.
D. The Hartman’s pouch.
E. The gallbladder fundus.
24. Which of following approaches has the best specificity to identify site of lower GI tract bleeding?
A. Tagged RBC scan
B. Mesenteric angiography
C. Surgical exploration
D. Barium enema
E. Abdomen CT with iv and oral contrast
Clear selection
25. A13 year old boy present to your office because of family history of medullary carcinoma of
thyroid. Physical examination is normal. Which of the following test should be performing?
A. VMA
B. Serum gastrin
C. Serum somatostatin
D. Serum insulin
E. Serum glucagon

26. In which of the following cases should a sentinel lymph node biopsy be performed?
A. A 0.7-mm melanoma over the right tibia with a clinically negative right groin
B. A 1.0-mm melanoma overlying the left groin
C. A 1.5-mm melanoma of the right upper arm and clinically negative right axilla
D. A 1.7- to 5-mm melanoma of the left thigh with palpable adenopathy of the left groin
E. A 0.6-mm melanoma of the left face

27. One of the following is not one of the bad effects of radiation in the human body:
2 points
A. Reddening of the skin (erythema)
B. Loss of hair.
C. Stiffening (fibrosis) of the lung.
D. Memory loss.
E. Cataract
28. One is false regarding the pelvic organs:
A. The ischiorectal fossa is bounded laterally by the fascia of the obturator internus muscle
B. The lower third of the rectum has no peritoneum coverage.
C. The urinary bladder is an intraperitoneal structure.
D. The sciatic nerve passes through the greater sciatic foramen into the gluteal region.
E. The blood supply to the prostate comes mainly from branches of the inferior vesicle artery.

Dr. wessam alzaidat - General surgery


29. Regarding the diaphragm, one is false:

A. The lateral arcuate ligament is a thickening of the fascia over the psoas muscle.
B. The costal part arises in slips from the lower six costal cartilages.
C. The caval opening is at eigth thoracic verterbra level.
D. The left phrenic nerve pierces the central tendon.
E. On posterior anterior chest radiography, the right dome is higher than the left by 2cm in most people.

30. All are true regarding the trachea except:

A. Extend anterior to third cervical vertebra till sixth cervical vertebra.


B. The thyroid cartilage forms the anterior and lateral border of the larynx.
C. The false cords are superior to the true cords.
D. The space between the false and true cords is called the vestibule.
E. The larynx is elliptical in shape at the level of the true cords

Dr. wessam alzaidat - General surgery


1. A 60-year-old man presents for evaluation of a small right inguinal hernia. The hernia is easily
reducible and is not associated with untoward symptoms. It has not limited the patient's activities. The
patient would like to pursue nonoperative management for now. What is the most likely indication for
surgical management of the hernia in the future?
Cosmesis
Impotence
Pain
Strangulation
Bowel incarceration
Clear selection

2. A 54-year-old man presents with abdominal pain, elevated serum lipase, and a CT scan consistent
with acute necrotizing pancreatitis. Which of the following statements regarding the management of
necrotizing pancreatitis is true?
Immediate surgery is an independent predictor of poor outcomes.
. Management via the open-abdomen approach is preferred.
Anatomic resection and necrosectomy are associated with similar rates of postoperative exocrine and endocrine
insufficiency.
Percutaneous management cannot replace surgery.
The step-up approach is associated with more delay -related complications.

3. A 65-year-old man with painless jaundice undergoes a triple-phase pancreas protocol CT scan. An ill-
defined hypodense mass is seen in the pancreas phase in the pancreatic head. The mass extends to the
superior mesenteric artery with less than 180° abutment and has contact along the superior
mesenteric vein confluence with an irregularity of the vein. He is otherwise healthy. What is the next
step in his management?

Neoadjuvant chemoradiation therapy and restaging CT scan


Pancreaticoduodenectomy
Palliative biliary and enteric bypass
Percutaneous transhepatic biliary drainage
Pancreaticoduodenectomy with en bloc resection of superior mesenteric artery and superior mesenteric vein
with reconstruction

4. During the initial open repair of an 8-cm ventral hernia, the lowest risk of recurrence is with
primary repair with nonabsorbable suture.
sublay mesh repair with synthetic mesh.
sublay mesh repair with biologic mesh.
underlay mesh repair with synthetic mesh.
onlay mesh repair with synthetic mesh.

5. Which of the following is the most appropriate indication for hyperthermic intraperitoneal
chemotherapy in patients with metastatic gastrointestinal adenocarcinoma?
Intrahepatic metastasis
Biliary obstruction
Retroperitoneal lymphadenopathy
A fixed tumor shelf on the pelvis
Diffuse peritoneal tumor seeding

6. A 55-year-old healthy woman presents to the emergency department with lower abdominal pain. A
CT scan of the abdomen/pelvis shows a 2.5-cm mass in the pancreatic tail. Ultrasound-guided biopsy of
the pancreatic mass shows a low-grade neuroendocrine tumor. What is the recommended treatment?

Enucleation of the mass


Total pancreatectomy
Dr. wessam alzaidat - General surgery
Observation with repeat imaging in 6 months
Distal pancreatectomy with splenectomy
Octreotide

7. A 52-year-old man has chronic hepatitis B without evidence of cirrhosis. What is the most
appropriate way to screen for hepatocellular carcinoma?

Liver function tests


Alfa-fetoprotein
Ultrasound
CT scan
MRI

8. A 65-year-old woman undergoes a laparoscopic cholecystectomy for cholecystitis. Her final


pathology shows an adenocarcinoma invasive into the subserosal soft tissue, less than 1 mm from the
cauterized margin. A CT scan shows no residual or metastatic disease. The appropriate next step is

port-site excision.
chemoradiation.
resection of liver segments IVb/V with portal lymphadenectomy.
common bile duct resection with portal lymphadenectomy.
observation.

9. which of the following statements is true and can be used to counsel patients with thyroid disease
concerning the use of intraoperative nerve monitoring (IONM)?
Operative time is reduced.
Operative cost is reduced.
Injurv to the recurrent laryngeal nerve during the initial operation is decreased.
Recurrent laryngeal nerve injury is decreased in thyroid reoperative surgery.
Vagus nerve stimulation is a substitute for actual identification of the recurrent laryngeal nerve at surgery.

10. Which of the following statements is true regarding postoperative fluid management?
Intravenous fluids should be administered to maintain a urine output of mL/kg/hour.
Urine output of less than 0.5 mL/kg/hour is associated with increased hospital stay.
Decreased urine output is a physiologic response to surgical stress.
Fluid overload reduces postoperative morbidity.
Enhanced recovery after surgery pathways improve outcomes by fluid loading.

11. A 48-year-old woman with a history of deep venous thrombosis treated with rivaroxaban presents
with 12 hours of right upper quadrant pain. Physical examination, laboratory values, and ultrasound
are consistent with acute cholecystitis. Laparoscopic cholecystectomy is planned. Given her normal
renal function coagulation profile when will her coagulation profile normalize?
6 hours
12 hours
36 hours
72 hours
96 hours
12. A previously healthy patient presents with peritonitis secondary to perforated diverticulitis. After
adequate source control, she should receive 4 days of
gentamicin.
tigecycline.
clindamycin.
fluconazole.
piperacillin-tazobactam.
13-Which of the following should be monitored postoperatively in a patient with von Willebrand
Dr. wessam alzaidat - General surgery
disease who has undergone an elective abdominal operation?
Platelet count
Fibrinogen level
Von Willebrand Factor activity
Thromboelastography
Factor VIII activity
14. Heparin, all are true except:
A. Is a heterogeneous mixture of sulphated polypeptides
B. Potentiates the actions of antithrombin III
C. Has a half life of 90 min
D. Can be reversed by protamine sulphate
E. Can induce an idiosyncratic thrombocytopenia

15. Regarding malignant melanoma


A. The incidence is doubling every 10 years
B. Superficial spreading melanoma is the commonest clinico-pathological type
C. Tumour thickness is the most important prognostic indicator
D. Regional lymphadenectomy provides survival benefit in patients with 'intremediatethick' tumours
E. Acrallentiginoustumours are usually found on the trunk and proximal limbs

16. Regarding benign breast disease, which is false?


A. Cyclical mastalgia is the commonest reason for referral to the breast clinic
B. Fibroadenomas are derived from the breast lobule
C. Lactational breast abscesses are usually due to Staph aureus
D. Duct ectasia is less common in smokers
E. Atypical lobular hyperplasia is associated with an increased risk of breast cancer

17. Regarding Calcium, all of the following cause hypercalcemia except:


A. Hyperparathyroidism
B. Sarcoidosis
C. Pancreatitis
D. Thyrotoxicosis
E. Familial hypocalciurichypercalcemia

18. Staphylococcus aureus produces all of the following except:


A. Cell wall peptidoglycan
B. Enterotoxin
C. Epiderolytic toxin
D. Neuroexotoxin
E. Protease

19. The decreased insulin to glucagon ratio seen during simple starvation allows:
A. Increase lipogenesis
B. Increase lipolysis
C. Increase protein synthesis
D. Increase glycogen production
E. Decrease lipolysis

20. Which of the following disturbances is associated with tumor lysis syndrome?
A. Hypocalcemia
B. Hypouricemia
C. Hypokalemia
D. Hypomagnesemia
E. Hypophosphatemia

Dr. wessam alzaidat - General surgery


21. Regarding pancreatic anatomy, which of the following is correct?
A. The uncinate process is located anterior to the superior mesenteric artery
B. The blood supply of the pancreas is partly derived from the inferior mesenteric artery
C. The body of the pancreas lies to the right of the SMA
D. In 60% of cases, the ducts of Wirsung and Santorini empty into the duodenum independently
E. The splenic artery usually runs posterior to the pancreatic body

22. All of the following are relative contraindications to thrombolytic therapy except:
A. Pregnancy
B. Recent surgery (less than 10 days)
C. Recent trauma
D. Recent CVA(less than 2 months)
E. Liver disease

23. Concerning gangrenous cholecystitis, the most common site of perforation is:
A. The gallbladder neck.
B. The body of the gallbladder.
C. The areas were the gallbladder stone stuck.
D. The Hartman’s pouch.
E. The gallbladder fundus.

24. Which of following approaches has the best specificity to identify site of lower GI tract bleeding?
A. Tagged RBC scan
B. Mesenteric angiography
C. Surgical exploration
D. Barium enema
E. Abdomen CT with iv and oral contrast

25. A13 year old boy present to your office because of family history of medullary carcinoma of
thyroid. Physical examination is normal. Which of the following test should be performing?
A. VMA
B. Serum gastrin
C. Serum somatostatin
D. Serum insulin
E. Serum glucagon

26. In which of the following cases should a sentinel lymph node biopsy be performed?
A. A 0.7-mm melanoma over the right tibia with a clinically negative right groin
B. A 1.0-mm melanoma overlying the left groin
C. A 1.5-mm melanoma of the right upper arm and clinically negative right axilla
D. A 1.7- to 5-mm melanoma of the left thigh with palpable adenopathy of the left groin
E. A 0.6-mm melanoma of the left face

27. One of the following is not one of the bad effects of radiation in the human body:
A. Reddening of the skin (erythema)
B. Loss of hair.
C. Stiffening (fibrosis) of the lung.
D. Memory loss.
E. Cataract
28. One is false regarding the pelvic organs:
A. The ischiorectal fossa is bounded laterally by the fascia of the obturator internus muscle
B. The lower third of the rectum has no peritoneum coverage.
Dr. wessam alzaidat - General surgery
C. The urinary bladder is an intraperitoneal structure.
D. The sciatic nerve passes through the greater sciatic foramen into the gluteal region.
E. The blood supply to the prostate comes mainly from branches of the inferior vesicle artery.

29. Regarding the diaphragm, one is false:

A. The lateral arcuate ligament is a thickening of the fascia over the psoas muscle.
B. The costal part arises in slips from the lower six costal cartilages.
C. The caval opening is at eigth thoracic verterbra level.
D. The left phrenic nerve pierces the central tendon.
E. On posterior anterior chest radiography, the right dome is higher than the left by 2cm in most people.

30. All are true regarding the trachea except:

A. Extend anterior to third cervical vertebra till sixth cervical vertebra.


B. The thyroid cartilage forms the anterior and lateral border of the larynx.
C. The false cords are superior to the true cords.
D. The space between the false and true cords is called the vestibule.
E. The larynx is elliptical in shape at the level of the true cords

Dr. wessam alzaidat - General surgery


1. Which of the following regarding inflammatory bowel disease is true:
A. Bloody diarrhea is more common in crohns disease than in ulcerative colitis.
B. Associated extra intestinal disease is less common in ulcerative colitis than in crohns disease
C. Crohns disease frequently involves the entire colon
D. Crohns is more commonly associated with primary sclerosing cholangitis than is ulcerative
colitis.
E. Surgical intervention for ulcerative colitis is limited to management of complications of the
primary disease process.
2. Which of the following statements concerning wound healing is true?
A. Keloids contain an overabundance of fibroblasts.
B. A hypertrophic scar extends beyond the boundaries of the original wound.
C. Improvement is usually seen with keloid excision followed by intralesional steroid injection.
D. An incision placed perpendicular to the lines of natural skin tension will result in the least
obvious scar.
E. Hypertrophic scars occur most commonly on the lower extremities.
3. Which of the following statement regarding intestinal obstruction is false?
A. In dynamic intestinal obstruction 60% are due to adhesions.
B. Proximal to an obstruction the gas is made up of 90% nitrogen and hydrogen sulphide.
C. Dehydration in intestinal obstruction is solely due to vomiting.
D. In dynamic obstruction the bowel distal to the obstruction initially functions normally.
E. In adynamic obstruction peristalsis is absent or inadequate.
4. The substrate depleted earliest in the postoperative period is:
A. Branched-chain amino acids
B. Non-branched-chain amino acids
C. Ketone
D. Glycogen
E. Glucose
5. Which of the following inhalation anesthetics accumulates in air-filled cavities during
general anesthesia?
A. Diethyl ether
B. Nitrous oxide
C. Halothane
D. Methoxyflurane
E. Trichloroethylene
6. Which of the following fractures or dislocations of the extremities induced by blunt
trauma is associated with significant vascular injuries?
A. Knee dislocation
B. Closed posterior elbow dislocation
C. Midclavicular fracture
D. Supracondylar femur fracture
E. Tibial plateau fracture
7. All of the following arises from the descending aorta except:
A. Nine pairs of posterior intercostal arteries.
B. Four-Five esophageal branches
C. Two to three bronchial arteries.
D. Left vertebral artery.
E. Mediastinal branches.

Dr. wessam alzaidat - General surgery


8. Which of the following is not a clinical feature of desmoid tumors?
A. Desmoid tumors are also called aggressive fi bromatosis .
B. These tumors are locally invasive and aggressive, and show fi broblastic proliferation.
C. Desmoid tumors are locally invasive and frequently metastasize.
D. They can develop in the abdominal wall of pregnant women, and form intraabdominal
mesenteric masses.
E. Abdominal desmoids may be a component of familial adenomatous polyposis Syndrome
9. Which one of the following matches is correct?
A. Krukenberg tumor-cul de sac implantation
B. Blummer shelf-ovarian implantation
C. Virchow’s node-periumbilical lymph node
D. Sister Mary Joseph nodule-inguinal lymph node metastasis
E. Irish nodule-axillary lymph node metastasis
10. Which of the following is not correct for testicular cancer?
A. AFP and b -hCG increase in 80–85% of extensive germ cell tumors.
B. b -hCG may cause false positivity in hypogonadism.
C. Pure seminomas have increased AFP.
D. AFP may increase in liver diseases and hepatitis.
E. Testis biopsy is not recommended for cases with cancer suspicion
11. What is the clinical nodal stage of a breast cancer patient with ipsilateral internal
mammary lymph node (+) with no axillary lymph node?
A. NX
B. N1
C. N2a
D. N2b
E. N3a
12. Which of the following is the most common symptom of nasopharyngeal cancer?
A. Nasal bleeding
B. Cervical mass
C. Dry eyes
D. Ophthalmoplegia
E. Diplopia
13. These are contents of the posterior triangle of the neck except:
A. Occipital artery.
B. Transverse cervical artery.
C. External jugular vein.
D. Hypoglossal nerve.
E. Brachial plexus.
14. Which is not true about PDS(polydioxine) suture:
A. It is monofilament.
B. Synthetic material.
C. Absorbable.
D. Good knot security.
E. Good tensile strength.
15. Which of the following tissues heals by repair :
A. Bone.

Dr. wessam alzaidat - General surgery


B. Liver.
C. Dermis.
D. Alimentary tract epithelium.
E. Tracheobronchial epithelium.
16. The spinal accessory nerve passes through the posterior cervical triangle:
A. Superficial to the platysma muscle.
B. In the superficial cervical fascia.
C. Between the investing fascia and the prevertebral fascia.
D. Within the carotid sheath.
E. Deep to the prevertebral fascia.
17. The fascia of neck, one is false:
A. The deep cervical fascia lies deep to the platysma.
B. The deep cervical fascia splits to invest trapezius muscle.
C. The anterior layer of the pretracheal fascia split to enclose the thyroid gland.
D. The pretracheal fascia split around the infrahyoid muscle.
E. The constituent of the ansa cervicalis are embedded in carotid sheeth.
18. The muscle which divides the submandibular space into two is:
A. Styloglossus.
B. Mylohyoid.
C. Digastric.
D. Omohyoid.
E. Sternocleidomastoid.
19. Which The cricoid cartilage lies at this vertebral level:
A. C2
B. C4
C. C6
D. C8
E. T2
20. The narrowest portion of an infant’s upper airway is :
A. The carina
B. The vocal cords
C. The subglottic space
D. The nasal airway
E. The pharynx
21. All the muscles of the larynx are innervated by recurrent laryngeal nerve except:
A. Cricothyroid.
B. Posterior cricoarytenoid
C. Lateral cricoarytenoid.
D. Interarytenoid.
E. Thyroarytenoid.
22.Concerning assessment of proliferative capacity, G2 phase include:
A. Preparation for mitosis
B. Normal cell activity phase
C. DNA synthesis
D. Preparation for DNA synthesis
E. mitosis

Dr. wessam alzaidat - General surgery


23. -Paraneoplasic syndrome includes all of the following statements EXCEPT:
A. Sub acute encephalitis
B. Medial temporal lobe, cingulate gyrus and insula are predominantly affected
C. It is more predominant in female
D. Anti-Hu-antibodies- cause sensory neuropathy and associated with oat cell pulmonary
carcinoma or lymphoma.
E. It is associated with small cell lung carcinoma
24. In case of brain abscess, which is incorrect?
A. Posterior fossa is a common site in infants
B. Total excision is often not the aim
C. Tapping of the abscess enhances antibiotic penetration
D. Posterior fossa is a common site in patients with AIDS
E. Commoner in children with congenital heart disease
25. Each of the following occurs in venous air embolism except
A. Decrease in cardiac output.
B. Increase in end-tidal pCO2.
C. Increase in pulmonary artery pressure.
D. Increase in pulmonary vascular resistance.
E. Ventilation-perfusion mismatch.

Dr. wessam alzaidat - General surgery


1.An 18-year-old man presents to the emergency department with a gunshot wound to the
left chest in the anterior axillary line in the seventh intercostal space. A rushing sound is
audible during inspiration. Immediate management is which of the following?

(A) Exploratory laparotomy


(B) Exploratory thoracotomy
(C) Pleurocentesis
(D) Closure of the hole with sterile dressing
(E) Insertion of chest tube
Lange (D)

2.Carotid bifurcation level?


C2
C3
C4
C6
Mrcs. ans (c4)

3. A 20-year-old woman presents to the emergency department with a stab wound to


the neck above the angle of the mandible. The patient’s blood pressure is 110/80 mm
Hg, pulse rate is 100 bpm, and respiration rate is 24 breaths per minute. Between
initial presentation and inser- tion of intravenous lines, the hematoma in the upper
neck enlarges significantly. What should be the next step in the patient’s
management?
(A) Barium swallow
(B) Flexible endoscopy
(C) Operative exploration
(D) Doppler ultrasound
(E) Angiography
Lange (E)

4. A 66-year-old man presents with acute severe pancreatitis.


Which of the following statements best describes the complications of acute
pancreatitis?
A Empirical antibiotic therapy is recommended for pancreatic necrosis associated
with ongoing pyrexia
B Immediate fine-needle aspiration of pancreatic necrosis >30% volume should be
obtained
C Pancreatic pseudocysts requiring surgical drainage require an open approach
D Staged laparotomies with open abdominal wound management systems
successfully manage pancreatic necrosis
E Sterile necrosis should be drained (surgically/radiologically); without drainage,
the sequelae will be worse
(D) FRCS recall

5. A 65-year-old man undergoing abdominal ultrasound scan is diagnosed with


hepatocellular carcinoma (HCC).
Which of the following statements best describes primary HCC?
A Alpha-fetoprotein and CT should be used as screening tools at regular intervals
B HCC screening is of value in the general population
C Hepatitis B virus is more carcinogenic than hepatitis C virus
D Men are relatively more at risk of HCC than women
E Most patients with HCC are part of screening programs and have early,
rdesectable disease.
(D) frcs recall
6. A 70-year-old woman with obstructive jaundice is diagnosed with
cholangiocarcinoma.
Which of the following statements best describes cholangiocarcinoma?
A 5-year survival rates of 25–40% exist for those undergoing complete, curative
resections
B Effective treatment options include chemoradiation for frail patients, or those
with non-resectable disease
C Liver transplantation can be considered as a therapeutic option in an
appropriate specialist centre
D No surgical options exist for locally advanced disease unlikely to achieve clear
resection margins
E The overall prognosis of this tumour has seen marked improvement in recent
times
(A) FRCS recall
7

8
9

10
11

12
13

14

15
16

17
18
Ans (B) lang

19
20
21

22

Dopamine action at high rate


23Least for contraction
Full thikness
24 Glucagonoma what is not a true?

25. A 74-year-old man presents with anorexia and self-limited hematemesis. During
endoscopy a mass is discovered and a biopsy is done. A hematopathologis diagnoses
non-Hodgkin’s lymphoma. What is the recommended therapy?
a. (A) Chemotherapy alone
b. (B) Immunotherapy
c. (C) Radiation and chemotherapy
d. (D) Surgery, radiation, and chemotherapy
e. (E) Surgery alone
Answer C Lange
26. Liver abscess of a previous alcohol man with liver trsnplanet 6 weeks ago
Staph
Salmonella
kleibsellia
C diff

27. Most common radtion thyroid cancer?


Papillary
Medullary
Follicular
Anaplastic
ans (papillary)

28. Most common to have erythrmaous breast and hottness


Female with breast feeding

29.female with 2 cm induration at the rt lower breast and she is bresr feeding with
blisster and fissure
Staph areius

30 descsion for tpn or entraal feeding based on?


Nutrional support needed
Risk of aspiration
Smal bowel non function

31given warfrin and later has symptoms?


Protein s
Protien c
(protien C ) ans

32 milk formula asa for gastric emptig


1 hours
2 hours
3 hours
4 hours
6 hours

33 9. All of the following are negative outcomes that have been


directly associated with perioperative hypothermia except:
A. Coagulopathy
B. Wound infections
C. Nosocomial pneumonia
D. Myocardial ischemia
E. Delayed wound healing
ANSWER: C
34. Question about fistula
CT cystogram

35.Burn victim in icu (chronic) most commin gastric disturbance


Curling ulcer
If acute (errosive gastritis)
By OF
1. Which of the following statements best describes early oesophageal cancer?
A. A Distinction between squamous and adenocarcinomas are not required
B. B Early oesophageal squamous cell carcinomas are common in the Western world
C. C It excludes pT1 lesions
D. D It excludes high-grade intraepithelial neoplasia
E. E The chances of a cure are excellent
2. A 66-year-old man has just been diagnosed with Barrett’s disease. Which of the
following statements best describes Barrett’s disease?
A. A Columnar mucosa is replaced by squamous mucosa
B. B Endoscopic mucosal resection and ablative therapy is appropriate treatment
C. C It is a congenital condition
D. D It is found in 5–10% of patients undergoing upper gastrointestinal endoscopy
E. E No surveillance is required for Barrett’s disease
3. A 48-year-old woman is diagnosed on oesophageal biopsies to have loss of ganglion
cells in the myenteric plexus. What is the most likely diagnosis?
A. A Achalasia
B. B Barrett’s disease
C. C Oesophageal cancer
D. D Pulsion diverticulum
E. E Sclerosis
4. Which of the following statements best describes gastric cancer?
A. A Gastrectomy and D2 lymphadenectomy should be performed routinely for all gastric
cancers
B. B Removal of station 10 lymph nodes include nodes along the splenic artery
C. C There is minimal additional risk associated with D2 lymphadenectomy for distal stomach
cancers
D. D Total gastrectomy with en-bloc pancreaticosplenectomy should be performed for distal
gastric cancers
E. E Total gastrectomy is not indicated for proximal cancers
5. A 69-year-old woman has undergone a gastrojejunostomy and complains
postoperatively that she has pain after eating food. This pain is relieved by vomiting which
is projectile and contains bile only. What is the most likely diagnosis?
A. A Chronic afferent loop syndrome
B. B Dumping syndrome
C. C Enterogastric reflux
D. D Gastro-oesophageal reflux
E. E Small stomach syndrome
6. A 66-year-old man has undergone a left carotid endarterectomy and develops
symptoms as a result of an iatrogenic nerve injury. Damage to which of the following
nerves would result in the accompanying neurological deficit?
A. A Facial nerve, resulting in drooping of the contralateral corner of the mouth
B. B Glossopharyngeal nerve, resulting in difficulty in swallowing
C. C Hypoglossal nerve, resulting in altered taste
D. D Superior laryngeal nerve, resulting in hoarseness and loss of effective cough
E. E Vagus nerve, resulting in diaphragmatic paralysis
7. A 22-year-old woman has marked lymphoedema in her right leg. What is the most
appropriate management?
A. A Diuretic therapy
B. B Elevation and elastic support stockings
C. C Excision of scarred skin and fibrotic subcutaneous tissue
D. D Lymphatic bypass
E. E Massage therapy or mechanical compression

Dr. wessam alzaidat - General surgery


8. A 65-year-old woman has a graft infection following a femoropopliteal bypass (using a
prosthetic graft). What is most likely causative organism?
A Escherichia coli
B Haemophilus
C Pseudomonas
D Staphylococcus
E Streptococcus

9. A 27-year-old man has been found to have a left-sided carotid body tumour. Which of
the following statements does not describe tumours of the carotid body?
A The tumours arise from paraganglion cells
B They are commonly benign
C They are commonly familial
D They are mobile side-to-side
E They are hard solid, painless highly vascular tumours
10. A 77-year-old woman undergoes a CT of the abdomen to stage her colorectal cancer.
This identifies an incidental visceral artery aneurysm. Which artery does a visceral arterial
aneurysm occur most commonly in?
A Coeliac
B Hepatic
C Inferior mesenteric
D Splenic
E Superior mesenteric
11. A 22-year-old man develops compartment syndrome in his left calf following a closed
tibial fracture. Which compartment is most commonly affected by compartment
syndrome?
A Anterior compartment
B Deep posterior compartment
C Lateral compartment
D Peroneal compartment
E Superficial posterior compartment
12. A 31-year-old woman has suffered from Graves’ disease for 8 years and is
increasingly troubled by thyroid eye disease. Her employment involves working with
young children. She required hospitalisation for neutropenia during an earlier course of
carbimazole, although she understands there are alternative drugs available. She wants
to be treated again and her thyroid function is consistent with toxicity. What is the most
appropriate treatment option?
A Beta-blockade, then total thyroidectomy, pretreated with Lugol’s iodine
B Beta-blockade, then subtotal thyroidectomy, pretreated with Lugol’s iodine
C Propylthiouracil medication
D Radiation therapy to her eyes
E Radioiodine
13. A 40-year-old woman presents to thyroid outpatient clinic with tiredness, weight loss
and heat intolerance; she has used lithium in the past for bipolar disease. She complains
of a generalised neck swelling, and there is a family history of thyroid disease. On
examination, she is clinically hyperthyroid, with lid lag and agitation. On examination, she
has a smooth, palpable goitre with one dominant swelling (3 cm) in the right lobe. What is
the most likely diagnosis?
A Autoimmune thyroid disease
B Colloid degeneration
C Graves’ disease
D Lithium-induced thyroid disease
E Toxic adenoma

Dr. wessam alzaidat - General surgery


14. A 52-year-old man who is heparin naive is started on subcutaneous unfractionated
heparin (UFH) for postoperative deep venous thromboembolism prophylaxis. The next
day, his platelet count falls from 260,000/mm3 to 102,000/ mm3 (150,000-400,000/mm3).
Management of his deep vein thrombosis prophylaxis should entail which of the
following?
A. Discontinue UFH and start low molecular weight heparin.
B. Continue UFH.
C. Discontinue UFH and start fondaparinux.
D. Discontinue UFH.
E. Discontinue heparin and start argatroban.
15. A 55-year-old man undergoes a laparoscopic sigmoid colectomy for diverticulitis.
Which of the following interventions is associated with a decreased rate of surgical site
infection in this patient?
A. Intraoperative normothermia
B. Prophylactic antibiotics for 48 hours
C. Iodine-impregnated adhesive drapes
D. Preoperative chlorhexidine shower
E. Incisional gentamicin sponge
16. A 52-year-old patient is critically ill in the intensive care unit 5 days after urgent
hepatic resection for bleeding hepatocellular cancer. He suddenly develops bright red
blood in one of the surgical drains. Intravascular contrast for CT angiography is planned
as a diagnostic study to consider angioembolization of the source of bleeding. What
strategy is effective in reducing the risk of contrast associated acute kidney injury?
A. Isotonic saline before and after
B. Sodium bicarbonate after
C. Sodium bicarbonate during contrast infusion
D. Hypertonic saline before and after
E. Albumin after
17. A 7-year old girl is admitted after she was a restrained passenger in a T-bone motor
vehicle crash with impact on her side of the car. She is tachycardic and has an acute
abdomen. At surgery, she has an isolated duodenal perforation in the second portion
opposite the ampulla of Vater involving 25% of the circumference. What is the best
operative plan?
A. Primary repair of the duodenum
B. Pyloric exclusion
C. Repair the duodenum, gastrostomy, jejunostomy, and duodenostomy
D. Duodenal diverticularization
E. Gastrojejunostomy, nasogastric decompression, nasojejunal feeding tube
18. A 31-year-old man presents after a gunshot wound to the right buttock. Exam under
anesthesia identifies blood in the rectum at 8 to 10 cm from the dentate line concerning
for an extraperitoneal rectal injury. In addition to confirming, an extraperitoneal rectal
injury, what is the most appropriate next step in this patient's management?
A. Direct repair
B. Direct repair with proximal diversion
C. Proximal diversion
D. Proximal diversion and presacral drain placement
E. Proximal diversion, presacral drain placement, and distal rectal washout
19. In nerve injuries, poor outcomes are associated with which of the following :
A. Sharp injury
B. Young age
C. Proximal injury
D. Early intervention
E. Distal injury

Dr. wessam alzaidat - General surgery


20. Injury to which of the following is the most frequent cause of life-threatening pelvic
bleeding in trauma patients?
A. Internal iliac artery
B. External iliac artery
C. Cancellous bony surfaces
D. Pudendal artery
E. Common iliac vein
21. Which of the following incomplete spinal cord injury syndromes has the WORST
prognosis for recovery?
A. Anterior cord syndrome
B. Posterior cord syndrome
C. Central cord syndrome
D. Brown-Séquard syndrome
E. Nerve root injury syndrome
22. The goal of resuscitation to maximize survival and minimize morbidity in patients with
shock include all the following except
A. Urine output more than 0.5 mL/kg/h
B. CVP 8 to 12 mm Hg
C. MAP 65 to 90 mm Hg
D. ScvO2more than 70%
E. Arterial pH 7.25
23. Which of the following is the most common single organ system injury associated with
death in children?
a. Abdominal injury
b. Cervical injury
c. Head injury
d. Thoracic injury
e. Lumbar injury
24. Which of the following is the least important when performing a Nissen fundoplication
for reflux disease?
A. Use of pledgets to prevent suture tears
B. Lengthening the intraabdominal esophagus
C. Division of the short gastric vessels
D. Hiatal dissection and closure
E. Short and floppy fundoplication around the esophagus with a bougie
25. Regarding liver transplantation for patients chronically infected with HCV, which of the
following statements is Not true?
A. The most common indication for liver transplantation worldwide is HCV infection.
B. After liver transplantation, these patients uniformly become re-infected within a matter of days.
C. Posttransplant infection with HCV cause cirrhosis in approximately 5% to over 20% of patients
at 5 years after liver transplantation.
D. Posttransplant treatment of HCV is a contraindication due to the extensive drug interactions
between DAAs and immunosuppressive agents.
E. The most common of death and re-transplantation in recipient with HCV infection is
recurrence of hepatitis C.

Dr. wessam alzaidat - General surgery


1- Arterial injuries that can be safely ligated in the unstable trauma
patient include all except :

*a- SMA
b- radial art
c- inferior mesentric art
d- internal iliac artery
e- celiac art

2-in which of the following venous injuries is ligation most


tolerated :

*a- infra renal vena cava


b- supra renal vena cava
c- common femoral vein
d- popliteal vein
e- innominate vein

3-following blunt abdominal trauma , mandatory exploration


is indicated for any non expanding hematoma identified on
CT – scan in which of the following areas :

a- Rt . perinephric
*b- mid line infra mesocolic
c- lateral pelvic area
d- retro hepatic
e- lt. perinephric

4-which structure is the most sensitive to hypoxia?

a-skin
b- bone
*c-non myelinated nerve
d- myelinated nerve
e- skeletal muscle

Pulmonary artery catheter is used to measure which variable


-directly :

a-cardiac index
*b- systemic vascular resistance
c- mixed- venous oxygen saturation
d- left ventricular end diastolic index
e- pulmonary vascular resistance index

Dr. wessam alzaidat - General surgery


6-An example of anti-inflammatory cytokine is:
a-IL-2
b-IFN-y
c-Lymphotoxin-alpha
*d-IL-4
e-TNF-alpha

7-Trendelenburg position considered a viable treatment


option for which category of shock?

a-Cardiogenic
*b-Neurogenic
c-Hypovolemic
d-Septic
e-Cardiac compressive

8-All of the following symptoms are seen with


hypermagnesemia except:

*a-Tachypnea
b-Depression of reflexes
c-Arrythmias
d-hypotension
e-Central nervous system depression

9-The most likely cell of origin for gastrointestinal


stromal tumor(GIST):

*a-Interstitial cells of Cajal


b-Kulchitsky cells
c-Myofibroblasts
d-G-cells
e-Paneth cells

Dr. wessam alzaidat - General surgery


10-Which of the following regarding inflammatory bowel
disease is true:

a-Bloody diarrhea is more common in crohns disease than


in ulcerative colitis.
*b-Associated extra intestinal disease is less common in
ulcerative colitis than in crohns disease
c-Crohns disease frequently involves the entire colon
d-Crohns is more commonly associated with primary
sclerosing cholangitis than is ulcerative colitis.
e-Surgical intervention for ulcerative colitis is limited to
management of complications of the primary disease
process.

11-Each of the following is associated with familial


polyposis syndrome except:

a-Gardners syndrome
b-Turcots syndrome
*c-MEN type 2b
d-Peutz-Jeghers syndrome
e-Juvenile polyposis syndrome.

12-Surgical exploration for patient with primary


hyperparathyroidism reveals all four glands to be:
enlarged.What is the most appropriate way to manage?

a-Closure with localization study


b-Incision biopsy of all glands
*c-Subtotal parathyroidectomy
d-Excision of the largest enlarged gland
e-Selective venous PTH sampling.

Dr. wessam alzaidat - General surgery


-The most common clinical manifestation of MEN 1 is:
13

a-Hypoglycemia.
b-Renal stones
c-Galactorrhea
d-Osteoporosis
*e-Peptic ulcer disease.

14-In MEN 1, the most common pancreaticodoudenal


tumors are
:
a-Gastrinomas
*b-Non functioning tumors
c-Insulinomas.
d-Somatostatinomas.
e-VIPomas.

15-Cholecystokinin:
*a-Relaxes the sphincter of oddi.
b-Inhibits gastric acid secretion.
c-Inhibits gall bladder contraction.
d-Causes mesenteric vasodilatation.
e-Inhibits pancreatic exocrine secretions.

16-Rectus sheath hematomas


*a-Can be caused by coughing.
b-Are rarely associated with anticoagulative therapy.
c-Usually occur at the semicircular line of Douglas at the
entry site of superior epigastric artery into the rectus sheath.
d-Are infrequently palpable on physical examination.
e-Usually require operative drainage.

Dr. wessam alzaidat - General surgery


17-Most primary and metastatic tumors to liver derive
nearly all their vascular inflow from branches of:
A-Portal vein.
b-Collateral circulation.
*c-Hepatic artery.
d-Celiac axis.
e-Unnamed branches of aorta.

Which of the following is not a complication of Caroli


-disease:
a-Biliary stone formation.
b-Recurrent cholangitis.
c-Septicemia.
d-Cholangiocarcinoma.
*e-Renal disorders.

19-Which of the following is not considered a risk factor


for gall bladder cancer?
a-Gall stones>3cm in size.
*b-Multiple small gall stones.
c-porcelain gall bladder
d-Biliary salmonella typhi infection.
e-Gall bladder adenoma.

20-Which is true about surgical anatomy of pancreas:


a-The head lies over the first lumber vertebra.
b-The accessory duct enters the second part of duodenum.
c-The superior mesenteric vessels pass behind the uncinate
process.
*d-The SMA lies to the right of the superior mesenteric vein.
e-The superior mesenteric vein join the portal vein behind
the head of pancreas.

Dr. wessam alzaidat - General surgery


21-Most common indication for splenectomy for a red
cell enzymatic defect is:
a-Hereditary spherocytosis.
b-G6PD.
*c-Private kinase deficiency.
d-Hereditary high red blood cell phosphatidyl choline
anemia.
e-Cold-agglutinin syndrome.

22-Which of the following is true of retroperitoneal soft


tissue sarcomas:

a-The most common histological cellular type is malignant


fibrous histiosarcoma.
b-Pre-operative biopsy is contraindicated because of tumor
seeding risk.
*c-Resection of contaguous organs that are not involve with
tumor is indicated to obtain adequate margins.
d-Death usually results from distant metastases.
e-Lung is the most common site of distant metastases.

23-Which of the following structures form the medial


border of the (triangle of doom) to be avoided in
laparascopic inguinal herniorrhaphy?

a-Inferior epigastric vessels.


*b-Vas deferens.
c-Spermatic vessels.
d-Iliac vessels.
e-Lateral femoral cutaneous nerve.

24-A 45 year old man who has been HIV positive for 15 years has a
painful 3-cm neck mass. A 2 week course of antibiotics does not change
the mass. Biopsy will most reveal:

a-Hodgkin’s lymphoma.
*b-B cell lymphoma.
c- T cell lymphoma.
d- Metastatic lung carcinoma.
e- Kaposi’s sarcoma.

Dr. wessam alzaidat - General surgery


25-Risk factors for male breast cancer include:
a-Gynecomastia
b-BRCA 1 gene mutation
*c- BRCA 2 gene mutation
d- Anabolic abuse.
e- Cannabis abuse.

26-A 2-cm gastric ulcer in the antrum of the stomach is associated with
all of the following except:

a-Helicobacter pylori.
*b-Increased acid secretion.
c- Malignancy.
d-nonsteroidal anti-inflammatory drugs.
e-Atrophic gastritis.

27-The most common presenting clinical sign in Budd-Chiari syndrome


is:
*a-Ascitis.
b-Esophageal varices.
c-Jaundice.
d-Encephalopathy.
e-Hemorroids.

28-Percutaneous image guided drainage of intra abdominal abscesses


is LEAST successful in which of the following sites?

a-Subphrenic.
b-Hepatic.
*c-Peripancreatic.
d-Diverticular.
e-Appendiceal.

Dr. wessam alzaidat - General surgery


29-In the workup on a patient for possible appendicitis, CT scanning should be
performed:
a) Before consulting the surgeon, by the emergency physician
b) In patients with equivocal physical findings
c) Routinely, in all patients with right lower quadrant pain
d) With equal frequency in men and women
e) Never

b—

30-The gastric mucosal cell that secretes intrinsic factor is the


a) G cell
b) Parietal cell
c) D cell
d) Enterochromaffin-like cell
d) Chief cell

b—

31-Hypertrophic pyloric stenosis is likeliest to occur in a(n)


a) Firstborn child
b) African-American infant
c) Child 6 to 9 months of age
d) Female infant
e) Infant born prematurely

a—

32-Overwhelming postsplenectomy infection (OPSI) a) Occurs more frequently


after resection for trauma than hematologic disease
b) Occurs with equal frequency in children and adults
c) Is most frequently caused by Streptococcus pneumoniae
d) Usually occurs within 2 years after splenectomy
e) Generally has an identifiable site of infection

c—

33-The characteristic feature of Crohn's colitis that best distinguishes the


clinical entity from ulcerative colitis is
a) Perianal disease
b) Rectal bleeding
c) Risk of malignancy
d) Obstructive symptoms
e) Pseudopolyps

a—

Dr. wessam alzaidat - General surgery


34-Type 1 gastric ulcers are
a) At the incisura
b) Along the greater curvature
c) Prepyloric
d) Associated with simultaneous duodenal ulcers
e) Close to the esophagogastric junction

a—

35-Radiation enteritis
a) Usually presents with perforation
b) Is caused by thrombosis of mucosal vessels
c) Occurs after 3,000 cGy of abdominal radiation
d) Routinely requires operative therapy
e) Is likely in patients who have undergone laparotomy

e–

36-The small bowel tumor with the greatest propensity for bleeding is
a) Carcinoid
b) Lymphoma
c) Adenocarcinoma
d) Hamartoma
e) Leiomyoma

e—

37-Gastric stump cancers (gastric adenocarcinomas in patients who


underwent gastric resection)

a. Are the consequence of Helicobacter pylori infection


b. Are generally of the polypoid type
c. Have a 5-year survival rate of 40%
d. Occur within 5 years after gastrectomy
e. Are caused by bile reflux

e—

38-A grade III laceration of the spleen is characterized by

a. Intraparenchymal hematoma > 5 cm


b. Capsular tear 1-3 cm deep
c. Hilar vessel disruption
d. Subcapsular hematoma involving 10%-50% of surface area
e. Active hemorrhage

a—
Dr. wessam alzaidat - General surgery
39-The spleen filters all of the following particles/cells EXCEPT

a. Malformed erythrocytes

b. T lymphocytes

c. Malarial parasites

d. Streptococcus pneumoniae

e. Platelets

b—

40-Acute appendicitis

a. Occurs most commonly in the second and third decades of life


b. Can be cured readily by antibiotics
c. Is most commonly caused by a fecalith
d. Carries an overall mortality rate of 7%
e. Induces leukocytosis in 90% of patients

41-The stomach bed does not include :

A. Splenic artery.
B. Celiac trunk.
C. Transverse mesocolon.
D. Left adrenal gland.
E. Neck of the pancreas.

E
42-which of the following is the strongest of all other risk factors in the
development of Gastric carcinoma :

A. Helicobacter pylori.
B. Atrophic gastritis.
C. Blood group A.
D. Pernicious anemia.
E. Low socioeconomic class.

Dr. wessam alzaidat - General surgery


43-whilst performing a small bowel resection for strictures following crohn”s
disease, you realize that on inspection, there are marked differences between
jejunal and ileal anatomy. Such differences include all the following Except:

A. Wider lumen in the jejunum.


B. Less lymphatics in the jejunal mesentry compared to ileal.
C. More prominent and multiple arcades of vessels in the ileum.
D. Thicker wall of the jejunum.
E. Thicker and more fat-laden mesentry increasing towards the ileum.

44-A 54 years old woman is referred to your surgical team with a diagnosis of
small bowel obstruction. Which one of the following clinical signs would you
look for in trying to identify the commonest cause of this condition :

A. surgery scar.
B. Lump in the groin above & medial to the pubic tubercle.
C. Lump in the groin below & lateral to the pubic tubercle.
D. Cachexia & nodule at the umbilicus.
E. Circumoral pigmentation & a family history of previous obstruction.

45-True statements regarding appendiceal neoplasms include which of the


following?

A. Carcinoid tumors of the tip of the appendix less than 1.5 cm are adequately
treated by simple appendectomy
B. Appendiceal carcinoma is associated with secondary tumors of the GI tract in up
to 60% of patients
C. Survival following right colectomy for a Dukes’ stage C appendiceal carcinoma is
markedly better than that for a similarly staged colon cancer at 5 years
D. Mucinous cystadenocarcinoma of the appendix is adequately treated by simple
appendectomy, even in patients with rupture and mucinous ascites
E. Up to 50% of patients with appendiceal carcinoma have metastatic disease, with
the liver as the most common site of spread

:A

Dr. wessam alzaidat - General surgery


46-All of the following statements about carcinoma of the gallbladder are
correct Except :

A. The neoplasm usually starts in the cystic duct and neck of the gallbladder.
B. It is found more commonly in women than men.
C. It is associated with the presence of gallstones in > 85% of cases.
D. Prognosis is generally poor with < 1 year survival with local invasion.
E. Chemotherapy and radiotherapy do not alter disease progression.

47-All of he following statements are true regarding diverticular disease


Except :

A. It is found more commonly in the developed world.


B. Surgical treatment is usually unnecessary in acute uncomplicated cases.
C. Diverticulae are more commonly found in the descending colon.
D. Perforation and fistula formation can result from an attack of acute diverticulitis.
E. Resolution of the diverticulae can occur with high fiber diet and adequate
hydration.

48-All of the following arteries are branches of the superior mesenteric


artery, except:

a. Ileocolic
b. Replaced left hepatic
c. Inferior pancreaticoduodenal
d. Jejunal
e. Replaced right hepatic

b—

Dr. wessam alzaidat - General surgery


49-Mesenteric cysts

a. Occur primarily in the mesocolon


b. Are discovered mainly in children
c. Can contain chyle or serous fluid
d. Require total enucleation for a cure
e. Are neoplastic

c—

50-Decompression for abdominal compartment syndrome should be


performed

a. If the urine output falls to 30 mL/hour


b. When the patient's respiratory rate increases to 24 breaths per minute
c. Based purely on physical findings
d. When bladder pressure exceeds 35 mm Hg
e. If the patient becomes hypoxemic

d—

51-In laparoscopic surgery, the vessel most likely to be punctured during


trochar placement is the

a. Aorta
b. Right common iliac artery
c. Vena cava
d. Left common iliac artery
e. Right common iliac vein

b—

52-A walled-off pelvic abscess secondary to sigmoid diverticulitis is Hinchey


stage

a. I
b. II
c. III
d. IV
e. V

b—

Dr. wessam alzaidat - General surgery


53-Following resolution of acute appendicitis, an interval appendectomy

a. In 90% of interval appendectomies and no pathologic abnormality.


b. Always reveals luminal occlusion of the appendix
c. Should be performed because of the high incidence of recurrent appendicitis
d. Is not indicated in patients older than 40 years
e. Needs to be performed open because of the fibrosis induced by appendicitis

54-True statements concerning the diagnosis and management of


retroperitoneal fibrosis include all of the following except:

A. Most patients present with dull, non-colicky back, flank, or abdominal pain
B. Evidence of impaired renal function with an elevated blood urea nitrogen is
common
C. The diagnosis is most commonly suggested by intravenous pyelography although
contrast studies with CT scan or MRI are useful in further defining the disease
D. Most patients will need operative intervention.
E. The prognosis for nonmalignant retroperitoneal fibrosis is grim with progression of
disease until death occurring in most patients

:E

55-Lynch Syndrome all are true except?

a) It is due to mutation in MMR gene

b) Associated with APC gene

c) It has Autosomal Dominant Inheritence

d) Adenomas in patients with Lynch syndrome display high grade dysplasia


than adenoma in patients with sporadic colorectal cancer?

e) It occurs predominantly on right side, has increased incidence of


synchronous and metachronous disease.

Dr. wessam alzaidat - General surgery


56-About chronic pancreatitis all are true except?

a) Parlington Rochelle operation is longitudanal Pancreaticojejunostomy

b) Whipple Pancreaticoduodenectomy removes 60% of pancreatic parenchyma


and has 25% incidence of Diabetes

c) Frey's operation is Duodenal Presereving Pancreatectomy

d) Beger's operation is Duodenal Preserving Pancreatectomy

e) is a different disease entity from acute pancreatitis

57-Which of the folowing about Pancreatic Ascites is not true

a) Conservative treatment effective in only 1/4th of patients

b) ERCP should be done before surgery

c) It is exudative

d) Metaplastic cells are present

e) Pancreatic fluid has high amylase and high albumin

58-Tetanus all are true except?


a. Is due to an infection with a gram-negative spore forming rod
b. The organism produces a powerful exotoxin
c. The toxin prevents the release of inhibitory neurotransmitter
d. Clostridium tetani is sensitive to penicillin
e. Risus sardonicus is the typical facial spasm

Dr. wessam alzaidat - General surgery


59-Regarding gas gangrene
a. It is due to Clostridium botulinum infection
b. Clostridial species are gram-negative spore forming anaerobes
c. The clinical features are due to the release of protein endotoxin
d. Gas is invariably present in the muscle compartments
e. antibiotics alone are the main stay of treatment

60-A 34-year-old morbidly obese diabetic woman underwent a gastric


bypass about 12 hours ago. The operation was technically difficult but
finally went well. You are called because she now has a temperature of 99.2‫؛‬
F, pulse of 134, and some pain in her incision and her back. She looks well;
the incision is clean; and her examination is otherwise negative. A bolus of
500 ml. of dextrose/lactated Ringer's did not change her vital signs, except
that her pulse rose to 140 without an increase in urine output. Your next
step should be:

A. Another bolus of crystalloids.


B. Posterioanterior and lateral chest films.
C. Obtain white cell count, differential count, and electrolyte values.
D. Call the operating room and warn them that you need to re-explore for a leak.
E. Increase her pain medication.

Answer: D

61-All of the following statements about the embryology of Meckel's


diverticulum are true except:

A. Meckel's diverticulum usually arises from the ileum within 60 cm. of the ileocecal
valve.
B. Meckel's diverticulum results from the failure of the vitelointestinal duct to
obliterate.
C. The incidence of Meckel's diverticulum in the general population is 5%.
D. Meckel's diverticulum is a true diverticulum possessing all layers of the intestinal
wall.
E. Gastric mucosa is the most common ectopic tissue found within a Meckel's
diverticulum.

Answer: C

Dr. wessam alzaidat - General surgery


62-A 43-year-old woman presents with complaints of anal pain and spotting of
blood with defecation. Physical examination reveals a 2 3 cm area of ulceration
within the anal canal. The remainder of the physical examination is normal.
Incisional biopsy is positive for squamous cell carcinoma. Appropriate
management includes which of the following?

A. Abdominoperineal resection
B. Wide local excision, skin grafting, proximal diverting colostomy
C. Primary radiation therapy
D. Local excision and primary closure
E. chemotherapy

Answer: C

63-The initial goal of therapy for acute toxic cholangitis is to:

A. Prevent septicemia by decompressing the duct system.


B. Remove the obstructing stone, if one is present.
C. Alleviate jaundice and prevent permanent liver damage.
D. Prevent the development of gallstone pancreatitis.
E. Prepare the patient for urgent cholangiography.

Answer: A

64-The clinical picture of gallstone ileus includes all of the following Except?

A. Air in the biliary tree.


B. Small bowel obstruction.
C. A stone at the site of obstruction.
D. Acholic stools.
E. Associated bouts of cholangitis.

Answer: D

65- The following Nyhus classification of hernias is correct except for:


A. Recurrent direct inguinal hernia—Type IVa.
B. Indirect inguinal hernia with a normal internal inguinal ring—Type I.
C. Femoral hernia—Type IIIc.
D. Direct inguinal hernia—Type IIIa.
E. Indirect inguinal hernia with destruction of the transversalis fascia of Hesselbach's
triangle—Type II.
Answer: E

66- The following statements about the repair of inguinal hernias are true
except:

Dr. wessam alzaidat - General surgery


A. The conjoined tendon is sutured to Cooper's ligament in the Bassini hernia repair.
B. The McVay repair is a suitable option for the repair of femoral hernias.
C. The Shouldice repair involves a multilayer, imbricated repair of the floor of the inguinal
canal.
D. The Lichtenstein repair is accomplished by prosthetic mesh repair of the inguinal canal
floor in a tension-free manner.
E. The laparoscopic transabdominal preperitoneal (TAPP) and totally extraperitoneal
approach (TEPA) repairs are based on the preperitoneal repairs of Cheattle, Henry, Nyhus,
and Stoppa.
Answer: A

67-Which of the following statements is true concerning the diagnosis and


management of epigastric hernias?

A. A large peritoneal sac containing abdominal viscera is common


B. At the time of surgical repair, a careful search for other defects should be performed
C. Recurrent epigastric hernias after simple closure is uncommon
D. Patients with symptoms of a painful midline abdominal mass frequently will contain
incarcerated small bowel
Answer: b

68- Chylous ascites is the accumulation of chyle within the peritoneal cavity.
Which of the following statement(s) is/are true concerning chylous ascites?

a. The cisterna chyli lies at the anterior surface of the first and second lumbar vertebrae
and receives lymphatic fluid from the mesenteric lymphatics
b. Chylous ascites is most commonly associated with abdominal lymphoma
c. Paracentesis and analysis of chylous fluid typically reveals elevated triglycerides,
protein, and leukocyte levels. The cytology is seldom positive despite the presence of
malignancy.
d. Treatment of chylous ascites with dietary manipulation will be successful in most cases
e. The mortality rate in adults with chylous ascites is in excess of 50%

Answer:d

69- A 48-year-old woman maintained on Warfarin for a history of cardiac


valvular replacement and a history of recent upper respiratory infection
presents with severe abdominal pain exacerbated by movement. Her physical
examination shows tenderness in the right paramedian area with voluntary
guarding but no peritoneal signs. Which statement is true concerning the
diagnosis and management of this patient.

a. Urgent laparotomy should be performed because of concern for arterial mesenteric


embolus
b. The correct diagnosis could likely be made by CT scan and operation avoided
c. The status of her anticoagulation should be checked and if her prothrombin time is
excessively prolonged,surgery is necessary

Dr. wessam alzaidat - General surgery


d. If untreated, hemodynamic instability is common
e. The usual cause is severe trauma.
Answer: b

70-True statements concerning the diagnosis and management of


retroperitoneal fibrosis include:

a. Most patients present with dull, non-colicky back, flank, or abdominal pain
b. Evidence of impaired renal function with an elevated blood urea nitrogen is common
c. The diagnosis is most commonly suggested by intravenous pyelography although
contrast studies with CT scan or MRI are useful in further defining the disease
d. Most patients can be managed nonoperatively
e. The prognosis for nonmalignant retroperitoneal fibrosis is grim with progression of
disease until death occurring in most patients
Answer: a, b, c

- In biliary atresia:
a. Medical treatment can be curative.
b. Operative success for bile drainage improved if surgery delayed after 3 months.
c. its rarely progress to cirrhosis.
d. indirect bilirubin level above 2 mg\dl is suggestive of the diagnosis.
e. liver biopsy shows inflammation and proliferation of the bile ducts.
Answer is: E

2- The best of the following to diagnose Hirschsprung’s disease is:


a. Aganglionosis of dilated bowel segment.
b. Presence of hypertrophied nerve trunks in rectal biopsy specimen.
c. Barium enema examination.
d. Failure to pass meconium in the first 24 hours.
e. Relaxation of internal sphincter during anorectal manometry study.
Answer is: B

3- A infant with intussusception:


a- A pathologic lead points is usually identified.
b- The site of origin is usually in the small bowel.
c- Triad of colicks, red currant Jelly stool and palpable abdominal mass presents in less than
50% of cases.
d- Surgical resection is mostly required.
e- Peaks around 4 years of age.
Answer is: C
4- 10 years boy presented to ER with abdominal pain after minor trivial blunt Lt lower chest wall
trauma, on exam: the HR 130, BP 70/40, the chest exam was unremarkable, the abdomen distended
and tender all over and absent bowel sounds. His mother reported history of cervical
lymphadenopathy with Flu like illness 2 weeks ago. Most likely this child’s scenario resulted from:
a- Mesenteric lymphadenitis.
b- Acute non obstructive appendicitis.
c- Meckle’s diverticulitis.
d- Splenic injury.
e- Fractured rib.
Answer is: D

Dr. wessam alzaidat - General surgery


5- 3 years old girl noticed by her mother to have a lump around her labia that’s firm and tender
which can’t be pushed back in and the physician appreciated tenderness in Rt lower quadrant, she
most likely have:
a- Acute appendicitis
b- Inguinal lymphadenopathy.
c- Incarcerated ovary in indirect inguinal hernial sac.
d- She experienced sexual abuse.
e- Bartholin gland abscess.
Answer is: C
6- A 5 year old child who suffered from vomiting for 2 days, when presented he was lethargic,
tachypnic and febrile and his abdominal exam revealed generalized tenderness and rigidity without
localization, the next step:
a- Order an abdominal CT scan with oral contrast and IV contrast.
b- Rule out intussusception by Barium enema.
c- Start broad spectrum IV antibiotics and observe.
d- Diagnostic lap. and accordingly.
e- Consider rigid esophagoscopy.
Answer is: D

7- A 6 years old girl with anterior neck mass with overlying redness which rises when she sticks
out her tongue, the mom reports same presentation 2 months earlier which improved with oral
antibiotic but persisting lump, which of the following is true:
A- Prepare for surgery to incise and drain the presumed abscess.
B- Excise the body of hyoid bone during surgery to avoid recurrence.
C- Start on IV steroids to decrease the edema.
D- Tracheostomy is mandatory.
E- Secure the airway with endotracheal tube.
Answer is: B
8- On the 30th day of a healthy looking newborn, he started to have persistent strongly projectile
non-bilious vomiting after feeding for 2 days, you suspect hypertrophic pyloric stenosis, which of
the following is true:
a- An impalpable abdominal mass rules out the above diagnosis.
b- A double bubble sign on X-ray film.
c- Immediate Pyloromyotomy must be done to resume feeding soon.
d- Rule out other important congenital anomalies (VACTERL).
e- Ultrasonography is superior to upper GI series.
Answer is: E

9- All the following are associated with Tracheoesophageal fistula except:


a- Congenital heart disease.
b- Hemivertebrae.
c- Omphalocele.
d- Horseshoe kidney.
e- Imperforate anus.
Answer is: C
10- The best management of irreducible inguinal hernia in a 2 months old infant is:
a- Reduction if successful and follow up till the age of one year.
b- Reduction if successful and surgery after 2-5 days.
c- Immediate surgical management.
d- Consider mesh repair to prevent recurrence.
e- Reduction if successful and surgery after 1 month.
Answer is: B

Dr. wessam alzaidat - General surgery


1. which of the following amino acids is a critical nutrient supporting the
immune function:
a) Taurine.
b) Branched chain amino acids (BCAA).
c) Alanine. *
d) Arginine.
e) Tyrosine.

2. The optimal initial treatment of aspiration pneumonia is:


a) Bronchoalveolar lavage with antacids.
b) Bronchoalveolar lavage with saline.
c) Parenteral corticosteroids.
d) Maintenance of oxygenation. *
e) Early prophylactic antibiotics.

3. All are true about ultrasound, EXCEPT:


a) Can detect a lesion less than 1 cm in the liver. *
b) Is useful about differentiating a thyroid cyst.
c) It helps in the diagnosis of acute appendicitis.
d) Its not the best modality to visualize the pancreas.
e) Investigation of choice in GB stones.
4. Which statement is correct regarding neurapraxia:
a) The injury involves the axon without loss of continuity of nerve sheath.
b) Momentary loss of conduction through the nerve without any organic
lesion. *
c) Partial or complete division of the nerve.
d) Spontaneous recovery is impossible.
e) Wallerian degeneration occurs.

5. The most important factor in the development of pressure ulcers in a spinal


cord injury patient is:
a) Malnutrition.
b) Septic episodes.
c) Anemia.
d) Local pressure. *
e) Diminished sensation.

6. The earliest sign of Volkmann's ischemic contracture is:


a) Absence of the radial pulse.
b) Pain on passive finger extension. *
c) Loss of voluntary finger motion.
d) Absent sensation in the median nerve distribution.
e) Pallor of the nail beds.

Dr. wessam alzaidat - General surgery


7. Regarding the oxygen dissociation curve, one is TRUE:
a) Is shifted to the left by increase 2,3-DPG.
b) Is sigmoid shape for both hemoglobin and myoglobin.
c) Is shifted to the left when one ascends to high altitude.
d) Is shifted to the right by an increase in the PH.
e) Is ideally 50% saturated at a PO2 of 28mmHg. *

8. Regarding insulin, all are true EXCEPT:


a) It is a polypeptide composed of two amino acid chains.
b) Its secretion is affected by glucagon.
c) It stimulates gluconeogenesis. *
d) It has little or no effect on the uptake of glucose in the brain.
e) If lacking, it may be a cause for fatty liver.

9. Which of the following statements regarding hemorrhoids is NOT TRUE:


a) Hemorrhoids are specialized "cushions" present in every one to aid
continence.
b) External hemorrhoids are covered by skin, while internal hemorrhoids are
lined by mucosa.
c) Pain is often associated with uncomplicated hemorrhoids. *
d) Haemorrhoidectomy is reserved for third and fourth degree hemorrhoids.
e) They are classified and treated according to the degree of their symptoms.
10. Infection affects wound healing by all the following mechanisms EXCEPT:
a) Prolonging oedema.
b) Decreasing tissue PO2.
c) Increasing collagenolysis.
d) Decreasing the inflammatory phase. *
e) Increasing the inflammatory phase.

11. All the followings may be linked to the etiology of malignant melanoma
EXCEPT:
a) Fair complexion.
b) Intermittent sun exposure.
c) Positive family history.
d) Junctional nevus. *
e) Dysplastic nevus.

12. The most reliable indicator of the adequacy of burn resuscitation is:
a) Central venous pressure.
b) Pulmonary capillary wedge pressure.
c) Urine output. *
d) Blood pressure and heart rate.
e) Mental status.

Dr. wessam alzaidat - General surgery


13. The most common site for squamous cell carcinoma of the lip is the:
a) Upper lip midline.
b) Upper lip laterally.
c) Lower lip midline.
d) Lower lip laterally. *
e) Oral commissure.
14. As the amount of total body fat increase, the total body water:
a) Increases.
b) Decrease. *
c) Unchanged.
d) Depends on patient's age.
e) Depends on patient's sex.
15. One week of starvation results in a loss of approximately 100 gm of nitrogen
(13.8 gm N/ day). This is associated with protein loss of:
a) 625 gm. *
b) 100 gm.
c) 1000 gm.
d) 50 gm.
e) 16 gm.
16. The HIGHEST potassium content is in:
a) Saliva. *
b) Gastric juice.
c) Bile.
d) Ileal.
e- Duodenal
17. During vessel wall injury, the initial step in clot formation is:
a) Local mast cells release of adenosine diphosphate (ADP).
b) Fibrin polymerization.
c) Plasminogen activation.
d) Platelet adherence to sub endothelial collagen. *
e) None of the above.
18. Postoperative staphylococcal wound infection usually detected in:
a) 1st to 2nd day.
b) 3rd to 4th day.
c) 5th to 8th day. *
d) 9th to 14th day.
e) After the 14th day.
19. A palpable radial pulse indicates a systolic blood pressure greater than:
a) 40 mmHg.
b) 60 mmHg.
c) 80 mmHg. *
d) 100 mmHg.
e) 120 mmHg.

Dr. wessam alzaidat - General surgery


20. What is the first parameter to change in early hypovolemic shock?
a) Systolic blood pressure.
b) Pulse rate. *
c) Diastolic blood pressure.
d) Respiratory rate.
e) Level of consciousness.
21. A previously healthy 18 years old man involved in automobile accident. He
lost consciousness, but regains it and appeared normal. An hour later he
became somnolent and has weakness on his right side, with dilated pupil on
the left side. Most likely he has:
a) Acute epidural hematoma. *
b) Acute subdural hematoma.
c) Subarachnoid hemorrhage.
d) Intraventricular hemorrhage.
e) Brain concussion.
22. Concerning factors that increase the risk of pulmonary embolism, all are
correct EXCEPT:
a) Superficial phlebitis one year ago. *
b) Estrogen therapy.
c) Obesity.
d) Pregnancy.
e) Deep venous thrombosis one year ago.

23. The TRUE statement regarding small intestine is:


a) The entire small intestine is intraperitoneal.
b) The jejunum is longer in length, larger in diameter, and thinner-walled than
ileum.
c) The muscularis, the muscle layer, provide strength for placement of sutures
for creation of a bowel anastomosis.
d) Peyer’s patches are most prominent in the distal ileum *
e) The marginal artery of Drummond provides blood supply to the duodenum.

24. A 40 years old female had a subtotal thyroidectomy, several hours later she
complained of difficulty in breathing and she had stridor and a markedly
swollen, tense neck wound the first step in the management of this patient
should be to:
a) Intubate with an endotracheal tube.
b) Perform a tracheostomy.
c) Control the bleeding site in the operating room.
d) Open the wound and evacuate the hematoma at bed side *
e) Aspirate the hematoma.

Dr. wessam alzaidat - General surgery


25. Patients with acute pancreatitis who are at increased risk to develop
complications include those with each of the following EXCEPT:
a) Ransons score> 3.
b) An APACHI II score >12.
c) Poorly enhancing areas of 50% of the pancreas on a dynamic CT scan.
d) Amylase values more than eight times upper limits of normal. *
e) Partial obliteration of the lesser sac and acute fluid collections near the
splenic hilum and inferior to pancreatic head.

26. Regarding acute suppurative ( toxic ) cholangitis, one is TRUE :


a) Third generation cephalosporin are the antibiotic of choice for patients with
renal impairment.
b) Urgent decompression of the biliary tree is indicated. *
c) Emergency surgical decompression is indicated for patients with known
cholilithiasis.
d) The catheter for percutaneous drainage should be placed through the common
duct into the duodenum.
e) Nasobiliary tube decompression is an effective form of drainage for
obstructing lesion in the upper third of the common duct.
27. The most common presenting symptom in colorectal carcinoma in patients
under age of 40 is:
a) Abdominal pain.
b) Weight loss.
c) Back pain.
d) Rectal bleeding. *
e) Abdominal distension.

28-A 24 years old man has appendectomy, histopathological report notes acute
inflammation with a 1.2 cm carcinoid tumor on the mid of appendix. This
patient should have:

f) No further treatment. *
g) Chemotherapy.
h) Right hemicolectomy.
i) Serial urinary 5- hydroxyindole acetic levels.
j) Regional radiation therapy.
29- Gastric ulcers are:
A- Malignant when located in the greater curvature.
B- More common in women.
C- Most commonly complicated by perforation.*
D- Associated with hypersecretion of acid in over 90% of cases.
E- Not associated with Helicobacter pylori.

Dr. wessam alzaidat - General surgery


30- Which is the most commonly cultured hospital acquired organism in critical
care patients with aspiration pneumonia?
a- Streptococcus pneumoniae
b- Staphylococcus aureus
c- Anaerobic species
d- Pseudomonas aeruginosa*
e- Haemophilus influenzae
31- Which of the following will immediately delay or cancel an elective surgical
case if not obtained appropriately preoperatively?
a- CBC
b-Urinanalysis
c- CXR
d- Informed consent*
e- ECG
32- The most common cause of nipple discharge is
a- carcinoma
b- fibrocystic disease
c- intraductal papilloma*
d- duct-ectasia
e- trauma
33- ALL OF THE FOLLOWING ARE SINGS OF ATROPINE POISONING
EXCEPT
a) dry mouth
b) red face
c) small pupils *
d) tachycardia
e) drowsiness
34- PAIN IS CARRIED TO THE BRAIN BY THE FOLLOWING TRACT
a) spino cerebellar tract
b) spino thalamic tract *
c) cortico- spino tract
d) reticular tract
e) olvospinal tract
35- A CETYLE CHOLINE IS THE CHEMICAL TRANSMITTER IN
a) all; pre and post ganglionic sympathetic system
b) all; pre and post ganglionic parasy pathetic system *
c) all post ganglionic sympathetic system
d) in adrenal medulla
e) all pre ganglionic sympathetic system
36- ALL OF THE FOLLOWING AFFECTING NERVE
CONDUCTION EXCEPT:
a) myelination of the nerve
b) nerve fiber diameter
c) hypoxia
d) autonomic nervous system *
e) inter nodal distance
Dr. wessam alzaidat - General surgery
37- THE MOST HELPFULL CLINICAL SIGN FOR SPINAL CORD TUMOUR
AT T10 LEVEL IS:
a) exaggerated of knee jerk
b) anesthesia up to umbilicus level *
c) urine retention
d) anesthesia up to hypogastrium
e) dissociated sensory loss
38- THE EARLY TREATMENT OF CERVICAL SPINES INJURY INCLUDES
ALL OF THE FOLLWING EXEPT:
a) immobilization by neck collar
b) the patient should be covered
c) lateral spine x-ray while patient on stretcher
d) change the patient position to take off his clothes in E.R *
e) treatment of spinal shock
39- THE MOST IMPORTANT FUNCTION OF INTERVERTEBRAL DISC IS:
a) Preserve spinal curvature
b) Protects spinal cord
c) Shock absorber *
d) Transmit weight of the body to the lower limbs
e) Has no significant function on adult age
40 – BLOOD SUPPLY OF SPINA LCORD IS USUALLY COME FROM :
a) vertebral arteries only
b) ant , post spinal arteries and radicular arteries *
c) vertebral arteries and abdominal aorta
d) vertebral arteries and ascending cervical arteries only
e) radicular arteries from the aorta only
41- FRACTURE OF THE L 4 VERTEBRAE WITH LOWER LIMBS
WEAKNESS IS DUE TO :
a) spinal cord contusion at that level
b) only dura injury
c) dura and nerves injury *
d) spinal cord , dura and nerves injury
e) spinal concussion without any nerve injury
42 – BARO RECEPTORS IS LOCATED IN :
a) skin
b) wall of the great vessels in the neck *
c) heart
d) in the brain at the medulla level
e) in the vessels of the brain

43- THE MOST SERIOUS COMPLICATION WHICH AFFECT THE


SEQUALE OF HEAD INJURY IS DUE TO:
a) brain infection and brain abscess
b) brain hypoxia *

Dr. wessam alzaidat - General surgery


c) secondary bleeding
d) decrease intracranial pressure
e) CSF leakage
44-A LADY IN CHILDBEARING AGE WITH REPEATED PREGNANCES
AND HAS POOR INTAKE OF ANIMAL DIET IS PRONE TO:
Osteoporosis
a) Osteomalacia *
b) Rickety bone
c) Vitamin B deficiency
d) Compressed spinal fractures
45- ALL OF THE FOLLOWING STRUCTURES PASSING UNDER THE FLEXOR
RETINACULUM EXCEPT:
Flexor Digitorum longus
a) Flexor Digitorum brevis
b) Median nerve
c) Flexor indices
d) Palmaris longus *
46- THE ORIGION OF SYMPATHETIC SYSTEM FROM THE SPINAL
CORD IS:
Cranio cervical outflow
a) Thoraco lumbar outflow *
b) Cranio sacral outflow
c) Only lumbar outflow
d) Whole spinal cord
47-METABOLIC ACIDOSIS MAY BE BROUGHT BY:
a) loss of CO2 by increase ventilation
b) retention of CO2 by respiratory obstruction
c) persistent vomiting
d) absorption of excessive amount of Na HCO3
e) excessive exercise *

48-THE MOST POTENT STIMULANT OF RESPIRATION WOULD BE:


a) a two fold increase in PCO2 of inspired air *
b) a two fold increase in PO2 of inspired air
c) a 50% decrease in PCO2 of inspired air
d) a 50% decrease in PO2 of inspired air
e) a & d are equally potent stimuli
49-The psoas major muscle
a. flexes the thigh at the hip. *
b. extends the thigh at the hip.
c. adducts the thigh at the hip.
d. abducts the thigh at the hip.
e. assists in full contraction of the diaphragm

Dr. wessam alzaidat - General surgery


50-The renal arteries typically branch from the abdominal aorta a the
level of the:

a. 12th thoracic vertebral body.


b. first lumbar vertebral body.
c. second lumbar vertebral body. *
d. third lumbar vertebral body.
e. fourth lumbar vertebral body.

51-A 20–year–old man is undergoing retroperitoneal dissection for a


testicular germ cell tumor. The inferior mesenteric artery is divided
during reflection of the intestines to expose the retroperitoneum. This
can be expected to result in:

a. ischemia of the descending colon.


b. ischemia of the sigmoid colon.
c. ischemia of the rectum.
d. ischemia of the splenic flexture
e. none of the above. *

52-The cremaster muscle is innervated by:

a. the ilioinguinal nerve.


b. the iliohypogastric nerve.
c. the obturator nerve.
d. the genital branch of the genitofemoral nerve. *
e. the femoral branch of the genitofemoral nerve.

53-As one proceeds outward from the adrenal medulla, the three
separate functional layers of the adrenal cortex are, in correct order:

a. the zona reticularis, zona fasciculata, then zona glomerulosa. *


b. the zona fasciculata, zona reticularis, then zona glomerulosa.
c. the zona glomerulosa, zona fasciculata, then zona reticularis.
d. the zona glomerulosa, zona reticularis, then zona fasciculata.
e. the zona reticularis, zona glomerulosa, then zona fasciculata.

54-Proceeding from posterior to anterior, the structures encountered in


the renal hilum are, in correct order:

a. the renal artery, renal vein, and renal pelvis.


b. the renal pelvis, renal artery, and renal vein. *
c. the renal pelvis, renal vein, and renal artery.
d. the renal vein, renal artery, and renal pelvis.
e. the renal artery, renal pelvis, and renal vein.

Dr. wessam alzaidat - General surgery


55-During inguinal hernia repair in a male patient, the ilioinguinal nerve
is injured in the canal, which will most likely produce:

a. anesthesia over the dorsum of the penis.


b. anesthesia over the pubis and scrotum, and loss of cremasteric
contraction.
c. anesthesia over the pubis and anterior scrotum only. *
d. anesthesia over the anterior and medial thigh.
e. anesthesia over the pubis only.

56-Hematuria is distinguished from hemoglobinuria or myoglobinuria


by:

a. dipstick testing.
b. the simultaneous presence of significant leukocytes.
c. microscopic presence of erythrocytes. *
d. examination of serum.
e. evaluation of hematocrit.

57-Glucose will be detected in the urine when the serum level is above:

a. 75 mg/dl.
b. 100 mg/dl.
c. 150 mg/dl.
d. 180 mg/dl. *
e. 225 mg/dl.

58-What proportion of the cardiac output is delivered to the kidney?

a. 5%
b. 20% *
c. 50%
d. 85%
e. 100%

59-The predominant buffering system in humans is:

a. bicarbonate. *
b. titratable acids.
c. ammonium (NH4+).
d. sodium lactate
e. phosphate

Dr. wessam alzaidat - General surgery


60-After a 7–hour–long, complex urethral reconstruction performed in
the extended lithotomy position, the patient has severe thigh and
buttock pain. The creatine phosphokinase levels are dramatically
elevated. To prevent ARF, the next step should be:

a. dopamine infusion.
b. plasmapheresis.
c. dobutamine infusion.
d. forced alkaline diuresis. *
e. percutaneous nephrostomy.

61-The renal structure at greatest risk for ischemic injury is the:

a. vasa recta.
b. cortical collecting duct.
c. juxtaglomerular apparatus.
d. medullary thick ascending loop of Henle. *
e. distal convoluted tubule.

62-Acute pyelonephritis is best diagnosed by:

a. chills, fever, and flank pain. *


b. bacteriuria and pyuria.
c. focal scar in renal cortex.
d. delayed renal function.
e. vesicourethral reflux.

63-The most reliable early clinical indicator of septicaemia is:

a. chills.
b. fever.
c. hyperventilation.*
d. lethargy.
e. change in mental status.

64-Which of the following is true regarding testosterone?

a. Testosterone is synthesized by the Sertoli cells of the testes.


b. Testosterone is synthesized by the Leydig cells of the testes.*
c. Testosterone is a direct precursor of pregnenolone.
d. 5α–Reductase is an enzyme that converts DHT into testosterone.
e. Aromatase converts estrogens into testosterone

Dr. wessam alzaidat - General surgery


65-Calcium is maximally absorbed in which portion of the
gastrointestinal tract?

a. Stomach
b. Jejunum
c. Jejunum and proximal ileum *
d. Ileum
e. Ascending colon

66- the most common type of congenital diaphragmatic hernia is caused by:
a. a defect in the central tendon
b. eventration of the diaphragm in the fetus
c. a defect through the space of Larry
d. a defect through the pleuroperitoneal fold. *
e. all of the above.
67- The calorie-nitrogen ratio for infant should be maintained at:
a. 75:1
b. 100:1
c. 50:1
d. 150:1 *
e. 25:1

68- A patient has undergone an ileal resection . Which of the following


conditions would he be least likely to develop?

a- alopecia *

b- megaloblastic anemia

c- nephrolithiasis

d- cholelithiasis

e- steatorrhea

69.With regard to Hirschsprung’s disease ( aganglionosis), all the following are


true except:
a- It is more common in males.
B. It may be complicated by enterocolitis.
C. Barium enema study may be normal.
D. It is best diagnosed by full thickness rectal biopsy.
E .Surgery can’t be accomplished without colostomy. *

Dr. wessam alzaidat - General surgery


70. In regard to malignant hyperthermia, one is true.
A. It triggered by Depolarizing muscle relaxing agents *
B. Non polarizing muscle agents are contra indicated .
C. Dantroline must be given after induction as prophylaxis.
D. Manitole and fluid support are not useful to prevent renal failure.
E. Alkalosis is the main metabolic disturbance.
71. Incompatible blood transfusion during surgery under general anesthesia,
which of the following is NOT true.
A. Unexplained bleeding.
B. Hematuria may be present.
C. Skin rash may be seen
D. Hypertension is well know problem in spite transfusion *
E. Volume support is mandatory.
72. The most common cause of death in Paediatric age group is :
A. Infection
B. Trauma. *
C. Malignancy.
D. Congenital anomalies.
E. Malnutrition.
73-In the management of pyloric stenosis, which of the following is true.
A. Acidosis should be corrected first.
B. Chloride level is normal in spite alkalosis.
C. Hypokalaemia should be corrected. *
D. Usually there is compensatory hypocarbia.
E. Paradoxical aciduria is not a feature.
74 - The ligamentum teres represents an obliterated
a- Ductus venosus
b- Ductus arteriosus
c- Internal iliac artery
d- Umbilical vein *
e- porta hepatis
75- Tributaries of the portal vein include all of the the following except:
a- Superior mesenteric
b- Para umbilical
c- uterine *
d- pyloric
e- lienal ( splenic )

76- The following statements are true regarding the liver anatomy except :
a- Is attached to the diaphragm and anterior abdominal wall by the Falciform
ligament
b- Is totally covered by peritoneum *
c- Drains by hepatic veins into the inferior vena cava
d- Has a lymph drainage to both the mediastinal and porta hepatic nodes
e- Is directly related to the right suprarenal gland

Dr. wessam alzaidat - General surgery


77- An animal is in negative nitrogen balance when :
a- The intake exceeds output
b- New tissue is being synthesized
c-Output exceeds intake *
d- Intake is equal to output
e- The urine is nitrogen – free
78- Bilirubin secreted into the intestine is subjected to enzymatic degradation ,
the final product being :
a- Biliverdin
d- Bilirubinogen
c- Urobilinogen
d- Stercobilin *
e- Mesobilirubinogen
79- In mammals , Norepinephrine is synthesized from :
a- Pyruvate
b- Arginine
c- Catechol
d- Tyrosine *
e- Tryptamine
80- Tissue graft within the same individual is known as :
a- Allograft
b- Isograft
c- Autograft *
d- Xenograft
e- Non of the above
81- The most effective means of sterilization by heat is :
a- steam
b- Boiling
c- Hot air
d- Fractional sterilization
e- Steam under pressure *
82- One of the following statements is true regarding Exotoxins:
a- Heat stable
b- Lipopolysaccharide in nature
c- part of cell wall of becteria
d- protein in nature *
e- less potent than endotoxins

83- Gall stones associated with pernicious anemia are most likely to be :
a- pure cholesterol
b- Calcium bilirubinate *
c- Calcium carbornte
d- Mixed gall stones
e- combined gall stones

Dr. wessam alzaidat - General surgery


84- In familial polyposis coli, polyps tend to appear most frequently :

a- At birth
b- During the first decade
c-during the second and third decades of life *
d- During the fourth decades of life
e- After age of 40

85- The substance involved with protein synthesis

a- DNA
b- RNA *
c- Adenine
d- Guanine
e- Cytosine

86- In surgical skin wound 80% of tensile strength of unwounded skin is


restored by :
a- 1 week
b-2 weeks
c- 4 weeks
d- 12 weeks *
e- 24 weeks

87- Calcitonin sometimes is elaborated by which one of the following tumors :


a-Medullary carcinoma of the breast
b- Medullary carcinoma of the thyroid *
c- Giant cell carcinoma a of pancrease
d- Carcinoid tumor of appendix
e- Ganglioneuromas

88- Normal " prothrombin time " as measured by usual clinical tests depends on
normal levels of all of the following factors excepts :
a- Factor II
b- Factor V
c- Factor VII
d- Factor VIII *
e- Factor X
89- Side effects of large doses of cortisone include all of the following except :
a- Hirsutism and osteoporosis
b- Mental changes
c- Recurrence or appearance of new peptic ulcers
d- Increased susceptibility to infection
e- Depletion of serum sodium *

Dr. wessam alzaidat - General surgery


90- The average daily loss of water through the lungs and skin (
insensible water loss ) is approximately :
a- 10 ml
b- 100ml
c- 1000ml *
d- 2000ml
e- Any of these depending on circumstances
91- The most common cause of gastric outlet obstruction in adults is;
A- Adenocarcinoma of the stomach.*
B- Hypertrophic pyloric stenosis.
C- Duodenal stenosis secondary to peptic ulceration.
D- Bezoar.
E- Gastric lymphoma.
92- About colorectal carcinoma associated with Crohn’s disease, one is TRUE:

A- It is usually common in women.


B- The frequency of carcinoma is similar in patients with extensive, long
standing ulcerative colitis.*
C- The right colon is involved in over 70% of patients.
D- The mean age of patients with colorectal carcinoma is 35.
E- The prognosis is the same for Ca associated with Crohn's dis and Ca in
colon without Crohn's dis,

93- Regarding pancreatic carcinoma all are true Except:-

A- 90% are ductal adenocarcinomas

B- Less than 20% occur in the head of the gland *

C- The usual presentation is with pain, weight loss and obstructive jaundice

D- Ultrasound has a sensitivity of 80-90% in the detection of the tumour

E- Less than 20% of patients are suitable for curative surgery

94- Heparin all are true Except:-

A-Is a heterogeneous mixture of sulphated polypeptides *

B- Potentiates the actions of antithrombin III

C- Has a half life of 90 minutes.

D-Can be reversed by protamine sulphate

E-Can induce an idiosyncratic thrombocytopenia

Dr. wessam alzaidat - General surgery


95- All of the following affect Gastrin release EXCEPT:
a. antral acidification.
b. antral alkalization.
c. carbohydrates in antrum. *
d. gastric distension.
e. somatostatin release.

96- What portion of the colon absorb the majority of fluid?


a. ascending colon *
b. transverse colon
c. descending colon
d. sigmoid colon
e. rectum

97- The concentration of which electrolyte in pancreatic secretion increases as


the rate of secretion increases?
a. sodium
b. potassium
c. chloride
d. bicarbonate *
e. calcium

98- All of the following promote LES relaxation EXCEPT:


a. atropine *
b. nitric oxide
c. cholecystokinin (CCK)
d. gastric distention
e. pharyngeal stimulation

99- Intraabdominal adhesions following abdominal surgery have been associated


with all of the following EXCEPT:
a. small bowel obstruction
b. infertility
c. chronic pelvic pain
d. intestinal malabsorption *
e. increased risk for enterotomy on subsequent laparatomy

100- Which of the following is true regarding small bowel carcinoid tumors?
a. most are biochemically atypical tumors lacking the enzyme dopa decarboxylase
b. they are most common GI carcinoid tumors
c. regional lymph node involvement is common in tumors less than 1 cm in size *
d. diagnosis is frequently made in patients prior to surgery
e. among carcinoid tumors, they are associated with the lowest rate of second primary

Dr. wessam alzaidat - General surgery


malignancy

KEY 1 C 29 C 66 D
2 D 67 D
30 D
3 A 68 A
31 D
4 B 69 E
32 C
5 D 70 A
33 C
6 B 71 D
34 B
7 E 72 B
35 B
8 C 73 C
36 D
9 C 74 D
37 B
10 D 75 C
38 D
11 D 76 B
39 C
77 C
12 C 40 B
78 D
41 C
13 D 79 D
42 B
80 C
14 B 43 B
81 E
44 B
15 A 82 D
45 E
83 B
16 A 46 B
84 C
47 E
17 D 85 B
48 A
86 D
18 C 49 A
87 B
50 C
19 C 88 D
51 E
20 B 89 E
52 D
90 C
21 A 53 A
91 A
54 B
22 A 92 B
55 C
93 B
23 D 56 C
94 A
57 D
24 D 95 C
58 B
96 A
25 D 59 A
97 D
60 D
26 B 98 A
61 D
99 D
62 A
27 D 100 C
63 C
28 A 64 B
65 C

Dr. wessam alzaidat - General surgery


1-To maintain a normal hydrogen balance, total daily excretion of H+ should equal
the daily: -
a) Fixed acid production plus fixed acid ingestion.
b) HCO3- excretion
c) Pseudomonas aeruginosa.
d) S.Fecales.
e) Staph. Aureus.

2- A 45-year-old obese man undergoes subtotal gastrectomy for an antral carcinoma.


On the fifth postoperative day, non-foul-smelling serosanguineous drainage appears
from the wound. All the following may have been contributing factors EXCEPT
a) malnutrition
b) atelectasis
c) anemia
d) poor technique
e) ascites

3- A 32-year-old man is admitted to a hospital where he undergoes exploratory


surgery for a gunshot wound of the left colon. Subsequently, he is maintained on
intravenous hyperalimentation; his vital signs are stable, and his urine output
averages 250 mL/h. On the fourteenth hospital day, he is reexplored under general
anesthesia for drainage of a left subphrenic abscess. Parenteral nutrition is continued
during surgery. In the recovery room 6 h later, the patient is stable and his urine
output is brisk, but he remains comatose with spontaneous respirations.
The most likely cause for the patient's delayed emergence from anesthesia is
a) hypoglycemia
b) nonketonic, hyperosmolar coma
c) partial curarization
d) volume overload
e) hypoventilation and hypoxemia

4- A patient with a non obstructing carcinoma of the sigmoid colon is being prepared
for elective resection. To minimize the risk of postoperative infectious complications,
your planning should include
a) A single preoperative parenteral dose of antibiotic effective against aerobes and
anaerobes
b) Avoidance of oral antibiotics to prevent emergence of Clostridium difficile
c) Postoperative administration for 2–4 days of parenteral antibiotics effective against
aerobes and anaerobes
d) Postoperative administration for 5–7 days of parenteral antibiotics effective against
aerobes and anaerobes
e) Operative time less than 5 h.

Dr. wessam alzaidat - General surgery


5- Regarding Parathyroid cancer, which one of the following statements is true
a) Is a frequent cause of hyperparathyroidism
b) Most patients present with hypercalcemia and a palpable mass

c) The histopathological criteria to differentiate a parathyroid adenoma and


parathyroid carcinoma are well defined
d) It is associated with multiple endocrine neoplasia type 1
e) Localizing studies are mandatory before neck exploration.

6- The rational for Endoscopic management of cystic duct leak after laparoscopic
cholecystectomy is: -
a) Covering the cystic duct-common bile duct junction with plastic stent to prevent
further leak.
b) Increase in the transpapillary pressure gradient by papillotomy.
c) Promoting flow and reducing the amount of bile flowing through the leak by
placing a plastic stent and also performing a papillotomy.
d) The reduction in the transpapillary pressure gradient by insertion of a temporary
stent.
e) Promoting flow and reducing the amount of bile flowing through the leak by
placing a naso- biliary drain.

7- If tacks/staples are placed to affix the mesh during laparoscopic inguinal hernia
repair, they should NOT be placed:-
a) Into Cooper's ligament.
b) Inferior and lateral to the internal inguinal ring.
c) Superior to the inguinal ligament In to the posterior aspect of the rectus muscle.
d) Superior and medial to the internal inguinal ring.
e) At the pubic bone.

8- What are the early signs and symptoms of gastrojejunostomy leak in obese
patients?
a) Fever.
b) Tachycardia
c) Peritonitis.
d) Oliguria.
e) Hypotension.

9- A 33-year-old man underwent laparoscopic Roux-en-Y gastric byoass 15 months


prior to his presentation to an emergency room for sudden onset of abdominal pain.
He has no other health problems. The pain extends "like a band" across his mid-
abdomen and causes him to bend over. Abdominal exam demonstrates minimal
tenderness in the left upper quadrant of the abdomen and no signs of peritonitis. Lab
tests including complete blood count and comprehensive chemistry panel are normal.
Dr. wessam alzaidat - General surgery
What is the most likely cause for his pain?
a) Anastomotic chronic stricture.
b) Internal herniation.
c) Marginal ulcer.
d) Late dumping syndrome.
e) Gastrointestinal leak.

10- The most common complication of PEG ( Percuatnious Endoscopic Gastrostomy)


placement is?
a) Bleeding
b) Infection
c) Peritonitis
d) Gastro-colic fistula
e) Tube dislodgment

11- About colorectal carcinoma associated with Crohn’s disease, one is TRUE:
a) It is usually common in women.
b) The frequency of carcinoma is similar in patients with extensive, long-standing
ulcerative colitis.
c) The right colon is involved in over 70% of patients.
d) The mean age of patients with colorectal carcinoma is 35.
e) The occurrence of carcinoma is unrelated to the duration of Crohn’s disease.

12- Prolonged absence of enteral nutrition during critical illness causes all of the
following changes in the small intestine EXCEPT:
a) Decrease villous height and cellular mass.
b) Reduction in the production of brush border enzymes.
c) Decrease nutrition absorption.
d) Increased translation in gut bacteria.
e) Irreversible changes in mucosal absorption.

13- Two yrs after resection of a T2N0M0 colon cancer, a chest X-ray reveals a
solitary peripheral pulmonary mass, chest and abdominal CT confirms an isolated
pulmonary lesion without other abnormalities. Which of the following statements is
true?
a) Staging with PET is not indicated
b) Percutaneous biopsy is not indicated because the risk of intrathoracic spread.
c) A normal serum carcinoembryonic antigen (CEA) level excludes metastatic
disease.
d) Resection is indicated.
e) Primary treatment should be Cytotoxic chemotherapy rather than resection.
Dr. wessam alzaidat - General surgery
14- A 35 year old female patient with BMI of 48 who complains of obesity:-
What comorbidities might you expect NOT to improve with weight loss?
a) Type II diabetes.
b) Hypertension.
c) Gastroesophageal reflux.
d) Cholesterol.
e) COPD.
15- A smooth, rubbery 3 cm lesion is removed from the breast of a 35-year-old
woman with a preoperative diagnosis of fibroadenoma. Histologically, this lesion is
found to be a phyllode tumor. Appropriate management at this time is
a) observation only.
b) reexcision of the area with a 1 cm margin.
c) total (simple) mastectomy.
d) total mastectomy with axillary dissection.
e) tamoxifen therapy.

16- Which is true regarding operations for morbid obesity?

a) Bacterial overgrowth in the bypassed segment is a complication of jejunoileal


bypass b) Dumping symptoms are greater following a vertical banded
gastroplasty than after a Roux-en-Y gastric bypass
c) Following Roux-en-Y gastric bypass, the patient may be permitted to resume
normal eating habits
d) Long-term weight loss is not sustained after a jejunoileal bypass operation
e) Ulceration in the bypassed antrum has been a problem after Roux-en-Y gastric
bypass operations

17- Colorectal pseudo-obstruction has been associated with all of the following
EXCEPT
a) excess parasympathetic tone
b) malignant infiltration of the celiac plexus
c) neuroleptic medications
d) opiate usage
e) severe metabolic illness

18- During brief endoscopic abdominal procedures, the largest reserve of body
buffers to absorb CO2 is found in
a) bone
b) kidney
c) liver
d) lung
e) straited muscle
Dr. wessam alzaidat - General surgery
19- Abdominal insufflation to an intraabdominal pressure of 15 mmHg produces all
of the following effects EXCEPT
a) decreased cardiac output
b) decreased glomerular filtration rate
c) increased left heart filling pressure
d) increased peak inspiratory pressure
e) increased right heart filling pressure

20- Regarding surgery for ulcerative colitis all are true EXCEPT:-
a). 30% patients with total colitis will require surgery within 5 years.
b). Panproctocolectomy and pouch formation is appropriate as an elective operation.
c). Pouches can be fashioned as 'S' 'J' or 'W' loops.
d). Over 90% patients with a pouch have perfect continence.
e). With a pouch the mean stool frequency is about 6 times per day.

21- Six weeks after undergoing a laparoscopic Roux-en-Y gastric bypass, a 42-year-
old man develops intolerance of solids and most liquids. Pre-operatively, he weighed
135 kg and he had a BMI of 45 kg/m2. His current weight is 114 kg and his BMI is
38 kg/m2. Consumption of a three-ounce meal leads to dysphagia and vomiting. He is
referred for an upper endoscopy. The endoscopic can’t go through the gastro-
jejunostomy.
Which of the following is the most appropriate therapy?
a) Nasogastric decompression of the gastric pouch and proximal small intestine.
b) Take down of the gastric bypass with small bowel resection and redo-
gastrojejunostomy.
c) Aggressive fluid resuscitation with normal saline.
d) Dilation of the gastrojejunostomy anastomosis followed by observation and
supportive care.
e) Administrations of a proton pump inhibitor and antibiotic therapy for possible
Helicobacter pylori with repeat endoscopy in six weeks

22- After insertion of the veress needle, ways to assume correct position of the needle
include:
a) Aspirate with a syringe to see what returns.
b) Inject 5cc saline and try to immediately get the fluid back.
c) Leave a drop of saline on the hub of the needle and see if it flows freely into the
abdomen.
d) Hook up the insufflator and see if low pressure flow results.
e) All of the above methods can, and usually should, be used.

Dr. wessam alzaidat - General surgery


23-Concerning base skull fracture of the anterior fossa all of the following is true
except:
a) Epistaxis.
b) CSF rhinorrhea.
c) Blindness.
d) Raccon eyes.
e) Battle sign.

24- One of the followings is true regarding Undescended testes:


a) Incidence is 1 in 10 000 boys
b) Most common location is the femoral region
c) Is associated with high rate of infertility
d) Orchidopexy is indicated at five years age
e) Increased risk of malignancy

25- Persistent, conjugated hyprebilirubinaemia may be caused by all of the


followings, EXCEPT:-
a) Alpha-1- antitrypsin deficiency
b) Hypothyroidism
c) Hemolytic disease
d) Cytomegalovirus infection
e) Cystic fibrosis

26- Neoadjuvant combined chemotherapy and radiation therapy in rectal carcinoma:


a) Is not effective in down staging tumors and should not be considered in patients
with fixed lesion.
b) Has been proven to be effective in patients with tumors less than 3 cm in diameter.
c) Should be followed by definitive operation in eight to ten weeks.
d) Is well tolerated by most patients but often result in higher operative morbidity.
e) Decrease local recurrence rate when compared with postoperative radiation
therapy.

27- About gastric lymphoma one is TRUE:


a) Gastrointestinal bleeding is the most common symptom.
b) Endoscopic biopsy establishes the diagnosis in all cases.
c) Primary therapy is surgery.
d) Primary therapy is radiation.
e) The long term survival is similar to the adenocarcinoma.

28- In patients with carotid artery disease:-


a) A bruit is a reliable sign of the degree of stenosis.
b) Atheroma is most commonly seen in the external carotid artery.
c) An embolic event often results in an ipsilateral hemiplegia.
d) Prophylactic aspirin reduces the risk of a stroke.
Dr. wessam alzaidat - General surgery
e) Surgery is of proven benefit in those with asymptomatic stenoses.

29- Which of the following statements regarding malignant hyperthermia is TRUE?


a) It is inherited as an autosomal recessive trait.
b) It is more common in the elderly than in children.
c) Patients with a family history of malignant hyperthermia cannot safely undergo
elective surgery if general anesthetics are to be used.
d) Termination of the procedure and intravenous use of danazol are the recommended
treatment for patients experiencing an acute intraoperative episode of malignant
hyperthermia.
e) The pathogenesis is related to intramuscular calcium transport

30- Oxygen delivery is dependent on all of the following EXEPT:-


a) Cardiac output
b) Hemoglobin
c) Oxygen saturation
d) Arterial oxygen pressure
e) Metabolic acidosis

31. The major indications for performing orchidopexy.


a) To enhance fertility.
b) To prevent the likelihood of development of cancer.
c) To repair a concomitant hernia.
d) To reduce the likelihood of torsion, trauma, and pain.
e) For psychological effect and cosmesis.

32. Clinical features of blunt trauma of children, All of the following are correct
EXCEPT:
a) The most common cause of death and disability following severe trauma in
children, is related to CNS injury.
b) Thoracic injuries in children are second to head injury as a cause of mortality.
c) Massive hemothorax is the most common indication for urgent thoracotomy in the
injured child.
d) CT scan of the abdomen is superior to both U/S and Isotope scan, as it provides
more information and clearer images.
e) Flail chest is primarily a problem of children under 12 years of age.

33. All the following are manifestations of hepatocellular failure EXCEPT:


a) Gynecomastia
b) Polycythemia
c) Hypoalbuminemia
d) Secondary hyperaldosteronism
e) Increased sensitivity to morphine

Dr. wessam alzaidat - General surgery


34.Radiotherapy plays an important role in the treatment of the following tumors
except
a) Rectal carcinoma
b) Wilms tumor
c) Gastric carcinoma
d) Medulloblastoma (intracranial)
e) Hodgkin’s diseas

35. In patient receiving assisted ventilation with positive end expiratory pressure
(PEEP).
The sudden occurrence of hypotension most likely caused by
a) Hypovolemia
b) Acute congestive cardiac failure
c) Haemothorax
d) Massive atelactasis
e) Tension pneumothorax

36. The most reliable indicator of the adequacy of burn resuscitation is.
a) Central venous pressure.
b) Pulmonary capillary wedge pressure.
c) Urine output.
d) Blood pressure and heart rate.
e) Mental status

37.Infection affects wound healing by all the following mechanisms except:


a) Prolonging oedema.
b) Decreasing tissue PO2.
c) Increasing collagenolysis.
d) Decreasing the inflammatory phase.
e) Increasing the inflammatory phase.

38. In females with UTI the best way to collect urine for culture is
a) Mid stream urine
b) Suprapubic aspiration
c) Initial stream
d) By catheterization
e) By cleaning the area and any stream

39. When a renal mass identified by I.V.P., features suggestive of malignancy include
all except
a) Calcifications within the mass
b) Increase tissue density
c) Irregularity of the margin
d) Decrease tissue density
e) Distortion of the collecting system
Dr. wessam alzaidat - General surgery
40. Contraction of bladder as a whole generally requires stimulation by
a) Sympathetic nerve
b) Parasympathetic nerve
c) Somatic nerve
d) It contracts spontaneously
e) Sympathetic and parasympathetic

41. Microscopic BPH describes


a) Enlarged” prostate
b) a proliferative process of the stromal and epithelial elements of the prostate
c) a proliferative process of the stromal elements
d) a proliferative process of the epithelial elements
e) prostate size should be more than 40 gm

42. Chronic scrotal pain is most often due to


a) Testicular torsion
b) Trauma
c) cryptochidism
d) hydrocele
e) orchitis

43. The most common complication after TURP is


a) Failure to void
b) Hemorrhage requiring transfusion
c) Clot retention
d) UTI
e) TUR syndrome

44. What is the most sensitive test for identifying residual freagments after PCNL
a) Nephrestomytubogram
b) MRI
c) Ultrasonography
d) Noncontrast CT
e) CT with contrast

45. The standered method of urinary tract reconstruction during renal transpantationis
a) ureteropyeloplasty
b) ureteroureterostomy
c) ureteroneocystostomy
d) vesicopyeloplasty
e) cutaneous ureterostomy

46. In electrolyte disorders that occur when jejunum is used for urinary intestinal
diversion, one is not happened:
Dr. wessam alzaidat - General surgery
a) hyponatremia
b) hyperchloremia
c) hyperkalemia
d) azotemia
e) acidosis

47. The landmark in retroperitoneoscopy:


a) Psoas muscle
b) Renal artery
c) Renal vein
d) Ureter
e) Transversalis muscle

48. In Renal Trauma, one is true:


a) The degree of hematuria and the severity of the renal injury correlate consistently
b) All blunt trauma patients with gross hematuria should undergo renal imaging.
c) Renal image is not mandatory in penetrating injuries with microscopic hematuria
d) The preferred imaging study for renal trauma is non-contrast-enhanced CT
e) patients with microscopic hematuria and hemodynamically stable should undergo
renal imaging

49. Definitive diagnosis of a ruptured bladder is made by performing a (n):


a) IVP.
b) Ultrasound of the abdomen.
c) CT of the abdomen.
d) CT cystogram.
e) VCU.

50. Adequate urine output for adult postoperative surgical patients is greater than
a) 35 ml|hr regardless of body size
b) 50 ml|hr regardless of body size
c) 0,5 ml|kg|hr
d) 1,0 ml|kg|hr
e) 1,5 ml|kg|hr

51. For pediatric patients with appendicitis, which of the following statements is
NOT true?
a) The rate for misdiagnosis is highest in children under 3 years old.
b) CT scan is less accurate in children than adults.
c) CT scan may be used to rule out peri-appendiceal abscess.
d) Urinary sepsis is the commonest misdiagnosis.
e) WBC count does not effectively differentiate perforated from nonperforated
appendicitis.

52. A 38-year-old man has a painless, nontender mass in his left neck that moves with
Dr. wessam alzaidat - General surgery
swallowing. Fine-needle aspiration shows medullary carcinoma. The right side
appears normal. The best course of action would be
a) Left thyroid lobectomy
b) Subtotal right lobectomy and left lobectomy
c) Total thyriodectomy
d) Total thyroidectomy with central neck dissection
e) Total thyroidectomu with radical neck dissection

53. Bacteremia in patients with biliary tract sepsis is most likely due to
a) Bacteroides Fragilis
b) Enterobacter
c Escherichia coli
d) Enterococcus
e) Coagulase-negative Staphylococcus

54. A 24-year old man undergoing laparotomy for symptomatic Crohn’s disease has a
2-cm stricture of the mid-ileum without any evidence of disease elsewhere in the
gastrointestinal tract. The best surgical option for this lesion would be

a) Resection with primary anastomosis


b) Heineke-Mikulicz strictureplasty
c) Heineke-Mikulicz strictureplasty with biopsy
d) Mechanical dilatation
e) Isoperistaltic side-to-side strictureplasty

55. Incompatible blood transfusion during surgery under general anesthesia, which
of the following is NOT true.
a) Unexplained bleeding.
b) Hematuria may be present.
c) Skin rash may be seen
d) Hypertension is well know problem in spite transfusion
e) Volume support is mandatory.

56. In regard to ventilation in pediatric age group, which of the following is the most
deleterious?
a) CPAP.
b) High volume ( Volutrauma).
c) High pressure ( Barotrauma)
d) PEEP.
e) High ventilatory rate.

57. Regarding urethral injuries, all are true except:


a) Prostatic hematoma may be present.
b) Perineal hematoma may be seen.
c) Fracture pelvis may be the cause.
Dr. wessam alzaidat - General surgery
d) Descending urethrogram is the investigation of choice.
e) May be treated with suprapubic urinary bladder catheterization.

58. Regarding overwhelming post splenectomy sepsis, which of the following is true.
a) Pneumococcal vaccination is not protective.
b) It is common after traumatic splenectomy.
c) Mortality rate is low.
d) It is common after splenectomy for blood diseases.
e) It is common after 10 years post surgery.

59. Regarding abdominal trauma in paediatric age group, which of the following is
true:

a) Clinical examination, stabilization and resuscitation are not of priority.


b) Ultrasound is the diagnostic tool of choice.
c) Blood transfusion should be started immediately.
d) Peritoneal lavage is contra-indicated.
e) Splenectomy is not a routine management for all types of splenic injuries

60. With regard to Hirschsprung’s disease ( aganglionosis), all the following are true
except:
a) It is more common in males.
b) It may be complicated by enterocolitis.
c) Barium enema study may be normal.
d) It is best diagnosed by full thickness rectal biopsy.
e) Surgery can’t be accomplished without colostomy.

61. Which of the followings is not a complication of esophageal atresia and


tracheoesophageal fistula repair?:
a) Esophageal stricture.
b) Anastomotic leakage.
c) Gastroesophageal reflux.
d) Recurrent fistula.
e) Massive variceal bleeding.

62. The most common condition, which needs liver transplantation in pediatric age
group, is:
a) Metabolic liver disease.
b) Malignant liver tumors.
c) Biliary atresia.
d) Choledochal cyst.
e) Viral hepatitis.

63. The least blood loss during burn wound excision occurs when escharectomy is
performed:
Dr. wessam alzaidat - General surgery
a) During the first 48 hours.
b) On days 3-5 post-burn.
c) On days 6-10 post-burn.
d) After day 10 post-burn.
e) When eschar is infected.

64. Contraindications to breast reconstruction after mastectomy include:


a) Diabetes Mellitus.
b) Age over 60 years.
c) Obesity.
d) Bilateral breast cancer.
e) None of the above.

65. All the following muscles are supplied by the trigeminal nerve EXCEPT:
a) Tensor veli palatini.
b) Tensor tympani.
c) Masseter.
d) Posterior belly of digastric.
e) Mylohyoid.

66. The advantages of early excision and graft for full-thickness burns in the first 48
hours post-burn include all of the following EXCEPT:
a) Less blood loss.
b) Less pain.
c) Improved cosmetic result.
d) Decreased hospital stay.
e) Ease of differentiation between partial and full-thickness burns.

67. All of the following statements are correct concerning Corticosteroids EXCEPT:
a) Inhibit fibroblast migration into the wound.
b) Inhibit the activity of prolylhydroxylase.
c) Prevent collagen deposition.
d) Inhibit wound contracture.
e) Vitamin A restores the harmful effects of steroids.

68. The burn that is difficult to estimate in percentage is the:


a) Electrical burn
b) Sun burn
c) Scald burn
d) Direct flame burn
e) Chemical burn

Dr. wessam alzaidat - General surgery


69. All of the following are signs of burn inhalation injury EXCEPT:
a) Erythema of oral mucosa
b) Coughing
c) Black sputum
d) Red colored burn
e) Burn of nasal hair

70. A blow-out fracture of the orbit is by definition:


a) A fracture that results in rupture of the globe.
b) A fracture that results in disruption of the orbital rim.
c) A fracture of the thin walls of the orbit allowing herniation of orbital
contents.
d) A fracture that results from an explosion.
e) A fracture that includes bones adjacent to the orbit.
71. A patient who has a penetrating wound to the extremity has the arteriogram
shows less than 5mm intimal defects and psudoaneurysm. Management of this injury
should be
a) immediate operation
b) Anticoagulant for 6 wks.
c) Observation
d) Elective repair
e) Antiplatelet therapy for 6 wks.

72. Which of the following statements about carotid endarterectomy is not true?
a) Patients with a symptomatic critical stenosis benefit from carotid
endarterectomy
b) The risk of contralateral stroke is reduced after carotid endarterectomy
c) Operative mortality is less than 2%
d) It prolongs survival when compared with patients treated medically
e) It should be accompanied by antiplatelet medication

73. Which of the following is not associated with left-sided portal hypertension
a) Gastric varices
b) Abdominal pain
c) Splenomegally
d) Hepatomegally
e) Chronic pancreatitis

74. A 15 years old boy complains of excessive sweating of his hands. They are cold,
clammy, and damp to the touch. Perspiration dripping from his hands forms a small
puddle during the examination. The remainder of medical history is unremarkable.
Which of the following statements is TRUE
a) the result of sympathectomy for this disorder are generally good
b) the sweat glands involved here are apocrine
c) if not treated, this will likely be permanent
Dr. wessam alzaidat - General surgery
d) topical aluminum salts create ulceration
e) a short course of methotrexate is ofen helpful

75. Which of the following statements about blunt tracheobronchial trauma is true
a) Repair should include a prophylactic tracheostomy
b) Delayed repair is preferred
c) Aneasthesia is safer if the endotracheal tube can be passed beyond the site of
injury
d) Massive hemoptysis is common at presentation
e) Bronchoscopy is hazardous and should be avoided

76. Regarding the functions of a tracheostomy all are true, except:


a) Bypasses an upper airway obstruction.
b) Increases the anatomical dead space.
c) Decreases airway resistance.
d) Protects against aspiration.
e) Allows frequent airway suction.

77. With regard to protein loss after injury, which of the following statements is true?
a) It results from impaired protein synthesis.
b) It occurs primarily from catabolism of skeletal muscle.
c) It occurs primarily from acute renal failure.
d) It occurs primarily from the site of injury.
e) It can be prevented by total parenteral nutrition.

78. Which of the followings is the most important stimulus for triggering the
endocrine response to injury?
a) Tissue acidosis.
b) Local wound factors.
c) Afferent nerve stimuli from the injured area.
d) Hypovolemia.
e) Temperature changes.

79. All of the followings are true regarding perforated appendicitis, except:
a) Higher rate in children.
b) Higher rate in old patients.
c) Usually due to delay in presentation and diagnosis.
d) Appendicular mass is felt in more than 50% of the cases.
e) The commonest complication is wound infection.

80. Factors which contribute to wound dehiscence include all the followings, except:
a) Old age.
b) Coughing.
c) Hypoproteinemea.
d) Anaemia.
Dr. wessam alzaidat - General surgery
e) Malignancy.

81.The lesions in familial adenomatous polyposis with the smallest malignant


potential are
a) Gastric polyps
b) Duodenal polyps
c) Abdominal desmoids
d) Brain tumors
e) Ileal polyps

82. Regarding hernias All true except


a) One portion of the bowel wall is herniated , but not entire
lumen is called Richter hernia
b) Spigelian hernia is Associated with the semilunar line
c) Femoral hernias are more common in females than males
d) Paradoudenal hernia is an internal hernia where the superior
mesenteric vein forms part of its boundaries
e) Obturator hernia Causes anterior thigh pain with walking

83. All of the following are consequences of obstructive jaundice except


a) Wound healing is impaired
b) The function of kupffer cells is increased
c) There is a decrease in the absorption of fat soluble vitamins
d) There is an increased risk of renal failure
e) There is an increased chance of the bile being infected in
malignant obstruction

84. In patients with upper gastrointestinal haemorrhage due to a peptic


ulcer All true except
a) A visible vessel in the ulcer bed at endoscopy is a risk factor
for rebleeding
b) An older patient would be expected to have a higher chance
of rebleeding
c) An adherent clot seen at endoscopy reduces the risk of
rebleeding
d) H2 antagonists do not reduce the bleeding rate
e) A gastric ulcer is more likely to bleed than a duodenal ulcer

85.In the management of trauma patient all true except

Dr. wessam alzaidat - General surgery


a) Mouth to mouth breathing provides a maximum inspired
oxygen concentration of 10%
b) Criciod pressure should be not be used to aid endotrachial
intubation in vomiting patients.
c) Nasogastric intubation is contraindicated in patients who are
suspected to have fracture of the base of skull
d) Needle cricothyroidotomy aids effective ventilation for up to 30
minutes
e) Insertion of (oropharyngial airway) should be attempted in
only on unconscious patients

86. Local anesthetic agents


a) Should not be given intravenously
b) Lignocaine (xylocaine) is four times more potent than
bupivacaine
c) The maximum safe dose for Lignocaine is 3mg/kg and up to
7mg/kg if mixed with adrenaline
d) Can be used safely in the distal limbs even if mixed with
adrenaline
e) Bupivacaine is less cardiotoxic than lignocaine
87. Regarding anal fissures, all are true except:
a) 10% occur in the posterior midline
b) Multiple fissures suggests the diagnosis of TB or crohn's
disease
c) 50% of the fissures heal with the use of bulking agent
d) Sphinterotomy has a success rate of over 90%
e) Sphinterotomy is associated with minor fecal incontinence in
over 15% of patients

88. All of the following are indications for surgery in patients with
spontaneous pnuemothorax except;
a) Scuba divers
b) Individuals living in remote area
c) Airline pilots
d) Age more than 60 years
e) Previous contralateral pnuemothorax

89. Low molecular weight Heparin as its primary inhibitory effect on one of
the following:
a. Factor II.
Dr. wessam alzaidat - General surgery
b. Factor IXa.
c. Antithrombin III.
d. Factor Xa.
e. Factor XIIa.

90. The O2 dissociation curve is shifted to the right by :


a) Decreased CO2 tension
b) Increased CO2 tension
c) Increased PH
d) Increased N2 tension
e) Decreased N2 tension

91. The Glasgow coma scale is the dependant upon all of the following
except
a) response to speech
b) respnse to pain
c) response of the pupils
d) motor response
e) response of the patient

92. In traumatic subarachnoid haemorrhage all of the following are correct


except .
a) is commoner than aneurysmal haemorrage
b) is commoner than subarachnoid haemorrhage associated with AVM.
c) may cause vasospasm and cerebral infarction
d) is often associated with frontal skull fracture
e) is more common in elderly people

93. The adverse prognostic factors for the development of acute subdural
hematoma involve all of the following except
a) old age
b) young age
c) chronic alcoholism
d) skull fracture
e) temporal agenesis

94. Extradural hematoma has the following characters except


a) is more common than acute subdural hematoma
b) is more often associated with vault skull fracture
c) is less often associated with primary brain injury
d) is more like to expand
e) is more likely to be arterial in origin

95. The following are indication for admission to the hospital after a minor
head injury except
Dr. wessam alzaidat - General surgery
a) reduce level of consiousness
b) concussion without skull fracture
c) clinical and radiological evidence of skull frature
d) focal neurological dysfunction
e) difficulty in assessing the patient

96. Which anastamosis location has the highest leakage rate?


a) Jejunojejunostomy after a Roux-en-Y reconstruction
b) Pancreaticojejunostomy
c) Esophagojejunostomy
d) Rectosigmoid anastamosis
e) Gastrojejunostomy

97. Bilibrubin
a) Is conjugated to glucuronic acid in the gallbladder
b) Is transported in hepatic sinusoidal blood bound to albumin
c) When conjugated, is secreted into bile by passive diffusion
d) Is converted to urobilinogen by jejunal enterocytes
e) Is produced predominantly by early phase (< 3 days)
erythrocyte heme breakdown
98.Exclusion criteria for laparoscopic colectomy for diverticulitis include all of
the following EXCEPT
a) Multiple areas of colonic involvement
b) Purulent peritonitis
c) BMI > 30 kg/m2
d) Free peritoneal perforation
e) Multilocular abscess

99.Forty-eight hours after performing a laparoscopic cholecystectomy, a


surgeon realizes that one of the clips was placed across the common bile
duct. The correct approach would be
a) Drainage via percutaneous transhepatic cholangiogram
b) Operative removal of the clips
c) Choledochoduodenostomy
d) Loop choledochojejunostomy
e) Roux-en-Y choledochojejunostomy

100. The ilioinguinal nerve

Dr. wessam alzaidat - General surgery


a) Is a branch of the femoral nerve
b) Has a motor component
c) Can be injured easily at the internal spermatic ring
d) Provides sensation for the penis and upper scrotum
e) Runs along the hypogastric vein

1-A
2-C
3-B
4-C
5-B
6-D
7-B
8-B
9-B
10-B
11-B
12-E
13-D
14-E
15-B
16-A
17-A
18-A
19-A
20-D
21-D
22-E
23-E
24-E
25-C
26-E
27-C
28-D
29-E
30-E
31-B
32-E
33-B
34-C
35-E
36-C
37-D
38-D
39-D
40-B
41-B
42-D
43-A
44-D
Dr. wessam alzaidat - General surgery
45-c
46-b
47-a
48-b
49-d
50-c
51-d
52-d
53-c
54-a
55-d
56-b
57-d
58-d
59-e
60-e
61-e
62-c
63-a
64-e
65-d
66-e
67-c
68-a
69-d
70-c
71-c
72-d
73-d
74-a
75-c
76-b
77-b
78-c
79-d
80-d
81-a
82-d
83-b
84-c
85-a
86-c
87-a
88-d
89-d
90-b
91-c
92-d
93-b
94-a
Dr. wessam alzaidat - General surgery
95-b
96-b
97-b
98-c
99-e
100-d

Dr. wessam alzaidat - General surgery


 Crohn's disease:
A. Is caused by Mycobacterium paratuberculosis.
B. Is more common in Asians than in Jews.
C. Tends to occur in families.
D. Is less frequent in temperate climates than in tropical ones.
E. Is improved by smoking.
(c )

 A 15-year-old boy is admitted with a history and physical findings consistent with
appendicitis. Which finding is most likely to be positive?

 A Pelvic crepitus

 B Iliopsoas sign

 C Murphy sign

 D Flank ecchymosis

 E Periumbilical ecchymosis

 (b )

 A 50-year-old man is admitted with massive bright red rectal bleeding. He recently
had a barium enema that demonstrated no diverticular or space-occupying lesion.
Nasogastric suction reveals no blood but does produce yellow bile. The patient
continues to bleed. What is the next diagnostic step?

 A Repeat barium enema

 B Colonoscopy

 C Upper gastrointestinal series

 D Mesenteric angiography

 E Small bowel follow-through with barium

 (D )

 Recurrence after operation for Crohn's disease:


A. Occurs after operations for ileal Crohn's but not colonic Crohn's.
B. Is usually found just proximal to an enteric anastomosis.
C. Rarely requires reoperation.
D. Occurs in 1% of patients at risk per year during the first 10 years after the
operation.
E. Is prevented by maintenance therapy with corticosteroids.
(B )

Dr. wessam alzaidat - General surgery


 A 15-year-old boy awakens with sudden onset of right lower quadrant and scrotal
tenderness accompanied by nausea and vomiting. Which of the following is the most
appropriate diagnosis and represents a surgical emergency?

 A Acute prostatitis

 B Acute epididymitis

 C Torsion of the testicle

 D Acute appendicitis

 E Gastroenteritis

 (C )

 Factors that decrease collagen synthesis include all of the following except:
 A. Protein depletion.
 B. Infection.
 C. Anemia.
 D. Advanced age.
 E. Hypoxia.
 (C)

 A 47-year-old woman presents with dysphagia to both solids and liquids equally. She
has experienced a 10-kg weight loss over the last several months. A barium swallow
reveals a birdlike narrowing in the distal esophagus. What is the underlying cause of
her symptoms?

 A Disorganized, strong nonperistaltic contractions in the esophagus

 B Failure of the lower esophageal sphincter to relax

 C Hiatal hernia

 D Barrett's esophagus

 E Esophageal stricture secondary to untreated gastroesophageal reflux

 (B)

 A 45-year-old male executive is seen because he is vomiting bright red blood. There
are no previous symptoms. The man admits to one drink a week and has no other
significant history. In the hospital, he bleeds five units of blood before endoscopy.
What is the most likely diagnosis?

 A Gastritis

 B Duodenal ulcer

 C Esophagitis

 D Mallory-Weiss tear

Dr. wessam alzaidat - General surgery


 E Esophageal varices

 (B)

 45-year-old man is seen in the emergency department after vomiting bright red blood.
He has no previous symptoms. He drinks one alcoholic beverage a day.

 What is the most reliable method for locating the lesion responsible for the bleeding?

 A Upper gastrointestinal series

 B Exploratory laparotomy

 C Upper endoscopy

 D Arteriography

 E Radionuclide scanning

 (c)

 After several hours in the hospital, he begins to have recurrent bleeding. He is


transferred to a critical care bed and is persistently hypotensive despite trasnfusion of
nine units of packed red blood cells. Which is the most appropriate next step in
management of this patient?

 A Upper endoscopy with attempt at cauterization of bleeding

 B Transport to the interventional radiology unit to identify and embolize bleeding


source

 C Placement of a Blakemore tube to temporarily tamponade bleeding and to allow for


stabilization of blood pressure

 D Laparotomy to control bleeding

 E Infusion of vasopressin and additional units of blood

 (D)

 A 25-year-old man is admitted with a history of sudden onset of severe midepigastric


abdominal pain. Upright chest radiograph reveals free intraperitoneal air. What is the
therapy for this patient?

 A Upper endoscopy

 B Barium swallow

 C Gastrografin swallow

 D Observation

 E Laparotomy

 (E)

Dr. wessam alzaidat - General surgery


 Concerning severe pancreatitis:

 A. Hypocalcaemia is the most common metabolic problem

 B. Coagulopathy is usually the first organ system failure to manifest itself

 C. Failure of two organ systems is associated with 90% mortality

 D. Solid, infected pancreatic necrosis will often respond to intravenous


antibiotics

 E. Positive end expiratory pressure (PEEP) may be useful in managing


respiratory failure

 (E)

 A 55-year-old female patient is evaluated for new onset of diabetes mellitus. Her
medical history is largely unremarkable. Her physical examination is unrevealing
except for the presence of an erythematous skin rash. Her further evaluation should
include an investigation of the possibility of which of the following?

 A Insulinoma

 B Glucagonoma

 C Gastrinoma

 D Carcinoid tumor

 E Pancreatic cholera

 (B)

 A 60-year-old female patient has a workup for episodic symptoms of palpitations,


nervousness, and bizarre behavior, all of which tend to occur during fasting states.
Biochemically, she is diagnosed as having an insulinoma. What is the best choice for
localizing this tumor?

 A CT scan

 B MRI

 C Selective arteriography

 D Percutaneous catheterization of the portal vein with selective venous sampling

 E Surgical exploration and intraoperative ultrasound

 (E)

 3. A 55-year-old woman with progressive but episodic muscle weakness is diagnosed


as having myasthenia gravis. Her chest radiograph is normal and reveals no evidence

Dr. wessam alzaidat - General surgery


of mediastinal mass or tumor. What is the most definitive treatment that can be
offered this patient?

 A Prednisone

 B Neostigmine

 C Thymectomy

 D Plasmapheresis

 E Atropine

 (c)

 A 50-year-old hypertensive man has definitive biochemical evidence of a


pheochromocytoma. Computed tomography (CT) scan and magnetic resonance
imaging (MRI) do not reveal any abnormalities, and m-iodobenzylguanidine scanning
is not readily available. What should be the next step in the management of this
patient?

 A Abdominal exploration

 B Continued clinical observation

 C Mediastinoscopy

 D Selective venous sampling

 E Mediastinal exploration

 (D)

 The hemodynamic value characteristic of septic shock is


 A Cardiac index, 2.8 L/min/m2
 B Systemic vascular resistance index, 350 dynes cm-5 sec/m2
 C Oxygen consumption, 135 mL/min/m2
 D Oxygen delivery, 700 mL/min/m2
 E Pulmonary capillary wedge pressure, 6.0 mm Hg
 (c)

Dr. wessam alzaidat - General surgery


MCQ for Residents
1.Hyperglycemia in a surgical patient receiving TPN may best be managed by

a. Oral hypoglycemic drugs

b Decreasing the dextrose load and doubling the amount of fat

c. Adding regulalar insulin to the TPN

e. Increasing concentration of protein and carbohydrate calories and decreasing that of

lipids

2.which amino acids can be metabolized outside the liver and are a local source of energy

For muscle?

a. Leucine, isoleucine, valine


b. Alanine, argenine, lysine.
c. Ethionone, glutamine, lysine.
d. Phenyalanine,tyrosine,histidine.
e. None of the above.
3. The gastrointestinal tract can secrete and absorb how much water in the form of gastric
juices per day in 70-kg adult male?
a. 1-2 L/day
b. 4-5 L/day
c. 6-7 L/day
d. 8-10 L/day
e. 50 L/day

4. The protein –sparing effect of glucose administration begin to be manifested after


administration of how much glucose?
a. 1 L of 5% dextrose in water (DƽW)
b. 2 L of (DƽW)
c. 3 L of(DƽW)
d. 4 L of (DƽW)
e. 5 of (DƽW).

5. TNF-α release :
a. can be effectively blocked by anti-TNF-α antibodies to halt systemic
inflammatory syndrome ( SIRS).
b. Doesn't have any beneficial effects in the early phases of the inflammatory
response.
c. Is primarily from leukocytes
d. Promotes polymorphonuclear (PMN) cell adherence and further cytokines
release.
Dr. wessam alzaidat - General surgery
e. Is always deleterious.

6. you suspect that a patient has ARF secondary to hypovolemia.


All of the following are appropriate initial treatment except:
a. check the hemoglobin level
b. Give intravenous boluses.
c. Start vasopressor infusion to keep mean arterial pressure greater than
65 mm Hg
d. Calculate the fractional excretion of sodium
e. Rule out causes of outflow obstruction.

7. Which one of the following suggest an acute adrenal crisis:


a. Random cortisole level of 34 mcg/dl
b. Hypothermia
c. Hyperglycemia
d. Hypokalemia
e. Increase cortisole of 5 mcg/dl after stimulation with cosyntropin

8. The syndrome of multi-organ-failure (MOF):

a. Involve sequential insults that lead to systemic hyperinflammation


b. Require the documentation of active infection
c. Has decreased in incidence over the past decade
d. Require diagnosis within 3 days of the systemic insult
e. Demonstrate consistent improvement after blood transfusion.

9. The systemic parameters in the definition of SIRS include all of the following except:
a. Temperature lower than 36ºC
b. Respiratory rate greater than 20 breath/mint
c. Paco2 less than 32 mm Hg
d. Systolic blood pressure lower than 90 mm Hg
e. Heart rate greater than 90 beats/mint.

10. Strategies that have been suggested to decrease the risk for postoperative
Pulmonary complications include all of the following except:

a. Routine nasogastric tube decompression


b. Lung expansion maneuvers
c. Preoperative smoking cessation.
d. Postoperative epidural anesthesis
e. use of intraoperative short-acting neuromuscular blocking agents.

11. All of the following are true concerning the sympathetic nervous system except:
a. Circulating epinephrine is produced mainly in the adrenal glands and secreted as a
Dr. wessam alzaidat - General surgery
hormone.
b. Most circulating norepinephrine is derived from synaptic nerve clefts.
C. Activation of the sympathetic nervous system result in vasoconstriction,
Tachycardia, and tachypnea.
d. Norepinephrine acts as neurotransmitter.
e. Up to 5% of norepinephrine and 15% of dopamine are produced by the enteric
nervous system.

12. Acute repiratory syndrome (ARDS), histologic examination of an alveolar biopsy


In the first 24 hours would demonstrate.
a. influx of protein-rich leukocyte.
b. Preservation of type II pneumocytes.
c. Bacterial colonization.
d. Alveolar hemorrhage.
e. High level of collagen and fibronectin.

13. which of the following statements regarding the inflammatory phase of the wound
healing.
a. it last up to 24 hours after the injury is incurred.
b. Initial vasodilation is followed by subsequent vasoconstriction.
c. Bradykinin causes vasoconstriction, which inhibits migration of neutrophils
to the healing wound.
d. The complement component C5a and platelet factor attract neutrophil to
the wound.
e. the presence of neutrophil in the wound is essential for normal wound healing.

14. Which of the following statements is true about growth factors.


a. Epidermal growth factor (EGF) stimulates the production of collagen.
b. Vascular endothelial growth factor (VEGF) and PDGF both stimulate angiogesis
by binding to a common receptor.
c. Fibroblast growth factor (FGF) stimulate wound contracture.
d. Transforming growth factor-B (TGF-B) is stored in endothelial cells.
e. Tumor necrosis factorα (TNF-α) inhibits angiogenesis.

15. Which of the following statements regarding scar revision is true?


a. scar maturation refers to the change in size of the wound in the first 1-2 months
b. scar revision should have been performed in the first 3 months after injury
to minimize fibrosis.
c. revision should be performed earlier in children than in adult
d. It corrects undesirable pigmentation
e. Scar revision should be delayed approximately 1 year to allow maturation.

16. Which of the following statements regarding wound healing is true?


Dr. wessam alzaidat - General surgery
a. Vitamin A is needed for hydroxylation of lysine and proline.
b. High doses of vitamin C improve wound healing
c. Vitamin E is involved in the stimulation of fibroplasias, collagen cross-linking.
d. Zinc deficiency results in delayed early wound healing.
e. Iron deficiency had been linked to defects in long-term wound healing.

17 . Which portion of the cell cycle in actively dividing cells is most sensitive to
Ionizing radiation?
a. S phase
b. M phase
c. G1 phase
d. G2 phase
e. All phases are equally radiosensitive.

18. Which of the following statements regarding chromosome is not true?


a. The nucleus contains the entire cellular DNA
b. Histones compact and organize the DNA strands.
c. interactions between DNA and proteins expose specific genes and control
their expression.
d. During mitosis, the spindle apparatus attaches to the chromosome at centromere.
e. Telomeres maintain chromosomal length replication cycles .

19. In DNA replication, what type of mutation is specifically associated with generation
Of a stop codon?.
a. Point mutation
b. Missense mutation.
c. Nonsense mutation
d. Frameshift mutation
e. Neutral mutation.

20. Which of the following condition is associated with an isolated prothrombin time
(PT) prolongation.

a. Von willebrand disease.


b. Factor VIII deficiency ( hemophilia).
c. Common pathway deficiency ( factor II, V, X, and fibrinogen).
d. Therapeutic anticoagulation with warfain
e. Therapeutic anticoagulation with heparin.

21. Regarding complication of blood transfusion, which is true?


a. Febrile reaction is rare.
b. Gram-positive organisms are the most common contaminants of stored blood.
c. Screening for minor antigens should be repeated every week when multiple
Dr. wessam alzaidat - General surgery
transfusions are given.
d. A small amount (more than 0.1 ml air) is well tolerated.
e. Malaria, chagas disease,( HTL-V-1 virus,) ( AIDS) and hepatitis can be transmitted
by blood transfusion.

22. which of the following statements is true regarding the stored blood?
a. Packed RBC stored in additive solution (AS-3) and kept at 4ºC are suitable
for transfusion for 3 months.
b. Platelets in banked blood retain their functions for 3 days.
C. Factors II, VII, IX, and XI are stable at 4ºC.
d. A decrease in RBC oxygen affinity occurs during storage as a result of
a decrease in 2,3-DPG levels.
e. There a significant rate of hemolysis in stored blood.

23. Which of the following statements regarding daily fluid balance is incorrect?
a. Daily water intake is 2000-2500ml.
b. Average stool loss, 1000ml.
c. Average insensible loss, 600ml.
d. Average urine volume, 800ml-1500ml.
e. Average increase in insensible loss in febrile patient, 250 ml/day for each degree
of fever.

24. Which of the following statements concerning Na +concentration of various fluids


Is incorrect.
a. pancreatic secretion, 140 mEq/L
b. Sweat, 40 mEq/L
C. Gasric secretion, 50 mEq/L
d. Saliva, 100mEq/L
e. Ileostomy output, 125 mEq/L.

25. Which of the following condition is not associated with hypernatremia?


a. Diabetes insipidus
b. Tumor lysis syndrome.
c. Steven-Johnson syndrome.
d. Primary hypodepsia.
e. Enterocutaneous fistula.

26. Which of the following clinical signs or symptoms is not associated with serum Na+
Concentration below 125 mEq/L?
a. Headache.
b. Hallucination.
c. Bradycardia
d. Hypoventilation
Dr. wessam alzaidat - General surgery
e. Hyperthermia

27. Which of the following (ECG) findings is not associated with hyperkalemia?
a. Peaked T waves
b. Prolong PR interval
c. Loss of P wave
d. Narrowing of QRS complex.
e. T waves higher than R waves in more than one level.

28. Which of the following is least useful in the immediate treatment of hyperkalemia?
a. Calcium salts.
b. NaHco3
c. Potassium-binding resins.
d. Glucose and insulin
e. Hemodylesis

29. With regard to hypokalemia, which of the following statements is not true?.
a. K+ and H+ are exchanged for Na+ in renal tubule.
b. Respiratory acidosis is associated with increase renal K+ loss.
c. hypokalemia can cause decrease deep tendon reflexes.
d. Flattened T waves and a prolonged QT interval are associated with hypokalemia.
e. Intravenous K+ administration should not exceed 40 to 60 mEq/hour.

30. Which of the following clinical scenario is not associated with acute hypocalcemia?
a. Fluid resuscitation from shock.
b. Rapid infusion of blood products.
c. Improper administration of phosphates.
d. Vitamin D-deficient diets.
e. Acute pancreatic.

31. with regard to acid-base buffering. Which of the following is false?


a. The major extracellular buffer is Hco3
b. Intracellular PH and extracellular PH are usually the same.
c. The major intracellular buffer consists of protein and phosphate salts.
d. H+ cannot directly pass though cell membrane.
e. Treating acidosis with Hco3 infusion can cause cell death.

32. Which of the following statement is not correct with regard to cardiac output(CO)
a. CO alone is not an indicator of myocardial contractility.
b. Ventricular end-Diastolic volume (EDV),vascular resistance, and myocardial contractility
determine stroke volume (SV).
c. Arterial blood pressure alone is an accurate indicator of CO.
Dr. wessam alzaidat - General surgery
d. CO varies directly with pulse rate of up to 160 beats/min in sinus rhyhtm, after
which it decrease.
e. Atrial contraction contributes up to 30% of EDV.

33. With regards to ventilator mechanics, which of the following statements is false?.
a. The work of breathing at rest consume 2% of total oxygen consumption.
b. COPD is associated with increase in the work breathing as a result of increase inspiratory
work.
c. The work of breathing may increase to 50% of total oxygen consumption in
postoperative patients.
d. Airway pressure reflects the compliance of the chest wall and diaphragm
as well as that of lungs.
e. Comliance is measured as the change in volume divided by change in pressure.

34. With regard to Multi organ Failure(MOF). Which of the following statement is false?.
a. Sepsis is the major risk
b. Injury to the microvascular endothelium is uniformly present.
c. Neutrophil-mediated injury is dependent on adherence to the microvascular
endothelium .
d. There is a bimodal pattern to the development of MOF.
e. An increase in the gastrointestinal barrier is often present.

35. Regarding pelvic , fracture which is true?


a. mortality rate is 30% if hypotension present.
b.A pubic symphysis diastasis of 3 cm double the volume of potential pelvic space
for hematoma.
c. It is imperative to make large surgical incision from xiphoid to symphysis to allow
better visualization of the pelvis.
d. The most common cause of death in pelvic fracture is overwhelming sepsis.
e. The external iliac artery is commonly involved in severe pelvic fracture.

36. Select the correct statement regarding flail chest.


a. It occur when three or more adjacent ribs are fractured.
b. work of breathing is increased secondary to paradoxical chest wall infection.
c. Patients with flail chest should be aggressively resuscitated because of probable
development of pulmonary contusion.
d. Patients with this condition should be prophylactically intubated secondary
to a high likelihood of respiratory failure.
e. If a patient doesn't require mechanical ventilation , it is important to avoid
use of positive end-expiratory pressure.

37. A 25 year old male, sustained flame burn, on his right arm circumferentially,
Bilateral on his legs, and on perineum. What the approximate % TBSA?
Dr. wessam alzaidat - General surgery
a. 28%.
b. 36%
c. 46%
d. 64%
e. 52%

38. Which of the following is a potential premalignant precursor of melanoma?


a. Keratoacanthoma
b. Actinic keratosis
c. Seborrheic keratosis
d. dysplastic nevus
e. Bowen disease.

39. The preferred diagnostic biopsy method for pigmented skin lesion is:
a. Punch biopsy
b. Incisional biopsy
c. Shave biopsy
d. Excisional biopsy
e. Excision with 0.5 –cm margin.

40. The most common histologic type of melanoma is :


a. Superficial spreading
b. Nodular
c. Lentigo
d. Acral lentiginous
e. Desmoplastic

41. Which of the following statements is false regarding direct inguinal hernias?
a. The most likely cause is destruction of connective tissue as a result of physical
stress.
b. Direct hernia should be repaired promptly because of the risk of incarceration.
c. A direct hernia may be a sliding involving apportion of the bladder wall.
d. A direct hernia may pass through the external ring.
e. An indirect hernia may present as well.

42. Inguinal hernia:


a. Are best treated with laparoscopic technique.
b. Should always be repaired
c. Benefit from the use of mesh in reducing recurrence.
d. Are associated with a high incidence of strangulation.
e. Are rare in infants.

43.Which of the following is not a characteristic of medullary breast cancer


Dr. wessam alzaidat - General surgery
a. Lymphocystic infiltrate.
b. Benign appearance on ultrasound.
c. High rate of lymph node metastasis
d. statistically better than average prognosis
e. Usually manifest as a palpable mass.

44. Which of the following is not an indication for postmastectomy radiotherapy?


a. T3 Tumors
b. Multicentric DCIS larger than 6 cm
c. inflammatory breast cancer.
d. four or more positive axillary lymph nodes.
e. Gross extranodal extension.

45. with regars to phyllodes tumor of the breast, which statement is not true?
a. It is histologically characterized by epithelial cystlike spaces.
b. Examination reveals a firm, moble, well circumscribed mass.
c. 10% to 15% are malignant
d. The benign version can grow aggressively and recur locally.
e. It commonly metastasizes to lymph nodes.

46. Regarding molecular marker in breast cancer, which of the following


Statements is correct?
a. Carrier of BRCA2 mutations are more likely to have triple –negative cancer.
b. HER2-positive cancers are unlikely to respond to treatment with trastuzumab.
c. ER-positive/HER2-negative patient should be treated by endocrine therapy
d. All breast cancer are sensitive to endocrine therapy.
e. Triple –negative patients have an excellent prognosis.

47. Which of the following is associated with appearance of invasive lobular carcinoma
On mammography?
a. Discrete bilateral mass
b. partially cystic appearance
c. Indistinct mass with poorly defined border.
d. mass with microcalcification.
e. Branching pleomrphic calcification.

48. what is percentage of false negative rate for sentinel lymph node biopsy(SLNB)?
a. 1%
b.8%
c. 15%
d. 20%
e. 25%

Dr. wessam alzaidat - General surgery


49. With regard to thyroid anatomy, which of the following statements is incorrect?
a. The inferior thyroid artery arise directly fro the external carotid artery.
b. the thyroid ima artery arises directly from the aorta in 3% of cases.
c. The ligament of berry is located near the entry point of RLN.
d. venous drainage of the thyroid gland is via superior, middle, and inferior branches.
e. The superior and middle thyroid vein drain into the jugular vein.

50.Which of the following statements regarding Hurthle cell carcinoma is incorrect?


a. It represent a subtype of follicular carcinoma
b. Hurthle cell carcinoma account for 3% of all thyroid cancer.
c. It is more likely to be multifocal
d. It demonstrate poor radioactive iodine.
e. Lymph node dissection is indicated for all patients.

Dr. wessam alzaidat - General surgery


1. With regard to total body water, all of the following are true except:

A. 50 to 70% of total body weight is water.


B. In general the percentage of total body weight that is water is higher in males
than in females.
C. Lean individuals have a greater proportion of water (relative to body weight)
than do obese individuals.
D. The percentage of total body weight that is water increases with age.*
E. Body water is divided into extracellular (i.e. intravascular and interstitial) and
intracellular functional compartments.

2. With regards to the distribution and composition of the body fluid


compartments, which of the following statement is incorrect:

A. The majority of intracellular water is in skeletal muscle.


B. The major intracellular cation is sodium.*
C. The major intracellular anions are the proteins and phosphates.
D. The major extracellular cation is sodium.
E. The major extracellular anions are chloride and bicarbonates.

3. With regard to respiratory acidosis and alkalosis, all of the following


are true except:

A. Respiratory acidosis is associated with an increased denominator of the


Henderson- Hasselbalch ratio that is due to CO2 retention, resulting in a ratio
of less than 20:1.
B. Respiratory alkalosis is associated with a decreased denominator that is due to
a loss of CO2, resulting in a ratio of greater than 20:1.
C. Compensation for respiratory acidosis is primarily renal.
D. Compensation for respiratory alkalosis is primarily pulmonary.*

4. With regard to risk factors for breast cancer, one of the following is
true:

A. Incidence does not appear to be age related among those older than age 35
years.
B. Family history is not a major predictor of risk for developing breast cancer.
C. Late first pregnancy increases the risk.*
D. Diet and weight have no association with breast cancer risk.

Dr. wessam alzaidat - General surgery


5. Which of the following thyroid adenomas may in rare instances
behave in a malignant manner:

A. Colloid adenoma.
B. Embryonal adenoma.
C. Fetal adenoma.
D. Hurthle cell adenoma.*

6. With regard to the technique of needle aspiration biopsy of the


thyroid, one is true:
A. It is generally contraindicated because of the extremely vascular nature of
the thyroid gland.
B. False positive results are rare.*
C. False negative results occur less often than false positive results.
D. There is a 3% risk that cancer cell will implant.
E. Benign versus malignant follicular neoplasm are fairly easily differentiated.

7. With regard to vitamin D physiology, one is true:


A. The major non-dietary source of vitamin D is hepatic synthesis.
B. Hydroxylation of vitamin D3 results in a loss of metabolic activity.
C. Vitamin D decreases intestinal absorption of dietary calcium.
D. Vitamin D has a direct effect on bone resulting in ossification of the bone.*
E. Increased level of PTH inhibits the hydroxylation of 25-
hydroxycholiecalciferol in the kidneys.

8. Which of the following is the most important mechanism for


maintaining competence of the gastroeosophageal junction:

A. Intact vagus nerve.


B. Diaphragmatic crural pinch.
C. Angle of the esophageal entry into the stomach.
D. Orientation of muscle fibers of the esophageal sphincter.
E. Intraabdominal segment of the esophagus.*

9. With regard to Para esophageal hiatal hernia, all of the following are true except:

A. Symptomatic gastroeosophageal reflux is the most common complication.


B. Often there is an associated sliding hernia.
C. Large asymptomatic hernia necessitate repair.
D. Surgical treatment usually involves an antireflux procedure.
E. Pain is common.

10. One of the following is true regarding perforated duodenal ulcer:

Dr. wessam alzaidat - General surgery


A. Can be ruled out if pneumoperitoneum is demonstrated.
B. Can be treated non-operatively.
C. Requires a definitive anti ulcer procedure at the time of the operation.
D. Most commonly occur with posterior ulcers.
E. Is a contraindication to parietal cell vagotomy because of the risk of
mediastinal contamination.

General
1.D 2.B 3.D 4.C 5.D 6.B 7.D 8.E 9.A
10.B
1. All the following structures lie in the transpyloric plane except:

A. The neck of the pancreas.


B. The left kidney.
C. The fundus of the gallbladder.
D. Duodenojejunal junction.
E. The celiac trunk.*

2. All of the following muscles have attachment to the costal cartilage


except:

A. Pectoralis major.
B. Diaphragm.
C. External oblique.*
D. Transversus abdominis.

3. Regarding the sternal angle (angle of Louis) all true except:

A. Marks the demarcation between the superior and inferior mediastinum.


B. Marks the level of the top of the arch of the Aorta.*
C. Marks the level of the second costal cartilage.
D. Marks the level where the azygous vein drains into the superior vena cava.
E. Corresponds to the level of intervertebral disc between 4th and 5th thoracic
vertebrae.

4. All of the following veins drain into the left brachiocephalic vein
except:
A. Vertebral.
B. Thymic.
C. Internal thoracic.
D. Left supreme intercostal vein.
E. Superior thyroid.*

Dr. wessam alzaidat - General surgery


5. During exercise, all true except:

A. Pulmonary ventilation increases up to a maximum of 10- fold.*


B. The depth of ventilation increases before the rate.
C. Oxygen debt can occur up to a maximum of 20 liters.
D. The total peripheral resistance falls.
E. Energy is derived from the local glycogen store rather than from extra
muscular carbohydrate.

6. On ascent to high altitude, all true except:

A. There is increased erythropoieten secretion.


B. Mountain sickness is mainly caused by cerebral edema.
C. The cardiac output is reduced.*
D. The oxygen dissociation curve is shifted to the right.
E. Pulmonary edema can occur in the unacclimatized individual.

7. Regarding thyroxin, all true except:

A. Acts via cyclic adenosine monophosphate.*


B. Can bind to albumin.
C. Has a half-life of about 7 days.
D. Is mainly metabolized by muscle and liver.

8. All of the following are true regarding absorption in the G.I. tract
except:
A. Active absorption of calcium occurs mainly in the duodenum.
B. Glucose and galactose compete for the same carrier system.
C. Protein may be absorbed as di and tri peptides.
D. Bile acids are predominantly absorbed in the terminal ileum.*

9. All of the following are true regarding bile, except:


A. Its electrolyte basis is an alkaline solution resembling that of the pancreatic
juice.
B. The main primary bile acids are colic acid and deoxycolic acid.*
C. It is concentrated in the gall bladder.
D. Bile salts entering the colon when the terminal ileum is resected prevent sodium
and water reabsorption.

Dr. wessam alzaidat - General surgery


10. Regarding calcium, all true except:
A. Total plasma levels are about 2.5mmol/L (10microgram/dl).
B. Is actively absorbed in the intestine.
C. Given intravenously as gluconate.
D. Is useful in the management of hypokalemia.*
E. Levels are raised in blood taken from a standing subject.

1.E 2.C 3.B 4.E 5.A 6.C 7.A 8.D 9.B


10.D

1. Alveolar ventilation(V) & perfusion(Q) are;


A. Evenly distributed through out the lung
B. Segmental & cannot be measured
C. Inversely related through out the lungs
D. Highest in the bases while V/Q is low*
E. Low in the apices & in the bases

2. If arterial pCO2 has markedly increased(pCO2 70 torr) one should


A.Rapidly decrease pCO2 to 40 torr.
B. Rapidly hyperventilate the patient
C.Decrease pCO2 slowly (1 torr.per minute)*
D.Infuse a pressure agent
E.Increase concentration of O2.

3. The main objective of therapy in ARDS is


A.Increase the number of functioning alveoli*
B. Increase PaO2 to more than 70 torr.
C. Increase pulmonary compliance
D. To maintain an open airway
E.To decrease the work of breathing

4.Which of the following is characteristic of Barrett’s ulcer?

A.Benign histology*

B. Location at the leading edge of the squamocolumnar junction

C. Occurrence in 30% 0f patients with Barrett’s esophagus

D. Presentation with cervical dysphagia

E.Responsiveness to H2 blockers

Dr. wessam alzaidat - General surgery


5. All the following are characteristics of the thoracic duct EXCEPT:
A. It takes its origin in the abdomen
B. It enters the chest through the esophageal hiatus*
C. In the neck, it is located behind the carotid sheet & jugular vein
D.It contains many valves to protect it against blood flow
E.Injury to the duct below T5 usually results into a right sided chylothorax
.
6. Regarding the right main bronchus, all the following are true EXCEPT:
A. It is longer & wider than the left*
B. It extends from the carina down to the origin of middle lobe bronchus
C. Its structure is identical to trachea
D. The right upper lobe bronchus leaves the main bronchus outside the hilum
E. It is more vertical than the left

7. The diagnosis of neurogenic thoracic outlet syndrome is usually established by:

A. Cervical spine Xray.


B. History & physical examination*
C. Nerve conduction studies
D. Digital plethysmography
E. Adson’s test

8. Causes of metabolic acidosis include all the following EXCEPT:

A. Diabetes millets
B. Vomiting*
C. Starvation
D. Renal failure
E. Small bowel fistula

9. All the following components or qualities of stored whole blood tend to

decrease over time EXCEPT:

A. Red blood cell viability


B. Potassium concentration*
C. pH
D. Platelet activity
E. O2 carrying capacity

Dr. wessam alzaidat - General surgery


10.Clinical features of Plummer-Vinson (Paterson-Killey) syndrome include all

the following EXEPT:

A. Atrophic oral mucosa


B. Subacute combined degeneration of the cord*
C. Koilonychia
D. Anemia
E.Dysphagia

11. Which of the following is most useful for determining the need for surgery in

a patient with a pharyngeal diverticulum (Zenker’s)?

A. Clinical history & esophagogram*


B. Flexible fiber-optic esophagoscopy
C. Manometry of the upper esophageal sphincter
D. Measured thickness of the cricopharyngeus sphincter on the esophagogram
E.24-hour distal esophageal pH monitoring

12. The procedure responsible for the largest number of esophageal perforations

is:

A. Esophageal bouginage
B. Esophageal sclerotherapy
C. Flexible esophagoscopy*
D. Insertion of naso-gastric tube
E.Pneumatic dilatation

13. Each of the following steps is appropriate in the initial management of patients

with acute corrosive injury to the esophagus, EXCEPT:

A. Administration of antibiotics
B. Administration of emetic agents*
C. Cricothyroidotomy or tracheostomy
D. Fiberoptic laryngoscopy
E. Placement of large bore intravenous cannula

Dr. wessam alzaidat - General surgery


14. The primary pathophysiologic effect of a pericardial effusion that produces

tamponade is:

A. Cardiac irritability
B. Impairment of left ventricular filling
C. Impairment of right ventricular filling*
D. Production of pulmonary edema
E. Shift of the superior & inferior vena cava

15. Each of the following statement regarding the anatomy and function of the

phrenic nerve is correct EXCEPT:

A. An accessory phrenic nerve commonly occurs in the neck but is rare in the thorax.
B. Diaphragmatic pacing requires intact lower motor neurons of the phrenic nerves
and viable cell bodies of C3, C4, and C5.
C. The left phrenic nerve passes anterior to the scalenus anterior muscle.
D. The phrenic nerve is the sole motor nerve supply to the diaphragm.
E. The phrenic nerve supplies sensory fibers only to the superior surface of the
diaphragm.*

16. The commonest tumor of trachea is:

A. Squamous cell carcinoma


B. Adenocarcinoma
C. Adenoid cystic carcinoma (cylendroma)*
D. Carcinoid tumor
E. Oat cell carcinoma

17. The bronchial mucosa is composed of all the following EXEPT:

A. Pseudostratified epithelium
B. Basal cells
C. Goblet cells
D. Adeno-cystic cells*
E. Cilia

Dr. wessam alzaidat - General surgery


18. Spinal cord injury at the level of C2, results IN:

A. Transient apnea
B. Permanent apnea
C. Loss of breathing automaticity(Ordine curse)*
D. Cheyne-Stokes respiration
E. Kussmaul respiration

19.Hoarseness secondary to bronchogenic carcinoma is usually due to extension of

the tumor in to which structure?

A. Vocal cord
B. Superior laryngeal nerve
C. Left recurrent laryngeal nerve*
D. Right vagus nerve
E. Larynx

1 --- D
2 --- C
3 --- A
4 --- A
5 --- B
6 --- A
7 --- B
8 --- B
9 --- B
10 – B
11 –A
12 ---C
13 ---B
14 --- C
15 --- E
16 --- C
17 --- D
18 --- C
19 --- C

Dr. wessam alzaidat - General surgery


1. Generalized edema results from all of the following disorders EXCEPT
A. Systemic hypertension.
B. Congestive heart failure.
C. Cirrhosis.
D. Nephrotic syndrome.
E. Hyperaldosteronism. (A)

2. Disorders that predispose to thrombosis include all of the following


EXCEPT:
A. Pancreatic carcinoma.
B. Pregnancy.
C. Vitamin K deficiency.
D. Sickle cell anemia.
E. Severe burns
(C)

3. The feature most important in differentiating a malignant from benign


tumor IS:
A. Lack of encapsulation.
B. High mitotic rate.
C. Presence of necrosis.
D. Presence of metastases.
E. Nuclear pleomorphism (anaplasia)

(D)

4. In inflammatory response, neutrophils release molecules that have all


of the following effects EXCEPT:
A. Chemotaxis of monocytes.
B. Chemotaxis of lymphocytes.
C. Degranulation of mast cells.
D. Increase vascular permeability independent of histamine
release.
E. Connective tissue digestive.

(B)

Dr. wessam alzaidat - General surgery


5. All of the following tumors are malignant EXCEPT:
A. Glomus tumor.
B. Ewing’s sarcoma.
C. Wilms’ tumor.
D. Seminoma.
E. Histocytosis X.
(A)

6. Malignancies that commonly occur in children younger than 5 years of


age include all of the following EXCEPT:
A. Leukemia.
B. Wilms’ tumor.
C. Retinoblastoma.
D. Rhabdomyosarcoma.
E.Osteosarcoma. (E)

7. All the following statements about vitamin A are true EXCEPT:


A. Deficiency causes blindness.
B. Liver disease causes deficiency.
C. Toxicity causes liver disease.
D. Deficiency causes squamous metaplasia in glandular
epithelium.
E. Toxicity is associated with increase infections.
(B)
8. Which of the following statements about vitamin K deficiency is
TRUE:
A. Deficiency causes defects in both the intrinsic and extrinsic
coagulation pathways.
B. Deficiency produces defects in platelets function.
C. Deficiency is associated with strict vegetarian diet.
D. Interstitial bacteria produce enough vitamin K to prevent
deficiency under normal conditions.
E. Deficiency develops less frequently in breast fed infants
than in bottle fed infants.
(A)

Dr. wessam alzaidat - General surgery


9. All of the following commonly contribute to postoperative atelectasis
after uncomplicated abdominal surgery EXCEPT:
A. Adult respiratory distress syndrome.
B. Diaphragmatic elevation.
C. Voluntary suppression of coughing.
D. Excessive bronchial secretion.
E. Limitation of respiratory movements.
(A)

10. diffuse alveolar damage (adult respiratory distress syndrome) is the


major pattern of pulmonary damage produced by all of the following
EXCEPT:
A. Oxygen toxicity.
B. Narcotic overdose.
C. Septic shock.
D. Cardiopulmonary bypass surgery.
E. Pneumothorax. (E)

11. All of the following statements about disseminated intravascular


coagulation are true EXCEPT:
A. the disorder is characterized by widespread thrombosis.
B. The disorder is characterized by widespread of
hemorrhage
C. It most often present as a primary (idiopathic condition)
D. The brain is the organ most often involved
E. The disorder is associated with mucin-secretion
adenocarcinoma
(C)
----------------------------------------------------------------------------------
----------------------------------------------------------------------------------
----------------------------------------------------------------------------------
----------------------------------------------------------------------------------

Dr. wessam alzaidat - General surgery


1. Which one of the following statements about duodenal atresia
and stenosis in the newborn is not true:

a. prenatal detection of duodenal atresia has remained constant


over the past three decades.
b. a mucosal web with a normal muscular wall is the most
common duodenal abnormality.
c. the double-bubble sign on plain films is the classic x-Ray
finding.
d. downs syndrome is identified in up to 25%of infants.
e. cardiac abnormalities are commonly associated with
duodenal atresia.

(a)

2. Jaundice in jejunal atresia:

a. Occur in 85% of cases.


b. More common in ileal atresia than in jejunal atresia.
c. Mainly due to unconjugated fraction of bilirubin.
d. Occur in less than 5%.
e. More common in males than females.
©

3. Which of the following statements about hypertrophic


pyloric stenosis in the newborn is not true?

a. There is familial predisposition.


B. Hypochloremia and hypokalemia may be present.

Dr. wessam alzaidat - General surgery


c. The pylorus can frequently be palpated on physical
examination.
d. Emesis on feeding is common in the early postoperative
period.
e. It is best diagnosed by an upper gastrointestinal study.

(e)

4. For pediatric patients with appendicitis, which of the


following statements is not true:

a. The rate of misdiagnosis is highest in children under 3


years old.
b. Computed tomographic (CT) scanning is less accurate in
children than adults.
c. CT scan is used to rule out peri-appendiceal abscess.
d. Urinary sepsis is the most common misdiagnosis.
e. WBC count does not effectively differentiate perforated
from nonperforated appendicitis.

(d)

Dr. wessam alzaidat - General surgery


5. Eight hours after an infant exhibits excessive drooling and
mild respiratory distress. An abdominal radiograph shows
complete lack of air in the gastrointestinal tract. Which is the
most likely diagnosis?

a. Hirschsprung”s disease.
b. Tracheoesophageal fistula H type.
c. Pyloric atresia.
d. Choanal atresia.
e. Esophageal atresia without Tracheoesophageal fistula

(E)

Dr. wessam alzaidat - General surgery


Residents 3+4:
1. The most common condition which needs liver transplantation in pediatric age group is:

A. Metabolic liver disease.


B. Malignant liver tumors.
C. Biliary atresia.
D. Choledochal cyst.
E. Viral hepatitis.

Answer is C.

2. Regarding urethral injuries, all are true except:


A. Prostatic hematoma may be present.
B. Perineal hematoma may be seen.
C. Fracture pelvis may be the cause.
D. Descending urethrogram is the investigation of choice.
E. May be treated with suprapubic urinary bladder catheterization.

Answer is D.

3. Regarding overwhelming post splenectomy sepsis, which of the following is true.


A.Pneumococcal vaccination is not protective.
B. It is common after traumatic splenectomy.
C.Mortality rate is low.
D. It is common after splenectomy for blood diseases.
E. It is common after 10 years post surgery.
Answer is D

4- Preoperative bowel preparation helps reduce which of the following postoperative complications
by 40%?
a- Pneumonia
b- Urinary tract infection (UTI)
c-Dehiscence
d- Wound infection
e- Intraabdominal abscess

Answer is :D

Dr. wessam alzaidat - General surgery


7- The single most effective method of diagnosis for a 57-year-old man with a 6-month history of
hoarseness and a neck mass is
a- Fine-needle aspiration of the neck mass
b- Computed tomographic (CT) scan of the head neck and chest
c- Incisional biopsy of the neck mass
d- Excisional biopsy of the neck mass
e- Laryngoscopy with biopsy
Answer is : A

8- A 24-year-old man undergoing laparotomy for symptomatic Crohn's disese has a 2-cm stricture
of the mid-ileum without any evidence of disease elsewhere in the gastrointestinal tract .
The best surgical option for this lesion would be
a- Resection with primary anastomosis
b- Heineke-Mikulicz strictureplasty
c- Heineke –Mikulicz strictureplasty with biopsy
d- Mechanical dilatation
e- Isoperistaltic side – to –side strictureplasty
Answer is : A

9- A 30 year –old-woman with recurrent peptic ulcer disease has a markedly elevated serum gastrin
level. The most likely site for her tumor is
a- Duodenum
b- Liver
c- Stomach
d- Lymph node
e- Pancreas
Answer is : A

10- Which of the following statements about aspiration of pancreatic cyst contents is true
a- It is not beneficial when the patient has a history of pancreatitis
b- Resection is not necessary if cyst fluid amylase level is elevated
c- It should be obtained if there is a central "starburst" calcification in the cyst
d- Cytology is useful in excluding malignancy
e- High cyst carcinoembryonic antigen (CEA)level excludes serous cystadenoma

Answer is : E

Dr. wessam alzaidat - General surgery


11- The most effective treatment for thromboangitis obliterans in the lower extremity is
a- Lumber sympathectomy
b- Cessation of smoking
c- Amputation
d- Inframalleolar bypass
e- Prostacycline analogue
Answer is : B

12. Eight hours after an infant exhibits excessive drooling and mild respiratory distress. An
abdominal radiograph shows complete lack of air in the gastrointestinal tract. Which is the most
likely diagnosis?

a. Hirschsprung”s disease.
b. Tracheoesophageal fistula H type.
c. Pyloric atresia.
d. Choanal atresia.
e. Esophageal atresia without Tracheoesophageal fistula

(E)

13. All of the following form radiolucent stones except:


A.Xanthine
B.Cysteine
C.Allopurinol
D. Orotic acid
E. purine

Ans:B

14. The feature most important in differentiating a malignant from benign tumor IS:
A. Lack of encapsulation.
B. High mitotic rate.
C. Presence of necrosis.
D. Presence of metastases.
E. Nuclear pleomorphism (anaplasia)

(D)

Dr. wessam alzaidat - General surgery


.
15- All the following pharmacologic agents can be used in the treatment of thyrotoxicosis to block
the production of thyroid hormone, EXCEPT?

a.Propylthiouracil
b.Propranolol
c.Methimazole
d.Carbimazole
e. Iodine
(B)
16- Which of the following statements about the diagnosis and treatment of esophageal leiomyomas
is/are correct :
. A.The majority are diagnosed after they cause dysphagia and chest pain.
B. Biopsy is indicated at the time of esophagoscopy, to rule out carcinoma.
C. Full-thickness elliptical excision of the esophageal wall is the preferred surgical approach.
D. Endoscopic ultrasonography is a reliable means of following leiomyomas conservatively.
E. Recurrence of resected leiomyomas is minimized by wide local excision.
(D)
17- Which of the following is contraindications to gastric bypass surgery?
A.Diabetes.
B.Hypertension.
C.Pickwickian syndrome
.D.Failure to agree to long term follow up
E,Sleep apnea.

(D)

18. Which of the following statements about the lower esophageal sphincter (LES) mechanism, or
high-pressure zone (HPZ), is true?
A. The LES is a circular smooth muscle ring that is 3 to 5 cm. long.
B. In assessing esophageal manometric data, mean HPZ pressure less than 6 mm. Hg or overall
length less than 2 cm. is more likely to be associated with incompetence of the LES and
gastroesophageal reflux.
C. Esophageal manometry and the acid perfusion (Bernstein) test reliably identify the patient with
an incompetent LES mechanism.
D. Distal HPZ relaxation occurs within 5 to 8 seconds of initiating a swallow.
E. Twenty-four–hour distal esophageal pH monitoring is achieved with an intraesophageal pH
electrode positioned at the esophagogastric junction.
Answer: B

19. Which of the following statements about achalasia is/are correct?

Dr. wessam alzaidat - General surgery


A. In most cases in North America the cause is a parasitic infestation by Trypanosoma cruzi.
B. Chest pain and regurgitation are the usual symptoms.
C. Distal-third esophageal adenocarcinomas may occur in as many as 20% of patients within 10
years of diagnosis.
D. Manometry demonstrates failure of LES relaxation on swallowing and absent or weak
simultaneous contractions in the esophageal body after swallowing.
E. Endoscopic botulinum toxin injection of the LES, pneumatic dilatation, and esophagomyotomy
provide highly effective curative therapy for achalasia.
Answer: D

20. Which of the following statements about epiphrenic diverticula of the esophagus is/are correct?
A. They are traction diverticula that arise close to the tracheobronchial tree.
B. They characteristically arise proximal to an esophageal reflux stricture.
C. The degree of dysphagia correlates with the size of the pouch.
D. They are best approached surgically through a right thoracotomy.
E. The operation of choice is a stapled diverticulectomy, long esophagomyotomy, and partial
fundoplication.
Answer: E

21. Which of the following statements about the presence of gallstones in diabetes patients is/are
correct?
A. Gallstones occur with the same frequency in diabetes patients as in the healthy population.
B. The presence of gallstones, regardless of the presence of symptoms, is an indication for
cholecystectomy in a diabetes patient.
C. Diabetes patients with gallstones and chronic biliary pain should be managed nonoperatively
with chemical dissolution and/or lithotripsy because of severe complicating medical conditions and
a high operative risk.
D. The presence of diabetes and gallstones places the patient at high risk for pancreatic cancer.
E. Diabetes patients with symptomatic gallstones should have prompt elective cholecystectomy, to
avoid the complications of acute cholecystitis and gallbladder necrosis.
Answer: E

. 22. Which of the following statements regarding unusual hernias is incorrect?


A. An obturator hernia may produce nerve compression diagnosed by a positive Howship-Romberg
sign.
B. Grynfeltt's hernia appears through the superior lumbar triangle, whereas Petit's hernia occurs
through the inferior lumbar triangle.
C. Sciatic hernias usually present with a painful groin mass below the inguinal ligament.
D. Littre's hernia is defined by a Meckel's diverticulum presenting as the sole component of the
hernia sac.
E. Richter's hernia involves the antimesenteric surface of the intestine within the hernia sac and

Dr. wessam alzaidat - General surgery


may present with partial intestinal obstruction.
Answer: C.

23. Staples may safely be placed during laparoscopic hernia repair in each of the following
structures except:
A. Cooper's ligament.
B. Tissues superior to the lateral iliopubic tract.
C. The transversus abdominis aponeurotic arch.
D. Tissues inferior to the lateral iliopubic tract.
E. The iliopubic tract at its insertion onto Cooper's ligament.
Answer: D

24. The following statements about the repair of inguinal hernias are true except:
A. The conjoined tendon is sutured to Cooper's ligament in the Bassini hernia repair.
B. The McVay repair is a suitable option for the repair of femoral hernias.
C. The Shouldice repair involves a multilayer, imbricated repair of the floor of the inguinal canal.
D. The Lichtenstein repair is accomplished by prosthetic mesh repair of the inguinal canal floor in
a tension-free manner.
E. The laparoscopic transabdominal preperitoneal (TAPP) and totally extraperitoneal approach
(TEPA) repairs are based on the preperitoneal repairs of Cheattle, Henry, Nyhus, and Stoppa.
Answer: A

25. The following statement(s) is/are true concerning umbilical hernias in adults.

a. Most umbilical hernias in adults are the result of a congenital defect carried into adulthood
b. A paraumbilical hernia typically occurs in multiparous females
c. The presence of ascites is a contraindication to elective umbilical hernia repair.
d. Incarceration is uncommon with umbilical hernias
e. Repair always requires mesh

Answer: b

26 A number of special circumstances exist in the repair of inguinal hernias. The following
statement(s) is/are correct.

a.Simultaneous repair of bilateral direct inguinal hernias can be performed with no significant
increased risk of recurrence

Dr. wessam alzaidat - General surgery


b. The preperitoneal approach may be appropriate for repair of a multiple recurrent hernia

c. femoral hernia repair can best be accomplished using a Bassini or Shouldice repair

d, Management of an incarcerated inguinal hernia with obstruction is best approached via


laparotomy incision

Answer: b

27. An absolute contraindication to breast-conserving


surgery for breast cancer is:
(a) large tumor
(b) tumor of high grade
(c) early pregnancy
(d) retroareolar tumor
(e) clinical axillary nodes

Answer :( c)

28. Gastrointestinal stromal tumors (GIST):


(a) occur most commonly in the duodenum
(b) are almost always malignant
(c) can be treated adequately with enucleation
(d) are often radioresistant
(e) spread mainly via the lymphatics
Answer D

29.Biliary-enteric fistula most commonly connects:


(a) gallbladder and ileum
(b) gallbladder and duodenum
(c) common bile duct and jejunum
(d) gallbladder and jejunum
(e) common bile duct and ileum
Answer :B.

Dr. wessam alzaidat - General surgery


30. The hepatic caudate lobe:
(a) drains directly into the inferior vena cava
(b) represents segment IV
(c) is supplied by the left portal vein only
(d) is supplied by the right portal vein only
(e) lies to the right of the inferior vena cava
Answer a

31. Small bowel obstruction in an elderly female


without external hernia or previous surgery is most likely caused by:
(a) small bowel neoplasm
(b) volvulus
(c) gallstone ileus
(d) abdominal abscess
(e) obturator hernia
Answer: c

32. The development of thrombocytopenia and arterial


thrombosis with heparin requires:
(a) continuation of heparin and platelet trans-
fusion
(b) continuation of heparin and thrombolysis
(c) doubling the heparin dosage
(d) changing the route of heparin administration
e) discontinuation of heparin
answer e
33.The risk of overwhelming postsplenectomy sepsis is
highest for patients requiring splenectomy for:
(a) thalassemia
(b) trauma
(c) immune thrombocytopenic purpura
(d) hereditary spherocytosis
(e) acquired hemolytic anemia
Answer a

34 After mastectomy, winging of the scapula results


from injury to:
(a) the medial pectoral nerve
(b) the lateral pectoral nerve

Dr. wessam alzaidat - General surgery


(c) the long thoracic nerve
(d) the thoracodorsal nerve
(e) the intercostal-brachial nerve
Answer c
35 A sudden onset of glucose intolerance in patients
receiving total parenteral nutrition often indicates:
(a) diabetes mellitus
(b) sepsis
(c) hypophosphatemia
(d) adrenal insufficiency
(e) zinc insufficiency
Answer b

36 Intra-aortic balloon pump:


(a) increases pulmonary wedge pressure
(b) increases afterload
(c) increases diastolic pressure
(d) increases duration of systole
(e) decreases duration of diastole
Answer c
37: Which of the following distinguishes adrenal insuf-
ficiency from sepsis?
(a) hypotension
(b) fever
(c) tachycardia
(d) altered mental status
(e) hypoglycemia
Answer e
38. The most common etiologic factor for hepatocellu-
lar carcinoma worldwide is:
(a) hepatitis C virus
(b) hepatitis B virus
(c) alcoholic cirrhosis
(d) aflatoxin ingestion
(e) schistosomiasis
Answer b

39. The main complication of topical silver nitrate is:


(a) metabolic acidosis
(b) metabolic alkalosis

Dr. wessam alzaidat - General surgery


(c) hyperkalemia
(d) hypocalcemia
(e) hyponatremia
Answer :e
40. The adverse effects of steroids on wound healing
can be reversed with:
(a) vitamin C
(b) vitamin A
(c) copper
(d) vitamin D
(e) vitamin E
Answer b
41 Which of the following is least appropriate
when evaluating a 14-year-old girl with a breast lump?
(a) ultrasound
(b) clinical follow-up
(c) mammography
(d) fine-needle aspiration
(e) excisional biopsy
Answer c

42. Smoke inhalation is most reliably excluded by:


(a) absence of carbonaceous sputum
(b) normal carboxyhemoglobin level
(c) normal xenon-133 inhalation scan
(d) normal chest x-ray
(e) normal flexible bronchoscopy

Answer e
43. Hypotension develops after pneumoperitoneum
and trocar placement for laparoscopic cholecystec-
tomy. The next action is to:
(a) convert to open cholecystectomy
(b) deflate the abdomen
(c) give intravenous fluids
(d) place the patient in head down position
(e) check for bowel injury
Answer b

44. The most common cause of esophageal perforation

Dr. wessam alzaidat - General surgery


is:
(a) penetrating neck injury
(b) iatrogenic
(c) spontaneous
(d) foreign body
(e) malignancy
answer b

45 . The most common site for accessory splenic


tissue is:
(a) gastrosplenic ligament
(b) gastrocolic ligament
(c) splenic hilum
(d) splenocolic ligament
(e) the pelvis
Answer c

46 Lobular carcinoma in situ:


(a) is mostly found in premenopausal women
(b) usually presents as a breast lump
(c) has characteristic calcification pattern on mam-
mography
(d) is precancerous
(e) associated cancer is lobular in nature

Answer a

47 Optimum approach to inflammatory breast car-


cinoma is:
(a) total mastectomy
(b) modified radical mastectomy
(c) lumpectomy and radiotherapy
(d) chemotherapy
(e) chemotherapy, modified radical mastectomy,
and radiotherapy
answer e

48 Appropriate management of 3-cm squamous cell


carcinoma of the anal canal is:
(a) chemotherapy
(b) abdominoperineal resection

Dr. wessam alzaidat - General surgery


(c) local excision
(d) radiotherapy  local excision
(e) chemotherapy  radiation
Answer e

49. Pleomorphic parotid adenoma:


(a) requires core biopsy before resection
(b) is adequately treated with enucleation
(c) commonly undergoes malignant transformation
(d) commonly results in facial palsy
(e) is the most common parotid neoplasm
Answer e

50 The most common cause of nipple discharge is:


(a) duct ectasia
(b) breast cancer
(c) intraductal papilloma
(d) pituitary adenoma
(e) fibrocystic disease

Answer c
51 The optimal treatment for bleeding gastric varices
in chronic pancreatitis is:
(a) distal pancreatectomy
(b) splenorenal shunt
(c) portocaval shunt
(d) splenectomy
(e) transjugular intrahepatic portosystemic shunt
Procedure
Answer d

Dr. wessam alzaidat - General surgery


Dr. wessam alzaidat - General surgery
2102/5/2 ‫امتحان الجراحة العامه للمقيمين‬

: 1. Most mets. Of esophageal cancer

Liver Lung

:2. Retrosternal goiter , incision type

Low collar

:3. Papillary thyroid cancer ,wrong

Worst prognosis

4. Transplant between different species

Xenograft

5. All causes rise in CVP except

PE

Myocardial contusion

Cardiac taponade

Tension pneumothorax

6. Mostly causes pancreatitis in children

Trauma

7. Mosly causes liver abscess in children

TrUma

Perforated appendicitis

Immunocompromised patients

8. Ischemic colitis

May present with colonic stricture

Obstruction on IMA

9. Liver hemangioma wrong

It's the 2nd most common benign liver tumor

:10. All are causes of AAA except

dm><

Dr. wessam alzaidat - General surgery


Htn

Enzymes

11. Typical presentation of diverteculitis and the pt. Looks toxic, Tt

IV antibiotics and observation

Hartmans procedure

:12. Patient with sever bleeding , with inr 11 , what to give

Prothrombin concebtrates and vit k

Vit k

Ffp+vit k

:13. Cause lung fubrosis

Bleomycin

Adrimycin

Methotrexat

:14. M.c salivary gland cancer

Mucoepidermoid cancer

:15. All of the following cause compartment syndrome except

Massive fluid resuscitation-

penetrating injury-

combined arterial and venous injury -

Poplitial fossa trauma

:16. Q about salivary gland stones , whats wrong

Stones are radiolucent

Most common in submandibular gland

Cause distruction for the gland

: 17. Gas pressure in lap.chole

12

:18. All of the following are true except

Dr. wessam alzaidat - General surgery


Atypical ductal hyperplasia does not increase risk of breast Ca

19. Ngt coiling + gassles stomach :

esophageal atresia no fistula

?20. 70 kg patient , with na 120 , what is sodium deficit

041

: 21. Best approach in cardiac tambonade

Left lateral thoracotomy

Subexphoid

Median sternotomy

Rt lateral thoracotomy

:22. Most common complication of hepatic adenoma

Infection

Rupture and henorrahge

Malignancy transformation

23. PBS :

anti metacondrial

24. all are contraindication for pt to be a kidney doner except:

Gestational DM once

Bp 140/100

25. all are criteria to d/c patient post ambulatory sugery except :

Patient ambulate

Good pain control

solid food

26. Pt presented with jaundice and wt loss .. palpable nontender gallbladder

Courvoisers sign

27. Conservation breast surgery :

restricted to 1 quad only

Dr. wessam alzaidat - General surgery


central

28. 2 years old child with with proximal jeujenal atresia 20 cm , distal ilium atresia 8cm , the best
management :

Double barrel loop ilostomy then definitive surgery

29. Wrong about Liver trauma

Mostly needs laparatomy

30. Most common to have opss after splenectomy

Thalsemia

Itp

Trauma

31. all are true about laproscopic surgery true except :

Increase risk of wound infection

:32. You have near total left colonic obstruction what next step

metastatic work up and manage accordingly -

metastatic work up abd colectomy regardless of mets state -

33. about pancreatic tumers true except :

Less than 20% involve head of pancreas

34. Peutz jeghers all true except

Bleeding is the most common presentation

35. Not an indication of bariatric surgery

Psychiatric illness

:36. What false

Clostridium perfringes are gram negative soore forming bacetria that produce toxins

37. All are true about bariatric surgery except

Sleeve is malabsortive with a restrictive component

R EN Y restrictive with malabsorbtive component

38. synchronous colonic tumors in an obstructing

Dr. wessam alzaidat - General surgery


Palpation of the rest colon intra op

Ct colonograph

Barium enema

Colectomy with colonoscopy after 3 months

39. Most common gastric outlet obstruction :

PUD

Cancer

External compression

40. about maxillofacial face trauma:

Mandible most common fracture

41. during trocars insertion in laproscopic surgery , the most common organ to be injured :

Bowel

Aorta

Common iliac

42. Hx of intestinal obstruction with air in the biliary tree

Gallstone ileus

:43. All are increase surgical infection except

Wound tension

Depth

Shaving

Postive op room pressure

44. Pt did appendectomy, antibiotoc prophylaxis for infective endocarditis s needed in this pt if he
:have

hx of coronary artery surgery -

?defibrilator and endicardial leads -

:45. All are complication of TPN except

Hyperglycemia -

Dr. wessam alzaidat - General surgery


Hypoglycemia -

Hyperkalemia -

Hypercalcemia -

Hypoproteinemia -

:46. All are DDx for intermittent claudication except

symptomatic baker cyst -

diabetic neuropathy -

hip osteoarthritis -

spinal canal stenosis -

:47. Level I, II, III axillary lymph nodes are classified acording to their relation with

Pectoralis minor muscle

:48. True except

Hemangioma second most common benign liver tumor

:49. Hyponatremia pt can be caused by except

adrenocortical overactivity -

small bowel obstruction -

diarrhea -

:50. Pre-op thyroid surgery preperation except

potassium iodide can be given for 10 days -

usually used are anthyroid drugs -

antithyroid meds can take from 2 weeks - 3 months for symptons contral -

B- blockers should be used for tachycardia cobtrol -

:51. All are true regarding Medullaru thyroid CA except

origniate from parafollicular C cella -

produce calcitonin -

can be part of MEN 2 syndrome -

operation of choice is total thyroidectomy -

Dr. wessam alzaidat - General surgery


10% sporadic

:52. Most specific test for lung subcarinal lymph node mets

Bronchoscopy with endobronchial ultrasound -

Ct chest -

Vats mediastinonoscopy

:53. All of the follwing can be presentation of varicose vein except

bleeding from varix -

ulceration -

pain in calfs aggravated by ealking and releived by rest + -

heaviness -

:54. Most common presentation of PVD

Asymptomatic

Rest pain

Intermittent claufication

55. 40 years old man , hx of travil to Africa , 1 week later RUQ pain with fever , what is true :

Anchovy sauce

56. You have a patient that is already anticoagulated, and you want to investigate for
:thrombophilia, all can be done except

Protein C&S -

lupus anticoagulant -

prothromin mutation test -

factor V leiden -

:57. All of the following drugs can lower cardioembolic events in atrial fibrillation patients expect

Dabigatran -

Apixaban -

Rivaroxban -

Warfarin -

Dr. wessam alzaidat - General surgery


Clopidogrel -

58. contraindication for mitral valve replacement except :

Normal echo stress

:59. Pt presnted with lower GI bleeding, after resuscitationnext step

Tagged rbc test

Colonoscopy

Proctoscopy

:60. All are are true except

Sepsis is SIRS in presence of infection -

Septic shock is sepsis with hypotension -

leucocytosis or leuckopenia is sign if sepsis -

Temp of >38 or <36 is sign of sepsis -

tachycardia and tachypnea can be signs of sepsis -

:61. All are indications of surgical intervention Except

Second ipsilateral pneumothorax -

First contralateral pbeumothorax -

Bilateral pneumothorax -

spontaneous Hemothorax -

62. one is an indication for surgery:

Gastric content in chest tube

200 cc serosanguenous over 3 hours

1000 cc serous after insertion of chest tube

63. Wound healing questions

Does not increase wound infx

Microorganism virulence

Does not affect wound healing

Dr. wessam alzaidat - General surgery


Depth of wound

:64. Q about premature

premature have higher risk of infection -

premature requirrment of fluid s is higher -

oxygen index of 20 is indication for ECMO -

lung hypoplasia is the cause of death -

65. q about bilirubin one is false :

Found in urine as unconjugated form

66. all are true about deep partial thikness burn except :

Painfull

Blisters

67. About bone scan one is false

Same dose of ct

…..

68. Q about epidural and peipheral analgesia...fentanyl lipid soluable comaoring to morphine

69. Intrnsic factor which is false

Polysacharid

Acts on terminal ilium

Need vit b12 injection every 2-3 months

70. Post cardiac cath abdominal.pain and bloody diarrhea

Mesentric ischemia

71. Question about branchial cleft cyst excision is dangerous because it is close to

Ext carotid

Int carotid

Hypoglossal nerve

None of the above

All of the above

Dr. wessam alzaidat - General surgery


72. Q about gallstone pancreatitis best mangment

ERCP then lap cholecytectomy

Observation then lap chole in same admission

: 73. Most common cause of burn in child < 5 years old

Scald burn

Flame burn

Chemical

Inhalatiinal

:74. Patient presented with multiple trauma, GCS 7 ... Most important

xray -

ct scan -

enditracheal intubation -

75. CEA: not used for screening for colonic CA

:76. All are indication for sleeve except

Motivated -

Psychatric illness + -

77. desmoids tumer:

:78. Flial chest true except

consist of two consecutive ribs fracture at 2 different sites -

surgery is rarely indicated -

have paradoxical chest movment withvrespiration -

:79. Most commin indication for surgery in goiter

Cosmotic -

pt anxiety about mass -

:80. Fast scan is used to asses

retroperitoneal organs injury -

Dr. wessam alzaidat - General surgery


intraperitoneal otgans injury -

intraabdominal vascular injuy -

:81. Best initial test to evaluate pneumothorax

upright erect CXR -

lateral decubitus -

Expiratory film

:82. All are normal ABG readings except

Base 0 +-2

PH 7.35-7.45

PCO2 35-45

HCO3 32-36 +

83. Question about rectus sheath hematoma

All cases need surgery

….

:84. Question about linitus plastica all are true except

Answer : mucosal ulceration is a marked feature

85.

:PM, 5/3/2017] Dr.salah 3'araybeh ORTHO: Question about linitus plastica all are true except 4221[

Answer : mucosal ulceration is a marked feature

: PM, 5/3/2017] Dr.salah 3'araybeh ORTHO: Question about staging 4220[

Answer clinical assesment of invasion and spread

Dr. wessam alzaidat - General surgery


1.Regarding Peutz Jeghers syndrome all are true, except:
a. Is an autosomal dominant condition.
b. Often presents with anaemia in childhood.
c. Is characterised by circumoral mucocutaneous pigmented lesions.
d. Is associated with adenomatous polyps of the small intestine.
e. Malignant change occurs in 2-3% of polyps.

2. Regarding peptic ulceration all are true, except:


a. H. pylori is a gram-negative bacillus.
b. Duodenal is more common than gastric ulceration.
c. Zollinger-Ellison syndrome is associated with gastrin hyposecretion.
d. H2-blockers will heal 85-95% of duodenal ulcers in 8 weeks.
e. Triple therapy can eradicate H. pylori in 80% of patients in one week.

3.Regarding head Injuries all are true, except:


a. More than 1 million people are seen in UK hospitals each year with head injuries.
b. Skull X-rays can not exclude an intracerebral haematoma.
c. Raised intracranial pressure is associated with an increase in cerebral perfusion.
d. Cushing's response consists of a rise in blood pressure and fall in heart rate.
e. Pupillary dilatation usually occurs on the same side of the intracerebral
haematoma.

4. Regarding stones in the gallbladder, one is correct:


a. Pigment stones are the most common.
b. Pigment stones are due to increased excretion of polymerised conjugated bilirubin.
c. Are not a risk factor for the development of gallbladder carcinoma.
d. 90% of gallstones are radio-opaque.
e. A mucocele of the gallbladder is mostly caused by a stone impacted in Hartmann's
pouch.

5.Regarding stones in the common bile duct, all are true, except:
a. Are found in 10 to 15% of patients undergoing cholecystectomy (Without pre-op
ERCP).
b. Can present with Charcot's Triad.
c. Are suggested by a bile duct diameter >8mm on ultrasound.
d. ERCP, sphincterotomy and balloon clearance is now the treatment of choice.
e. If removed by exploration of the common bile duct the T-tube can be removed
after 3days.

6. Regarding crystalloid solutions, all are true, except:


a. Normal saline contains 154 mmol sodium and 154 mmol chloride.
b. 3 litres of dextrose 1/5 th saline in a day will provide 90 mmol of sodium.
c. 2 grams of potassium chloride is equal to 57 mmol of the salt.
d. Hartmann's solution contains calcium and bicarbonate.
e. The daily maintenance potassium requirement of a 40 Kg woman is about 40
mmol.
1
Dr. wessam alzaidat - General surgery
7. Regarding colloid solutions, all are true, except:
a. Human albumin has a molecular weight of 69 kDa.
b. Albumin has a half life in the circulation of about 15 days.
c. Gelatins (e.g. Haemaccel®) are polysaccharides with a MW of about 35 kDa.
d. Dextrans reduce platelet aggregation and can induce anaphylaxis.
e. 6% Hydroxylethyl Starch (HES) is a synthetic polysaccharide derived from
amylopectin.

8. Regarding central parenteral nutrition, all are true, except:


a. Is a hypo-osmolar solution.
b. Typically contains 14-16g nitrogen as L-amino acids.
c. Typically contains about 250g glucose.
d. Is associated with metabolic disturbances in about 5% of patients.
e. Can induce derangement of liver function tests.

9.Regarding solitary thyroid nodules, all are true, except:


a. Are more prevalent in women.
b. In the adult population, less than 10% are malignant.
c. More than 50% of scintigraphically cold nodules are malignant.
d. The risk of a hot nodule being malignant is negligible.
e. Should not be surgically removed in all patients.

10. Regarding the femoral canal, one is correct:


a. Lies lateral to the femoral vein.
b. Has no relation to the inguinal ligament.
c. Has the lacunar ligament as its lateral border.
d. Has the pectineal ligament as its posterior border.
e. Contains the lymph node of Lund.

11.Regarding the pathology of ulcerative colitis, one is true:


a. Shows full thickness bowel inflammation.
b. The rectum is almost always not involved.
c. 10% of patients have terminal ileal disease.
d. Enterocutaneous or intestinal fistulae are common.
e. The serosa is usually affected.

12. Regarding benign breast disease, all are true, except:


a. Cyclical mastalgia is the commonest reason for referral to the breast clinic.
b. Fibroadenomas are derived from the breast lobule.
c. Lactational breast abscesses are usually due to Staph aureus.
d. Duct ectasia is less common in smokers.
e. Atypical lobular hyperplasia is associated with an increased risk of breast cancer.

2
Dr. wessam alzaidat - General surgery
13.Regarding infantile hypertrophic pyloric stenosis, one is true:
a. Occurs with a male : female ratio of 4:1.
b. Sons of affected mothers have a 80% risk of developing the lesion.
c. Invariably presents between six and eights months of age.
d. Typically presents with bile stained projectile vomiting.
e. Surgical treatment is by Heller's cardiomyotomy.

14. Regarding oesophageal atresia , all are true, except:


a. Is often associated with a distal tracheo-oesophageal fistula.
b. Polyhydramnios is often present late in pregnancy.
c. 50% have other associated congenital abnormalities.
d. Contrast x-ray studies are necessary to confirm the diagnosis.
e. Post-operatively over 15% develop oesophageal strictures.

15.All the following are features of Fallot's Tetralogy, except:


a. Atrial septal defect.
b. Pulmonary stenosis.
c. Right ventricular hypertrophy.
d. A right to left cardiac shunt.
e. Cyanotic attacks during feeding and crying.

16.Regarding phaeochromocytomas, one is correct:


a. Are neuroendocrine tumours of the adrenal cortex.
b. Are benign in 10% of cases.
c. Occur as part of the MEN type I syndrome.
d. Can be imaged with a meta-iodobenzylguanidine scan.
e. Require only beta-blockage drug prior to surgery.

17.Regarding the functions of a tracheostomy all are true, except:


a. Bypasses an upper airway obstruction.
b. Increases the anatomical dead space.
c. Decreases airway resistance.
d. Protects against aspiration.
e. Allows frequent airway suction.

18. Regarding hepatitis B, all are true, except:


a. It is due to a DNA virus.
b. It has an incubation period of 2 to 6 days.
c. It can be transmitted via blood products.
d. About 1% of the UK population are hepatitis B surface antigen positive.
e. About 10% of infected patients become chronic carriers.

3
Dr. wessam alzaidat - General surgery
19.The best surgical modality to treat recurrent primary spontaneous
pneumothorax in a 20 year old man is:
a.bullectomy.
b.pleurectomy.
c.bullectomy and pleurectomy.
d.pleurodesis.
e.decortication.

20-Regarding the anatomy of diaphragm, all are true except:


a.The hiatal opening is situated at the level of the tenth thoracic vertebra.
b.The inferior vena cava passes through the right side of diaphragm at the level of
12th thoracic vertebra.
c.The principle blood supply of the diaphragm is derived directly from the aorta.
d.The left diaphragm is lower than the right diaphragm in a normal adult.
e.Nerve supply to the diaphragm is via phrenic nerve which arises from 3rd,4th&5th
cervical nerve roots.

21.Which one of the following diagnostic methods is the best to diagnose


pneumothorax?
a.Standing CXR, expiratory film.
b.Standing CXR, inspiratory film.
c.Supine CXR, expiratory film.
d.Supine CXR, inspiratory film.
e.Lateral CXR.

22.Regarding chest trauma, all are true, except:


a.Rib fracture in a child is usually a marker of severe injury.
b.Thoracostomy tube should be inserted prior to CXR in any patient with suspected
pneumothorax associated with hypotension.
c.A patient with flail chest, should be considered for ventilatory support if his
respiratory rate is above 35/min.
d.Exploratory thoracotomy should be done if the amount of blood production through
the chest tube is more than 200 cc hourly for 4 hours.
e.Chest tube should be clamped during the transport of the patient.

23. The best site to insert a chest tube is:


a.2nd intercostal space, anterior axillary line.
b.3rd intercostal space,posterior axillary line.
c.4th intercostal space ,posterior axillary line.
d.5th intercostal space, anterior axillary line.
e.7th intercostal space, mid axillary line.

4
Dr. wessam alzaidat - General surgery
24. The best non invasive diagnostic procedure to diagnose diaphragmatic injury
after trauma is:
a.Chest xray.
b.Chest CT scan.
c.Chest MRI.
d.Chest ultrasound.
e.Barium meal.

25.Regarding the anatomy of the tracheobronchial tree and the lung, all are
true, except:
a.The right lung is larger than the left lung.
b.The major (oblique fissure) separates the lower lobe from the upper and middle
lobe.
c.There are 8 segments in the right lung.
d.The right main bronchus is shorter and wider than the left main bronchus.
e.The posterior wall of the trachea is membranous.

26. All the followings are true data about the anatomy and the physiology of
lower oesophageal sphincter, except:
a.In adults, it measures 3 to 5 cm in length.
b.In normal individuals, manometry demonstrates pressure of 5 mm Hg within the
lower oesophageal sphincter.
c.The lower oesophageal sphincter remains contracted at rest.
d.It is not a true sphincter.
e.It normally relaxes with the approach of the primary peristaltic wave.

27.Which of the following is probably a posterior mediastinal mass rather than


an anterior one?
a.Thyroid tumour.
b.Neurogenic tumour.
c.Thymic carcinoma.
d.Teratoma.
e.Lymphoma.

28. Regarding thoracic duct,one is correct:


a.Is the main collecting vessel of the lymphatic system and is far smaller than the
right terminal lymphatic duct.
b.Originates from the cysterna chyle at the level of 4th lumbar vertebra.
c.Originates in the left side.
d.Ascends to enter the chest through the inferior vena cava hiatus.
e.Lies on the anterior surface of the vertebral column behind the esophagus and
between the aorta and the azygous vein.

5
Dr. wessam alzaidat - General surgery
29. The maximum safe dose of local anesthetic administered subcutaneously in a
70 kg man is:
a.10 to 20 mL of 1% lidocaine.
b.40 to 50 mL of 2% lidocaine with epinephrine.
c.40 to 50 mL of 1% lidocaine with epinephrine.
d.40 to 50 mL of 1% bupivacaine.
e.40 to 50 mL of 1% lidocaine without epinephrine.

30.Which of the following vessels of the lower limb is least likely to be occluded
by atherosclerosis?
a.Anterior tibial.
b.Peroneal.
c.Proximal posterior tibial.
d.Distal posterior tibial.
e.Dorsalis pedis.

31. After thoracotomy, a woman has a loss of sensation of the nipple areolar
complex,injury to which intercostal nerve is responsible for this?
a.Second.
b.Third.
c.Fourth.
d.Fifth.
e.Sixth.

32. Oxygen delivery can be increased by increasing all of the followings, except:
a.Hemoglobin.
b.Atmospheric pressure.
c.Cardiac output.
d.Inspired oxygen concentration.
e.Oxygen extraction.

33. Which of the followings is the best abdominal site to assess the bowel sounds
using the stethoscope?
a.Right upper quadrant.
b.Right lower quadrant.
c.Left upper quadrant.
d.Left lower quadrant.
e.Periumbilical.

6
Dr. wessam alzaidat - General surgery
34. Brain stem death is diagnosed in the absence of these brain stem reflexes,
except:
a- Corneal reflexes.
b- Pupillary reflexes.
c- Doll's eye movements.
d- Caloric response.
e- Apnoeic test.

35. The followings are indications for admission to hospital after a minor head
injury, except:
a- reduced level of consciousness.
b- Concussion without skull fracture.
c- Clinical or radiological evidence of skull fracture.
d- Focal neurological dysfunction.
e- Difficulty in assessing the patient.

36. In Extradural haematomas all the followings occur, except:


a- One in ten occur in the frontal region.
b- Less than half exhibit a lucid interval.
c- 20% occur in the temporal area.
d- The first symptom is often progressive paroxysmal headache.
e- Nearly always associated with vault skull fracture.

37. The source of bleeding in the extradural space are the followings, except:
a- Diploic vessels of the skull vault.
b- The cerebral veins.
c- The middle meningeal vessels.
d- The dural sinuses.
e- The dural arterioles.

38.The following regimens are used to treat raised intracranial pressure


(ICP), except:
a- Hypoventilation .
b- Mannitol 20% solution, 1g mannitol/kg body weight.
c- Drainage of CSF.
d- Barbiturates.
e- Surgical decompression.

39. Cerebrospinal fluid (CSF) characterizes by the followings ,except:


a. Circulates at about 0.5 ml/min.
b. Secreted at about 500 ml/24 hours.
c. Secreted in the lateral,third,and fourth ventricles.
d. Secreted predominantly by active secretion.
e. Absorbed by the choroid villi.
7
Dr. wessam alzaidat - General surgery
40. Definitive diagnosis of Pulmonary embolism is made by one of the
followings:
a- ECG (EKG).
b- Chest X-ray.
c- VQ scan.
d- ABG.
e- Pulmonary angiography.

41.In the body metabolism, 10g of protein, would produce approximately:


a. 20 kcal.
b.41 kcal.
c.410 kcal.
d.4100 kcal.
c. 900 kcal.

42. The Glasgow coma scale is dependant upon all of the followings, except:
a.Response to speech.
b.Response to pain.
c.Response of the pupils.
d.Motor response.
e.Response of the patient.

43. Which one of the following spinal segments causes signs of wasting of the
intrinsic muscles of the hand?
a- C5.
b- C7/8.
c- T1.
d- L5.
e- S1.

44. The methods by which bacteria gain access into the brain include all the
followings, except:
a- Direct spread from the surrounding tissues.
b- Haematogenous.
c- Via implantation during trauma.
d- Lymphatics.
e- During surgery.

45. All the followings are premalignant lesions of the skin, except:
a. Actinic keratosis.
b. Bowen’s disease.
c. Seborrheic keratosis.
d. Dysplastic nevi.
e. Nevus sebaceous.

8
Dr. wessam alzaidat - General surgery
46. The commonest cause of death after major burns in modern burn units is:
a) Inadequate resuscitation.
b) Pneumonia.
c) Burn wound sepsis.
d) Renal failure.
e) Bacterial endocarditis.

47. Complications of mandibular fractures include all the followings, except:


a. Malocclusion.
b. Lower lip paraesthesia.
c. Trismus.
d. Ankylosis of the temperomandibular joint.
e. Paralysis of the lower lip.

48. An ulnar nerve injury at the elbow would result in all the followings, except:
a. Clawing of the ring and little fingers.
b. Inability to abduct the little finger.
c. Inability to flex the ring and little fingers.
d. Inabilility to adduct the thumb.
e. Radial deviation of the wrist in flexion.

49. The most important factor which decide the efficacy of laser treatment for a
particular condition is:
a. Age of patient.
b. Depth of lesion.
c. Energy Fluence (Power density x Duration of treatment).
d. Spot size.
e. Wavelength as decided by the lasing medium.

50. A blow-out fracture of the orbit is by definition:


a. A fracture that results in rupture of the globe.
b. A fracture that results in disruption of the orbital rim.
c. A fracture of the thin walls of the orbit allowing herniation of the orbital contents.
d. A fracture that results from an explosion.
e. A fracture that includes bones adjacent to the orbit.

51. Which of the followings is considered the primary blood supply to the skin?
a. Fascial plexus.
b. Subcutaneous plexus.
c. Subdermal plexus.
d. Dermal plexus.
e. Subepidermal plexus.

9
Dr. wessam alzaidat - General surgery
52. Principles of skin wound repair include all of the followings, except:
a. Obliteration of the dead space.
b. Eversion of the skin edges.
c. Tension free closure.
d. Layered tissue closure.
e. Use of nylon simple sutures at the skin edge.

53. The most reliable indicator of the adequacy of burn resuscitation is:
a. Central venous pressure.
b. Pulmonary capillary wedge pressure.
c. Urine output.
d. Blood pressure and heart rate.
e. Mental status.

54. The most common site for squamous cell carcinoma of the lip is the:
a. Upper lip midline.
b. Upper lip laterally.
c. Lower lip midline.
d. Lower lip laterally.
e. Oral commissure.

55. Which of the following cell types is essential for normal wound healing?
a. Leukocytes.
b. Monocytes.
c. Platelets.
d. Erythrocytes.
e. All of the above.

56. The most important factor in the development of ulcers in a spinal cord
injury patient is:
a. Malnutrition.
b. Septic episodes.
c. Anemia.
d. Local pressure.
e. Diminished sensation.

57. Which vitamin D metabolite stimulate intestinal calcium absorption?

a. 1,25–dihydroxyvitamin D.
b. 1,25–dihydroxyvitamin D1.
c. 1,25–dihydroxyvitamin D2.
d. 1,25–dihydroxyvitamin D3.
e. 1,25–dihydroxyvitamin D4.

11
Dr. wessam alzaidat - General surgery
58. Urinary stones generally do not pass spontaneously if they are larger than?

a. 2 mm.
b. 3 mm.
c. 4 mm.
d. 7 mm.
e. 8 mm.

59.Which of the following studies replace intravenous urography in the initial


evaluation of patients with renal colic?

a. Kidney, ureter and bladder study.


b. Renal tomography.
c. Ultrasonography.
d. CTScan.
e. MRI.

60. Initial hormonal evaluation of an infertile man with a sperm count of 5


million sperm/ml should include assays of testosterone level and:

a. Prolactin.
b. Follicle–stimulating hormone (FSH).
c. FSH, luteinizing hormone (LH) and prolactin.
d. FSH and thyroid function.
e. Sex steroid hormone binding globulin (SHBG).

61. Where does benign prostatic hyperplasia (BPH) originate?

a. In the transition zone.


b. In the peripheral zone.
c. In the periurethral glands.
d. In the transition zone and periurethral zone.
e. In the whole organ.

62. It is advisable in a man with BPH and a slightly elevated creatinine level to
perform a(an):

a. Transurethral resection of the prostate (TURP).


b. Intravenous pyelography.
c. Renal sonography.
d. Urodynamic study.
e. kidney, ureter and bladder X-ray.

11
Dr. wessam alzaidat - General surgery
63.A 20 year old man has undergone a retroperitoneal dissection for a testicular
germ cell tumor. The inferior mesenteric artery has been divided during
reflection of the intestines to expose the retroperitoneum. This can be expected
to result in:

a. Ischemia of the descending colon.


b. Ischemia of the sigmoid colon.
c. Ischemia of the rectum.
d. Ischemia of the transverse colon.
e. None of the above.

64. The predominant buffering system in humans is:

a. Bicarbonate.
b. Titratable acids.
c. Ammonium (NH4+).
d. Urea.
e. Phosphate.

65. Terminal hematuria (at the end of the urinary stream) is usually due to:

a. Bladder neck or prostatic inflammation.


b. Bladder cancer.
c. kidney stones.
d. Ureteric calculi.
e. Ureteral stricture disease.

66. Pneumaturia may be due to all of the followings, except:

a. Diverticulitis.
b. Colon cancer.
c. Recent urinary tract instrumentation.
d. Inflammatory bowel disease.
e. Ectopic ureter.

67. As one proceeds outward from the adrenal medulla, the three separate
functional layers of the adrenal cortex are, in correct order:

a. The zona reticularis, zona fasciculata, then zona glomerulosa.


b. The zona fasciculata, zona reticularis, then zona glomerulosa.
c. The zona glomerulosa, zona fasciculata, then zona reticularis.
d. The zona glomerulosa, zona reticularis, then zona fasciculata.
e. The zona reticularis, zona glomerulosa, then zona fasciculata.

12
Dr. wessam alzaidat - General surgery
68. What proportion of the cardiac output is delivered to the kidney?

a. 5%.
b. 20%.
c. 50%.
d. 85%.
e. 100%.

69. During surgical dissection, the ureter can be identified as it enters the pelvis:

a. At the aortic bifurcation.


b. Crossing the superior border of the sacrum.
c. Crossing the common iliac artery at the branching of the internal iliac artery.
d. Crossing the uterine artery.
e. At the internal inguinal ring.

70. All the followings are consequences of major haemorrhage, except:


a- Rise in serum osmolality.
b- Rise in intracellular osmotic pressure.
c- Decrease in plasma oncotic pressure .
d- Peak albumin synthesis in 6 hrs.
e- Rise in blood sugar.

71. Signs of cardiac tamponade may include all of the followings, except:
a- Agitation and confusion.
b- Hypotension.
c- Congested neck veins.
d- Muffled heart sounds.
e- Deviated trachea .

72. About achalasia, one is not true :

a- It is the most common primary esophageal disorder.


b- Manometry reveals non relaxation of the lower esophageal sphincter at rest.
c- Acceptable treatment includes Endoscopic myotomy.
d- It is most often associated with hyperperistalsis of the esophageal body.
e- The intraluminal pressure is elevated.

73. All are causes of metabolic acidosis, except:


a- Untreated diabetes mellitus.
b- Duodenal Ulcer.
c- Uremia.
d- Shock.
e- Liver failure .

13
Dr. wessam alzaidat - General surgery
74. All are recognized complications of acute pancreatitis, except:
a- Adult respiratory distress syndrome .
b- Tetany.
c- Acute renal failure .
d- Disseminated intravascular coagulation .
e- Hypercalcaemia .

75. The following body fluids have high risk of HIV transmission, except:
a- Semen .
b- Blood.
c- Urine.
d- Breast milk.
e- Fresh frozen plasma.

76. Factors which contribute to wound dehiscence include all the followings,
except:
a- Old age.
b- Coughing.
c- Hypoproteinemea.
d- Anaemia.
e- Malignancy.

77. The most common extracranial solid tumour in children is:


a. Wilm's tumour.
b. Neuroblastoma.
c. Hepatoblastoma.
d. Hepatocellular carcinpma.
e. Seminoma.

78. In intussusception, all are true, except:


a. Idiopathic causes are the most common in the infantile age group.
b. Ileo-colic intussusception are seen most commonly.
c. Abdominal colic and vomiting present in most of the cases.
d. Bloody stool is an indication for open surgical reduction.
e. In older children, a pathological leading point must be excluded.

79. Regarding inguinal hernia in children, all are true, except:


a. Is more common in pre-term babies.
b. Is more common in males.
c. Testicular atrophy might be a long term complication.
d. Complications are more common in pre-terms than in term babies.
e. Contralateral groin exploration is routinely done to exclude bilateral hernias.

14
Dr. wessam alzaidat - General surgery
80. One of the followings is true regarding undescended testes:
a. The incidence is 1 in 10 000 boys.
b. The most common location is in the femoral region.
c. Is associated with high rate of infertility.
d. Orchidopexy is indicated at the age of five years.
e. Bilateral cases are more frequent than unilateral cases.

81. The most common indication for splenectomy in children is:


a. Splenic injury in blunt abdominal trauma.
b. Sarcoidosis.
c. Idiopathic thrombocytopenic purpura ( ITP ).
d. Hereditary spherocytosis.
e. Thalassemia.

82. Recurrent UTI in children mostly raise the suspicion of:


a. Urethral diverticulum.
b. Vesico ureteric reflux.
c. Ureteric duplication.
d. Renal ectopia.
e. Neurogenic rectum.

83. The absolute contraindication for circumcision in children is:


a. Hypospadias.
b. Blood dyscrasia.
c. Recurrent UTI.
d. Para-phimosis.
e. Phimosis.

84. A child weighs 15 Kg, his fluid requirement is:


a. 50 ml per hour.
b. 50 ml per kg per hour.
c. 1000 ml per 24 hours.
d. 10 ml per kg per hour.
e. 150 ml per hour.

85. Biliary Atresia is definitely diagnosed by:


a. HIDA scan.
b. Abdominal ultrasound.
c. Liver function test.
d. Abdominal C.T scan with i.v contrast.
e. Intra-operative cholangiogram.

15
Dr. wessam alzaidat - General surgery
86. Two years old infant previously healthy, presented with sudden onset of
respiratory distress, Choking and transient cyanosis. The most likely diagnoses
is:
a. F.B aspiration.
b. Pneumothorax.
c. Pneumonia.
d. Hemothorax.
e. Hemo-pneumothorax.

87. All are causes of bile-stained vomiting in newborns, except:


a. Sepsis.
b. Duodenal atresia.
c. Meconium ileus.
d. Complicated intestinal malrotation.
e. Esophageal atresia.

88. All are normal blood tests in the term newborn at day one of age, except:
a. Blood glucose 45mg-60mg / dl.
b. WBC 20. 000 to 30.000.
c. Creatinine up to 1.3mg%.
d. PCV 55%- 65%.
e. ESR 60mm/first hour.

89. The most common cause of death in old children is:


a. Malignancy.
b. Congenital heart disease.
c. Trauma.
d. Renal Anomalies
e. CNS abnormalities.

90. Causes of abdominal calcification on x-ray include all of the followings,


except:
a. Faecolith.
b. Wilm's tumour.
c. Neuroblastoma.
d. Nephrocalcinosis.
e. Calcified lymph nodes.

91. Which of the followings is the most important stimulus for triggering the
endocrine response to injury?

a. Tissue acidosis.
b. Local wound factors.
c. Afferent nerve stimuli from the injured area.
d. Hypovolemia.
e. Temperature changes.
16
Dr. wessam alzaidat - General surgery
92. Which of the following substances is elevated during the acute response to
injury?

a. Glucagon.
b. Insulin.
c. Thyroid stimulatring hormone (TSH).
d. Prolactin.
e. Calcitonin.

93. With regard to protein loss after injury, which of the following statements is
true?
a. It results from impaired protein synthesis.
b. It occurs primarily from catabolism of skeletal muscle.
c. It occurs primarily from acute renal failure.
d. It occurs primarily from the site of injury.
e. It can be prevented by total parenteral nutrition.

94. Normal wound healing is accelerated by one of the followings?


a. Ascorbic acid.
b. Vitamin A.
c. Zinc.
d. Increased local oxygen tension.
e. Frequent dressing.

95. All of the followings are true regarding perforated appendicitis, except:
a. Higher rate in children.
b. Higher rate in old patients.
c. Usually due to delay in presentation and diagnosis.
d. Appendicular mass is felt in more than 50% of the cases.
e. The commonest complication is wound infection.

96. With regard to blunt abdominal trauma, the commonest intra abdominal
organ which can be affected is?
a. Urinary bladder.
b. Large intestine.
c. Pancreas.
d. Stomach.
e. Spleen.

97. One of the followings is not a cause of hypokalemia?


a. Inadequate oral intake.
b. Potassium-free intravenous fluids.
c. High output nasogastric tube.
d. Blood transfusion.
e. Massive vomiting and diarrhea.

17
Dr. wessam alzaidat - General surgery
98. With regard to wound classification, penetrating abdominal trauma is?
a. Clean wound.
b. Clean/contaminated wound.
c. Contaminated wound.
d. Dirty wound.
e. Not classified.

99. The most common cause of hypercalcemia in hospitalized patients is?


a. Malignancy.
b. Over oral intake.
c. Bone fracture.
d. Blood transfusion.
e. Parathyroid adenoma.

100. One of the following is not a cause of dilutional hyponatremia?


a. Excessive oral water intake.
b. Iatrogenic (intravenous) excess of free water administration.
c. Postoperative increase of antidiuretic hormone secretion.
d. Decreased oral intake of sodium.
e. Some drugs which cause water retention.

18
Dr. wessam alzaidat - General surgery
Key Answers
1. D
2. C
3. C
4. E
5. E
6. C
7. C
8. A
9. C
10. D
11. C
12. D
13. A
14. D
15. A
16. D
17. B
18. B
19. C
20. B
21. A
22. E
23. D
24. C
25. E
26. B
27. B
28. E
29. C
30. C
31. C
32. E
33. B
34. C
35. B
36. C
37. B
38. A
39. E
40. E
19
Dr. wessam alzaidat - General surgery
41. B
42. C
43. C
44. D
45. C
46. B
47. E
48. C
49. E
50. C
51. C
52. E
53. C
54. D
55. B
56. D
57. D
58. C
59. D
60. B
61. D
62. C
63. E
64. A
65. A
66. E
67. A
68. B
69. C
70. B
71. A
72. D
73. B
74. E
75. D
76. D
77. B
78. D
79. E
80. C
81. D
82. B
83. A
84. A
21
Dr. wessam alzaidat - General surgery
85. E
86. A
87. E
88. E
89. C
90. A
91. C
92. A
93. B
94. C
95. D
96. E
97. D
98. D
99. A
100. D

21
Dr. wessam alzaidat - General surgery
1. In the workup on a patient for possible appendicitis, CT scanning should be
performed:
a) Before consulting the surgeon, by the emergency physician
b) In patients with equivocal physical findings
c) Routinely, in all patients with right lower quadrant pain
d) With equal frequency in men and women
e) Never

b—

2. Acute appendicitis

a. Occurs most commonly in the second and third decades of life


b. Can be cured readily by antibiotics
c. Is most commonly caused by a fecalith
d. Carries an overall mortality rate of 7%
e. Induces leukocytosis in 90% of patients

3. The gastric mucosal cell that secretes intrinsic factor is the


a) G cell
b) Parietal cell
c) D cell
d) Enterochromaffin-like cell
d) Chief cell

b—

4. Hypertrophic pyloric stenosis is likeliest to occur in a(n)


a) Firstborn child
b) African-American infant
c) Child 6 to 9 months of age
d) Female infant
e) Infant born prematurely

a—

5. Overwhelming postsplenectomy infection (OPSI) a) Occurs more frequently


after resection for trauma than hematologic disease
b) Occurs with equal frequency in children and adults
c) Is most frequently caused by Streptococcus pneumoniae
d) Usually occurs within 2 years after splenectomy
e) Generally has an identifiable site of infection

c—

Dr. wessam alzaidat - General surgery


6. The characteristic feature of Crohn's colitis that best distinguishes the
clinical entity from ulcerative colitis is
a) Perianal disease
b) Rectal bleeding
c) Risk of malignancy
d) Obstructive symptoms
e) Pseudopolyps

a—

7. Type 1 gastric ulcers are


a) At the incisura
b) Along the greater curvature
c) Prepyloric
d) Associated with simultaneous duodenal ulcers
e) Close to the esophagogastric junction

a—

8. Radiation enteritis
a) Usually presents with perforation
b) Is caused by thrombosis of mucosal vessels
c) Occurs after 3,000 cGy of abdominal radiation
d) Routinely requires operative therapy
e) Is likely in patients who have undergone laparotomy

e–

9. The small bowel tumor with the greatest propensity for bleeding is
a) Carcinoid
b) Lymphoma
c) Adenocarcinoma
d) Hamartoma
e) Leiomyoma

e—

Dr. wessam alzaidat - General surgery


10. A grade III laceration of the spleen is characterized by

a. Intraparenchymal hematoma > 5 cm


b. Capsular tear 1-3 cm deep
c. Hilar vessel disruption
d. Subcapsular hematoma involving 10%-50% of surface area
e. Active hemorrhage

a—

11. The spleen filters all of the following particles/cells EXCEPT

a. Malformed erythrocytes

b. T lymphocytes

c. Malarial parasites

d. Streptococcus pneumoniae

e. Platelets

b—

12. which of the following is the strongest of all other risk factors in the
development of Gastric carcinoma :

Dr. wessam alzaidat - General surgery


A. Helicobacter pylori.
B. Atrophic gastritis.
C. Blood group A.
D. Pernicious anemia.
E. Low socioeconomic class.

13. whilst performing a small bowel resection for strictures following crohn”s
disease, you realize that on inspection, there are marked differences
between jejunal and ileal anatomy. Such differences include all the
following Except:

A. Wider lumen in the jejunum.


B. Less lymphatics in the jejunal mesentry compared to ileal.
C. More prominent and multiple arcades of vessels in the ileum.
D. Thicker wall of the jejunum.
E. Thicker and more fat-laden mesentry increasing towards the ileum.

14. A 54 years old woman is referred to your surgical team with a diagnosis
of small bowel obstruction. Which one of the following clinical signs would
you look for in trying to identify the commonest cause of this condition :

A. surgery scar.
B. Lump in the groin above & medial to the pubic tubercle.
C. Lump in the groin below & lateral to the pubic tubercle.
D. Cachexia & nodule at the umbilicus.
E. Circumoral pigmentation & a family history of previous obstruction.

15. True statements regarding appendiceal neoplasms include which of the


following?

A. Carcinoid tumors of the tip of the appendix less than 1.5 cm are
adequately treated by simple appendectomy
B. Appendiceal carcinoma is associated with secondary tumors of the GI tract in up
to 60% of patients

Dr. wessam alzaidat - General surgery


C. Survival following right colectomy for a Dukes’ stage C appendiceal carcinoma is
markedly better than that for a similarly staged colon cancer at 5 years
D. Mucinous cystadenocarcinoma of the appendix is adequately treated by simple
appendectomy, even in patients with rupture and mucinous ascites
E. Up to 50% of patients with appendiceal carcinoma have metastatic disease, with
the liver as the most common site of spread

:A

16. All of the following statements about carcinoma of the gallbladder are
correct Except :

A. The neoplasm usually starts in the cystic duct and neck of the
gallbladder.
B. It is found more commonly in women than men.
C. It is associated with the presence of gallstones in > 85% of cases.
D. Prognosis is generally poor with < 1 year survival with local invasion.
E. Chemotherapy and radiotherapy do not alter disease progression.

17. All of he following statements are true regarding diverticular disease


Except :

A. It is found more commonly in the developed world.


B. Surgical treatment is usually unnecessary in acute uncomplicated cases.
C. Diverticulae are more commonly found in the descending colon.
D. Perforation and fistula formation can result from an attack of acute diverticulitis.
E. Resolution of the diverticulae can occur with high fiber diet and
adequate hydration.

18. All of the following arteries are branches of the superior mesenteric
artery, except:

a. Ileocolic
b. Replaced left hepatic
c. Inferior pancreaticoduodenal
Dr. wessam alzaidat - General surgery
d. Jejunal
e. Replaced right hepatic

b—

19. Mesenteric cysts

a. Occur primarily in the mesocolon


b. Are discovered mainly in children
c. Can contain chyle or serous fluid
d. Require total enucleation for a cure
e. Are neoplastic

c—

20. Decompression for abdominal compartment syndrome should be


performed

a. If the urine output falls to 30 mL/hour


b. When the patient's respiratory rate increases to 24 breaths per minute
c. Based purely on physical findings
d. When bladder pressure exceeds 35 mm Hg
e. If the patient becomes hypoxemic

d—

21. In laparoscopic surgery, the vessel most likely to be punctured during


trochar placement is the

a. Aorta
b. Right common iliac artery
c. Vena cava
d. Left common iliac artery
e. Right common iliac vein

b—

22. Following resolution of acute appendicitis, an interval appendectomy

a. In 90% of interval appendectomies no pathologic abnormality


could be found.
b. Always reveals luminal occlusion of the appendix
c. Should be performed because of the high incidence of recurrent appendicitis
d. Is not indicated in patients older than 40 years
Dr. wessam alzaidat - General surgery
e. Needs to be performed open because of the fibrosis induced by appendicitis

23. True statements concerning the diagnosis and management of


retroperitoneal fibrosis include all of the following except:

A. Most patients present with dull, non-colicky back, flank, or abdominal pain
B. Evidence of impaired renal function with an elevated blood urea nitrogen is
common
C. The diagnosis is most commonly suggested by intravenous pyelography although
contrast studies with CT scan or MRI are useful in further defining the disease
D. Most patients will need operative intervention.
E. The prognosis for nonmalignant retroperitoneal fibrosis is grim with
progression of disease until death occurring in most patients

:E

24. Lynch Syndrome all are true except?

a) It is due to mutation in MMR gene

b) Associated with APC gene

c) It has Autosomal Dominant Inheritence

d) Adenomas in patients with Lynch syndrome display high grade dysplasia than
adenoma in patients with sporadic colorectal cancer?

e) It occurs predominantly on right side, has increased incidence of


synchronous and metachronous disease.

25. Which of the folowing about Pancreatic Ascites is not true

a) Conservative treatment effective in only 1/4th of patients

b) ERCP should be done before surgery

c) It is exudative

d) Metaplastic cells are present

e) Pancreatic fluid has high amylase and high albumin

Dr. wessam alzaidat - General surgery


A

26. Tetanus all are true except?


a. Is due to an infection with a gram-negative spore forming rod
b. The organism produces a powerful exotoxin
c. The toxin prevents the release of inhibitory neurotransmitter
d. Clostridium tetani is sensitive to penicillin
e. Risus sardonicus is the typical facial spasm

A
27. Which of the following statements about the surgical treatment of
esophageal carcinoma is/are correct?

A. The finding of severe dysphagia in association with Barrett's mucosa is an indication


for an antireflux operation to prevent subsequent development of carcinoma.
B. Long-term survival is improved by radical en bloc resection of the esophagus with
its contained tumor, adjacent mediastinal tissues, and regional lymph nodes.
C. The morbidity and mortality rates for cervical esophagogastric
anastomotic leak are substantially less than those associated with
intrathoracic esophagogastric anastomotic leak.
D. The leading complications of transthoracic esophagectomy and intrathoracic
esophagogastric anastomosis are bleeding and wound infection.
E. Transhiatal esophagectomy without thoracotomy achieves better long-term survival
than transthoracic esophagectomy.

Answer: C

28. The most effective therapy for morbid obesity, in terms of weight control,
is:

A. Intensive dieting with behavior modification.


B. A multidrug protocol with fenfluramine, phenylpropanolamine, and mazindol.
C. A gastric bypass with a 40-ml. pouch, a 10- to 20-cm. Roux-en-Y
gastroenterostomy.
D. A gastric bypass with a 15-ml. pouch, a 40- to 60-cm. Roux-en-Y
gastroenterostomy.
E. Daily exercise with strong emphasis on utilizing all four limbs.

Answer: D

29. All of the following statements about the embryology of Meckel's


diverticulum are true except:
Dr. wessam alzaidat - General surgery
A. Meckel's diverticulum usually arises from the ileum within 90 cm. of the ileocecal
valve.
B. Meckel's diverticulum results from the failure of the vitelline duct to obliterate.
C. The incidence of Meckel's diverticulum in the general population is 5%.
D. Meckel's diverticulum is a true diverticulum possessing all layers of the intestinal
wall.
E. Gastric mucosa is the most common ectopic tissue found within a Meckel's
diverticulum.

Answer: C

Dr. wessam alzaidat - General surgery


1-To maintain a normal hydrogen balance, total daily excretion of H+
should equal the daily: -
a) Fixed acid production plus fixed acid ingestion.
b) HCO3- excretion
c) Pseudomonas aeruginosa.
d) S.Fecales.
e) Staph. Aureus.

2- A 45-year-old obese man undergoes subtotal gastrectomy for an antral


carcinoma. On the fifth postoperative day, non-foul-smelling
serosanguineous drainage appears from the wound. All the following may
have been contributing factors EXCEPT
a) malnutrition
b) atelectasis
c) anemia
d) poor technique
e) ascites

3- A 32-year-old man is admitted to a hospital where he undergoes


exploratory surgery for a gunshot wound of the left colon. Subsequently,
he is maintained on intravenous hyperalimentation; his vital signs are
stable, and his urine output averages 250 mL/h. On the fourteenth hospital
day, he is reexplored under general anesthesia for drainage of a left
subphrenic abscess. Parenteral nutrition is continued during surgery. In
the recovery room 6 h later, the patient is stable and his urine output is
brisk, but he remains comatose with spontaneous respirations.
The most likely cause for the patient's delayed emergence from
anesthesia is
a) hypoglycemia
b) nonketonic, hyperosmolar coma
c) partial curarization
d) volume overload
e) hypoventilation and hypoxemia

4- A patient with a non obstructing carcinoma of the sigmoid colon is


being prepared for elective resection. To minimize the risk of
postoperative infectious complications, your planning should include
a) A single preoperative parenteral dose of antibiotic effective against
aerobes and anaerobes
b) Avoidance of oral antibiotics to prevent emergence of Clostridium

Dr. wessam alzaidat - General surgery


difficile
c) Postoperative administration for 2–4 days of parenteral antibiotics
effective against aerobes and anaerobes
d) Postoperative administration for 5–7 days of parenteral antibiotics
effective against aerobes and anaerobes
e) Operative time less than 5 h.

5- Regarding Parathyroid cancer, which one of the following statements


is true
a) Is a frequent cause of hyperparathyroidism
b) Most patients present with hypercalcemia and a palpable mass

c) The histopathological criteria to differentiate a parathyroid adenoma


and parathyroid carcinoma are well defined
d) It is associated with multiple endocrine neoplasia type 1
e) Localizing studies are mandatory before neck exploration.

6- The rational for Endoscopic management of cystic duct leak after


laparoscopic cholecystectomy is: -
a) Covering the cystic duct-common bile duct junction with plastic stent
to prevent further leak.
b) Increase in the transpapillary pressure gradient by papillotomy.
c) Promoting flow and reducing the amount of bile flowing through the
leak by placing a plastic stent and also performing a papillotomy.
d) The reduction in the transpapillary pressure gradient by insertion of a
temporary stent.
e) Promoting flow and reducing the amount of bile flowing through the
leak by placing a naso- biliary drain.

7- If tacks/staples are placed to affix the mesh during laparoscopic


inguinal hernia repair, they should NOT be placed:-
a) Into Cooper's ligament.
b) Inferior and lateral to the internal inguinal ring.
c) Superior to the inguinal ligament In to the posterior aspect of the
rectus muscle.
d) Superior and medial to the internal inguinal ring.
e) At the pubic bone.

Dr. wessam alzaidat - General surgery


8- What are the early signs and symptoms of gastrojejunostomy leak in
obese patients?
a) Fever.
b) Tachycardia
c) Peritonitis.
d) Oliguria.
e) Hypotension.

9- A 33-year-old man underwent laparoscopic Roux-en-Y gastric byoass


15 months prior to his presentation to an emergency room for sudden
onset of abdominal pain. He has no other health problems. The pain
extends "like a band" across his mid-abdomen and causes him to bend
over. Abdominal exam demonstrates minimal tenderness in the left upper
quadrant of the abdomen and no signs of peritonitis. Lab tests including
complete blood count and comprehensive chemistry panel are normal.
What is the most likely cause for his pain?
a) Anastomotic chronic stricture.
b) Internal herniation.
c) Marginal ulcer.
d) Late dumping syndrome.
e) Gastrointestinal leak.

10- The most common complication of PEG ( Percuatnious Endoscopic


Gastrostomy) placement is?
a) Bleeding
b) Infection
c) Peritonitis
d) Gastro-colic fistula
e) Tube dislodgment

11- About colorectal carcinoma associated with Crohn’s disease, one is


TRUE:
a) It is usually common in women.
b) The frequency of carcinoma is similar in patients with extensive, long-
standing ulcerative colitis.
c) The right colon is involved in over 70% of patients.

Dr. wessam alzaidat - General surgery


d) The mean age of patients with colorectal carcinoma is 35.
e) The occurrence of carcinoma is unrelated to the duration of Crohn’s
disease.

12- Prolonged absence of enteral nutrition during critical illness causes all
of the following changes in the small intestine EXCEPT:
a) Decrease villous height and cellular mass.
b) Reduction in the production of brush border enzymes.
c) Decrease nutrition absorption.
d) Increased translation in gut bacteria.
e) Irreversible changes in mucosal absorption.

13- Two yrs after resection of a T2N0M0 colon cancer, a chest X-ray
reveals a solitary peripheral pulmonary mass, chest and abdominal CT
confirms an isolated pulmonary lesion without other abnormalities.
Which of the following statements is true?
a) Staging with PET is not indicated
b) Percutaneous biopsy is not indicated because the risk of intrathoracic
spread.
c) A normal serum carcinoembryonic antigen (CEA) level excludes
metastatic disease.
d) Resection is indicated.
e) Primary treatment should be Cytotoxic chemotherapy rather than
resection.

14- A 35 year old female patient with BMI of 48 who complains of


obesity:-
What comorbidities might you expect NOT to improve with weight
loss?
a) Type II diabetes.
b) Hypertension.
c) Gastroesophageal reflux.
d) Cholesterol.
e) COPD.
15- A smooth, rubbery 3 cm lesion is removed from the breast of a 35-
year-old woman with a preoperative diagnosis of fibroadenoma.
Histologically, this lesion is found to be a phyllode tumor. Appropriate

Dr. wessam alzaidat - General surgery


management at this time is
a) observation only.
b) reexcision of the area with a 1 cm margin.
c) total (simple) mastectomy.
d) total mastectomy with axillary dissection.
e) tamoxifen therapy.

16- Which is true regarding operations for morbid obesity?

a) Bacterial overgrowth in the bypassed segment is a complication of


jejunoileal bypass b) Dumping symptoms are greater following a
vertical banded gastroplasty than after a Roux-en-Y gastric
bypass
c) Following Roux-en-Y gastric bypass, the patient may be permitted to
resume normal eating habits
d) Long-term weight loss is not sustained after a jejunoileal bypass
operation
e) Ulceration in the bypassed antrum has been a problem after Roux-en-
Y gastric bypass operations

17- Colorectal pseudo-obstruction has been associated with all of the


following EXCEPT
a) excess parasympathetic tone
b) malignant infiltration of the celiac plexus
c) neuroleptic medications
d) opiate usage
e) severe metabolic illness

18- During brief endoscopic abdominal procedures, the largest reserve of


body buffers to absorb CO2 is found in
a) bone
b) kidney
c) liver
d) lung
e) straited muscle

19- Abdominal insufflation to an intraabdominal pressure of 15 mmHg


produces all of the following effects EXCEPT

Dr. wessam alzaidat - General surgery


a) decreased cardiac output
b) decreased glomerular filtration rate
c) increased left heart filling pressure
d) increased peak inspiratory pressure
e) increased right heart filling pressure

20- Regarding surgery for ulcerative colitis all are true EXCEPT:-
a). 30% patients with total colitis will require surgery within 5 years.
b). Panproctocolectomy and pouch formation is appropriate as an elective
operation.
c). Pouches can be fashioned as 'S' 'J' or 'W' loops.
d). Over 90% patients with a pouch have perfect continence.
e). With a pouch the mean stool frequency is about 6 times per day.

21- Six weeks after undergoing a laparoscopic Roux-en-Y gastric bypass,


a 42-year-old man develops intolerance of solids and most liquids. Pre-
operatively, he weighed 135 kg and he had a BMI of 45 kg/m2. His
current weight is 114 kg and his BMI is 38 kg/m2. Consumption of a
three-ounce meal leads to dysphagia and vomiting. He is referred for an
upper endoscopy. The endoscopic can’t go through the gastro-
jejunostomy.
Which of the following is the most appropriate therapy?
a) Nasogastric decompression of the gastric pouch and proximal small
intestine.
b) Take down of the gastric bypass with small bowel resection and redo-
gastrojejunostomy.
c) Aggressive fluid resuscitation with normal saline.
d) Dilation of the gastrojejunostomy anastomosis followed by observation
and supportive care.
e) Administrations of a proton pump inhibitor and antibiotic therapy for
possible Helicobacter pylori with repeat endoscopy in six weeks

22- After insertion of the veress needle, ways to assume correct position
of the needle include:
a) Aspirate with a syringe to see what returns.
b) Inject 5cc saline and try to immediately get the fluid back.

Dr. wessam alzaidat - General surgery


c) Leave a drop of saline on the hub of the needle and see if it flows
freely into the abdomen.
d) Hook up the insufflator and see if low pressure flow results.
e) All of the above methods can, and usually should, be used.

23-Concerning base skull fracture of the anterior fossa all of the


following is true except:
a) Epistaxis.
b) CSF rhinorrhea.
c) Blindness.
d) Raccon eyes.
e) Battle sign.

24- One of the followings is true regarding Undescended testes:


a) Incidence is 1 in 10 000 boys
b) Most common location is the femoral region
c) Is associated with high rate of infertility
d) Orchidopexy is indicated at five years age
e) Increased risk of malignancy

25- Persistent, conjugated hyprebilirubinaemia may be caused by all of


the followings, EXCEPT:-
a) Alpha-1- antitrypsin deficiency
b) Hypothyroidism
c) Hemolytic disease
d) Cytomegalovirus infection
e) Cystic fibrosis

26- Neoadjuvant combined chemotherapy and radiation therapy in rectal


carcinoma:
a) Is not effective in down staging tumors and should not be considered in
patients with fixed lesion.
b) Has been proven to be effective in patients with tumors less than 3 cm
in diameter.
c) Should be followed by definitive operation in eight to ten weeks.
d) Is well tolerated by most patients but often result in higher operative
morbidity.
e) Decrease local recurrence rate when compared with postoperative
radiation therapy.

Dr. wessam alzaidat - General surgery


27- About gastric lymphoma one is TRUE:
a) Gastrointestinal bleeding is the most common symptom.
b) Endoscopic biopsy establishes the diagnosis in all cases.
c) Primary therapy is surgery.
d) Primary therapy is radiation.
e) The long term survival is similar to the adenocarcinoma.

28- In patients with carotid artery disease:-


a) A bruit is a reliable sign of the degree of stenosis.
b) Atheroma is most commonly seen in the external carotid artery.
c) An embolic event often results in an ipsilateral hemiplegia.
d) Prophylactic aspirin reduces the risk of a stroke.
e) Surgery is of proven benefit in those with asymptomatic stenoses.

29- Which of the following statements regarding malignant hyperthermia


is TRUE?
a) It is inherited as an autosomal recessive trait.
b) It is more common in the elderly than in children.
c) Patients with a family history of malignant hyperthermia cannot safely
undergo elective surgery if general anesthetics are to be used.
d) Termination of the procedure and intravenous use of danazol are the
recommended treatment for patients experiencing an acute intraoperative
episode of malignant hyperthermia.
e) The pathogenesis is related to intramuscular calcium transport

30- Oxygen delivery is dependent on all of the following EXEPT:-


a) Cardiac output
b) Hemoglobin
c) Oxygen saturation
d) Arterial oxygen pressure
e) Metabolic acidosis

31. The major indications for performing orchidopexy.


a) To enhance fertility.
b) To prevent the likelihood of development of cancer.
c) To repair a concomitant hernia.
d) To reduce the likelihood of torsion, trauma, and pain.
e) For psychological effect and cosmesis.

Dr. wessam alzaidat - General surgery


32. Clinical features of blunt trauma of children, All of the following are
correct EXCEPT:
a) The most common cause of death and disability following severe
trauma in children, is related to CNS injury.
b) Thoracic injuries in children are second to head injury as a cause of
mortality.
c) Massive hemothorax is the most common indication for urgent
thoracotomy in the injured child.
d) CT scan of the abdomen is superior to both U/S and Isotope scan, as it
provides more information and clearer images.
e) Flail chest is primarily a problem of children under 12 years of age.

33. All the following are manifestations of hepatocellular failure


EXCEPT:
a) Gynecomastia
b) Polycythemia
c) Hypoalbuminemia
d) Secondary hyperaldosteronism
e) Increased sensitivity to morphine

34.Radiotherapy plays an important role in the treatment of the following


tumors except
a) Rectal carcinoma
b) Wilms tumor
c) Gastric carcinoma
d) Medulloblastoma (intracranial)
e) Hodgkin’s diseas

35. In patient receiving assisted ventilation with positive end expiratory


pressure (PEEP).
The sudden occurrence of hypotension most likely caused by
a) Hypovolemia
b) Acute congestive cardiac failure
c) Haemothorax
d) Massive atelactasis
e) Tension pneumothorax

36. The most reliable indicator of the adequacy of burn resuscitation is.
a) Central venous pressure.

Dr. wessam alzaidat - General surgery


b) Pulmonary capillary wedge pressure.
c) Urine output.
d) Blood pressure and heart rate.
e) Mental status

37.Infection affects wound healing by all the following mechanisms


except:
a) Prolonging oedema.
b) Decreasing tissue PO2.
c) Increasing collagenolysis.
d) Decreasing the inflammatory phase.
e) Increasing the inflammatory phase.

38. In females with UTI the best way to collect urine for culture is
a) Mid stream urine
b) Suprapubic aspiration
c) Initial stream
d) By catheterization
e) By cleaning the area and any stream

39. When a renal mass identified by I.V.P., features suggestive of


malignancy include all except
a) Calcifications within the mass
b) Increase tissue density
c) Irregularity of the margin
d) Decrease tissue density
e) Distortion of the collecting system

40. Contraction of bladder as a whole generally requires stimulation by


a) Sympathetic nerve
b) Parasympathetic nerve
c) Somatic nerve
d) It contracts spontaneously
e) Sympathetic and parasympathetic

41. Microscopic BPH describes


a) Enlarged” prostate
b) a proliferative process of the stromal and epithelial elements of the
prostate
c) a proliferative process of the stromal elements

Dr. wessam alzaidat - General surgery


d) a proliferative process of the epithelial elements
e) prostate size should be more than 40 gm

42. Chronic scrotal pain is most often due to


a) Testicular torsion
b) Trauma
c) cryptochidism
d) hydrocele
e) orchitis

43. The most common complication after TURP is


a) Failure to void
b) Hemorrhage requiring transfusion
c) Clot retention
d) UTI
e) TUR syndrome

44. What is the most sensitive test for identifying residual freagments
after PCNL
a) Nephrestomytubogram
b) MRI
c) Ultrasonography
d) Noncontrast CT
e) CT with contrast

45. The standered method of urinary tract reconstruction during renal


transpantationis
a) ureteropyeloplasty
b) ureteroureterostomy
c) ureteroneocystostomy
d) vesicopyeloplasty
e) cutaneous ureterostomy

46. In electrolyte disorders that occur when jejunum is used for urinary
intestinal diversion, one is not happened:
a) hyponatremia
b) hyperchloremia
c) hyperkalemia
d) azotemia
e) acidosis

Dr. wessam alzaidat - General surgery


47. The landmark in retroperitoneoscopy:
a) Psoas muscle
b) Renal artery
c) Renal vein
d) Ureter
e) Transversalis muscle

48. In Renal Trauma, one is true:


a) The degree of hematuria and the severity of the renal injury correlate
consistently
b) All blunt trauma patients with gross hematuria should undergo renal
imaging.
c) Renal image is not mandatory in penetrating injuries with microscopic
hematuria
d) The preferred imaging study for renal trauma is non-contrast-enhanced
CT
e) patients with microscopic hematuria and hemodynamically stable
should undergo renal imaging

49. Definitive diagnosis of a ruptured bladder is made by performing a


(n):
a) IVP.
b) Ultrasound of the abdomen.
c) CT of the abdomen.
d) CT cystogram.
e) VCU.

50. Adequate urine output for adult postoperative surgical patients is


greater than
a) 35 ml|hr regardless of body size
b) 50 ml|hr regardless of body size
c) 0,5 ml|kg|hr
d) 1,0 ml|kg|hr
e) 1,5 ml|kg|hr

51. For pediatric patients with appendicitis, which of the following


statements is NOT true?
a) The rate for misdiagnosis is highest in children under 3 years old.
b) CT scan is less accurate in children than adults.

Dr. wessam alzaidat - General surgery


c) CT scan may be used to rule out peri-appendiceal abscess.
d) Urinary sepsis is the commonest misdiagnosis.
e) WBC count does not effectively differentiate perforated from
nonperforated appendicitis.

52. A 38-year-old man has a painless, nontender mass in his left neck that
moves with swallowing. Fine-needle aspiration shows medullary
carcinoma. The right side appears normal. The best course of action
would be
a) Left thyroid lobectomy
b) Subtotal right lobectomy and left lobectomy
c) Total thyriodectomy
d) Total thyroidectomy with central neck dissection
e) Total thyroidectomu with radical neck dissection

53. Bacteremia in patients with biliary tract sepsis is most likely due to
a) Bacteroides Fragilis
b) Enterobacter
c Escherichia coli
d) Enterococcus
e) Coagulase-negative Staphylococcus

54. A 24-year old man undergoing laparotomy for symptomatic Crohn’s


disease has a 2-cm stricture of the mid-ileum without any evidence of
disease elsewhere in the gastrointestinal tract. The best surgical option for
this lesion would be

a) Resection with primary anastomosis


b) Heineke-Mikulicz strictureplasty
c) Heineke-Mikulicz strictureplasty with biopsy
d) Mechanical dilatation
e) Isoperistaltic side-to-side strictureplasty

55. Incompatible blood transfusion during surgery under general


anesthesia, which of the following is NOT true.
a) Unexplained bleeding.
b) Hematuria may be present.
c) Skin rash may be seen
d) Hypertension is well know problem in spite transfusion
e) Volume support is mandatory.

Dr. wessam alzaidat - General surgery


56. In regard to ventilation in pediatric age group, which of the following
is the most deleterious?
a) CPAP.
b) High volume ( Volutrauma).
c) High pressure ( Barotrauma)
d) PEEP.
e) High ventilatory rate.

57. Regarding urethral injuries, all are true except:


a) Prostatic hematoma may be present.
b) Perineal hematoma may be seen.
c) Fracture pelvis may be the cause.
d) Descending urethrogram is the investigation of choice.
e) May be treated with suprapubic urinary bladder catheterization.

58. Regarding overwhelming post splenectomy sepsis, which of the


following is true.
a) Pneumococcal vaccination is not protective.
b) It is common after traumatic splenectomy.
c) Mortality rate is low.
d) It is common after splenectomy for blood diseases.
e) It is common after 10 years post surgery.

59. Regarding abdominal trauma in paediatric age group, which of the


following is true:

a) Clinical examination, stabilization and resuscitation are not of priority.


b) Ultrasound is the diagnostic tool of choice.
c) Blood transfusion should be started immediately.
d) Peritoneal lavage is contra-indicated.
e) Splenectomy is not a routine management for all types of splenic
injuries

60. With regard to Hirschsprung’s disease ( aganglionosis), all the


following are true except:
a) It is more common in males.
b) It may be complicated by enterocolitis.
c) Barium enema study may be normal.
d) It is best diagnosed by full thickness rectal biopsy.

Dr. wessam alzaidat - General surgery


e) Surgery can’t be accomplished without colostomy.

61. Which of the followings is not a complication of esophageal atresia


and tracheoesophageal fistula repair?:
a) Esophageal stricture.
b) Anastomotic leakage.
c) Gastroesophageal reflux.
d) Recurrent fistula.
e) Massive variceal bleeding.

62. The most common condition, which needs liver transplantation in


pediatric age group, is:
a) Metabolic liver disease.
b) Malignant liver tumors.
c) Biliary atresia.
d) Choledochal cyst.
e) Viral hepatitis.

63. The least blood loss during burn wound excision occurs when
escharectomy is performed:
a) During the first 48 hours.
b) On days 3-5 post-burn.
c) On days 6-10 post-burn.
d) After day 10 post-burn.
e) When eschar is infected.

64. Contraindications to breast reconstruction after mastectomy include:


a) Diabetes Mellitus.
b) Age over 60 years.
c) Obesity.
d) Bilateral breast cancer.
e) None of the above.

65. All the following muscles are supplied by the trigeminal nerve
EXCEPT:
a) Tensor veli palatini.
b) Tensor tympani.
c) Masseter.

Dr. wessam alzaidat - General surgery


d) Posterior belly of digastric.
e) Mylohyoid.

66. The advantages of early excision and graft for full-thickness burns in
the first 48 hours post-burn include all of the following EXCEPT:
a) Less blood loss.
b) Less pain.
c) Improved cosmetic result.
d) Decreased hospital stay.
e) Ease of differentiation between partial and full-thickness burns.

67. All of the following statements are correct concerning Corticosteroids


EXCEPT:
a) Inhibit fibroblast migration into the wound.
b) Inhibit the activity of prolylhydroxylase.
c) Prevent collagen deposition.
d) Inhibit wound contracture.
e) Vitamin A restores the harmful effects of steroids.

68. The burn that is difficult to estimate in percentage is the:


a) Electrical burn
b) Sun burn
c) Scald burn
d) Direct flame burn
e) Chemical burn

69. All of the following are signs of burn inhalation injury EXCEPT:
a) Erythema of oral mucosa
b) Coughing
c) Black sputum
d) Red colored burn
e) Burn of nasal hair

70. A blow-out fracture of the orbit is by definition:


a) A fracture that results in rupture of the globe.
b) A fracture that results in disruption of the orbital rim.
c) A fracture of the thin walls of the orbit allowing herniation of
orbital contents.
d) A fracture that results from an explosion.

Dr. wessam alzaidat - General surgery


e) A fracture that includes bones adjacent to the orbit.
71. A patient who has a penetrating wound to the extremity has the
arteriogram shows less than 5mm intimal defects and psudoaneurysm.
Management of this injury should be
a) immediate operation
b) Anticoagulant for 6 wks.
c) Observation
d) Elective repair
e) Antiplatelet therapy for 6 wks.

72. Which of the following statements about carotid endarterectomy is


not true?
a) Patients with a symptomatic critical stenosis benefit from carotid
endarterectomy
b) The risk of contralateral stroke is reduced after carotid
endarterectomy
c) Operative mortality is less than 2%
d) It prolongs survival when compared with patients treated medically
e) It should be accompanied by antiplatelet medication

73. Which of the following is not associated with left-sided portal


hypertension
a) Gastric varices
b) Abdominal pain
c) Splenomegally
d) Hepatomegally
e) Chronic pancreatitis

74. A 15 years old boy complains of excessive sweating of his hands.


They are cold, clammy, and damp to the touch. Perspiration dripping
from his hands forms a small puddle during the examination. The
remainder of medical history is unremarkable. Which of the following
statements is TRUE
a) the result of sympathectomy for this disorder are generally good
b) the sweat glands involved here are apocrine
c) if not treated, this will likely be permanent
d) topical aluminum salts create ulceration
e) a short course of methotrexate is ofen helpful

75. Which of the following statements about blunt tracheobronchial

Dr. wessam alzaidat - General surgery


trauma is true
a) Repair should include a prophylactic tracheostomy
b) Delayed repair is preferred
c) Aneasthesia is safer if the endotracheal tube can be passed beyond
the site of injury
d) Massive hemoptysis is common at presentation
e) Bronchoscopy is hazardous and should be avoided

76. Regarding the functions of a tracheostomy all are true, except:


a) Bypasses an upper airway obstruction.
b) Increases the anatomical dead space.
c) Decreases airway resistance.
d) Protects against aspiration.
e) Allows frequent airway suction.

77. With regard to protein loss after injury, which of the following
statements is true?
a) It results from impaired protein synthesis.
b) It occurs primarily from catabolism of skeletal muscle.
c) It occurs primarily from acute renal failure.
d) It occurs primarily from the site of injury.
e) It can be prevented by total parenteral nutrition.

78. Which of the followings is the most important stimulus for triggering
the endocrine response to injury?
a) Tissue acidosis.
b) Local wound factors.
c) Afferent nerve stimuli from the injured area.
d) Hypovolemia.
e) Temperature changes.

79. All of the followings are true regarding perforated appendicitis,


except:
a) Higher rate in children.
b) Higher rate in old patients.
c) Usually due to delay in presentation and diagnosis.
d) Appendicular mass is felt in more than 50% of the cases.
e) The commonest complication is wound infection.

Dr. wessam alzaidat - General surgery


80. Factors which contribute to wound dehiscence include all the
followings, except:
a) Old age.
b) Coughing.
c) Hypoproteinemea.
d) Anaemia.
e) Malignancy.

81.The lesions in familial adenomatous polyposis with the smallest


malignant potential are
a) Gastric polyps
b) Duodenal polyps
c) Abdominal desmoids
d) Brain tumors
e) Ileal polyps

82. Regarding hernias All true except


a) One portion of the bowel wall is herniated , but not
entire lumen is called Richter hernia
b) Spigelian hernia is Associated with the semilunar
line
c) Femoral hernias are more common in females than
males
d) Paradoudenal hernia is an internal hernia where
the superior mesenteric vein forms part of its
boundaries
e) Obturator hernia Causes anterior thigh pain with
walking

83. All of the following are consequences of obstructive


jaundice except
a) Wound healing is impaired
b) The function of kupffer cells is increased
c) There is a decrease in the absorption of fat soluble
vitamins

Dr. wessam alzaidat - General surgery


d) There is an increased risk of renal failure
e) There is an increased chance of the bile being
infected in malignant obstruction

84. In patients with upper gastrointestinal haemorrhage due to


a peptic ulcer All true except
a) A visible vessel in the ulcer bed at endoscopy is a
risk factor for rebleeding
b) An older patient would be expected to have a
higher chance of rebleeding
c) An adherent clot seen at endoscopy reduces the
risk of rebleeding
d) H2 antagonists do not reduce the bleeding rate
e) A gastric ulcer is more likely to bleed than a
duodenal ulcer

85.In the management of trauma patient all true except


a) Mouth to mouth breathing provides a maximum
inspired oxygen concentration of 10%
b) Criciod pressure should be not be used to aid
endotrachial intubation in vomiting patients.
c) Nasogastric intubation is contraindicated in patients
who are suspected to have fracture of the base of
skull
d) Needle cricothyroidotomy aids effective ventilation
for up to 30 minutes
e) Insertion of (oropharyngial airway) should be
attempted in only on unconscious patients

86. Local anesthetic agents


a) Should not be given intravenously
b) Lignocaine (xylocaine) is four times more potent
than bupivacaine
c) The maximum safe dose for Lignocaine is 3mg/kg
and up to 7mg/kg if mixed with adrenaline
d) Can be used safely in the distal limbs even if mixed
with adrenaline
e) Bupivacaine is less cardiotoxic than lignocaine

Dr. wessam alzaidat - General surgery


87. Regarding anal fissures, all are true except:
a) 10% occur in the posterior midline
b) Multiple fissures suggests the diagnosis of TB or
crohn's disease
c) 50% of the fissures heal with the use of bulking
agent
d) Sphinterotomy has a success rate of over 90%
e) Sphinterotomy is associated with minor fecal
incontinence in over 15% of patients

88. All of the following are indications for surgery in patients


with spontaneous pnuemothorax except;
a) Scuba divers
b) Individuals living in remote area
c) Airline pilots
d) Age more than 60 years
e) Previous contralateral pnuemothorax

89. Low molecular weight Heparin as its primary inhibitory


effect on one of the following:
a. Factor II.
b. Factor IXa.
c. Antithrombin III.
d. Factor Xa.
e. Factor XIIa.

90. The O2 dissociation curve is shifted to the right by :


a) Decreased CO2 tension
b) Increased CO2 tension
c) Increased PH
d) Increased N2 tension
e) Decreased N2 tension

91. The Glasgow coma scale is the dependant upon all of the
following except
a) response to speech
b) respnse to pain

Dr. wessam alzaidat - General surgery


c) response of the pupils
d) motor response
e) response of the patient

92. In traumatic subarachnoid haemorrhage all of the following


are correct except .
a) is commoner than aneurysmal haemorrage
b) is commoner than subarachnoid haemorrhage associated
with AVM.
c) may cause vasospasm and cerebral infarction
d) is often associated with frontal skull fracture
e) is more common in elderly people

93. The adverse prognostic factors for the development of acute


subdural hematoma involve all of the following except
a) old age
b) young age
c) chronic alcoholism
d) skull fracture
e) temporal agenesis

94. Extradural hematoma has the following characters except


a) is more common than acute subdural hematoma
b) is more often associated with vault skull fracture
c) is less often associated with primary brain injury
d) is more like to expand
e) is more likely to be arterial in origin

95. The following are indication for admission to the hospital after a
minor head injury except
a) reduce level of consiousness
b) concussion without skull fracture
c) clinical and radiological evidence of skull frature
d) focal neurological dysfunction
e) difficulty in assessing the patient

96. Which anastamosis location has the highest leakage rate?


a) Jejunojejunostomy after a Roux-en-Y reconstruction
b) Pancreaticojejunostomy
c) Esophagojejunostomy

Dr. wessam alzaidat - General surgery


d) Rectosigmoid anastamosis
e) Gastrojejunostomy

97. Bilibrubin
a) Is conjugated to glucuronic acid in the gallbladder
b) Is transported in hepatic sinusoidal blood bound to
albumin
c) When conjugated, is secreted into bile by passive
diffusion
d) Is converted to urobilinogen by jejunal enterocytes
e) Is produced predominantly by early phase (< 3
days) erythrocyte heme breakdown
98.Exclusion criteria for laparoscopic colectomy for diverticulitis
include all of the following EXCEPT
a) Multiple areas of colonic involvement
b) Purulent peritonitis
c) BMI > 30 kg/m2
d) Free peritoneal perforation
e) Multilocular abscess

99.Forty-eight hours after performing a laparoscopic


cholecystectomy, a surgeon realizes that one of the clips was
placed across the common bile duct. The correct approach
would be
a) Drainage via percutaneous transhepatic cholangiogram
b) Operative removal of the clips
c) Choledochoduodenostomy
d) Loop choledochojejunostomy
e) Roux-en-Y choledochojejunostomy

100. The ilioinguinal nerve


a) Is a branch of the femoral nerve
b) Has a motor component

Dr. wessam alzaidat - General surgery


c) Can be injured easily at the internal spermatic ring
d) Provides sensation for the penis and upper scrotum
e) Runs along the hypogastric vein

1-A
2-C
3-B
4-C
5-B
6-D
7-B
8-B
9-B
10-B
11-B
12-E
13-D
14-E
15-B
16-A
17-A
18-A
19-A
20-D
21-D
22-E
23-E
24-E
25-C
26-E
27-C
28-D
29-E
30-E
31-B
32-E
33-B
34-C
35-E
36-C
37-D
38-D
39-D
40-B
41-B

Dr. wessam alzaidat - General surgery


42-D
43-A
44-D
45-c
46-b
47-a
48-b
49-d
50-c
51-d
52-d
53-c
54-a
55-d
56-b
57-d
58-d
59-e
60-e
61-e
62-c
63-a
64-e
65-d
66-e
67-c
68-a
69-d
70-c
71-c
72-d
73-d
74-a
75-c
76-b
77-b
78-c
79-d
80-d
81-a
82-d
83-b
84-c
85-a
86-c

Dr. wessam alzaidat - General surgery


87-a
88-d
89-d
90-b
91-c
92-d
93-b
94-a
95-b
96-b
97-b
98-c
99-e
100-d

Dr. wessam alzaidat - General surgery


1.Regarding Peutz Jeghers syndrome all are true, except:
a. Is an autosomal dominant condition.
b. Often presents with anaemia in childhood.
c. Is characterised by circumoral mucocutaneous pigmented lesions.
d. Is associated with adenomatous polyps of the small intestine.
e. Malignant change occurs in 2-3% of polyps.

2. Regarding peptic ulceration all are true, except:


a. H. pylori is a gram-negative bacillus.
b. Duodenal is more common than gastric ulceration.
c. Zollinger-Ellison syndrome is associated with gastrin hyposecretion.
d. H2-blockers will heal 85-95% of duodenal ulcers in 8 weeks.
e. Triple therapy can eradicate H. pylori in 80% of patients in one week.

3.Regarding head Injuries all are true, except:


a. More than 1 million people are seen in UK hospitals each year with head injuries.
b. Skull X-rays can not exclude an intracerebral haematoma.
c. Raised intracranial pressure is associated with an increase in cerebral perfusion.
d. Cushing's response consists of a rise in blood pressure and fall in heart rate.
e. Pupillary dilatation usually occurs on the same side of the intracerebral
haematoma.

4. Regarding stones in the gallbladder, one is correct:


a. Pigment stones are the most common.
b. Pigment stones are due to increased excretion of polymerised conjugated bilirubin.
c. Are not a risk factor for the development of gallbladder carcinoma.
d. 90% of gallstones are radio-opaque.
e. A mucocele of the gallbladder is mostly caused by a stone impacted in Hartmann's
pouch.

5.Regarding stones in the common bile duct, all are true, except:
a. Are found in 10 to 15% of patients undergoing cholecystectomy (Without pre-op
ERCP).
b. Can present with Charcot's Triad.
c. Are suggested by a bile duct diameter >8mm on ultrasound.
d. ERCP, sphincterotomy and balloon clearance is now the treatment of choice.
e. If removed by exploration of the common bile duct the T-tube can be removed
after 3days.

6. Regarding crystalloid solutions, all are true, except:


a. Normal saline contains 154 mmol sodium and 154 mmol chloride.
b. 3 litres of dextrose 1/5 th saline in a day will provide 90 mmol of sodium.
c. 2 grams of potassium chloride is equal to 57 mmol of the salt.
d. Hartmann's solution contains calcium and bicarbonate.
e. The daily maintenance potassium requirement of a 40 Kg woman is about 40
1
Dr. wessam alzaidat - General surgery
mmol.

7. Regarding colloid solutions, all are true, except:


a. Human albumin has a molecular weight of 69 kDa.
b. Albumin has a half life in the circulation of about 15 days.
c. Gelatins (e.g. Haemaccel®) are polysaccharides with a MW of about 35 kDa.
d. Dextrans reduce platelet aggregation and can induce anaphylaxis.
e. 6% Hydroxylethyl Starch (HES) is a synthetic polysaccharide derived from
amylopectin.

8. Regarding central parenteral nutrition, all are true, except:


a. Is a hypo-osmolar solution.
b. Typically contains 14-16g nitrogen as L-amino acids.
c. Typically contains about 250g glucose.
d. Is associated with metabolic disturbances in about 5% of patients.
e. Can induce derangement of liver function tests.

9.Regarding solitary thyroid nodules, all are true, except:


a. Are more prevalent in women.
b. In the adult population, less than 10% are malignant.
c. More than 50% of scintigraphically cold nodules are malignant.
d. The risk of a hot nodule being malignant is negligible.
e. Should not be surgically removed in all patients.

10. Regarding the femoral canal, one is correct:


a. Lies lateral to the femoral vein.
b. Has no relation to the inguinal ligament.
c. Has the lacunar ligament as its lateral border.
d. Has the pectineal ligament as its posterior border.
e. Contains the lymph node of Lund.

11.Regarding the pathology of ulcerative colitis, one is true:


a. Shows full thickness bowel inflammation.
b. The rectum is almost always not involved.
c. 10% of patients have terminal ileal disease.
d. Enterocutaneous or intestinal fistulae are common.
e. The serosa is usually affected.

12. Regarding benign breast disease, all are true, except:


a. Cyclical mastalgia is the commonest reason for referral to the breast clinic.
b. Fibroadenomas are derived from the breast lobule.
c. Lactational breast abscesses are usually due to Staph aureus.
d. Duct ectasia is less common in smokers.
e. Atypical lobular hyperplasia is associated with an increased risk of breast cancer.

2
Dr. wessam alzaidat - General surgery
13.Regarding infantile hypertrophic pyloric stenosis, one is true:
a. Occurs with a male : female ratio of 4:1.
b. Sons of affected mothers have a 80% risk of developing the lesion.
c. Invariably presents between six and eights months of age.
d. Typically presents with bile stained projectile vomiting.
e. Surgical treatment is by Heller's cardiomyotomy.

14. Regarding oesophageal atresia , all are true, except:


a. Is often associated with a distal tracheo-oesophageal fistula.
b. Polyhydramnios is often present late in pregnancy.
c. 50% have other associated congenital abnormalities.
d. Contrast x-ray studies are necessary to confirm the diagnosis.
e. Post-operatively over 15% develop oesophageal strictures.

15.All the following are features of Fallot's Tetralogy, except:


a. Atrial septal defect.
b. Pulmonary stenosis.
c. Right ventricular hypertrophy.
d. A right to left cardiac shunt.
e. Cyanotic attacks during feeding and crying.

16.Regarding phaeochromocytomas, one is correct:


a. Are neuroendocrine tumours of the adrenal cortex.
b. Are benign in 10% of cases.
c. Occur as part of the MEN type I syndrome.
d. Can be imaged with a meta-iodobenzylguanidine scan.
e. Require only beta-blockage drug prior to surgery.

17.Regarding the functions of a tracheostomy all are true, except:


a. Bypasses an upper airway obstruction.
b. Increases the anatomical dead space.
c. Decreases airway resistance.
d. Protects against aspiration.
e. Allows frequent airway suction.

18. Regarding hepatitis B, all are true, except:


a. It is due to a DNA virus.
b. It has an incubation period of 2 to 6 days.
c. It can be transmitted via blood products.
d. About 1% of the UK population are hepatitis B surface antigen positive.
e. About 10% of infected patients become chronic carriers.

3
Dr. wessam alzaidat - General surgery
19.The best surgical modality to treat recurrent primary spontaneous
pneumothorax in a 20 year old man is:
a.bullectomy.
b.pleurectomy.
c.bullectomy and pleurectomy.
d.pleurodesis.
e.decortication.

20-Regarding the anatomy of diaphragm, all are true except:


a.The hiatal opening is situated at the level of the tenth thoracic vertebra.
b.The inferior vena cava passes through the right side of diaphragm at the level of
12th thoracic vertebra.
c.The principle blood supply of the diaphragm is derived directly from the aorta.
d.The left diaphragm is lower than the right diaphragm in a normal adult.
e.Nerve supply to the diaphragm is via phrenic nerve which arises from 3rd,4th&5th
cervical nerve roots.

21.Which one of the following diagnostic methods is the best to diagnose


pneumothorax?
a.Standing CXR, expiratory film.
b.Standing CXR, inspiratory film.
c.Supine CXR, expiratory film.
d.Supine CXR, inspiratory film.
e.Lateral CXR.

22.Regarding chest trauma, all are true, except:


a.Rib fracture in a child is usually a marker of severe injury.
b.Thoracostomy tube should be inserted prior to CXR in any patient with suspected
pneumothorax associated with hypotension.
c.A patient with flail chest, should be considered for ventilatory support if his
respiratory rate is above 35/min.
d.Exploratory thoracotomy should be done if the amount of blood production through
the chest tube is more than 200 cc hourly for 4 hours.
e.Chest tube should be clamped during the transport of the patient.

23. The best site to insert a chest tube is:


a.2nd intercostal space, anterior axillary line.
b.3rd intercostal space,posterior axillary line.
c.4th intercostal space ,posterior axillary line.
d.5th intercostal space, anterior axillary line.
e.7th intercostal space, mid axillary line.

4
Dr. wessam alzaidat - General surgery
24. The best non invasive diagnostic procedure to diagnose diaphragmatic injury
after trauma is:
a.Chest xray.
b.Chest CT scan.
c.Chest MRI.
d.Chest ultrasound.
e.Barium meal.

25.Regarding the anatomy of the tracheobronchial tree and the lung, all are
true, except:
a.The right lung is larger than the left lung.
b.The major (oblique fissure) separates the lower lobe from the upper and middle
lobe.
c.There are 8 segments in the right lung.
d.The right main bronchus is shorter and wider than the left main bronchus.
e.The posterior wall of the trachea is membranous.

26. All the followings are true data about the anatomy and the physiology of
lower oesophageal sphincter, except:
a.In adults, it measures 3 to 5 cm in length.
b.In normal individuals, manometry demonstrates pressure of 5 mm Hg within the
lower oesophageal sphincter.
c.The lower oesophageal sphincter remains contracted at rest.
d.It is not a true sphincter.
e.It normally relaxes with the approach of the primary peristaltic wave.

27.Which of the following is probably a posterior mediastinal mass rather than


an anterior one?
a.Thyroid tumour.
b.Neurogenic tumour.
c.Thymic carcinoma.
d.Teratoma.
e.Lymphoma.

28. Regarding thoracic duct,one is correct:


a.Is the main collecting vessel of the lymphatic system and is far smaller than the
right terminal lymphatic duct.
b.Originates from the cysterna chyle at the level of 4th lumbar vertebra.
c.Originates in the left side.
d.Ascends to enter the chest through the inferior vena cava hiatus.
e.Lies on the anterior surface of the vertebral column behind the esophagus and
between the aorta and the azygous vein.

5
Dr. wessam alzaidat - General surgery
29. The maximum safe dose of local anesthetic administered subcutaneously in a
70 kg man is:
a.10 to 20 mL of 1% lidocaine.
b.40 to 50 mL of 2% lidocaine with epinephrine.
c.40 to 50 mL of 1% lidocaine with epinephrine.
d.40 to 50 mL of 1% bupivacaine.
e.40 to 50 mL of 1% lidocaine without epinephrine.

30.Which of the following vessels of the lower limb is least likely to be occluded
by atherosclerosis?
a.Anterior tibial.
b.Peroneal.
c.Proximal posterior tibial.
d.Distal posterior tibial.
e.Dorsalis pedis.

31. After thoracotomy, a woman has a loss of sensation of the nipple areolar
complex,injury to which intercostal nerve is responsible for this?
a.Second.
b.Third.
c.Fourth.
d.Fifth.
e.Sixth.

32. Oxygen delivery can be increased by increasing all of the followings, except:
a.Hemoglobin.
b.Atmospheric pressure.
c.Cardiac output.
d.Inspired oxygen concentration.
e.Oxygen extraction.

33. Which of the followings is the best abdominal site to assess the bowel sounds
using the stethoscope?
a.Right upper quadrant.
b.Right lower quadrant.
c.Left upper quadrant.
d.Left lower quadrant.
e.Periumbilical.

34. Brain stem death is diagnosed in the absence of these brain stem reflexes,
except:
a- Corneal reflexes.
b- Pupillary reflexes.
c- Doll's eye movements.

6
Dr. wessam alzaidat - General surgery
d- Caloric response.
e- Apnoeic test.

35. The followings are indications for admission to hospital after a minor head
injury, except:
a- reduced level of consciousness.
b- Concussion without skull fracture.
c- Clinical or radiological evidence of skull fracture.
d- Focal neurological dysfunction.
e- Difficulty in assessing the patient.

36. In Extradural haematomas all the followings occur, except:


a- One in ten occur in the frontal region.
b- Less than half exhibit a lucid interval.
c- 20% occur in the temporal area.
d- The first symptom is often progressive paroxysmal headache.
e- Nearly always associated with vault skull fracture.

37. The source of bleeding in the extradural space are the followings, except:
a- Diploic vessels of the skull vault.
b- The cerebral veins.
c- The middle meningeal vessels.
d- The dural sinuses.
e- The dural arterioles.

38.The following regimens are used to treat raised intracranial pressure


(ICP), except:
a- Hypoventilation .
b- Mannitol 20% solution, 1g mannitol/kg body weight.
c- Drainage of CSF.
d- Barbiturates.
e- Surgical decompression.

39. Cerebrospinal fluid (CSF) characterizes by the followings ,except:


a. Circulates at about 0.5 ml/min.
b. Secreted at about 500 ml/24 hours.
c. Secreted in the lateral,third,and fourth ventricles.
d. Secreted predominantly by active secretion.
e. Absorbed by the choroid villi.

7
Dr. wessam alzaidat - General surgery
40. Definitive diagnosis of Pulmonary embolism is made by one of the
followings:
a- ECG (EKG).
b- Chest X-ray.
c- VQ scan.
d- ABG.
e- Pulmonary angiography.

41.In the body metabolism, 10g of protein, would produce approximately:


a. 20 kcal.
b.41 kcal.
c.410 kcal.
d.4100 kcal.
c. 900 kcal.

42. The Glasgow coma scale is dependant upon all of the followings, except:
a.Response to speech.
b.Response to pain.
c.Response of the pupils.
d.Motor response.
e.Response of the patient.

43. Which one of the following spinal segments causes signs of wasting of the
intrinsic muscles of the hand?
a- C5.
b- C7/8.
c- T1.
d- L5.
e- S1.

44. The methods by which bacteria gain access into the brain include all the
followings, except:
a- Direct spread from the surrounding tissues.
b- Haematogenous.
c- Via implantation during trauma.
d- Lymphatics.
e- During surgery.

45. All the followings are premalignant lesions of the skin, except:
8
Dr. wessam alzaidat - General surgery
a. Actinic keratosis.
b. Bowen’s disease.
c. Seborrheic keratosis.
d. Dysplastic nevi.
e. Nevus sebaceous.

46. The commonest cause of death after major burns in modern burn units is:
a) Inadequate resuscitation.
b) Pneumonia.
c) Burn wound sepsis.
d) Renal failure.
e) Bacterial endocarditis.

47. Complications of mandibular fractures include all the followings, except:


a. Malocclusion.
b. Lower lip paraesthesia.
c. Trismus.
d. Ankylosis of the temperomandibular joint.
e. Paralysis of the lower lip.

48. An ulnar nerve injury at the elbow would result in all the followings, except:
a. Clawing of the ring and little fingers.
b. Inability to abduct the little finger.
c. Inability to flex the ring and little fingers.
d. Inabilility to adduct the thumb.
e. Radial deviation of the wrist in flexion.

49. The most important factor which decide the efficacy of laser treatment for a
particular condition is:
a. Age of patient.
b. Depth of lesion.
c. Energy Fluence (Power density x Duration of treatment).
d. Spot size.
e. Wavelength as decided by the lasing medium.

50. A blow-out fracture of the orbit is by definition:


a. A fracture that results in rupture of the globe.
b. A fracture that results in disruption of the orbital rim.
c. A fracture of the thin walls of the orbit allowing herniation of the orbital contents.
d. A fracture that results from an explosion.
e. A fracture that includes bones adjacent to the orbit.

51. Which of the followings is considered the primary blood supply to the skin?
a. Fascial plexus.
b. Subcutaneous plexus.
9
Dr. wessam alzaidat - General surgery
c. Subdermal plexus.
d. Dermal plexus.
e. Subepidermal plexus.

11
Dr. wessam alzaidat - General surgery
52. Principles of skin wound repair include all of the followings, except:
a. Obliteration of the dead space.
b. Eversion of the skin edges.
c. Tension free closure.
d. Layered tissue closure.
e. Use of nylon simple sutures at the skin edge.

53. The most reliable indicator of the adequacy of burn resuscitation is:
a. Central venous pressure.
b. Pulmonary capillary wedge pressure.
c. Urine output.
d. Blood pressure and heart rate.
e. Mental status.

54. The most common site for squamous cell carcinoma of the lip is the:
a. Upper lip midline.
b. Upper lip laterally.
c. Lower lip midline.
d. Lower lip laterally.
e. Oral commissure.

55. Which of the following cell types is essential for normal wound healing?
a. Leukocytes.
b. Monocytes.
c. Platelets.
d. Erythrocytes.
e. All of the above.

56. The most important factor in the development of ulcers in a spinal cord
injury patient is:
a. Malnutrition.
b. Septic episodes.
c. Anemia.
d. Local pressure.
e. Diminished sensation.

57. Which vitamin D metabolite stimulate intestinal calcium absorption?

a. 1,25–dihydroxyvitamin D.
b. 1,25–dihydroxyvitamin D1.
c. 1,25–dihydroxyvitamin D2.
d. 1,25–dihydroxyvitamin D3.
e. 1,25–dihydroxyvitamin D4.

11
Dr. wessam alzaidat - General surgery
58. Urinary stones generally do not pass spontaneously if they are larger than?

a. 2 mm.
b. 3 mm.
c. 4 mm.
d. 7 mm.
e. 8 mm.

59.Which of the following studies replace intravenous urography in the initial


evaluation of patients with renal colic?

a. Kidney, ureter and bladder study.


b. Renal tomography.
c. Ultrasonography.
d. CTScan.
e. MRI.

60. Initial hormonal evaluation of an infertile man with a sperm count of 5


million sperm/ml should include assays of testosterone level and:

a. Prolactin.
b. Follicle–stimulating hormone (FSH).
c. FSH, luteinizing hormone (LH) and prolactin.
d. FSH and thyroid function.
e. Sex steroid hormone binding globulin (SHBG).

61. Where does benign prostatic hyperplasia (BPH) originate?

a. In the transition zone.


b. In the peripheral zone.
c. In the periurethral glands.
d. In the transition zone and periurethral zone.
e. In the whole organ.

62. It is advisable in a man with BPH and a slightly elevated creatinine level to
perform a(an):

a. Transurethral resection of the prostate (TURP).


b. Intravenous pyelography.
c. Renal sonography.
d. Urodynamic study.
e. kidney, ureter and bladder X-ray.

12
Dr. wessam alzaidat - General surgery
63.A 20 year old man has undergone a retroperitoneal dissection for a testicular
germ cell tumor. The inferior mesenteric artery has been divided during
reflection of the intestines to expose the retroperitoneum. This can be expected
to result in:

a. Ischemia of the descending colon.


b. Ischemia of the sigmoid colon.
c. Ischemia of the rectum.
d. Ischemia of the transverse colon.
e. None of the above.

64. The predominant buffering system in humans is:

a. Bicarbonate.
b. Titratable acids.
c. Ammonium (NH4+).
d. Urea.
e. Phosphate.

65. Terminal hematuria (at the end of the urinary stream) is usually due to:

a. Bladder neck or prostatic inflammation.


b. Bladder cancer.
c. kidney stones.
d. Ureteric calculi.
e. Ureteral stricture disease.

66. Pneumaturia may be due to all of the followings, except:

a. Diverticulitis.
b. Colon cancer.
c. Recent urinary tract instrumentation.
d. Inflammatory bowel disease.
e. Ectopic ureter.

67. As one proceeds outward from the adrenal medulla, the three separate
functional layers of the adrenal cortex are, in correct order:

a. The zona reticularis, zona fasciculata, then zona glomerulosa.


b. The zona fasciculata, zona reticularis, then zona glomerulosa.
c. The zona glomerulosa, zona fasciculata, then zona reticularis.
13
Dr. wessam alzaidat - General surgery
d. The zona glomerulosa, zona reticularis, then zona fasciculata.
e. The zona reticularis, zona glomerulosa, then zona fasciculata.

68. What proportion of the cardiac output is delivered to the kidney?

a. 5%.
b. 20%.
c. 50%.
d. 85%.
e. 100%.

69. During surgical dissection, the ureter can be identified as it enters the pelvis:

a. At the aortic bifurcation.


b. Crossing the superior border of the sacrum.
c. Crossing the common iliac artery at the branching of the internal iliac artery.
d. Crossing the uterine artery.
e. At the internal inguinal ring.

70. All the followings are consequences of major haemorrhage, except:


a- Rise in serum osmolality.
b- Rise in intracellular osmotic pressure.
c- Decrease in plasma oncotic pressure .
d- Peak albumin synthesis in 6 hrs.
e- Rise in blood sugar.

71. Signs of cardiac tamponade may include all of the followings, except:
a- Agitation and confusion.
b- Hypotension.
c- Congested neck veins.
d- Muffled heart sounds.
e- Deviated trachea .

72. About achalasia, one is not true :

a- It is the most common primary esophageal disorder.


b- Manometry reveals non relaxation of the lower esophageal sphincter at rest.
c- Acceptable treatment includes Endoscopic myotomy.
d- It is most often associated with hyperperistalsis of the esophageal body.
e- The intraluminal pressure is elevated.

14
Dr. wessam alzaidat - General surgery
73. All are causes of metabolic acidosis, except:
a- Untreated diabetes mellitus.
b- Duodenal Ulcer.
c- Uremia.
d- Shock.
e- Liver failure .

74. All are recognized complications of acute pancreatitis, except:


a- Adult respiratory distress syndrome .
b- Tetany.
c- Acute renal failure .
d- Disseminated intravascular coagulation .
e- Hypercalcaemia .

75. The following body fluids have high risk of HIV transmission, except:
a- Semen .
b- Blood.
c- Urine.
d- Breast milk.
e- Fresh frozen plasma.

76. Factors which contribute to wound dehiscence include all the followings,
except:
a- Old age.
b- Coughing.
c- Hypoproteinemea.
d- Anaemia.
e- Malignancy.

77. The most common extracranial solid tumour in children is:


a. Wilm's tumour.
b. Neuroblastoma.
c. Hepatoblastoma.
d. Hepatocellular carcinpma.
e. Seminoma.

78. In intussusception, all are true, except:


a. Idiopathic causes are the most common in the infantile age group.
b. Ileo-colic intussusception are seen most commonly.
c. Abdominal colic and vomiting present in most of the cases.
d. Bloody stool is an indication for open surgical reduction.
e. In older children, a pathological leading point must be excluded.

79. Regarding inguinal hernia in children, all are true, except:


15
Dr. wessam alzaidat - General surgery
a. Is more common in pre-term babies.
b. Is more common in males.
c. Testicular atrophy might be a long term complication.
d. Complications are more common in pre-terms than in term babies.
e. Contralateral groin exploration is routinely done to exclude bilateral hernias.

80. One of the followings is true regarding undescended testes:


a. The incidence is 1 in 10 000 boys.
b. The most common location is in the femoral region.
c. Is associated with high rate of infertility.
d. Orchidopexy is indicated at the age of five years.
e. Bilateral cases are more frequent than unilateral cases.

81. The most common indication for splenectomy in children is:


a. Splenic injury in blunt abdominal trauma.
b. Sarcoidosis.
c. Idiopathic thrombocytopenic purpura ( ITP ).
d. Hereditary spherocytosis.
e. Thalassemia.

82. Recurrent UTI in children mostly raise the suspicion of:


a. Urethral diverticulum.
b. Vesico ureteric reflux.
c. Ureteric duplication.
d. Renal ectopia.
e. Neurogenic rectum.

83. The absolute contraindication for circumcision in children is:


a. Hypospadias.
b. Blood dyscrasia.
c. Recurrent UTI.
d. Para-phimosis.
e. Phimosis.

84. A child weighs 15 Kg, his fluid requirement is:


a. 50 ml per hour.
b. 50 ml per kg per hour.
c. 1000 ml per 24 hours.
d. 10 ml per kg per hour.
e. 150 ml per hour.

16
Dr. wessam alzaidat - General surgery
85. Biliary Atresia is definitely diagnosed by:
a. HIDA scan.
b. Abdominal ultrasound.
c. Liver function test.
d. Abdominal C.T scan with i.v contrast.
e. Intra-operative cholangiogram.

86. Two years old infant previously healthy, presented with sudden onset of
respiratory distress, Choking and transient cyanosis. The most likely diagnoses
is:
a. F.B aspiration.
b. Pneumothorax.
c. Pneumonia.
d. Hemothorax.
e. Hemo-pneumothorax.

87. All are causes of bile-stained vomiting in newborns, except:


a. Sepsis.
b. Duodenal atresia.
c. Meconium ileus.
d. Complicated intestinal malrotation.
e. Esophageal atresia.

88. All are normal blood tests in the term newborn at day one of age, except:
a. Blood glucose 45mg-60mg / dl.
b. WBC 20. 000 to 30.000.
c. Creatinine up to 1.3mg%.
d. PCV 55%- 65%.
e. ESR 60mm/first hour.

89. The most common cause of death in old children is:


a. Malignancy.
b. Congenital heart disease.
c. Trauma.
d. Renal Anomalies
e. CNS abnormalities.

90. Causes of abdominal calcification on x-ray include all of the followings,


except:
a. Faecolith.
b. Wilm's tumour.
17
Dr. wessam alzaidat - General surgery
c. Neuroblastoma.
d. Nephrocalcinosis.
e. Calcified lymph nodes.

91. Which of the followings is the most important stimulus for triggering the
endocrine response to injury?

a. Tissue acidosis.
b. Local wound factors.
c. Afferent nerve stimuli from the injured area.
d. Hypovolemia.
e. Temperature changes.

92. Which of the following substances is elevated during the acute response to
injury?

a. Glucagon.
b. Insulin.
c. Thyroid stimulatring hormone (TSH).
d. Prolactin.
e. Calcitonin.

93. With regard to protein loss after injury, which of the following statements is
true?
a. It results from impaired protein synthesis.
b. It occurs primarily from catabolism of skeletal muscle.
c. It occurs primarily from acute renal failure.
d. It occurs primarily from the site of injury.
e. It can be prevented by total parenteral nutrition.

94. Normal wound healing is accelerated by one of the followings?


a. Ascorbic acid.
b. Vitamin A.
c. Zinc.
d. Increased local oxygen tension.
e. Frequent dressing.

95. All of the followings are true regarding perforated appendicitis, except:
a. Higher rate in children.
b. Higher rate in old patients.
c. Usually due to delay in presentation and diagnosis.
d. Appendicular mass is felt in more than 50% of the cases.
e. The commonest complication is wound infection.

18
Dr. wessam alzaidat - General surgery
96. With regard to blunt abdominal trauma, the commonest intra abdominal
organ which can be affected is?
a. Urinary bladder.
b. Large intestine.
c. Pancreas.
d. Stomach.
e. Spleen.

97. One of the followings is not a cause of hypokalemia?


a. Inadequate oral intake.
b. Potassium-free intravenous fluids.
c. High output nasogastric tube.
d. Blood transfusion.
e. Massive vomiting and diarrhea.

98. With regard to wound classification, penetrating abdominal trauma is?


a. Clean wound.
b. Clean/contaminated wound.
c. Contaminated wound.
d. Dirty wound.
e. Not classified.

99. The most common cause of hypercalcemia in hospitalized patients is?


a. Malignancy.
b. Over oral intake.
c. Bone fracture.
d. Blood transfusion.
e. Parathyroid adenoma.

100. One of the following is not a cause of dilutional hyponatremia?


a. Excessive oral water intake.
b. Iatrogenic (intravenous) excess of free water administration.
c. Postoperative increase of antidiuretic hormone secretion.
d. Decreased oral intake of sodium.
e. Some drugs which cause water retention.

19
Dr. wessam alzaidat - General surgery
Key Answers

21
Dr. wessam alzaidat - General surgery
1. D
2. C
3. C
4. E
5. E
6. C
7. C
8. A
9. C
10. D
11. C
12. D
13. A
14. D
15. A
16. D
17. B
18. B
19. C
20. B
21. A
22. E
23. D
24. C
25. E
26. B
27. B
28. E
29. C
30. C
31. C
32. E
33. B
34. C
35. B
36. C
37. B
38. A
39. E
40. E
41. B
42. C
43. C

21
Dr. wessam alzaidat - General surgery
44. D
45. C
46. B
47. E
48. C
49. E
50. C
51. C
52. E
53. C
54. D
55. B
56. D
57. D
58. C
59. D
60. B
61. D
62. C
63. E
64. A
65. A
66. E
67. A
68. B
69. C
70. B
71. A
72. D
73. B
74. E
75. D
76. D
77. B
78. D
79. E
80. C
81. D
82. B
83. A
84. A
85. E
86. A

22
Dr. wessam alzaidat - General surgery
87. E
88. E
89. C
90. A
91. C
92. A
93. B
94. C
95. D
96. E
97. D
98. D
99. A
100. D

23
Dr. wessam alzaidat - General surgery
1. which of the following amino acids is a critical nutrient
supporting the immune function:
a) Taurine.
b) Branched chain amino acids (BCAA).
c) Alanine. *
d) Arginine.
e) Tyrosine.

2. The optimal initial treatment of aspiration pneumonia is:


a) Bronchoalveolar lavage with antacids.
b) Bronchoalveolar lavage with saline.
c) Parenteral corticosteroids.
d) Maintenance of oxygenation. *
e) Early prophylactic antibiotics.

3. All are true about ultrasound, EXCEPT:


a) Can detect a lesion less than 1 cm in the liver. *
b) Is useful about differentiating a thyroid cyst.
c) It helps in the diagnosis of acute appendicitis.
d) Its not the best modality to visualize the pancreas.
e) Investigation of choice in GB stones.
4. Which statement is correct regarding neurapraxia:
a) The injury involves the axon without loss of continuity of
nerve sheath.
b) Momentary loss of conduction through the nerve without any
organic lesion. *
c) Partial or complete division of the nerve.
d) Spontaneous recovery is impossible.
e) Wallerian degeneration occurs.

5. The most important factor in the development of pressure ulcers


in a spinal cord injury patient is:
a) Malnutrition.
b) Septic episodes.
c) Anemia.
d) Local pressure. *
e) Diminished sensation.

6. The earliest sign of Volkmann's ischemic contracture is:


a) Absence of the radial pulse.
b) Pain on passive finger extension. *
c) Loss of voluntary finger motion.
d) Absent sensation in the median nerve distribution.
e) Pallor of the nail beds.

Dr. wessam alzaidat - General surgery


7. Regarding the oxygen dissociation curve, one is TRUE:
a) Is shifted to the left by increase 2,3-DPG.
b) Is sigmoid shape for both hemoglobin and myoglobin.
c) Is shifted to the left when one ascends to high altitude.
d) Is shifted to the right by an increase in the PH.
e) Is ideally 50% saturated at a PO2 of 28mmHg. *

8. Regarding insulin, all are true EXCEPT:


a) It is a polypeptide composed of two amino acid chains.
b) Its secretion is affected by glucagon.
c) It stimulates gluconeogenesis. *
d) It has little or no effect on the uptake of glucose in the brain.
e) If lacking, it may be a cause for fatty liver.

9. Which of the following statements regarding hemorrhoids is NOT


TRUE:
a) Hemorrhoids are specialized "cushions" present in every one to
aid continence.
b) External hemorrhoids are covered by skin, while internal
hemorrhoids are lined by mucosa.
c) Pain is often associated with uncomplicated hemorrhoids. *
d) Haemorrhoidectomy is reserved for third and fourth degree
hemorrhoids.
e) They are classified and treated according to the degree of their
symptoms.
10. Infection affects wound healing by all the following mechanisms
EXCEPT:
a) Prolonging oedema.
b) Decreasing tissue PO2.
c) Increasing collagenolysis.
d) Decreasing the inflammatory phase. *
e) Increasing the inflammatory phase.

11. All the followings may be linked to the etiology of malignant


melanoma EXCEPT:
a) Fair complexion.
b) Intermittent sun exposure.
c) Positive family history.
d) Junctional nevus. *
e) Dysplastic nevus.

12. The most reliable indicator of the adequacy of burn resuscitation


is:
a) Central venous pressure.
b) Pulmonary capillary wedge pressure.
Dr. wessam alzaidat - General surgery
c) Urine output. *
d) Blood pressure and heart rate.
e) Mental status.
13. The most common site for squamous cell carcinoma of the lip is
the:
a) Upper lip midline.
b) Upper lip laterally.
c) Lower lip midline.
d) Lower lip laterally. *
e) Oral commissure.
14. As the amount of total body fat increase, the total body water:
a) Increases.
b) Decrease. *
c) Unchanged.
d) Depends on patient's age.
e) Depends on patient's sex.
15. One week of starvation results in a loss of approximately 100 gm
of nitrogen (13.8 gm N/ day). This is associated with protein loss
of:
a) 625 gm. *
b) 100 gm.
c) 1000 gm.
d) 50 gm.
e) 16 gm.
16. The HIGHEST potassium content is in:
a) Saliva. *
b) Gastric juice.
c) Bile.
d) Ileal.
e- Duodenal
17. During vessel wall injury, the initial step in clot formation is:
a) Local mast cells release of adenosine diphosphate
(ADP).
b) Fibrin polymerization.
c) Plasminogen activation.
d) Platelet adherence to sub endothelial collagen. *
e) None of the above.
18. Postoperative staphylococcal wound infection usually detected in:
a) 1st to 2nd day.
b) 3rd to 4th day.
c) 5th to 8th day. *
d) 9th to 14th day.
e) After the 14th day.
19. A palpable radial pulse indicates a systolic blood pressure greater
than:
Dr. wessam alzaidat - General surgery
a) 40 mmHg.
b) 60 mmHg.
c) 80 mmHg. *
d) 100 mmHg.
e) 120 mmHg.
20. What is the first parameter to change in early hypovolemic
shock?
a) Systolic blood pressure.
b) Pulse rate. *
c) Diastolic blood pressure.
d) Respiratory rate.
e) Level of consciousness.
21. A previously healthy 18 years old man involved in automobile
accident. He lost consciousness, but regains it and appeared
normal. An hour later he became somnolent and has weakness on
his right side, with dilated pupil on the left side. Most likely he
has:
a) Acute epidural hematoma. *
b) Acute subdural hematoma.
c) Subarachnoid hemorrhage.
d) Intraventricular hemorrhage.
e) Brain concussion.
22. Concerning factors that increase the risk of pulmonary
embolism, all are correct EXCEPT:
a) Superficial phlebitis one year ago. *
b) Estrogen therapy.
c) Obesity.
d) Pregnancy.
e) Deep venous thrombosis one year ago.

23. The TRUE statement regarding small intestine is:


a) The entire small intestine is intraperitoneal.
b) The jejunum is longer in length, larger in diameter, and thinner-
walled than ileum.
c) The muscularis, the muscle layer, provide strength for placement
of sutures for creation of a bowel anastomosis.
d) Peyer’s patches are most prominent in the distal ileum *
e) The marginal artery of Drummond provides blood supply to the
duodenum.

24. A 40 years old female had a subtotal thyroidectomy, several


hours later she complained of difficulty in breathing and she had
stridor and a markedly swollen, tense neck wound the first step in
the management of this patient should be to:
a) Intubate with an endotracheal tube.
Dr. wessam alzaidat - General surgery
b) Perform a tracheostomy.
c) Control the bleeding site in the operating room.
d) Open the wound and evacuate the hematoma at bed side *
e) Aspirate the hematoma.

25. Patients with acute pancreatitis who are at increased risk to


develop complications include those with each of the following
EXCEPT:
a) Ransons score> 3.
b) An APACHI II score >12.
c) Poorly enhancing areas of 50% of the pancreas on a dynamic CT
scan.
d) Amylase values more than eight times upper limits of normal. *
e) Partial obliteration of the lesser sac and acute fluid collections
near the splenic hilum and inferior to pancreatic head.

Dr. wessam alzaidat - General surgery


26. Regarding acute suppurative ( toxic ) cholangitis, one is TRUE :
a) Third generation cephalosporin are the antibiotic of choice for
patients with renal impairment.
b) Urgent decompression of the biliary tree is indicated. *
c) Emergency surgical decompression is indicated for patients with
known cholilithiasis.
d) The catheter for percutaneous drainage should be placed through
the common duct into the duodenum.
e) Nasobiliary tube decompression is an effective form of drainage
for obstructing lesion in the upper third of the common duct.
27. The most common presenting symptom in colorectal carcinoma
in patients under age of 40 is:
a) Abdominal pain.
b) Weight loss.
c) Back pain.
d) Rectal bleeding. *
e) Abdominal distension.

28-A 24 years old man has appendectomy, histopathological report


notes acute inflammation with a 1.2 cm carcinoid tumor on the mid
of appendix. This patient should have:

f) No further treatment. *
g) Chemotherapy.
h) Right hemicolectomy.
i) Serial urinary 5- hydroxyindole acetic levels.
j) Regional radiation therapy.
29- Gastric ulcers are:
A- Malignant when located in the greater curvature.
B- More common in women.
C- Most commonly complicated by perforation.*
D- Associated with hypersecretion of acid in over 90% of cases.
E- Not associated with Helicobacter pylori.
30- Which is the most commonly cultured hospital acquired
organism in critical care patients with aspiration pneumonia?
a- Streptococcus pneumoniae
b- Staphylococcus aureus
c- Anaerobic species
d- Pseudomonas aeruginosa*
e- Haemophilus influenzae

31- Which of the following will immediately delay or cancel an


elective surgical case if not obtained appropriately preoperatively?
a- CBC
b-Urinanalysis
Dr. wessam alzaidat - General surgery
c- CXR
d- Informed consent*
e- ECG
32- The most common cause of nipple discharge is
a- carcinoma
b- fibrocystic disease
c- intraductal papilloma*
d- duct-ectasia
e- trauma
33- ALL OF THE FOLLOWING ARE SINGS OF ATROPINE
POISONING EXCEPT
a) dry mouth
b) red face
c) small pupils *
d) tachycardia
e) drowsiness
34- PAIN IS CARRIED TO THE BRAIN BY THE FOLLOWING
TRACT
a) spino cerebellar tract
b) spino thalamic tract *
c) cortico- spino tract
d) reticular tract
e) olvospinal tract
35- A CETYLE CHOLINE IS THE CHEMICAL TRANSMITTER
IN
a) all; pre and post ganglionic sympathetic system
b) all; pre and post ganglionic parasy pathetic system *
c) all post ganglionic sympathetic system
d) in adrenal medulla
e) all pre ganglionic sympathetic system
36- ALL OF THE FOLLOWING AFFECTING NERVE
CONDUCTION EXCEPT:
a) myelination of the nerve
b) nerve fiber diameter
c) hypoxia
d) autonomic nervous system *
e) inter nodal distance

37- THE MOST HELPFULL CLINICAL SIGN FOR SPINAL CORD


TUMOUR AT T10 LEVEL IS:
a) exaggerated of knee jerk
b) anesthesia up to umbilicus level *
c) urine retention
d) anesthesia up to hypogastrium
e) dissociated sensory loss
Dr. wessam alzaidat - General surgery
38- THE EARLY TREATMENT OF CERVICAL SPINES INJURY
INCLUDES ALL OF THE FOLLWING EXEPT:
a) immobilization by neck collar
b) the patient should be covered
c) lateral spine x-ray while patient on stretcher
d) change the patient position to take off his clothes in E.R *
e) treatment of spinal shock
39- THE MOST IMPORTANT FUNCTION OF INTERVERTEBRAL DISC
IS:
a) Preserve spinal curvature
b) Protects spinal cord
c) Shock absorber *
d) Transmit weight of the body to the lower limbs
e) Has no significant function on adult age
40 – BLOOD SUPPLY OF SPINA LCORD IS USUALLY COME FROM :
a) vertebral arteries only
b) ant , post spinal arteries and radicular arteries *
c) vertebral arteries and abdominal aorta
d) vertebral arteries and ascending cervical arteries only
e) radicular arteries from the aorta only
41- FRACTURE OF THE L 4 VERTEBRAE WITH LOWER LIMBS
WEAKNESS IS DUE TO :
a) spinal cord contusion at that level
b) only dura injury
c) dura and nerves injury *
d) spinal cord , dura and nerves injury
e) spinal concussion without any nerve injury
42 – BARO RECEPTORS IS LOCATED IN :
a) skin
b) wall of the great vessels in the neck *
c) heart
d) in the brain at the medulla level
e) in the vessels of the brain

43- THE MOST SERIOUS COMPLICATION WHICH AFFECT THE


SEQUALE OF HEAD INJURY IS DUE TO:
a) brain infection and brain abscess
b) brain hypoxia *
c) secondary bleeding
d) decrease intracranial pressure
e) CSF leakage
44-A LADY IN CHILDBEARING AGE WITH REPEATED PREGNANCES
AND HAS POOR INTAKE OF ANIMAL DIET IS PRONE TO:
Osteoporosis
a) Osteomalacia *
Dr. wessam alzaidat - General surgery
b) Rickety bone
c) Vitamin B deficiency
d) Compressed spinal fractures
45- ALL OF THE FOLLOWING STRUCTURES PASSING UNDER THE
FLEXOR RETINACULUM EXCEPT:
Flexor Digitorum longus
a) Flexor Digitorum brevis
b) Median nerve
c) Flexor indices
d) Palmaris longus *
46- THE ORIGION OF SYMPATHETIC SYSTEM FROM THE SPINAL
CORD IS:
Cranio cervical outflow
a) Thoraco lumbar outflow *
b) Cranio sacral outflow
c) Only lumbar outflow
d) Whole spinal cord
47-METABOLIC ACIDOSIS MAY BE BROUGHT BY:
a) loss of CO2 by increase ventilation
b) retention of CO2 by respiratory obstruction
c) persistent vomiting
d) absorption of excessive amount of Na HCO3
e) excessive exercise *

48-THE MOST POTENT STIMULANT OF RESPIRATION WOULD BE:


a) a two fold increase in PCO2 of inspired air *
b) a two fold increase in PO2 of inspired air
c) a 50% decrease in PCO2 of inspired air
d) a 50% decrease in PO2 of inspired air
e) a & d are equally potent stimuli
49-The psoas major muscle
a. flexes the thigh at the hip. *
b. extends the thigh at the hip.
c. adducts the thigh at the hip.
d. abducts the thigh at the hip.
e. assists in full contraction of the diaphragm

50-The renal arteries typically branch from the abdominal


aorta a the level of the:

Dr. wessam alzaidat - General surgery


a. 12th thoracic vertebral body.
b. first lumbar vertebral body.
c. second lumbar vertebral body. *
d. third lumbar vertebral body.
e. fourth lumbar vertebral body.

51-A 20–year–old man is undergoing retroperitoneal


dissection for a testicular germ cell tumor. The inferior
mesenteric artery is divided during reflection of the intestines
to expose the retroperitoneum. This can be expected to result
in:

a. ischemia of the descending colon.


b. ischemia of the sigmoid colon.
c. ischemia of the rectum.
d. ischemia of the splenic flexture
e. none of the above. *

52-The cremaster muscle is innervated by:

a. the ilioinguinal nerve.


b. the iliohypogastric nerve.
c. the obturator nerve.
d. the genital branch of the genitofemoral nerve. *
e. the femoral branch of the genitofemoral nerve.

53-As one proceeds outward from the adrenal medulla, the


three separate functional layers of the adrenal cortex are, in
correct order:

a. the zona reticularis, zona fasciculata, then zona glomerulosa. *


b. the zona fasciculata, zona reticularis, then zona glomerulosa.
c. the zona glomerulosa, zona fasciculata, then zona reticularis.
d. the zona glomerulosa, zona reticularis, then zona fasciculata.
e. the zona reticularis, zona glomerulosa, then zona fasciculata.

54-Proceeding from posterior to anterior, the structures


encountered in the renal hilum are, in correct order:

a. the renal artery, renal vein, and renal pelvis.


b. the renal pelvis, renal artery, and renal vein. *
c. the renal pelvis, renal vein, and renal artery.
d. the renal vein, renal artery, and renal pelvis.
e. the renal artery, renal pelvis, and renal vein.

Dr. wessam alzaidat - General surgery


55-During inguinal hernia repair in a male patient, the
ilioinguinal nerve is injured in the canal, which will most likely
produce:

a. anesthesia over the dorsum of the penis.


b. anesthesia over the pubis and scrotum, and loss of
cremasteric contraction.
c. anesthesia over the pubis and anterior scrotum only. *
d. anesthesia over the anterior and medial thigh.
e. anesthesia over the pubis only.

56-Hematuria is distinguished from hemoglobinuria or


myoglobinuria by:

a. dipstick testing.
b. the simultaneous presence of significant leukocytes.
c. microscopic presence of erythrocytes. *
d. examination of serum.
e. evaluation of hematocrit.

57-Glucose will be detected in the urine when the serum level


is above:

a. 75 mg/dl.
b. 100 mg/dl.
c. 150 mg/dl.
d. 180 mg/dl. *
e. 225 mg/dl.

58-What proportion of the cardiac output is delivered to the


kidney?

a. 5%
b. 20% *
c. 50%
d. 85%
e. 100%

59-The predominant buffering system in humans is:

a. bicarbonate. *
b. titratable acids.
c. ammonium (NH4+).

Dr. wessam alzaidat - General surgery


d. sodium lactate
e. phosphate

60-After a 7–hour–long, complex urethral reconstruction


performed in the extended lithotomy position, the patient has
severe thigh and buttock pain. The creatine phosphokinase
levels are dramatically elevated. To prevent ARF, the next step
should be:

a. dopamine infusion.
b. plasmapheresis.
c. dobutamine infusion.
d. forced alkaline diuresis. *
e. percutaneous nephrostomy.

61-The renal structure at greatest risk for ischemic injury is


the:

a. vasa recta.
b. cortical collecting duct.
c. juxtaglomerular apparatus.
d. medullary thick ascending loop of Henle. *
e. distal convoluted tubule.

62-Acute pyelonephritis is best diagnosed by:

a. chills, fever, and flank pain. *


b. bacteriuria and pyuria.
c. focal scar in renal cortex.
d. delayed renal function.
e. vesicourethral reflux.

63-The most reliable early clinical indicator of septicaemia is:

a. chills.
b. fever.
c. hyperventilation.*
d. lethargy.
e. change in mental status.

64-Which of the following is true regarding testosterone?

Dr. wessam alzaidat - General surgery


a. Testosterone is synthesized by the Sertoli cells of the testes.
b. Testosterone is synthesized by the Leydig cells of the testes.*
c. Testosterone is a direct precursor of pregnenolone.
d. 5α–Reductase is an enzyme that converts DHT into
testosterone.
e. Aromatase converts estrogens into testosterone

65-Calcium is maximally absorbed in which portion of the


gastrointestinal tract?

a. Stomach
b. Jejunum
c. Jejunum and proximal ileum *
d. Ileum
e. Ascending colon

66- the most common type of congenital diaphragmatic hernia is


caused by:
a. a defect in the central tendon
b. eventration of the diaphragm in the fetus
c. a defect through the space of Larry
d. a defect through the pleuroperitoneal fold. *
e. all of the above.
67- The calorie-nitrogen ratio for infant should be maintained at:
a. 75:1
b. 100:1
c. 50:1
d. 150:1 *
e. 25:1

68- A patient has undergone an ileal resection . Which of the


following conditions would he be least likely to develop?

a- alopecia *

b- megaloblastic anemia

c- nephrolithiasis

d- cholelithiasis

e- steatorrhea

69.With regard to Hirschsprung’s disease ( aganglionosis), all the


following are true except:
a- It is more common in males.
Dr. wessam alzaidat - General surgery
B. It may be complicated by enterocolitis.
C. Barium enema study may be normal.
D. It is best diagnosed by full thickness rectal biopsy.
E .Surgery can’t be accomplished without colostomy. *

70. In regard to malignant hyperthermia, one is true.


A. It triggered by Depolarizing muscle relaxing agents *
B. Non polarizing muscle agents are contra indicated .
C. Dantroline must be given after induction as prophylaxis.
D. Manitole and fluid support are not useful to prevent renal failure.
E. Alkalosis is the main metabolic disturbance.
71. Incompatible blood transfusion during surgery under general
anesthesia, which of the following is NOT true.
A. Unexplained bleeding.
B. Hematuria may be present.
C. Skin rash may be seen
D. Hypertension is well know problem in spite transfusion *
E. Volume support is mandatory.
72. The most common cause of death in Paediatric age group is :
A. Infection
B. Trauma. *
C. Malignancy.
D. Congenital anomalies.
E. Malnutrition.
73-In the management of pyloric stenosis, which of the following is
true.
A. Acidosis should be corrected first.
B. Chloride level is normal in spite alkalosis.
C. Hypokalaemia should be corrected. *
D. Usually there is compensatory hypocarbia.
E. Paradoxical aciduria is not a feature.
74 - The ligamentum teres represents an obliterated
a- Ductus venosus
b- Ductus arteriosus
c- Internal iliac artery
d- Umbilical vein *
e- porta hepatis
75- Tributaries of the portal vein include all of the the following
except:
a- Superior mesenteric
b- Para umbilical
c- uterine *
d- pyloric
e- lienal ( splenic )

Dr. wessam alzaidat - General surgery


76- The following statements are true regarding the liver anatomy
except :
a- Is attached to the diaphragm and anterior abdominal wall by the
Falciform ligament
b- Is totally covered by peritoneum *
c- Drains by hepatic veins into the inferior vena cava
d- Has a lymph drainage to both the mediastinal and porta hepatic nodes
e- Is directly related to the right suprarenal gland
77- An animal is in negative nitrogen balance when :
a- The intake exceeds output
b- New tissue is being synthesized
c-Output exceeds intake *
d- Intake is equal to output
e- The urine is nitrogen – free
78- Bilirubin secreted into the intestine is subjected to enzymatic
degradation , the final product being :
a- Biliverdin
d- Bilirubinogen
c- Urobilinogen
d- Stercobilin *
e- Mesobilirubinogen
79- In mammals , Norepinephrine is synthesized from :
a- Pyruvate
b- Arginine
c- Catechol
d- Tyrosine *
e- Tryptamine
80- Tissue graft within the same individual is known as :
a- Allograft
b- Isograft
c- Autograft *
d- Xenograft
e- Non of the above
81- The most effective means of sterilization by heat is :
a- steam
b- Boiling
c- Hot air
d- Fractional sterilization
e- Steam under pressure *
82- One of the following statements is true regarding Exotoxins:
a- Heat stable
b- Lipopolysaccharide in nature
c- part of cell wall of becteria
d- protein in nature *
e- less potent than endotoxins
Dr. wessam alzaidat - General surgery
83- Gall stones associated with pernicious anemia are most likely to
be :
a- pure cholesterol
b- Calcium bilirubinate *
c- Calcium carbornte
d- Mixed gall stones
e- combined gall stones

84- In familial polyposis coli, polyps tend to appear most frequently :

a- At birth
b- During the first decade
c-during the second and third decades of life *
d- During the fourth decades of life
e- After age of 40

85- The substance involved with protein synthesis

a- DNA
b- RNA *
c- Adenine
d- Guanine
e- Cytosine

86- In surgical skin wound 80% of tensile strength of unwounded


skin is restored by :
a- 1 week
b-2 weeks
c- 4 weeks
d- 12 weeks *
e- 24 weeks

87- Calcitonin sometimes is elaborated by which one of the following


tumors :
a-Medullary carcinoma of the breast
b- Medullary carcinoma of the thyroid *
c- Giant cell carcinoma a of pancrease
d- Carcinoid tumor of appendix
e- Ganglioneuromas

Dr. wessam alzaidat - General surgery


88- Normal " prothrombin time " as measured by usual clinical tests
depends on normal levels of all of the following factors excepts :
a- Factor II
b- Factor V
c- Factor VII
d- Factor VIII *
e- Factor X
89- Side effects of large doses of cortisone include all of the following
except :
a- Hirsutism and osteoporosis
b- Mental changes
c- Recurrence or appearance of new peptic ulcers
d- Increased susceptibility to infection
e- Depletion of serum sodium *
90- The average daily loss of water through the lungs and skin
( insensible water loss ) is approximately :
a- 10 ml
b- 100ml
c- 1000ml *
d- 2000ml
e- Any of these depending on circumstances

91- The most common cause of gastric outlet obstruction in adults is;
A- Adenocarcinoma of the stomach.*
B- Hypertrophic pyloric stenosis.
C- Duodenal stenosis secondary to peptic ulceration.
D- Bezoar.
E- Gastric lymphoma.
92- About colorectal carcinoma associated with Crohn’s disease, one
is TRUE:

A- It is usually common in women.


B- The frequency of carcinoma is similar in patients with
extensive, long standing ulcerative colitis.*
C- The right colon is involved in over 70% of patients.
D- The mean age of patients with colorectal carcinoma is 35.
E- The prognosis is the same for Ca associated with Crohn's dis
and Ca in colon without Crohn's dis,

93- Regarding pancreatic carcinoma all are true Except:-

A- 90% are ductal adenocarcinomas

B- Less than 20% occur in the head of the gland *


Dr. wessam alzaidat - General surgery
C- The usual presentation is with pain, weight loss and obstructive
jaundice

D- Ultrasound has a sensitivity of 80-90% in the detection of the tumour

E- Less than 20% of patients are suitable for curative surgery

94- Heparin all are true Except:-

A-Is a heterogeneous mixture of sulphated polypeptides *

B- Potentiates the actions of antithrombin III

C- Has a half life of 90 minutes.

D-Can be reversed by protamine sulphate

E-Can induce an idiosyncratic thrombocytopenia

95- All of the following affect Gastrin release EXCEPT:


a. antral acidification.
b. antral alkalization.
c. carbohydrates in antrum. *
d. gastric distension.
e. somatostatin release.

96- What portion of the colon absorb the majority of fluid?


a. ascending colon *
b. transverse colon
c. descending colon
d. sigmoid colon
e. rectum

97- The concentration of which electrolyte in pancreatic secretion


increases as the rate of secretion increases?
a. sodium
b. potassium
c. chloride
d. bicarbonate *
e. calcium

98- All of the following promote LES relaxation EXCEPT:


Dr. wessam alzaidat - General surgery
a. atropine *
b. nitric oxide
c. cholecystokinin (CCK)
d. gastric distention
e. pharyngeal stimulation

99- Intraabdominal adhesions following abdominal surgery have


been associated with all of the following EXCEPT:
a. small bowel obstruction
b. infertility
c. chronic pelvic pain
d. intestinal malabsorption *
e. increased risk for enterotomy on subsequent laparatomy

100- Which of the following is true regarding small bowel carcinoid


tumors?
a. most are biochemically atypical tumors lacking the enzyme dopa
decarboxylase
b. they are most common GI carcinoid tumors
c. regional lymph node involvement is common in tumors less than 1 cm
in size *
d. diagnosis is frequently made in patients prior to surgery
e. among carcinoid tumors, they are associated with the lowest rate of
second primary malignancy

KEY 1 C
2 D
3 A
4 B
5 D
6 B
7 E
8 C
9 C
10 D

Dr. wessam alzaidat - General surgery


11 D
12 C
13 D
14 B
15 A
16 A
17 D
18 C
19 C
20 B
21 A
22 A
23 D
24 D
25 D
26 B

28 A
29 C
30 D
31 D
32 C
33 C
34 B
35 B
36 D
37 B
38 D
39 C
40 B
41 C
42 B
43 B
44 B
45 E
Dr. wessam alzaidat - General surgery
46 B
47 E
48 A
49 A
50 C
51 E
52 D
53 A
54 B
55 C
56 C
57 D
58 B
59 A
60 D
61 D
62 A
63 C
64 B
65 C
66 D
67 D
68 A
69 E
70 A
71 D
72 B
73 C
74 D
75 C
76 B
77 C
78 D
79 D
80 C
81 E
82 D
83 B
84 C
85 B
86 D
87 B
88 D
Dr. wessam alzaidat - General surgery
89 E
90 C
91 A
92 B
93 B
94 A
95 C
96 A
97 D
98 A
99 D
100 C

27---------------D

Dr. wessam alzaidat - General surgery


Senior exam December 2022 ‫الخدمات الطبيه‬
1. an 18 y.o healthy college student presents to your office c/o chronic recurrent pain and
drainage from a wound at the superior aspect of his gluteal cleft, on examination he has a single
midline pit with minimal serous discharge and no surrounding erythema , there are no surronding
sinus tracts, he has never received medical attention for this issue . the most appropriate
management :
A. cleft lift procedure
B. Observation
C. Topical clindamycin
D. Excision with off mid line closure
E. I and d
Answer : D source Rush anorectal q 18

2. In which of the following settings should a LAR be performed :


A. 56 y.o male with circumferential villious adenoma beginning at the dentate line and extending 8
cm proximaly
B. Palliation of obstructing rectal cancer just above the dentate line with minimal liver mets in a
60 y.o female
C. An 80 y.o male with a large rectal cancer that produces anal pain and tenesmus and involves
the anal sphicter
D. A 45 y.o female with anastomotic recurrence after LAR of a distal rectal cancer
E. A 75 y.o male with urinary incontinence and a rectal cancer 5 cm above dentate line
Answer : E source Rush colon rectum q 7

3. cecal diverticula are different from sigmoid diverticula in that :


A. Sigmoid diverticula are true while cecal diverticula are false diverticula
B. Cecal diverticula is usual distinguishable from cancer
C. Cecal diverticula are consider congenital in origin.
D. Asymptomatic cecal diverticula found on barium enema or colonoscopy should be treated
operatively because of the high incidence of complications
E. Feculent peritonitis from perforation of cecal diverticulum may be treated with resection and
primary anastomosis in most cases
Answer : C source rush colon rectum q12

4. a 20 y.o healthy man with no previous medical problems is being evaluated for chronic
constipation his electrolyte level are normal , he denies recent travel and is not currently taking
any medications , plain radio graphs show dilated colon, transit studies are abnormal with slow
transit time , what is the best next step in management of this patient :
A. Flexible sigmoidoscope
B. Modification of diet and antiboitics
C. Placement of a rectal tube proximal to the normal caliber aganglionic bowel to decompress the
dilated nondiseased bowel
D. Anal monometry rectal bx ,and barium enema
E. Exploratory laparotomy
Answer : D source rush colon rectum q 5

5. A screening colonoscopy identifies a broad sessile villous apperaing lesion of the rectum
beginning 4 cm above the anal verge and extending 5 cm proximally , bx show a villoius adenoma
with dysplasia endorectal us shows that the muscularis propria is not involved , no suspicious

Dr. wessam alzaidat - General surgery


lymph nodes are seen , which of the following approaches is the most appropriate management of
this patient:
A. Repeat biopsy
B. Fulguration
C. Tran-anal excsion
D. APR
E. Itracavitary radiotherapy
Answer : C Source rush colon rectum q 6

6. An 89 y.o multi-parous female with COPD presents with a 12 month history of a worsening anal
bulge and fecal incontinence , on examination she has a 5 cm full thickness rectal prolapse , what
is the best surgical management for this patent :
A. Low ANterior Resection
B. Perineal rectosigmoidectomy
C. Trans abdominal mesh rectopexy
D. Laparoscopic sigmoid resection
E. Robotic sigmoid resection and mesh rectopexy
Answer B source Rush anorectal q 14

7. A pt with History of trauma laparotomy presents with small bowel obstruction , A NG tube is
placed that aspirates greater than 4 liter of fluid per day , in this pt the decreased insulin-glucagon
ration seen during simple starvation allows :
A. Increased lipogenesis
B. Increased lipolysis
C. Increased protein synthesis
D. Increased glycogen production
E. Decreased lipolysis
Answer B source rush nutrition q 4

8. which of the following statements regarding hypervolemia in postoperative pt is true :


A. Hypervolemia can be produced by the administration of isotonic salt solutions in amounts that
exceed the loss of volume
B. Acute over expansion of the ECF space is typically not well tolerated in in healthy individuals
C. Excess administration of normal saline can result in metabolic derangement, most commonly
hypercloremic metabolic alkolosis
D. The most reliable sign of volume excess is peripheral edema
E. Daily weight measurement in the post operative period does not help determine fluid status
Answer : A source rush fluid q 11

9. concerns with the use of propofol to supplement anaesthesia would be the greatest in a
patient with :
A. ESRD
B. Poor cardiac function
C. Cirrhosis
D. Neutropenia
E. Recent Burn
Answer : B source devigillio anaesthesia q 20

10: The best way to confirm proper placement of endotracheal tube is :

Dr. wessam alzaidat - General surgery


A. Direct visualization of tube passing through the vocal cords.
B. Auscultation of the lungs
C. Observation and condensation within the tube
D. Chest x-ray
E. End tidal co2
Answer : E source Devergillio Anastheisa q 15

11:which of the following is true regarding fluid homoeostasis during and immediately after a 4
hour laparotomy :
A. The majority of postoperativly oral fluid replacement will be primary absorbed in the colon
B. Laparoscopic and open abdominal causes have similar unmeasured fluid losses
C. Administration of 5% albumin pulls third space fluid in to the intravascular space
D. Insensible fluid deficit may be high as 4 liter
E. About 50% of crystalloid bolus will remain intravascular
Answer : D source devirgillo fluid q 6

12:which of the following is true regarding the risk of hypoglycemia following cessation of TPN :
A. It commonly occurs in patients with liver disease
B. TPN should be tapered in most pts to avoid this complication
C. This complication is relatively uncommon
D. It is more likely to occur in diabetic patients
E. It is more likely to occur in patient with renal disease
Answer : D source devirgillo nutrition q 3

13: the hernia bounded by the lattisimus dorsi muscle , illiac crest , and external oblique muscle is
known as :
A. Grynfelt hernia
B. Ricther hernia
C. Petit hernia
D. Litter hernia
E. Obturator hernia
Answer : C source Devergillo Hernia Q7

14:which of the following is a contraindications for negative pressure wound therapy :


A. Newly grafted skin
B. Wounds with fistula
C. Diabetic wounds
D. Ischemic wounds
E. Venous stasis wounds
Answer D source devergillo plastic surgery q4

15:what is the mainstay of postoperative flap monitoring :


A. Doppler ultrasound
B. Pulse oximetry
C. Clinical observation
D. Qaunititive flurometry
E. Surface temperature probing
Answer C source devirgillo plastic surgery q 13

Dr. wessam alzaidat - General surgery


16: Thromboangitits obliterans ( buergers disease ) is characterized by :
A. Frequent coronary artery involvement
B. Frequent involvement of aortoilliac bifurcation
C. Disease limited to pedal arteries
D. Successful treatment with saphenous vein bypass
E. Cockscrew collaterals
Answer : E source Devirgillo vascular q 12

17:which of the following is not true regarding hypertrophic scarring and keloid :
A. Both have increase deposition of collagen
B. Both have normal amount of fibroblast
C. Both have a genetic predisposition
D. Both have a propensitiy to develope following a postsurgicval wound infection
E. Both tend to recur with re-excsion
Answer : C source devirgillo wound healing q 3

18:which are the following true regarding skin antiseptic techniques before surgery :
A. Iodine based preps are superior to chlorhexidine for preventing surgical site infection
B. Chlorhexidine based preps are safe on all body surfaces as a preoperative cleanser
C. Preoperative bathing with cholrhexidine has been shown to reduce incidence of surgical site
infection
D. The bactericidal effect of iodine is form its ability to form an extracellular crystal matrix and
destabilize cell membranes
E. Povidone iodine was formulated to decrease the availabiltiy to molecular iodine
Answer E source devergillo wound healing q 10
19: which of the following is true regarding hepato-renal syndrome :
A. Type 2 is rapidly progressive with a poor prognosis
B. Is is associated with intense renal vasodilation
C. It is associated with splanchnic vasoconstriction
D. The urine sodium is typically less than 10 MEq /L
E. Type 1 is relatively stable
Answer : D source devergillo critical care q 29

20. an asymptomatic pt is found to have a serum calcium level of 13.5mg/dl which of the following
medications should be avoided ?
A. Biphosphonates
B. Thiazide diuretic
C. Mithramycin
D. Calcitonin
E. Corticosteroids
Answer : B source Rush fluid electrolytes q5

21: Impaired platelet aggregation in uremia is responsive to all of the following except :
A. Hemodialysis
B. Cryoprecepitate
C. DDAVP
D. Platelet transfusion
E. Desmopressin
Answer : D source rush trasnfusion q7

Dr. wessam alzaidat - General surgery


22: which of the following urologic complications does not occur with hernia repair
A. Ishcemic orchitits
B. Transection of the vas deferns
C. Prostatitis
D. Testicaular atrophy
E. Ovarian torsion within the hernia sac
Answer C source Rush henria q23

23. A 68 y.o diabetic man undergoes a below knee amputation , the pt postoperative course is
complicated by severe depression and anorexia , before discharge the pt is started on
multivitamins regimen , which of the following statements regarding wound healing is true?
A. vitamin A is needed for hydroxylation of lysine and proline in collagen synthesis
B. High doses of vitamin c improve wound healing
C. Vitamin E is involved in the stimulation of fibroplasia, collagen cross linking and
epithelialization
D. Zinc deficiency results in delayed early wound healing
E. Iron deficiency has been linked to defects in long term wound remodeling
Answer : d source rush wound healing q 6

24: a 25 y.o female ballet dancer with a history of anorexia nervosa arrives at the ER with rt lower
quadrant pain , after an appendectomy , a wound infection at the surgical site requires
debridement , the pt is placed on an antibiotic regimen , and the wound is packed with wet-to dry
dressing , regarding wound healing and malnutrition , which of the following statements is true ?
A. Hypoprotenimea leads to decrease levels of arginine and glutamine which are essential in
wound healing
B. Cell membranes rapidly to become dehydrated in the absence of vitamin E resulting in delayed
wound healing
C. Zinc is essential to the fibroblast ability to cross link collagen
D. Vitamin D servers an immunomodulatory in wound healing
E. The pt should be treated with high does vitamin c , vitamin a , zinc
Answer : D source rush wpound healing q14

25: which of the following is not proven etiology of DIC ?


A. Gram negative sepsis
B. Trauma
C. Retained products of conception
D. Malignancy
E. Supratheraputic UFH drip
Answer : E source rush homoeostasis q9

26: a full term baby in born with drooling , coughing and cyanosis after the first feeding but theses
resolve quickly and spontaneously the nest best step in management should be:
A. immediate intubation
B. Placement of orogastric tube
C. Two view abdominal x-ray
D. Two view chest x-ray
E. Upper GI contrast series
Answer : B source devirgillo pediatric surgery q 4

Dr. wessam alzaidat - General surgery


27: 3 y.o girl presents to the ER with crampy abdominal pain for 24 hours , the pain is increasing in
frequency and duration ultrasound demonstrated target sign the most common pathological lead
point is :
A. Appendix
B. Small bowel polyp
C. Ectopic pancreatic tissue
D. Meckels diverticulum
E. Intralumianl hematoma
Answer D source devirgillo pediatic surgery q 24

28: which condition is associated with the development of soft tissue sarcoma: \
A. Famillial retinoblastoma
B. Juvenile polyposis syndrome
C. Von hippel lindau syndrome
D. Asbestos syndrome
E. MEN TYPE 1
Answer : A source rush soft tissue q 7

29: which of the following is true regarding sarcomas ?


A. kaposi sarcoma is the common cause of death in patients with AIDS
B. Embryonal sub-type is a rare childhood rhambdomyosarcoma
C. Embryonal subtyoe has the worst prognosis is childhood rhambdomyosarcoma
D. Osteosarcoma arises from stromal cells
E. Osteosarcoma is one of the rarest malignant bone tumors
Answer : D source devirgillo skin and soft tissue q 8a

30:which of the following is true regarding FAP ?


A. Micro satellite instability is a major contributor to this disease
B. It is not associated with extra-intestinal manifestations
C. Patients with this gene should begin screening with flexible sigmoidoscopy at age 20
D. Patients with prophylactic proctocolectomy have a lower risk of developing periampulary ca
E. Upper endoscopy should be performed every 1 to 3 years
Answer: E source devirgillo large intestine q10

Dr. wessam alzaidat - General surgery

You might also like